You are on page 1of 555

MCQ Categories in each Chapter

EBD_7207
• Corporate Office : 45, 2nd Floor, Maharishi Dayanand Marg, Corner Market,
Malviya Nagar, New Delhi-110017
Tel. : 011-26692293 / 26692294

by
Preetima Bajpai

Typeset by Disha DTP Team

DISHA PUBLICATION
All Rights Reserved

© Copyright Publisher
No part of this publication may be reproduced in any form without prior permission of the publisher. The publisher do
not take any legal responsibility for any errors or misrepresentations that might have crept in. We have tried and made
our best efforts to provide accurate up-to-date information in this book.

For further information about the books from DISHA,


Log on to www.dishapublication.com or email to info@dishapublication.com

(ii)
Contents
1. Some Basic Concepts of Chemistry 1-18

2. Structure of Atom 19-38

3. Classification of Elements and Periodicity in Properties 39-52

4. Chemical Bonding and Molecular Structure 53-76

5. States of Matter 77-94

6. Thermodynamics 95-110

7. Equilibrium 111-134

8. Redox Reactions 135-146

9. Hydrogen 147-156

10. The s-Block Elements 157-168

11. The p-Block Elements (Group 13 and 14) 169-180

12. Organic Chemistry – Some Basic Principles and Techniques 181-202

13. Hydrocarbons 203-226

14. Environmental Chemistry 227-238

15. The Solid State 239-254


16. Solutions 255-274

17. Electrochemistry 275-294


(iii)
EBD_7207
18. Chemical Kinetics 295-316

19. Surface Chemistry 317-332

20. General Principles and Processes of Isolation of Elements 333-344

21. The p-Block Elements (Group 15, 16, 17 and 18) 345-366

22. The d-and f-Block Elements 367-382

23. Coordination Compounds 383-404

24. Haloalkanes and Haloarenes 405-422

25. Alcohols, Phenols and Ethers 423-440

26. Aldehydes, Ketones & Carboxylic acids 441-462

27. Amines 463-478

28. Biomolecules 479-498

29. Polymers 499-510

30. Chemistry In Everyday Life 511-520

Mock Test - 1 MT-1-6

Mock Test - 2 MT-7-12

Mock Test - 3 MT-13-20

Mock Test - 4 MT-21-26

Mock Test - 5 MT-27-32

(iv)
1
SOME BASIC CONCEPTS OF
CHEMISTRY
FACT / DEFINITION TYPE QUESTIONS 8. Choose the correct statement.
(a) The particle s in liquids are more closely held than
1. A mixture of sand and iodine can be separated by gases but less free to move than solids.
(a) crystallisation (b) distillation (b) The particles of solids are arranged in orderly fashion
(c) sublimation (d) fractionation but they can move as freely as liquids.
2. Difference in density is the basis of (c) The particles of gases are far apart as compared to
(a) ultrafiltration (b) molecular sieving solids and liquids and their movement is easy and
(c) molecular attraction (d) gravity separation fast.
3. Which of the following is an example of a heterogeneous (d) The particles of gases moves faster than liquids only
substance? when the gases are heated.
(a) Bottled water (b) Table salt 9. A mixture contains two or more substances in ________
(c) Pieces of copper (d) Candle which are called its ________.
4. Which of the following substances cannot be separated in (a) fixed ratio, compounds
to its constituents by physical methods? (b) fixed ratio, elements
(a) Sugar and water solution
(c) any ratio, components
(b) Salt and sugar
(d) any ratio, elements
(c) Solid glucose
10. Which one of these is not a pure compound?
(d) Both (a) and (b)
5. Which of the following pair of substances contain (a) O3 (b) H2O2
element and compound within a pair ? (c) H2O (d) Sucrose solution
(A) O2, CH4 (B) H2, O2 11. One fermi is
(C) N2, CO2 (D) Na, CO (a) 10–15 cm (b) 10–13 cm
–10
(c) 10 cm (d) 10–12 cm
(a) A, C, D (b) B only
(c) C and D (d) All of these 12. The prefix 1018 is
6. Which of the following statements about a compound is (a) giga (b) kilo
incorrect? (c) exa (d) nano
(a) A molecule of a compound has atoms of different 13. The prefix zepto stands for (in m)
elements. (a) 109 (b) 10–12
(b) A compound cannot be separated into its constituent –15
(c) 10 (d) 10–21
elements by physical methods of separation.
14. The unit J Pa–1 is equivalent to
(c) A compound retains the physical properties of its
constituent elements. (a) m3 (b) cm3
(c) dm 3 (d) None of these
(d) The ratio of atoms of different elements in a compound
is fixed. 15. Which has highest weight ?
7. Choose the correct combination (a) 1 m3 of water (b) A normal adult man
Element Compound Mixture (c) 10 litre of Hg (d) All have same weight
(a) Ammonia Sodium Air 16. Which one of the following set of units represents the
(b) Water Sugar Aqueous sugar solution smallest and largest amount of energy respectively?
(c) Hydrogen Oxygen Water (a) J and erg (b) erg and cal
(d) Silver Water Air (c) cal and eV (d) eV and L-atm
EBD_7207
2 SOME BASIC CONCEPTS OF CHEMISTRY
17. A measured temperature on Fahrenheit scale 25. In the final answer of the expression
is 200 °F. What will this reading be on Celsius scale ?
(a) 40° C (b) 94° C (29.2 20.2) (1.79 105 )
(c) 93.3 °C (d) 30° C 1.37
18. Which of the following is not a SI unit? the number of significant figures is :
(a) metre (b) candela (a) 1 (b) 2
(c) mole (d) litre (c) 3 (d) 4
19. The prefix 10–24 is 26. The number of significant figures for the three numbers
(a) yotta (b) zeta 161 cm, 0.161 cm, 0.0161 cm are
(c) yocto (d) zepto (a) 3,4 and 5 respectively (b) 3,4 and 4 respectively
20. Many countries use Fahrenheit scale for expressing (c) 3,3 and 4 respectively (d) 3,3 and 3 respectively
temperature of atmosphere. If temperature in any such 27. Given P = 0.0030m, Q = 2.40m, R = 3000m, Significant figures
country is measured 41°F then what is its value in celcius in P, Q and R are respectively
scale and would you expect hot or cold atmosphere in that (a) 2, 2, 1 (b) 2, 3, 4
country? (c) 4, 2, 1 (d) 4, 2, 3
(a) 15°C, cold (b) 25°C, normal 28. If the density of a solution is 3.12 g mL–1, the mass of 1.5 mL
(c) 5°C, cold (d) 41°C, hot solution in significant figures is______.
21. A sample was weighted using two different balances. The (a) 4.7 g (b) 4680 × 10–3 g
results were (c) 4.680 g (d) 46.80 g
(i) 3.929 g (ii) 4.0 g 29. In which of the following number all zeros are significant?
How would the weight of the sample be reported? (a) 0.0005 (b) 0.0500
(a) 3.93 g (b) 3g (c) 50.000 (d) 0.0050
(c) 3.9 g (d) 3.929 g 30. The correctly reported answer of addition of 29.4406, 3.2
22. Two students performed the same experiment separately and 2.25 will have significant figures
and each one of them recorded two readings of mass which (a) 3 (b) 4
are given below. Correct reading of mass is 3.0 g. On the (c) 2 (d) 5
basis of given data, mark the correct option out of the 31. The number of significant figures in 10.3106 g is
following statements. (a) 2 (b) 3
Students Readings (c) 1 (d) 6
(i) (ii) 32. Choose the correct option that represents the result of the
A 3.01 2.99 given calculation to the appropriate number of significant
B 3.05 2.95 figures:
(a) Results of both the students are neither accurate nor 43.0 0.0243
precise. 0.340 4
(b) Results of student A are both precise and accurate.
(a) 0.768 (b) 0.77
(c) Results of student B are neither precise nor accurate.
(c) 0.76 (d) 0.7683
(d) Results of student B are both precise and accurate.
33. Arrange the numbers in increasing no. of significant figures.
23. 0.00016 can be written as ...A... in scientific notaiton. Here,
0.002600, 2.6000, 2.6, 0.260
A refers to
(a) 2.6 < 0.260 < 0.002600 < 2.6000
(a) 1.6 × 10–4 (b) 24.50 × 10–9
–8 (b) 2.6000 < 2.6 < 0.002600 < 0.260
(c) 2.450 × 10 (d) 24.50 × 10–7
(c) 0.260 < 2.6 < 0.002600 < 2.6000
24. If the true value for an experimental result is 6.23 and the
(d) 0.002600 < 0.260 < 2.6 < 2.6000
results reported by three students X, Y and Z are :
34. Dimension of pressure are same as that of
X : 6.18 and 6.28
(a) Energy (b) Force
Y : 6.20 and 6.023
(c) Force per unit volume (d) Energy per unit volume
Z : 6.22 and 6.24
35. n g of substance X reacts with m g of substance Y to form
Which of the following option is correct : p g of substance R and q g of substance S. This reaction
(a) X precise, Y accurate, Z precise and accurate. can be represented as, X + Y = R + S. The relation which
(b) X precise and accurate, Y not precise, Z precise can be established in the amounts of the reactants and the
(c) Both X & Z precise & accurate, Y not precise. products will be
(d) Both X & Y neither precise nor accurate, Z both precise (a) n – m = p – q (b) n + m = p + q
and accurate. (c) n = m (d) p = q
SOME BASIC CONCEPTS OF CHEMISTRY 3
36. 20 g of CaCO3 on heating gave 8.8 g of CO2 and 11.2 g of 42. The percentage of copper and oxygen in samples of CuO
CaO. This is in accordance with obtained by different methods were found to be the same.
(a) The law of conservation of mass. The illustrate the law of
(b) The law of constant composition. (a) constant proportions (b) conservation of mass
(c) The law of reciprocal proportion. (c) multiple proportions (d) reciprocal proportions
(d) None of these 43. The law of definite proportions was given by –
37. Which of the following is the best example of law of (a) John Dalton (b) Humphry Davy
conservation of mass? (c) Proust (d) Michael Faraday
(a) 12 g of carbon combines with 32 g of oxygen to form 44. Which one of the following pairs of compounds illustrate
44 g of CO2 the law of multiple proportions ?
(b) When 12 g of carbon is heated in a vacuum there is no (a) H2O and Na2O (b) MgO and Na2O
change in mass (c) Na2O and BaO (d) SnCl2 and SnCl4
(c) A sample of air increases in volume when heated at 45. Among the following pairs of compounds, the one that
constant pressure but its mass remains unaltered illustrates the law of multiple proportions is
(d) The weight of a piece of platinum is the same before (a) NH3 and NCl3 (b) H2S and SO2
and after heating in air (c) CS2 and FeSO4 (d) CuO and Cu2O
38. Which of the following statements is correct about the 46. Two samples of lead oxide were separately reduced to
reaction given below ? metallic lead by heating in a current of hydrogen. The weight
of lead from one oxide was half the weight of lead obtained
4Fe(s) 3O 2 (g) 2Fe 2 O 3 (g)
from the other oxide. The data illustrates
(a) Total mass of iron and oxygen in reactants = total mass (a) law of reciprocal proportions
of iron and oxygen in product therefore, it follows law (b) law of constant proportions
of conservation of mass.
(c) law of multiple proportions
(b) Total mass of reactants = total mass of product;
(d) law of equivalent proportions
therefore, law of multiple proportions is followed.
47. In compound A, 1.00g of nitrogen unites with 0.57g of
(c) Amount of Fe2O3 can be increased by reducing the
oxygen. In compound B, 2.00g of nitrogen combines with
amount of any one of the reactants (iron or oxygen).
2.24g of oxygen. In compound C, 3.00g of nitrogen combines
(d) Amount of Fe2O3 produced will decrease if the amount with 5.11g of oxygen. These results obey the following law
of any one of the reactants (iron or oxygen) is taken in
(a) law of constant proportion
excess.
(b) law of multiple proportion
39. In an experiment 4.2 g of NaHCO3 is added to a solution of
(c) law of reciprocal proportion
acetic acid weighing 10.0 g, it is observed that 2.2 g of CO2
is released into the atmosphere. The residue left behind is (d) Dalton’s law of partial pressure
found to weigh 12.0 g 48. Which of the following statements indicates that law of
The above observations illustrate multiple proportion is being followed.
(a) law of definite proportions. (a) Sample of carbon dioxide taken from any source will
always have carbon and oxygen in the ratio 1 : 2.
(b) law of conservation of mass
(b) Carbon forms two oxides namely CO2 and CO, where
(c) law of multiple proportions
masses fo oxygen which combine with fixed mass of
(d) None of these
carbon are in the simple ration 2 : 1.
40. In one experiment, 4g of H2 combine with 32g of O2 to form
(c) When magnesium burns in oxygen, the amount of
36g of H2O. In another experiment, when 50g of H2 combine
magnesium taken for the reaction is equal to the amount
with 400g of O2 then 450g of H2O is formed. Above two
of magnesium in magnesium oxide formed.
experiments follow
(d) At constant temperature and pressure 200 mL of
(a) The law of conservation of mass
hydrogen will combine with 100 mL oxygen to produce
(b) The law of constant composition 200 mL of water vapour.
(c) Both (a) and (b) 49. The molecular weight of O2 and SO2 are 32 and 64
(d) Neither (a) nor (b) respectively. At 15°C and 150 mm Hg pressure, one litre of
41. Irrespective of the source, pure sample, of water always yields O2 contains ‘N’ molecules. The number of molecules in two
88.89% mass of oxygen and 11.11% mass of hydrogen. This is litres of SO2 under the same conditions of temperature and
explained by the law of pressure will be :
(a) conservation of mass (b) multiple proportions (a) N/2 (b) 1N
(c) constant composition (d) constant volume (c) 2N (d) 4N
EBD_7207
4 SOME BASIC CONCEPTS OF CHEMISTRY
50. 10 dm 3 of N2 gas and 10 dm3 of gas X at the same 59. What is the average atomic mass of bromine from the
temperature contain the same number of molecules, the gas following data : (abundance is in %)
X is Isotope Mass Abundance
(a) CO2 (b) CO 79Br 78.9183361 50.69
(c) H2 (d) NO 81Br 80.916289 49.31
51. One mole of a gas occupies a volume of 22.4 L. This is (a) 79.9 (b) 76.6
derived from (c) 75.9 (d) 69.9
(a) Berzelius’ hypothesis (b) Gay-Lussac’s law 60. What is the mass of an atom of oxygen (in gm)?
(c) Avogadro’s law (d) Dalton’s law (a) 2.656 × 10–23 (b) 1.567 × 10–22
52. One of the following combination which illustrates the law (c) 2.0 × 10 –22 (d) 3.5 × 10–23
of reciprocal proportions ? 61. If the mass of the one atom is found to be 2.324784×10–23g,
(a) N2O3, N2O4, N2O5 (b) NaCl, NaBr, NaI then this atom can be ?
(c) CS2, CO2, SO2 (d) PH3, P2O3, P2O5 (a) Oxygen (b) Carbon
53. Equal volumes of two gases A and B are kept in a container (c) Fluorine (d) Nitrogen
at the same temperature and pressure. Avogadro’s law is 62. What is the mass of 1 molecule of CO.
invalid if
(a) 2.325 × 10–23 (b) 4.65 × 10–23
(a) the gases are reactive –23
(c) 3.732 × 10 (d) 2.895 × 10–23
(b) the gases are non-reactive
63. Calculate the volume at STP occupied by 240 gm of SO2.
(c) gas A has more number of molecules than gas B.
(a) 64 (b) 84
(d) None of these
(c) 59 (d) 73
54. Molecular mass is defined as the
64. At S.T.P. the density of CCl4 vapours in g/L will be nearest
(a) mass of one atom compared with the mass of one to:
molecule
(a) 6.87 (b) 3.42
(b) mass of one atom compared with the mass of one atom
(c) 10.26 (d) 4.57
of hydrogen
65. The number of gram molecules of oxygen in 6.02 × 1024
(c) mass of one molecule of any substance compared with
CO molecules is
the mass of one atom of C-12
(a) 10 gm molecules (b) 5 gm molecules
(d) None of the above
(c) 1 gm molecules (d) 0.5 gm molelcules
55. 1 amu is equal to
66. The number of oxygen atoms in 4.4 g of CO2 is
1 1 (a) 1.2 × 1023 (b) 6 × 1022
(a) of O-16 (b) of C-12 23
14 12 (c) 6 × 10 (d) 12 × 1023
(c) 1 g of H2 (d) 1.66 × 10–23 kg 67. Which has maximum number of molecules?
56. The modern atomic weight scale is based on (a) 7 gm N2 (b) 2 gm H2
(a) O16 (b) C12 (c) 16 gm NO2 (d) 16 gm O2
(c) H 1 (d) C13 68. Number of atoms in 558.5 gram Fe (at. wt. of Fe = 55.85
57. The percentage weight of Zn in white vitriol [ZnSO4.7H2O] g mol–1) is
is approximately equal to ( Zn 65, S 32, O 16 and (a) twice that in 60 g carbon
(b) 6.023 1022
H = 1)
(c) half that in 8 g He
(a) 33.65 % (b) 32.56 %
(d) 558.5 6.023 1023
(c) 23.65 % (d) 22.65 %
69. The number of molecules in 16 g of methane is
58. The average atomic mass of neon based on following data
is : 16
(a) 3.0 × 1023 (b) 10 23
Isotope Relative abundance 6.02
20 Ne 0.9051
21 Ne 0.0027 16
(c) 6.023 × 1023 (d) 10 23
22 Ne 0.0922 3.0
(a) 0.33 u (b) 20.187 u 70. Number of g of oxygen in 32.2 g Na2SO4.10 H2O is
(c) 6.729 u (d) 18.058 u (a) 20.8 (b) 2.24
(c) 22.4 (d) 2.08
SOME BASIC CONCEPTS OF CHEMISTRY 5
71. The number of moles of oxygen in one litre of air containing 83. Which has the maximum number of molecules among the
21% oxygen by volume, under standard conditions are following ?
(a) 0.0093 mole (b) 0.21 mole (a) 44 g CO2 (b) 48 g O3
(c) 2.10 mole (d) 0.186 mole (c) 8 g H2 (d) 64 g SO2
72. The number of molecules in 8.96 litre of a gas at 0ºC and 1
84. The weight of one molecule of a compound C 60 H122 is
atm. pressure is approximately
(a) 6.023 × 1023 (b) 12.04 × 1023 (a) 1.2 10 20
gram (b) 1.4 10 21
gram
(c) 18.06 × 10 23 (d) 24.08 × 1022
73. The mass of a molecule of water is (c) 5.025 10 23 gram (d) 6.023 10 23 gram
(a) 3 × 10–25 kg (b) 3 × 10–26 kg
85 The simplest formula of a compound containing 50% of
–26
(c) 1.5 × 10 kg (d) 2.5 × 10–26 kg element X (atomic mass 10) and 50% of element Y (atomic
74. One mole of CO2 contains : mass 20) is
(a) 3 g atoms of CO2 (a) XY (b) XY3
(b) 18.1 × 1023 molecules of CO2
(c) X2Y (d) X2Y3
(c) 6.02 × 1023 atoms of O
86. Empirical formula of hydrocarbon containing 80% carbon
(d) 6.02 × 1023 atoms of C
and 20% hydrogen is :
75. Volume of a gas at NTP is 1.12 × 10–7cm3. The number of
molecules in it is : (a) CH3 (b) CH4
(a) 3.01 × 1012 (b) 3.01 × 1024 (c) CH (d) CH2
(c) 3.01 × 10 23 (d) 3.01 × 1020 87. The empirical formula of a compound is CH2. One mole of
76. How many atoms are contained in one mole of sucrose this compound has a mass of 42 grams. Its molecular formula
(C12 H22O11)? is :
(a) 20 × 6.02 × 1023 atoms/mol (a) C3H6 (b) C3H8
(b) 45 × 6.02 × 1023 atoms/mol
(c) CH2 (d) C2H2
(c) 5 × 6.02 × 1023 atoms/mol
88. A compound contains 54.55 % carbon, 9.09% hydrogen ,
(d) None of these
36.36% oxygen. The empirical formula of this compound is
77. One litre oxygen gas at S.T.P will weigh :
(a) 1.43 g (b) 2.24 g (a) C3H 5O (b) C 4 H 8O 2
(c) 11.2 g (d) 22.4 g (c) C 2 H 4O 2 (d) C 2 H 4O
78. Number of moles of NaOH present in 2 litre of 0.5 M NaOH 89. In a hydrocarbon, mass ratio of hydrogen and carbon is
is : 1:3, the empirical formula of hydrocarbon is
(a) 1.5 (b) 2.0 (a) CH4 (b) CH2
(c) 1.0 (d) 2.5 (c) C2H (d) CH3
79. O2, N2 are present in the ratio of 1 : 4 by weight. The ratio of
90. An organic compound contains carbon, hydrogen and
number of molecules is :
oxygen. Its elemental analysis gave C, 38.71% and H, 9.67%.
(a) 7 : 32 (b) 1 : 4
The empirical formula of the compound would be :
(c) 2 : 1 (d) 4 : 1
(a) CH3O (b) CH2O
80. How many moles of Al2(SO4)3 would be in 50 g of the
(c) CHO (d) CH4O
substance ?
(a) 0.083 mole (b) 0.952 mole 91. A hydrocarbon is composed of 75% carbon. The empirical
formula of the compound is
(c) 0.481 mole (d) 0.140 mole
(a) CH2 (b) CH3
81. The mass of 1 mole of electrons is
(a) 9.1 × 10–28g (b) 1.008 mg (c) C2H5 (d) CH4
(c) 0.55 mg (d) 9.1 × 10–27 g 92. 12 gm of Mg (atomic mass 24) will react completely with
82. 10 g of hydrogen and 64 g of oxygen were filled in a steel hydrochloric acid to give
vessel and exploded. Amount of water produced in this (a) One mol of H2
reaction will be: (b) 1/2 mol of H2
(a) 3 mol (b) 4 mol (c) 2/3 mol of O2
(c) 1 mol (d) 2 mol (d) both 1/2 mol of H2 and 1/2 mol of O2
EBD_7207
6 SOME BASIC CONCEPTS OF CHEMISTRY
93. 20.0 kg of N2(g) and 3.0 kg of H2(g) are mixed to produce 104. Two solutions of a substance (non electrolyte) are mixed in
NH3(g). The amount of NH3(g) formed is the following manner. 480 ml of 1.5 M first solution + 520 ml
(a) 17 kg (b) 34 kg of 1.2 M second solution. What is the molarity of the final
(c) 20 kg (d) 3 kg mixture?
94. 20.0 kg of H2(g) and 32 kg of O2(g) are reacted to produce (a) 2.70 M (b) 1.344 M
H2O(l). The amount of H2O (l) formed after completion of (c) 1.50 M (d) 1.20 M
reaction is
(a) 62 kg (b) 38 kg STATEMENT TYPE QUESTIONS
(c) 42 kg (d) 72 kg
95. What is the weight of oxygen required for the complete 105. Which of the following statements are correct ?
combustion of 2.8 kg of ethylene ? (i) Both solids and liquids have definite volume.
(a) 2.8 kg (b) 6.4 kg (ii) Both liquids and gases do not have definite shape.
(c) 9.6 kg (d) 96 kg (iii) Both solids and gases take the shape of the container.
96. In the reaction (a) (i) and (iii) (b) (ii) and (iii)
4 NH3 ( g ) 5O2 ( g ) 4 NO( g ) 6H2 O(l ) (c) (i) and (ii) (d) (i), (ii) and (iii)
When 1 mole of ammonia and 1 mole of O2 are made to react 106. Choose correct option based on following statements. Here
to completion, T stands for true statement and F for false statement.
(a) 1.0 mole of H2O is produced (i) Homogeneous mixture has uniform composition
(b) 1.0 mole of NO will be produced throughout.
(c) all the oxygen will be consumed (ii) All components of a heterogeneous mixture are
(d) all the ammonia will be consumed observable to naked eyes.
97. What is the molarity of 0.2N Na 2 CO 3 solution? (iii) All solutions are homogeneous in nature.
(a) 0.1 M (b) 0 M (iv) Air is an example of heterogeneous mixture.
(c) 0.4 M (d) 0.2 M (a) TTFF (b) TFTF
98. The molar solution of H 2SO 4 is equal to : (c) FFTT (d) TFFF
107. Read the following and choose the incorrect statements.
(a) N/2 solution (b) N solution
(c) 2N solution (d) 3N solution (i) Both weight and mass are same quantities used for
99. Volume of water needed to mix with 10 mL 10N HNO3 to get measurement of amount of matter present in a
0.1 N HNO3 is : substance
(a) 1000 mL (b) 990 mL (ii) Mass and weight of a substance vary from one place
(c) 1010 mL (d) 10 mL to another due to change in gravity.
100. One kilogram of a sea water sample contains 6 mg of (iii) SI unit of mass is kilogram and while SI unit of weight
dissolved O2. The concentration of O2 in the sample in is gram.
ppm is (a) (i) and (iii) (b) (ii) and (iii)
(a) 0.6 (b) 6.0 (c) (i) and (ii) (d) All of these
(c) 60.0 (d) 16.0 108. Moon takes 27.3 days to complete one orbit around the
101. A 5 molar solution of H2SO4 is diluted from 1 litre to a Earth. Now read the following statements and choose the
volume of 10 litres, the normality of the solution will be : correct code. Here T is for true statement and F is for ‘False
(a) 1N (b) 0.1N statement’.
(c) 5N (d) 0.5N
(i) Moon takes 655.2 hours to complete one orbit around
102. With increase of temperature, which of these changes?
the Earth.
(a) Molality (b) Weight fraction of solute
(ii) Moon takes 39312 seconds to complete one orbit
(c) Molarity (d) Mole fraction
20
around the earth.
103. 6.02 × 10 molecules of urea are present in 100 ml of its
(iii) Moon takes 1638 minutes to complete one orbit around
solution. The concentration of urea solution is
the Earth.
(a) 0.02 M (b) 0.01 M
(c) 0.001 M (d) 0.1 M (a) F T F (b) T T T
(Avogadro constant, NA = 6.02 × 1023 mol–1) (c) T F F (d) T F T
SOME BASIC CONCEPTS OF CHEMISTRY 7
109. Give the correct order of initials T or F for following 113. Match the columns
statements. Use T if statement is true and F if it is false. Column-I Column-II
(i) Gay-Lussac’s law of gaseous volumes is actually the (Number) (Significant figures)
law of definite proportion by volume. (A) 29900. (p) 2
(ii) Law of conservation of mass is true for physical (B) 290 (q) 1
change, but not for chemical change. (C) 1.23 × 1.331 (r) 4
(iii) The percentage of oxygen in H2O2 is different from (D) 20.00 (s) 3
that in H2 O. Hence, it violates law of definite
(E) 2.783 – 1 (t) 5
proportions.
(a) A – (r), B – (q), C – (t), D – (p), E – (s)
(iv) Fixed mass of A reacts with two different masses of B
(say x and y), then the ratio of x/y can be any positive (b) A – (t), B – (p), C – (s), D – (r), E – (q)
integer. (c) A – (p), B – (t), C – (s), D – (r), E – (q)
(v) At STP, 5 mL of N2 and H2 have different no. of (d) A – (t), B – (s), C – (r), D – (q), E – (p)
molecules. 114. Match the columns
(a) TTFTF (b) FTTFT Column-I Column-II
(c) TFFTF (d) TFTTF (Laws of chemical (Scientist)
110. Consider the following statements. combinations)
(i) Atoms of H, O, N and C have identical properties but (A) Law of definite (p) Antoine Lavoisier
different mass. proportions
(ii) Matter is divisible into atoms which are further (B) Law of multiple (q) Gay Lussac
indivisible. proportions
(iii) The ratio of N: H in NH3 is 1 : 3 and N : O in nitric oxide (C) Law of conservation (r) Dalton
is 2 : 1. of mass
(iv) Dalton’s atomic theory support law of conservation (D) Law of gaseous (s) Joseph Proust
of mass.
volumes
Which of the following pairs of statements is true according
(a) A – (s), B – (r), C – (p), D – (q)
to Dalton’s atomic theory ?
(b) A – (p), B – (r), C – (s), D – (q)
(a) (i) and (ii) (b) (ii) and (iii)
(c) A – (r), B – (p), C – (s), D – (q)
(c) (ii) and (iv) (d) (i) and (iv)
(d) A – (q), B – (s), C – (r), D – (p)
111. Choose the correct option based on following statements.
Here ‘T’ stands for true and ‘F’ stands for false statement. 115. Match the columns
(i) Molecular mass of cane sugar (C12H22O11) is 182 amu. Column-I Column-II
(ii) 1 mole of cane sugar contains 6.022 × 1023 molecules (A) C6H5NH2 (p) 84
of cane sugar. (B) C6H6 (q) 100
(iii) 34.20 g of cane sugar contains 6.022 × 1021 molecules (C) C6H12 (r) 93
of cane sugar. (D) CaCO3 (s) 78
(a) TTF (b) TFT (a) A – (p), B – (r), C – (q), D – (s)
(c) FTF (d) FTT (b) A – (r), B – (s), C – (p), D – (q)
(c) A – (r), B – (p), C – (q), D – (s)
MATCHING TYPE QUESTIONS (d) A – (r), B – (q), C – (s), D – (p)
112. Match the items of Column I, II and III appropriately and 116. Match the columns.
choose the correct option from the codes given below. Column-I Column-II
Column I Column II Column III (A) 88 g of CO2 (p) 0.25 mol
(Multiple) (Prefix) (Symbol) (B) 6.022 × 1023 molecules (q) 2 mol
(A) 10–15 (p) Kilo (i) m of H2O
(B) 10 –3 (q) yotta (ii) f (C) 5.6 litres of O2 at STP (r) 1 mol
(C) 103 (r) milli (iii) k (D) 96 g of O2 (s) 6.022 × 1023 molecules
(D) 1024 (s) femto (iv) Y (E) 1 mol of any gas (t) 3 mol
(a) A – (s), (ii); B – (r), (i); C – (p), (iii); D – (q), (iv) (a) A – (q), B – (r), C – (p), D – (t), E – (s)
(b) A – (p), (ii); B – (q), (iii); C – (r), (i); D – (s), (iv) (b) A – (r), B – (q), C – (p), D – (t), E – (s)
(c) A – (q), (iv); B – (p), (ii); C – (p), (i); D – (r), (iii) (c) A – (q), B – (p), C – (r), D – (t), E – (s)
(d) A – (r), (iii); B – (p), (ii); C – (s), (i); D – (q), (iv) (d) A – (q), B – (r), C – (p), D – (s), E – (t)
EBD_7207
8 SOME BASIC CONCEPTS OF CHEMISTRY

117. Match the mass of elements given in Column I with the 122. Assertion : Volume of a gas is inversely proportional to the
number of moles given in Column II and mark the appropriate number of moles of gas.
choice. Choose the correct codes formt he options given Reason : The ratio by volume of gaseous reactants and
below. products is in agreement with their mole ratio.
Column-I Column-II 123. Assertion : Equal moles of different substances contain
(A) 28 g of He (p) 2 moles same number of constituent particles.
(B) 46 g of Na (q) 7 moles Reason : Equal weights of different substances contain
the same number of constituent particles.
(C) 60 g of Ca (r) 1 mole
124. Assertion : The empirical mass of ethene is half of its
(D) 27 g of Al (s) 1.5 mole molecular mass.
(a) A – (s), B – (r), C – (q), D – (p) Reason : The empirical formula represents the simplest
(b) A – (p), B – (r), C – (q), D – (s) whole number ratio of various atoms present in a
(c) A – (r), B – (q), C – (p), D – (s) compound.
(d) A – (q), B – (p), C – (s), D – (r)
118. Match the columns. CRITICAL THINKING TYPE QUESTIONS
Column-I Column-II 125. What are the significant figure(s) in a broken “ruler”
(Physical quantity) (Unit) show below?
(A) Molarity (p) mol
(B) Mole fraction (q) Unitless 0.0 1.0 2.0
(C) Mole (r) mol L–1 (A) 1 (B) 2
(D) Molality (s) mol kg–1 (C) 3 (D) 0
(a) A, B and C (b) A, B, D
(a) A – (r), B – (q), C – (s), D – (p)
(c) A only (d) A and B
(b) A – (r), B – (p), C – (q), D – (s)
126. Which one of the following sets of compounds correctly
(c) A – (r), B – (q), C – (p), D – (s)
illustrate the law of reciprocal proportions?
(d) A – (q), B – (r), C – (p), D – (s) (a) P2O3, PH3, H2O (b) P2O5, PH3, H2O
(c) N2O5, NH3, H2O (d) N2O, NH3, H2O
ASSERTION-REASON TYPE QUESTIONS
127. If we consider that 1/6, in place of 1/12, mass of carbon
Directions : Each of these questions contain two statements, atom is taken to be the relative atomic mass unit, the mass
Assertion and Reason. Each of these questions also has four of one mole of a substance will
alternative choices, only one of which is the correct answer. You (a) decrease twice
have to select one of the codes (a), (b), (c) and (d) given below. (b) increase two fold
(a) Assertion is correct, reason is correct; reason is a correct (c) remain unchanged
explanation for assertion. (d) be a function of the molecular mass of the substance
(b) Assertion is correct, reason is correct; reason is not a 128. The maximum number of molecules are present in
correct explanation for assertion (a) 15 L of H2 gas at STP (b) 5 L of N2 gas at STP
(c) Assertion is correct, reason is incorrect (c) 0.5 g of H2 gas (d) 10 g of O2 gas
(d) Assertion is incorrect, reason is correct. 129. How many moles of magnesium phosphate, Mg3(PO4)2 will
119. Assertion : Significant figures for 0.200 is 3 whereas for contain 0.25 mole of oxygen atoms?
200 it is 1. (a) 1.25 × 10–2 (b) 2.5 × 10–2
Reason : Zero at the end or right of a number are significant (c) 0.02 (d) 3.125 × 10–2
provided they are not on the right side of the decimal point. 130. Volume occupied by one molecule of water
120. Assertion : 1.231 has three significant figures. (density = 1 g cm–3) is : ]
Reason : All numbers right to the decimal point are (a) 9.0 × 10–23 cm3 (b) 6.023 × 10– 23 cm3
significant. –23
(c) 3.0 × 10 cm 3 (d) 5.5 × 10– 23 cm3
121. Assertion : One atomic mass unit is defined as one twelfth 131. The number of atoms in 0.1 mol of a triatomic gas is :
of the mass of one carbon - 12 atom. (NA = 6.02 ×1023 mol–1)
Reason : Carbon-12 isotope is the most abundunt isotope (a) 6.026 × 1022 (b) 1.806 × 1023
of carbon and has been chosen as standard. (c) 3.600 × 1023 (d) 1.800 × 1022
SOME BASIC CONCEPTS OF CHEMISTRY 9

132. 1 c.c. N2O at NTP contains : 140. Which of the following is the correct empirical and
molecular formulae of a compound, if the molecular mass
1.8
(a) 10 22 atoms of a compound is 80 and compound contains 60% of C,
224 5% of H and 35% of N ?
6.02 (a) C2H2N ; C4H4N2 (b) C3H4N2 ; C6H8N4
(b) 10 23 molecules (c) C2H4N2 ; C4H8N4 (d) C2H2N ; C2H2N
22400
141. Which of the following is the correct empirical and
1.32 molecular formulae of a compound, if the molecular mass
(c) 10 23 electrons
224 of a compound is 93 and compound containing 77.43% of
(d) All of the above C, 7.53% of H and 15.05% of N ?
(a) C3H3.5N1.5 and C6H7N
133. How much time (in hours) would it take to distribute one
(b) C6H7N and C6H7N
Avogadro number of wheat grains if 10 20 grains are
(c) C3H3N and C6H7N
distributed each second ?
(d) C3H3N and C6H6N2
(a) 0.1673 (b) 1.673
142. Liquid benzene (C6H6) burns in oxygen according to the
(c) 16.73 (d) 167.3
equation 2C 6 H 6 (l ) 15O 2 ( g ) 12 CO 2 ( g ) 6 H 2 O ( g )
134. Arrange the following in the order of increasing mass (atomic
How many litres of O2 at STP are needed to complete the
mass: O = 16, Cu = 63, N = 14)
combustion of 39 g of liquid benzene?(Mol. wt. of O2 = 32,
I. one atom of oxygen C6H6 = 78)
II. one atom of nitrogen (a) 74 L (b) 11.2 L
III. 1 × 10–10 mole of oxygen (c) 22.4 L (d) 84 L
IV. 1 × 10–10 mole of copper 143. Assuming fully decomposed, the volume of CO2 released at
(a) II < I < III < IV (b) I < II < III < IV STP on heating 9.85 g of BaCO3 (Atomic mass, Ba = 137)
will be
(c) III < II < IV < I (d) IV < II < III < I
(a) 2.24 L (b) 4.96 L
135. If 1.5 moles of oxygen combines with Al to form Al 2O3,
(c) 1.12 L (d) 0.84 L
the mass of Al in g [Atomic mass of Al = 27] used in the
144. The mass of BaCO3 produced when excess CO2 is bubbled
reaction is
through a solution of 0.205 mol Ba(OH)2 is :
(a) 2.7 (b) 54
(a) 81 g (b) 40.5 g
(c) 40.5 (d) 81
(c) 20.25 g (d) 162 g
136. Which one of the following is the lightest?
145. For the reaction Fe2O3 + 3CO2 2Fe + 3CO2, the volume
(a) 0.2 mole of hydrogen gas
of carbon monoxide required to reduce one mole of ferric
(b) 6.023 × 1022 molecules of nitrogen
oxide is
(c) 0.1 g of silver
(a) 67.2 dm3 (b) 11.2 dm3
(d) 0.1 mole of oxygen gas
(c) 22.4 dm3 (d) 44.8 dm3
137. In a compound C, H and N atoms are present in 9 : 1 : 3.5 by
146. How many moles of lead (II) chloride will be formed from a
weight. Molecular weight of compound is 108. Molecular reaction between 6.5 g of PbO and 3.2 g of HCl ?
formula of compound is (a) 0.044 (b) 0.333
(a) C2H6N2 (b) C3H4N (c) 0.011 (d) 0.029
(c) C6H8N2 (d) C9H12N3. 147. Fat is an important source of energy and water, this is
138. The empirical formula of an acid is CH2O2, the probable important for the desert animals like camel which store fat
molecular formula of acid may be : in its hump and provide water and energy. How many
(a) C3H6O4 (b) CH2O grams and moles of H2O are produced from the
combustion of fat C57H110O6 from 450 gram of fat stored
(c) CH2O2 (d) C2H4O2
in hump of camel ?
139. A gaseous hydrocarbon gives upon combustion 0.72 g of
water and 3.08 g. of CO2. The empirical formula of the 163
C57 H110O6 O2 57CO 2 55H 2O
hydrocarbon is : 2
(a) C2H4 (b) C3H4 (a) 500.56 , 27.80 (b) 450, 26.80
(c) 580, 25.0 (d) 400, 26.6
(c) C6H5 (d) C7H8
EBD_7207
10 SOME BASIC CONCEPTS OF CHEMISTRY
148. Which of the following option represents correct limiting 150. If maximum fluoride ion presence was set to be 4 ppm
reagents in reactions (i), (ii) and (iii) respectively. number of moles of fluoride in 10 ml drinking water ?
(i) C + O2 CO2
(a) 2.10 × 10–3 (b) 2.10 × 10–2
(26g) (20g)
(ii) N2 + 3H2 2NH3 (c) 3.10 × 10–3 (d) 3.3 × 10–2
(60g) (80g) 151. The increasing order of molarity with 25 gm each of
(iii) P4 + 3O2 P4O6 NaOH, LiOH, Al(OH)3, KOH, B(OH)3 in same volume of
(100g) (200g) water?
(a) C, N2, O2 (b) C, N2, P4
(a) Al(OH)3 < B(OH)3 < KOH < NaOH < LiOH
(c) O2, H2, P4 (d) O2, N2, P4
149. 10 mL of 2(M) NaOH solution is added to 200 mL of (b) LiOH < NaOH < KOH < B(OH)3 < Al(OH)3
0.5 (M) of NaOH solution. What is the final concentration ? (c) LiOH < NaOH < B(OH)3 < KOH < Al(OH)3
(a) 0.57 (M) (b) 5.7 (M)
(c) 11.4 (M) (d) 1.14 (M) (d) NaOH < LiOH < B(OH)3 < Al(OH)3 < KOH
SOME BASIC CONCEPTS OF CHEMISTRY 11

18. (d) Litre (L) is not an SI unit. It is used for measurement of


FACT / DEFINITION TYPE QUESTIONS volume of liquids.
1. (c) By sublimation since I2 sublimes. 19. (c) yocto = 10–24
2. (d) It forms the basis of gravity separation. 5 5
20. (c) C F 32 41 32 = 5°C
3. (d) Candle is a heterogeneous mixture of wax and threads. 9 9
Copper is an element while bottled water and table salt It will be cold.
are compounds. 21. (a) Out of two 3.929 g is more accurate and will be reported
4. (c) Glucose is a pure substance hence its constituents as 3.93 after rounding off.
cannot be separated by simple physical method. 22. (b)
5. (a) In case of B, none of O2 and H2 is a compound since 23. (a) 0.00016 can be written as 1.6 × 10–4 in scientic notation.
compound consist of two or more different atoms. 24. (d) Both Y and X are neither precise nor accurate as the
6. (c) two values in each of them are not close. With respect
7. (d) Silver is an element, water is a compound and air is a to X & Y, the values of Z are close & agree with the
mixture. true value. Hence, both precise & accurate.
8. (c) Attraction between particles in solid is maximum and 25. (c) On calculation we find
hence their movement is minimum amongst the
phases. (29.2 – 20.2)(1.79 ´105 )
= 1.17×106
Attraction between particles in gases is minimum and 1.37
hence their movements is maximum amongst the three As the least precise number contains 3 significant
phases. figures therefore, answers should also contains 3
Attraction between the particles and their movements significant figures.
in liquids is intermediate i.e., between solids and gases. 26. (d) We know that all non-zero digits are significant and
9. (c) A mixture may contain any number of components in the zeros at the beginning of a number are not
any ratio. significant. Therefore number 161 cm, 0.161 cm and
e.g. air is a mixture of various gases. 0.0161cm have 3, 3 and 3 significant figures
10. (d) It is a solution and is a mixture of sucrose and water. respectively.
11. (b) One fermi is 10–13 cm. 27. (b) Given P = 0.0030m, Q = 2.40m & R = 3000m. In P(0.0030)
12. (c) Exa = 1018 initial zeros after the decimal point are not significant.
13. (d) 1 zepto = 10–21 Therefore, significant figures in P(0.0030) are 2.
Similarly in Q (2.40) significant figures are 3 as in this
14. (a) Joule is the unit of work and Pascal is unit of pressure.
case final zero is significant. In R = (3000) all the zeros
J Work Nm
JPa 1 m3 are significant hence, in R significant figures are 4
Pa Pressure 2
Nm because they come from a measurement.
15. (a) 1 m3 of water 106 cm3 of water 28. (a)
Mass of 106 cm3 water 29. (c) If zero is used to locate the decimal point it is
= 106 cm3 × 1 g cm3 ( density of H2O = 1 g cm3) considered as a significant figure. In 50.000 all zero are
significant.
10 6 30. (a) Sum of the figures 29.4406, 3.2 and 2.25 is 34.8906. The
= 106 g = kg = 103 kg = 1000 kg
103 sum should be reported to the first place of decimal as
(b) Weight of normal adult man = 65 kg 3.2 has only one decimal place. After rounding off the
Weight of 1 m3 of water is highest. sum is 34.9. Hence number of significant figures is
(c) Density of Hg = 13.6 g cm–3 three.
Volume of Hg = 10 L = 10 × 1000 = 104 cm3 31. (d) 10.3106 g has 6 significant figures. Since all
non-zero digits are significant and a zero becomes
Weight of Hg = 13.6 × 104 = 136000 g = 136 kg
significant if it appears between two non-zero digits.
16. (d) Smallest and largest amount of energy respectively
are eV and L-atm. 43.0 0.0243
32. (b) = 0.7683088
1 eV = 1.6 × 10–19 J 0.340 4
1L -atm = 101.325 J The least precise term has two significant figures
17. (c) (leaving the exact number). Hence after rounding off
correct answer is 0.77.
EBD_7207
12 SOME BASIC CONCEPTS OF CHEMISTRY
33. (a) 2.6 has two significant figures. 50. (b) The number of molecules of N2 and X are same. Hence
0.260 has three significant figures. they must have the same molecular weights.
0.002600 has four significant figures. X is CO.
2.6000 has five significant figures. 51. (c)
Energy 52. (c) In law of reciprocal proportions, the two elements
34. (d) which can be shown combining with the third element, must combine with
volume
Pressure each other in the same ratio or multiple of that Ratio of
S and O when combine with C is 2 : 1. Ratio of S and O is
= Force Work (energy/di stance) Energy
SO2 is 1 : 1
area Area Volume 53. (d) Avogadro’s law is independent of the reactive or
35. (b) X Y R S unreactive nature of the gases.
ng mg pg qg
According to Avogadro’s law equal volumes of gases
n + m = p + q by law of conservation of mass. at the same temperature and pressure should contain
36. (a) CaCO3 CaO + CO2 equal number of molecules.
20 g 8.8 g 11.2 g
mass of reactant = mass of products = 20g. 54. (c)
Hence the law of conservation of mass is obeyed. 1
37. (a) 38. (a) 55. (b) 1 amu = of the mass of C-12.
12
39. (b) NaHCO3 CH3COOH Residue CO 2 56. (b) The modern atomic weight scale is based on C12.
4.2g 10.0g 12.0g 2.2g
57. (d) Molecular weight of ZnSO 4 .7H 2O
Mass of reactants = 4.2 + 10.0 = 14.2 g
Mass of products = 12.0 + 2.2 = 14.2 g = 65 + 32 + (4 × 16) + 7(2 × 1 + 16) = 287.
Hence, given reaction illustrate law of conservation
percentage mass of zinc (Zn)
of mass.
mass of H 2 combined 4 1 65
40. (c) I experiment : 100 22.65%
mass of O 2 combined 32 8 287
58. (b) Average atomic mass of neon
mass of H 2 combined 50 1 = 20 × 0.9051 + 21 × 0.0027 + 22 × 0.0922
II experiment :
mass of O 2 combined 400 8 = 20.187 u
Hence both law of conservation of mass and constant 59. (a) (78.9183361) × (0.5069) + (80.916289) × (0.4931)
composition is obeyed. 60. (a) Mass of oxygen atom is 15.995 amu, becasue 1 amu
41. (c) The H : O ratio in water is fixed, irrespective of its = 1.66056 × 10–24 g, hence 15.995 × value of 1 amu
source. Hence it is law of constant composition. give the value equal to option (a).
42. (a) Constant proportions according to which a pure 23
chemical compound always contains same elements 2.824784 10
61. (d) = 14 amu
combined together in the same definite proportion of 1.66056 10–24
weight. Where 1.66056 × 10–24 is equal to one atomic mass
43. (c) (amu)
44. (d) SnCl2 SnCl4 62. (b) Gram molecular weight of CO = 12 + 16 = 28 g
119 : 2 × 35.5 119 : 4 × 35.5 6.023 × 1023 molecules of CO weight 28 g
Chlorine ratio in both compounds is
28 4.65 10 23 g
= 2 × 35.5 : 4 × 35.5 = 1 : 2 1 molecule of CO weighs =
6.02 1023
45. (d) In CuO and Cu2O the O : Cu is 1 : 1 and 1 : 2 respectively.
This is law of multiple proportion. 63. (b) Molecular weight of SO2 = 32 + 2 × 16 = 64
46. (c) 64 g of SO2 occupies 22.4 litre at STP
47. (b) Law of multiple proportion. As the ratio of oxygen 22.4
which combine with fix weights of 1 g of nitrogen bears 240 g of SO2 occupies = 240 84 litre at STP
64
a simple whole number ratio
0.57 : 1 : 12 : 1.7031 : 2 : 3 64. (a) 1 mol CCl4 vapour = 12 + 4 × 35.5
48. (b) = 154 g 22.4 L at STP
49. (c) According to Avogadro's law "equal volumes of all 154
gases contain equal number of molecules under similar Density = gL 1 6.875 gL 1
22.4
conditions of temperature and pressure". Thus if 1 L 65. (b) 6.02 × 1023 molecules of CO =1mole of CO
of one gas contains N molecules, 2 L of any other gas 6.02 × 1024 CO molecules = 10 moles CO
under the same conditions of temperature and pressure = 10 g atoms of O = 5 g molecules of O2
will contain 2N molecules.
SOME BASIC CONCEPTS OF CHEMISTRY 13

4.4 1
66. (a) 4.4 g CO2 = = 0.1 mol CO2 (mol. wt. of CO2 = 44) No. of molecules of O2 =
44 32
= 6 × 10 molecules = 2 × 6 × 1022 atoms of O.
22
4
67. (b) 2g of H2 means one mole of H2, hence contains No. of molecules of N2 =
28
6.023 × 1023 molecules. Others have less than one mole,
1 4 1 1
so have less no. of molecules. Ratio of no. of molecules = : : = 7 : 32
32 28 32 7
558.5
68. (a) Fe (no. of moles) = = 10 moles = 10NA atoms.
55.85 weight 50
No. of moles in 60 g of C = 60/12 = 5 moles = 5NA 80. (d) No. of moles = = = 0.14 mole
mol. wt . 342
atoms.
69. (c) 16 g CH4 is 1 mol. Hence number of molecules 81. (c) Mass of 1 electron = 9.11 × 10–28 g
= Avogadro number = 6.023 × 1023. Mass of 1 mole (6.02 × 1023) electrons
70. (c) M. Wt of Na2SO4.10H2O is 322 g which contains 224 = 9.11 × 10–28 × 6.02 × 1023g
g oxygen. = 55 × 10–5 g = 55 × 10–5 × 103 mg = 0.55 mg.
32.2 g will contain 22.4 g oxygen. 1
82. (b) H2 + O H2 O
71. (a) 21% of 1 litre is 0.21 litre. 2 2
22.4 litres = 1 mole at STP 10g 64g
10 64
0.21 5 mol 2 mol
0.21 litre = 0.0093 mol 2 32
22.4 In this reaction oxygen is the limiting agent. Hence
72. (d) At S.T.P. 22.4 litre of gas contains 6.023 × 1023
molecules amount of H2O produced depends on the amount of
O2 taken
molecules in 8.96 litre of gas
0.5 mole of O2 gives H2O = 1 mol
6.023 10 23 8.96 2 mole of O2 gives H2O = 4 mol
= 24.08 10 22
22.4 83. (c) No. of molecules
73. (b) Mass of one molecule of Water 44
Moles of CO2 = 1 NA
44
18
= 3 10 23 g 3 10 26 Kg 48
6.023 10 23 Moles of O3 = 1 NA
48
74. (d) 1 molecule of CO2 has one atom of C and two atoms of
8
oxygen. Moles of H2 = 4 4NA
1 mole of CO2 has = 6.02 × 1023 atoms of C 2
= 2 × 6.02 × 1023 atoms of O 64
1
Moles of SO2 = NA
75. (a) Given, V = 1.12 × 10–7 cm3 64
22400 cm3 at NTP = 6.02 × 1023 molecules 84. (b) Molecular weight of C60H122 = (12 × 60) + 122 = 842.
Therefore weight of one molecule
6.02 1023
1.12 × 10–7 cm3 at NTP = 1.12 10 7 Molecular weight of C 60 H 122
22400 =
= 3.01 × 1012 molecules. Avagadro' s number
76. (b) Total atoms in 1 molecule of C12H22O11 842
1.4 10 21 g
= 12 + 22 + 11 = 45 6.023 10 23
Total atoms in 1 mole of C12H22O11 85. (c) 50% of X (Atomic mass 10), 50% of Y (Atomic mass 20).
= 45 × 6.02 × 1023 atoms/mol.
Relative number of atoms of X = 50 5 and than
77. (a) 22.4 L of O 2 at STP = 32 g 10
50
Y= 2.5
32 20
1 L of O2 at STP = 1 = 1.428 g = 1.43 g
22.4 Simple Ratio 2 : 1. Formula X2Y
78. (c) Given V = 2 L, Molarity = 0.5M, Moles = ? 86. (a) Element % Atomic Relative Simple ratio
mass no.of atoms of atoms
Molarity No. of moles of solute or Moles
0.5
2 80 6.66
V of solution in L C 80 12 6.66 1
12 6.66
Moles = 2 × 0.5 = 1.0
20 20.0
79. (a) Let mass of O2 = 1 g H 20 1 20.0 3
1 6.66
Mass of N2 = 4g
Empirical formula is CH3
EBD_7207
14 SOME BASIC CONCEPTS OF CHEMISTRY
87. (a) Empirical formula of compound = CH2 5
Molecular mass of the compound = 42 1 mole of NH3 requires = = 1.25 mole of O2.
4
n = 42/14 = 3 Hence O2 is consumed completely.
Hence molecular formula = C3H6
Equivalent mass
88. (d) C 54.55 54.55/12 = 4.5 4.5/2.27 = 2 97. (a) Molarity = Normality ×
H 9.099.09/1= 9.09 9.09/2.27 = 4 Molecular mass
O 36.36 36.36/16 = 2.27 2.27/2.27= 1 M
= 0.2 = 0.1 M
Hence empirical formula of the compound = C2H4O 2 M
89. (a) Mass ratio of H : C = 1 : 12 Normality
However, given mass ratio of H : C = 1 : 3 98. (a) Molarity
Replaceable hydrogen atom
Therefore, for every C atom, there are 4 H atoms, hence
empirical formula = CH4 H2SO4 is dibasic acid.
Molar solution of H2SO4 = N/2 H2SO4
90. (a) Element Percentage Atomic Atomic Simple
weight ratio ratio 99. (b) Given N1 = 10N, V1 = 10 ml, N2 = 0.1N, V2 = ?

C 38.71 12 38.71 3.23 N1V1 N 2V2


3.23 1
12 3.23 or 10 × 10 = 0.1 × V2
H 9.67 1 9.67 9.67
9.67 3 10 10
1 3.23 or V2 , V2 = 1000 ml
O 100 16 51.62 3.23 0.1
(38.71 9.67) 3.23 1 Volume of water to be added
16 3.23
51.62 = V2 – V1 = 1000 – 10 = 990 ml.
Mass of solute
Thus empirical formula is CH3O. 100. (b) ppm 106
Mass of solution
91. (d) At. Rel.No. of Simple
Element % 6 10 –3
Mass Atoms Ratio ppm 10 6.
1000
C 75 12 75 /12 6.25 1 101. (a) 5 M H2SO4 = 10 N H2SO4,
H 25 1 25 /1 25 4 ( Basicity of H2SO4 = 2)
N1V1 = N2V2,
Empirical formula is CH4. 10 × 1 = N2 × 10 or N2 = 1 N
92. (b) Mg 2 HCl MgCl 2 H2 102. (c) Among all the given options molarity is correct
1 mole 1 mole because the term molarity involve volume which
1
mole
1
mole (12g of Mg
1
mol ) increases on increasing temperature.
2 2 2
6.02 1020
93. (a) We know that 103. (b) Moles of urea present in 100 ml of sol.=
6.02 10 23
N2 + 3H2 2NH3
28 g 6 g 34 g 6.02 10 20 1000
M 0.01M
14 g 3 g 17 g 6.02 10 23 100
Here given H2 is 3 kg and N2 is 20 kg but 3 kg of H2 can [ M = Moles of solute present in 1L of solution]
only react with 14 g of N2 and thus the obtained NH3 104. (b) From the molarity equation.
will be of 17 kg. M1V1 + M2V2 = MV
94. (d) 2H2 + O2 2H2O Let M be the molarity of final mixture,
4g 32g 36g M1V1 +M V2
4kg 32 kg 36kg 2
M= where V = V1 + V2
V
95. (c) C2H4 + 3 O2 2CO2 + 2H2O
28 g 96 g 480 1.5 520 1.2
M 1.344 M
28 g of C2H4 undergo complete combustion by 480 520
= 96 g of O2 STATEMENT TYPE QUESTIONS
2.8 kg of C2H4 undergo complete combustion by
= 9.6 kg of O2. 105. (c) Both solids and liquids have definite volume, but
96. (c) According to stoichiometry they should react as follow gases do not.
Solids have their own shape, but liquids and gases
4NH 3 5O 2 4NO 6H 2 O
takes the shape of the container in which they are put
4 mole of NH3 requires 5 mole of O2. in.
SOME BASIC CONCEPTS OF CHEMISTRY 15
106. (b) For statement (ii), it is not necessary that all 114. (a) 115. (b) 116. (a)
components of a h eterogeneous mixture are
28
observable to naked eyes for example blood is a 117. (d) A : 28 g of He = = 7 mol
heterogeneous mixture whose components are not 4
visible to naked eyes. For statement (iv) air is a 46
homogeneous mixture of various gases. B : 46 g of Na = = 2 mol
23
107. (d) Mass of a substance is the amount of matter present
in it while weight is the force exerted by gravity on an 60
object. C : 60 g of Ca = = 1.5 mol
40
Mass is constant while weight may vary from one
place to another due to gravity. 27
D : 27 g of Al = = 1 mol
SI unit of both mass and weight is kilogram. 27
108. (c) 27.3 days = 27.3 × 24 hours 118. (c)
= 655.2 hours
27.3 days = 27.3 × 24 × 60 minutes ASSERTION- REASON TYPE QUESTIONS
= 39312 minutes
27.3 days = 27.3 × 24 × 60 × 60 seconds 119. (c)
= 2358720 seconds 120. (d) 1.231 has four significant figures all no. from left to
109. (c) For statement (i), T = The other name of Gay-Lussac’s right are counted, starting with the first digit that is
law is law of definite proportions by volume. not zero for calculating the no. of significant figure.
For statement (ii), F = Law of conservation of mass is 121. (b)
valid for both physical and chemical change. 122. (d) We know that from the reaction H2 + Cl2 2HCl that
For statement (iii), F = Law of definite proportion is the ratio of the volume of gaseous reactants and
valid for each compound individually and not for products is in agreement with their molar ratio.The ratio
comparing two different compounds. of H2 : Cl2 : HCl volume is 1: 1: 2 which is the same as
For statement (iv), T = x/y must be a simple whole their molar ratio. Thus volume of gas is directly related
number ratio and must be a positive integer. to the number of moles. Therefore, the assertion is
For statement (v), F = Equal volumes of all gases false but reason is true.
under similar conditions of temperature and pressure
123. (c) Equal moles of different substances contain same
contain equal number of molecules.
number of constituent particles but equal weights of
110. (c) For statement (i) : H, O, C, N = All have different different substances do not contain the same number
chemical properties. of consituent particles.
For statement (ii) : It is true as per Dalton’s postulate.
124. (a)
For statement (iii) : N : O = 1 : 1 (NO)
For statement (iv) : Dalton’s postulates says, atoms
CRITICAL THINKING TYPE QUESTIONS
can neither be created nor destroyed.
111. (c) Molecular mass of cane sugar (C12H22O11) 125. (b) For, 0.0 significant figure is zero. For 0.1 to 0.9
= 12 × 12 + 22 × 1 + 11 × 16 significant figure will be 1 whereas from 1.0 to 2.0
= 342 amu significant figures will be 2.
1 mole of cane sugar (C12H22O11) = 342 g 126. (a) In law of reciprocal proportions, the two elements
(Molecular mass of cane sugar = 342 g) combining with the third element, must combine with
342 g of cane sugar contain = 6.022 × 1023 molecules each other in the same ratio or multiple of that ratio.
6.022 10 23 P2O3, PH3 and H2O correctly illustrate the law of
34.20 g of cane sugar contain = 34.20 reciprocal proportions. Ratio in the number of atoms
342
of hydrogen and oxygen combining with one P is 3 :
= 6.022 × 1022 molecules.
1.5 i.e., 2 : 1.
MATCHING TYPE QUESTIONS 127. (a) Relative atomic mass
Mass of one atom of the element
112. (a) = th
113. (b) Terminal zeros are not significant if there is no 1/12 part of the mass of one atom of Carbon 12
decimal i.e., 290 contains two significant figures Mass of one atom of the element
whereas in 29900. there are 5 significant figures; or 12
1.23 × 1.331 = 1.63713 but keeping the mind the 1.23 mass of one atom of the C 12
has only few significant figures i.e., only three Now if we use 1/6 in place of 1/12 the formula becomes
significant figures, so result should also be reported Mass of one atom of element
in three significant figures only. Thus 1.6373 should Relative atomic mass = 6
Mass of one atom of carbon
be rounded off to 1.64. Value 1.783 is rounded off to
Relative atomic mass decrease twice
2, so has only one significant figure.
EBD_7207
16 SOME BASIC CONCEPTS OF CHEMISTRY

128. (a) No. of molecules in different cases 131. (b) The number of atoms in 0.1 mole of a triatomic gas
(a) 22.4 litre at STP contains = 0.1 × 3 × 6.023 × 1023.
= 6.023 × 1023 molecules of H2 = 1.806 × 1023
15 132. (d) At NTP 22400 cc of N2O = 6.02 × 1023 molecules
15 litre at STP contains = 6.023 10 23
22.4 6.02 10 23
= 4.03 × 1023 molecules of H2 1 cc N2O = molecules
22400
(b) 22.4 litre at STP contains
3 6.02 10 23 1. 8
= 6.023×1023 molecules of N2 atoms = 1022 atoms
22400 224
5
5 litre at STP contains = 6.023 10 23 No. of electrons in a molecule of N 2O = 7 + 7 + 8 = 22
22.4
Hence no. of electrons
= 1.344 × 1023 molecules of N2
6.02 10 23
22 electrons = 1.32 ´10
23
(c) 2 gm of H2= 6.023×1023 molecules of H2
22400 224
0.5 133. (b) If 10 grains are distributed in one sec, 6.023 × 1023
20
0.5 gm of H2= 6.023 10 23
2 grains will be distributed in
= 1.505 × 1023 molecules of H2
6.023 1023 1
(d) Similarly 10 g of O2 gas 1.673 hrs
1020 60 60
10 134. (a) Mass of 6.023 × 1023 atoms of oxygen = 16 g
= 6.023 10 23 molecules of O2
32 Mass of one atom of oxygen
= 1.88 × 1023 molecules of O2 16 23
= 2.66 10 g
Thus (a) will have maximum number of molecules 6.023 10 23
129. (d) 1 Mole of Mg3(PO4)2 contains 8 mole of oxygen atoms Mass of 6.023 × 1023 atoms of nitrogen = 14 g
8 mole of oxygen atoms 1 mole of Mg3(PO4)2 Mass of one atom of nitrogen
1 14
0.25 mole of oxygen atom 0.25 mole of 2.32 10 23
g
8 6.023 1023
Mg3(PO4)2 Mass of 1 × 10–10 mole of oxygen = 16 × 10–10
Mass of 1 mole of copper = 63 g
2 mole of Mg3(PO4)2
3.125 10 Mass of 1 mole of oxygen = 16 g
Mass Mass of 1 × 10–10 mole of copper = 63 × 1 × 10–10
130. (c) Density
Volume = 63 × 10–10
So, the order of increasing mass is II < I < III < IV.
1gram
1 gram cm–3 = 135. (b) The equation for the formation of Al 2 O3 can be
cm 3 represented as
Mass 1gram 2Al 3 / 2O 2 Al2 O3
Volume = 3
1cm 3 2 moles
Density 1gram cm 1.5 moles 1 mole

Thus, 1 mole of alumina is obtained by the reaction of


Volume occupied by 1 gram water = 1 cm3
1.5 moles of oxygen and 2 moles of aluminium. Thus,
or Volume occupied by the amount of aluminium
6.023 1023 = 2 × 27 g = 54 g. [mol. mass of Al = 27]
molecules of water = 1 cm3 136. (c) (a) Weight of H2 = mole × molecular wt.
18
= 0.2 × 2 = 0.4 g
1 23
(b) 6.023 × 10 = 1 mole
[ 1g water = moles of water]
18 Thus 6.023 × 1022 = 0.1 mole
Thus volume occupied by 1 molecule of water Weight of N2 = 0.1 × 28 = 2.8 g
1 18 (c) Weight of silver = 0.1 g
= 23
cm 3 = 3.0×10–23 cm3. (d) Weight of oxygen = 32 × 0.1 = 3.2 g
6.023 10
SOME BASIC CONCEPTS OF CHEMISTRY 17

137. (c) Percentage R.N.A Simplest ratio 15.05


Number of moles of N = = 1.075
9 3 14.00g / mol
C 9 = 3 1.074 is the smallest value, division by it gives a ratio
12 4
of C : H : N
1 = 5.9 : 6.9 : 1
H 1 =1 4 =6:7:1
1
Empirical formula = C6H7N
3.5 1 Empirical formula weight = 6 × 12 + 7 + 14 = 93
N 3.5 = 1
14 4 Molecular mass
n 1
Empirical formula = C3H4N Empirial formula weight
(C3H4N)n = 108 Molecular formula = 1 × C6H7N = C6H7N
(12 × 3 + 4 × 1 + 14)n = 108 142. (d) 2C 6 H 6 15O 2 (g ) 12 CO 2 (g ) 6 H 2 O(g )
(54)n = 108 2 (78) 15(32 )
108 156 gm of benzene required oxygen = 15 × 22.4 litre
n= =2
54 15 22.4
molecular formula = C6 H8N2 1 gm of benzene required oxygen = litre
156
138. (c) The acid with empirical formula CH2O2 is formic acid, 39 gm of Benzene required oxygen
H—COOH.
15 22.4 39
139. (d) 18 gm, H2O contains = 2 gm H = 84.0 litre
156
2 143. (c) BaCO3 BaO CO 2
0.72 gm H2O contains = 0.72 gm 0.08 gm H
18 197 gm
44 gm CO2 contains = 12 gm C 197 gm of BaCO3 released carbon dioxide
12
= 22.4 litre at STP
3.08 gm CO2 contains = 3.08 0.84 gm C 22.4
44 1 gm of BaCO3 released carbon dioxide = litre
0.84 0.08 197
C:H= : = 0.07 : 0.08 = 7 : 8 9.85 gm of BaCO3 released carbon dioxide
12 1
Empirical formula = C7H8 22.4
140. (a) Let 100 g of compound be there. = 9.85 = 1.12 litre
197
35 144. (b) Ba(OH)2 CO2 BaCO3 H2 O
Number of moles of Nitrogen = = 2.5
14 n mol n mol
5 n mol Ba(OH)2 = n mol BaCO3
Number of moles of Hydrogen = = 4.9 0.205 mol Ba (OH) 2 0.205 mol BaCO3
1.008
Wt. of substance = No. of moles × Molecular mass
60
Number of moles of Carbon = = 4.9 = 0.205 × 197.3 = 40.5 g
12.01 145. (a) Fe2O3 + 3CO 2Fe + 3CO2
Since 2.5 is the smallest value division by it give 1 vol. 3 vol. 2 vol. 3 vol.
ratio 1 mol. 3 mol. 2 mol. 3 mol.
N:H:C ( vol% = mol%)
1 : 1.96 : 1.96 One gram mol of any gas occupies 22.4 litre at NTP.1
mol of Fe2O3 requires 3 mol of CO for its reduction
=1:2:2
i.e., 1 mol of Fe2O3 requires 3 × 22.4 litre or 67.2 dm3
Empirical formula = C2H2N
CO to get itself reduced.
Empirical formula weight = 2 × 12 + 2 + 14 = 40 146. (d) Writing the equation for the reaction, we get
Molecular mass = 80 PbO + 2HCl PbCl2 + H2O
Molecular formulae = n (C2H2N) 207 + 16 2 × 36.5 207 + 71
= 223 g = 73g = 278g
80 6.5
= 2 (C2H2N) n = C4H4N2 No. of moles of PbO 0.029
40 223
141. (b) Let 100 g of compound be there. 3.2
No. of moles of HCl 0.0877
77.43g 36.5
Number of moles of C = = 6.44 Thus PbO is the limiting reactant 1 mole of PbO produce
12.01g / mol
7.53g 1 mole PbCl2.
Number of moles of H = = 7.47 0.029 mole PbO produces 0.029 mole PbCl2.
1.008g / mol
EBD_7207
18 SOME BASIC CONCEPTS OF CHEMISTRY
149. (a) From molarity equation
163
147. (a) C57H110O6 + O2 57CO2 + 55H2O M1V1 + M2V2 = MV(total)
2
10 200 210
890 gram of fat produces 990 gram of H2O 2 0.5 M
1000 1000 1000
990 120 = M × 210
450 gram fat produces 450
890 120
M= 0.57 M
= 500.56 g of H2O 210
500.56g 150. (a) 1 ppm = 1 mg / 1 litre (for liquids)
Moles of H2O = = 27.80 4 ppm = 4 mg / 1 litre
18g mol
1 litre contains 4 mg of fluoride ions
26g 4
148. (d) nC = 2.16 10 ml contains 10 = 0.04 mg
12 g/mol 1000

20 g 0.04g
nO2 = 0.625 Number of moles of fluoride =
32 g/mol 19g / mol
O2 will be a limiting reagent in reaction (i) = 2.10 × 10–3
60 g No. of moles of solute
nN 2 = 2.14 151. (a) Molarity (M)
28g/mol Volume of solution in litres
Molarity nsolute
nH 2 = 40
25
According to balanced equation, nNaOH = = 0.625
40
1 mol of N2 requires 3 mole of N2
2.14 mol of N2 require 6.42 mol of N2 25
nLiOH = = 1.04
N2 will be a limiting reagent in reaction (ii) 24
100g 25
nP4 = = 0.86 nO2 = 6.25 nAl(OH)3 = = 0.32
4 31 (17 3 17)
According to balanced equation
25
1 mol of P4 require 3 mol of O2 nKOH = = 0.45
0.86 mol of P4 require 2.58 mol of O2 (39 17)
So P4 is a limiting reagent in reaction (iii) 25
nB(OH)3 = = 0.403
(11 17 3)
2
STRUCTURE OF ATOM

7. Which is not true with respect to cathode rays?


FACT / DEFINITION TYPE QUESTIONS
(a) A stream of electrons
1. Which of the scientist were able to prove that atom is no (b) Charged particles
longer non-divisible? (c) Move with speed same as that of light
(a) Dalton (b) Michael Faraday (d) Can be deflected by magnetic fields
(c) Thomson (d) Chadwick 8. What is the optimum conditions required to study the
2. Which of the following is never true for cathode rays ? conduction of electricity through gases.
(a) They possess kinetic energy. (a) High pressure and low voltage
(b) They are electromagnetic waves. (b) High pressure and high voltage
(c) They produce heat. (c) Low pressure and high voltage
(d) Low pressure and low voltage
(d) They produce mechanical pressure.
9. In discharge tube experiment stream of negatively charged
3. Cathode rays are deflected by
particles travel from
(a) an electric field only (b) magnetic field only (a) anode to cathode (b) cathode to anode
(c) by both (d) by none (c) Both (a) and (b) (d) Electrons does not travel
4. Which of the following statement is not correct about the 10. Millikan performed an experiment method to determine
characteristics of cathode rays? which of the following ?
(a) They start from the cathode and move towards the (a) Mass of the electron (b) Charge of the electron
anode. (c) e/m ratio of electron (d) Both (a) and (b)
(b) They travel in straight line in the absence of an external 11. The discovery of neutron became very late because :
electrical or magnetic field. (a) neutrons are present in nucleus
(c) Characteristics of cathode rays do not depend upon (b) neutrons are chargeless
the material of electrodes in cathode ray tube. (c) neutrons are fundamental particles
(d) Characteristics of cathode rays depend upon the (d) all of the above
nature of gas present in the cathode ray tube. 12. Which is correct statement about proton ?
5. Which of the following statements about the electron is (a) Proton is nucleus of deuterium
incorrect? (b) Proton is -particle
(a) It is negatively charged particle (c) Proton is ionized hydrogen molecule
(b) The mass of electron is equal to the mass of neutron. (d) Proton is ionized hydrogen atom
(c) It is a basic constituent of all atoms. 13. The lightest particle is :
(d) It is a constituent of cathode rays. (a) -particle (b) positron
6. While performing cathode ray experiments, it was observed (c) proton (d) neutron
that there was no passage of electric current under normal 14. When beryllium is bombarded with alpha particles
conditions. Which of the following can account for this (Chadwick’s experiment) extremely penetrating radiations,
observation ? which cannot be deflected by electrical or magnetic field are
(a) Dust particles are present in air given out. These are :
(b) Carbon dioxide is present in air (a) A beam of protons
(c) Air is a poor conductor of electricity under normal (b) Alpha rays
conditions (c) A beam of neutrons
(d) A beam of neutrons and protons
(d) None of the above
EBD_7207
20 STRUCTURE OF ATOM
15. Neutron is discovered by (b) -rays which impinged on a metal foil and ejected
(a) Chadwick (b) Rutherford electrons
(c) Yukawa (d) Dalton (c) helium atoms, which impinged on a metal foil and got
16. Suppose beam containing all three fundamental subatomic scattered
particles are allowed to pass through an electric field as (d) helium nuclei, which impinged on a metal foil and got
shown in figure. The subatomic particles detected at three scattered
points A, B and C on the screen respectively are ? 24. Which of the following scientists explained his model on
the basis of centrifugal force ?
+
A (a) Thomson (b) Dalton
Beam of (c) Millikan (d) Rutherford
B
particles 25. The number of neutrons in dipositive zinc ion with mass
C number 70 is

(a) 34 (b) 36
(a) Protons, neutrons, electrons (c) 38 (d) 40
(b) Electrons, neutrons, protons
40 1
(c) Electrons, protons, neutrons 26. The number of electrons in 19 K is
(d) Neutrons, protons, electrons
17. Which of the following properties of atom could be explained (a) 20 (b) 40
correctly by Thomson Model of atom? (c) 18 (d) 19
(a) Overall neutrality of atom. 27. Which of the following does not contain number of neutrons
(b) Spectra of hydrogen atom. equal to that of 40
18 Ar ?
(c) Position of electrons, protons and neutrons in atom.
(a) 41 (b) 43
(d) Stability of atom. 19 K 21 Sc
18. Arrange the following in terms of penetrating power. 40 42
(c) 21 Sc (d) 20 Ca
-rays, -rays, -rays
(a) > > (b) < < 28. Number of protons, neutrons and electrons in the element
(c) > < (b) > > 231
89 X is
19. Which of the rays are not deflected by the electric and
(a) 89, 89, 242 (b) 89, 142, 89
magnetic field ?
(c) 89, 71, 89 (d) 89, 231, 89
(a) -rays (b) X-rays
29. An element has atomic number 11 and mass number 24.
(c) -rays (d) Both (a) and (b)
What does the nucleus contain?
20. Rutherford’s experiment on the scattering of -particles
(a) 11 protons, 13 neutrons
showed for the first time that the atom has :
(b) 11 protons, 13 neutrons, 13 electrons
(a) electrons (b) protons
(c) 13 protons, 11 neutrons
(c) nucleus (d) neutrons'
(d) 13 protons, 11 electrons
21. When atoms are bombarded with alpha particles, only, a
30. The number of electrons and neutrons of an element is 18
few in million suffer deflection, others pass out undeflected. and 20 respectively. Its mass number is
This is because (a) 2 (b) 17
(a) the force of repulsion on the moving alpha particle is (c) 37 (d) 38
small 31. ‘A’ represents mass no. and Z represents atomic no. then
(b) the force of attraction between alpha particle and - decay is characterized by
oppositely charged electrons is very small (a) Z increases by 2, A decreases by 4
(c) there is only one nucleus and large number of electrons (b) Z decreases by 2, A increases by 4
(d) the nucleus occupies much smaller volume compared (c) Z decreases by 2, A decreases by 4
to the volume of the atom (d) Z increases by 2, A increases by 4.
22. Rutherford’s -particle dispersion experiment concludes 32. Nucleons are
(a) all positive ions are deposited at small part (a) only neutrons
(b) all negative ions are deposited at small part (b) neutrons + protons
(c) proton moves around the electron (c) neutrons + protons + electrons
(d) neutrons are charged particles. (d) neutrons + electrons
23. Rutherford’s experiment which established the nuclear 33. Atoms with same mass number but different atomic numbers
model of the atom used a beam of are called
(a) -particles which impinged on a metal foil and got (a) isotopes (b) isobars
absorbed (c) isochores (d) None of these
STRUCTURE OF ATOM 21
34. Which of the following pairs will have same chemical
properties ? T2 > T1
(a) 14 and 15 (b) O2– and F–
6 C 7 N T2
(d) Wavelength
(c) 40 and 40 (d) 35 and 37
18 Ar 19 K 17 Cl 17 Cl T1
35. What is the difference between two species if one has
atomic mass = 14 and atomic number = 7 whereas the
other has atomic mass = 14 and atomic number = 6 ? Intensity
(a) Neutrons (b) Protons
39. The ideal body, which emits and absorbs radiations of all
(c) Electrons (d) All of these
frequencies, is called a black body and the radiation emitted
36. From the data given below A, B, C and D respectively are,
by such a body is called
(A) 10 e–, atomic no. 11 (B) 10 e–, atomic no. 6 (a) white body radiation (b) black body radiation
(C) 10 e–, atomic no. 10 (D) 10 e–, atomic no. 9 (c) black body emission (d) None of these
(a) Na+, C4–, Ne, F– (b) C4–, Ne, Na–, F– 40. Which one of the following is not the characteristic of
(c) F–, Na+, Ne, C4– (d) F–, Na+, C4–, Ne Planck’s quantum theory of radiation ?
37. If the wavelength of the electromagnetic radiation is (a) The energy is not absorbed or emitted in whole number
increased to thrice the digital value, then what will be the or multiple of quantum
percent ch ange in the value of frequen cy of the (b) Radiation is associated with energy
electromagnetic radiation. (c) Radiation energy is not emitted or absorbed
(a) Increases by 33% (b) Decreases by 33% continuously but in the form of small packets called
(c) Increases by 66% (d) Decreases by 66% quanta
38. Which is the correct schematic representation of the graph (d) This magnitude of energy associated with a quantum
of black body radiation. is proportional to the frequency.
41. Which of the following is related with both wave nature
(a) and particle nature ?
T1 > T2
(a) Interference (b) E = mc2
(c) Diffraction (d) E = h
42. The value of Planck's constant is 6.63 × 10–34 Js. The
Intensity T1 T2
velocity of light is 3.0 × 108 m s–1. Which value is closest to
the wavelength in nanometers of a quantum of light with
frequency of 8 × 1015 s–1 ?
Wavelength
(a) 3 × 107 (b) 2 × 10–25
(c) 5 × 10 –18 (d) 4 × 101
43. In the photo-electron emission, the energy of the emitted
electron is
T1 > T2 (a) greater than the incident photon
(b) same as than of the incident photon
(c) smaller than the incident photon
(b) Intensity T2 (d) proportional to the intensity of incident photon
T1
44. When a metal surface is exposed to solar radiations
(a) The emitted electrons have energy less than a maximum
value of energy depending upon frequency of incident
Wavelength radiations
(b) The emitted electrons have energy less than maximum
value of energy depending upon intensity of incident
radiation
T1 > T2
(c) The emitted electrons have zero energy
T2 (d) The emitted electrons have energy equal to energy of
(c) photons of incident light
Wavelength T1 45. In photoelectric effect, at which frequency electron will be
ejected with certain kinetic energy ( 0 = threshold
frequency).
Intensity
(a) > 0 (b) 0 >
(c) 0 (d) 0
EBD_7207
22 STRUCTURE OF ATOM
46. In continous spectrum light of _(i)
___ wavelength is deviated 56. The radius of hydrogen atom in the ground state is 0.53 Å.
the ___
ii The radius of Li2+ ion (atomic number = 3) in a similar state is
(a) (i) = longest, least (b) (ii) = shortest, least (a) 0.17 Å (b) 0.265 Å
(c) (i) = shortest, most (d) (i) = longest, most (c) 0.53 Å (d) 1.06 Å
47. Which of the following statements do not form a part of 57. The energy of an electron in the nth Bohr orbit of hydrogen
Bohr’s model of hydrogen atom ? atom is
(a) Energy of the electrons in the orbits are quantized
(b) The electron(s) in the orbit nearest to the nucleus has 13.6 13.6
(a) – 4
eV (b) – eV
the lowest energy n n3
(c) Electrons revolve in different orbits around the nucleus
13.6 13.6
(d) The position and velocity of the electrons in the orbit (c) – eV (d) – eV
2 n
cannot be determined simultaneously n
48. An electron from one Bohr stationary orbit can go to next 58. The energy of second Bohr orbit of the hydrogen atom
higher orbit is 328 kJ mol 1 ; hence the energy of fourth Bohr orbit
(a) by emission of electromagnetic radiation would be:
(b) by absorption of any electromagnetic radiation (a) 41 kJ mol 1 (b) 82 kJ mol 1
(c) by absorption of electromagnetic radiation of particular (c) 164 kJ mol 1 (d) 1312 kJ mol 1
frequency 59. In a hydrogen atom, if energy of an electron in ground state
(d) without emission or absorption of electromagnetic is 13.6. eV, then that in the 2nd excited state is
radiation (a) 1.51 eV (b) 3.4 eV
49. For a Bohr atom angular momentum M of the electron is
(c) 6.04 eV (d) 13.6 eV.
(n = 0, 1, 2, .....) :
60. The energy of an electron in second Bohr orbit of hydrogen
(a) nh 2 (b) n2 h2 atom is :
4 4 (a) –5.44 × 10–19eV (b) –5.44 × 10–19cal
(c) –5.44 × 10 kJ–19 (d) –5.44 × 10–19J
nh2 nh
(c) (d) 61. The energy of electron in first energy level is 21 .79 10 12
4 2 erg per atom. The energy of electron in second energy level
50. According to Bohr's theory, the angular momentum of an is :
electron in 5th orbit is
(a) 54.47 10 12 erg atom 1
(a) 10 h / (b) 2.5 h /
12 1
(b) 5.447 10 erg atom
(c) 25 h / (d) 1.0 h /
51. In Bohr’s model, atomic radius of the first orbit is y, the (c) 0.5447 10 12 erg atom 1
radius of the 3rd orbit, is (d) 0.05447 10 12 erg atom 1
(a) y / 3 (b) y 62. The ionisation potential of a hydrogen atom is –13.6 eV.
(c) 3y (d) 9y What will be the energy of the atom corresponding to n = 2.
52. The radius of 1st Bohr’s orbit for hydrogen atom is ‘r’. The (a) – 3.4 eV (b) – 6.8 eV
radius of second Bohr’s orbit is (c) – 1.7 eV (d) –2.7 eV
(a) 4r (b) r3 +
2 63. The line spectrum of He ion will resemble that of
(c) 4r (d) r1/3
(a) hydrogen atom (b) Li+ ion
53. The third line of the Balmer series, in the emission spectrum
of the hydrogen atom, is due to the transition from the (c) helium atom (d) lithium atom
(a) fourth Bohr orbit to the first Bohr orbit 64. What does negative sign in the electronic energy for
(b) fifth Bohr orbit to the second Bohr orbit hydrogen atom convey.
(c) sixth Bohr orbit to the third Bohr orbit (a) Energy of electron when n =
(d) seventh Bohr orbit to the third Bohr orbit (b) The energy of electron in the atom is lower than the
54. Which one of the following pairs is not correctly matched ? energy of a free electron in motion
(a) Rutherford-Proton (c) The energy of electron in the atom is lower than the
(b) J.J. Thomson-Electron energy of a free electron of rest
(c) J.H. Chadwick-Neutron (d) The energy of electron decreases as it moves away
(d) Bohr-Isotopes from nucleus
55. If r is the radius of the first orbit, the radius of nth orbit of 65. In which of the following Bohr’s stationary state, the
H-atom is given by electron will be at maximum distance from the nucleus ?
(a) rn2 (b) rn (a) IInd (b) Ist
(c) r/n (d) r2 n2 (c) Vth (d) IIIrd
STRUCTURE OF ATOM 23
66. The wavelength of the radiation emitted, when in a hydrogen 74. Bohr model can explain :
atom electron falls from infinity to stationary state 1, would (a) the solar spectrum
be (Rydberg constant = 1.097×107 m–1) (b) the spectrum of hydrogen molecule
(a) 406 nm (b) 192 nm (c) spectrum of any atom or ion containing one electron
(c) 91 nm (d) 9.1×10–8 nm only
67. The frequency of radiation emitted when the electron falls (d) the spectrum of hydrogen atom only
from n = 4 to n = 1 in a hydrogen atom will be (Given :
75. Which of the following statements do not form a part of
ionization energy of H=2.18 ×10–18J atom–1and h = 6.625 ×
Bohr’s model of hydrogen atom ?
10–34 J s )
(a) Energy of the electrons in the orbits are quantized
(a) 1.54 1015 s 1 (b) 1.03 1015 s 1 (b) The electron in the orbit nearest the nucleus has the
(c) 3.08 1015 s 1 (d) 2.00 1015 s 1 lowest energy
68. Which of the following transitions of electrons in the (c) Electrons revolve in different orbits around the nucleus
hydrogen atom will emit maximum energy ? (d) The position and velocity of the electrons in the orbit
(a) n5 n4 (b) n4 n3 cannot be determined simultaneously.
(c) n3 n2 (d) all will emit same energy 76. Bohr’s theory can be applied to which of the following
69. The first emission line of hydrogen atomic spectrum in the ions.
Balmer series appears is (R = Rydberg constant) (a) Na+ (b) Be2+
5 3 (c) Li+ (d) Li2+
(a) R cm 1 (b) R cm 1 77. Bohr’s model is not able to account for which of the
36 4
following.
7 9
(c) R cm 1 (d) R cm 1 (a) Stability of atom.
144 400 (b) Spectrum of neutral helium atom.
70. According to the Bohr theory, which of the following
(c) Energy of free electron at rest.
transitions in the hydrogen atom will give rise to the least
energetic photon ? (d) Calculation of radii of the stationary states.
(a) n = 6 to n = 1 (b) n = 5 to n = 4 78. If electron, hydrogen, helium and neon nuclei are all moving
(c) n = 6 to n = 5 (d) n = 5 to n = 3 with the velocity of light, then the wavelength associated
71. The wavelength (in cm) of second line in the Lyman series with these particles are in the order
of hydrogen atomic spectrum is (Rydberg constant (a) Electron > hydrogen > helium > neon
= R cm–1) (b) Electron > helium > hydrogen > neon
(c) Electron < hydrogen < helium < neon
8R 9
(a) (b) (d) Neon < hydrogen < helium < electron
9 8R 79. The de Broglie wavelength of a tennis ball of mass 60 g
4 3R moving with a velocity of 10 metres per second is
(c) (d) approximately
3R 4
(a) 10–31 metres (b) 10–16 metres
72. The shortest wavelength in hydrogen spectrum of Lyman –25
(c) 10 metres (d) 10–33 metres
series when RH = 109678 cm–1 is
Planck’s constant, h = 6.63 × 10–34 Js
(a) 1002.7Å (b) 1215.67Å
80. If the energy difference between the ground state of an
(c) 1127.30Å (d) 911.7Å
atom and its excited state is 4.4 × 10–4 J, the wavelength of
73. What is the expression of frequency ( ) associated with
photon required to produce the transition :
absorption spectra of the photon.
(a) 2.26 × 10–12 m (b) 1.13 × 10–12 m
(c) 4.52 × 10 m –16 (d) 4.52 × 10–12 m
RH 1 1
(a) ni nf 81. The mass of a photon with a wavelength equal to
h n2 n 2
i f
1.54 × 10–8 cm is
(a) 0.8268 × 10–34 kg (b) 1.2876 × 10–33 kg
RH 1 1
(b) nf ni (c) 1.4285 × 10–32 kg (d) 1.8884 × 10–32 kg
h n2 n 2
i f 82. If the Planck’s constant h = 6.6×10–34 Js, the de Broglie
wavelength of a particle having momentum of 3.3 × 10–24
RH 1 1 kg ms –1 will be
(c) nf ni
h n2 n 2 (a) 0.002 Å (b) 0.5Å
i f
(c) 2Å (d) 500Å
(d) All the above are correct
EBD_7207
24 STRUCTURE OF ATOM
83. The values of Planck's constant is 6.63 × 10–34 Js. The 93. Which of the following sets of quantum numbers is correct
velocity of light is 3.0 × 108 m s–1. Which value is closest to for an electron in 4f orbital ?
the wavelength in nanometres of a quantum of light with (a) n = 4, = 3, m = + 1, s = + ½
frequency of 8 × 1015 s–1? (b) n = 4, = 4, m = – 4, s = – ½
(a) 5 × 10–18 (b) 4 × 101 (c) n = 4, = 3, m = + 4, s = + ½
(c) 3 × 107 (d) 2 × 10–25 (d) n = 3, = 2, m = – 2, s = + ½
84. The de Broglie wavelength of a car of mass 1000 kg and 94. What is the correct orbital designation of an electron with
velocity 36 km/hr is : the quantum number, n = 4, = 3, m = – 2, s = 1/2 ?
(a) 6.626 × l0–34 m (b) 6.626 × 10–38 m (a) 3s (b) 4 f
(c) 6.626 × 10 m –31 (d) 6.626 × 10– 30 m (c) 5p (d) 6s
85. Heisenberg uncertainty principle can be explained as 95. Which of the following represents correct set of the four
P h h quantum numbers for an electron in a 4d subshell ?
(a) x (b) x P (a) 4, 2, 1, 0 (b) 4, 2, 1, – 1/2
4 4
(c) 4, 3, 2, + 1/2 (d) 4, 3, – 2, – 1/2
h h 96. The total number of electrons that can be accommodated in
(c) x P (d) P
x all orbitals having principal quantum number 2 and azimuthal
86. Heisenberg's uncertainity principle is applicable to quantum number 1 is
(a) atoms only (b) electron only (a) 2 (b) 4
(c) nucleus only (d) any moving object (c) 6 (d) 8
87. The position of both, an electron and a helium atom is 97. For azimuthal quantum number = 3, the maximum number
known within 1.0 nm. Further the momentum of the of electrons will be
electron is known within 5.0 × 10–26 kg ms–1 . The (a) 2 (b) 6
minimum uncertainty in the measurement of the (c) 0 (d) 14
momentum of the helium atom is 98. Which of the following is not permissible arrangement of
(a) 50 kg ms–1 (b) 80 kg ms–1 electrons in an atom?
–26
(c) 8.0 × 10 kg ms –1 (d) 5.0 × 10–26 kg ms–1 (a) n = 5, l = 3, m = 0, s = + 1/2
88. Uncertainty in the position of an electron (b) n = 3, l = 2, m = – 3, s = – 1/2
(mass = 9.1 × 10–31 kg) moving with a velocity 300 ms–1, (c) n = 3, l = 2, m = – 2, s = – 1/2
accurate upto 0.001% will be (h = 6.63 × 10–34 Js) (d) n = 4, l = 0, m = 0, s = – 1/2
(a) 1.92 × 10–2 m (b) 3.84 × 10–2 m 99. Which of the following sets of quantum numbers represents
(c) 19.2 × 10 m –2 (d) 5.76 × 10–2 m the highest energy of an atom?
89. The uncertainty in the position of an electron (mass = (a) n = 3, l = 0, m = 0, s = +1/2
9.1 × 10–28 g) moving with a velocity of 3.0 × 104 cm s–1 (b) n = 3, l = 1, m = 1, s = +1/2
accurate upto 0.011% will be (c) n = 3, l = 2, m = 1, s = +1/2
(a) 1.92 cm (b) 7.68 cm (d) n = 4, l = 0, m = 0, s = +1/2
(c) 0.175 cm (d) 3.84 cm. 100. Which set of quantum numbers are not possible?
90. The Heisenberg uncertainity principle will be most n l m s
significant for which of the following object ? (a) 3 2 0 +1/2
(a) Object A of mass 9.11 × 10–30 kg (b) 2 2 1 +1/2
(b) Object B of mass 9.11 × 10–28 g (c) 1 0 0 –1/2
(c) Object C of mass 9.11 × 10–24 mg (d) 3 2 –2 +1/2
(d) Object D of mass 9.11 × 10–28 kg 101. What will be the sum of all possible values of l and m for
91. The orientation of an atomic orbital is governed by n=5?
(a) Spin quantum number (a) 12 (b) 13
(b) Magnetic quantum number (c) 4 (d) 9
(c) Principal quantum number 102. The following quantum numbers are possible for how many
orbital(s) n = 3, l = 2, m = +2 ?
(d) Azimuthal quantum number
(a) 1 (b) 3
92. For which one of the following sets of four quantum
(c) 2 (d) 4
numbers, an electron will have the heighest energy?
103. The orbitals are called degenerate when
n l m s
(a) they have the same wave functions
(a) 3 2 1 1/2
(b) they have the same wave functions but different
(b) 4 2 –1 1/2
energies
(c) 4 1 0 –1/2
(c) they have different wave functions but same energy
(d) 5 0 0 –1/2
(d) they have the same energy
STRUCTURE OF ATOM 25
104. The number of spherical nodes in 3p orbitals are 112. The order of filling of electrons in the orbitals of an atom will
(a) one (b) three be
(c) two (d) None of these (a) 3d, 4s, 4p, 4d, 5s (b) 4s, 3d, 4p, 5s, 4d
105. Which of the following graph correspond to one node (c) 5s, 4p, 3d, 4d, 5s (d) 3d, 4p, 4s, 4d, 5s
113. The number of d-electrons retained in Fe2+
(At. no. of Fe = 26) ion is
(a) 4 (b) 5
(c) 6 (d) 3
(a) (b) 114. The electronic configuration of an element is 1s2, 2s2 2p6,
3s2 3p6 3d 5, 4s1. This represents its
a0 a0 (a) excited state (b) ground state
(c) cationic form (d) anionic form
115. Number of unpaired electrons in N2+ is
(a) 2 (b) 0
(c) 1 (d) 3
116. An ion has 18 electrons in the outermost shell, it is
(c) (d) (a) Cu+ (b) Th4+
a0 (c) Cs+ (d) K+
a0
117. In a given atom no two electrons can have the same values
for all the four quantum numbers. This is called
106. If there are five radial nodes, then what can be the correct
(a) Hund’s Rule (b) Aufbau principle
representation of the orbital for n = 8.
(c) Uncertainty principle (d) Pauli’s exclusion principle
(a) 8d (b) 8s
118. The electronic configuration of Cu (atomic number 29) is
(c) 8p (d) 8f
107. What can be the representation of the orbital having 3 (a) 1s 2 , 2 s 2 2 p 6 ,3s 2 3 p 6 , 4 s 2 3d 9
angular nodes and n = 5. (b) 1s 2 ,2 s 2 2 p 6 ,3s 2 3 p 6 3d 10 , 4 s1
(a) 5d (b) 5f (c) 1s2 ,2s2 2 p6 ,3 p 2 3 p 6 ,4s2 4 p6 ,5s2 5 p1
(c) 5p (d) 5s
(d) 1s 2 , 2 s 2 2 p 6 ,3 p 2 3 p 6 , 4 s 2 4 p 6 3d 3
108. The number of orbitals present in the fifth shell will be
(a) 25 (b) 10 119. The orbital diagram in which the Aufbau principle is violated
is :
(c) 50 (d) 20
2s 2p
109. Arrange the orbital of same shell in the increasing order of
shielding strength of the outer shell of electrons. (a)
s, f, d, p
(a) s < p < d < f (b) s > p < d < f (b)
(c) s > p > d < f (d) s > p > d > f
110. Which of the following is not correct for electronic (c)
distribution in the ground state ?
(d)
(a) Co [Ar] 120. If n = 6, the correct sequence for filling of electrons will be :
(a) ns (n – 2) f (n – 1) d np
(b) Ni [Ar] (b) ns (n – 1) d (n – 2) f np
(c) ns (n – 2) f np (n – 1) d
(d) ns np (n – 1) d (n – 2) f
(c) Cu [Ar] 121. Maximum number of electrons in a subshell of an atom is
determined by the following:
(d) All of the above
(a) 2 l + 1 (b) 4 l – 2
111. The electronic configuration of gadolinium (Atomic number 2
(c) 2 n (d) 4 l + 2
64) is
122. The correct order of increasing energy of atomic orbitals is
(a) [Xe] 4f 8 5d 0 6s2 (b) [Xe] 4f 3 5d 5 6s2
6 2 2 (a) 5 p < 4 f < 6 s < 5 d (b) 5 p < 6 s < 4 f < 5 d
(c) [Xe] 4f 5d 6s (d) [Xe] 4f 7 5d 1 6s2
(c) 5 p < 5 d < 4 f < 6 s (d) None of these
EBD_7207
26 STRUCTURE OF ATOM
123. For which element, the valence electron will be present in 127. Which of the following statement(s) is/are incorrect
the highest energy orbital. regarding photoelectric effect?
(a) 3Li (b) 16 S (i) The number of electrons ejected is proportional to the
(c) 20Ca (d) 21 Sc intensity of light.
124. Which of the following electronic configuration of d-orbital (ii) There is some time lag between the striking of light
will have highest affinity for gaining an electron? beam on the metal surface and the ejection of electrons.
(iii) The kinetic energy of ejected electrons depends upon
(a)
the brightness of light.
(iv) The kinetic energy of the ejected electron is
(b) proportional to the frequency of the incident radiation.
(a) (i) and (ii) (b) (ii) and (iii)
(c) (c) (ii) only (d) (ii) and (iv)
128. For Balmer series in the spectrum of atomic hydrogen, the
(d)
1 1
wave number of each line is given by v RH –
STATEMENT TYPE QUESTIONS n12 n 22
125. On the basis of figure given below which of the following where RH is a constant and n 1 and n 2 are integers. Which
statement(s) is/are correct ? of the following statement(s) is (are) correct?
(i) As wavelength decreases, the lines in the series
+ converge.
Cathode Anode
(ii) The integer n 1 is equal to 2.
(iii) The ionization energy of hydrogen can be calculated
from the wave number of these lines.
(iv) The line of longest wavelength corresponds to n 2 = 3.
(a) (i), (ii) and (iii) (b) (ii), (iii) and (iv)
– Fluorescent screen (c) (i), (i) and (iv) (d) (ii) and (iv)
129. Which of the following statements of quantum mechanics
Magnet
was in agreement with Bohr’s model?
(i) At point B, when only electric field is applied. (i) The path of an electron in an atom can never be
determined accurately.
(ii) At point C, when both electric and magnetic field is
applied. (ii) The energy of electrons in atom is quantized i.e., can
(iii) At point B, when both electric and magnetic fields are only have specific values.
balanced. (iii) An orbital cannot contain more than two electrons.
(iv) At point C, when only magnetic field is applied. (a) Only (i) (b) (i) and (ii)
Which of the following is/are correct? (c) Only (ii) (d) (ii) and (iii)
(a) (i) and (ii) (b) only (iii) 130. Which of the following statements concerning the quantum
(c) (iii) and (iv) (d) (i) and (iii) numbers are correct ?
126. Which of the following statements are not correct about (i) Angular quantum number determines the three-
electromagnetic radiation ? dimensional shape of the orbital.
(i) Electromagnetic waves require medium to travel. (ii) The principal quantum number determines the
(ii) Different electromagnetic radiations travel at same orientation and energy of the orbital.
speed in vaccum. (iii) Magnetic quantum number determines the size of the
(iii) The oscillating electric and magnetic fields produced orbital.
by oscillating charged particles are perpendicular to
(iv) Spin quantum number of an electron determines the
each other, but not to the direction of propagation.
orientation of the spin of electron relative to the chosen
(iv) The oscillating electric field and magnetic field are
axis.
perpendicular to each other, and also to the direction
of propagation. The correct set of option is
(a) (i), (ii) and (iii) (b) (ii) and (iii) (a) (i) and (ii) (b) (i) and (iv)
(c) (i) and (iii) (d) (i) and (iv) (c) (iii) and (iv) (d) (ii), (iii) and (iv)
STRUCTURE OF ATOM 27
135. Match the columns.
MATCHING TYPE QUESTIONS
Column-I Column-II Column-III
131. Match the columns. (Sub shell) (Number of (Angular/Azimuthal
Column-I Column-II orbitals) Quantum Number)
(A) d (p) 1 (i) 1
(A) 1 H, 2 H and 3 H (p) Isobars (B) f (q) 3 (ii) 2
1 1 1
(C) s (r) 5 (iii) 0
(B) 14 and 14 (q) Isotopes
6C 7N (D) p (s) 7 (iv) 3
(C) Na+and Mg2+ (r) Isoelectronic species (a) A – (r) – (ii), B – (s) – (iv), C – (p) – (iii), D – (q) – (i)
(a) A – (p), B – (q), C – (r) (b) A – (q) – (i), B – (s) – (iv), C – (p) – (iii), D – (r) –(ii)
(b) A – (q), B – (p), C – (r) (c) A – (p) – (iii), B – (s) – (iv), C – (r) – (ii), D – (q) – (i)
(c) A – (r), B – (q), C – (p) (d) A – (r) – (ii), B – (p) – (iii), C – (s) – (iv), D – (q) – (i)
(d) A – (p), B – (r), C – (q) 136. Match the columns.
132. Match the columns. Column-I Column-II
Column-I Column-II
z
(A) X-rays (p) v = 100 – 104 Hz
(B) UV (q) v = 1010 Hz
(C) Long radio waves (r) v = 1016 Hz
(D) Microwave (s) v = 1018 Hz
(A) d x 2 y2 (p)
(a) A – (s), B – (r), C – (p), D – (q)
(b) A – (r), B – (s), C – (p), D – (q) y x
(c) A – (s), B – (p), C – (r), D – (q)
(d) A – (s), B – (r), C – (q), D – (p)
133. Match the columns.
Column-I Column-II z

(A) 2 (p) Energy can be emitted


or absorbed in packets
(B) d xy (q)
(B) de Brolie (q) Significant only for
motion of microscopic
objects. y x
(C) Heisenberg (r) The probability of
finding an electron at a
point within an atom z
(D) Planck’s (s) Every object in motion
has a wave character.
(a) A – (q), B – (s), C – (r), D – (p)
(C) d yz (r)
(b) A – (r), B – (p), C – (q), D – (s)
(c) A – (r), B – (s), C – (q), D – (p)
(d) A – (s), B – (p), C – (r), D – (q) x
134. Match the columns. y
Column-I Column-II
(Quantum number) (Information provided) z
(A) Principal (p) orientation of the orbital
quantum number
(B) Azimuthal (q) energy and size of orbital
(D) d (s)
quantum number z2
(C) Magnetic (r) spin of electron
quantum number x
(D) Spin quantum (s) shape of the orbital y
number
(a) A – (q), B – (s), C – (p), D – (r) (a) A – (p), B – (s), C – (r), D – (q)
(b) A – (s), B – (q), C – (p), D – (r) (b) A – (s), B – (p), C – (r), D – (q)
(c) A – (q), B – (p), C – (s), D – (r) (c) A – (s), B – (p), C – (q), D – (r)
(d) A – (q), B – (s), C – (r), D – (p) (d) A – (s), B – (r), C – (p), D – (q)
EBD_7207
28 STRUCTURE OF ATOM
137. Match the columns 140. Assertion : The radius of the first orbit of hydrogen atom is
Column-I Column-II 0.529Å.
(Rules) (Statements) Reason : Radius of each circular orbit (rn) - 0.529Å (n2/Z),
(A) Hund’s Rule (p) No two electrons in an atom where n = 1, 2, 3 and Z = atomic number.
can have the same set of four 141. Assertion : All isotopes of a given element show the same
quantum numbers. type of chemical behaviour.
(B) Aufbau Principle (q) Half-filled and completely Reason : The chemical properties of an atom are controlled
filled orbitals have extra by the number of electrons in the atom.
stablity. 142. Assertion : Black body is an ideal body that emits and
(C) Pault Exclusion (r) Pairing of electrons in the absorbs radiations of all frequencies.
Principle orbitals belonging to the same Reason : The frequency of radiation emitted by a body
subshell does not take place goes from a lower frequency to higher frequency with an
until each orbital is singly increase in temperature.
occupied. 143. Assertion : It is impossible to determine the exact position
(D) Heisenberg’s (s) It is impossible to determine and exact momentum of an electron simultaneously.
Uncertainty the exact position and exact Reason : The path of an electron in an atom is clearly defined.
Principle momentum of a subatomic
particle simultaneously. CRITICAL THINKING TYPE QUESTIONS
(t) In the ground state of atoms,
144. What is the ratio of mass of an electron to the mass of a
orbitals are filled in the order
proton?
of their increasing energies.
(a) 1 : 2 (b) 1 : 1
(a) A – (r), B – (p), C – (t), D – (s)
(c) 1 : 1837 (d) 1 : 3
(b) A – (r), B – (t), C – (s), D – (p)
145. The increasing order for the values of e/m (charge/mass) is
(c) A – (r), B – (t), C – (p), D – (s)
(a) e, p, n, (b) n, p, e,
(d) A – (t), B – (r), C – (p), D – (s)
(c) n, p, , e (d) n, , p, e
138. Match the columns.
146. In which of the following the amount of deviation from
Column-I Column-II their path in the presence of electric and magnetic field will
(Atom / Ion) (Electronic configuration) be maximum?
(A) Cu (p) 1s2 2s2 2p6 3s2 3p6 3d10 (a) N2– (b) N3–
(B) Cu 2+ (q) 1s2 2s2 2p6 3s2 3p6 3d10 4s2 (c) N – (d) N
(C) Zn2+ (r) 1s2 2s2 2p6 3s2 3p6 3d10 4s1 147. The deflection of the particles from their path in presence
(D) Cr 3+ (s) 1s2 2s2 2p6 3s2 3p6 3d9 of electric and magnetic field will be maximum in which of
(t) 1s2 2s2 2p6 3s2 3p6 3d3 the following.
(a) A – (s), B – (r), C – (p), D – (t) (a) O (b) N
(b) A – (r), B – (s), C – (p), D – (t) (c) U (d) He
(c) A – (r), B – (s), C – (t), D – (p) 148. Which of the following pairs have identical values of e/m?
(d) A – (r), B – (s), C – (p), D – (s) (a) A proton and a neutron
(b) A proton and deuterium
ASSERTION-REASON TYPE QUESTIONS (c) Deuterium and an -particle
Directions : Each of these questions contain two statements, (d) An electron and -rays
Assertion and Reason. Each of these questions also has four 149. If the alpha-particles are projected against the following
alternative choices, only one of which is the correct answer. You atoms Fe, Be, Mg, Al then increasing order in which the
have to select one of the codes (a), (b), (c) and (d) given below. alpha-particle feel repulsion will be
(a) Assertion is correct, reason is correct; reason is a correct (a) Be, Mg, Al, Fe (b) Be, Al, Mg, Fe
explanation for assertion. (c) Mg, Al, Mg, Fe (d) Al, Mg, Fe, Be
(b) Assertion is correct, reason is correct; reason is not a 150. Chlorine exists in two isotopic forms, C1-37 and C1-35 but
correct explanation for assertion its atomic mass is 35.5. This indicates the ratio of C1-37 and
(c) Assertion is correct, reason is incorrect C1-35 is approximately
(d) Assertion is incorrect, reason is correct. (a) 1 : 2 (b) 1 : 1
139. Assertion : The position of an electron can be determined (c) 1 : 3 (d) 3 : 1
exactly with the help of an electron microscope. 151. The number of electrons, neutrons and protons in a species
are equal to 10, 8 and 8 respectively. The proper symbol of
Reason : The product of uncertainty in the measurement of
the species is
its momentum and the uncertainty in the measurement of
(a) 16 O 8 (b) 18 O 8
the position cannot be less than a finite limit.
(c) 18 Ne10 (d) 16 O82–
STRUCTURE OF ATOM 29
152. What will be the difference between electromagnetic 159. Which of the following levels of H and He+
have same
radiation shown in A and B respectively ? energy respectively ?
(A) 1, 2 (B) 3, 4
(C) 2, 4 (D) 3, 6
(a) A and D (b) A and B
(A)
(c) C and D (d) A, C and D
160. Bohr radius of nth orbit of an atom is given by the expression:
n2 h2 nh
(a) r (b) r 2
4 2
me 2
4 mZe2
(B) n2 h2 n 2 h2
(c) r (d) r
4 2 mZ 4 2 me 2 Z
(i) Velocity (ii) Wavelength 161. The ratio between kinetic energy and the total energy of the
(iii) Frequency (iv) Energy electrons of hydrogen atom according to Bohr’s model is
(a) (ii) only (b) (ii) and (iv) (a) 2 : 1 (b) 1 : 1
(c) 1 : – 1 (d) 1 : 2
(c) (ii), (iii) and (iv) (d) (iv) only
162. The potential energy of electron present in ground state of
153. Arrange the electromagnetic radiations a, b, c, d and e in
Li2+ ion is represented by :
increasing order of energy. Frequencies of a, b and c are
1015, 1014 and 1017 respectively whereas wavelength of 3e 2 3e
(d) and (e) are 350 nm and 100 nm respectively ? (a) (b)
4 0r 4 0r
(a) a, b, c, d, e (b) a, b, d, e, c
(c) a, d, b, e, c (d) b, d, a, e, c 3e 2 3e 2
154. An electron, e1 is moving in the fifth stationary state, and (c) 2 (d)
4 0r 4 0r
another electron e2 is moving in the fourth stationary state.
The radius of orbit of electron, e1 is five times the radius of 163. In hydrogen atomic spectrum, a series limit is found at
orbit of electron, e2 calculate the ratio of velocity of electron 12186.3 cm–1. Then it belong to
e1 (v1) to the velocity of electron e2 (v2). (a) Lyman series (b) Balmer series
(a) 5 : 1 (b) 4 : 1 (c) Paschen series (d) Brackett series
(c) 1 : 5 (d) 1 : 4 164. Which transition in the hydrogen atomic spectrum will have
2+
155. The Li ion is moving in the third stationary state, and its the same wavelength as the transition, n = 4 to n = 2 of He+
linear momentum is 7.3 × 10–34 kg ms–1. Calculate its angular spectrum?
momentum. (a) n = 4 to n = 3 (b) n = 3 to n = 2
(a) 1.158 × 10–45 kg m2s–1 (c) n = 4 to n = 2 (d) n = 2 to n = 1
(b) 11.58 × 10–48 kg m2s–1 165. Arrange the following elements in the order of ease of
(c) 11.58 × 10–47 kg m2s–1 detection of wave properties, in the de Broglie experiment.
(d) 12 × 10–45 kg m2s–1 H, Li, Be, B, K
156. The Bohr orbit radius for the hydrogen atom (n = 1) is (a) H < Be, B < Li < K. (b) H > Li > K > Be > B
approximately 0.530 Å. The radius for the first excited state (c) H > Li > Be > B > K (d) H < Li < Be < B < K
(n = 2) orbit is (in Å) 166. A 600 W mercury lamp emits monochromatic rediation of
(a) 0.13 (b) 1.06 wavelength 331.3 nm. How many photons are emitted from
the lamp per second ? (h = 6.626 × 10–34 Js; velocity of light
(c) 4.77 (d) 2.12
= 3 × 108 ms–1)
157. According to Bohr’s theory the energy required for an electron
(a) 1 × 1019 (b) 1 × 1020
in the Li2+ ion to be emitted from n = 2 state is (given that 21
(c) 1 × 10 (d) 1 × 1023
the ground state ionization energy of hydrogen atom is
167. Calculate the velocity of ejected electron from the metal
13.6 eV)
surface when light of frequency 2 × 1015 Hz fall on the
(a) 61.2 eV (b) 13.6 eV
metal surface and the threshold frequency is 7 × 1014 Hz
(c) 30.6 eV (d) 10.2 eV for metal ?
158. Among species H, Li2+, He+, Be3+ and Al3+ Bohr’s model (a) 1.37 × 106 (b) 1.26 × 106
was able to explain the spectra of (c) 1.45 × 10 7 (d) 1.37 × 107
(a) All of these 168. What is the wavelength (in m) of the electron emitted in
(b) None of these the above question (Q. no. 167) ?
(c) all other species except Be3+ (a) 5.308 × 10–10 (b) 5.89 × 10–11
(d) all other species except Al3+ (c) 4.37 × 10 –13 (d) 3.98 × 10–10
EBD_7207
30 STRUCTURE OF ATOM
169. The velocity of particle A is 0.1 ms–1 and that of particle B is (b) n = 3, l = 3, m = +3, s = + ½
0.05 ms–1. If the mass of particle B is five times that of (c) n = 6, l = 0, m = +1, s = – ½
particle A, then the ratio of de-Broglie wavelengths (d) n = 4, l = 2, m = +2, s = 0
associated with the particles A and B is 176. Which combinations of quantum numbers, n, , m and s for
(a) 2 : 5 (b) 3 : 4 the electron in an atom does not provide a permissible
(c) 6 : 4 (d) 5 : 2 solution of the wave equation ?
170. Two fast moving particles X and Y are associated with
de Broglie wavelengths 1 nm and 4 nm respectively. If mass 1 1
(a) 3, 2,1, (b) 3,1,1,
of X in nine times the mass of Y, the ratio of kinetic energies 2 2
of X and Y would be 1 1
(a) 3 : 1 (b) 9 : 1 (c) 3, 3,1, (d) 3, 2, 2,
2 2
(c) 5 : 12 (d) 16 : 9
177. An electron has principal quantum number 3. The number of
171. Uncertainty in position of a n electron (mass = 9.1 × 10–28 g)
its (i) subshells and (ii) orbitals would be respectively
moving with a velocity of 3 × 104 cm/s accurate upto 0.001%
(a) 3 and 5 (b) 3 and 7
will be (use h/4 ) in uncertainty expression where
h = 6.626 ×10–27 erg-second). (c) 3 and 9 (d) 2 and 5
(a) 1.93 cm (b) 3.84 cm 178. The electrons identified by quantum numbers n and :
(c) 5.76 cm (d) 7.68 cm (A) n = 4, = 1 (B) n = 4, = 0
172. The measurement of the electron position is associated with (C) n = 3, = 2 (D) n = 3, = 1
an uncertainty in momentum, which is equal to can be placed in order of increasing energy as :
1×10–18 g cm s–1. The uncertainty in electron velocity is, (a) (C) < (D) < (B) < (A) (b) (D) < (B) < (C) < (A)
(mass of an electron is 9 × 10– 28 g) (c) (B) < (D) < (A) < (C) (d) (A) < (C) < (B) < (D)
(a) 1 × 109 cm s–1 (b) 1 × 106 cm s–1 179. The five d-orbitals are designated as d xy , d yz , d xz , d
5 –1 x2 y2
(c) 1 × 10 cm s (d) 1 × 1011 cm s–1
and d 2. Choose the correct statement.
173. In an atom, an electron is moving with a speed of 600 m/s z
with an accuracy of 0.005%. Certainity with which the (a) The shapes of the first three orbitals are similar but
position of the electron can be located is ( h = 6.6 × 10 –34 kg that of the fourth and fifth orbitals are different
m2s–1, mass of electron, em = 9.1 × 10–31 kg) (b) The shapes of all five d-orbitals are similar
(a) 5.10 × 10 –3 m (b) 1.92 × 10 –3 m (c) The shapes of the first four orbitals are similar but that
(c) 3.84 × 10 m –3 (d) 1.52 × 10 –4 m of the fifth orbital is different
174. If uncertainty in position and momentum are equal, then (d) Ths shapes of all five d-orbitals are different
uncertainty in velocity is : 180. If the nitrogen atom has electronic configuration 1s7, it would
have energy lower than that of the normal ground state
1 h h configuration 1s22s22p3, because the electrons would be
(a) (b)
2m 2 closer to the nucleus. Yet 1s7 is not observed because it
violates.
1 h h (a) Heisenberg uncertainty principle
(c) (d)
m (b) Hund's rule
175. Which of the following sets of quantum numbers is correct? (c) Pauli exclusion principle
(a) n = 5, l = 4, m = 0, s = + ½ (d) Bohr postulate of stationary orbits
STRUCTURE OF ATOM 31

26. (a) 19 + 1e– = 20 electrons.


FACT / DEFINITION TYPE QUESTIONS
27. (c) 18Ar40 contains 22 neutrons and 21Sc40 contains 19
1. (b) neutrons. The number of neutrons = (A – Z)
2. (b) Cathode rays are never electromagnetic waves. 28. (b) Number of p = number of e– = 89 and neutrons
3. (c) Cathode rays are made up of negatively charged 231 – 89 = 142.
particles (electrons) which are deflected by both the 29. (a) Z = 11, A = 24. Hence protons = 11 the neutrons
electric and magnetic fields. (24 – 11) = 13.
4. (d) 5. (b) 6. (c) 7. (c) 30. (d) For neutral atom . No. of p = No. of e– = 18 and
8. (c) The electrical discharge through the gases could be A = Z + No. of neutrons = 18 + 20 = 38.
observed only at low pressure and high voltage. 31. (c) When an alpha particle is emitted from a nucleus its
9. (b) The cathode rays (negatively charged particles atomic number decreases by two and its atomic mass
stream) originates from cathode and move towards decreases by four e.g.
anode. –
Z XA Z 2X
A 4
10. (b) Millikan determined the value of charge on the
electron by using oil drop experiment. 32. (b) Nucleons are total number of protons and neutrons.
11. (b) Both of these are collectively known as nucleons.
12. (d) Proton is the nucleus of H-atom (H-atom devoid of its 33. (b) Atoms with mass number but different atomic numbers
electron) are called isobars. Examples; 146C, 147N are isobars.
13. (b) Positron (positive electron, +1e0) is positively charged 34. (d) 35 and 37 are isotopes, so they will have
17 Cl 17 Cl
electron without any mass, so it is the lightest particle
same chemical properties.
among given choices.
35. (d) Atomic number is equal to number of protons or
14. (c) As the neutron is a chargeless particle, hence, the beam
number of electrons. Thus if two species have
of neutrons is not deflected by electrical or magnetic
different atomic number they must contain different
field.
number of protons and electrons. Number of neutrons
15. (a) James Chadwick in 1932 discovered the neutrons. = Atomic mass – Atomic number. Therefore due to
16. (b) Since electrons are negatively charged particles they difference of atomic numbers two species also have
got deflected toward positively charged electrode different number of neutrons.
whereas proton being positively charged will get 36. (a)
deflected toward negative electrode. Since neutrons
c
are neutral, so they went straight. 37. (d) 1
17. (a) l
18. (b) -rays have the least penetrating power, followed by c c
-rays (100 times that of -rays) and -rays (1000 times 2
2 3 1
that of -rays).
2 1 100
19. (d) X- and -rays are not deflected by the electric and % change in frequency =
magnetic field. 1
20. (c) Rutherford’s -ray scattering experiment first showed c c 2c
the existence of a small positivily charged entity in the 100
3 1 1 3 1
centre of atom, called nucleus. = 100
c c
21. (d) The nucleus occupies much smaller volume compared
to the volume of the atom. 1 1
22. (a) All positive ions are deposited at small part. (nucleus = – 66%
of atom). 38. (b)
23. (d) Rutherford used doubly charged helium particle. 39. (b) The ideal body, which emits and absorbs radiations of
( - particle) all frequencies, is called a black-body and the radiation
24. (d) Electrons are revolving around the nucleus, and emitted by such a body is called black-body radiation.
centrifugal force is balancing the force of attraction. 40. (a) Energy is always absorbed or emitted in whole number
25. (d) Number of neutrons = Mass number – Atomic number or multiples of quantum.
= 70 – 30 = 40.
EBD_7207
32 STRUCTURE OF ATOM
41. (d) (i) Interference and diffraction support the wave Atomic number of Li (Z) = 3.
nature of electron.
n2 (1) 2
(ii) E = mc2 supports the particle nature of electron. Radius of Li2+ ion 0.53 r1 0.17
Z 3
hc 57. (c) Energy of an electron in Bohr's orbit is given by the
(iii) E h is de-Broglie equation and it
13.6
supports both wave nature and particle nature of relationship. En eV.
electron. n2
hc c 1312
42. (d) E h or 58. (b) We know that En = kJ mol 1
n2
3 108 n = 4 (Fourth Bohr orbit)
3.75 10 8 m 1312
8 1015 Given E4 = –82 kJ mol–1
In nanometer = 3.75× 10 42
which is closest to 4 × 101 59. (a) 2nd excited state will be the 3rd energy level.
43. (c) K.E. of emitted electron 13.6 13.6
En eV or E eV 1.51 eV.
= hv hv0 (i.e. smaller than hv ). n2 9
44. (a)
45. (a) At a frequency > 0, the ejected electrons come out 13.6 Z 2
60. (d) For H atom, En eV
with certain kinetic energy. n2
46. (a) For second orbit, n = 2
47. (d) This statement is known as uncertainty principle which Z = At. no. = 1 (for hydrogen)
was given by Heisenberg it is not a Bohr’s postulate.
13.6 (1) 2 13.6
48. (c) Since the energy difference between two consecutive E2 eV
2 4
Bohr orbits is quantized and the energy of higher orbit (2)
is more than that of lower orbit, so an electron from 13.6 1.6 10 19 19
one Bohr stationary orbit can go to next higher orbit = J = 5.44 10 J
by absorption of electromagnetic radiation of particular 4
wavelength or frequency. 61. (b) If we assume the atom to be hydrogen like, energy of
nth energy level
49. (d) For a Bohr atom, angular momentum M of the electron
E
nh
= . En = 21 where E1 is energy of first energy level
2 n
50. (b) Angular momentum of an electron in nth orbit is given E1 21.79 10 12
E1
E2 =
by 2 2 44
nh –12
= – 5.447 × 10 erg per atom.
mvr
2 E0
For n = 5, we have 62. (a) Energy of an electron E
n2
5h 2.5h For energy level (n = 2)
Angular momentum of electron
2 13.6 13.6
E 3.4 eV.
51. (d) y n2 2
2 4
For 1st orbit y = 1 63. (a) H,He+ and Li2+ are single electron species thus show
similar line spectra.
For IIIst orbit y 32 9
64. (c)
So it will 9y. 65. (c) Vth stationary state, as radii of stationary state is given
52. (a) rn = r1 × n2 (for hydrogen atom) as rn = n2 × a0 n=5
rn = r × n2
as r1 = r (given) 1 1 1
66. (c) R
r2 = r × 22 (n = 2, for second Bohr’s orbit) n12 n22
= 4r 1 1 1
53. (b) 54. (d) 1.097 107 1.097 107 m 1
1
55. (a) Radius of nth orbit = r1 n2 . (for H-atom)
56. (a) For hydrogen atom (n) = 1 (due to ground state) 9
91.15 10 m 91nm
Radius of hydrogen atom (r) = 0.53 Å.
STRUCTURE OF ATOM 33

6.626 10 34 Js
1 1 1 m
67. (c) IE 1.54 10 10 m 3 108 m sec 1
h n12 n22
1.4285 10 32 kg
18
2.18 10 1 1
6.625 10 34 1 16 3.08 1015 s 1
34
h 6.6 10 10
82. (c) 24
2 10 m 2Å
p 3.3 10
1 1
68. (c) E for two energy levels 21.79 J/atom
n12 n22 ch c
83. (b) E h ;and
1 1
69. (a) E R ;
n12 n22 3.0 108
8 1015
First line in Balmer series results in the transition :
n2 = 3 to n1 = 2.
70. (c) Energy of photon obtained from the transition n = 6 to 3.0 108
0.37 10 7 37.5 10 9 m 4 101 nm
n = 5 will have least energy. 8 1015
1 1
E 13.6Z 2 h
n12 n22 84. (b)
mv
1 1 1 h = 6.6 × 10–34 J/s
71. (a) R –
n12 n 22 m = 1000 kg
For second line in lyman series
36 103
n2 = 3 v = 36 km/hr = m/sec = 10 m/sec
60 60
1 1 1 1 1 8R
R R – =
1 2
3 2 1 9 9 6.6 10 34
= 6.6 10 38 m
3
72. (d) The shortest wavelength in hydrogen spectrum of 10 10
Lyman series is given by formula : 85. (b) Heisenberg uncertainity principle can be explained by
1 RH RH 109678 the relation
2 2 1
n 1 h
x. P
= 9.117 × 10–6 cm 4
= 911.7 × 10–10 m = 911.7 Å. where x = uncertainity in position
73. (b) P = uncertainity in momentum
74. (c) Bohr model can explain spectrum of any atom or ion
86. (d) Heisenberg's uncertainty Principle is applicable to any
containing one electron only (that is H-like species)
moving object.
75. (d) Uncertainty principle which was given by Hiesenberg
and not Bohr’s postulate. h
87. (d) By Heisenberg uncertainty Principle x p
76. (d) Bohr’s model can be applied to one electron system 4
only. (which is constant)
77. (b) Bohr model can only explain one electron system As x for electron and helium atom is same thus
78. (a) = h/ mv ; for the same velocity, varies inversely momentum of electron and helium will also be same
with the mass of the particle. therefore the momentum of helium atom is equal to
5 × 10–26 kg. m.s–1.
h 6.6 10 34
79. (d) 10 33 m 88. (a) Given m = 9.1 × 10–31kg
mv 60 10 3 10
h = 6.6 × 10–34Js
80. (d) Given E = 4.4 × 10–4 j , =?
300 .001
hc 6.6 10 34 3 108 v= = 0.003ms–1
= 4.5 10 22 m 100
E 4
4.4 10
h h From Heisenberg's uncertainity principle
81. (c) We know that ; m
mv v 6.62 10 34
The velocity of photon (v) = 3 × 108 m sec–1 x 31
1.92 10 2 m
4 3.14 0.003 9.1 10
1.54 10 8 cm 1.54 10 10 meter
EBD_7207
34 STRUCTURE OF ATOM
100. (b) Value of l = 0 ..........(n – 1)
h h
89. (c) x. p or x.m v ; l cannot be equal to n.
4 4 101. (b) For n = 5, l = n – 1 = 5 – 1 = 4
0.011 m = 2l + 1 = 2(4) + 1 = 9
v 3 10 4 3.3 cms 1 Sum of values of l and m = 9 + 4 = 13
100
102. (a) Quantum number n = 3, l = 2, m = +2 represent an
27 orbital with
6.6 10
x 0.175 cm
4 3.14 9.1 10 28 3.3 s
1
3d xy or 3d 2 2
2 x y
90. (b) x. v value will be large for object of smallest mass
and is therefore the most significant for calculating which is possible only for one electron.
uncertainity. 103. (d) The orbitals which have same energy are called
91. (b) Magnetic quantum no. represents the orientation of degenerate orbitals eg. px , p y and pz .
atomic orbitals in an atom. For example px, py & pz 104. (a) No. of radial nodes in 3p-orbital = (n – – 1)
have orientation along X-axis, Y-axis & Z-axis [for p ortbital = 1]
92. (b) The sub-shell are 3d, 4d, 4p and 4s, 4d has highest =3–1–1=1
energy as n + value is maximum for this. 105. (b)
93. (a) The possible quantum numbers for 4f electron are 106. (a) As n – l – 1 = 5 or 8 – l – 1 = 5 l = 2.
1 107. (b) According to given information n = 5 and l = 3.
n = 4, = 3, m = – 3, –2 –1, 0, 1 , 2 , 3 and s 108. (a) The number of allowed orbitals are given by n 2.
2
Thus when n = 5
Of various possiblities only option (a) is possible.
(5)2 = 25
94. (b) n = 4 represents 4th orbit
109. (d) Spherical shaped s-orbital shields the outer shell
= 3 represents f subshell electrons move effectively than p-orbital, which in turn
m = – 2 represents orientation of f-orbital shields more effectively than d-orbital and so on.
s = 1/2 represents direction of spin of electron. 110. (d) According to Hund’s rule electron pairing in p, d and f
The orbital is 4f. orbitals cannot occur until each orbital of a given
95. (b) For 4d orbitals, n = 4, l = 2 subshell contains one electron each or is singly
occupied.
For s orbital l 0
111. (d) We know that atomic number of gadolinium is 64.
For p orbital l 1 Therefore the electronic configuration of gadolinium
For d orbital l = 2 is [Xe] 4f 7 5d1 6s2. Because the half filled and fully
m = –2, –1, 0, +1 or + 2 filled orbitals are more stable.
112. (b) The sub-shell with lowest value of (n + ) is filled up
1 1
s = + and first. When two or more sub-shells have same (n + )
2 2 value the subshell with lowest value of 'n' is filled up
1 first therefore the correct order is
Thus choice b having n = 4, l = 2, d = 1 and s = is
2 orbital 4s 3d 4p 5s 4d
correct. n+ 4+ 0 3+ 2 4 +1 5 + 0 4 + 2
96. (c) n = 2, l = 1 means 2p–orbital. Electrons that can be value =4 =5 =5 =5 =6
accommodated = 6 as p sub-shell has 3 orbital and 113. (c) Fe++ (26 – 2 = 24) = 1s2 2s2 2p6 3s2 3p6 4s0 3d 6 hence
each orbital contains 2 electrons. no. of d electrons retained is 6. [Two 4s electron are
97. (d) = 3 means f -subshell. Maximum no. of removed]
electrons = 4 + 2 = 4 × 3 + 2 = 14 114. (b) This configuration represents ground state electronic
98. (b) m = – l to +l, through zero thus for l = 2, values of m configuration of Cr.
will be – 2, –1, 0, + 1, + 2. 1s2 2s2 2p6 3s23p63d 5 4s1
Therefore for l = 2, m cannot have the value –3. 115. (c) N(7) = 1s2 2s2 2p3
99. (c) (a) n = 3, 0 means 3s-orbital and n + = 3 N2 1s 2 , 2 s 2 2 p1x
(b) n = 3, 1 means 3p-orbital n + = 4 Unpaired electrons = 1.
116. (a) Cu+ = 29 – 1 = 28 e–
(c) n = 3, 2 means 3d-orbital n + = 5
thus the electronic confingration of Cu+ is
(d) n = 4, 0 means 4s-orbital n + = 4
2 2 6 2 6 10
Increasing order of energy among these orbitals is Cu+ (28) = 1s 2s 2 p 3s 3 p 3d
3s < 3p < 4s < 3d 18e
3d has highest energy.. 117. (d) This is as per the definition of Pauli’s exclusion
principle.
STRUCTURE OF ATOM 35
118. (b) Electronic configuration of Cu (29) is 1s2 2s 2 2p6 3s2 135. (a) For d-subshell Number of orbitals = 5, l = 2
3p6 3d 10 4s1 and not 1s2, 2s2 2p6 3s2 3p6 3d 94s2 due f-subshell Number of orbitals = 7, l = 3
to extra stability of fully filled orbitals. s-subshell Number of orbitals = 1, l = 0
119. (b) According to Aufbau principle, the orbital of lower p-subshell Number of orbitals = 3, l = 1
energy (2s) should be fully filled before the filling of 136. (b) 137. (c) 138. (b)
orbital of higher energy starts.
120. (a) ns (n 2)f (n 1)d np [n = 6] ASSERTION-REASON TYPE QUESTIONS
121. (d) The number of sub shell is (2 l + 1). The maximum 139. (d) The statement-1 is false but the statement-2 is true
number of electrons in the sub shell is 2 (2 l + 1) exact position and exact momentum of an electron can
= (4 l + 2). never be determined according to Heisenberg’s
122. (b) uncertainty principle. Even not with the help of electron
5p 4f 6s 5d microscope because when electron beam of electron
(n + l) 5+ 1 4+ 3 6+ 0 5+ 2 microscope strikes the target electron of atom, the
impact causes the change in velocity and position of
6 7 6 7 electron .
Hence the order is 5p < 6s < 4f < 5d 140. (a) Both assertion and reason are true and reason is the
123. (d) correct explanation of assertion.
124. (d) The d-orbital represented by option (d) will become
completely filled after gaining an electron. Therefore n2 h2 n2
Radius, rn 2
0.529Å.rn
option (d) is correct. 4 e mZ Z
For first orbit of H-atom
STATEMENT TYPE QUESTIONS n=1
125. (c) When both electric and magnetic field is applied, (1)2
electron strikes at point B, and at point C when only r1 0.529Å = 0.529Å
1
magnetic field is applied.
126. (c) 141. (a) 142. (b) 143. (c)
127. (b) For statement (ii) there is no time lag between striking
of light beam and the ejection of electrons. For
CRITICAL THINKING TYPE QUESTIONS
statement (iii) refer statement (iv). 144. (c)
128. (c) (i) Beyond a certain wavelength the line spectrum
e 0
becomes band spectrum. 145. (d) for (i) neutron 0
(ii) For Balmer series n1 = 2 m 1
(iii) For calculation of longest wavelength use nearest 2
(ii) -particle 0.5
value of n 2. Hence for longest wavelength in Balmer 4
series of hydrogen spectrum,
1
n1 = 2 & n2 = 3. (iii) proton 1
129. (c) Statement (i) is related to Heisenberg’s uncertainity 1
principle. Statement (iii) belongs to Pauli’s exclusion 1
(iv) electron 1837
principle. 1 1837
130. (b) Angular quantum number determines the 3d shape of 146. (b) N3– The amount of deviation depends upon the
the orbital. magnitude of negative charge on the particle.
Spin quantum number of an electron determines the 147. (d) The lesser is the mass of particle, greater is the
orientation of the spin of electron relative to the chosen deflection.
axis. 148. (c) Deuterium and an -particle have identical values of
e/m.
MATCHING TYPE QUESTIONS 149. (a) Considering the core of an atom, higher the positive
charge concentrated in the nucleus, greater the
131. (b) Isotopes have same atomic number. Isobars have repulsion for an alpha-particle.
same mass number, whereas isoelectronic species
have same number of electrons although the (A) has kq1 ( ze )
Coulombic force of repulsion =
same number of electrons but the protons they carry r2
are same while in case of isolelectronic species q1 = charge on -particle
number of protons they carry are different. (ze) = charge on nucleus of atom
132. (a) 133. (c) 134. (a) 150. (c)
EBD_7207
36 STRUCTURE OF ATOM
151. (d) Atomic number = No. of protons = 8 Also, linear momentum (mv) = 7.3 × 10 –34 kg ms–1
Mass number = No. of protons + No. of neutrons Then angular momentum will be
= 8 + 8 = 16 = (mv) × r
Since the no. of electrons are two more than the no. of = (7.3 × 10–34 kg ms–1) (158.7 pm)
protons, hence, it is a binegative species. Thus, the = 7.3 × 10–34 kg ms–1 × (158.7 × 10–12 m)
species is 16 O82 . = 11.58 × 10–48 kg m2 s–1
152. (c) e/m waves shown in figure A has higher wavelength = 11.58 × 10–45 g m2s–1
in comparison to e/m waves shown in figure B. 156. (d) Given : Radius of hydrogen atom = 0.530 Å, Number of
Thus these waves also differ in frequency and excited state (n) = 2 and atomic number of hydrogen
c atom (Z) = 1. We know that the Bohr radius.
energy.
n2 (2)2
(r ) Radius of atom 0.530
Z 1
1
4 0.530 2.12 Å
hc
(A) E1
z2
1 157. (c) Energy of electron in 2nd orbit of Li+2 13.6
n2

13.6 (3) 2
2 = = –30.6 eV
(B) E2
hc (2) 2
2 Energy required = 0 – (–30.6) = 30.6 eV
158. (d) Except Al3+ all contain one electron and Bohr’s
model could explain the spectra for one electron
153. (d) E = h system, Bohr’s model was not able to explain the
spectra of multielectron system.
c
and
Z2 2.18 10 18
159. (d) EnH 2.18 10 18 2 J J
= 1015, b = 1014,
a nH 2
nH
17 15
c = 10 , d = 0.85 × 10
and 15
e = 10 × 10 , + Z2 2.18 10 18 4
154. (d) From the expression of Bohr’s theory, we know that EnHe 2.18 10 18 J J
n2 + n2 +
h He He
me v1r1 n1
2
+ 1 4
EnH EnHe nHe+ = 2 × nH
h 2
nH n2 +
& me v 2 r2 n2 He
2
If nH = 1 Then nHe+ = 2
m e v1r1 n1 h 2 If nH = 2 Then nHe+ = 4
m e v2 r2 n2 2 h If nH = 3 Then nHe+ = 6
Given, r1 = 5 r2, n1 = 5, n2 = 4 n2 h 2
160. (d) Radius of nth orbit
me v1 5r2 5 4 2 me2 Z
me v2 r2 4 where n = no. of orbit
h = Plank’s constant
v1 5 1 e = charge on one electron
1: 4
v2 4 5 4 m = mass of one electron
155. (b) Z = 3 for Li2+ ions Z = atomic number
161. (c)
52.9 n 2
So rn Ze 2
Z 162. (d) In S.I. units the P.E. .
n = 3, Z = 3 4 0r
For Li2+, Z = 3.
2
52.9 3
rn pm 3e 2
3 P.E. .
= 158.7 pm 4 0r
STRUCTURE OF ATOM 37
163. (c) Series limit is the last line of the series, i.e. n2 = . h / mA v A m Bv B
A
1 1 1 1 1 R B h / mB v B m A vA
R R
n12 n 22 n12 2
n12 5mA 0.05
5 0.5 2.5 5 / 2
mA 0.1
09677.76
12186.3
n12 A : B 5:2

109677.76 h
n12 9 n1 3 170. (d) de Broglie wavelength
12186.3 mv
The line belongs to Paschen series. 1 m 2 v2 1 1 v2
;
2 m1v1 4 9 v1
1 1 1
164. (d) For He+ ion, Z2 R
n12 n 22 v2 9
v1 4
2 1 1 3R
2 R
22 42 4 v1 4
v2 9
1 1 1
For hydrogen atom, R
n12 n 22 1
KE mv 2
2
3R 1 1 1 1 3 2
R or KE1 m1 v12 9 4 16
4 n12 n 22 n12 n 22 4
KE2 m2 v 22 1 9 9
n1 = 1 and n2 = 2.
171. (a) Given mass of an electron(v) 28
165. (c) The wavelengths of elements decreases with increase 9.1 10 g;
h Velocity of electron (v) 3 10 4 cm/s;
in their mass.
mv
0.001
hc Accuracy in velocity 0.001% ;
166. (c) Energy of a photon, E 100
Actual velocity of the electron
34
6.626 10 (Js) 3 108 (ms 1 )
= 6× 10–19 J 0.001
331.3 10 9 (m) ( v) 3 104 0.3 cm/s .
100
No. of photons emitted per second
Planck’s constant (h) = 6.626×10–27erg-sec.
600 (J)
=1021
6 10 9 (J) Uncertainty in the position of the electron
1 2 h 6.626´10-27 ´ 7
167. (a) m h h 0 ( x) =
2 4 m v 4 ´ 22 ´ (9.1´10-28 )´ 0.3
1 2 =1.93 cm
mv h( 0)
2 172. (a) p=m v
Substituting the given values of x and m, we get
2h 1×10–18 g cm s–1 = 9×10–28 g × v
v ( 0)
m 18
1 10
168. (a) According to de-Broglie, or v
9 10 28
h = 1.1 × 109 cm s–1 1×109 cm s–1
mv i.e. option (a) is correct.
where m = mass of electron, v = velocity 173. (b) According to Heisenberg uncertainty principle.
169. (d) Given, vA = 0.1 ms–1 and vB = 0.05 ms–1 also, h h
mB = 5mA x.m v x
4 4 m v
h 600 0.005
de-Broglie wavelength, Here v 0.03
mv
100
EBD_7207
38 STRUCTURE OF ATOM

34 176. (c) Possible values of and m depend upon the value of n


6.6 10
So, x = 0 to (n – 1)
4 3.14 9.1 10 31 0.03 m = – to + through zero
= 1.92 × 10–3 meter
1 1
h s= and
174. (a) We know p. x 2 2
4 Thus for n = 3,
since p = x (given) may be 0, 1 or 2; but not 3
h m may be –2, –1, 0, +1 or +2
p. p
4 1 1
s may be or
h 2 2
or m v m v. [ p= m v]
4 177. (c)
2 h 178. (b) (A)4 p (B) 4 s
or v (C) 3 d (D) 3 p
4 m2
According to Bohr Bury's (n + ) rule, increasing order
h 1 h of energy will be (D) < (B) < (C) < (A).
or v
4 m 2 2m Note : If the two orbitals have same value of (n + )
Thus option (a) is the correct option. then the orbital with lower value of n will be filled first.
175. (a) For n = 5, l may be 0, 1, 2, 3 or 4 179. (c) First four orbitals contain four lobes, while fifth orbital
For l = 4, m = 2l + 1 = 2 × 4 + 1 = 9 consists of only two lobes. The lobes of dxy orbital lie
between x and y axis. Similarly in the case of dyz and
= –4, – 3, – 2, –1, 0, +1, +2, +3, +4
dzx. their lobes lie between yz and zx axis respectively.
1
For m = 0, s Four lobes of d 2 2 orbital are lying along x and y
2 x y
Hence, (a) is correct option. axis while two lobes of d orbital are lying along z-
z2
(b) For any value of n, the value of l cannot be equal or axis.
greater than value of n, hence it is incorrect. 180. (c) As per Pauli exclusion principle "no two electrons in
(c) For l = 0, m = 0 hence it is incorrect. the same atom can have all the four quantum numbers
(d) The value of s can never be zero. Thus this option is equal or an orbital cannot contain more than two
also incorrect. electrons and it can accommodate two electrons only
when their directions of spin are opposite".
3
CLASSIFICATION OF ELEMENTS AND
PERIODICITY IN PROPERTIES
10. According to Newlands theory, when elements are
FACT / DEFINITION TYPE QUESTIONS
arranged in the order of increasing atomic weight which
1. Periodic classification of elements is used to examine the number element had similar properties to the first element.
(a) periodic trends in physical properties of elements (a) third (b) seventh
(b) periodic trends in chemical properties of elements (c) eighth (d) sixth
(c) Both (a) and (b) 11. Lothar Meyer plotted the physical properties such as atomic
(d) None of the above volume, melting point and ...X... against atomic weight. Here,
X refers to
2. Cl, Br, I, if this is Dobereiner’s triad and the atomic masses
(a) mass (b) boiling point
of Cl and I are 35.5 and 127 respectively the atomic mass of
Br is (c) surface tension (d) None of these
12. The most significant contribution towards the development
(a) 162.5 (b) 91.5
of periodic table was made by
(c) 81.25 (d) 45.625
(a) Mendeleev (b) Avogadro
3. If the two members of a Dobereiner triad are phosphorus (c) Dalton (d) Cavendish
and antimony, the third member of this triad is
13. Noble gases were included in Mendeleev’s periodic table in
(a) arsenic (b) sulphur the
(c) iodine (d) calcium (a) 1st group (b) 7th group
4. The law of triads is applicable to a group of (c) 8th group (d) None of these
(a) Cl, Br, I (b) C, N, O 14. Mendeleev classified elements in
(c) Na, K, Rb (d) H, O, N (a) increasing order of atomic groups
5. In 1800, only ....X.... elements were known. Here, X refers to (b) eight periods and eight groups
(a) 63 (b) 31 (c) seven periods and nine groups
(c) 114 (d) 92 (d) eight periods and seven groups
6. Johann Doberiner gave the idea of trends among physical 15. Select the correct chronological order for the discoveries
and ...X... of several groups of three elements. Here, X refers to of the following scientists.
(a) atomic number (b) atomic mass Dobereiner, Newlands, Chancourtois, Mendeleev
(c) chemical properties (d) None of these (a) Chancourtois, Dobereiner, Newlands, Mendeleev
(b) Dobereiner, Chancourtois, Newlands, Mendeleev
7. Which of the following is the correct set of elements to
Dobereiner’s triads ? (c) Dobereiner, Newlands, Chancourtois, Mendeleev
(d) Chancourtois, Newlands, Dobereiner, Mendeleev
(a) Li Na K (b) Br Cl I 16. The molecular formula of chloride of Eka-Aluminium and
7 23 39 80 35.5 127
(c) (d) Data insufficient Eka-Silicon respectively are
Fe Ni Co
55.85 58.71 58.93 (a) GaCl3 and SiO4 (b) GaCl3 and AlCl3
8. On which of the following Dobereiner’s Triad law is not (c) AlCl3 and SiCl4 (d) GaCl3 and GeCl4
applicable? 17. Who developed long form of the periodic table?
(a) Cl, Br, I (b) Ca, Sr, B (a) Lothar Meyer (b) Neils Bohr
(c) F, Cl, Br (d) Li, Na, K (c) Mendeleev (d) Moseley
9. Newlands could classify elements only upto – 18. At present, how many elements are known
(a) copper (b) chlorine (a) 110 (b) 112
(c) calcium (d) chromium (c) 113 (d) 118
EBD_7207
40 CLASSIFICATION OF ELEMENTS AND PERIODICITY IN PROPERTIES
19. Which of the scientists given below discovered that 31. In the modern periodic table one of the following does not
periodic table should be based on the atomic number ? have appropriate position –
(a) Mendeleev (b) Newlands (a) transition elements
(c) Moseley (d) Lothar Meyer (b) inert gases
20. How many elements are there in 6th period of periodic table? (c) inner transition elements
(a) 18 (b) 8 (d) halogens
(c) 30 (d) 32 32. If the atomic number of an element is 33, it will be placed in
21. Modern periodic table is based on the atomic number of the the periodic table in the
elements. The experiment which proved the significance of
(a) First group (b) Third group
the atomic number was
(c) Fifth group (d) Seventh group.
(a) Mulliken’s oil drop experiment
33. An atom has electronic configuration 1s2 2s2 2p6 3s2 3p6
(b) Mosley’s work on X-ray spectra
3d3 4s2, you will place it in which group?
(c) Bragg’s work on X-ray diffraction
(a) Fifth (b) Fifteenth
(d) Discovery of X-rays by Rontgen
22. The period number in the periodic table corresponds to the (c) Second (d) Third
...A... principal quantum number (n) of the elements. Here, A 34. Which of the following is not an actinoid ?
refers to (a) Curium (Z = 96) (b) Californium (Z = 98)
(a) lowest (b) highest (c) Uranium (Z = 92) (d) Terbium (Z = 65)
(c) middle (d) None of these 35. The period number in the long form of the periodic table is
23. The symbol and IUPAC name for the element with atomic equal to
number 120, respectively are (a) magnetic quantum number of any element of the period.
(a) Ubn and unbinilium (b) Ubn and unbiunium (b) atomic number of any element of the period.
(c) Ubn and unnibium (d) Ubn and unnilium (c) maximum Principal quantum number of any element of
24. Element with which of the following atomic number was the period.
named by American Society as Rutherfordium, while by (d) maximum Azimuthal quantum number of any element
Soviet Society it was named as Kurchatovium? of the period.
(a) 108 (b) 104 36. The electronic configuration of four elements are given
(c) 114 (d) 110 below. Which elements does not belong to the same family
25. What is the IUPAC name of the element with atomic number as others?
114 ?
(a) Unununnium (b) Unnilquadium (a) [Xe]4f 14 5d10 ls 2 (b) [Kr]4d10 5s 2
(c) Ununquadium (d) Unnilennium. (c) [Ne]3s23p5 (d) [Ar] 3d10 4s2
26. Element with electronic configuration 1s2 2s2 2p6 3s2 3p6 37. The elements with atomic numbers 9, 17, 35, 53 and 85 belong to
3d10 4s2 4p6 4d10 5s2 5p3 belongs to the following group of (a) alkali metals (b) alkaline earth metals
the periodic table (c) halogens (d) noble gases
(a) 2nd (b) 5th 38. Which of the following pairs has both members from the
(c) 3rd (d) 7th same period of the periodic table.
27. The long form of periodic table consists of
(a) Na – Ca (b) Na – Cl
(a) seven periods and eight groups
(c) Ca – Cl (d) Cl – Br
(b) seven periods and eighteen groups
(c) eight periods and eighteen groups 39. The elements which are characterized by the outer electronic
(d) eighteen periods and eight groups configuration ns1 to ns2 np6 are collectively called
28. All the members in a group in long form of periodic table (a) Transition elements
have the same (b) Representative elements
(a) valence (c) Lanthanides
(b) number of valence electrons (d) Inner transition elements
(c) chemical properties 40. f-block elements are called inner transition elements because
(d) All of the above
(a) they have properties similar to those of transition
29. Elements of which group form anions most readily?
elements
(a) Oxygen family (b) Nitrogen family
(c) Halogens (d) Alkali metals (b) they exist in between transition elements
30. Element having atomic no. of 56 belongs to which of the (c) the last electron enters into the f-orbital of the
following block of periodic table? penultimate shell
(a) p- block (b) d-block (d) the last electron enters into any orbital of penultimate
(c) f- block (d) s-block shell
CLASSIFICATION OF ELEMENTS AND PERIODICITY IN PROPERTIES 41
41. An element, which belongs to third period and group 16 in 54. Which ionisation potential (IP) in the following equations
the periodic table has electronic configuration. involves the greatest amount of energy ?
(a) 1s2, 2s2 2p6, 3s2 3p3 (b) 1s2, 2s2 2p6, 3s2 3p4 (a) Na Na e (b) K K2 e
(c) 1s2, 2s2 2p6, 3s2 3p5 (d) 1s2, 2s2 2p4
(c) C 2 C3 e (d) Ca Ca 2 e
42. Which of the following is non-metallic ?
55. Arrange S, P, As in order of increasing ionisation energy
(a) B (b) Be (a) S < P < As (b) P < S < As
(c) Mg (d) Al (c) As < S < P (d) As < P < S
43. Which group of the periodic table contains coinage metal ? 56. The statement that is not correct for periodic classification
(a) IIA (b) IB of elements is :
(c) IA (d) None of these (a) The properties of elements are periodic function of
44. The only non-metal which is liquid at ordinary temperature is their atomic numbers.
(a) Hg (b) Br2 (b) Non-metallic elements are less in number than metallic
(c) NH3 (d) None of these elements.
45. Which is a metalloid? (c) For transition elements, the 3d-orbitals are filled with
(a) Pb (b) Sb electrons after 3p-orbitals and before 4s-orbitals.
(c) Bi (d) Zn (d) The first ionisation enthalpies of elements generally
46. In the long form of the periodic table all the non-metals are increase with increase in atomic number as we go along
placed in a period.
(a) s-block (b) p-block 57. Consider the following changes
(c) f-block (d) d-block A A e : E1 and A A 2 e : E2
47. Arrange the following elements in the order of their The energy required to pull out the two electrons are E1 and
increasing non-metallic character E2 respectively. The correct relationship between two
Li, O, C, Be, F energies would be
(a) F < O < C < Be < Li (b) Li < Be < C < O< F (a) E1 < E2 (b) E1 = E2
(c) F < O < C < Be < Li (d) F < O < Be < C < Li (c) E1 > E2 (d) E1 E2
48. Which is the correct order of ionic sizes (At. No. : Ce = 58, 58. Of the given electronic configurations for the elements,
Sn = 50, Yb = 70 and Lu = 71) ? which electronic configuration indicates that there will be
(a) Ce > Sn > Yb > Lu (b) Sn > Ce > Yb > Lu abnormally high difference in the second and third ionization
(c) Lu > Yb > Sn > Ce (d) Sn > Yb > Ce > Lu energy for the element?
49. The order of increasing sizes of atomic radii among the (a) 1s2 2s2 2p6 3s2 (b) 1s2 2s2 2p6 3s1
elements O, S, Se and As is : (c) 1s2 2s2 2p6 3s2 3p1 (d) 1s2 2s2 2p6 3s2 3p2
(a) As < S < O < Se (b) Se < S < As < O 59. Alkali metals are powerful reducing agents because
(c) O < S < As < Se (d) O < S < Se < As (a) these are metals
(b) their ionic radii are large
50. In the ions P3–, S2– and Cl–, the increasing order of size is
(c) these are monovalent
(a) Cl–, S2–, P3– (b) P3–, S2–, Cl–
2– – 3– (d) their ionisation potential is low
(c) S , Cl , P (d) S2–, P3–, Cl– 60. Which of the following metals requires the radiation of
51. Which of the following is correct ? highest frequency to cause the emission of electrons ?
(a) Isoelectronic ions have same nuclear charge (a) Na (b) Mg
(b) Isoelectronic ions have same neutrons (c) K (d) Ca
(c) Isoelectronic ions have same number of electrons 61. Halogens and chalcogens family have highly ...P.. electron
(d) All are correct gain enthalpy. Here, P refers to
52. On going down a main sub-group in the periodic table (a) negative (b) positive
(example Li to Cs in IA or Be to Ra in IIA), the expected (c) zero (d) infinity
trend of changes in atomic radius is a 62. Which of the following represents the correct order of
(a) continuous increase increasing electron gain enthalpy with negative sign for the
(b) continuous decrease elements O, S, F and Cl ?
(c) periodic one, an increase followed by a decrease (a) Cl < F < O < S (b) O < S < F < Cl
(d) decrease followed by increase (c) F < S < O < Cl (d) S < O < Cl < F
53. Why the size of an anion is larger than the parent atom? 63. The electron affinity for the inert gases is –
(a) Due to increased repulsion among the electrons. (a) zero (b) high
(c) negative (d) positive
(b) Due to decrease in effective nuclear charge.
64. The element with positive electron gain enthalpy is
(c) Due to increased in effective nuclear charge.
(a) hydrogen (b) sodium
(d) Both (a) and (b) (c) oxygen (d) neon
EBD_7207
42 CLASSIFICATION OF ELEMENTS AND PERIODICITY IN PROPERTIES
65. Which of the following will have the least negative electron 74. Which of the following statement(s) about the modern
gain enthalpy? periodic table is/are incorrect ?
(a) P (b) S (i) The elements in the modern periodic table are arranged
(c) Cl (d) F on the basis of their decreasing atomic number
(ii) The elements in the modern periodic table are arranged
66. Which is the correct order of electronegativity ?
on the basis of their increasing atomic masses
(a) F > N < O > C (b) F > N > O > C (iii) Isotopes are placed in adjoining group(s) in the periodic
(c) F > N > O < C (d) F < N < O =C table
67. The correct order of decreasing electronegativity values (iv) The elements in the modern periodic table are arranged
among the elements I-beryllium, II-oxygen, III-nitrogen and on the basis of their increasing atomic number
IV-magnesium is (a) (i) only (b) (i), (ii) and (iii)
(a) II > III > I > IV (b) III > IV > II > I (c) (i), (ii) and (iv) (d) (iv) only
(c) I > II > III > IV (d) I > II > IV > III 75. Consider the following statements:
68. An element having electronic configuration (i) The discovery of inert gases later on did not disturb
Mendeleev's arrangement.
1s 2 2s 2 2p 6 3s 2 3p 6 4s1 forms (ii) In the present periodic table, periodicity in the
(a) Acidic oxide (b) Basic oxide properties of elements is related to the periodicity in
their electronic configurations.
(c) Amphoteric oxide (d) Neutral oxide
Which of these statement(s) is/are correct ?
69. Diagonal relationship is shown by (a) (i) only (b) (ii) only
(a) All elements with their diagonally downward elements (c) Both (i) and (ii) (d) Neither (i) nor (ii)
towards right 76. Which of the following statements are correct?
(b) Most of the elements of second period (i) The second period (n = 2) starts with lithium and third
(c) All 3d series elements electron enters the 2s orbital. The next element,
(d) None of the above beryllium has four electrons and has the electronic
70. In any period the valency of an element with respect to configuration 1s22s2. From the next element boron,
oxygen the 2p orbitals are filled with electrons when the L
(a) Increases one by one from IA to VIIA shell is completed at neon (2s22p6). Thus there are 8
(b) Decreases one by one form IA to VIIA elements in the second period.
(c) Increases one by one from IA to IVA and then (ii) Successive filling of 3s and 3p orbitals gives rise to
decreases from VA to VIIA one by one the third period of 8 elements from sodium to argon.
(d) Decreases one by one from IA to IVA and then (iii) The fourth period (n = 4) starts at potassium and the
increases from VA to VIIA one by one added electron fill up the first 4s and 4p orbitals than
71. What will be the formula of the compound formed by the 3d orbital is filled.
(iv) Fifth period begins with rubidium with the filling of 5s
silicon and bromine ?
orbital and ends at xenon with the filling up of the 5p
(a) SiBr2 (b) SiBr4
orbital.
(c) SiBr3 (d) SiBr (a) (i) and (ii) (b) (i), (ii) and (iii)
72. Which of the following sequence correctly represents the (c) (iii) and (iv) (d) (i), (ii) and (iv)
decreasing acidic nature of oxides ? 77. With reference to the chemical element with atomic number
(a) Li2O > BeO > B2O3 > CO2 > N2O3 17, consider the following statements:
(b) N2O3 > CO2 > B2O3 > BeO > Li2O (i) It belongs to second period in the periodic table of
(c) CO2 > N2O3 > B2O3 > BeO > Li2O chemical elements.
(d) B2O3 > CO2 > N2O3 > Li2O > BeO (ii) It forms anion with unit negative charge.
Which of the statement(s) given above is/are correct ?
STATEMENT TYPE QUESTIONS (a) (i) only (b) (ii) only
(c) Both (i) and (ii) (d) Neither (i) nor (ii)
73. Choose the correct coding for following statements. Here 78. Choose the correct codes for the following statements
T stands for True and F stands for False statement. related to s-block elements. Here ‘T’ stands for true and F
(i) Mendeleev left several gaps in his periodic table for stands for false statement.
the undiscovered elements. (i) They are all reactive metals with low ionization
(ii) The gap under aluminium and a gap under silicon was enthalpies.
left and these elements were called Eka aluminium and (ii) Their metallic character and reactivity increase as we
Eka silicon. go down the group.
(iii) Germanium was placed in place of Eka-aluminium and (iii) They are found in pure form in nature.
gallium was placed in place of Eka silicon. (iv) All the compounds of s-block elements are ionic in
(a) TTT (b) TFF nature.
(c) TTF (d) TFT (a) TTFF (b) TTFT
(c) TTTF (d) TFFF
CLASSIFICATION OF ELEMENTS AND PERIODICITY IN PROPERTIES 43
79. Consider the following statements: 85. Read the following three statements and choose the correct
(i) The elements silicon, germanium and arsenic are called option. Here T stands for true and F stands for false
metalloids. statement.
(ii) Metalloids have properties quite different from those (i) Boron has a smaller first ionization enthalpy than
of metals and non-metals. beryllium.
Which of these statement(s) is/are correct ? (ii) Nitrogen has smaller first ionization enthalpy than
(a) (i) only (b) (ii) only oxygen.
(c) Both (i) and (ii) (d) Neither (i) nor (ii) (iii) The first ionization enthalpy increases across a
80. Consider the following statements: period.
(i) Metals will be found on the right side of the periodic (a) FTT (b) TFT
table. (c) TFF (d) FFT
(ii) The element P, S and O belong to the same period.
86. Consider the following statements
Which of these statement(s) is/are correct ?
(i) The radius of an anion is larger than that of the parent
(a) (i) only (b) (ii) only
atom.
(c) Both (i) and (ii) (d) Neither (i) nor (ii)
(ii) The ionization energy generally increases with
81. Consider the following statements:
increasing atomic number in a period.
(i) Atomic radii decreases across a row of the periodic
(iii) The electronegativity of elements increases on moving
table when we move from left to right.
down across a group.
(ii) Atomic radii increases down the column as we move
from top to bottom.
Which of the above statements is/are correct?
(iii) Although the order of elements is based on atomic (a) (i) alone (b) (ii) alone
numbers, vertical families share similar chemical (c) (i) and (ii) (d) (ii) and (iii)
properties.
Which of the statement(s) given above is/are correct? MATCHING TYPE QUESTIONS
(a) (i) and (ii) (b) (i) and (iii) 87. Match the Column-I and Column-II and select correct
(c) (ii) and (iii) (d) (i), (ii) and (iii) answer by given codes.
82. Consider the following statements: Column-I Column-II
(i) Fluorine has the highest electron affinity in the periodic (Year) (The number of elements
table. discovered)
(ii) Noble gases are placed extremely left in periodic table. (A) 1800 (p) 118
(iii) Magnesium is more metallic in nature than sodium. (B) 1865 (q) 63
Which of these statement(s) is/are correct ?
(C) At present (r) 31
(a) (i) and (ii) (b) (i) and (iii)
(a) A – (q), B – (p), C – (r)
(c) Only (i) (d) Only (ii)
(b) A – (r), B – (p), C – (q)
83. Which of the following statement(s) is/are incorrect?
(c) A – (q), B – (r), C – (p)
(i) Ionization enthalpy is expressed in units of kJmol–1.
(d) A – (r), B – (q), C – (p)
(ii) Ionization enthalpy is always positive.
88. Match the columns.
(iii) Second ionization enthalpy will be higher than the
Column-I Column-II
third ionization enthalpy.
(A) Newland law (p) Atomic mass vs
(a) Only (ii) (b) Only (iii)
of octaves Atomic volume
(c) (ii) and (iii) (d) None of these
(B) Mendeleev (q) Li, Na, K
84. Consider the following statements:
(C) Electronic (r) One to seven groups sub-
(i) There are 16 groups and 7 periods in the modern
configuration divided into group A and B
periodic table.
(D) Lother Meyer (s) Periodic repetition of
(ii) Electro-positive character decreases on moving down
a group. properties of elements
(iii) Electro-negativity in a period increases right from the (E) Dobereiner's triad (t) Only 56 elements known
alkali metal to the inert gas element. (a) A–(t); B–(s); C–(r); D–(p); E–(q)
Which of these statement(s) is/are correct ? (b) A–(t); B–(r); C–(s); D–(p); E – (q)
(a) (i) and (ii) (b) (i) and (iii) (c) A–(t); B–(r); C–(s); D–(q); E – (p)
(c) All are correct (d) All are incorrect (d) A–(r); B–(t); C–(s); D–(p); E – (q)
EBD_7207
44 CLASSIFICATION OF ELEMENTS AND PERIODICITY IN PROPERTIES
89. Match the columns : 93. Match the columns.
Column-I Column-II Column-I Column-II
(A) On arraging in order of (p) Mendeleev (A) ‘s’ block elements (p) Cr
atomic weights, physical (B) ‘p’ block elements (q) Na
and chemical properties (C) ‘d’ block elements (r) Ce
are repeated at regular (D) ‘f’ block elements (s) Si
intervals.
(a) A – (s), B – (q), C – (p), D – (r)
(B) Elements are arranged in (q) Lothar Meyer
(b) A – (q), B – (s), C – (r), D – (p)
the order of increasing
atomic weights. (c) A – (q), B – (p), C – (s), D – (r)
(C) Elements were arranged (r) Moseley (d) A – (q), B – (s), C – (p), D – (r)
on the basis of similar 94. Match the columns.
properties ignoring order Column-I Column-II
of atomic weights (A) Element with largest size (p) Boron
(D) Atomic number is a (s) Chancourtois in second period
more fundamental (B) Element with smallest size (q) Fluorine
property of an element in group 13
than its atomic mass (C) Element with maximum (r) Bromine
(a) A – (p), B – (q), C – (s), D – (r) non-metallic character
(b) A – (r), B – (s), C – (r), D – (p)
(D) Element with smallest size (s) Lithium
(c) A – (q), B – (p), C – (s), D – (r)
in fourth period
(d) A – (q), B – (s), C – (p), D – (r)
90. Match Column-I (IUPAC nomenclature of element) with (E) Element with most metallic (t) Lead
Column-II (IUPAC official name). character in group 14
Column-I Column-II (a) A – (s), B – (p), C – (q), D – (t), E – (r)
(A) Unnilhexium (p) Lawrencium (b) A – (p), B – (s), C – (q), D – (r), E – (t)
(B) Unniltrium (q) Dubnium (c) A – (s), B – (q), C – (p), D – (r), E – (t)
(C) Unnilunium (r) Seaborgium (d) A – (s), B – (p), C – (q), D – (r), E – (t)
(D) Unnilpentium (s) Mendelevium 95. Match the columns.
(a) A – (s), B – (p), C – (r), D – (q) Column-I Column-II
(b) A – (r), B – (p), C – (s), D – (q) (A) Electronegativity (p) Isotopes
(c) A – (r), B – (s), C – (p), D – (q) (B) Lanthanides (q) increases along a period
(d) A – (q), B – (r), C – (p), D – (s) (C) Transition elements (r) f-group of elements
91. Match the columns.
(D) Ionisation energy (s) d-group of elements
Column-I Column-II
(Name of element) (Group of element) (E) Elements of same (t) decreases along a group
(A) Nitrogen (p) 15 atomic number
(B) Aluminium (q) 16 but different
(C) Chlorine (r) 17 mass number
(D) Oxygen (s) 13 (a) A – (q), B – (r), C – (s), D – (p), E – (t)
(E) Copper (t) 11 (b) A – (r), B – (q), C – (s), D – (t), E – (p)
(a) A – (p), B – (s), C – (r), D – (q), E – (t) (c) A – (q), B – (r), C – (s), D – (t), E – (p)
(b) A – (s), B – (p). C – (r), D – (q), E – (t)
(d) A – (q), B – (s), C – (r), D – (t), E – (q)
(c) A – (p), B–(s), C – (q), D – (r), E – (t)
(d) A – (p), B – (s), C – (r), D – (t), E – (q) 96. Match Column-I with Column-II and select the correct
92. Match the columns. answer by the given codes.
Column-I Column-II Columnn-I Column-II
(Name of element) (Period of element) (Atoms) (Properties)
(A) Hydrogen (p) 3 (A) He (p) High electronegative
(B) Sodium (q) 4 (B) F (q) Most electropositive
(C) Calcium (r) 6
(C) Rb (r) Strongest reducing agent
(D) Barium (s) 1
(E) Iodine (t) 5 (D) Li (s) Highest ionisation energy
(a) A – (p), B – (s), C – (q), D – (r), E – (t) (a) A – (s), B – (q), C – (r), D – (p)
(b) A – (s), B – (p), C – (q), D – (r), E – (t) (b) A – (p), B – (s), C – (q), D – (r)
(c) A – (s), B – (q), C – (p), D – (r), E – (t) (c) A – (s), B – (p), C – (r), D – (q)
(d) A – (s), B – (p), C – (q), D – (t), E – (r) (d) A – (s), B – (p), C – (q), D – (r)
CLASSIFICATION OF ELEMENTS AND PERIODICITY IN PROPERTIES 45
97. Match the Column-I and Column-II and select the correct 103. Assertion : Second period consists of 8 elements.
answer by given codes. Reason : Number of elements in each period is four times
Column-I Column-II the number of atomic orbitals available in the energy level
(Elements) (Properties) that is being filled.
(A) Li+ < Al3+ < Mg2+ < K+ (p) DEA (Electron affinity) 104. Assertion : Helium is placed in group 18 along with p-block
(B) Li+ > Al3+ > Mg2+ > K+ (q) Ionic radii elements.
(C) Cl > F > Br > I (r) EN (Electronegativity) Reason : It shows properties similar to p-block elements.
(D) F > Cl > Br > I (s) Effective nuclear charge 105. Assertion : Hydrogen can be placed in group 1.
(a) A – (q), B – (s), C – (r), D – (p) Reason : Hydrogen can gain an electron to achieve a noble
(b) A – (q), B – (s), C – (p), D – (r) gas arrangement.
(c) A – (s), B – (q), C – (r), D – (p) 106. Assertion : Atomic size increases along a period.
(d) A – (s), B – (q), C – (p), D – (r) Reason : Effective nuclear charge increases as the atomic
98. Match the columns on the basis of diagonal relationship number increases resulting in the increased attraction of
between elements. electrons to the nucleus.
Column-I Column II 107. Assertion : Second ionization enthalpy will be higher the
(A) Li (p) Na first ionization enthalpy.
(B) Be (q) Al Reason : Ionization enthalpy is a quantitative measure of
(C) B (r) Si the tendency of an element to lose electron.
(s) Mg 108. Assertion : Alkali metals have least value of ionization energy
(a) (A) – (s), B – (r), C – (p) within a period.
(b) (A) – (s), B – (q), C – (r) Reason : They precede alkaline earth metals in periodic
(c) (A) – (s), B – (q), C – (p) table.
(d) (A) – (q), B – (s), C – (p) 109. Assertion : Electron gain enthalpy can be exothermic or
99. Match the columns endothermic.
Column-I Column-II Reason : Electron gain enthalpy provides a measure of the
(A) [BF4]– (p) 7, +7 ease with which an atom adds an electron to form anion.
(B) [A1F6]3– (q) 4, +4 110. Assertion : Smaller the size of an atom greater is the
(C) OF2 (r) 6, +3 electronegativity.
(D) SiF4 (s) 2, +2 Reason : Electronegativity refers to the tendency of atom
(E) IF7 (t) 4, +3 so share electrons with other atom.
(a) A – (s), B – (q), C – (t), D – (r), E – (p)
(b) A – (t), B – (r), C – (s), D – (q), E – (p) CRITICAL THINKING TYPE QUESTIONS
(c) A – (q), B – (r), C– (t), D – (s), E – (p) 111. Which fact is not valid for Dobereiner's triads?
(d) A – (r), B – (q), C – (s), D – (p), E – (t) (a) The atomic weight of middle element is roughly average
of the other two elements
ASSERTION-REASON TYPE QUESTIONS (b) The properties of middle element is roughly average
Directions : Each of these questions contain two statements, of the other two elements
Assertion and Reason. Each of these questions also has four (c) The elements of triads belong to the same group of
alternative choices, only one of which is the correct answer. You modern periodic table
have to select one of the codes (a), (b), (c) and (d) given below. (d) The elements of triads have same valency electrons.
(a) Assertion is correct, reason is correct; reason is a correct 112. In the Mendeleev periodic table, which of the following
explanation for assertion. element instead of having lower atomic weight was placed
(b) Assertion is correct, reason is correct; reason is not a after the element of higher atomic weight thereby ignoring
correct explanation for assertion the order of increasing atomic weights.
(c) Assertion is correct, reason is incorrect (a) Iodine (b) Antimony
(d) Assertion is incorrect, reason is correct. (c) Bromine (d) Molybdenum
100. Assertion : In a triad, the three elements present have same 113. Which of the following is correct about Eka-Aluminium and
gaps of atomic masses. Eka-Silicon ?
Reason : Elements in a triad have similar properties. (a) Oxides of Eka-Aluminium is Al2O3 and Eka-Silicon is
101. Assertion : According to Mendeleev, periodic properties Si2O3
of elements is a function of their atomic number. (b) Oxides of Eka-Aluminium is Ga2O3 and Eka-Silicon is
Reason : Atomic number is equal to the number of protons. GeO2
102. Assertion : Atomic number of the element ununbium is 112. (c) Melting point of Eka-Aluminium is lower than the
Reason : Name for digits 1 and 2 is un- and bi-respectively melting point of Eka-Silicon
in latin words. (d) Both (a) and (c)
EBD_7207
46 CLASSIFICATION OF ELEMENTS AND PERIODICITY IN PROPERTIES
114. Which of the following elements are found in pitch blende? 122. An element X belongs to fourth period and fifteenth group
(a) Actinium and protoactinium of the periodic table. Which one of the following is true
(b) Neptunium and plutonium regarding the outer electronic configuration of X ? It has
(c) Actinium only (a) Partially filled d-orbitals and completely filled s-orbitals
(d) Both (a) and (b) (b) Completely filled s-orbital and completely filled
115. Which of the following period contain most of the man- p-orbitals
made radioactive elements? (c) Completely filled s-orbital and half-filled p-orbitals
(a) Seventh (b) Fifth (d) Half-filled d-robitals and completely filled s-orbitals
(c) Sixth (d) Both (a) and (c) 123. An element has electronic configuration 1s2 2s2 2p6 3s2 3p4.
116. The electronic configuration of an element is (a) Period = 3rd, block = p, group = 16
1s 2 2s 2 2 p 6 3s 2 3 p3 . What is the atomic number of the (b) Period = 5th, block = s, group = 1
element, which is just below the above element in the (c) Period = 3rd, block = p, group = 10
periodic table? (d) Period = 4th, block = d, group = 12
(a) 33 (b) 34 124. The periodic table of elements does not
(c) 36 (d) 49 (a) include the inert gases
117. Which of the following elements show the given (b) tell us about the arrangement of atoms in a molecule
properties? (c) allow us to make accurate guess of the properties of
(i) All elements are metals. undiscovered elements
(ii) Most of the elements form coloured ions, exhibit (d) reveal regularities in the occurance of elements with
variable valence and paramagnetism.
similar properties
(iii) Oftently used as catalysts.
125. The lightest liquid metal is
(a) Chalcogens
(a) Hg (b) Ga
(b) Transition elements
(c) Cs (d) Fr
(c) Inner transition elements
126. The correct sequence which shows decreasing order of the
(d) Representative elements
ionic radii of the elements is
118. Which of the given elements A, B, C, D and E with atomic
number 2, 3, 7, 10 and 30 respectively belong to the same (a) Al3 Mg 2 Na F O2
period?
(b) Na Mg 2 Al3 O2 F
(a) A, B, C (b) B, C, D
(c) A, D, E (d) B, D, E (c) Na F Mg 2 O2 Al3
119. According to Mendeleev's periodic classification, the (d) O2 F Na Mg 2 Al3
electronic configuration of hydrogen atom resembles that
of alkali metals, which are given below as : 127. The ionic radii (Å) of C 4– and O2– respectively are 2.60 and
H = 1s1, Li = 2s1, Na = 3s1, K = 4s1 1.40. The ionic radius of the isoelectronic ion N3– would be
On the other hand like halogens, hydrogen also exist as (a) 2.6 (b) 1.71
diatomic molecules, such as : H2, Cl2, Br2, I2, etc. (c) 1.4 (d) 0.95
On the basis of above information hydrogen can be placed 128. Which of the following species will have the smallest and
with : the largest size?
(a) Alkali metals (b) Halogens
Cl, Na, Cl–, Al3+, Mg2+, Na+
(c) Both (a) and (b) (d) None of these
(a) Smallest = Na+, Largest = Cl–
120. Which of the following statements is incorrect from the
point of view of modern periodic table ? (b) Smallest = Al3+, Largest = Cl–
(a) Elements are arranged in the order of increasing atomic (c) Smallest = Al3+, Largest = Cl
number (d) Smallest = Na, Largest = Cl
(b) There are eighteen vertical columns called groups 129. Covalent radii of atoms varies in range of 72 pm to 133 pm
(c) Transition elements fit in the middle of long periods from F to I while that of noble gases He to Xe varies from
(d) Noble gases are arbitrarily placed in eighteenth group 120pm to 220pm. This is because in case of noble gases
121. Element X forms a chloride with the formula XCl2, which is (a) covalent radius is very large
a solid with a high melting point. X would most likely be in
(b) van der Waal radius is considered
the same group of the periodic table as –
(c) metallic radii is considered
(a) Na (b) Mg
(d) None of these
(c) Al (d) Si
CLASSIFICATION OF ELEMENTS AND PERIODICITY IN PROPERTIES 47

130. The van der Waal and covalent radii of fluorine atom 135. If ionisation enthalpy of oxygen is lesser than nitrogen
respectively from the following figure are. because of two of the four 2p– electrons occupy same 2p-
orbital than why such case is not possible with fluorine
294 pm which contain greater no of paired electrons because.
(a) greater size of atomic orbitals
(b) smaller size of orbitals
(c) nuclear charge overpower electronic repulsions.
F F F F (d) None of these
136. Which of the following statements is wrong ?
(a) van der Waal’s radius of iodine is more than its covalent
144 pm 144 pm radius
(b) All isoelectronic ions belong to same period of the
(a) 219pm, 72pm (b) 75pm, 72pm
periodic table
(c) 147pm, 72pm (d) 147pm, 144pm (c) I.E.1 of N is higher than that of O while I.E.2 of O is
131. Arrange the following in increasing order of ionic radii? higher than that of N
C4–,N3–,F–,O2– (d) The electron gain enthalpy of N is almost zero while
(a) C4– < N3– < O2– < F– that of P is 74.3 kJ mol–1
(b) N3– < C4– < O2– < F– 137. Which one of the following statements is incorrect ?
– 2– 3– 4– (a) Greater the nuclear charge, greater is the electron
(c) F < O < N < C
affinity
(d) O2– < F– < N3– < C4– (b) Nitrogen has zero electron affinity
132. The first ( iH1) and second ( iH2) ionization enthalpies (c) Electron affinity decreases from fluorine to iodine in
(in kJ mol –1 ) and the electron gain enthalpy ( egH) 17th group
(in kJ mol–1) of the elements I, II, III, IV and V are given (d) Chlorine has highest electron affinity
below 138. The elements with zero electron affinity are
Element iH 1 iH 2 eg H
(a) Boron and Carbon
I 520 7300 – 60 (b) Beryllium and Helium
(c) Lithium and Sodium
II 419 3051 – 48
(d) Fluorine and Chlorine
III 1681 3374 – 328
139. Which of the following property of element is directly
IV 1008 1846 – 295 related to electronegativity?
V 2372 5251 + 48 (a) Atomic radius (b) Ionization enthalpy
The most reactive metal and the least reactive non-metal of (c) Non-metallic character (d) None of these
these are respectively 140. Which is not the correct order for the stated property.
(a) I and V (b) V and II (a) Ba > Sr > Mg ; atomic radius
(c) II and V (d) IV and V (b) F > O > N ; first ionization enthalpy
(c) Cl > F > I ; electron affinity
133. Among the following transition elements, pick out the
(d) O > Se > Te ; electronegativity
element/elements with highest second ionization energy.
141. In which of the following arrangements, the order is NOT
(A) V (At. no = 23) (B) Cr (At. no = 24) according to the property indicated against it?
(C) Mn (At. no = 25) (D) Cu (At. no = 29) (a) Li < Na < K < Rb :
(E) Zn (At. no = 30) Increasing metallic radius
(a) (A) and (C) (b) (B) and (D) (b) I < Br < F < Cl :
(c) (B) and (E) (d) Only (D) Increasing electron gain enthalpy
(with negative sign)
134. As we move across the second period from C to F ionisation
(c) B < C < N < O
enthalpy increases but the trend from C to F for ionisation
Increasing first ionization enthalpy
enthalpy is C < O < N < F why it is not C < N < O < F. This is
because (d) Al3 Mg 2 Na F
(a) atomic radii of O > atomic radii of N Increasing ionic size
(b) electronic configuration of N is more stable than 142. The compounds of the s-block elements, with the exception
electronic configuration of O of lithium and ...X... are predominantly ionic. Here, X refers
(c) atomic radii of N > atomic radii of O to
(a) hydrogen (b) helium
(d) None of these
(c) magnesium (d) beryllium
EBD_7207
48 CLASSIFICATION OF ELEMENTS AND PERIODICITY IN PROPERTIES
143.Among Al2O3, SiO2, P2O3 and SO2 the correct order of acid 145. Which of the following is the reason for the different
strength is chemical behaviour of the first member of a group of
(a) Al2O3 < SiO2< SO2 < P2O3 elements in the s- and p-blocks compared to that of the
(b) SiO2< SO2 < Al2O3 < P2O3 subsequent members in the same group?
(c) SO2< P2O3 < SiO2 < Al2O3 (i) Small size
(d) Al2O3 < SiO2< P2O3 < SO2 (ii) Large charge / radius ratio
144. Observe the following periodic table :
(iii) Low electronegativity of the element
H He
2
(a) (i) and (iii) (b) (i), (ii) and (iii)
1
Li Be B C Y O F Ne
(c) (i) and (ii) (d) (ii) and (iii)
2, 2 2, 3 2, 4 2, 5 2, 7 2, 8
2, 1 2, 6 146. Which of the following statement(s) is/are correct ?
Na Ag Al Z P S Cl Ar
2, 8, 1 2, 8, 2 2, 8, 3 2, 8, 4 2, 8, 6 2, 8, 7
(i) Aluminium react with HCl to form Al3+ and H2 is
2, 8, 5 2, 8, 8
liberated
K X
2, 8, 8, 1 2, 8, 8, 2 (ii) Aluminium dissolve in NaOH to form NaAl(OH)4 and
Arrange the following elements X, Y, Z in increasing order H2
of their valencies : (a) (i) and (ii) (b) Only (ii)
(a) X > Z > Y (b) Y > Z > X (c) Only (i) (d) Neither (i) nor (ii)
(c) Z > Y > X (d) X > Y > Z
CLASSIFICATION OF ELEMENTS AND PERIODICITY IN PROPERTIES 49

FACT / DEFINITION TYPE QUESTIONS 16. (d) Chloride formulas


(i) Eka-Aluminium = GaCl3(ECl3)
1. (c) Periodic classification of elements follow a logical (ii) Eka-Silicon = GeCl4(ECl4)
consequence of electronic configuration of atoms Mendeleef arranged elements in horizontal rows and
which is used to examine the physical and chemical vertical columns of a table in order to their increasing
properties of the elements. atomic weights.
2. (c) According to Dobereneir’s triad the atomic mass of Br 17. (b)
will be average of the atomic masses of Cl & I
18. (d) 118 elements are known at present. The recently
35.5 127 discovered elements are man-made.
81.25
2 19. (c) Moseley discovered that atomic number is more
3. (a) fundamental property than atomic mass.
4. (a) According to the law of triads the atomic wt of the 20. (d) 6th period consists of 32 elements.
middle element is arithmatic mean of I and III. 21. (b)
At.wt of Cl At wt of I 22. (b) The period number corresponds to the highest
At wt of Br principal quantum number (n) of the element.
2
5. (b) In 1800, only 31 elements were known by 1865 the 23. (a) Atomic number (Z) = 120
number of identified elements had more than doubled IUPAC name = Unbinilium
to 63. At present 116 elements are known. Of them the Symbol = Ubn
recently discovered elements are man-made. 24. (b) Element with atomic number 104 was named by
6. (c) Johann Dobereiner in early 1800’s was the first to American society as Rutherfordium and Kurchatovium
consider the idea of trends among properties of by soviet society.
elements. By 1829 he noted a similarity among the 25. (c) Digit Name
physical and chemical properties of several groups of 1 un
three elements (triads). 4 quad
7. (a) According to law of triad,
Using above notation IUPAC name of element 114 is
Li Na K Ununquadium.
¯ ¯ ¯ 26. (b) Its valence shell has 5 electrons (ns2, np3). It belongs
39 + 7 46 to 5th group of the periodic table.
= = 23
2 2 27. (b)
8. (c) Arithmetic mean of atomic mass of F and Br 28. (d) Because of the presence of same number of valence
electrons the elements of same group have similar
19 80 chemical properties.
= = 49.5.
2 29. (c) Halogens are most electronegative elements i.e., they
Atomic mass of Cl = 35.5 are likely to form anions most readily.
Arithmetic mean of atomic masses of F and Br 30. (d) Barium has atomic number 56. It is an alkaline earth
Atomic mass of Cl. metal i.e., found in s-block.
9. (c) 31. (c)
10. (c) Every eighth element had the similar properties to the 32. (c) Element with Z = 33
first element.
(1s2 2s p6 3s2 p6d10 4s 2 p3 ) lies in fifth (or 15th) group.
11. (b) Lothar Meyer plotted the physical properties such as
atomic volume, melting point and boiling point against 33. (a) The electronic configuration clearly suggest that
atomic weight and obtained a periodically repeated it is a d-block element (having configuration
pattern. (n – 1) d 1– 10 ns 0 – 2 ) which starts from III B and goes
12. (a) 13. (d) 14. (c) till II B. Hence with d3 configuration it would be
classified in the group.
15. (b) Correct order is Dobereiner, Chancourtois, Newlands,
Mendeleev. 34. (d) 35. (c)
EBD_7207
50 CLASSIFICATION OF ELEMENTS AND PERIODICITY IN PROPERTIES
36. (c) Elements (a), (b) and (d) belong to the same group 62. (b) O < S < F < Cl
since each one of them has two electrons in the s sub Electron gain enthalpy –141, –200, –333, – 349 kJ mol–
shell. In contrast, element (c) has seven electrons in 1
the valence shell and hence does not lie in the same 63. (a)
group in which elements (a), (b) and (d) lie. 64. (d) Noble gases have positive values of electron gain
37. (c) enthalpy because the anion is higher in energy than
38. (b) Na and Cl both belongs to III period. the isolated atom and electron.
39. (b) 40. (b) 65. (a) Within a group, electron gain enthalpy becomes less
41. 2 2 6 2
(b) 1s , 2s 2p , 3s 3p 4 negative down a group. However, adding an electron
to the 2p-orbital leads to greater repulsion than adding
¯
an electron to the larger 3p-orbital. Hence,
Number of shell = 3 phosphorus has the least negative electron gain
(Principal quantum number)
enthalpy.
Number of period = 3
66. (a)
Valence electrons = 6 i.e., 2 + 4
67. (a) Electronegativity values of given elements are as
Number of group = 16
follows:
42. (a) Metallic ch aracter decreases down group and
Be – 1.5 (I) Mg – 1.2 (IV)
increases along a period.
O – 3.5 (II) N – 3.0 (III)
43. (b) Cu, Ag and Au are coinage metals. They belong to group
i.e. II > III > I > IV
IB (d-block) of periodic table.
68. (b) It is electronic configuration of alkali metal. Hence it
44. (b) 45. (b)
46. (b) Non-metals are mainly placed in p-block elements. will form basic oxide.
47. (b) Non-metallic character increases on moving from left 69. (d) 70. (c)
to right in a period. 71. (b) Silicon has valence of 4 and bromine has valence of 1.
48. (b) Correct order of ionic size is Sn > Ce > Yb > Lu. Hence formula of compound will be SiBr4.
49. (c) On moving down in a group atomic radii increases due 72. (b) On passing from left to right in a period acidic character
to successive addition of extra shell hence of the normal oxides of the elements increases with
O < S < Se increase in electronegativity.
Further As is in group 15 having one less electron in
its p orbital hence have higher atomic radii than group STATEMENT TYPE QUESTIONS
16 elements. 73. (c) Gallium was placed in place of Eka aluminium and
i.e., O < S < Se < As germanium was placed in place of Eka silicon.
50. (a) 74. (b) 75. (c)
51. (c) Isoelectronic ions have same number of electrons. 76. (d) In fourth period filling up of 3d orbital becomes
52. (a) Continuous increase as no. of shells increases down energetically favourable before the 4p orbital is filled.
the group. 77. (b) The chemical element with atomic number 17 is
53. (d) The size of an anion will be larger than that of the chlorine. It belongs to third period in the periodic table
parent atom because the addition of one or more
and forms anion with unit negative charge (Cl–).
electron(s) would result in increased repulsion among
78. (a) For statement (iii) the s-block elements because of
the electrons and a decrease in effective nuclear
their high reactivity are never found pure in nature.
charge.
For statement (iv) the compounds of the s-block
54. (b) K K 2 e . Since e– is to be removed from stable elements with the exception of lithium and beryllium
configuration. are predominantly ionic.
55. (c) 56. (c) 79. (c) 80. (d) 81. (d)
57. (a) IE1 is always less than IE2. 82. (c) Noble gases are placed extremely right in periodic table.
58. (a) Mg 1s 2 2s 2 2p 6 3s 2 Sodium is more metallic than magnesium as it is more
After removing of 2 electron, the magnesium acquired electropositive and has low ionisation energy.
noble gas configuration hence removing of 3rd 83. (b) Second ionization enthalpy will be higher than the
electron will require large amount of energy. first ionization enthalpy but lower than the third
59. (d) ns1 configuration and lesser IE. ionization enthalpy.
60. (b) As I.E. of Mg is more 84. (d)
61. (a) The halogen (group-17) and the chalcogens (group- 85. (b) Oxygen has smaller first ionization enthalpy than
16) are two groups of elements having highly negative nitrogen.
electron gain enthalpies. 86. (c)
CLASSIFICATION OF ELEMENTS AND PERIODICITY IN PROPERTIES 51

MATCHING TYPE QUESTIONS 105. (b) Both the statements are correct but assertion is not
correct explanation for reason.
87. (d) A. 1800 31 elements were known 106. (c) Atomic size generally decreases along a period.
B. 1865 63 elements 107. (b) Both assertion and reason are true but reason is not
C. At present 118 the correct explanation of assertion.
88. (b) 89. (d) 90. (b) 91. (a) 92. (b) It is difficult to remove an electron from a positively
93. (d) 94. (d) 95. (c) charged ion than a neutral atom.
96. (d) Helium (He) 1s2 Highest ionisation 108. (b) Both assertion and reason are true but reason is not
energy due to noble gas
the correct explanation of assertion.
in nature.
Alkali metals belong to first group and have largest
Fluorine (F) 1s2, 2s22p3 High electronegativity in
size in a period and hence low I.E.
nature due to small size
and –1 oxidation state. 109. (b) Both assertion and reason are true but reason is not
Rubidium (Rb) Most electronegative the correct explanation of assertion.
element due to large Depending on the element, the process of adding an
atomic size. electron to the atom can be either endothermic or
Lithium (Li) Strongest reducing exothermic.
agent due to small size 110. (c) Assertion is true but reason is false.
and positive oxidation Electronegativity refers to the tendency of atom to
state (+1) attract bonding electrons.
97. (b) A. Li+ < Al2+ < Mg2+ < K+
The cation with the greater positive charge will CRITICAL THINKING TYPE QUESTIONS
have a smaller radius because of the greater
attraction of the electrons to the nucleus. Anion 111. (b)
with the greater negative charge will have the larger 112. (a) Iodine with lower atomic weight than that of tellurium
radius. (Group VI) was placed in Group VII along with fluorine,
chlorine, bromine because of similarities in properties.
1
Positive charge 113. (d) Oxides of Eka-Aluminium = Ga2O3
ionic radius Oxides of Eka-Silicon = SiO2
Negative charge ionic radius Melting point of Eka-Aluminium = Low (302 K)
B. Greater positive charge, increases effective nuclear Melting point of Eka-Silicon = High (1231 K)
charge in case of isoelectronic species. While for
114. (d) Neptunium and plutonium like actinium and
same group elements effective nuclear charge
protoactinium are also found in pitch.
decreases down the groups.
C. Cl > F > Br > I 115. (a) Seventh period includes most of the man-made radio-
electron affinity of Cl is highest in halogen family. active elements.
D. F > Cl > Br > I 116. (a) Atomic number of the given element is 15 and it
electronegativity of fluorine (F) is higher than Cl, belongs to 5th group. Therefore atomic number of the
Br and I. element below the above element
98. (b) 99. (b) = 15 + 18 = 33.
117. (b) These are characteristic properties of d-block
ASSERTION-REASON TYPE QUESTIONS elements.
118. (b) 119. (c) 120. (d) 121. (b) 122. (c)
100. (d) In a triad, the atomic mass of the middle element is the
mean of the atomic masses of the first and third 123. (a) By observing principal quantum number (n). Orbital
elements. (s, p, d, f ) and equating no. of e–’s we are able to find
101. (d) According to Mendeleev, periodic properties of the period, block and group of element in periodic table.
elements is a function of their atomic masses. 124. (b) Periodic table deals with elements and not molecules.
102. (a) Both assertion and reason are true and reason is the 125. (c) Cs is a metal. It is liquid at room temperature. It is
correct explanation of assertion. lighter than Hg (also a liquid metal).
103. (c) Number of elements in each period is twice the number 126. (d) All the given species contains 10 e – each i.e.
of atomic orbitals available in the energy level that is isoelectronic.
being filled. For isoelectronic species anion having high negative
104. (c) He (1s2) should be placed along with s-block elements charge is largest in size and the cation having high
because of its electronic configuration but it has a positive charge is smallest.
completely filled valence shell and as a result it exhibits 127. (b) The ionic radii of isoelectronic ions decrease with the
properties of noble gases, thus it is placed along with increase in the magnitude of the nuclear charge.
noble gases (ns2, np6). So, decreasing order of ionic radii is C4– > N3– > O2–.
EBD_7207
52 CLASSIFICATION OF ELEMENTS AND PERIODICITY IN PROPERTIES
128. (b) Anions will be larger and cations will be smaller than 140. (b) On moving along the period, ionization enthalpy
the parent atoms. Among isoelectronic species (Na+, increases.
Mg2+ and Al3+), the one with the larger positive In second period, the order of ionization enthalpy
nuclear charge will have a smaller radius. should be as follows :
Largest = Cl– and smallest = Al3+ F > O > N.
129. (b) In case of halogens covalent radius is considered this But N has half-filled structure, therefore, it is more
bond is formed by overlapping of electron clouds; stable than O. That is why its ionization enthalpy is
while noble gases remain monoatomic, in this case higher than O. Thus, the correct order of IE is
only way to obtain radius is through van der Waal
F > N > O.
radii.
141. (c) In a period the value of ionisation potential increases
130. (c) Covalent radius is radius of an atom in its bound state
from left to right with breaks where the atoms have
i.e., in fluorine it is half of distance between two
some what stable configuration. In this case N has
covalently bonded fluorine atoms; van der Waal radii
half filled stable orbitals. Hence has highest ionisation
is one-half of the distance between the nuclei of two
energy. Thus the correct order is
identical non-bonded isolated atoms. These atoms are
attracted toward each other through weak van der B< C< O < N
Waal’s force hence van der Waal radii are very large. and not as given in option (c)
131. (c) All the given species are isoelectronic. In case of 142. (d) With the exception of lithium and beryllium compounds
isoelectronic species ionic radii increases with of s-block elements are predominantly ionic.
increase in negative charge on anions. 143. (d) As the size increases the basic nature of oxides
132. (c) I represents Li, II represents K changes to acidic nature i.e., acidic nature increases.
III represents Br, IV represents I
SO 2 P 2O 3 SiO 2 Al 2 O 3
V represents He
Acidic Weak Amphoteric
So, amongst these, II represents most reactive metal acidic
and V represents least reactive non-metal.
133. (b) 134. (b) 135. (c) SO2 and P2O3 are acidic as their corresponding acids
136. (b) In the isoelectronic species, all isoelectronic anions H2SO3 and H3PO3 are strong acids.
belong to the same period and cations to the next 144. (c)
period. 145. (c) The anomalous behaviour of first member of a group
137. (c) Electron affinity of 9F is less than that of 17Cl of element in the s- and p-block element is due to their
138. (b) Fully filled electronic configuration. small size, large charge/radius ratio and high
139. (c) The increase in the electronegativities across a period electronegativity.
is accompanied by an increase in non-metallic 146. (a) Because Al is amphotoric in nature so it dissolve in
properties (or decrease in metallic properties) of both acid and base.
elements.
4
CHEMICAL BONDING AND
MOLECULAR STRUCTURE
8. Electrovalence of calcium and chlorine respectively is
FACT / DEFINITION TYPE QUESTIONS
(a) + 2, – 1 (b) + 1, – 1
1. The attractive force which holds various constituents (c) + 1, – 2 (d) + 2, – 2
(atoms, ions etc.) together in different chemical species is 9. When a metal atom combines with non-metal atom, the
called a non-metal atom will
(a) chemical bond (b) chemical compound (a) lose electrons and decrease in size
(c) ionic bond (d) covalent bond (b) lose electrons and increase in size
2. The evolution of various theories of valence and the (c) gain electrons and decrease in size
interpretation of the nature of chemical bonds have closely (d) gain electrons and increase in size
been related to the developments in the understanding of 10. Who introduced the term covalent bond ?
(a) structure of atom (a) Lewis (b) Langmuir
(b) electronic configuration of elements (c) Nyholm and Gillespie (d) Heitler and London
(c) periodic table 11. Which of the following is/are not the condition(s) for
(d) All of the above Lewis dot structure?
3. Who provide explanation of valence based on intertness (i) Each bond is formed as a result of sharing of an
of noble gases ? electron pair between the atoms.
(a) Lewis (b) Ko ssel-Lewis (ii) From the two combining atoms only one atom
contribute electron(s) to the shared pair.
(c) Langmuir (d) Sidgwick & Powell
(iii) The combining atoms attain the outer shell noble
4. In the formation of a molecule which of the following take
gas configurations as a result of the sharing of
part in chemical combination? electrons.
(a) cation (b) anion (a) (i) and (iii) (b) (ii) and (iii)
(c) valence electron (d) inner shell electron (c) (ii) only (d) (iii) only
5. Which of the following do(es) not represent correct Lewis 12. Which of the following does not represent the correct
symbols? Lewis dot structure?
:C : O Ne Be B
I II III IV V Cl Cl H O H
(a) I, IV & V (b) II, III & IV – –
8e 8e 2e

8e

2e

(c) II only (d) II & III
6. The bond formed as a result of the electrostatic attraction (A) (B)
between the positive and negative ions is termed as ...
(a) Chemical bond (b) Electrovalent bond O C O
(c) Co-ordinate bond (d) Covalent bond
– – –
7. Cation and anion combines in a crystal to form following 8e 8e 8e
type of compound (C)
(a) ionic (b) metallic (a) A (b) B
(c) covalent (d) dipole-dipole (c) C (d) A and C
EBD_7207
54 CHEMICAL BONDING AND MOLECULAR STRUCTURE
13. Which of the following statements are correct based on 19. What is X, Y and Z in the following expression of formal
given Lewis dot structure ? charge.
Formal charge (F.C) on an atom in a Lewis structure
(i) N N 1
= X –Y (Z )
2
(a) X = Total number of non bonding electrons
H H Y = Total number of bonding electrons
(ii) C C Z = Total number of valence electrons in the free
atom
H H (b) X = Total number of valence electrons in the free
8e– 8e–
atom
Y = Total number of bonding electrons
(iii) O C O Z = Total number of non bonding electrons
(c) X = Total number of valence electrons in the free
8e– 8e

8e

atom
Y = Total number of non bonding electrons
(iv) H C C H Z = Total number of bonding electrons
(d) X = Total number of electrons in the free atom
– –
8e 8e Y = Total number of non bonding electrons
(a) (i) and (iv) represents formation of triple bond Z = Total number of valence electrons
(b) Only (iii) represents formation of double bond 20. The lowest energy structure is the one with the ..........
(c) Only (ii) represents formation of single bond formal charges on the atoms.
(d) (ii) and (iii) both represents formation of single bond. (a) smallest (b) highest
14. Which of the following Lewis representation of the (c) zero (d) negative
21. 3–
In PO4 ion, the formal charge on each oxygen atom and
molecules NF3, O3 and HNO3 is correct ?
+
P—O bond order respectively are
(a) –0.75, 0.6 (b) – 0.75, 1.0
: :
: — :
: :

: :
: — :
: :

:F – N – F: O O=N–O–H
: —

(c) – 0.75, 1.25 (d) –3, 1.25



: :

O O:
:

:F: :O: 22. In the cyanide ion, the formal negative charge is on
I II III (a) C
Choose the correct option(s). (b) N
(a) Only I (b) Only II (c) Both C and N
(c) Only III (d) I, II and III (d) Resonate between C and N
15. In N 2 molecule, the number of electrons shared by each 23. What are the exceptions of the octet rule ?
(a) The incomplete octet of central atom
nitrogen atom is
(b) An odd number of electrons on central atom.
(a) 1 (b) 2
(c) Expanded octet of the central atom
(c) 3 (d) 5
(d) All of these
16. Which of the following represents the Lewis structure of
24. In which of the following molecules octet rule is not
N2 molecule?
followed?
(a) (b) N N (a) NH3 (b) CH4
N N
(c) CO2 (d) NO
(c) N N (d) N N 25. In which of the following compounds octet is complete and
incomplete for all atoms :
17. Which of the following shows the Lewis dot formula for Al2Cl6 Al2(CH3)6 AlF3 Dimer of Dimer of
CO2 ? BeCl2 BeH2
(a) IC IC IC C C
: : : :
:
: : :

(a) (b) :O:C::O:


:

:O ::C:: O: (b) C IC IC C IC
:

: :

(c) :O:: C:O: (d) :O:C :O: (c) C IC C IC IC


18. Which of the following is the correct electron dot structure (d) IC C IC IC IC
of N2O molecule? (Note : C for complete octet and IC for incomplete octet.)
.. .. 26. Which of the following molecule(s) obey the octet rule?
(a) :N N O: (b) : N N O
.. : (i) [BF4]–, (ii) [AlCl4]–, (iii) SO2, (iv) CCl4
.. .. .. .. (a) (i), (ii), (iii), (iv) (b) (ii), (iii), (iv)
(c) N N O: (d) : N N O :
.. .. (c) (i), (iii), (iv) (d) (i), (ii), (iii)
CHEMICAL BONDING AND MOLECULAR STRUCTURE 55
27. Among the following the electron deficient compound is (a) increases (b) decreases
(a) BCl3 (b) CCl4 (c) remains the same (d) decreases slowly
(c) PCl5 (d) BeCl2 36. Which of the following combination will form an
28. Which of the following is the electron deficient electrovalent bond ?
molecule? (a) P and Cl (b) NH3 and BF3
(a) C2 H6 (b) B2H6 (c) H and Ca (d) H and S
(c) SiH4 (d) PH3 37. Among the following which compound will show the highest
29. Which of the following compounds does not follow the lattice energy ?
octet rule for electron distribution? (a) KF (b) NaF
(a) PCl5 (b) PCl3 (c) CsF (d) RbF
(c) H2O (d) PH3 38. Which of the following bond will have highest ionic
30. A pair of compound which have odd electrons in the group character?
NO, CO, ClO2, N2O5, SO2 and O3 are (a) H–I (b) H–F
(a) NO and ClO2 (b) CO and SO2 (c) H–Cl (d) H–Br
(c) ClO2 and CO (d) SO2 and O3 39. Which of the following pairs will form the most stable ionic
31. Which of the following statements is incorrect ? bond ?
(a) The formation of ionic compounds depend upon the (a) Na and Cl (b) Mg and F
ease of formation of the positive and negative ions (c) Li and F (d) Na and F
from the respective neutral atoms. 40. Which of the following methods is used for measuring
(b) Formation of ionic compounds depend upon bond length ?
arrangement of the positive and negative ions in the (a) X-ray diffraction
solid. (b) Electron-diffraction
(c) Formation of positive ion involves addition of (c) Spectroscopic techniques
electron(s) while that of negative ion involves (d) All of these
removal of electron(s). 41. .......... is measured as the radius of an atom’s core which
(d) None of these is in contact with the core of an adjacent atom in a
32. Complete the following statement by choosing the bonded situation.
appropriate option. (a) van der Waal’s radius
Ionic bonds will be formed more easily between elements (b) Bond length
with comparatively A and elements with (c) Covalent radius
comparatively high negative value of B . (d) Ionic radius
(a) A = low electronegativity 42. Following figure represent a chlorine molecule. Identify A
B = ionization enthalpy B and C in the given figure.
(b) A = low ionization enthalpy
C
B = electron gain enthalpy A
(c) A = high ionization enthalpy
B = electron gain enthalpy
(d) A = high electronegativity
B = ionization enthalpy
33. In ionic solids how crystal structure get stabilized
(a) By the energy released in the formation of crystal
lattice. B
(b) By achieving octet of electrons around the ionic
pm

species in gaseous state.


360

(c) By electron gain enthalpy and the ionization enthalpy.


(d) None of these
34. Energy required to completely separate one mole of a
solid ionic compound into gaseous constituent ions is (a) A = Bond length, B = van der Waal’s radius
called .......... . C = Covalent radius
(a) Ionisation enthalpy (b) A = Covalent radius, B = Bond length
(b) Electron gain enthalpy C = Ionic radius
(c) Bond dissociation enthalpy (c) A = Ionic radius, B = van der Waal’s radius
(d) Lattice enthalpy C = Covalent radius
35. The effect of more electronegative atom on the strength of (d) A = Covalent radius, B = van der Waal’s radius
ionic bond C = Bond length
EBD_7207
56 CHEMICAL BONDING AND MOLECULAR STRUCTURE
43. Which of the following statement is correct? 48. Which of the following is/are misconception(s) associated
(a) Amount of energy required to break one mole of with resonance ?
bonds of a particular type between two atoms in a (i) The molecule exist for a certain fraction of time in
gaseous state is called bond enthalpy. one cannonical form and for other fractions of time
(b) The unit of bond enthalpy is kJ mol–1 in other cannonical forms.
(c) Larger the bond dissociation enthalpy, stronger will (ii) The cannonical forms have no real existence.
be the bond in the molecule (iii) There is no such equilibrium between the cannonical
(d) All of these forms.
44. Complete the following statements. (a) (i) only (b) (ii) and (iii)
With A in bond order, B increases (c) (i) and (iii) (d) (iii) only.
and C decreases.
49. The number of possible resonance structures for CO32 is
(a) A = increase, B = bond length, C = bond enthalpy
(b) A = decrease, B = bond enthalpy, C = bond length (a) 2 (b) 3
(c) A = increase, B = bond enthalpy, C = bond length (c) 6 (d) 9
(d) A = increase, B = bond angle, C = bond enthalpy 50. Which one of the following is not the resonance structure
45. Which of the following molecules have same bond order ? of CO2?
(a) O = C = O (b) – O – C O+
H 2 , Cl2 , CO, Br2 , N 2 (c) + O C–O – (d) O C = O
I II III IV V
51. All the bond lengths of sulphur – oxygen in sulphate ion,
Choose the correct option. are equal because of:
(a) I, II and IV have same bond order
(a) symmetry
(b) III and V have same bond order
(b) resonance
(c) Both (a) and (b) are correct
(c) high electronegativity of oxygen
(d) None of the above
(d) None of these
46. Which one of the following is not correct representation of
52. Resonance is due to
resonance ?
(a) delocalization of sigma electrons
– .. + + .. –
A. :O::C::O: (b) delocalization of pi electrons
: :
:

: :

:O::C::O: :O::C:O:
:

I II III (c) migration of protons


– – (d) Both (a) and (b)
:

:O: :O: :O: 53. Which one of the following pairs of molecules will have
permanent dipole moments for both members ?
B. C C C (a) NO2 and CO2 (b) NO2 and O3
– – : (c) SiF4 and CO2 (d) SiF4 and NO2
:O:– –
:O:–
: :

:O: :O: :O O
: 54. The molecule which has zero dipole moment is
I II III
:

(a) CH3Cl (b) NF3


Choose the correct option. (c) BF3 (d) ClO2
(a) Only A (b) Only B 55. Which of the following has dipole moment?
(c) Both A and B (d) None of the above (a) CO2 (b) p-dichlorobenzene
47. Which of the following structure represents structure of (c) NH3 (d) CH4
O3 more accurately? 56. Identify the non polar molecule in the following compounds
(a) H2 (b) HCl
O O
12

14
pm

pm

(c) HF and HBr (d) HBr


8
1

pm
pm
8

57. A neutral molecule XF3 has a zero dipole moment. The


14

12

O O O O element X is most likely


(a) chlorine (b) boron
I II
(c) nitrogen (d) carbon
58. Among the following, the molecule of high dipole moment
O is
pm

12

(a) CCl4 (b) NH3


8
8

pm
12

(c) H2O (d) CHCl3


O O
59. Which one of the following molecules is expected to have
III
zero dipole moment?
(a) I (b) II (a) H2O (b) CO2
(c) III (d) I and II (c) SO2 (d) CaF2
CHEMICAL BONDING AND MOLECULAR STRUCTURE 57

60. The correct order of dipole moments of HF, H 2S and H 2 O is 71. According to VSEPR theory the geometry of a covalent
molecules depends upon
(a) HF H 2 S H 2 O (b) HF H 2 S H 2 O
(a) the number of bond pairs of electrons
(c) HF H 2S H 2O (d) HF H 2 O H 2 S (b) the number of lone pairs of electrons
61. The most polar bond is (c) the number of electron pairs present in the outer shell
(a) C – F (b) C – O of the central atom
(c) C – Br (d) C – S (d) All the above
62. Which of the following possess dipole moment SF6(a), 72. The geometry of ClO– ion according to Valence Shell Electron
SO2(b), H2S(c) , SF4(d) ?
Pair Repulsion (VSEPR) theory will be
(a) b and c (b) a and c
(a) planar triangular (b) pyramidal
(c) b, c and d (d) a and b
(c) tetrahedral (d) square planar
F 73. In BrF3 molecule, the lone pairs occupy equatorial positions
F to minimize
F F F
.. (a) lone pair - bond pair repulsion only
S : S
S S (b) bond pair - bond pair repulsion only
F F O O F (c) lone pair - lone pair repulsion and lone pair - bond pair
H H
F F repulsion
(d) lone pair - lone pair repulsion only
63. According to Fajan’s rule, covalent bond is favoured by 74. Which of the correct increasing order of lone pair of electrons
(a) Large cation and small anion on the central atom?
(b) Large cation and large anion (a) IF7 < IF5 < CIF3 < XeF2
(c) Small cation and large anion (b) IF7 < XeF2 < CIF2 < IF5
(d) Small cation and small anion (c) IF7 < CIF3 < XeF2 < IF5
64. Arrange the following in increasing order of covalent (d) IF7 < XeF2 < IF5 < CIF3
character (i) NaCl, (ii) RbCl, (iii) MgCl2, (iv) AlCl3 ? 75. The number of lone pair and bond pair of electrons on the
(a) (i), (ii), (iii), (iv) (b) (iv), (ii), (i), (iii) sulphur atom in sulphur dioxide molecule are respectively
(c) (ii), (i), (iii), (iv) (d) (iii), (i), (ii), (iv) (a) 1 and 3 (b) 4 and 1
65. The correct sequence of increasing covalent character (c) 3 and 1 (d) 1 and 4
is represented by 76. A molecule has two lone pairs and two bond pairs around
(a) LiCl < NaCl < BeCl2 (b) BeCl2 < LiCl < NaCl the central atom. The molecule shape is expected to be
(c) NaCl < LiCl < BeCl2 (d) BeCl2 < NaCl < LiCl (a) V-shaped (b) triangular
66. Which of the following salt shows maximum covalent (c) linear (d) tetrahedral
character? 77. Using VSEPR theory, predict the species which has square
(a) AlCl3 (b) MgCl 2 pyramidal shape
(a) SnCl2 (b) CCl4
(c) CsCl (d) LaCl3 (c) SO3 (d) BrF5
67. Polarisibility of halide ions increases in the order 78. Among the following molecules : SO2, SF4, CIF3, BrF5 and
(a) F –, I – , Br–, Cl– (b) Cl –, Br – , I–, F– XeF4, which of the following shapes does not describe any
– – –
(c) I , Br , Cl , F – (d) F –, Cl – , Br–, l– of the molecules mentioned?
68. The covalent bond length is the shortest in which one of (a) Bent (b) Trigonal bipyramidal
the following bonds? (c) See-saw (d) T-shape
(a) C — O (b) C — C 79. Which of the following structure is most stable ?
(c) C N (d) O — H F .. F
69. Hydrogen chloride molecule contains F
..

..

(a) polar covalent bond (b) double bond Cl F F Cl Cl F


(c) co-ordinate bond (d) electrovalent bond
..

70. Sodium chloride is an ionic compound whereas hydrogen .. F F ..


F
chloride is mainly covalent because I II III
(a) sodium is less reactive Choose the correct option.
(b) hydrogen is non-metal (a) Only I
(c) hydrogen chloride is a gas (b) Only II
(d) electronegativity difference in the case of hydrogen (c) Only III
and chlorine is less than 2.1. (d) All three have same stability
EBD_7207
58 CHEMICAL BONDING AND MOLECULAR STRUCTURE
80. A -bonded molecule MX3 is T-shaped. The number of 89. The angle between the overlapping of one s-orbital and
non-bonding pairs of electron is one p-orbital is
(a) 0 (a) 180° (b) 120°
(b) 2 (c) 109 28' (d) 120° 60'
(c) 1 90. The enolic form of a acetone contains
(d) can be predicted only if atomic number of M is known.
(a) 9 sigma bonds, 1 pi bond and 2 lone pairs
81. Shape of methane molecule is
(b) 8 sigma bonds, 2 pi bonds and 2 lone pairs
(a) tetrahedral (b) pyramidal
(c) 10 sigma bonds, 1 pi bond and 1 lone pair
(c) octahedral (d) square planar
82. The shape of stannous chloride molecule is (d) 9 sigma bonds, 2 pi bonds and 1 lone pair
(a) see-saw (b) square planar 91. Linear combination of two hybridized orbitals belonging to
(c) trigonal pyramidal (d) bent two atoms and each having one electron leads to a
83. Look at the following potential energy curve which of the (a) sigma bond
following correctly represents the most stable state of (b) double bond
hydrogen molecule. (c) co-ordinate covalent bond
(d) pi bond.
92. Which of the following statements is not correct ?
(a) Double bond is shorter than a single bond
(b) Sigma bond is weaker than a (pi) bond
(c) Double bond is stronger than a single bond
D (d) Covalent bond is stronger than hydrogen bond
A
O 93. Which of the following represents zero overlap of atomic
C
orbitals.
Energy

(a) z
pz
s

B (b) z
Internuclear distance pz
s
(a) A (b) B
(c) C (d) D
84. Which of the following statements is false ?
(a) H2 molecule has one sigma bond
(c) z
(b) HCl molecule has one sigma bond
(c) Water molecule has two sigma bonds and two lone pairs s
(d) Acetylene molecule has three pi bonds and three sigma
bonds px
85. The number of sigma ( ) and pi ( ) bonds present in 1,3,5,7 (d) All of these
octatetraene respectively are
94. As the s-character of hybridised orbital increases, the bond
(a) 14 and 3 (b) 17 and 4
angle
(c) 16 and 5 (d) 15 and 4
(a) increase (b) decrease
86. Allyl cyanide molecule contains
(c) becomes zero (d) does not change
(a) 9 sigma bonds, 4 pi bonds and no lone pair
(b) 9 sigma bonds, 3 pi bonds and one lone pair 95. Which of the following is/are not essential condition(s)
for hybridisation?
(c) 8 sigma bonds, 5 pi bonds and one lone pair
(d) 8 sigma bonds, 3 pi bonds and two lone pairs (i) The orbitals present in the valence shell of the atom
87. The molecule not having -bond is are hybridised.
(a) Cl 2 (b) O2 (ii) The orbitals undergoing hybridisation should have
(c) N2 (d) CO2 almost equal energy.
88. In hexa-1, 3-diene-5-yne the number of C — C , C — C (iii) Promotion of electron is essential prior to
and C — H bonds, respectively are hybridisation
(a) 5, 4 and 6 (b) 6, 3 and 5 (iv) Only half filled orbitals participate in hybridisation.
(c) 5, 3 and 6 (d) 6, 4 and 5 (a) (i) only (b) (iii) only
(c) (iv) only (d) (iii) and (iv)
CHEMICAL BONDING AND MOLECULAR STRUCTURE 59
96. The nature of hybridisation in the ammonia molecule is 109. In which of the following species is the underlined carbon
(a) sp 2 (b) dp 2 having sp3 - hybridisation ?
(c) sp (d) sp 3 (a) CH – COOH (b) CH CH OH
97. The shape of sulphate ion is
(c) CH COCH (d) CH CH CH
(a) square planar (b) triagonal
110. 3
A sp -hybrid orbital contains
(c) trigonal planar (d) tetrahedral
98. The strength of bonds formed by s–s and p–p, s–p overlap (a) 25% s-character (b) 75% s-character
in the order of (c) 50% s-character (d) 25% p-character
(a) s–p > s–s > p–p (b) p–p > s–s > s–p 111. The types of hybridisation of the five carbon atoms from
left to right in the molecule
(c) s–s > p–p > s–p (d) s–s > s–p > p–p
CH3 — CH == C == CH — CH3 are
99. Which of the following will have sp3 d3 hybridisation?
(a) sp3, sp2, sp2, sp2, sp3 (b) sp3, sp, sp2, sp2, sp3
(a) BrF5 (b) PCl5
(c) sp3, sp2, sp, sp2, sp3 (d) sp3, sp2, sp2, sp, sp3
(c) XeF6 (d) SF6
112. Pick out the incorrect statement from the following
100. The shape of CO 2 molecule is (a) sp hybrid orbitals are equivalent and are at an angle
(a) linear (b) tetrahedral of 180° with each other
(b) planar (d) pyramidal (b) sp2 hybrid orbitals are equivalent and bond angle
101. The hybridisation state of carbon in fullerene is between any two of them is 120°
(a) sp (b) sp2 (c) sp3d2 hybrid orbitals are equivalent and are oriented
towards corners of a regular octahedron
(c) sp3 (d) sp3 d
(d) sp3d3 hybrid orbitals are not equivalent
102. Which of the following statements is true for an ion having
113. All carbon atoms are sp2 hybridised in
sp3 hybridisation?
(a) 1, 3-butadiene (b) CH2 = C = CH2
(a) all bonds are ionic
(c) cyclohexane (d) 2-butene
(b) H-bonds are situated at the corners of a square 114. Which one of the following is not correct in respect of
(c) all bonds are co-ordinate covalent hybridization of orbitals?
(d) H-atoms are situated at the corners of tetrahedron (a) The orbitals present in the valence shell only are
103. Which of the following molecule does not have a linear hybridized
arrangement of atoms ? (b) The orbitals undergoing hybridization have almost
(a) H2S (b) C2H2 equal energy
(c) BeH2 (d) CO2 (c) Promotion of electron is not essential condition for
104. In which one of the following molecules the central atom hybridization
said to adopt sp2 hybridization? (d) Pure atomic orbitals are more effective in forming stable
(a) BeF2 (b) BF3 bonds than hybrid orbitals
(c) C2H2 (d) NH3 115. Molecular orbital theory was given by
105. Considering the state of hybridization of carbon atoms, find (a) Kossel (b) Mosley
out the molecule among the following which is linear ? (c) Mulliken (d) Werner
(a) CH3– CH = CH–CH3 116. Atomic orbital is monocentric while a molecular orbital is
(b) CH3 – C C – CH3 polycentric. What is the meaning of above statements?
(c) CH2 = CH – CH2 – C CH (a) Electron density in atomic orbital is given by the
(d) CH3 – CH2 – CH2 – CH3 electron distribution around a nucleus in an atom.
While in molecular orbital it is given by the electron
106. Equilateral shape has
distribution around group of nuclei in a molecule.
(a) sp hybridisation (b) sp2 hybridisation
3
(b) While an electron in an atomic orbital is influenced
(c) sp hybridisation (d) None of these by one nucleus, in a molecular orbital it is influenced
107. In an octahedral structure, the pair of d orbitals involved in by two or more nuclei depending upon the number
d 2 sp3 hybridization is of atoms in the molecule.
(c) The electron in an atomic orbital is present in one
(a) d d (b) d xz, d nucleus while in molecular orbital electrons are
x2 y 2 , z 2 x2 y 2
d xy, d yz present on more than one nuclei depending upon
(c) d d (d)
z 2 , xz the number of atoms in the molecule.
108. The trigonal bipyramidal geometry is obtained from the (d) All of these
hybridisation 117. With increasing bond order, stability of bond
(a) dsp 3 or sp 3d (b) dsp 2 or sp 2d (a) Remain unaltered (b) Decreases
2 3
(c) d sp or sp d 3 2 (d) None of these (c) Increases (d) None of these
EBD_7207
60 CHEMICAL BONDING AND MOLECULAR STRUCTURE
118. The given increasing order of energies of various 130. The correct statement with regard to H2+ and H2– is
molecular orbitals is not true for which of the following (a) both H2+ and H–2 are equally stable
molecule? (b) both H2+ and H2– do not exist
1s < s < 2s < 2s < ( 2px = 2py) < 2pz < ( 2px
= 2py) < 2pz (c) H2– is more stable than H2
(a) B2 (b) C2
(d) H2+ is more stable than H2
(c) N2 (d) O2
119. Which of the following corresponds unstable molecule? 131. Mark the incorrect statement in the following
Here Nb is number of bonding electrons and Na is number (a) the bond order in the species O2 , O2+ and O2 –
of antibonding electrons.
decreases as O 2 O2 O2
(a) Nb > Na (b) Nb < Na
(c) Na = Nb (d) Both (b) and (c) (b) the bond energy in a diatomic molecule always
120. If Nx is the number of bonding orbitals of an atom and Ny is increases when an electron is lost
the number of antibonding orbitals, then the molecule/atom (c) electrons in antibonding M.O. contribute to repulsion
will be stable if between two atoms.
(a) Nx > Ny (b) Nx = Ny (d) with increase in bond order, bond length decreases
(c) Nx < Ny (d) Nx Ny and bond strength increases.
132. According to molecular orbital theory which of the following
121. In the molecular orbital diagram for O2+ ion, the highest
statement about the magnetic character and bond order is
occupied orbital is
(a) MO orbital (b) MO orbital correct regarding O 2
(c) * MO orbital (d) * MO orbital (a) Paramagnetic and Bond order < O2
122. The theory capable of explaining paramagnetic behaviour (b) Paramagnetic and Bond order > O2
of oxygen is (c) Diamagnetic and Bond order < O2
(a) resonance theory (d) Diamagnetic and Bond order > O2
(b) V.S.E.P.R. theory 133. Bond order is a concept in the molecular orbital theory. It
(c) molecular orbital theory depends on the number of electrons in the bonding and
(d) valence bond energy antibonding orbitals. Which of the following statements is
123. In an anti-bonding molecular orbital, electron density is true about it ? The bond order
minimum (a) can have a negative quantity
(a) around one atom of the molecule (b) has always an integral value
(b) between the two nuclei of the molecule (c) can assume any positive or integral or fractional value
(c) at the region away from the nuclei of the molecule including zero
(d) at no place (d) is a non-zero quantity
124. When two atomic orbitals combine, they form 134. Which of the following does not exist on the basis of
(a) one molecular orbital (b) two molecular orbital molecular orbital theory ?
(c) three molecular orbital (d) four molecular orbital (a) H2+ (b) He2+
125. Paramagnetism is exhibited by molecules (c) He2 (d) Li2
(a) not attracted into a magnetic field 135. The paramagnetic property of the oxygen molecule is due
(b) containing only paired electrons to the presence of unpaired electrons present in
(c) carrying a positive charge
(d) containing unpaired electrons (a) ( 2 px )1 and ( * 2p x )1
126. The difference in energy between the molecular orbital
formed and the combining atomic orbitals is called (b) ( 2 px )1 and ( 2p y )1
(a) bond energy (b) activation energy
(c) stabilization energy (d) destabilization energy (c) ( * 2p y )1 and ( * 2p z )1
127. The bond order in N2+ is
(a) 1.5 (b) 3.0 (d) ( * 2p x )1 and ( * 2p z )1
(c) 2.5 (d) 2.0 136. In which of the following state of compound the
128. Which molecule has the highest bond order? magnitude of H-bonding will be maximum and in which
(a) N2 (b) Li2 case it will be minimum ?
(c) He2 (d) O2 (a) Maximum = Solid, Minimum = Liquid
129. Which one of the following molecules is expected to exhibit (b) Maximum = Liquid, Minimum = Gas
diamagnetic behaviour ? (c) Maximum = Solid, Minimum = Gas
(a) C2 (b) N2 (d) Maximum = Gas, Minimum = Solid
(c) O2 (d) S2
CHEMICAL BONDING AND MOLECULAR STRUCTURE 61

137. Which of the following are correctly classified ? OH


Intermolecular Intramolecular 146. The vapour pressure of is higher than
H-bonding H-bonding NO 2
(a) HF H2O
(b) CH3OH HF OH
(c) H2O o-nitrophenol due to
(d) HF p-nitrophenol O 2N
138. Intramolecular hydrogen bond exists in (a) dipole moment (b) dipole-dipole interaction
(a) ortho nitrophenol (b) ethyl alcohol (c) H-bonding (d) lattice structure
(c) water (d) diethyl ether 147. The reason for exceptionally high boiling point of water is
(a) its high specific heat
139. The boiling point of p-nitrophenol is higher than that of
(b) its high dielectric constant
o-nitrophenol because
(c) low ionization of water molecule
(a) NO2 group at p-position behave in a different way (d) hydrogen bonding in the molecules of water
from that at o-position. 148. Acetic acid exists as dimer in benzene due to
(b) intramolecular hydrogen bonding exists in (a) condensation reaction
p-nitrophenol (b) hydrogen bonding
(c) there is intermolecular hydrogen bonding in (c) presence of carboxyl group
p-nitrophenol (d) presence of hydrogen atom at -carbon
(d) p-nitrophenol has a higher molecular weight than 149. Hydrogen bonding is formed in compounds containing
hydrogen and
o-nitrophenol.
(a) highly electronegative atoms
140. Which one of the following is the correct order of interactions ? (b) highly electropositive atoms
(a) Covalent < hydrogen bonding < vander Waals < dipole- (c) metal atoms with d-orbitals occupied
dipole (d) metalloids
(b) vander Waals < hydrogen bonding < dipole < covalent
(c) vander Waals < dipole-dipole < hydrogen bonding < STATEMENT TYPE QUESTIONS
covalent 150. Read the following statements and choose the correct
(d) Dipole-dipole < vander Waals < hydrogen bonding < sequence of T and F from the given codes. Here T
covalent. represents true and F represents false statement.
141. Strongest hydrogen bond is shown by (i) The number of dots in Lewis symbol represents the
number of valence electrons.
(a) water (b) ammonia
(ii) Number of valence electrons helps to calculate
(c) hydrogen fluoride (d) hydrogen sulphide group valence of element.
142. The low density of ice compared to water is due to (iii) Group valence is given as 8 minus the number of
(a) induced dipole-induced dipole interactions inner shell electrons.
(b) dipole-induced dipole interactions (a) T T T (b) T F F
(c) T T F (d) F F F
(c) hydrogen bonding interactions
151. Based on the following Lewis dot structure which of the
(d) dipole-dipole interactions given statement(s) is/are correct?
143. Methanol and ethanol are miscible in water due to
2–
(a) covalent character O
(b) hydrogen bonding character O C O
(c) oxygen bonding character
(d) None of these (i) There is formation of a double bond and two single
144. The hydrogen bond is shortest in bonds.
(ii) There are two additional electrons than those
(a) S — H --- S (b) N — H --- O
provided by the neutral atoms.
(c) S — H --- O (d) F — H --- F (iii) The least electropositive atom occupies the central
145. Hydrogen bonding is maximum in position in the molecule/ion.
(a) C2H5OH (b) CH3OCH3 (a) (i) and (iii) (b) (i), (ii) and (iii)
(c) (CH3)2 C = O (d) CH3CHO (c) (iii) only (d) (i) and (ii)
EBD_7207
62 CHEMICAL BONDING AND MOLECULAR STRUCTURE
152. Choose the correct sequence of T and F for following 156. Which of the following statements is/are not correct for
statements. Here T stands for true statement and F stands combination of atomic orbitals?
for false statement. (i) The combining atomic orbitals must have the same
(i) Formal charge in the Lewis structure helps in or nearly the same energy.
keeping track of the valence electrons in the (ii) Greater the extent of overlap, the greater will be the
molecule. electron density between the nuclei of a moleculer
(ii) Formal charge indicates the actual charge separation orbital.
within the molecule.
(iii) 2pz orbital of one atom can combine with either of
(iii) Formal charges help in the selection of the lowest
2px, 2py or 2pz orbital of other atom as these orbitals
energy structure from a number of possible Lewis
have same energy.
structures.
(a) T T F (b) T F T (a) (i) and (ii) (b) (iii) only
(c) T T T (d) F T T (c) (i) only (d) (ii) and (iii)
153. Read the following statements and choose the correct
option. Here T stands for True and F stands for False MATCHING TYPE QUESTIONS
statement.
157. Match the columns
(i) The smaller the size of the cation and the larger the
size of the anion, the greater the covalent character Column-I Column-II
of an ionic bond. (A) BeH2 (p) Odd electron molecules
(ii) The smaller the charge on the cation, the greater the (B) SF6 (q) Expanded octet
covalent character of the ionic bond. (C) NO2 (r) Incomplete octet of
(iii) For cations of the same size and charge, the one, central atom
with electronic configuration (n – 1)d nns0, typical of (a) A – (p), B – (q), C – (r)
transition metals, is more polarising than the one (b) A – (q), B – (r), C – (p)
with a noble gas configuration, ns2 np6, typical of
(c) A – (r), B – (q), C – (p)
alkali and alkaline earth metal cations.
(d) A – (r), B – (p), C – (q)
(a) T T T (b) T T F
(c) T F T (d) F T T 158. Match the columns
154. Choose the correct sequence of T and F for following Column-I Column-II
statements. Here T stands for True and F for False
(A) HCl (p) Covalent compound with
statement.
(i) Sigma bond is formed by head on overlap of directional bond
bonding orbitals along the internuclear axis. (B) CO2 (q) Ionic compound with
(ii) Pi bond is formed when atomic orbitals overlap in non-directional bonds
such a way that their axes remain parallel to each
other and perpendicular to the internuclear axis. (C) NaCl (r) Polar molecule
(iii) Half-filled s-orbital of one atom and half filled (D) CCl4 (s) Non-polar molecule
p-orbitals of another atom forms. bond on (a) A – (p, q, r), B – (q, r), C – (p, q), D – (r)
overlapping. (b) A – (q), B – (r), C – (p), D – (s)
(iv) Overlapping in case of pi-bond takes place to a
(c) A – (p, r), B – (p, s), C – (q), D – (p, s)
larger extent as compared to sigma bond.
(a) T T T T (b) T F T F (d) A – (q), B – (r), C – (p, q), D – (s)
(c) T T F F (d) T T F T 159. Match Column-I with Column-II and Column-III and choose
155. Give the correct order of initials T or F for following the correct option from the given codes.
statements. Use T if statement is true and F if it is false : Column-I Column-II Column-III
(i) The order of repulsion between different pair of Molecule (No. of lone (Shape of molecule)
electrons is lp – lp > lp – bp > bp – bp pairs and
(ii) In general, as the number of lone pair of electrons on bond pairs)
central atom increases, value of bond angle from (A) NH3 (i) 1, 2 (p) Bent
normal bond angle also increases
(B) SO2 (ii) 1, 4 (q) Trigonal pyramidal
(iii) The number of lone pair on O in H2O is 2 while on N in
NH3 is 1 (C) SF4 (iii) 2, 3 (r) T-shape
(iv) The structures of xenon fluorides and xenon (D) ClF3 (iv) 1, 3 (s) See-Saw
oxyfluorides could not be explained on the basis of (a) A – (iv, q); B – (ii, p); C – (i, r); D – (iii, s)
VSEPR theory (b) A – (iv, q); B – (i, p); C – (ii, s); D – (iii, r)
(a) TTTF (b) TFTF (c) A – (i, p); B – (iii, s); C – (iv, r); D – (ii, q)
(c) TFTT (d) TFFF (d) A – (iv, p); B – (i, r); C – (iii, q); D – (ii, s)
CHEMICAL BONDING AND MOLECULAR STRUCTURE 63

160. Match the columns (a) A – (p), B – (q), C – (p), D – (r)


Column-I Column-II (b) A – (p), B – (r), C – (q), D – (p)
(A) Trigonal planar (p) PCl5 (c) A – (q), B – (q), C – (p), D – (r)
: (d) A – (r), B – (p), C – (q), D – (p)
162. Match Column-I (molecule) with Column-II (type of
120° hybridisation) and choose the correct option from the
A codes given below.
Column-I Column-II
:
:

(B) Tetrahedral (q) NH4+ (Molecule) (Type of hybridisation)


(A) SF6 (p) sp 3 d
:

(B) PF5 (q) sp 3


109.5°
A (C) BCl3 (r) sp 3 d 2
(D) C2H6 (s) sp 2
: (a) A – (r), B – (p), C – (s), D – (q)
:

(b) A – (r), B – (p), C – (q), D – (s)


(c) A – (p), B – (r), C – (q), D – (s)
:

(C) Trigonal bipyramidal (r) SF6 (d) A – (p), B – (r), C – (s), D – (q)
163. Match the columns
:

Column-I Column-II
(A) Valence bond theory (p) Nyholm and Gillespie
90°
: : (B) Octet rule (q) F. Hund & R. S Mulliken
A (C) Molecular orbital (r) Heitler and London
120°
theory
:

(D) The valence shell (s) Ko ssel and Lewis


electron pair
:

(D) Octahedral (s) BF3 repulsion theory


(a) A – (p), B – (q),C – (r), D – (s)
:

90° (b) A – (q), B – (r),C – (s), D – (p)


:

(c) A – (p), B – (s),C – (q), D – (r)


:

90° A (d) A – (s), B – (r), C – (q), D – (p)


164. Match the columns
:

Column-I Column-II Column-III


:

(a) A – (p), B – (q), C – (r), D – (s)


(b) A – (s), B – (r), C – (q), D – (p)
+
(c) A – (s), B – (q), C – (p), D – (r) (A) 1s (p) – + – + (i)

(d) A – (r), B – (p), C – (q), D – (s)
161. Match the columns *2p
Column-I Column-II
(B) 2pz (q) (ii) – + –
+ + –

+

(A) – (p) negative overlap


*1s

+ – +
+ (C) 2px (r) (iii)
– – +
+

(B) – (q) zero overlap


(a) A– (q, iii), B – (r, i), C – (p, ii)
(C) – (r) positive overlap (b) A– (q, iii), B – (p, ii), C – (r, i)
+

(c) A– (p, iii), B – (q, ii), C – (r, i)


(D) – (d) A– (p, ii), B – (q, iii), C – (r, i)
+

+
EBD_7207
64 CHEMICAL BONDING AND MOLECULAR STRUCTURE
175. Assertion : The bond order of helium is always zero.
ASSERTION-REASON TYPE QUESTIONS
Reason : The number of electrons in bonding molecular
Directions : Each of these questions contain two statements, orbital and antibonding molecular orbital is equal.
Assertion and Reason. Each of these questions also has four 176. Assertion : Bonding molecular orbital has greater stability
alternative choices, only one of which is the correct answer. You than corresponding antibonding molecular orbital.
have to select one of the codes (a), (b), (c) and (d) given below. Reason : The electron density in a bonding molecular
(a) Assertion is correct, reason is correct; reason is a correct orbital is located away from the space between the nuclei
explanation for assertion. while in antibonding molecular orbital it is located
(b) Assertion is correct, reason is correct; reason is not a between the nuclei of the bonded atoms.
correct explanation for assertion 177. Assertion : Water is one of the best solvent.
(c) Assertion is correct, reason is incorrect Reason : H-bonding is present in water molecules.
(d) Assertion is incorrect, reason is correct.
165. Assertion : The correct Lewis structure of O3 may be drawn
CRITICAL THINKING TYPE QUESTIONS
as 178. What is the correct mode of hybridisation of the central
+ atom in the following compounds?
1
NO–2 SF4 PF6 –
:

O 2
(a) sp sp sp 3
– 2 3
2 sp 3 d 2
: :

(b) sp sp d
: :

O O: 3
(c) sp 2 sp 3 d 2 sp 3
Reason : The formal charges on atom 1, 2 and 3 are +1, 0 (d) sp 3 sp 3 sp 3 d 2
and –1 respectively.
179. Which of the following molecules has trigonal planar
166. Assertion : Atoms can combine either by transfer of geometry?
valence of electrons from one atom to another or by (a) BF3 (b) NH3
sharing of valence electrons.
(c) PCl3 (d) IF3
Reason : Sharing and transfer of valence electrons is 180. Which of the following molecules is planar?
done by atoms to have an octet in their valence shell. (a) SF4 (b) XeF4
167. Assertion : The lesser the lattice enthalpy more stable is (c) NF3 (d) SiF4
the ionic compound. 181. Hybridization present in ClF3 is
Reason : The lattice enthalpy is greater, for ions of highest (a) sp 2 (b) sp 3
charge and smaller radii. (c) dsp 2 (d) sp 3 d
168. Assertion : Sulphur compounds like SF6 and H2SO4 have 182. Which of the following represents the given mode of
12 valence electrons around S atom. hybridisation sp2 –sp2 – sp - sp from left to right ?
Reason : All sulphur compounds do not follow octet rule.
(a) H C CH C N (b) HC C–C CH
169. Assertion : BF3 molecule has zero dipole moment.
Reason : F is electronegative and B–F bonds are polar in CH2
nature. (c) H C C C CH (d) H2C
170. Assertion : CH2Cl2 is non-polar and CCl4 is polar molecule. 183. Hybridisation states of C in CH3+ and CH4 are
Reason : Molecule with zero dipole moment is non-polar in (a) sp2 & sp3 (b) sp3 & sp2
nature. (c) sp2 & sp2 (d) sp3 & sp3
171. Assertion : Lone pair-lone pair repulsive interactions are 184. The type of hybridization in xenon atom and the number of
greater than lone pair-bond pair and bond pair-bond pair lone pairs present on xenon atom in xenon hexafluoride
interactions. molecule are respectively
Reason : The space occupied by lone pair electrons is (a) sp3d3 , one (b) sp3d3 , two
3 3
(c) sp d , two (d) sp3d2 , zero
more as compared to bond pair electrons.
172. Assertion : In NH3, N is sp3 hybridised, but angle is found 185. In which of the following species, all the three types of
to be 107°. hybrid carbons are present?
Reason : The decrease in bond angle is due to repulsion (a) CH2 = C = CH2 (b) CH3 – CH= CH – CH+2
(c) CH3 – C C – CH2 + (d) CH3 – CH= CH – CH–2
between the lone pair.
173. Assertion : Shape of NH3 molecule is tetrahedral. 186. If an organic compound contain 92.3% C and 7.7% H, than
Reason : In NH3 nitrogen is sp3 hybridized. number of sp3,sp2 and sp hybridized carbon atoms in all
possible str uctures of compound respectively are
174. Assertion : pi bonds are weaker than bonds.
(molecular mass = 52g/mol)
Reason : pi bonds are formed by the overlapping of p-p
(a) 1, 2, 5 (b) 0, 4, 4
orbitals along their axes.
(c) 0, 8, 4 (d) None of these
CHEMICAL BONDING AND MOLECULAR STRUCTURE 65
187. Arrange the following in increasing order of bond length 197. The ground state electronic configuration of valence shell
(i) N2 (ii) electrons in nitrogen molecule (N 2 ) is written as
N2
KK 2s 2 , * 2s 2 , 2p 2x , 2p 2y 2p 2z Bond order in nitrogen
(iii) N 22
molecule is
(a) (ii), (i) and (iii) (b) (ii), (iii) and (i) (a) 0 (b) 1
(c) (iii), (ii) and (i) (d) (i), (ii) and (iii) (c) 2 (d) 3
188. Which of the following molecule exist ? 198. Bond order in benzene is
(a) He2 (b) Be2 (a) 1 (b) 2
(c) Li2 (d) Both (a) and (b) (c) 1.5 (d) None of these
189. Hybridization and structure of I3– are
199. In O2 , O2 and O 2 2 molecular species, the total number of
(a) sp2 and trigonal planar
(b) sp3d2 and linear antibonding electrons respectively are
(c) sp3d and linear (a) 7, 6, 8 (b) 1, 0, 2
(c) 6, 6, 6 (d) 8, 6, 8
(d) sp3 and T-shape
200. N2 and O2 are converted to monopositive cations N2+ and
190. What is the change in hybridization when AlCl3 changes
O2+ respectively. Which is incorrect ?
to [Al2Cl6]–3 ?
(a) In N2+ the N–N bond is weakened
(a) sp3 d to sp3d2 (b) sp3 to sp3 d
(b) In O2+ the bond order increases
(c) sp2 to sp3d2 (d) None of these
(c) In O2+ the paramagnetism decreases
191. From the given figure the van der Waal radius and covalent
(d) N2+ becomes diamagnetic
radius of the hydrogen atom respectively are
201. Bond order normally gives idea of stability of a molecular
62 pm species. All the molecules viz. H2, Li2 and B2 have the same
bond order yet they are not equally stable. Their stability
order is
(a) H2 > B2 > Li2 (b) Li2 > H2 > B2
240 pm (c) Li2 > B2 > H2 (d) B2 > H2 > Li2
202. According to MO theory which of the following lists ranks
the nitrogen species in terms of increasing bond order?

(a) 151, 31 (b) 120, 31 (a) N 2–


2 N 2– N2 (b) N2 N 22– N 2–
(c) 31, 100 (d) 30, 120 (c) N 2– N 2– N2 (d) N 2– N2 N 2–
2 2
192. Which of the following substances has the greatest ionic
203. Hydrogen bonding would not affect the boiling point of
character ?
(a) HI (b) H2O
(a) Cl2O (b) NCl3
(c) NH3 (d) CH3OH
(c) PbCl2 (d) BaCl2
204. Which one of the following molecules will form a linear
193. Among the following species, identify the pair having same
polymeric structure due to hydrogen bonding?
bond order CN–, O2– , NO+, CN+
(a) NH3 (b) H2O
(a) CN– and O2– (b) O2– and NO+
– + (c) HCl (d) HF
(c) CN and NO (d) CN– and CN+
205. Which among the following can form intermolecular
194. Which of the following is not correct with respect to bond H – bonding ?
length of the species ?
O
(a) C2 C22 (b) B2 B2
H C CHO CHO NO 2
(c) Li 2 Li 2 (d) O2 O2 OH
195. The molecule which has the highest bond order is
(a) C2 (b) N2 OH
(c) B2 (d) O2 OH OH
196. The compound which cannot be formed is (A) (B) (C) (D)
(a) He (b) He (a) A (b) B and D
(c) He 2 (d) 2
He (c) B, C and D (d) A and C
EBD_7207
66 CHEMICAL BONDING AND MOLECULAR STRUCTURE

FACT / DEFINITION TYPE QUESTIONS Cl Cl


1. (a) Chemical bond is an attractive force, which holds 8e

8e

various constituents (atoms, ions etc.) together in 13. (a) (i) represents : N N :
different chemical species. (iv) represents H – C C – H
2. (d) The evolution of various theories of valence and the (ii) and (iii) represents respectively :
interpretation of the nature of chemical bonds have
H H
closely been related to the developments in the C=C and O=C=O
understanding of structure of atom, electronic H H
configuration of elements and Periodic Table. 14. (a) All Lewis representation of the molecules NH3, O3
3. (b) Kossel and Lewis provide some logical explanation and HNO3 given in question are correct.
of valence which was based on the intertness of 15. (c) N2 ; N N
noble gases. 3 electrons are shared by each nitrogen atom
4. (c) In the formation of a molecule, only the outer shell 16. (a) Lewis structure of N2 is N N
electrons take part in chemical combination and they
17. (a) Step I : Skeleton OCO
are known as valence electrons.
Step II : A = 1 × 4 for C + 2 × 6 for O = 4 + 12
5. (d) Valence electrons in O = 6
= 16 electrons
Correct Lewis symbol = O Step III : Total no. of electrons needed to achieve noble
Similarly, Ne is a noble gas having valence electrons gas configuration (N)
=8 N = 1 × 8 + 2 × 8 = 24
Step IV : Shared electrons, S = N – A = 24 – 16
: :

Correct Lewis symbol for Ne = :Ne:


6. (b) Electrovalent bond is formed as a result of electrostatic = 8 electrons
attraction between the positive and negative ions. Step V : O::C::O
:

:
7. (a) The electrostatic force that binds the oppositely Step VI : :O::C ::O: : O = C = O:
charged ions which are formed by transfer of
electron(s) from one atom to another is called ionic ..
N N
bond. Cation and anion are oppositely charged 18. (b)
particles therefore they form ionic bond in crystal. F F F H H H
8. (a) Calcium is assigned a positive electrovalence of two,
while chlorine has a negative electrovalence of one. N –– N– O octet of each atom is complete.
9. (d) When a metal for example Na combines with a non 19. (c) Formal charge (F.C.) on an atom in a Lewis structure
metal e.g., Cl2. Following reaction occurs = [total number of valence electrons in the free atom]
2Na Cl 2 2NaCl – [total number of non bonding (lone pair) electrons]
In this process Na loses one electron to form Na + and – (1/2) [total number of bonding (shared) electrons]
Cl accepts one electron to form Cl– 20. (a) The lowest energy structure is the one with the
smallest formal charges on the atoms.
Na Na e
21. (c) Bond order between P – O
Cl e Cl
Therefore, in this process Cl gain electrons and hence no. of bonds in all possible direction 5
its size increases. 1.25
total no. of resonating structures 4
10. (b) Langmuir (1919) refined the Lewis postulations by –
abandoning the idea of the stationary cubical O O
arrangement of the octet, and by introducing the –
O P O

O P O

term covalent bond. – –


O O
11. (c) Each combining atom contributes at least one O

O

electron to the shared pair. – – –


O P O O P O
12. (a) In formation of Cl2 molecule a pair of electrons is

shared between the two chlorine atoms. Each O O
chlorine atom contribute one electron to the shared 3
pair. Formal charge on oxygen = 0.75
4
CHEMICAL BONDING AND MOLECULAR STRUCTURE 67
22. (b) In CN– ion formal negative charge is on nitrogen atom 30. (a)
due to lone pair of electrons. 31. (c) Formation of positive ion involves removal of
23. (d) According to octet role, the central atom must have 8 electron(s) from neutral atom and that of the
electrons but in some compounds the number of negative ion involves addition of electron(s) to the
electrons is more than 8, or less than 8 or an odd number neutral atom.
of electrons is left on the central atom e.g., PCl5, BF5, 32. (b) Ionic bonds will be formed more easily between
NO. elements with comparatively low ionization enthalpies
and elements with comparatively high negative
F value of electron gain enthalpy.
F 33. (a) In ionic solids, the sum of the electron gain enthalpy
and the ionization enthalpy may be positive but still
F P
the crystal structure gets stabilized due to the
F Cl energy released in the formation of the crystal
:

: :
: :
F Cl:B:Cl N=O lattice.
10 electrons 6 electrons Odd electrons 34. (d) Lattice enthalpy is required to completely separate
around P around B around N one mole of a solid ionic compound into gaseous
24. (d) constituent ions.
35. (a) An ionic bond tightly held the two ions of opposite
Cl Cl Cl charges together, so it is a dipole. More is the
25. (c) (i) Al Al
Cl Cl Cl electronegativity of anion higher will be the electron
(Complete octet) density and higher will be its charge which
consequently increses the strength of ionic bond.
CH3 CH3 CH3 36. (c) Higher the difference in electronegativity between the
(ii) Al Al two atoms, more will be electrovalent character of the
CH3 CH3 CH3
bond. Among given choices, calcium and hydrogen
(3c – 2e) bond
(incomplete octet) have maximum difference in their electronegativities.
37. (b) For compounds containing cations of same charge,
- lattice energy increases as the size of the cation
(iii) AlF3 ® Al3+ + 3F
¾¾
decrease. Thus, NaF has highest lattice energy. The
(ionic 2s 2 2p6 2s2 2p6 size of cations is in the order Na+ < K+ < Rb+ < Cs+
compound)
(octet (octet
complete) complete) 38. (b) Ionic character of a bond is directly proportional to
the difference of electro negativities of bonded atoms.
Cl So, H – F in which electronegativity difference is
(iv) Cl – Be Be – C (Incomplete octet) highest, will have highest ionic character.
Cl
(inomplete octet) 39. (b) The stability of the ionic bond depends upon the lattice
energy which is expected to be more between Mg and
H F due to +2 charge on Mg atom.
(v) H – Be Be – H (Incomplete octet) 40. (d) Bond lengths are measured by spectroscopic, X-ray
H diffraction and electron diffraction techniques.
(3c – 2e– ) bond 41. (c) The covalent radius is measured approximately as
the radius of an atom’s core which is in contact with
the core of an adjacent atom in a bonded situation.
.. .. .. .. ..
O .. S .. O 42. (d) A = Covalent radius
26. (d) .. ..
B = van der Waal’s radius
C = Bond length
Total no. of valence electron around sulphur in SO 2 is
10 while in case of other molecules total no. of 8 43. (d) All of the given statements are correct.
electrons are present in each. 1
44. (c) Bond order bond enthalpy
27. (a) Boron in BCl3 has 6 electrons in outermost shell. bond length
Hence BCl3 is a electron deficient compound.
28. (b) The compounds in which octet of central atom is 45. (c) In CO (three shared election pairs between C and O)
incomplete are known as electron deficient the bond order is 3. For N2 bond order is 3 and its
compounds. Hence B2H6 is a electron deficient Dg H - is 946 kJ mol–1, being one of the highest for a
compound.
diatomic molecule, isoelectronic moleculaes and ions
29. (a) PCl5 does not follow octet rule, it has 10 electrons in
have identical bond order for example F2 and O2–
2 have
its valence shell.
bond ordeer 1, N2, CO and NO+ have bond order 3.
EBD_7207
68 CHEMICAL BONDING AND MOLECULAR STRUCTURE
46. (c) Both representation of resonating structures in 59. (b) O = C = O
molecules of CO2 and CO32– are correct. 60. (a) The correct order of dipole moments of HF, H2S and
47. (c) I and II structure shown above constitute the H2O us
cannonical structure. III structure represents the HF < H2S < H2O
structure of O3 more accurately. This is also called
resonance hybrid. 61. (a) C F
48. (a) The molecule does not exist for a certain fraction of
Because difference between electronegativity of
time in one cannonical form and for other fractions
carbon and flourine is highest.
of time in other cannonical forms.
62. (c) In case of SF6 resultant dipole moment is zero while
49. (b) There are three resonance structures of CO32 ion. all other possess dipole moment.
O O– O– 63. (c)
64. (c) According to Fajan's rule, smaller size and greater
C C C charge on cation favour the formation of partial

O O – –
O O O O– covalent character in ionic bonds.
(I) (II) (III) 65. (c) As difference of electronegativity increases % ionic
50. (d) Choices (a), (b) and (c) are the resonance structures of character increases and covalent character decreases
CO2. i.e., electronegativity difference decreases covalent
character increases.
51. (b) 52. (b)
53. (b) Both NO2 and O3 have angular shape and hence will Further greater the charge on the cation and smaller
have net dipole moment. the size more will be its polarising power. Hence
covalent character increases.
54. (c) The dipole moment of symmetrical molecules is zero.
66. (a) According to Fajan’s rule, as the charge on the cation
F increases, and size decreases, its tendency to polarise
|
the anion increases. This brings more and more
| B
F F covalent nature to electrovalent compounds. Hence
|

Triangular planar (symmetrical molecule) AlCl3 shows maximum covalent character.


55. (c) Dipole moment is a vector quantity, hence the dipole 67. (d) In case of anions having same charge as the size of
moment of symmetrical molecules is zero. As CO2, anion increases, polarisibility of anion also increases.
p-dichlorobenzene and CH4 have regular symmetrical 68. (d) The electronegativity difference is maximum in O—H
shape. Hence = 0 bond hence O—H bond length is the smallest among
Cl the given set.
H 69. (a) A gaseous HCl molecule has hydrogen and chlorine
.. linked by a covalent bond. Here electronegativity of
O=C=O C H N H
chlorine is greater than that of hydrogen. Due to this
H H H H the shared pair of electron is more attracted towards
Cl chlorine. Thus, chlorine end of molecule has higher
However, NH3 has distorted structure due to presence electron density and becomes slightly negative and
of lone pair of electrons on N atom and thus has definite the hydrogen and slightly positive. Hence the covalent
dipole moment. bond in HCl has a polar character as shown below
56. (a) In H2, both atoms are identical, so the molecule is non
polar.
H Cl
57. (b) BF3 has planar and symmetrical structure thus as a
70. (b) Hydrogen is non metal and non metal atoms form
result the resultant of two bond moments, being equal
covalent bond.
and opposite to the third, cancels out and hence
71. (d)
molecule possess zero dipole moment.
F
72. (b) Hybridisation is sp3 and shape pyramidal.

; =0 F
F B
:

73. (c) Br F
F
58. (c) CCl4 and BF3 being symmetrical have zero dipole
moment. H2O, CHCl3 and NH3 have dipole moments :
of 1.84 D, 1.01 D and 1.46 D respectively. Thus among F
the given molecules H2O has highest dipole moment.
CHEMICAL BONDING AND MOLECULAR STRUCTURE 69
In BrF3, both bond pairs as well as lone pairs of 85. (b) H — C — C— C — C— C — C— C — C — H
electrons are present. Due to the presence of lone
pairs of electrons (lp) in the valence shell, the bond H H H H H H H H
angle is contracted and the molecule takes the T- 1, 3,5, 7 - octatetraene
17 and 4
shape. This is due to greater repulsion between two
lone pairs or between a lone pair and a bond pair than 86. (b) Allyl cyanide is :
between the two bond pairs. H H
H
74. (a) The number of lone pairs of electrons on central atom C C C C N
H ,
in various given species are
H
Species Number of lone pairs on
central atom It contains 9 sigma bonds, 3 pi bonds and 1 lone
pair of electrons.
IF7 nil
87. (a) Cl2 : Cl – Cl (1 , No )
IF5 1
O 2 : O = O (1 , 1 )
ClF3 2
N 2 : N N (1 , 2 )
XeF2 3
CO2 : O = C = O (2 , 2 )
Thus the correct increasing order is 88. (a) The given molecule is
IF7 < IF5 < ClF3 < XeF2
0 1 2 3 H2 C C C C C C H
2
..
S H H H
75. (d)
O O The number of C—C ‘ ’ bonds = 5
lp = 1 The number of C—C ‘ ’ bonds = 4
bp = 4 The number of C—C ‘ ’ bonds = 6
76. (a) V-shaped H2O like structure.
77. (d) BrF5 has square pyramidal geometry. 89. (a)
78. (b) SO2 – bent s-orbital p-orbital
SF4 – see-saw The overlap between s- and p-orbitals occurs along
ClF3 – T-shape internuclear axis and hence the angle is 180°.
BrF5 – square pyramidal O–H
XeF4 – square planar.
90. (a) CH3 C CH 2 has 9 ,1 and 2lone pairs.
79. (a) I is the most stable geometry because both the lone
pairs are present at equitorial position. Due to which 91. (a) Linear combination of two hybridized orbitals leads to
repulsion is minimum in molecule as compared to the the formation of sigma bond.
repulsion in other molecules where lone pair is in axial 92. (b) Sigma bond is stronger than -bond. The electrons in
position. the bond are loosely held. The bond is easily broken
X and is more reactive than -bond. Energy released
80. (b) during sigma bond formation is always more than
..

bond because of greater extent of overlapping


M X
93. (c) Option (c) represents zero overlapping.
..

94. (a) Bond angle increases with increase in s-character of


X hybridised orbital. The table given below shows the
Number of lone pair = 2 hybridised orbitals, their % s-chatracter and bond
81. (a) In methane molecule C is sp3 hybridised so its shape angles.
will be tetrahedral. Hybridised % s-character Bond
.. orbitals angle
Sn
82. (d) Bent sp 3 25 109.5°
Cl Cl
sp 2 33 120°
83. (b) The minimum in the energy curve corresponds to the
sp 50 180°
most stable state of H2.
95. (d) Promotion of electron is not an essential condition
84. (d) Structure of acetylene molecule
prior to hybridisation. It is not necessary that only
1 1 1 half filled orbitals participate in hybridisation. In
H— C C— H
2 some cases, even filled orbitals of valence shell take
Thus acetylene molecule has 3 bonds and 2 bonds. part in hybridisation.
EBD_7207
70 CHEMICAL BONDING AND MOLECULAR STRUCTURE
96. (d) The hybridisation in a molecule is given by 100. (a) O C O
1 C(6) = 1s2 2s2 2p2
H [V M C A] .......(1)
2 ground state
where V = no. of valency e– in central atom 2
2s 2px 2py 2pz
M = no. of monovalent atoms around central atom
C = charge on cation, A = charge on anion Excited state
For NH3; V = 5, M = 3, C = 0, A = 0
Putting these values in (1), we get sp hybridized
two electrons excluded
1 from hybridization and
H [5 3 0 0] = 4
2 partcipate in bonding
For H = 4, the hybridisation in molecule is sp3. with oxygens.
97. (d) Hybridisaiton of SO42– ion is given by 101. (b) In fullerene, carbons are sp2-hybridised like graphite.
1 Fullerenes are also the allotropes of carbon other than
H= [V + M + A– C] diamond and graphite but have been produced
2
synthetically, e.g.; C60.
Where V = valency of central metal atom 102. (d) Ions having sp3 hybridisation contain hydrogen atoms
M = no. of monovalent atoms surrounding central metal at the corners of tetrahedron.
atom. 103. (a) For linear arrangement of atoms the hybridisation
A = charge on anion, C = charge on cation should be sp(linear shape, 180° angle). Only H2S has
For SO24 ; V = 6, M = 0, A = 2, C = 0 sp3-hybridization and hence has angular shape while
C2H2, BeH2 and CO2 all involve sp - hybridization and
1 hence have linear arrangement of atoms.
H= [6 + 0 + 2 – 0] = 4 104. (b) BF3 involves sp2-hybridization.
2
i.e., sp3 hybridisation and tetrahedral shape. sp 3 sp sp sp 3
98. (d) The strength of a bond depends upon the extent of 105. (b) H3C — C C — CH3
overlapping. s-s and s-p overlapping results in the linear
formation of bond but extent of overlapping along 106. (b) Equilateral or triangular planar shape involves sp2
internuclear axis is more in case of s-s overlapping hybridization.
than in s-p. p-p overlapping may result in bond if 107. (a) Only those d orbitals whose lobes are directed along
overlapping takes place along internuclear axis or may X, Y and Z directions hybridise with s and p orbitals.
result in –bond if sideways overlapping takes place. In other three d orbitals namely dxy, dyz and dxz, the
In any case the extent of overlapping is lesser in p - p lobes are at an angle of 45° from both axis, hence the
than that of the other two, s-s and s-p. Hence the correct extent of their overlap with s and p orbitals is much
order is lesser than d and d orbitals.
s - s > s - p > p - p. x 2 y2 z2
99. (c) In case of Xenon compounds 108. (a) According to VSEPR theory, trigonal bipyramidal
1 geometry results from sp3d or dsp3 hybridisation.
Hybridization = [No. of valence electrons of dsp2 hybridisation results in square planar geometry,
2
Xe + number of monovalent atoms surrounding Xe – while d2sp3 leads to octahedral shape.
charge on cation + charge or an ion] 109. (b) In CH3 CH2OH underlined C is forming 4 bonds,
hence sp3 hybridisation. In others it is sp2 hybridised
1
In XeF6 = [8 + 6 – 0 + 0] = 7 i.e. sp3d3 hybridizaiton. (due to 3 bonds).
2 110. (a) Each sp3-hybrid orbital has 25% s-character and 75%
In case of SF6, Sulphur is sp3d2 hybridized. p-character.
In case of BrF5 .Bromine atom has seven valence
electrons and 5 are contributed by the fluorine atoms. 111. (c) H3C CH C CH CH 3
sp3 sp 2 sp 2
sp sp3
Total number of electron pairs in valence shell of central
112. (d)
7 5
atom = 6 sp 2 sp sp 2
2 113. (a) CH 2 C CH2
Number of shared pairs = 5
Number of one pairs = 1 sp3
The molecule is square pyramidal. sp3 sp3
In PCl5 P is sp3d hybridised. i.e. the correct answer is sp 3
sp3
sp3
XeF6 or option (c)
CHEMICAL BONDING AND MOLECULAR STRUCTURE 71

sp3 sp 2 sp 2 sp 3 131. (b) The removal of an electron from a diatomic molecule


CH3 CH CH CH 3 may increase the bond order as in the conversion
sp sp sp sp O2 (2) O 2 ( 2 .5) or decrease the bond order as
HC C C CH
in the conversion, N 2 (3.0) N 2 ( 2.5), As a
sp 2 sp 2 sp 2 sp2 result, the bond energy may increase or decrease.
CH 2 CH CH CH2
1, 3-butadiene Thus, statement (b) is incorrect.
114. (d) Statement (d) is incorrect. 132. (b) O2 : 1s 2 , *
1s 2 , 2s 2 , *
2 s 2 , 2 p z2 ,
115. (c) Molecular orbital theory was given by Mulliken.
116. (b) Atomic orbital is monocentric because an electron in 2 p 2x *
2 p1x
it is influenced by one nucleus. While molecular ,
orbital is polycentric as it is influenced by two or 2 p 2y *
2 p1y
more nuclei depending upon the number of atoms in
the molecule. 10 6
Bond order 2
117. (c) 2
118. (d) For oxygen correct increasing order is (two unpaired electrons in antibonding molecular orbital)
1s < s < 2s < 2s < 2pz < ( 2px = 2py) <
( px= *2py) < 2pz
119. (d) Nb < Na or Na = Nb 2 p x2 , *
2 p1x
O 2 : 1s 2 , *
1s 2 , 2s 2 , *
2s 2 , 2 p 2z ,
i.e., a negative or zero bond order corresponds to an 2 p 2y , *
2 p 0y
unstable molecule.
120. (a) 121. (c) 10 5
122. (c) Paramagnetism of O2 is best explained by molecular Bond order 2. 5
2
orbital theory.
(One unpaired electron in antibonding molecular
123. (b)
orbital)
124. (b) One bonding M.O. and one anti-bonding M.O.
125. (d) Molecules having unpaired electrons show Hence O2 as well as O 2 both are paramagnetic, and
paramagnetism.
126. (c) bond order of O 2 is greater than that of O2.
127. (c) N 2 = 7 + 7 – 1 = 13 electrons 133. (c)
Configuration is 134. (c) Helium molecule does not exist as bond order of
1s 2 , *1s 2 , 2s 2 , * 2s 2 , 2 px 2 He2 = 0.
= 2 p y 2 , 2 p1z 135. (c) The paramagnetic property in oxygen came through
Bond order = unpaired electron which can be explained by molecular
1 No. of e s in bonding No. of e s in antibonding orbital theory.
2 molecular orbital molecular orbital
1 1
= (9 4) 5 2.5
2 2 2p*x
128. (a)
129. (a, b) The molecular orbital structures of C2 and N2 are
N2 1s 2 *1s2 2s 2 *2s 2 2 px2 2 py2 2 pz2 2pz* 2py*
C2 1s 2 *1s 2 2 s 2 * 2 s 2 2 py 2 2 Pz2
Both N2 and C2 have paired electrons, hence they are px py pz px py pz
diamagnetic.
130. (d) H 2 ( 1s1)
1 1
Bond order = (1 0)
2 2
H 2 : ( 1s 2 ) ( *1s1 ) 2px bonding

1 1
Bond order = (2 1)
2 2 2p*y and 2p*z
So 2 unpaired of electron present in
The bond order of H 2 and H 2 are same but H 2 is more 136. (c) H-bonding is maximum in the solid state and
stable than H 2 . In H 2 the antibonding orbital is filled minimum in gaseous state.
with 1 electron so this causes instability.
EBD_7207
72 CHEMICAL BONDING AND MOLECULAR STRUCTURE
137. (c) H2O shows intermolecular hydrogen bonding while 151. (d) The least electronegative atom occupies the central
o-nitrophenol shows intramolecular H-bonding. position in the molecule/ion.
O
152. (b) Formal charges do not indicate real charge separation
within the molecule. Indicating the charges on the
138. (a) N (Intermolecular
O hydrogen bonding) atoms in the Lewis structure only helps in keeping
OH track of the valence electrons in the molecule.
153. (c) The greater the charge on the cation, the greater the
o-nitrophenol
covalent character of the ionic bond.
139. (c) The b.p. of p-nitrophenol is higher than that of 154. (d) Statement (c) and (d) are incorrect.
o-nitrophenol because in p-nitrophenol there is 155. (b) (ii) (F) In general as the number of lone pair of electrons
intermolecular H-bonding but in o-nitrophenol it is on central atom increases, value of bond angle from
intramolecular H-bonding. normal bond angle decreases due to lp – lp > lp – bp.
140. (b) The strength of the interactions follows the order (iv)(F) Structures of xenon fluorides and xenon
vander Waal’s < hydrogen – bonding < dipole-dipole oxyfluoride are explained on the basis of VSEPR theory.
< covalent. In SOBr2, S – O bond has minimum bond length in
141. (c) H–F shows str ongest H-bonds due to high comparison to S – O bond lengths in SOF2 and SOCl2,
electronegativity of F atom. because in SOBr 2, S – O bond has been formed by
142. (c) Ice has many hydrogen bonds which give rise to cage hybrid orbital containing less s-character.
like structure of water molecules. This, structure 156. (b) Atomic orbitals having same or nearly same energy
possess larger volume and thus makes the density of will not combine if they do not have the same
ice low. symmetry. 2pz orbital of one atom cannot combine
143. (b) Methanol and ethanol are soluble because of the with 2px or 2py orbital of other atom because of their
hydrogen bonding. different symmetries.
144. (d) F—H----F bond is shortest, because with the increase
of electronegativity and decrease in size of the atom to MATCHING TYPE QUESTIONS
which hydrogen is linked, the strength of the hydrogen
157. (c) BeH2 : Incomplete octet of central atom.
bond increases.
Be has 2 valence electrons
145. (a) Hydrogen bonding is possible only in compounds
SF6 : Expanded octet
having hydrogen attached with F, O or N.
There are 12 electrons around the S atom in SF6
C2 H 5 OH CH 3 O CH 3 NO2 : Odd electron molecules.
(H-bonding possible) (H-bonding not possible)
In molecules with an odd number of electrons like
O O NO2, the octet rule is not satisfied.
|| || 158. (c)
CH3 C CH 3 CH 3 C H
(H-bonding not possible) (H-bonding not possible) 159. (b) NH3 1lp, 3bp Trigonal pyramidal
SO2 1lp, 2bp Bent
146. (c) ortho-Nitrophenol has intramolecular H-bonding SF4 1lp, 4bp See-saw
.
. . OH ClF3 2lp, 3bp T-shape
. O and para-nitrophenol has 160. (c) Trigonal planar = BF3
||

. N Tetrahedral = NH +
O 4
Trigonal bipyramidal = PCl5
intermolecular H-bonding.
147. (d) Hydrogen bonding increases the boiling point of Octahedral = SF6
compound. 161. (c)
148. (b) 162. (a) SF6 sp3d2
149. (a) Hydrogen bond is formed when hydrogen is attached PF5 sp3d
with the atom which is highly electronegative and BCl3 sp2
having small radius. C2H6 sp3
163. (c) Valence bond theory = Heitler and London
STATEMENT TYPE QUESTIONS Octet rule = Ko ssel and Lewis
150. (c) The group valence of the elements is generally either Molecular orbital theory = F. Hund and R.S. Mulliken
equal to the number of dots in Lewis symbols or 8 VSEPR theory = Nyholm and Gillespie
minus the number of dots or valence electrons. 164. (b)
CHEMICAL BONDING AND MOLECULAR STRUCTURE 73

ASSERTION-REASON TYPE QUESTIONS 172. (a) Both assertion and reason are true and reason is the
correct explanation of assertion.
1 lone pair – lone pair repulsion > lone pair – bond pair
:
O repulsion > bond pair – bond pair repulsion. In the
165. (a) 2 : : 3 ammonia molecule, NH3 there are three bond pairs and
: :

O O: one lone pair. The three N – H bond pairs are pushed


closer because of the lone pair – bond pair repulsion,
1
Formal charge on O1 = 6 – 2 – (6) = +1 and HNH bond angle gets reduced from 109°23’ (the
2 tetrahedral angle) to 107°.
1 173. (d) Assertion is false but reason is true.
Formal charge on O2 = 6 – 4 – (4) = 0 NH3 molecule is pyramidal is shape, because out of
2
four electron pairs, three are bonding pairs and one is
1 lone pair.
Formal charge on O3 = 6 – 6 – × 3 = –1
2 174. (a) Both assertion and reason are true and reason is the
Hence, correct representation of O3 is correct explanation of assertion.
pi bonds are formed by the overlapping of p-p orbitals
(+1)
perpendicular to their axis i.e., sidewise overlap.
:

O
175. (a) Both assertion and reason are true and reason is the
(0) (–1)
: :
: :

O O: correct explanation of assertion.


Helium molecule is formed by linking two helium atoms.
166. (a) Atoms combine either by transfer of valence
Both have 1s orbitals. These will combine to form two
electrons from one atom to another or by sharing of
molecular orbitals (1s) and * (1s). Four available
valence electrons to have an octet in their valence
electrons are accommodated as (1s)2 and * (1s)2.
shell.
176. (c) The electron density in a bonding molecular orbital
167. (d) Assertion is false but reason is true.
is located between the nuclei of the bonded atoms
The greater the lattice enthalpy, more stable is the ionic because of which the repulsion between the nuclei
compound.
is very less while in case of an antibonding
168. (c) Sulphur forms many compounds in which the octet molecular orbital, most of the electron density is
rule is obeyed. For example SCl2 has an octet of located away from the space between the nuclei.
electrons around it. Electrons placed in a bonding molecular orbital tend
169. (b) Both assertion and reason are true but reason is not to hold the nuclei together and stabilise a molecule.
the correct explanation of assertion. 177. (b) Both assertion and reason are true but reason is not
BF3 is sp2 hybridized. Dipole moment is a vector the correct explanation of assertion.
quantity. The three bond moments give a net sum of Water is excellent solvent because it has high value of
zero, as the resultant of any two is equal and opposite dielectric constant. Due to high value of dielectric
to the third. constant, the electrostatic force of attraction between
F the ions decrease and these ions get separated and
ultimately get solvated by the solvent molecules.
F B =0 ( + )= 0
CRITICAL THINKING TYPE QUESTIONS
F 178. (b) Hybridisation of the central atom in compound is given
(a) (b)
by
170. (d) Assertion is false but reason is true. 1
CH2Cl2 is polar while CCl4 is non-polar because in H= [V M C A]
2
CCl4 net dipole moment cancels.
where V = No. of valency electrons in central metal
171. (a) While the lone pairs are localised on the central
atom,
atom, each bonded pair is shared between two
M = No. of monovalent atoms surrounding the central
atoms. As a result, the lone pair electrons in a
atom,
molecule occupy more space as compared to the
bonding pairs of electrons. This results in greater C = charge on cation and A = charge on anion
repulsion between lone pairs of electrons as compared 1
to the lone pair -bond pair and bond pair - bond pair For NO 2 , H = [5 0 0 1] 3
2
repulsions.
sp2 hybridisation
EBD_7207
74 CHEMICAL BONDING AND MOLECULAR STRUCTURE

1 (c) CH3 — C C— CH 2
For SF4, H = [6 4 0 0] 5 sp sp
2 sp 3 sp 2

sp3d hybridisation
(d) CH3 — CH2 CH— CH 2–
2
1 sp3 sp sp sp3
For PF6–, H =
[5 6 0 1] 6
2 Note : Carbocations and carboanions are sp2 and sp3
sp3d2 hybridisation. hybridised respectively.
So, option (a) is correct choice. 186. (c) Let amount of compound = 100 g
179. (a) BF3 is sp2 hybridised. So, it is trigonal planner. NH3, 92.3
PCl 3 has sp 3 hybridisation hence has trigonal No. of moles of C 7.69 7.7
12
bipyramidal shape, IF3, has sp3d hydridization and
has linear shape. 7.7
No. of moles of H 7.7
1 1
180. (b) XeF4 hybridisation is (V X C A)
2 Empirical formula = CH
hence V = 8 (no. of valence e– ) Empirical formula mass = 12 + 1 = 13 g/mol
X = 4 (no. of monovalent atom) Molecular mass = 52 g/mol
1 52
(8 4 0 0) 6 sp3d 2 n 4
2
13
C = 0 charge on cation
Molecular foumula = Empirical formula × 4
F F = C4H4
Xe Possible structures
A = 0 (charge on anion). The shape is square
F F H H
sp sp H H
planar shape. C C C C H C C C C
H H, H,
181. (d) Hybridisation present in a molecule can be find out by sp 2
sp 2 sp sp sp2 sp2
the following formula.
1 H H 2
H (V M C A) sp2 C C sp
2
Where V = No. of electrons in valence shell of central H C C H
atom 2
sp sp2
M = No. of singly charged atoms
C = charge on cation 187. (d) As the bond order decreases, bond length increases
A = charge on anion Bond order
1 No. of bonding e s No. of antibonding e s
So, Hybridisation (in ClF3) = [7 3 0 0] = 5
2 2
sp3d Hybridisation For N2, electronic configuration is
182. (a) CH 2 CH C N 1s 2 *1s2 2s2 * 2s 2 ( 2p 2x 2p2y ) 2p 2z
1 2 3 4
3 bonds (sp2 hybridisation); 2 bonds 10 4
(i) Bond order of N2 3
(sp - hybridisation) 2
C1 = 3 bonds, C2 = 3 bonds, 9 4
C3 = 2 bonds (ii) Bond order of N 2 2.5
2
183. (a) Hybridisation of carbon in CH 3 is sp2 and in CH4 its
hybridisation is sp3 8 4
(iii) Bond order of N 22 2
184. (a) XeF4 having one lone pair of electron show distorted 2
pentagonal bipyramidal shape and sp3d3 hybridisation. Hence, order of Bond length will be,
185. (c) (a) CH 2 C CH2 N2 N2 N2
sp
sp 2 sp 2
188. (c) According to molecular orbital theory, bond order of
(b) CH3 — CH2 CH— CH 2 Li2 is 1 , while in all other cases bond order is 0, so
2
sp3 sp sp sp 2 they do not exist. Li2 molecules are known to exist in
the vapour phase.
CHEMICAL BONDING AND MOLECULAR STRUCTURE 75
189. (c) I3 has – sp3d hybridisation and has linear structure 194. (d) On calculating bond order of species given in question
I 2
C2 = 2 C2 3
:
: I : B2 = 1
B2 0.5
I
Li2 0.5 Li2 = 1
190. (c)
N2 2.5 N2 = 3
sp2 hybridisation
O2 = 2.0 O2 1.5
For AlCl 3
1
(excited state) Bond length
Bond order
Cl Cl Cl
O2 O2
3 2
sp d hybridisation 195. (b) For C2 (12) :
( 1s)2 ( 1s)2 ( 2s)2 ( 2s)2( 2px)2 ( 2py)2
3–
For [Al2Cl6] 8 4
B.O. 2
(excited state) 2

Cl Cl Cl Cl– Cl– Cl– For N2(14) :


191. (b) Covalent radius is half of the distance between atoms ( 1s)2 ( *1s)2 ( 2s) 2 ( * 2s)2 ( 2p x ) 2 ( 2p y )2 ( 2p z )2
in bonding state, while van der Waal radius is half of
the distance between atoms in its non bonding state. 10 4
B.O. 3
192. (d) According to Fajan's rule : 2
1 For B2 (10)
Covalent character
size of cation = ( 1s)2 ( *1s) 2 ( 2s)2 ( * 2s) 2 ( 2p x )1 ( 2p y )1
size of anion
6 4
Among the given species order of size of cations B.O. 1
2
N3+ < O2+ < Pb2+ < Ba2+
For O2 (16)
order of size of anions O2– > Cl–.
Hence the order of covalent character is = ( 1s)2 ( *1s)2 ( 2s)2 ( * 2s) 2 ( 2p z ) 2 ( 2p x ) 2
NCl3 Cl2O PbCl2 BaCl2 ( 2p y ) 2 ( * 2p x )1 ( * 2p y )1
BaCl2 is most ionic in nature.
193. (c) M.O. electronic configuration of CN– is 1s2 *1s2 10 6
B.O. 2
s2 *2s2 2px2 2py2 2pz2 2
10 – 4 F2 is (18)
B.O. 3
2 = ( 1s)2 ( *1s)2 ( 2s)2 ( * 2s) 2 ( 2p z ) 2 ( 2p x ) 2
M.O. electronic configuration of O2– is
1s2 *1s2 2s2 2s2 2pz2 2px2 ( 2p y ) 2 ( * 2px ) 2 ( * 2p y ) 2
2py2 *2px2 *2py1
10 8
10 – 7 B.O. 1
B.O. 1.5 2
2
N2 has the highest bond order = 3.
M.O. electronic configuration of CN+
1s2 *1s2 2s2 *2s2 2px2 2py2 2pz1 1
196. (c) B.O. = [N b N a ] 0 ; cannot exist
9–4 2
B.O. 2.5
2 197. (d) In this configuration, there are four completely filled
M.O. electronic configuration of NO+ is bonding molecular orbitals and one completely filled
1s2 *1s2 2s2 *2s2 2pz2 2px2 2py2 antibonding molecular orbital. So that Nb = 8 and
10 – 4 Na 2.
B.O. 2
2 1 1
CN– and NO+ have bond order equal to 3 Bond order = ( N b Na ) (8 2) 3.
2 2
EBD_7207
76 CHEMICAL BONDING AND MOLECULAR STRUCTURE
198. (c) Benzene has the following resonance structures– Though the bond order of all the species are same
(B.O = 1) but stability is different. This is due to
difference in the presence of no. of anti-bonding
electron.
Hence, its bond order is Higher the no. of anti-bonding electron lower is the
stability hence the correct order is H2 > Li2 > B2
no of possible resonating structures 202. (a) Molecular orbital configuration of
1.5.
2
N 2–
2 1s 2 *1s 2 2 s 2 * 2 s 2 -
199. (a) Molecular orbital electronic configuration of these
species are : 2 px2 * 2 p1x
2 p2z
O2 (17e ) 1s 2 , *1s 2 , 2s 2 , * 2 s 2 , 2 pz2 , 2 p 2y * 2 p1y

2 px2 2 p2y , *2 px2 2* p1y 10 – 6


Bond order = 2
2
O2 (16e ) 1s 2 , *1s 2 , 2 s 2 , * 2 s 2 , 2 p z2 ,
2 px2 * 2 p1x
N 2– 1s 2 *1s 2 2s 2 * 2s 2 2 pz2
2 px2 2 p 2y , *2 p1x * 2 p1y 2 p 2y *2 p 0y
O22 (18e ) 1s 2 , *1s 2 , 2 s 2 , * 2 s 2 , 2 p z2 , 10 5
2.5
Bond order =
2
2 px2 2 p 2y , *2 px2 *2 p 2y
2 px2
Hence number of antibonding electrons are 7, 6 and 8 N2 1s 2 *1s 2 2 s 2 * 2 s 2 , 2 pz2
respectively. 2 p 2y
2
200. (d) b *a2 2
b
*2 2
a ( b
2 1
b ) b (N 2 13electrons) 10 – 4
Bond order = 3
it contains one unpaired electron hence paramagnetic. 2
201. (N) None of the given option is correct. The correct order is = N 22– N 2– N 2
The molecular orbital configuration of the given 203. (a) Hydrogen bonding is not possible in HI due to low
molecules is electronegativity of iodine. So, hydrogen bonding
H2 = 1s2 (no electron anti-bonding) would not affect boiling point of HI.
Li2 = 1s2 1s2 2s2(two anti-bonding electrons) 204. (d) HF form linear polymeric structure due to hydrogen
bonding.
B2 = 1s2 1s2 2s2 2s2 2p1y 2p1z 205. (c) B, C and D form intermolecular hydrogen bonding
(4 anti-bonding electrons) while A form intramolecular hydrogen bonding due to
proximity of oxygen and hydrogen.
5
STATES OF MATTER

FACT / DEFINITION TYPE QUESTIONS (a) more than unit electronic charge
(b) equal to unit electronic charge
1. Which of the following is not a type of van der Waal's (c) less than unit electronic charge
forces? (d) double the unit electronic charge
(a) Dipole - dipole forces 7. Dipole-dipole interaction is stronger than the London forces
(b) Dipole - induced dipole forces but is weaker than ion-ion interaction because
(c) Ion - dipole forces (a) only partial charges are involved
(d) London forces (b) only total charges are involved
2. Who proposed the concept of dispersion force ? (c) both (a) and (b)
(a) Heitler and London (b) van der Waal (d) sometimes (a) and sometimes (b)
(c) Gay Lussac (d) Fritz London 8. Induced dipole moment depend upon the
3. Which of the following option correctly represents the I. dipole moment present in the permanent dipole.
relation between interaction energy and distance between II polarisability of the electrically neutral molecules.
two interacting particles (r) for London forces ?
Identify the correct option.
1 1 (a) I is correct but II is wrong
(a) (b)
r r6 (b) I is wrong and II is correct
(c) Both I and II are wrong
1 1 (d) Both I and II are correct
(c) 12 (d)
r r3 9. Dipole-induced dipole interactions are present in which of
4. The interaction energy of London force is inversely the following pairs :
proportional to sixth power of the distance between two (a) Cl2 and CCl4 (b) HCl and He atoms
interacting particles but their magnitude depends upon (c) SiF4 and He atoms (d) H2O and alcohol
(a) charge of interacting particles 10. Which of the following exhibits the weakest intermolecular
(b) mass of interacting particles forces ?
(c) polarisability of interacting particles (a) NH3 (b) HCl
(d) strength of permanent dipoles in the particles. (c) He (d) H2O
5. London forces are always ...I... and interaction energy is 11. Strength of the hydrogen bond is determined by interaction
inversely proportional to the ...II... power of the distance between the
between two interacting particles. I. lone pair of the electronegative atom and the hydrogen
Here, I and II refer to atom of other atom.
(a) I repulsive, II sixth II. bond pair of the electronegative atom and the hydrogen
(b) I attractive, II fourth atom of other atom.
(c) I attractive, II sixth Identify the correct option.
(d) I repulsive, II fourth (a) Only I is correct
6. Dipole-dipole forces act between the molecules possessing (b) Only II is correct
permanent dipole. Ends of dipoles possess ‘partial charges’. (c) Both I and II are correct
The partial charge is (d) Neither I nor II are correct
EBD_7207
78 STATES OF MATTER
12. Which of the following statements regarding thermal energy
is correct ?
(a) Thermal energy is the measure of average kinetic III. P IV. P
energy of the particles of the matter and is thus
responsible for movement of particles. 1/V 1/V

(b) Intermolecular forces tend to keep the molecules (a) Only I (b) II and IV
together but thermal energy of the molecules tends to (c) I and III (d) Only III
keep them apart. 20. Which of the following represents Boyle's law in terms of
(c) Three states of matter are the result of balance between density ?
intermolecular forces and the thermal energy of the d
molecules. (a) d . p = k' (b) k'
p
(d) All of the above
13. Which of the following is the correct order of thermal energy dp
in three states of matter ? (c) k' (d) d k ' p
2
(a) Solid < Liquid < Gas (b) Liquid < Gas < Solid (Here d = density, p = pressure, k' = constant)
(c) Liquid < Solid < Gas (d) Gas < Solid < Liquid 21. Boyle’s law states that the
14. Which of the following are arrangement in the correct order?
(a) pressure of a gas is directly proportional to the
I. Gas > Liquid > Solid (Thermal energy)
temperature at constant volume
II. Solid > Liquid > Gas (Intermolecular force)
Select the correct option. (b) pressure of a gas is inversely proportional the volume
(a) I only (b) II only at constant temperature
(c) Both I and II (d) None of these (c) volume is directly proportional to the temperature at
15. Which one of the following statements is not correct about constant pressure
the three states of matter i.e., solid, liquid and gaseous ? (d) None of the above
(a) Molecules of a solid possess least energy whereas 22. 600 c.c. of a gas at a pressure of 750 mm of Hg is compressed
those of a gas possess highest energy. to 500 c.c. Taking the temperature to remain constant, the
(b) The density of solid is highest whereas that of gases increase in pressure, is
is lowest (a) 150 mm of Hg (b) 250 mm of Hg
(c) Gases like liquids possess definite volumes (c) 350 mm of Hg (d) 450 mm of Hg
(d) Molecules of a solid possess vibratory motion 23. The lowest hypothetical or imaginary temperature at which
16. Which of the following is true about gaseous state ? gases are supposed to occupy zero volume is called
(a) Thermal energy = Molecular attraction _________
(b) Thermal energy >> Molecular attraction (a) Kelvin temperature (b) absolute zero
(c) Thermal energy << Molecular attraction (c) Charle's temperature (d) constant temperature
24. 500 ml of nitrogen at 27°C is cooled to –5°C at the same
(d) Molecular force >> Those in liquids
pressure. The new volume becomes
17. The first reliable measurement on properties of gases was
(a) 326.32 ml (b) 446.66 ml
made by _____________
(c) 546.32 ml (d) 771.56 ml
(a) Gay Lussac (b) Jacques charles
25. What is the value of X in °C for given volume vs temperature
(c) Robert Boyle (d) Avogadro
curve ?
18. At constant temperature, for a given mass of an ideal gas p1 p < p < p < p
(a) the ratio of pressure and volume always remains p2 1 2 3 4
constant. p3
Volume p4
(b) volume always remains constant.
(c) pressure always remains constant.
X Temperature (°C)
(d) the product of pressure and volume always remains
(a) 0° C (b) 273.15° C
constant.
(c) – 273.15° C (d) 300° C
19. Which of following graph(s) represents Boyle's law
26. Which of the following expression at constant pressure
represents Charle’s law?
I. P II. P 1 1
(a) V (b) V
T T2
V V
(c) V T (d) V d
STATES OF MATTER 79

27. The following graph illustrates 34.

(a) Dalton’s law V1 < V2 < V3 < V4


V1
V

Pressure (bar)
(b) Charle’s law V2
V3
(c) Boyle’s law
V4
(d) Gay-Lussac’s law
Temp. (ºC) 0 100 200 300 400
Temperature (K)
28. Air at sea level is dense. This is a practical application of
The relationship which is shown in the figure is derived
(a) Boyle’s law (b) Charle’s law
from
(c) Kelvin’s law (d) Brown’s law I. Boyle's law.
29. Use of hot air balloons in sports and meteorological II. Avogadro law.
observations is an application of III. Charles' law.
(a) Boyle’s law (b) Charle’s law Which of the following is the correct option ?
(c) Kelvin’s law (d) Gay-Lussac’s law (a) I and II (b) II and III
30. An ideal gas will have maximum density when (c) I and III (d) Only I
(a) P = 0.5 atm, T = 600 K (b) P = 2 atm, T = 150 K 35. Which of the following represents Avogadro law ?
(c) P = 1 atm, T = 300 K (d) P = 1 atm, T = 500 K m
(a) V = kn (b) V = k
M
31. (c) M = kd (d) All of these
P1 < P2 < P 3 < P 4
36. At STP molar volume of an ideal gas or a combination of
P1 ideal gases is ___________
Volume (mL)

P2
(a) 22.71098 L mol–1 (b) 20.71098 L mol–1
P3 (c) 22.4139 L mol –1 (d) 24.78 L mol–1
P4 37. 4.4 g of a gas at STP occupies a volume of 2.24 L, the gas
can be
–273.5 (a) O2 (b) CO
–300 –200 –100 0 100 (c) NO2 (d) CO2
Temperature (°C)
38. An ideal gas is one which obeys the gas laws under
(a) a few selected experimental conditions
Arrange the pressures P1, P2, P3 and P4 in the increasing (b) all experimental conditions
order which are shown in the graphs. (c) low pressure alone
(a) P1 < P2 < P3 < P4 (b) P4 < P3 < P2 < P1 (d) high temperature alone
(c) P1 = P2 = P3 = P4 (d) P4 = P3 < P2 = P1 39. For an ideal gas, number of moles per litre in terms of its
32. On a ship sailing in pacific ocean where temperature is 23.4°C, pressure P, gas constant R and temperature T is
a balloon is filled with 2 L air. What will be the volume of the (a) PT/R (b) PRT
balloon when the ship reaches Indian ocean, where (c) P/RT (d) RT/P
temperature is 26.1°C ? 40. Select one correct statement. In the gas equation, PV = nRT
(a) 2.018 L (b) 2.8 L (a) n is the number of molecules of a gas
(b) V denotes volume of one mole of the gas
(c) 3.5 L (d) 1.5 L
(c) n moles of the gas have a volume V
33. Which of the following represents Gay Lussac's law ?
(d) P is the pressure of the gas when only one mole of gas
P is present.
I. constant II. P1T2 = P2T1
T 41. Correct gas equation is
III. P1V1 P2V2 V1T2 V2T1 PV
1 1 T1
(a) (b)
Choose the correct option. P1 P2 P2V2 T2
(a) I, II and III (b) II and III
PT
1 2 P2V2 V1V2
(c) I and III (d) I and II (c) (d) P1P2
V1 T2 T1T2
EBD_7207
80 STATES OF MATTER
42. The correct value of the gas constant ‘R’ is close to : 52. The pressure exerted by 6.0g of methane gas in a 0.03 m3
(a) 0.082 litre-atmosphere K vessel at 129°C is (Atomic masses : C = 12.01, H = 1.01 and
(b) 0.082 litre-atmosphere K–1 mol–1 R = 8.314 JK–1 mol –1)
(c) 0.082 litre – atmosphere–1 K mol–1 (a) 31684 Pa (b) 215216 Pa
(c) 13409 Pa (d) 41648 Pa
(d) 0.082 litre –1 atmosphere – 1 K mol
53. A gaseous mixture was prepared by taking equal mole of
43. If P, V, M, T and R are pressure, volume, molar mass,
CO and N2. If the total pressure of the mixture was found 1
temperature and gas constant respectively, then for an ideal
atmosphere, the partial pressure of the nitrogen (N2) in the
gas, the density is given by
mixture is
RT P (a) 0.5 atm (b) 0.8 atm
(a) (b)
PM RT (c) 0.9 atm (d) 1 atm
54. A bubble of air is underwater at temperature 15°C and the
M PM pressure 1.5 bar. If the bubble rises to the surface where the
(c) (d)
V RT temperature is 25°C and the pressure is 1.0 bar, what will
44. Pure hydrogen sulphide is stored in a tank of 100 litre happen to the volume of the bubble ?
capacity at 20° C and 2 atm pressure. The mass of the gas (a) Volume will become greater by a factor of 1.6.
will be (b) Volume will become greater by a factor of 1.1.
(a) 34 g (b) 340 g (c ) Volume will become smaller by a factor of 0.70.
(c) 282.68 g (d) 28.24 g (d) Volume will become greater by a factor of 2.5.
45. At N.T.P the volume of a gas is found to be 273 ml. What 55. A mixture contains 64 g of dioxygen and 60 g of neon at a
will be the volume of this gas at 600 mm of Hg and 273°C? total pressure of 10 bar. The partial pressures in bar of
(a) 391.8 ml (b) 380 ml dioxygen and neon are respectively (atomic masses O = 16,
Ne = 20)
(c) 691.6 ml (d) 750 ml
(a) 4 and 6 (b) 6 and 4
46. Pressure of a mixture of 4 g of O2 and 2 g of H2 confined
(c) 5 and 5 (d) 8 and 2
in a bulb of 1 litre at 0°C is
56. 500 mL of air at 760 mm pressure were compressed to 200
(a) 25.215 atm (b) 31.205 atm mL. If the temperature remains constant, what will be the
(c) 45.215 atm (d) 15.210 atm pressure after compression?
47. Gas equation PV = nRT is obeyed by (a) 1800 mm (b) 1900 mm
(a) Only isothermal process (c) 2000 mm (d) 1500 mm
(b) Only adiabatic process 57. Value of universal gas constant (R) depends upon
(c) Both (a) and (b) (a) Number of moles of gas
(d) None of these (b) Volume of gas
48. The total pressure of a mixture of two gases is : (c) Temperature of gas
(a) the sum of the partial pressures (d) None of these
58. Two vessels of volumes 16.4 L and 5 L contain two ideal
(b) the difference between the partial pressures
gases of molecular existence at the respective temperature
(c) the product of the partial pressures of 27°C and 227°C and exert 1.5 and 4.1 atmospheres
(d) the ratio of the partial pressures respectively. The ratio of the number of molecules of the
49. If three unreactive gases having partial pressures PA, PB former to that of the later is
and PC and their moles are 1, 2 and 3 respectively then their (a) 2 (b) 1
total pressure will be
1 1
(a) P = PA + PB + PC (b) P PA PB PC (c) (d)
2 3
6
59. 56 g of nitrogen and 96 g of oxygen are mixed isothermally
PA PB PC and at a total pressure of 10 atm. The partial pressures of
(c) P (d) None of these
3 oxygen and nitrogen (in atm) are respectively
50. Dalton’s law of partial pressure will not apply to which of (a) 4, 6 (b) 5, 5
the following mixture of gases (c) 2, 8 (d) 6, 4
(a) H2 and SO2 (b) H2 and Cl2 60. If 10–4 dm3 of water is introduced into a 1.0 dm3 flask at 300
(c) H2 and CO2 (d) CO2 and Cl2 K, how many moles of water are in the vapour phase when
equilibrium is established ?
51. Pressure exerted by saturated water vapour is called
(Given : Vapour pressure of H2O at 300 K is 3170 Pa;
_____________
R = 8.314 J K–1 mol–1)
(a) Aqueous tension (b) Partial pressure (a) 5.56× 10–3 mol (b) 1.53 × 10–2 mol
(c) Total pressure (d) Both (a) and (b) (c) 4.46 × 10–2 mol (d) 1.27 × 10–3 mol
STATES OF MATTER 81
61. The total pressure of a mixture of two gases is 68. Kinetic theory of gases proves
(a) the sum of the partial pressures (a) only Boyle’s law (b) only Charles’ law
(b) the difference between the partial pressures (c) only Avogadro’s law (d) All of these
(c) the product of the partial pressures 69. Which one of the following is the wrong assumption of
(d) the ratio of the partial pressures kinetic theory of gases ?
62. Ideal gas equation is the combination of (a) Momentum and energy always remain conserved.
I. Boyle's law (b) Pressure is the result of elastic collision of molecules
II. Charles's law with the container’s wall.
III. Avogadro law (c) Molecules are separated by great distances compared
IV. Dalton's law of partial pressure to their sizes.
Choose the correct option. (d) All the molecules move in straight line between collision
(a) Only I and II (b) I, II and III and with same velocity.
(c) II and III (d) I, III aor IV 70. When is deviation more in the behaviour of a gas from the
63. According to the kinetic theory of gases, in an ideal gas, ideal gas equation PV = nRT ?
between two successive collisions a gas molecule travels (a) At high temperature and low pressure
(a) in a wavy path (b) At low temperature and high pressure
(b) in a straight line path (c) At high temperature and high pressure
(c) with an accelerated velocity (d) At low temperature and low pressure
(d) in a circular path 71. In van der Waal’s equation of state of the gas law,
64. Gases consist of large number of identical particles (atoms theconstant ‘b’ is a measure of
or molecules) that are so small and so far apart on the average (a) volume occupied by the molecules
that the actual volume of the molecules is negligible in (b) intermolecular attraction
comparison to the empty space between them. (c) intermolecular repulsions
Above given statement explain which property of gases ? (d) intermolecular collisions per unit volume
(a) Gases occupy all the space available to them. 72. In van der Waal’s equation of state for a non-ideal gas, the
(b) Gases has fixed shape. term that accounts for intermolecular forces is
(c) Compressibility of gases. (a) (V – b) (b) RT
(d) None of these. a
65. If there were loss of kinetic energy, the motion of gas (c) P (d) (RT)–1
V2
molecules will _____A _______ and gas will ____B _______.
73. The values of van der Waal’s constant ‘a’ for the gases O2,
(a) A = increase, B = collide
N2, NH3 and CH4 are 1.360, 1.390, 4.170 and 2.253 L2 atm
(b) A = stop, B = settle down mol–2 respectively. The gas which can most easily be liquified is
(c) A = increase,
(a) O2 (b) N2
B = exert more pressure on walls of container (c) NH3 (d) CH4
(d) A = decrease, B = get liquified 74. A gas described by van der Waal’s equation
66. Which of the following assumption of kinetic molecular (i) behaves similar to an ideal gas in the limit of large
theory states that gases do not have fixed shape ? molar volume
(a) Particles of a gas move in all possible directions in (ii) behaves similar to an ideal gas in the limit of large
straight line. pressure
(b) Particles of a gas are always in constant and random (iii) is characterised by van der Waal's coefficients that are
motion. dependent on the identity of the gas but are
(c) Total energy of molecules before and after the collision independent of the temperature
remains same. (iv) has the pressure that is lower than the pressure exerted
(d) None of these by the same gas behaving ideally
67. Which of the following assumption of kinetic theory if hold (a) (i) and (ii) (b) (i) and (iii)
good than the pressure vs volume graph of experimental (c) (i), (ii) and (iii) (d) (ii) and (iv)
data (real gas) and that of theoretically calculated from Boyle's
75. The units of constant a in van der Waal’s equation is
law (ideal gas) should coincide ?
(a) dm6 atm mol–2 (b) dm3 atm mol–1
(i) There is no force of attraction between the molecules
(c) dm atm mol–1 (d) atm mol–1
of a gas.
76. The van der Waal’s constant ‘a’ for four gases P, Q, R and S
(ii) Volume of the molecules of a gas is negligibly small in
are 4.17, 3.59, 6.71 and 3.8 atm L2 mol–2 respectively.
comparison to the space occupied by the gas.
Therefore, the ascending order of their liquefaction is
(a) (i) only (b) (ii) only
(a) R < P < S < Q (b) Q < S < R < P
(c) Both (i) and (ii) (d) None of these
(c) Q < S < P < R (d) R < P < Q < S
EBD_7207
82 STATES OF MATTER
77. At low pressure the van der Waal’s equation is reduced to 85. Following table represents critical temperature of some
gases. Arrange these gases in their increasing order of
pVm ap pVm b
(a) Z 1 (b) Z 1 p liquifaction.
RT RT RT RT
pVm a Gas Tc / K
(c) pVm RT (d) Z 1
RT RT H2 33.2
78. The compressibility factor for a real gas at high pressure is : He 5.3
RT N2 126
(a) 1 + (b) 1 O2 154.3
pb

pb pb (a) He < N2 < H2 < O2 (b) H2 < He < N2 < O2


(c) 1 + (d) 1 – (c) He < H2 < N2 < O2 (d) O2 < N2 < H2 < He
RT RT
86. Choose the correct statement based on the following
79. The gas with the highest critical temperature is
isotherms of carbon dioxide at various temperature.
(a) H2 (b) He
(c) N2 (d) CO2 G
80. A gas is said to behave like an ideal gas when the relation H
PV/T = constant. When do you expect a real gas to behave 50°C
like an ideal gas ? P3
D 31.1°C
(a) When the temperature is low
(b) When both the temperature and pressure are low
73
(c) When both the temperature and pressure are high
Pressure
(Pc ) E
(d) When the temperature is high and pressure is low 33.98°C
C 21.5°C (Tc )
81. An ideal gas can’t be liquefied because P2 B
F
(a) its critical temperature is always above 0°C
(b) Its molecules are relatively smaller in size P1
13.1°C

(c) it solidifies before becoming a liquid A


(d) forces between its molecules are negligible
82. Gases possess characteristic critical temperature which V3 V2 Vc V1
depends upon the magnitude of intermolecular forces Volume
between the particles. Following are the critical temperature (i) We can move from point A to F vertically by increasing
of some gases. the temperature.
Gases H2 He O2 N2 (ii) We can reach the point G by compressing the gas at
Critical temperature 33.2 5.3 154.3 126 constant temperature.
in Kelvin (iii) We can move down from G towards D by increasing
From the above data what would be the order of liquefaction the temperature.
of these gases ? (iv) As soon as we cross the point D on the critical isotherm
Start writing the order from the gas liquefying first we get liquid.
(a) H2, He, O2, N2 (b) He, O2, H2, N2 (a) (i) and (ii) (b) (i), (ii) and (iii)
(c) N2, O2, He, H2 (d) O2, N2, H2, He (c) (i), (ii) and (iv) (d) (i), (ii), (iii) and (iv)
83. Above Boyle point, real gases show ______X _______ from 87. At 1 atm pressure boiling temperature is called _____X ____.
ideality and Z values are _______Y ________ than one. If pressure is 1 bar then the boiling point is called
(a) X = Negative deviation, Y = Less _____Y _____ of the liquid.
(b) X = Negative deviation, Y = Greater (a) X = Standard boiling point, Y = Normal boiling point
(c) X = Positive deviation, Y = Less (b) X = Normal boiling point, Y = Standard boiling point
(d) X = Positive deviation, Y = Greater (c) X = Critical boiling point, Y = Normal boiling point
84. Select the one that when used would be considered as best (d) X = Critical boiling point, Y = Standard boiling point
88. Which of the following statement is incorrect ?
condition for liquification of a gas.
(a) Standard boiling point of liquid is slightly lower than
(a) Increasing the temperature. the normal boiling point.
(b) Decreasing the pressure. (b) 1 atm pressure is slightly less than 1 bar pressure
(c) Increasing the pressure and decreasing the (c) The normal boiling point of water is 100°C and its
temperature. standard boiling point is 99.6°C
(d) Decreasing the pressure and increasing the temperature. (d) None of the above
STATES OF MATTER 83
89. A liquid can exist only 99. The correct order of viscosity of the following liquids will
(a) between triple point and critical temperature be
(b) at any temperature above the melting point (a) Water < methyl alcohol < dimethyl ether < glycerol
(c) between melting point and critical temperature (b) methyl alcohol < glycerol < water < dimethyl ether
(d) between boiling and melting temperature (c) dimethyl ether < methyl alcohol < water < glycerol
(d) glycerol < dimethyl ether < water < methyl alcohol
90. The kinetic energy of molecules in gaseous state is
(a) more than those in the liquid state STATEMENT TYPE QUESTIONS
(b) less than those in the liquid state
(c) equal to those in the liquid state 100. Which of the following statement(s) is/are true for London
force ?
(d) less than those in solid state
(i) These forces are always attractive.
91. A pin or a needle floats on the surface of water, the reason
(ii) These forces are important for long distance too.
for this is
(iii) Their magnitude depends on the polarisability of the
(a) surface tension (b) less weight
particle.
(c) upthrust of liquid (d) None of the above (a) (i) and (ii) (b) (i) only
92. The spherical shape of rain-drop is due to (c) (iii) only (d) (i) and (iii)
(a) density of the liquid (b) surface tension 101. Choose the correct sequence of T and F for following
(c) atmospheric pressure (d) gravity statements. Here T stands for true and F stands for false
93. Which of the following phenomena is caused by surface statement.
tension ? (i) Dipole - dipole forces act between the molecules
(a) Particles at the bottom of river remain separated but possessing permanent dipole.
they stick together when taken out. (ii) Partial charge possessed by these dipoles is always
(b) A liquid rise in a thin capillary. equal to unit electronic charge.
(c) Small drops of mercury from spherical bead instead of (iii) Dipole - dipole interaction is weaker than London forces
spreading on the surface. and ion - ion interaction.
(d) All of the above (a) TTF (b) TFF
(c) TTT (d) TFT
94. A drop of oil is placed on the surface of water. Which of the
following statement is correct ? 102. Which of the following statements are correct ?
(i) Hydrogen bonding is a special case of dipole - dipole
(a) It will remain on it as a sphere
interaction.
(b) It will spread as a thin layer
(ii) Energy of hydrogen bond varies between 10 to
(c) It will be partly as spherical droplets and partly as thin 100 kJ mol–1.
film (iii) Hydrogen bonds are powerful force in determining the
(d) It will float as a distorted drop on the water surface structure and properties of compounds like proteins,
95. When the temperature increases, the viscosity of nucleic acids etc.
(a) gases decreases and viscosity of liquids increases (iv) Strength of the hydrogen bond is determined by the
(b) gases increases and viscosity of liquids decreases coulombic interaction between the lone-pair electrons
(c) gases and liquids increases of the electronegative atom of one molecule and the
hydrogen atom of other molecule.
(d) gases and liquids decreases
(a) (i) and (ii) (b) (i), (ii) and (iii)
96. The surface tension of which of the following liquid is
(c) (ii), (iii) and (iv) (d) All of these
maximum?
103. Which of the following statements are correct regarding
(a) C2H5OH (b) CH3OH
the characteristic of gases ?
(c) H2O (d) C6 H6 (i) Gases are highly compressible.
97. In which phenomena water changes into water vapour below (ii) Gases exert pressure equally in all directions.
its B.P. ? (iii) Gases have much higher density than the solids and
(a) Evaporation (b) Condensation liquids.
(c) Boiling (d) No such phenomena exist (iv) Gases mix evenly and completely in all proportion
without any mechanical aid.
98. The liquid which has the highest rate of evaporation is
Choose the correct option.
(a) petrol (b) nail-polish remover (a) (i), (ii) and (iii) (b) (ii), (iii) and (iv)
(c) water (d) alcohol (c) (i), (ii) and (iv) (d) (i), (ii), (iii) and (iv)
EBD_7207
84 STATES OF MATTER
104. Read the following statements and choose the correct
MATCHING TYPE QUESTIONS
option. Here T stands for true and F stands for false
statement. 108. Match the columns
(i) Equation : V
nRT
will be applicable to any gas, under Column-I Column-II
P
(A) Attractive force that (p) Dipole-dipole
those conditions when behaviour of the gas
operates between the force
approaches ideal behaviour.
polar molecules having
(ii) Value of universal gas constant at 0° C and 1 atm permanent dipole and
pressure is 8.20578 × 10–2 L atm K–1 mol–1 the molecule lacking
(iii) Ideal gas equation describes the state of any gas, permanent dipole
therefore it is also called equation of state.
(B) Interaction in which (q) London force
(iv) Value of R in units of Pa m3 K–1 mol–1 is 8.314 × 10–2
interaction energy
(a) T T T T (b) T T T F
between stationary
(c) T F T F (d) T F F T
polar molecules is
105. Choose the correct sequence of T and F for following
statements. Here T stands for true and F stands for false. 1
proportional to
(i) There may be exchange of energy between colliding r3
molecules, their individual energies may change, but
(C) Force that are important (r) Dipole-induced
the sum of their energies remains constant.
(ii) At any particular time, different particles in the gas only at short distances dipole force
have different speeds and hence different kinetic (~ 500 pm)
energies. (a) A – (r), B – (p), C – (q)
(iii) Particles of a gas move in all possible directions in (b) A – (p), B – (r), C – (q)
straight lines. During their random motion, they collide
with each other and with the walls of the container. (c) A – (r), B – (q), C – (p)
(iv) In kinetic theory it is assumed that average kinetic (d) A – (q), B – (r), C – (p)
energy of the gas molecules is directly proportional to 109. Match the columns
the absolute temperature. Column-I Column-II
(a) TTTT (b) TTTF
(A) Volume of a fixed mass (p) Boyle's Law
(c) TFTT (d) TFFT
of a gas at constant
106. Which of the following statements are correct ?
(i) Real gases show deviations from ideal gas law because pressure is directly
molecules interact with each other. proportional to its
(ii) Due to interaction of molecules the pressure exerted absolute temperature
by the gas is given as :
(B) At constant volume, (q) Avogadro's Law
an 2 pressure of a fixed amount
p real = p ideal of a gas varies directly with
V2 the temperature.
(iii) Value of 'a' is measure of magnitude of intermolecular (C) Equal volumes of all (r) Charle's Law
attractive forces within the gas and depends on gases under the same
temperature and pressure of gas. conditions of temperature
(iv) At high pressure volume occupied by the molecules and pressure contain equal
also becomes significant because instead of moving number of molecules.
in volume V, these are now restricted to volume (V-nb)
(D) At constant temperature, (s) Gay Lussac's Law
(a) (i) and (iv) (b) (i), (ii) and (iii)
the pressure of a fixed amount
(c) (i), (iii) and (iv) (d) (i) and (iii)
(i. e., number of moles n) of
107. Choose the correct sequence of true and false for following
gas varies inversely with its
statements. Here T represents true and F represents false
volume.
statement.
(i) Greater the viscosity, the more slowly the liquid flows. (a) A – (s), B – (r), C – (q), D – (p)
(ii) Glass is an extremely viscous liquid. (b) A – (r), B – (s), C – (q.), D – (p)
(iii) Viscosity of liquid increases as the temperature rise. (c) A – (r), B – (q), C – (p), D – (s)
(a) TFF (b) FFT (d) A – (q), B – (p), C – (s), D – (r)
(c) TFT (d) TTF
STATES OF MATTER 85
110. Match the columns (a) A – (r), B – (p), C – (q)
Column-I Column-II (b) A – (r), B – (q), C – (p)
(A) Boyle's law (p) V n at constant T
(c) A – (q), B – (r), C – (p)
and P
(B) Charle's law (q) ptotal = p1+ p2+ p3+ .... (d) A – (p), B – (r), C – (q)
at constant T, V
ASSERTION-REASON TYPE QUESTIONS
pV
(C) Dalton's law (r) = Constant
T Directions : Each of these questions contain two statements,
(D) Avogadro law (s) V T at constant n Assertion and Reason. Each of these questions also has four
and p alternative choices, only one of which is the correct answer. You
have to select one of the codes (a), (b), (c) and (d) given below.
1
(t) pµ at constant n (a) Assertion is correct, reason is correct; reason is a correct
V
explanation for assertion.
and T
(a) A – (t), B – (s), C – (q), D – (p) (b) Assertion is correct, reason is correct; reason is not a
(b) A – (s), B – (q), C – (p), D – (t) correct explanation for assertion
(c) A – (r), B – (t), C – (q), D – (p) (c) Assertion is correct, reason is incorrect
(d) A – (t), B – (q), C – (s), D – (r) (d) Assertion is incorrect, reason is correct.
111. Match the graphs between the following variables (Column-I) 113. Assertion : Three states of matter are the result of balance
with their names (Column-II) : between intermolecular forces and thermal energy of the
Column-I Column-II molecules.
(Graphs) (Names)
Reason : Intermolecular forces tend to keep the molecules
(A) Pressure vs temperature (p) Isotherms
together but thermal energy of molecules tends to keep
graph at constant
them apart.
molar volume.
(B) Pressure vs volume (q) Constant temperature 114. Assertion : Gases expand and occupy all the space available
graph at constant curve to them
temperature Reason : There is no force of attraction between the particles
(C) Volume vs temperature (r) Isochores of a gas at ordinary temperature and pressure.
graph at constant 115. Assertion : Gases do not liquefy above their critical
pressure temperature, even on applying high pressure.
(s) Isobars
Reason : Above critical temperature, the molecular speed is
(a) A – (p), B – (r), C – (s)
high and intermolecular attractions cannot hold the
(b) A – (r), B – (p), C – (s)
molecules together because they escape because of high
(c) A – (r), B – (q), C – (p)
speed.
(d) A – (s), B – (q), C – (r)
112. Match the following graphs of ideal gas (Column-I) with 116. Assertion : At critical temperature liquid passes into
their co-ordinates (Column-II) : gaseous state imperceptibly and continuously.
Column-I Column-II Reason : The density of liquid and gaseous phase is equal
(Graphical (x and y to critical temperature.
representation) co-ordinates) 117. Assertion : The temperature at which vapour pressure of a
liquid is equal to the external pressure is called boiling
temperature.
(A) (p) pV vs. V Reason : At high altitude atmospheric pressure is high.
118. Assertion : Liquids tend to have maximum number of
molecules at their surface.
Reason : Small liquid drops have spherical shape.
(B) (q) p vs. V CRITICAL THINKING TYPE QUESTIONS
119. Arrange the following in increasing order their
intermolecular interaction
1 (A) HCl (B) SF6 and (C) NaCl
(C) (r) p vs. (a) A, B, C (b) A, C, B
V
(c) B, A, C (d) B, C, A
EBD_7207
86 STATES OF MATTER

120. When a sample of gas is compressed at constant temperature 124. From the given figure what can be said about the gases
from 15 atm to 60 atm, its volume changes from 76 cm3 to does not deviate much from ideal gases at
20.5 cm3. Which of the following statements are possible
explanations of this behaviour? Real

(1) The gas behaves non-ideally Ideal

(2) The gas dimerises

Pressure
(3) The gas is adsorbed into the vessel walls
(a) 1, 2 and 3 (b) 1 and 2 only
(c) 2 and 3 only (d) 1 only
Volume
121. Three different gases X, Y and Z of molecular masses 2, 16
(a) Higher pressure and low volume.
and 64 were enclosed in a vessel at constant temperature till
(b) Low pressure and low volume.
equilibrium is reached. Which of the following statement is
(c) High pressure and high volume.
correct?
(d) Low pressure and high volume.
(a) Gas Z will be at the top of the vessel 125. 16 g of oxygen and 3 g of hydrogen are mixed and kept at
(b) Gas Y will be at the top of the vessel 760 mm of Hg pressure and 0° C. The total volume occupied
by the mixture will be nearly
(c) Gas Z will be at the bottom and X will be at the top (a) 22.4 litres (b) 33.6 litres
(d) Gases will form homogenous mixture (c) 448 litres (d) 44800 ml
122. Which of the following volume (V) - temperature (T) plots 126. The density of neon will be highest at
represents the behaviour of one mole of an ideal gas at one (a) S.T.P. (b) 0ºC, 2 atm
atmospheric pressure ? (c) 273ºC, 1 atm. (d) 273ºC, 2 atm.
127. A plot of volume (V) versus temperature (T) for a gas at
constant pressure is a straight line passing through the
V(L) V(L)
(36.8 L (26.8 L origin. The plots at different values of pressure are shown
(22.4 L 373 K) (22.4 L 373 K) in figure. Which of the following order pressure is correct
(a) 273K) (b) 273K) for this gas ?

T(K) T(K)

p1
p2
Volume (mL)

V(L)
V(L)
(30.6 L p3
373 K) (22.4 L
(22.4 L 273K)
(c) (d) (14.2 L p4
273K) 373 K)

T(K) T(K)

Temperature (K)
123. Consider the case of hot air balloon, density of air at 20°
C is 1.2Kg/m3, if the air was heated to 99°C, density of (a) p1 > p2 > p3 > p4 (b) p1 = p2 = p3 = p4
air becomes 0.94kg/m3. What would be the volume (in m3) (c) p1 < p2 < p3 < p4 (d) p1 < p2 = p3 < p4
at 20°C if the volume at 99°C is 2800 m3 and how much 128. At constant temperature, for a given mass of an ideal gas
air (in kg) has been escaped at 99°C , if the air in inflated (a) The ratio of pressure and volume always remains
balloon was heated to 99°C (if the inflated volume of constant.
balloon was found to be 2800m3) respectively are (b) Volume always remains constant.
(c) Pressure always remains constant.
(a) 2243, 728 (b) 3495.3, 596
(d) The product of pressure and volume always remains
(c) 2687, 593 (d) 2956, 771 constant.
STATES OF MATTER 87

129. If 500 ml of gas A at 400 torr and 666.6 ml of B at 600 torr are 136. 0.5 mole of each H2, SO2 and CH\4 are kept in a container. A
placed in a 3 litre flask, the pressure of the system will be hole was made in the container. After 3 h, the order of partial
(a) 200 torr (b) 100 torr pressures in the container will be
(c) 550 torr (d) 366 torr (a) pSO 2 pCH 4 pH 2 (b) pH 2 pSO 2 pCH 4
130. What is the partial pressure (mmHg) of nitrogen if total
atmospheric pressure is 760mmHg ? (c) pH 2 pCH 4 pSO 2 (d) pSO 2 pH 2 pCH 4
(a) 159 (b) 300 137. For a person travelling to the peak of the mountain which
(c) 592.8 (d) 230 of the following statement(s) is/are correct ?
131. Cyclopropane and oxygen at partial pressures 170 torr and (i) Oxygen level goes on decreasing.
570 torr respectively are mixed in a gas cylinder. What is the (ii) Gas law can be applied to this situation.
ratio of the number of moles of cyclopropane to the number
(a) Both (i) and (ii)
of moles of oxygen (nC3H6/nO2)?
(b) Only (i)
170 42 170 170 570 (c) Only (ii)
(a) 0.39 (b) 0.19
570 32 42 42 32 (d) Neither (i) nor (ii)
138. Pressure in well inflated tyres of automobiles is almost
170 170
(c) 0.23 (d) 0.30 constant, but on a hot summer day this increases
740 570
considerably and tyre may burst if pressure is not adjusted
132. An evacuated glass vessel weights 50 g when empty, properly. During winters, on a cold morning one may find
144.0 g when filled with a liquid of density 0.47 g ml–1 and the pressure in the tyres of a vehicle decreased considerably.
50.5 g when filled with an ideal gas at 760 mm Hg at 300 Which of the following law explain the above observations?
K. The molar mass of the ideal gas is
(a) Charle's Law (b) Avogadro Law
(Given R = 0.0821 L atm K–1 mol–1)
(c) Boyle's Law (d) Gay Lussac's Law
(a) 61.575 (b) 130.98
139. What is the ratio of pressure of the 2gm of hydrogen to
(c) 123.75 (d) 47.87 that of 4 gm of helium at temperature of 298K, 20ml
133. The pressure of a 1:4 mixture of dihydrogen and dioxygen volume? (consider the ideal behaviour)
enclosed in a vessel is one atmosphere. What would be the (a) 1 : 2 (b) 2 : 1
partial pressure of dioxygen ?
(c) 1 : 1 (d) 2 : 2
(a) 0.8 × 105 atm (b) 0.008 Nm–2
140. In case of CO and CH4 curve goes to minima then
(c) 8 × 104 Nm–1 (d) 0.25 atm increases with increase in pressure but in case of H2
134. Two vessels containing gases A and B are interconected as and He the curve is linear because:
shown in the figure. The stopper is opened, the gases are
allowed to mix homogeneously. The partial pressures of A CO CH4
and B in the mixture will be, respectively H2

Gas A Gas B He

12 L 8L
8 atm 5 atm PV Ideal gas

(a) 8 and 5 atom (b) 9.6 and 4 atm


(c) 4.8 and 2 atm (d) 6.4 and 4 atm
P
135. A neon-dioxygen mixture contains 70.6 g O2 and 167.5 g
neon. If pressure of the mixture of gases in the cylinder is 25 (a) Intermolecular interactions for H2 and He are very
bar. What is the partial pressure of O2 and Ne in the mixture low.
respectively ?
(b) Molecular size or atomic size for H2 and He is small.
(a) 5.25 bar, 10 bar (b) 19.75 bar, 5.25 bar
(c) Both (a) and (b)
(c) 19.75 bar, 10 bar (d) 5.75 bar, 19.75 bar
(d) Neither (a) nor (b)
EBD_7207
88 STATES OF MATTER

141. Which among the following has lowest surface tension ? 143. Consider the case of honey flowing over a slope for this
(a) Hexane (b) Water situation which of the following statement(s) is/are
(c) CH3OH (d) CH3CH2OH correct ?
142. Water droplets was not able to maintain its spherical
shape in the presence of gravity but mercury does, why ?
(a) Force of attraction between atoms of mercury is very
high than that of molecules in case of water.
(b) Surface tension of mercury is very high.
(c) Both (a) and (b)
(d) Neither (a) nor (b) (B)
(A)

(i) Velocity with which honey is flowing is slower in A


than B ( being same in both cases)
(ii) Velocity increases with increase in temperature.
(a) (i) and (ii) (b) Only (i)
(c) Only (ii) (d) Neither (i) nor (ii)
STATES OF MATTER 89

FACT / DEFINITION TYPE QUESTIONS 19. (c)


P P
1. (c) Attractive forces between an ion and a dipole are
known as ion - dipole forces and these are not van der
Waals forces. V 1/V
2. (d) Fritz London explained the concept of dispersion force. Curve I Curve III

1 Both these graphs represents Boyle's law.


3. (b) Interaction energy 20. (b) According to Boyle's Law
r6
pv = k
4. (c)
5. (c) London forces are always attractive and interaction m
p k
energy is inversely proportional to the sixth power of d
the distance between two interacting particles.
p k
6. (c) k'
d m
7. (a) Dipole-dipole interaction is stronger than the London
forces but is weaker than ion-ion interaction because 21. (b) Boyle’s law
only partial charges are involved e.g., HCl molecules. 1
The attractive force decreases with the increase of P
V
distance between the dipoles.
K
8. (d) Induced dipole moment depends upon the dipole P
V
moment present in the permanent dipole and the
polarisability of the electrically neutral molecule. PV K
Molecules of large size can be easily polarized. High 22. (a) Given initial volume (V1) = 600 c.c.; Initial pressure
polarisability increases the strength of attractive (P1) = 750 mm of Hg and final volume (V2) = 500 c.c.
interactions. according to Boyle’s law,
9. (b) This type of attractive force operates between the polar P1V1 = P2V2
molecules having permanent dipole and the molecules
lacking permanent dipole. or 750 × 600 = P2 × 500
HCl is polar ( 0) and He is non polar ( = 0), thus
gives dipole-induced dipole interaction. 750 600
or P2 900 mm of Hg
10. (c) Nobel gases has no intermolecular forces due to 500
inertness. Therefore increase of pressure = (900 – 750) = 150 mm
of Hg
11. (a) Strength of the hydrogen bond is determined by
coulombic interaction between lone pair electrons of 23. (b) The lowest hypothetical or imaginary temperature at
the electronegative atom of one molecule and the which gases are supposed to occupy zero volume is
hydrogen atom of the other molecule. called absolute zero.
12. (d) 13. (a) 24. (b) Given initial volume (V1) = 500 ml ; Initial temperature
(T1) = 27ºC = 300 K and final temperature (T2) = –5ºC
14. (c) Gaseous state of substance has the maximum thermal
= 268 K.
energy.
From Charle’s law :
15. (c) Gases do not have definite shape and volume. Their
volume is equal to the volume of the container. V1 V2 500 V2
or
16. (b) It is characteristic of gases i.e., Thermal energy >> T1 T2 300 268
molecular attraction.
Where V2 = New volume of gas
17. (c) Robert Boyle made first reliable measurement on
properties of gases. 500
V2 268 446.66ml.
18. (d) According to Boyle’s law at constant temperature, 300
1 25. (c) At any given pressure, graph of volume vs temperature
V or PV = constant (in °C) is a straight line and on extending to zero volume
P
each line intercepts the temperature axis at – 273.15° C.
EBD_7207
90 STATES OF MATTER

V PV m
26. (c) According to Charle’s law V T or k 44. (c) n
T RT M
27. (b) Charle’s law V T at constant P.. MPV 34 2 100
m 282.68gm
MP RT 0.082 293
28. (a) d p, Boyle’s law, d . At sea level pressure is PV T 760 546
RT 45. (c) V2
1 1 1 273 691.6 ml
more, hence density of air is more. T1 P2 600 273
29. (b) Hot air is lighter due to less density (Charle’s law) 46. (a) Total moles
4 2
MP = 1.125; PV nRT
d 32 2
RT
P 1.125 .0821 273
MP P = 25.215 atm
30. (b) Higher P, lower T, greater the density. d
RT 47. (c) PV = nRT is for an ideal gas which follows both
31. (a) Order of pressure, p1 < p2 < p3 < p4. isothermal and adiabatic processes.
32. (a) V1 = 2 L, T2 = (26.1 + 273) K = 299.1 K, V2 = ? 48. (a) By Dalton’s law of partial pressures, the total pressure
T1 = (23.4 + 273) K = 296.4 K of a mixture of two gases is the sum of the partial
pressures of components of the mixture.
V1 V2 V1T2 49. (a)
From Charle's law, V2
T1 T2 T1 50. (b) Because H2 & Cl2 gases may react with each other to
produce HCl gas hence Dalton’s law is not applicable.
2L 299.1K 51. (a) Pressure exerted by saturated water vapour is called
V2 2L 1.009
296.4K aqueous tension.
= 2.018 L nRT 6 8.314 402
52. (d) P = 41648 Pa
P V 16.02 0.03
33. (d) constant (Gay Lussac's law)
T
53. (a) Given nCO = n N2
P1 P2
P1T2 P2T1
T1 T2 PCO + PN2 = 1 atm
PV = constant Partial pressure of a gas = mole fraction of gas × total
P1V1 = P2V2 [Boyle's law] pressure
34. (c) This relationship is derived from Boyle's and Charles' n N2
n N2
law. PN2 = 1 = 2n 1= 1 0.5 atm.
35. (d) All of the given relations are true for Avogadro's law. n CO n N2 N2 2
36. (a) At STP molar volume of an ideal gas or a combination 54. (a) Given
of ideal gases is 22.71098 L mol–1. P1 = 1.5 bar T1 = 273 + 15 = 288 K V1 = V
37. (d) 44g at STP occupies volume 22.4 litre which is molecular P2 = 1.0 bar T1 = 273 + 25 = 298K V2 = ?
mass of CO2. Molecular mass occupies 22.4 litre at P1V1 P2 V2
STP. T1 T2
38. (b) An ideal gas obeys the gas laws under all experimental
1.5 V 1 V2
conditions.
288 298
39. (c) PV = nRT
V2 = 1.55 V i.e., volume of bubble will be almost 1.6 time
n/V = P/RT
T. to initial volume of bubble.
40. (c) In the equation PV = nRT, n moles of the gas have 55. (a) Partial pressure = total pressure × mole fraction
volume V.
64
PV PV P2V2 Moles of oxygen = 2
41. (b) constant or 1 1 32
T T1 T2
60
PV
1 1 T1 Moles of neon = 3
20
P2V2 T2
2 2
42. (b) R = 0.0082 litre atm K–1 mole–1 . Mole fraction of oxygen =
2 3 5
43. (d) PV = nRT = m RT
M 2
PO 2 10 4 bar
m PM 5
or PM = RT = dRT d=
V RT
STATES OF MATTER 91

3 3 64. (c) Given statement explain the great compressibility of


Mole fraction of neon = gases.
2 3 5
65. (b) If there were loss of kinetic energy, the motion of
3 molecule will stop and gases will settle down.
PNe 10 6 bar
5 66. (b) Particles of a gas are always in constant and random
56. (b) P1V1 = P2V2 motion. If the particles were at rest and occupy fixed
760 × 500 = P2 × 200. positions, then a gas would have a fixed shape which
760 500 is not observed.
P2 = 1900mm Hg 67. (b) If assumption (ii) is correct, the pressure vs volume
200
graph of experimental data (real gas) and that of
57. (d) Value of gas constant depends only upon units of
theoretically calculated for Boyle's law (ideal gas)
measurement.
should coincide.
58. (a) Given conditions
68. (d) Kinetic theory of gases proves all the given gas laws.
V1 = 16.4 L, V2 = 5 L
69. (d) Molecules move very fast in all directions in a straight
P1 = 1.5 atm, P2 = 4.1 atm
line by colliding with each other but with different
T1 = 273 + 27 = 300 K, velocity.
T2 = 273 + 227 = 500 K 70. (b) At low temperature and high pressure.
P1V1 n1T1 71. (a) In van der waals equation ‘b’ is for volume correction
Applying gas equation, P V n 2 T2
2 2
a a
72. (c) P (V – b) = RT
T; Here P represents
n1 P1V1T1 V2 V2
n2 P2 V2 T2 the intermolecular forces.
1.5 16.4 500 2 73. (c) ‘a’ is directly related to forces of attraction. Hence
4.1 5 300 1 greater the value of ‘a’, more easily the gas gets
liquified.
59. (d) On applying Dalton's law,
74. (b)
Partial pressure of a component
(i) At very large molar volume
= Mole fraction × Total pressure
Given, mass of N2 = 56 g, mass of O2 = 96 g a
P P and Vm b Vm
Total pressure = 10 atm Vm2
n 56 96 (iii) According to van der Waals equation 'a' and 'b' are
N2 2, n O2 3
28 32 independent of temperature.
n
N2 2 n2a PV 2
x
N2 0.4, 75. (a) P ; a atm dm 6 mol-2
n
N2 n
O2 2 3 V2 n2
n
76. (c) Easily liquefiable gases have greater intermolecular
x O2 3 forces which is represented by high value of 'a'. The
O2 n n
0.6
N2 O2 2 3 greater the value of 'a' more will be liquefiability.
PN = 0.4 × 10 = 4 atm, PO = 0.6 × 10 = 6 atm
2 2
So, the order is Q < S < P < R.
60. (d) From the ideal gas equation : 77. (a) When pressure is low ‘b’ can be neglected, thus
PV nRT a
P V RT
PV 3170 10 3 V2
or n = 1.27 × 10–3
RT 8.314 300
a
61. (a) By Dalton’s law of partial pressures, the total pressure PV RT
of a mixture of two gases is the sum of the partial V
pressures. a
62. (b) An ideal gas equation is the combination of Boyle's PV RT
V
law, Charles' law and Avogadro law.
63. (b) According to kinetic theory the gas molecules are in a PV RT a
state of constant rapid motion in all possible directions RT RT VRT
colloiding in a random manner with one another and
PV a
with the walls of the container and between two Z 1
successive collisions molecules travel in a straight line RT VRT
path but show haphazard motion due to collisions.
EBD_7207
92 STATES OF MATTER

a a 91. (a) 92. (b)


78. (c) P (V b) RT at high pressure can be 93. (d) All these phenomena are caused by surface tension.
V2 V2
neglected 94. (b) 95. (b)
PV – Pb = RT and PV = RT + Pb 96. (c) Due to intermolecular H-bonding the surface tension
of H2O is more than other liquid. One H2O molecule is
PV Pb
1 joined with 4 another H2O molecule through H–bond.
RT RT
Hydrogen bonding is in order H2O > C2H5OH >
Pb CH3OH.
Z 1 ; Z > 1 at high pressure
RT
97. (a) Boiling point of water is 100°C whereas evaporation of
79. (d) CO2 has highest critical temperature of 304.2 K water into water vapours occurs at room temperature.
80. (d) At low pressure and high temperature: At low pressure
98. (a) As intermolecular forces are least in case of petrol.
volume correction for 1 mole of a gas in negligible, i.e b
=0 Thus, it has highest rate of evaporation.
thus the gas equation becomes 99. (c) The correct order of viscosity of the given liquids is
dimethyl ether < methyl alcohol < water < glycerol.
æ ö
çç P + a ÷÷ V = RT
çè 2 ø÷÷ STATEMENT TYPE QUESTIONS
V

PVm a 100. (d) These forces are important only at short distances
or Z = = 1- (~ 500 pm)
RT Vm RT
101. (b) For statement (ii), partial charge possessed by these
At higher pressure, the pressure correction for 1 mole
dipoles is always less than the unit electronic charge.
a For statement (iii), dipole - dipole interaction is stronger
of gas in negligible i.e =0
V2 than London forces but weaker than ion - ion
or (P + 0) (V – b) = RT interaction.
or P (Vm – b) = RT 102. (d) All of the given statements are correct for hydrogen
or PVm = RT + Pb bond.
PVm Pb 103. (c) Gases have much lower density than the solids and
or Z = = 1+ liquids.
RT RT
104. (b) Value of R = 8.314 Pa m3 K–1 mol–1
81. (d) In the ideal gas, the intermolecular forces of attraction
are negligible and hence it cannot be liquefied. 105. (a) All the given statements are true.
82. (d)
83. (d) Above Boyle point, real gases show positive deviation an 2
106. (a) For statement (ii), preal = p ideal
from ideality and Z values are greater than one. V2
84. (c) For statement (iii), value of 'a' is independent of
85. (c) More will be critical temperature easier is the temperature and pressure.
liquifaction of the gas. Hence correct order will be
107. (d) Viscosity of liquid decreases as temperature rise.
He < H2 < N2 < O2
86. (a) For statement (iii), we can move down from G towards MATCHING TYPE QUESTIONS
D by lowering the temperature.
For statement (iv), we get liquid as soon as we cross 108. (a) 109. (b) 110. (a) 111. (b) 112. (c)
point H.
87. (b) At 1 atm pressure boiling temperature is called normal ASSERTION-REASON TYPE QUESTIONS
boiling point. If pressure is 1 bar than the boiling point
is called standard boiling point of the liquid. 113. (a)
88. (b) 1 bar pressure is slightly less than 1atm pressure. 114. (a) Gases expand and occupy all the space available to
89. (d) A substance exists as a liquid above its m. pt. and them because there is no force of attraction between
below its b. pt. the particles of a gas at ordinary temperature and
90. (a) The kinetic energy of molecules in gaseous state is pressure.
more than those in the liquid state as the molecules in 115. (a) 116. (a)
gaseous state can move freely (with higher speed) as 117. (c) At high altitude atmospheric pressure is low.
compared in liquid state. 118. (d)
STATES OF MATTER 93
It means density of gas is directly proportional to
CRITICAL THINKING TYPE QUESTIONS
pressure and inversely proportional to temperature.
119. (c) In case of HCl molecules their is dipole-dipole Density of neon will be maximum at highest pressure
interaction which is stronger than London forces as and lowest temperature.
in case of SF6. Now between HCl and NaCl the ion- 127. (c)
ion interaction present in NaCl is far more stronger 128. (d) According to Boyle’s law at constant temperature,
than dipole-dipole interaction of HCl.
1
120. (d) Given, P1 = 15 atm, P2 = 60 atm V or PV = constant
V1 = 76 cm3, V2 = 20.5 cm3. P
If the gas is an ideal gas, then according to Boyle's law, 129. (a) Applying Boyle's law P1V1 = P2V2 for both gases
it must follow the equation, 500 200
P1V1 = P2V2 400 P 3 P
1000 3
P1 × V1 = 15 × 76 = 1140
P2 × V2 = 60 × 20.5 = 1230 666.6 400
600 P 3 P
1000 3
P1V1 P2 V2
The gas behaves non-ideally. 200 400 600
PT P P 200 tor
The given information is not sufficient to comment on 3 3 3
other statements. 130. (c) Percentage of nitrogen in atmosphere is 78% .Partial
121. (d) All the gases occupy the available volume and will pressure of N2 = 0. 78 × 760
form homogeneous mixture. 131. (d) By Ideal gas equation
V1 V2 P1V n1RT
122. (c) at const. pressure
T1 T2
n1 P1 and n 2 P2
22.4 V2
, V2 30.6 litre n1 P1 n1 170
273 373 0.30
n2 P2 n2 570
123. (a) Since atmospheric pressure remain constant
132. (a) Given weight of empty glass vessel = 50 g
V1 V2 V1 2800m 3 Weight of vessel filled with liquid = 144 g
T1 T2 298K 372K Weight of liquid = 144 – 50 = 94 g.
V1 = 2243 m3 Volume of liquid = Mass/density = 94/0.47
2800 m3 volume of inflated balloon. = 200 ml = 200 × 10–3 L.
Mass of air in inflated ballon = 2800 m3 × 0.94 kg m–3 Given, pressure of ideal gas = 760 mm Hg = 1 atm
= 2632 kg Temperature = 300 K
Keeping the volume same = 2800 m3 R = 0.0821 L atm K–1 mol–1
The mass of air, which occupies it with density Mass of ideal gas = 50.5 – 50 = 0.5 g
(1.2 kg/m3) is 2800 × 1.2 = 3360 kg According to ideal gas equation,
Amount of air which had been escaped = 3360 – 2632 w
= 728 kg PV = nRT = RT
M
124. (d)
0.5
16 1 1 × 200 × 10–3 0.0821 300
125. (d) n of O 2 M
32 2
0.5 0.0821 300
3 M 61.575
n of H2 200 10–3
2
133. (d) p1 = 1.5 atm, T1 = 15°C = (15 + 273)K = 288 K
3 1
Total no. of moles 2 p2 = 1 atm, T2 = 25°C = (25 + 273)K = 298 K
2 2
nRT 2 .082 273 p1V1 p 2 V2
V 44.8lit 44800 ml T1 T2
P 1
PM p1T2 V2 V2 1.5 298
126. (b) d = 1.55
RT T1p2 V1 V1 288 1
EBD_7207
94 STATES OF MATTER

p A vA 8 12 96 136. (a) Extent of diffusion H2 > CH4 > SO2 because rate of
134. (c) Moles of A, (n A)
RT RT RT 1
diffusion
molar mass
pB v B 8 5 40
Moles of B, (n B) Order of partial pressure after diffusion is
RT RT RT
Total pressure × total volume = (nA + nB) × RT pSO 2 pCH 4 pH 2

1 137. (a) As the height increases, atmospheric pressure


p (12 8) (96 40)RT decreases, so now the volume of the gas increases
RT
and gas tends to become less denser, hence the
p = 6.8
concentration of oxygen decreases.
Partial pressure of A = p × mole fraction of A
138. (d) The mathematical relationship between pressure and
96 96 40 temperature was given by Gay Lussac's law.
6.8
RT RT 139. (c) Number of moles, temperature and volume are same.
= 4.8 atm 140. (c) Due to small size of these species (H2 and He)
Partial pressure of B = 6.8 – 4.8 = 2 atm. intermolecular interactions (van der Waal forces) are
very low, therefore it is difficult to compress these .
70.6g
135. (d) Number of moles of O2 2.21 mol 141. (a) Since surface tension depends on the attractive
1
32g mol forces between the molecules, and hydrogen bonding
a special type of dipole-dipole interactions in (b), (c)
167.5g and (d) which is stronger than London forces of
Number of moles of Ne 8.375 mol
1
20g mol attraction in hexane.
142. (c)
2.21
Mole fraction of O2 0.21 143. (a) Force is required to maintain the flow of layer which
2.21 8.375
is inversely proportional to the area of contact of
Mole fraction of Ne = 1 – 0.21 = 0.79 layer therefore flow in B is greater than that in A as
Partial pressure of a gas = Mole fraction × total pressure the area of contact is greater in A. Also viscosity of
Partial pressure of O2 = 0.21 × 25 = 5.25 bar the fluid decreases with increase in temperature
Partial pressure of Ne = 0.79 × 25 = 19.75 bar therefore liquid flow increases.
6
THERMODYNAMICS

FACT / DEFINITION TYPE QUESTIONS 7. An isolated system is that system in which


(a) There is no exchange of energy with the surroundings
1. Thermodynamics is not concerned about____. (b) There is exchange of mass and energy with the
(a) energy changes involved in a chemical reaction. surroundings
(b) the extent to which a chemical reaction proceeds. (c) There is no exchange of mass or energy with the
(c) the rate at which a reaction proceeds surroundings
(d) the feasibility of a chemical reaction. (d) There is exchange of mass with the surroundings
2. Which of the following statements is not true regarding the 8. The state of a thermodynamic system is described by its
laws of thermodynamics ? measurable or macroscopic (bulk) properties. These are
(a) It deal with energy changes of macroscopic systems. (a) Pressure and volume
(b) It deal with energy changes of microscopic systems. (b) Pressure, volume, temperature and amount
(c) It does not depends on the rate at which these energy (c) Volume, temperature and amount
transformations are carried out. (d) Pressure and temperature
(d) It depends on initial and final states of a system 9. Which of the following are not state functions ?
undergoing the change. (I) q + w (II) q
3. A……………... in thermodynamics refers to that part of (III) w (IV) H - TS
universe in which observations are made and remaining (a) (I) and (IV) (b) (II), (III) and (IV)
universe constitutes the……………... (c) (I), (II) and (III) (d) (II) and (III)
(a) surroundings, system (b) system, surroundings 10. Among the following the state function(s) is (are)
(c) system, surroundings (d) system, boundary (i) Internal energy
4. The universe refers to
(ii) Irreversible expansion work
(a) only system
(iii) Reversible expansion work
(b) only surroundings
(iv) Molar enthalpy
(c) both system and surroundings
(a) (ii) and (iii) (b) (i), (ii) and (iii)
(d) None of these
(c) (i) and (iv) (d) (i) only
5. Which of the following statements is correct?
11. Enthalpy change ( H) of a system depends upon its
(a) The presence of reacting species in a covered beaker
(a) Initial state
is an example of open system.
(b) Final state
(b) There is an exchange of energy as well as matter
between the system and the surroundings in a closed (c) Both on initial and final state
system. (d) None of these
(c) The presence of reactants in a closed vessel made up 12. ………………. is a quantity which represents the total energy
of copper is an example of a closed system. of the system
(d) The presence of reactants in a thermos flask or any (a) Internal energy (b) Chemical energy
other closed insulated vessel is an example of a closed (c) Electrical energy (d) Mechanical energy
system. 13. Which of the following factors affect the internal energy of
6. Which of the following is closed system ? the system ?
(a) Jet engine (a) Heat passes into or out of the system.
(b) Tea placed in a steel kettle (b) Work is done on or by the system.
(c) Pressure cooker (c) Matter enters or leaves the system.
(d) Rocket engine during propulsion (d) All of the above
EBD_7207
96 THERMODYNAMICS
14. The system that would not allow exchange of heat between 20. When 1 mol of a gas is heated at constant volume,
the system and surroundings through its boundary is temperature is raised from 298 to 308 K. If heat supplied to
considered as the gas is 500 J, then which statement is correct ?
(a) isothermal (b) adiabatic (a) q = w = 500 J, U = 0 (b) q = U = 500 J, w = 0
(c) isobaric (d) isochoric (c) q = –w = 500 J, U = 0 (d) U = 0, q = w = –500 J
15. The enthalpy change of a reaction does not depend on 21. The work done during the expansion of a gas from a volume
(a) The state of reactants and products of 4 dm3 to 6 dm3 against a constant external pressure of 3
(b) Nature of reactants and products atm is (1 L atm = 101.32 J)
(c) Different intermediate reactions (a) – 6 J (b) – 608 J
(d) Initial and final enthalpy change of a reaction. (c) + 304 J (d) – 304 J
16. The q is ……………. when heat is transferred from the 22. Which of the following statements/relationships is not
surroundings to the system and q is……………………….. correct in thermodynamic changes ?
When heat is transferred from system to the surroundings. (a) U = (isothermal reversible expansion of a gas)
(a) positive , negative (b) negative , positive V2
(b) w = – nRT ln (isothermal reversible expansion of
(c) high, low (d) low, high V1
17. Adiabatic expansions of an ideal gas is accompanied by an ideal gas)
(a) decrease in E V2
(b) increase in temperature (c) w = nRT ln (isothermal reversible expansion of an
V1
(c) decrease in S
ideal gas)
(d) no change in any one of the above properties
(d) For a system of constant volume heat involved directly
18. Which of the following statements is incorrect?
changes to internal energy.
(a) q is a path dependent function. 23. An ideal gas expands in volume from 1×10–3 to 1 × 10–2 m3
(b) H is a state function. at 300 K against a constant pressure of 1×105 Nm–2. The
(c) Both H and q are state functions. work done is
(d) Both (a) and (b) (a) 270 kJ (b) – 900 kJ
19. Figure below is showing that one mole of an ideal gas is (c) – 900 J (d) 900 kJ
fitted with a frictionless piston. Total volume of the gas is 24. The difference between H and U is usually significant
Vi and pressure of the gas inside is p. If external pressure is for systems consisting of
pex which is greater than p is applied, piston is moved inward (a) only solids (b) only liquids
till the pressure inside becomes equal to pex.
(c) both solids and liquids (d) only gases
25. If a reaction involves only solids and liquids which of the
following is true ?
pex (a) H< E (b) H = E
(c) H> E (d) H = E + RT n
Pressure, p

26. During isothermal expansion of an ideal gas, its


Area = pex V (a) internal energy increases
(b) enthalpy decreases
(c) enthalpy remains unaffected
(d) enthalpy reduces to zero.
Vf Vi Volume, V 27. Assume each reaction is carried out in an open container.
pex For which reaction will H = E ?
(a) C(s) + 2H2O (g) 2H2 (g) + CO2 (g)
(b) PCl5 (g) PCl3 (g) + Cl2 (g)
(c) 2CO (g) + O2 (g) 2CO2 (g)
(d) H2 (g) + Br 2 (g) 2 HBr (g)
pex
1
28. For the reaction CO (g ) O 2 (g ) CO 2 (g )
2
l Which one of the statement is correct at constant T and P ?
(a) H E
What does the shaded area represents in the figure ? (b) H E
(a) Work done (b) Pressure change (c) H E
(c) Volume change (d) Temperature change
(d) H is independent of physical state of the reactants
THERMODYNAMICS 97
29. For a reaction in which all reactants and products are liquids, 41. Which of the following is not true regarding thermo-chemical
which one of the following equations is most applicable ? equations?
(a) H< E (b) H = S (a) The coefficients in a balanced thermo-chemical
(c) H= E (d) H = G equation refer to the number of moles of reactants and
30. The relationship between enthalpy change and internal products involved in the reaction
energy change is (b) The coefficients in a balanced thermo-chemical
(a) H= E+P V (b) H ( E V P) equation refer to the number of molecules of reactants
(c) H= E–P V (d) H= P V– E and products involved in the reaction
31. For the reaction (c) The numerical value of r H refers to the number of
C3H8 (g) 5O 2 (g) 3CO2 (g) 4H 2O(l) moles of substances specified by an equation.

at constant temperature, H – E is (d) Standard enthalpy change r H will have units as


(a) – RT (b) + RT kJ mol-1.
(c) – 3 RT (d) + 3 RT 42. The enthalpies of elements in their standard states are taken
as zero. The enthalpy of formation of a compound
32. Consider the reaction : N 2 3H 2 2 NH 3 carried out at
(a) is always negative
constant temperature and pressure. If H and U are the (b) is always positive
enthalpy and internal energy changes for the reaction, which (c) may be positive or negative
of the following expressions is true ? (d) is never negative
(a) H U (b) H U 43. If enthalpies of formation of C 2 H 4 (g) , CO 2 (g) and
(c) H U (d) H 0 H 2 O ( l) at 25°C and 1atm pressure are 52, – 394 and
33. Among the following, the intensive properties are – 286 kJ/mol respectively, the change in ethalpy is equal to
(i) molar conductivity (ii) electromotive force (a) – 141.2 kJ/mol (b) – 1412 kJ/mol
(iii) resistance (iv) heat capacity (c) + 14.2 kJ/mol (d) + 1412 kJ/mol
(a) (ii) and (iii) (b) (i), (ii) and (iii) 44. The enthalpy change for a reaction does not depend upon
(c) (i) and (iv) (d) (i) only
(a) use of different reactants for the same product
34. Which is an extensive property of the system ?
(b) the nature of intermediate reaction steps
(a) Volume (b) Viscosity
(c) the differences in initial or final temperatures of
(c) Temperature (d) Refractive index
involved substances
35. Which of the following is an example of extensive property?
(a) Temperature (b) Density (d) the physical states of reactants and products
(c) Mass (d) Pressure 45. On the basis of thermochemical equations (i), (ii) and (iii),
36. Which of the following factors do not affect heat capacity? find out which of the algebric relationships given in options
(a) Size of system (b) Composition of system (a) to (d) is correct.
(c) Nature of system (d) Temperature of the system (i) C (graphite) + O2(g) CO2(g); rH = x kJ mol–1
37. The heat required to raise the temperature of body by 1 C° is 1
(ii) C(graphite) + O2(g) CO (g); –1
called 2 rH = y kJ mol
(a) specific heat (b) thermal capacity
(c) water equivalent (d) None of these. 1 –1
(iii) CO (g) + O (g) CO2 (g); rH = z kJ mol
38. Equal volumes of two monoatomic gases, A and B, at same 2 2
temperature and pressure are mixed. The ratio of specific (a) z = x + y (b) x = y – z
heats (Cp/Cv) of the mixture will be :
(c) x = y + z (d) y = 2z – x
(a) 0.83 (b) 1.50
46. Given that bond energies of H – H and Cl – Cl are 430 kJ mol–
(c) 3.3 (d) 1.67 1 and 240 kJ mol–1 respectively and H for HCl is – 90 kJ
39. The molar heat capacity of water at constant pressure is 75 f
mol– 1, bond enthalpy of HCl is
JK–1 mol–1. When 1kJ of heat is supplied to 100 g of water,
(a) 380 kJ mol–1 (b) 425 kJ mol–1
which is free to expand, the increase in temperature of water –1
is (c) 245 kJ mol (d) 290 kJ mol–1
(a) 6.6 K (b) 1.2 K 47. Bond dissociation enthalpy of H2, Cl2 and HCl are 434 , 242
(c) 2.4 K (d) 4.8 K and 431 kJ mol–1 respectively. Enthalpy of formation of HCl
40. Calorie is equivalent to : is:
(a) 0.4184 Joule (b) 4.184 Joule (a) 93 kJ mol–1 (b) – 245 kJmol–1
(c) – 93 kJmol –1 (d) 245 kJmol–1
(c) 41.84 Joule (d) 418.4 Joule
EBD_7207
98 THERMODYNAMICS
48. From the following bond energies: (a) Standard enthalpy of a reaction
H – H bond energy: 431.37 kJ mol–1 (b) Born – Haber cycle of lattice enthalpy
C = C bond energy: 606.10 kJ mol–1 (c) Hess’s law of constant heat summation
C – C bond energy: 336.49 kJ mol–1 (d) Standard enthalpy of a solution
C – H bond energy: 410.50 kJ mol–1 55. The enthalpy change on breaking one mole of bonds
Enthalpy for the reaction, completely to obtain atoms in the gas phase is known as
(a) enthalpy of atomization.
H H H H
| | | | (b) enthalpy of formation
C C H H H C C H (c) enthalpy of sublimation
| | | | (d) enthalpy of vaporization
H H H H 56. Which of the following statements is true for the given
will be: reaction?
(a) – 243.6 kJ mol–1 (b) –120.0 kJ mol–1 Na s Na g ; H 108.4 kJ mol –1
(c) 553.0 kJ mol –1 (d) 1523.6 kJ mol–1 (a) The enthalpy of atomization is same as the enthalpy of
49. The following two reactions are known : vaporisation
Fe2O3(s) + 3CO (g) 2Fe(s) + 3CO2(g); H = –26.8 kJ (b) The enthalpy of atomization is same as the enthalpy of
FeO(s) + CO(g) Fe(s) + CO2(g); H = –16.5 kJ sublimation.
The value of H for the following reaction (c) The enthalpy of atomization is same as the bond
enthalpy
Fe2O3(s) + CO(g) 2FeO(s) + CO2(g) is;
(d) The enthalpy of atomization is same as the enthalpy of
(a) + 6.2 kJ (b) + 10.3 kJ
solution
(c) – 43.3 kJ (d) – 10.3 kJ
57. The heat of combustion of a substance is :
50. Consider the following processes :
(a) Always positive
H (kJ/mol)
(b) Always negative
1/2 A B +150
(c) Numerically equal to the heat of formation
3B 2C + D –125
(d) Unpredictable
E + A 2D +350
For B + D E + 2C, H will be : 58. During complete combustion of one mole of butane, 2658 kJ
of heat is released. The thermochemical reaction for above
(a) 525 kJ/mol (b) – 175 kJ/mol
change is
(c) – 325 kJ/mol (d) 325 kJ/mol
(a) 2C4H10(g) + 13O2(g) 8CO2(g) + 10H2O(l)
51. The H° for CO2(g) CO(g) and H2O(g) are –393.5, – –1
f cH = –2658.0 kJ mol
110.5 and –241.8 kJ/mol respectively, the standard enthalpy 13
change (in kJ) for the reaction (b) C4H10(g) + O (g) 4CO2(g) + 5H2O (g)
2 2
CO2(g) + H2(g) CO(g) + H2O(g) is : –1
(a) 524.1 (b) 41.2 cH = –1329.0 kJ mol
(c) – 262.5 (d) – 41.2 13
(c) C4H10(g) + O (g) 4CO2(g) + 5H2O (l)
52. If enthalpies of formation of C 2 H 4 ( g ) , CO2(g) and 2 2
H 2O(l ) at 25°C and 1atm pressure are 52, – 394 and – 286 –1
cH = –2658.0 kJ mol
kJ/mol respectively, the change in enthalpy is equal to
13
(a) – 141.2 kJ/mol (b) – 1412 kJ/mol (d) C4H10(g) + O (g) 4CO2(g) +5H2O (l)
2 2
(c) + 14.2 kJ/mol (d) + 1412 kJ/mol
–1
53. Hess’s law is used to calculate : cH = + 2658.0 kJ mol
(a) enthalpy of reaction. (b) entropy of reaction 59. Given that heat of neutralisation of strong acid and strong
(c) work done in reaction (d) All of the above base is – 57.1 kJ. The heat produced when 0.25 mole of HCl
54. Which thermochemical process is shown by the following is neutralised with 0.25 mole of NaOH in aqueous solution
is :
figure ?
(a) 14.275 kJ (b) 57.1 kJ
rH (c) 22.5 kJ (d) 28.6 kJ
X Y
60. For most of the ionic compounds, Hsol is …………and
rH 1 rH3
the dissociation process is…………….
rH 2 (a) positive ,exothermic (b) negative, exothermic
P Q
(c) positive, endothermic (d) negative,endothermic
THERMODYNAMICS 99
61. Pick out the wrong statement
2Zn(s) O2 (g) 2ZnO(s) ; G = – 480 kJ
(a) The standard free energy of formation of all elements
is zero Then G for the reaction :
(b) A process accompanied by decrease in entropy is 2ZnS(s) 3O 2 (g) 2ZnO(s) 2SO 2 (g)
spontaneous under certain conditions
(c) The entropy of a perfectly crystalline substance at will be :
absolute zero is zero (a) –357 kJ (b) –731 kJ
(d) A process that leads to increase in free energy will be (c) –773 kJ (d) –229 kJ
spontaneous 69. Identify the correct statement regarding entropy.
62. Identify the correct statement for change of Gibbs energy (a) At absolute zero temperature, entropy of a perfectly
for a system ( G system) at constant temperature and crystalline substance is taken to be zero.
pressure (b) At absolute zero temperature, the entropy of a perfectly
(a) If Gsystem = 0, the system has attained equilibrium crystalline substance is positive.
(b) If Gsystem = 0, the system is still moving in a particular (c) Absolute entropy of a substance cannot be
direction determined.
(c) If Gsystem < 0, the process is not spontaneous (d) At 0°C, the entropy of a perfectly crystalline substance
(d) If Gsystem > 0, the process is not spontaneous is taken to be zero
63. Identify the correct statement regarding a spontaneous 70. Unit of entropy is
process: (a) JK–1 mol–1 (b) J mol–1
–1
(c) J K mol –1 –1 (d) JK mol–1
(a) Lowering of energy in the process is the only criterion
for spontaneity. 71. Considering entropy (S) as a thermodynamic parameter,
(b) For a spontaneous process in an isolated system, the the criterion for the spontaneity of any process is
change in entropy is positive. (a) Ssystem Ssurroundings 0
(c) Endothermic processes are never spontaneous. (b) Ssystem Ssurroundings 0
(d) Exothermic processes are always spontaneous.
(c) Ssystem 0 only
64. A chemical reaction will be spontaneous if it is accompanied
by a decrease of (d) Ssurroundings 0 only
(a) entropy of the system. 72. In an exothermic reaction (reversible) which of the following
(b) enthalpy of the system. has positive value?
(c) internal energy of the system. (a) Enthalpy (b) Entropy
(d) free energy of the system. (c) Gibb’s free energy (d) None of these
65. In which of the following entropy decreases? 73. A reaction cannot take place spontaneously at any
(a) Crystallization of sucrose solution temperature when
(b) Rusting of iron (a) both H and S are positive
(c) Melting of ice (b) both H and S are negative
(d) Vaporization of camphor (c) H is negative and S is positive
66. A spontaneous reaction is impossible if (d) H is positive and S is negative
(a) both H and S are negative 74. A reaction is spontaneous at low temperature but non-
spontaneous at high temperature. Which of the following
(b) both H and S are positive is true for the reaction?
(c) H is negative and S is positive (a) H > 0, S > 0 (b) H < 0, S > 0
(d) H is positive and S is negative (c) H > 0, S = 0 (d) H < 0, S < 0
67. For the gas phase reaction, 75. At the sublimation temperature, for the process
PCl5(g) PCl3(g) + Cl2(g) CO2(s) CO2(g)
(a) H, S and G are all positive
which of the following conditions are correct ? (b) H > 0, S > 0 and G < 0
(a) H = 0 and S < 0 (b) H > 0 and S > 0 (c) H < 0, S > 0 and G < 0
(c) H < 0 and S < 0 (d) H > 0 and S < 0 (d) H > 0, S > 0 and G = 0
68. The factor of G values is important in metallurgy. The G 76. Choose the reaction with negative S value.
values for the following reactions at 800ºC are given as :
(a) 2NaHCO3(s) Na2CO3(s) + CO2(g) + H2O(g)
S2 (s) 2O2 (g) 2SO2 (g) ; G = – 544 kJ (b) Cl2(g) 2Cl(g)
(c) 2SO2(g) + O2(g) 2SO3(g)
2Zn(s) S2 (s) 2ZnS(s) ; G = – 293 kJ (d) 2KClO3(s) 2KCl(s) + 3O2(g)
EBD_7207
100 THERMODYNAMICS
77. A chemical reaction is spontaneous at 298 K but non- (ii) Variables like P, V and T are called state variables or
spontaneous at 350 K. Which one of the following is true state functions
for the reaction? (iii) Their values depend only on the state of the system
G H S and not on how it is reached.
(a) – – + (a) (i) and (ii) are correct
(b) + + + (b) (ii) and (iii) are correct
(c) – + – (c) (i) , (ii) and (iii) are correct
(d) – – – (d) Only (iii) is correct
78. For a particular reversible reaction at temperature T, H and 85. Read the following statements carefully and choose the
S were found to be both +ve. If Te is the temperature at
correct option
equilibrium, the reaction would be spontaneous when
(i) Internal energy, U, of the system is a state function.
(a) Te > T (b) T > Te (ii) –w shows, that work is done on the system.
(c) Te is 5 times T (d) T = Te
(iii) +w shows, that work is done by the system
79. Which of the following pairs of a chemical reaction is certain
(a) (i) and (ii) are correct (b) (ii) and (iii) are correct
to result in a spontaneous reaction?
(a) Exothermic and increasing disorder (c) (i) and (iii) are correct (d) Only (i) is correct
(b) Exothermic and decreasing disorder 86. Read the following statements carefully and choose the
(c) Endothermic and increasing disorder correct answer
(d) Endothermic and decreasing disorder (i) Expansion of a gas in vacuum (pex = 0) is called free
80. In which of the following reactions, standard entropy expansion.
change ( S°) is positive and standard Gibb’s energy change (ii) Work is done during free expansion of an ideal gas
( G°) decreases sharply with increasing temperature ? whether the process is reversible or irreversible
1 (iii) No work is done during free expansion of an ideal gas
(a) C graphite + O (g) CO(g) whether the process is reversible or irreversible
2 2
(iv) No work is done during free expansion of an ideal gas
1
(b) CO(g) + O (g) CO2(g) when the process is reversible
2 2
(a) Only statement (iii) is correct
1 (b) Statements (i) and (iii) are correct
(c) Mg(s) + O (g) MgO(s)
2 2 (c) Statements (ii) and (iv) are correct
1 1 1 (d) Statements (i) and (iv) are correct
(d) C graphite + O2(g) CO2(g)
2 2 2 87. Which of the following statement(s) is/are correct ?
81. Consider the following reaction occurring in an automobile (i) In case of expansion maximum amount of work can be
2C 8 H18 (g ) 25 O 2 (g ) 16CO 2(g) 18H 2O(g ) obtained under isothermal conditions by reversibly
carrying out the process rather than through
the sign of H, S and G would be irreversible route.
(a) +, –, + (b) –, +, – (ii) In case of compression, minimum amount of work can
(c) –, +, + (d) +, +, – be done on system by carrying out the process
82. A reaction occurs spontaneously if irreversibly than reversibly.
(a) T S < H and both H and S are + ve (a) (i) and (ii) (b) Only (i)
(b) T S > H and H is + ve and S is ve (c) Only (ii) (d) Neither (i) nor (ii)
(c) T S > H and both H and S are + ve
88. Read the following statements carefully and choose the
(d) T S = H and both H and S are + ve
correct option
83. The enthalpy of fusion of water is 1.435 kCal/mol.
(i) In case of diatomic molecules the enthalpy of
The molar entropy change for the melting of ice at 0°C is :
atomization is also the bond dissociation enthalpy.
(a) 10.52 cal / (mol K) (b) 21.04 cal / (mol K)
(ii) In case polyatomic molecules, bond dissociation
(c) 5.260 cal / (mol K) (d) 0.526 cal / (mol K)
enthalpy is different for different bonds within the same
STATEMENT TYPE QUESTIONS molecule.
(a) Both (i) and (ii) are correct
84. Read the following statements carefully and choose the (b) (i) is correct but (ii) is incorrect
correct option (c) (ii) is correct but (i) is incorrect
(i) The state of the system is specified by state functions
(d) Both (i) and (ii) are incorrect
or state variables.
THERMODYNAMICS 101
89. Read the following statements regarding spontaneity of a 93. Match the columns
process and mark the appropriate choice. Column-I Column-II
(i) When enthalpy factor is absent than randomness factor (A) pext = 0 (p) Free expansion of an
decides spontaneity of a process. ideal gas
(ii) When randomness factor is absent then enthalpy factor (B) q = pext (Vf – Vi) (q) Adiabatic change
decides spontaneity of a process.
(C) q = 2.303 nRT log (Vf / Vi) (r) Isothermal reversible
(iii) When both the factors take place simultaneously, the change
magnitude of both the factors decide spontaneity of a
(D) U = Wad (s) Isothermal irreversible
process.
change
(a) Statements (i) and (ii) are correct and (iii) is incorrect.
(a) A – (p), B – (s), C – (r), D – (q)
(b) Statement (iii) is correct, (i) and (ii) are incorrect.
(b) A – (p), B – (q), C – (r), D – (s)
(c) Statements (i), (ii) and (iii) are correct.
(c) A – (p), B – (r), C – (s), D – (q)
(d) Statements (i), (ii) and (iii) are incorrect.
90. Which of the following statement is incorrect ? (d) A – (p), B – (r), C – (q), D – (s)
(a) The standard enthalpy of reaction is the enthalpy 94. Match the columns
change for a reaction when all the participating Column-I Column-II
substances are in their standard states. (A) H 2 (g) Br2 (g) 2HBr (g) (p) H = U – 2RT
(b) The standard state of a substance at a specified
temperature is its pure form at 1 bar. (B) PCl5 (g) PCl3 (g) Cl 2 (g) (q) H = U + 3RT
(c) The standard state of solid iron at 298 K is pure iron at (C) N 2 (g) 3H 2 (g) 2NH3 (g) (r) H= U
1 bar
(d) Standard conditions are denoted by adding the (D) 2N 2O5 (g) 4NO 2 (g) O2 (g) (s) H = U + RT
superscript to the symbol H e.g., H (a) A – (r), B – (p), C – (q), D – (s)
(b) A – (r), B – (s), C – (p), D – (q)
MATCHING TYPE QUESTIONS
(c) A – (q), B – (p), C – (s), D – (r)
91. Match the columns (d) A – (s), B – (q), C – (p), D – (r)
Column-I Column-II 95. Match the columns
(A) Cm (p) Cv T Column-I Column-II
(B) q (q) C/n 13
(C) (r) Cp T (A) C4 H10 O (p) Enthalpy of atomisation
2 2
(D) H (s) C T
4CO2 5H 2O; H = – w
(a) A – (q), B – (s), C – (r), D – (p)
(b) A – (q), B – (s), C – (p), D – (r) (B) CH4 C + 4H ; H = x (q) Enthalpy of formation
(c) A – (s), B – (q), C – (p), D – (r) (C) H2+ Br2 2HBr ; H = y (r) Enthalpy of combustion
(d) A – (q), B – (p), C – (r), D – (s) (D) Na–(s) Na(g) ; H = z (s) Enthalpy of sublimation
92. Match the columns (a) A – (s), B – (p), C – (q), D – (r)
Column-I Column-II (b) A – (q), B – (r), C – (p), D – (s)
(c) A – (r), B – (p), C – (q), D – (s)
Vf
(A) Free expansion at (p) q = – w = nRT n V (d) A – (p), B – (q), C – (s), D – (r)
i
96. Match the columns
V= 0
Column-I Column-II
(B) Isothermal irreversible (q) U = wad
(A) Exothermic (p) H = 0, E = 0
change
(B) Spontaneous (q) G = 0
(C) Isothermal reversible (r) U = qv
change (C) Cyclic process (r) H is negative
(D) For adiabatic change (s) q = – w = Pex (Vf – Vi) (D) Equilibrium (s) G is negative
(a) A – (s), B – (p), C – (r), D – (q) (a) A – (q), B – (r), C – (p), D – (s)
(b) A – (r), B – (q), C – (p), D – (s) (b) A – (s), B – (p), C – (r), D – (q)
(c) A – (r), B – (s), C – (p), D – (q) (c) A – (p), B – (q), C – (s), D – (r)
(d) A – (q), B – (r), C – (s), D – (p) (d) A – (r), B – (s), C – (p), D – (q)
EBD_7207
102 THERMODYNAMICS
97. Match the columns 105. Assertion : The mass and volume of a substance are the
Column-I Column-II extensive properties and are proportional to each other.
(A) H = –ve ; S = –ve (p) Reaction will be Reason : The ratio of mass of a sample to its volume is an
G = –ve non-spontaneous at intensive property.
high temperature 106. Assertion : First law of thermodynamics is applicable to an
(B) H = –ve ; S = –ve (q) Reaction will be electric fan or a heater.
G = +ve non-spontaneous at Reason : In an electric fan, the electrical energy is converted
low temperature into mechanical work that moves the blades. In a heater,
(C) H = +ve ; S = +ve (r) Reaction will be electrical energy is converted into heat energy.
G = +ve spontaneous at low 107. Assertion : The value of enthalpy of neutralization of weak
temperature acid and strong base is numerically less than 57.1 kJ.
(D) H = +ve ; S = +ve (s) Reaction will be Reason : All the OH– ions furnished by 1 g equivalent of
G = –ve spontaneous at high strong base are not completely neutralized.
temperature
108. Assertion : When a solid melts, decrease in enthalpy is
(a) A – (q), B – (r), C – (p), D – (s)
observed.
(b) A – (r), B – (p), C – (q), D – (s)
Reason : Melting of a solid is endothermic.
(c) A – (r), B – (q), C – (s), D – (p)
109. Assertion : Many endothermic reactions that are not
(d) A – (q), B – (s), C – (p), D – (r)
spontaneous at room temperature become spontaneous at
ASSERTION-REASON TYPE QUESTIONS high temperature.
Reason : Entropy of the system increases with increase in
Directions : Each of these questions contain two statements, temperature.
Assertion and Reason. Each of these questions also has four 110. Assertion : An exothermic process which is non-
alternative choices, only one of which is the correct answer. You spontaneous at high temperature may become spontaneous
have to select one of the codes (a), (b), (c) and (d) given below. at a low temperature.
(a) Assertion is correct, reason is correct; reason is a correct Reason : There occurs a decrease in entropy factor as the
explanation for assertion. temperature is decreased.
(b) Assertion is correct, reason is correct; reason is not a
correct explanation for assertion CRITICAL THINKING TYPE QUESTIONS
(c) Assertion is correct, reason is incorrect
(d) Assertion is incorrect, reason is correct. 111. In an adiabatic process, no transfer of heat takes place
98. Assertion : T, P and V are state variables or state functions. between system and surroundings. Choose the correct
Reason : Their values depend on the state of the system option for free expansion of an ideal gas under adiabatic
and how it is reached. condition from the following.
99. Assertion : At constant temperature and pressure whatever (a) q = 0, T 0, w = 0 (b) q 0, T = 0, w = 0
heat absorbed by the system is used in doing work. (c) q = 0, T = 0, w = 0 (d) q = 0, T < 0, w 0
Reason : Internal energy change is zero. 112. According to the first law of thermodynamics which of the
100. Assertion : For an isothermal reversible process Q = –W following quantities represents change in a state function ?
i.e. work done by the system equals the heat absorbed by (a) q rev (b) qrev – Wrev
the system. (c) q rev /W rev (d) qrev + Wrev
Reason : Enthalpy change ( H) is zero for isothermal
113. If H is the change in enthalpy and E, the change in
process.
internal energy accompanying a gaseous reaction, then
101. Assertion : Absolute value of internal energy of a substance
cannot be determined. (a) H is always greater than E
Reason : It is impossible to determine exact values of (b) H < E only if the number of moles of the products
constitutent energies of the substances. is greater than the number of moles of the reactants
102. Assertion : A process is called adiabatic if the system does (c) H is always less than E
not exchange heat with the surroundings.
(d) H < E only if the number of moles of products is
Reason : It does not involve increase or decrease in less than the number of moles of the reactants
temperature of the system.
114. For an isothermal reversible expansion process, the value
103. Assertion : There is exchange in internal energy in a cyclic
of q can be calculated by the expression
process.
Reason : Cyclic proces is the one in which the sytem returns V2 V2
(a) q 2.303nRT log q 2.303nRT log
to its initial state after a number of reactions. V1 (b) V1
104. Assertion : Internal energy is an extensive property.
Reason : Internal energy depends upon the amount of the V1
(c) q Pexp nRT log
system. V2 (d) None of these
THERMODYNAMICS 103
115. A piston filled with 0.04 mol of an ideal gas expands 122. An ideal gas is allowed to expand both reversibly and
reversibly from 50.0 mL to 375 mL at a constant temperature irreversibly in an isolated system. If T i is the initial
of 37.0ºC. As it does so, it absorbs 208 J of heat. The values temperature and Tf is the final temperature, which of the
of q and w for the process will be: following statements is correct?
(R = 8.314 J/mol K) (ln 7.5 = 2.01) (a) (Tf)rev = (Tf)irrev
(a) q = + 208 J, w = – 208 J (b) Tf = Ti for both reversible and irreversible processes
(b) q = – 208 J, w = – 208 J (c) (Tf)irrev > (Tf)rev
(c) q = – 208 J, w = + 208 J (d) Tf > Ti for reversible process but Tf = Ti for irreversible
(d) q = + 208 J, w = + 208 J process
116. According to the first law of thermodynamics, U = q + W. 123. Given
In special cases the statement can be expressed in different Reaction Energy Change
ways. Which of the following is not a correct expression ? (in kJ)
(a) At constant temperature q = –W Li(s) Li(g) 161
(b) When no work is done U = q Li(g) +
Li (g) 520
(c) In gaseous system U = q + P V
(d) When work is done by the system : U = q + W 1
F (g) F(g) 77
117. The internal energy change when a system goes from state A 2 2
to B is 40 kJ/mole. If the system goes from A to B by a reversible F(g) + e– F–(g) (Electron gain
path and returns to state A by an irreversible path what would enthalpy)
be the net change in internal energy ? Li+ (g) + F–(g) Li F(s) – 1047
(a) > 40 kJ (b) < 40 kJ
1
(c) Zero (d) 40 kJ Li (s) + F (g) Li F(s) – 617
118. Under isothermal condition for one mole of ideal gas what 2 2
is the ratio of work done under reversible to irreversible Based on data provided, the value of electron gain enthalpy
process, initially held at 20 atm undergoes expansion from of fluorine would be :
1L to 2L, at 298K, under external pressure of 10 atm? (a) – 300 kJ mol–1 (b) – 350 kJ mol–1
(c) – 328 kJ mol –1 (d) – 228 kJ mol–1
(a) 1.7 (b) 2.0
(c) 1.4 (d) 1.0 124. The standard enthalpy of formation ( f H° 298) for methane,
119. Processes A to B, B to C and C to D shown in the figure CH4 is – 74.9 kJ mol–1. In order to calculate the average
below respectively are ? energy given out in the formation of a C – H bond from this
it is necessary to know which one of the following?
P1/ V1/ T1 P2/ V2/ T1 P2/ V3/ T2 P3/ V3/ T3 (a) The dissociation energy of the hydrogen molecule,
H2.
(b) The first four ionisation energies of carbon.
(c) The dissociation energy of H2 and enthalpy and
A B C D sublimation of carbon (graphite).
(d) The first four ionisation energies of carbon and
(a) Isothermal, isobaric and isochoric
electron affinity of hydrogen.
(b) Isobaric, isothermal and isochoric
125. For complete combustion of ethanol,
(c) Isothermal, isothermal and isobaric
C2 H5 OH l 3O2 g 2CO2 g 3H 2 O l ,
(d) Isobaric, isobaric and isothermal
120. What is the internal energy (kJ) change occurs when 36g the amount of heat produced as measured in bomb
of H2O(l) converted to H2O(g)? H°(vapourisation) = calorimeter, is 1364.47 kJ mol–1 at 25ºC. Assuming ideality
40.79kJ/mol the enthalpy of combustion, cH, for the reaction will be:
(a) (b) (R = 8.314 kJ mol–1)
(c) (d)
1 1
121. Consider the reaction : (a) 1366.95 kJ mol (b) 1361.95 kJ mol
4NO2 ( g ) O 2 ( g ) 2N 2O5 ( g ), (c) 1460.95 kJ mol 1
(d) 1350.50 kJ mol 1

rH = – 111 kJ. 126. Standard enthalpy of vapourisation vap H° for water at


If N2O5(s) is formed instead of N2O5(g) in the above reaction, 100°C is 40.66 kJ mol–1. The internal energy of vaporisation
the rH value will be : of water at 100°C (in kJ mol–1) is :
(given, H of sublimation for N2O5 is –54 kJ mol–1) (a) + 37.56 (b) – 43.76
(a) + 54 kJ (b) + 219 kJ (c) + 43.76 (d) + 40.66
(c) – 219 J (d) – 165 kJ (Assume water vapour to behave like an ideal gas).
EBD_7207
104 THERMODYNAMICS
127. Consider the following reactions: 131. The enthalpy changes for the following processes are listed
(i) H +(aq) + OH–(aq) = H2O(l), below :
H = – X1 kJ mol–1 Cl2(g) 2Cl(g), 242.3 kJ mol–1
I2(g) 2I(g), 151.0 kJ mol–1
1
(ii) H2(g) + O = H2O(l), ICl(g) I(g) + Cl(g), 211.3 kJ mol–1
2 2(g)
I2(s) I2(g), 62.76 kJ mol–1
H = – X2 kJ mol–1
Given that the standard states for iodine and chlorine are
(iii) CO2(g) + H2(g) = CO(g) + H2O,
I2(s) and Cl2(g), the standard enthalpy of formation for ICl(g)
H = – X3 kJ mol–1
is :
5 (a) +16.8 kJ mol–1 (b) +244.8 kJ mol–1
(iv) C 2 H 2(g) O2(g) = 2CO2(g) + H2O(l)’
2 (c) –14.6 kJ mol–1 (d) –16.8 kJ mol–1
H = + 4X4 kJ mol–1 132. What is the equilibrium constant if ATP hydrolysis by water
Enthalpy of formation of H2O (l) is produce standard free energy of –50 kJ/mole under normal
(a) + X3 kJ mol– 1 (b) – X4 kJ mol– 1 body conditions ?
(c) + X1 kJ mol – 1 (d) – X2 kJ mol– 1 (a) 2.66 × 108 (b) 5.81 × 108
128. Diborane is a potential rocket fuel which undergoes (c) 1.18 × 107 (d) 1.98 × 108
combustion according to the equation 133. A reaction with H = 0, is found to be spontaneous. This is
B2 H 6 ( g ) 3O 2 ( s ) B 2 O3 ( s ) 3H 2O( g ) due to
(a) S is negative (b) S is positive
Calculate the enthalpy change for the combustion of
diborane. Given (c) T S is positive (d) Both (b) and (c)
134. In an irreversible process taking place at constant T and P and
3
(i) 2B( s) O (g) B2 O3 ( s); H = –1273 kJ per mol in which only pressure-volume work is being done, the change
2 2 in Gibbs free energy (dG) and change in entropy (dS), satisfy
1 the criteria
(ii) H2 ( g ) O2 (g ) H 2O(l ); H = –286 kJ per mol (a) (dS)V, E > 0, (dG)T, P < 0
2
(b) (dS)V, E = 0, (dG)T, P = 0
(iii) H 2O(l ) H 2O( g ); H = 44 kJ per mol (c) (dS)V, E = 0, (dG)T, P > 0
(iv) 2B (s) + 3H2 (g) B2H6 (g); H = 36 kJ per mol (d) (dS)V, E < 0, (dG)T, P < 0
(a) + 2035 kJ per mol (b) – 2035 kJ per mol 135. In conversion of lime-stone to lime,
(c) + 2167 kJ per mol (d) – 2167 kJ per mol CaCO3(s) CaO(s) CO2(g) the values of H and S
129. How many molecules of ATP, undergo hydrolysis to raise
-1
are + 179.1 kJ mol and 160.2 J/K respectively at 298 K and
the temperature of 180 kg of water which was originally at
room temperature by 1°C ? C{P,m} water = 75.32J/mol/K, 1 bar. Assuming that H and S do not change with
H{P} for ATP hydrolysis= 7 kcal/mol temperature, temperature above which conversion of
(a) 1.5 × 1025 (b) 2.00 × 1023 limestone to lime will be spontaneous is
(c) 3.4 × 10 25 (d) 4.0 × 1024 (a) 1118 K (b) 1008 K
130. What is the amount of heat (in Joules) absorbed by 18 g of (c) 1200 K (d) 845 K.
water initially at room temperature heated to 100°C ? If 10g 136. For vaporization of water at 1 atmospheric pressure, the
of Cu is added to this water , than decrease in temperature values of H and S are 40.63 kJmol–1 and 108.8 JK–1 mol–1,
(in Kelvin) of water was found to be? C (p,m) for water respectively. The temperature when Gibbs energy change
75.32J/mol K ; C (p,m) for Cu = 24.47J/mol K. ( G) for this transformation will be zero, is:
(a) 5649, 369 (b) 5544, 324 (a) 293.4 K (b) 273.4 K
(c) 5278, 342 (d) 3425, 425 (c) 393.4 K (d) 373.4 K.
THERMODYNAMICS 105

22. (c) For isothermal reversible expansion.


FACT / DEFINITION TYPE QUESTIONS
V
1. (c) w = –nRT ln 2
V
2. (b) The laws of thermodynamics deal with energy changes
of macroscopic systems involving a large number of 23. (c) W P V 105 (1 10 2
1 10 3 ) 900J
molecules rather than microscopic systems containing 24. (d) The difference between H and U is not usually
a few molecules.
significant for systems consisting of only solids or
3. (b)
liquids. Solids and liquids do not suffer any significant
4. (c) The universe = The system + The surroundings volume changes upon heating. The difference,
5. (c) however, becomes significant when gases are
6. (c) Closed system can exchange energy and not matter involved.
with surroundings. Pressure cooker provides closed 25. (b) H = E + P V, for solid and liquid,
system.
V = or H = E + n RT, for solids and liquids n = 0.
7. (c) Isolated system can not exchange mass or energy.
26. (c) During isothermal expansion of an ideal gas,
8. (b) We can describe the state of a gas by quoting its
pressure (P), volume (V), temperature (T ), amount (n) T = 0. Now H = E + PV
etc. H E ( PV )
9. (d) We know that q (heat) and work (w) are not state
H = E + (nRT); T);
functions but (q + w) is a state function. H – TS (i.e. G)
is also a state functions. Thus II and III are not state Thus if T = 0., H = E
functions so the correct answer is option (d). i.e., remain unaffected
10. (c) Internal energy and molar enthalpy are state functions. 27. (d) We know that
Work (reversible or irreversible) is a path function. H= E + P V
11. (c) In the reactions, H2 + Br 2 2HBr there is no
12. (a) Internal energy is a quantity which represents the total change in volume or V = 0
energy of the system. It may be chemical, electrical, So, H = E for this reaction
and mechanical or any other type of energy you may 1 1
think of, the sum of all these is the internal energy of 28. (b) n ; H E RT ; E H
2 2
the system.
13. (d) 14. (b) 29. (c) As all reactant and product are liquid n (g) 0
15. (c) In accordance with Hess’s law.
H E nRT
16. (a)
17. (a) E = Q–W H E ( n 0)
For adiabatic expansion, Q = 0 30. (a) H E P V
E = –W 31. (c) H E nRT
The negative sign shows decrease in Internal energy, n = 3 – (1 + 5)
which is equal to the work done on the system by the
= 3 – 6 = –3
surroundings.
18. (d) q is a path dependent function, H is a state function H E ( 3RT )
because it depends on U, p and V, all of which are 32. (b) H U nRT for N 2 3H 2 2 NH 3
state functions. ng = 2 – 4 = – 2
19. (a) The shaded area shows work done on an ideal gas in a
H= U 2 RT or U= H+ 2R T U> H
cylinder when it is compressed by a constant external
pressure 33. (a) Mass independent properties (molar conductivity and
20. (b) As volume is constant hence work done in this proces electromotive force) are intensive properties.
is zero hence heat supplied is equal to change in Resistance and heat capacity are mass dependent,
internal energy. hence extensive properties.
21. (b) W = – p V 34. (a) Volume depends upon mass. Hence it is extensive
property.
3(6 4) 6 litre atmosphere
6 101.32 608 J
EBD_7207
106 THERMODYNAMICS
35. (c) An extensive property is a property whose value 48. (b) Enthalpy of reaction
depends on the quantity or size of matter present in = B.E(Reactant)– B.E(Product)
the system. For example, mass, volume, internal energy,
enthalpy, heat capacity, etc. are extensive properties = B.E (C C) 4 B.E.(C–H) B.E.(H H)
36. (d ) The magnitude of the heat capacity depends on the
size, composition and nature of the system. B.E.(C C) 6 B.E.(C H)
37. (b) The heat required to raise the temperature of body by = [606.1 + (4 × 410.5) + 431.37)] – [336.49 + (6 × 410.5)]
1C° is called thermal capacity or heat capacity. = –120.0 kJ mol–1
5 49. (a) Fe2O3(s)+ CO(g) 2FeO(s)+ CO2(g)
CP R
2 5
1.67 H = –26.8 + 33.0 = + 6.2 kJ
38. (d) CV 3 3 50. (b) Given H
R
2 1
39. (c) Given Cp = 75 JK–1 mol–1 A B + 150 ...(1)
2
100
n mole , Q = 1000 J T=? 3B 2C D –125 ...(2)
18 +350 ...(3)
E A 2D
1000 18 To calculate H operate
Q = nCp T T 2.4 K 2 × eq. (1) + eq. (2) – eq. (3)
100 75
40. (b) 1 calorie = 4.184 joule H = 300 – 125 – 350 = – 175
41. (b) The coefficients in a balanced thermo-chemical H H f products H f reactants
51. (b)
equation refer to the number of moles (not to molecules)
of reactants and products involved in the reaction.
H° [ H f (CO)(g) H f (H 2 O)(g)] –
42. (c)
43. (b) Enthalpy of formation of C 2 H 4 , CO2 and H 2 O are [ H f (CO 2 )(g) H f (H 2 )(g)]
52, – 394 and – 286 kJ/ mol respectively. (Given) = [– 110.5 + ( – 241.8)]– [– 393.5 + 0] = 41.2
The reaction is
52. (b) C 2 H 4 3O 2 2CO 2 2H 2O.
C 2 H 4 3O 2 2CO 2 2H 2O. Change in enthalpy,
change in enthalpy,
H H products H reactants
( H) H products H reactants
2 ( 394) 2 ( 286) (52 0)
2 ( 394) 2 ( 286) (52 0)
= –1412 kJ/ mol.
= – 1412 kJ/ mol. 53. (a) Hess’s law is used for calculating enthalpy of reaction.
44. (b) 45. (c)
54. (c) H
1 1
X Y
46. (b) H2 Cl 2 HCl H1 H2 H3
2 2 X P Q Y
H HCl B.E. of reactant H H1 H2 H3
55. (a) 56. (b)
B.E. of products
57. (b) Heat of combustion of a substance is always negative
as it is the amount of heat evolved (i.e. decrease in
1 1 1
90 430 240 B.E. of HCl enthalpy) when one mole of the substance is
2 2 2
completely burnt in air or oxygen.
B.E. of HCl = 215 + 120 + 90
58. (c)
= 425 kJ mol–1
59. (a) Conc. of HCl = 0.25 mole
47. (c) The reaction for formation of HCl can be written as
Conc. of NaOH = 0.25 mole
H2 + Cl2 2HCI
Heat of neutralization of strong acid by strong base
H – H + Cl – Cl 2 (H – Cl) = – 57.1 kJ
Substituting the given values, we get enthalpy of
HCl + NaOH NaCl + H2O – 57.1kJ
formation of
1 mole of HCl neutralise 1 mole of NaOH, heat evolved
2HCl = – ( 862 – 676) = –186 kJ.
= 57.1 kJ
Enthalpy of formation of
0.25 mole of HCl neutralise 0.25 mole of NaOH
186 Heat evolved = 57.1 × 0.25 = 14.275 kJ
HCl = kJ = –93 kJ.
2
THERMODYNAMICS 107
60. (c) For a reaction to be non-spontaneous at all
61. (d) A process is spontaneous only when there is decrease in temperatures, H should be +ve and S should be –ve
the value of free energy, i.e., G is –ve. G = +ve – T × (–ve); the value of G is always
62. (a) If Gsystem = 0 the system has attained equilibrium is positive for such a reaction and hence it will be non-
right choice. spontaneous at all temperatures.
In it alternative (d) is most confusing as when G > 0, 74. (d) We know that G = H – T S
the process may be spontaneous when it is coupled When H < 0 and S < 0 then G will be negative at
with a reaction which has G < 0 and total G is low temperatures (positive at high temperature) and
negative, so right answer is (a). the reaction will be spontaneous.
63. (b) Spontaneity of reaction depends on tendency to 75. (d) Since the process is at equilibrium G = 0 for G 0,
acquire minimum energy state and maximum they should be H > 0, S > 0.
randomness. For a spontaneous process in an isolated 76. (c) S has negative value if number of gaseous moles
system the change in entropy is positive. decreases during a reaction, ng = –ve
For the reaction
64. (d) G is negative for a spontaneous process.
2SO2 + O2 2SO3
65. (a) Crystallization of sucrose solution. Entropy is a
ng = 2 – 3 = –1
measure of randomness during the crystallisation of
77. (d) G= H– T S
sucrose solution liquid state is changing into solid
For a reaction to be spontaneous,
state hence entropy decreases.
H = –ve, S = +ve
66. (d) G = H – T S; G is positive for a reaction to be at all temperatures.
non-spontaneous when H is positive and S is
but at high temperature, G H T S
negative. ve ve
67. (b) For the reaction Thus the second term will have high positive value
PCl5 (g) PCl3 (g) Cl2 (g) and reaction will be non-spontaneous.
The reaction given is an example of decomposition 78. (b) At equilibrium G = 0
reaction and we know that decomposition reactions Hence, G = H – Te S = 0
are endothermic in nature, i.e, H > 0. H = Te S or Te = DH
Further DS
For a spontaneous reaction
n = (1 + 1) – 1= + 1
G must be negative which is possible only if
Hence more number of molecules are present in
H– T S<0
products which shows more randomness i.e. S > 0
( S is positive) DH
H < T S or T > ; Te < T
DS
68. (b) For the reaction
79. (a) Measure of disorder of a system is nothing but
2ZnS 2Zn + S2 ; G1º = 293 kJ ..........(1)
Entropy. For a spontaneous reaction, G < 0. As per
2Zn + O2 2ZnO ; G2º = – 480 kJ ..........(2) Gibbs Helmholtz equation,
S2 + 2 O2 2SO2 ; G3º = – 544 kJ .........(3) G= H– T S
Gº for the reaction Thus G is –ve only
2ZnS + 3 O2 2ZnO + 2SO2 When H = –ve (exothermic)
can be obtained by adding eqn. (1), (2) and (3) and S = +ve (increasing disorder)
Gº = 293 – 480 – 544 = – 731 kJ 80. (a) Since, in the first reaction gaseous products are
69. (a) Third law of Thermodynamics. forming from solid carbon hence entropy will increase
q i.e. S = +ve.
70. (a) S
T 1
C (gr.) + O2(g) CO(g); S° = + ve
q required heat per mole 2
T constant absolute temperatur e Since, G° = H° – T S hence the value of G decrease
Unit of entropy is JK–1 mol–1 on increasing temperature.
71. (a) For a spontaneous process, S total is always positive. 81. (b) This is combustion reaction, which is always exothermic
hence
72. (d) For an exothermic reaction all three enthalpy, entropy H = –ve
and Gibb's free energy change have negative values. As the no. of gaseous molecules are increasing hence
73. (d) Gibb’s-Helmholtz equation is entropy increases
G= H– T S now G H T S
EBD_7207
108 THERMODYNAMICS
For a spontaneous reaction ASSERTION-REASON TYPE QUESTIONS
G ve
98. (c) Values of state functions depend only on the state of
Which is possible in this case as H = –ve and S
the system and not on how it is reached.
= +ve.
99. (a) Q = –W if E = 0
82. (c) For a spontaneous reaction
100. (b) In an isothermal process change in internal energy
G(–ve), which is possible if S = +ve,
( E) is zero (as it is a function of temperature).
H = +ve
According to first law of thermodynamics
and T S > H [As G = H – T S]
Q + W = E. Hence Q = –W (if E = 0)
H 1.435 103 If a system undergoes a change in which internal
83. (c) S energy of the system remains constant (i.e. E = 0)
T 273
then –W = Q. This means that work done by the system
5.260 cal / (mol K) equals the heat absorbed by the system.
101. (a) It is fact that absolute values of internal energy of
STATEMENT TYPE QUESTIONS substances cannot be determined. It is also true that it
is not possible to determine exact values of constitutent
84. (c) Variables like P, V and T which describes the state of energies of a substance.
system are called state variables or state functions 102. (c) It may involve increase or decrease in temperature of
because their values depend only on the state of the the system. Systems in which such processes occur,
system and not on how it is reached. are thermally insulated from the surroundings.
85. (d) The positive sign expresses when work is done on the 103. (a) As internal energy is a state function so its value
system. Similarly, negative sign expresses when work depends on intial and final states of the system. In
is done by the system case of cyclic system initial and final states are same.
86. (b) 87. (a) 88. (a) So E = 0, and similarly H = 0.
89. (c) All the statements regarding spontaneity of a reaction 104. (a) The properties whose magnitude depends upon the
are correct. quantity of matter present in the system are called
90. (c) The standard enthalpy of reaction is the enthalpy extensive properties eg, internal energy.
change for a reaction when all the participating 105. (b) The mass and volume depend upon the quantity of
substances are in their standard states. The standard matter so these are extensive properties while ratio of
state of a substance at a specified temperature is its mass to its volume does not depend upon the quantity
pure form at 1 bar. For example, the standard state of of matter so this ratio is an extensive property.
liquid ethanol at 298 K is pure liquid ethanol at 1 bar. 106. (a) In case of electric fan electrical energy is converted
Standard state of solid ion at 500 K is pure iron at 1 bar. into mechanical energy and in case of heater, electrical
The standard conditions are denoted by adding the energy is converted into heat energy. Therefore, these
superscript to the symbol H e.g., H . follow the first law of thermodynamics.
107. (c) The value of enthalpy of neutralisation of weak acid
MATCHING TYPE QUESTIONS by strong base is less than 57.1 kJ. This is due to the
reason that the part of energy liberated during
91. (b) 92. (c) combination of H+ and OH+ ions is utilised in the
93. (a) A – (p), B – (s), C – (r), D – (q) ionisation of weak acid.
Expansion of a gas in vacuum (pext = 0) is called free 108. (d) When a solid melts, increase in enthalpy is observed.
expansion. 109. (b) The factor T S increases with increase in temperature.
For isothermal irreversible change 110. (b) Both assertion and reason are true but reason is not
q = –W = pext(Vf – Vi) the correct explanation of assertion.
for isothermal reversible change For a process to be spontaneous G must be negative.
q = – W = nRT ln (Vf/Vi) G= H–T S
= 2.303 nRT log Vf/Vi Exothermic process ( H is negative) is non-
For adiabatic change, q = 0, U = Wad spontaneous if S is negative and temperature is high
94. (b) (A) ng = 2 – 2 = 0 hence H = U because in such condition T S H.
(B) ng = 2 – 1 = 1 hence H = U + RT ( G H T S tive ). When temperature is
(C) ng = 2 – 4 = –2 hence H = U – 2RT
decreased, T S H ( G H T S tive)
(D) ng = 5 – 2 = 3 hence H = U + 3RT
and so the reaction becomes spontaneous.
95. (c) 96. (d) 97. (b)
THERMODYNAMICS 109

CRITICAL THINKING TYPE QUESTIONS 8.314 373


Uo = 40.79 kJ/mol kJ / mol
111. (c) Justification : free expansion w = 0 1000
adiabatic process q = 0 = (40.79 – 3.10) kJ/mol
U = q + w = 0, this means that internal energy remains kJ
constant. Therefore, = 37.69
mol
T = 0.
Internal energy change for 36 g of water
112. (d) Mathematical expression of first law of
thermodynamics kJ 36g
37.69
E = q + w, E is a state function. mol 18g / mol
113. (d) As H = E + ngRT U = 75.98 kJ
if np < nr; ng = np – n r = – ve. 4NO 2 (g) O 2 (g) 2N 2 O5 (g),
121. (d) rH = – 111 kJ
Hence H< E.
– 54 kJ
V2 H'
114. (a) q = –W 2.303nRTlog
V1 2N 2 O5 (s)
115. (a) Process is isothermal reversible expansion, hence
– 111 – 54 = H'
U = 0, therefore q = – W.
Since q = + 208 J, W = – 208 J H' = – 165 kJ
122. (c) In a reversible process the work done is greater than in
116. (d) When work is done by the system, U = q – W
irreversible process. Hence the heat absorbed in
117. (c) For a cyclic process the net change in the internal reversible process would be greater than in the latter
energy is zero because the change in internal energy case. So
does not depend on the path. Tf (rev.) < Tf (irr.)
reversible path
123. (c) Applying Hess’s Law
1
fH sub H diss H I.E. E.A lattice H
2
A B
617 161 520 77 E.A. (–1047)
E.A. = –617 + 289 = –328 kJ mol–1
irreversible path
electron affinity of fluorine
118. (a) –Wirreversible = Pext (V2 – V1) = –328 kJ mol–1
= 10 atm (2L – 1L) 124. (a) To calculate average enthalpy of C – H bond in
= 10 atm – L methane following informations are needed
(i) dissociation energy of H2 i.e.
V2
1
Pex dv H (g) H( g ); H x suppose
–Wreversible = 2 2
V1
(ii) Sublimation energy of C(graphite) to C(g)
V2 C( graphite) C( g ); H y Suppose
= 2.303 nRT log V Given
1
1
C( graphite ) 2H 2 ( g ) CH 4 ( g ); H 75 kJ mol
2
= 1 2.303 0.0821 atm–L /K /mol × log 125. (a) C 2 H 5 OH( ) 3O 2 ( g ) 2CO2 ( g ) 3H2O( )
1
Bomb calorimeter gives U of the reaction
= 16.96 atm–L
Given, U = –1364.47 kJ mol–1
Wreversible 16.96 ng = – 1
= 1.69 1.7
Wirreversible 10.00 1 8.314 298
H = U + ngRT = 1364.47
119. (a) 1000
120. (a) H2O (l ) H2O (g) = – 1366.93 kJ mol–1
Hvap = 40.79 kJ/mol 126. (a) H 2O ( ) H 2O (g) + Q
H = U + ngRT E = 37558 J / mol
40.79 kJ/mol = U + (1) (8.314 JK–1 mol–1) (373 K) E = 37.56 kJ mol–1
EBD_7207
110 THERMODYNAMICS
127. (d) This reaction shows the formation of H2O, and the X2 If now 10g of copper is added Cp, m = 24.47 J/ mol K
represents the enthalpy of formation of H2O because Amount of heat gained by Cu
as the definition suggests that the enthalpy of formation
is the heat evolved or absorbed when one mole of J 10g
= 24.47 (373 – 298) K
substance is formed from its constituent atoms. K mol 63g / mol
128. (b) For the equation = 291.3 J
B 2 H 6 ( g ) 3O 2 ( g ) B 2 O 3 ( g ) 3H 2 O( g ) Heat lost by water = 291.30 J
Eqs. (i) + 3 (ii) + 3 (iii) – (iv) J
H = – 1273 + 3(–286) + 3(44) – 36 – 291.30 J = 75.32 T2 373K
K
= – 1273 – 858 + 132 – 36
–3.947 K = T2 –373 K
= – 2035 kJ/mol
T2 = 369.05 K
129. (a) qp H Cp dT 131. (a) I 2 ( s ) Cl2 ( g ) 2ICl( g )
J rH = [ H(I2(s) I2(g)) + HI–I + HCl–Cl] – [ HI –
qp 75.32 299 298 K ]
K mol Cl
= 151.0 + 242.3 + 62.76 –2 × 211.3 = 33.46
J 33.46
qp 75.32
K mol Df H°(ICl) = = 16.73 kJ / mol
2
For 180 kg of water, no. of moles of water
132. (a) G RT ln K eq : Normal body temperature = 37oC
180 103 g
104 g moles kJ J
18g / mol 50 8.314 310 lnK eq
mol K mol
J 19.39 = lnKeq
qp 75.32 10 4 moles
mol
= 753.2 × 103 J = 753.2 kJ K eq 2.6 108
H for ATP = 7 kcal / mol 133. (b) G= H– T S
= 7 × 4.184 kJ/mol G = – T S (when H = 0 and S = +ve)
= 29.2 kJ/mol G = –ve
6.022 × 1023 molecules of ATP produce = 29.2 kJ 134. (a) For spontaneous reaction, dS > 0 and dG should be
29.2 kJ produced from 6.022 × 1023 molecules negative i.e. < 0.
75.8 135. (a) G° = H° – T S°
753.2 kJ produced from 6.022 × 1023 × For a spontaneous reaction G° < 0
29.2
= 1.5 × 1025 molecules or H° – T S° < 0
H
T
130. (a) 18gm of water at 100oC S
10gm of Cu at 25oC is added.
179.3 103
T> 1117.9K 1118K
160.2
1atm
o 136. (d) H 2O H2O(g)
100 C
H = 40630 J mol –1
S = 108.8 JK–1 mol –1
qp = Cp, m dT
G = H T S When G = 0,
J 18g H T S =0
= 75.32 373 298 K
K mol 18g / mol
H 40630 J mol 1
J T = S
=
108.8 J mol 1
= 373.4 K.
= 75.32 × 75 K
K
= 5.649 × 103 J
7
EQUILIBRIUM

FACT / DEFINITION TYPE QUESTIONS (b) The rate of condensation from vapour to liquid state is
equal to the rate of evaporation.
1. Which of the following is not a general characteristic of (c) The rate of condensation from vapour to liquid state is
equilibria involving physical processes ? much less than the rate of evaporation.
(a) Equilibrium is possible only in a closed system at a (d) The rate of condensation from vapour to liquid state is
given temperature. equal or less than the rate of evaporation.
(b) All measurable properties of the system remain 6. When pressure is applied to the equilibrium system
constant. Ice Water
(c) All the physical processes stop at equilibrium. Which of the following phenomenon will happen?
(d) The opposing processes occur at the same rate and (a) More ice will be formed
there is dynamic but stable condition. (b) Water will evaporate
2. The liquid which has a ………..vapour pressure is more (c) More water will be formed
volatile and has a ………….boiling point. (d) Equilibrium will not be formed
(a) Higher , higher (b) Lower, lower 7. A reaction is said to be in equilibrium when
(c) Higher, lower (d) Lower, higher (a) the rate of transformation of reactant to products is
3. Boiling point of the liquid depends on the atmospheric equal to the rate of transformation of products to the
pressure. It depends on the altitude of the place; at high reactants.
altitude the boiling point………….. (b) 50% of the reactants are converted to products.
(a) increases (c) the reaction is near completion and all the reactants
(b) decreases are converted to products.
(c) either decreases or increases (d) the volume of reactants is just equal to the volume of
(d) remains same the products.
4. In an experiment three watch glasses containing separately 8. Which of the following is not true about a reversible
1mL each of acetone, ethyl alcohol, and water are exposed reaction?
to atmosphere and the experiment with different volumes of (a) The reaction does not proceed to completion
the liquids in a warmer room is repeated, it is observed that (b) It cannot be influenced by a catalyst
in all such cases the liquid eventually disappears and the (c) Number of moles of reactants and products is always
time taken for complete evaporation in each case was equal
different. The possible reason is/are (d) It can be attained only in a closed container
(a) the nature of the liquids is different 9. If the synthesis of ammonia from Haber's process is carried
(b) the amount of the liquids is different out with exactly the same starting conditions (of partial
(c) the temperature is different pressure and temperature) but using D2 (deuterium) in place
(d) All of the above of H2. Then
5. A small amount of acetone is taken in a watch glass and it is (a) the equilibrium will be disturbed
kept open in atmosphere. Which statement is correct for (b) the composition of reaction mixture will remain same at
the given experiment? equilibrium.
(a) The rate of condensation from vapour to liquid state is (c) Use of isotope in reaction will not produce ammonia.
higher than the rate of evaporation. (d) At equilibrium rate of forward reaction will be greater
than the rate of reverse reaction
EBD_7207
112 EQUILIBRIUM
10. Consider the following graph and mark the correct statement. 1
(a) (b) 6
6
HI
1
(c) (d) 36
HI 36
Conc.
H2+I2 H 2 + I2 16. Given the reaction between 2 gases represented by A2 and
B2 to give the compound AB(g).
A2(g) + B2(g) 2 AB(g).
Time Equal Time
At equilibrium, the concentration
of A2 = 3.0 × 10–3 M
(a) Chemical equilibrium in the reaction, H 2 I2 2HI of B2= 4.2 × 10–3 M
of AB = 2.8 × 10–3 M
can be attained from other directions.
lf the reaction takes place in a sealed vessel at 527°C, then
(b) Equilibrium can be detained when H2 and I2 are mixed
the value of KC will be :
in an open vessel.
(a) 2.0 (b) 1.9
(c) The concentrations of H2 and I2 keep decreasing while
(c) 0.62 (d) 4.5
concentration of HI keeps increasing with time.
(d) We can find out equilibrium concentration of H2 and 17. A reaction is A + B C + D. Initially we start with
I2 from the given graph. equal concentrations of A and B. At equilibrium we find
11. What are the product formed when Deuterium is added that the moles of C is two times of A. What is the equilibrium
equilibrium reaction of H2 and I2 ? constant of the reaction?
(i) HD (ii) DI 1 1
(a) (b)
(iii) D2 (iv) HI 4 2
(a) (i), (ii) and (iv) (b) (i) and (ii) (c) 4 (d) 2
(c) (ii) and (iv) (d) All of these
12. If a system is at equilibrium, the rate of forward to the reverse 18. In A B C . The unit of equilibrium constant is :
(a) Litre mole –1 (b) Mole litre
reaction is :
(a) less (b) equal (c) Mole litre–1 (d) No unit
(c) high (d) at equilibrium 19. For the reaction C(s) CO 2 (g) 2CO(g) , the partial
13. K1 and K2 are equilibrium constant for reactions (1) and (2) pressures of CO2 and CO are 2.0 and 4.0 atm respectively at
N2(g) + O2 (g) 2 NO (g) .............(1) equilibrium. The Kp for the reaction is.
1 1 (a) 0.5 (b) 4.0
NO(g) N 2 (g ) O 2 (g ) ......(2) (c) 8.0 (d) 32.0
2 2
20. In which of the following equilibrium Kc and Kp are not
Then,
equal?
2
(a) K1 = 1 (b) K1 = K22 (a) 2 NO( g ) N 2 (g)+O 2 ( g )
K2
(b) SO 2 (g)+NO2 ( g ) SO3 (g)+NO( g )
1 (c) H 2 (g)+I 2 ( g ) 2HI(g)
(c) K1 = (d) K1 = (K2)0
K2
(d) 2C(s)+O 2 ( g ) 2CO2(g)
14. The equilibrium constant for the reversible reaction
21. For the following reaction in gaseous phase
N2 + 3H2 2NH3 is K and for reaction
1
1 3 CO( g ) O 2 ( g ) CO 2 ( g ), K p / K c is
N2 H2 NH3, the equilibrium constant is K ' 2
2 2 (a) (RT)1/2 (b) (RT)–1/2
The K and K ' will be related as: (c) (RT) (d) (RT)–1
(a) K × K ' = 1 (b) K= K' 22. The KP/KC ratio will be highest in case of
(c) K ' K (d) K K' 1
(a) CO (g) O (g) CO2 (g)
15. In the following equilibrium reaction 2 2
2A B + C, (b) H2 (g) + I2 (g) 2HI (g)
the equilibrium concentrations of A, B and C are 1 × 10–3 M,
2 × 10–3 M and 3 × 10–3 M respectively at 300 K. The value (c) PCl5 (g) PCl3(g) + Cl2 (g)
of Kc for this equilibrium at the same temperature is
(d) 7H2 (g) + 2NO2 (g) 2NH3 (g) + 4H2O(g)
EQUILIBRIUM 113
23. For a chemical reaction ; 30. The thermal dissociation of calcium carbonate showing
A (g) + B ( ) D (g) + E (g) heterogeneous equilibrium is
Hypothetically at what temperature, Kp = Kc CaCO 3 (s) CaO(s) CO 2 (g)
(when, R = 0.08 -atm/mole-K)
(a) T = 0 K (b) T = 1K For this reactions which of the following is/are true
(c) T = 12.5 K (d) T = 273 K (i) Kć = [CO2(g)]
24. Steam reacts with iron at high temperature to give hydrogen (ii) Kp = pCO2
gas and Fe3O4 (s). The correct expression for the equilibrium (iii) [CaCO 3 (s)] and [CaO(s)] are both con stant
constant is (iv) [ CO2(g)] is constant
(a) (i), (ii) and (iv) (b) (i), (ii) and (iii)
PH2 (PH 2 ) 4 (c) (ii) and (iv) (d) (i), (iii) and (iv)
2
(a) (b)
PH2 O (PH 2O ) 4 31. In a reversible chemical reaction having two reactants in
2
equilibrium, if the concentration of the reactants are doubled
then the equilibrium constant will
(PH 2 ) 4 [Fe 3O 4 ] [Fe 3O 4 ]
(c) (d) (a) Also be doubled (b) Be halved
4
(PH 2O ) [Fe] [Fe] (c) Become one-fourth (d) Remain the same
25. For the reaction C(s) + CO2(g) 2CO(g), Kp = 63 atm at 1000 32. On doubling P and V with constant temperature the
K. If at equilibrium : Pco = 10 Pco2, then the total pressure equilibrium constant will
of the gases at equilibrium is (a) remain constant (b) become double
(a) 6.3 atm (b) 6.93 atm (c) become one-fourth (d) None of these
(c) 0.63 atm (d) 0.693 atm 33. If for the reaction
26. The rate constant for forward and backward reaction of N2 + 3H2 2NH3, H = –92.38KJ/mole than what
hydrolysis of ester are 1.1 10 2 and 1 .5 10 3 per minute happens if the temperature is increased?
respectively. Equilibrium constant for the reaction
(a) Reaction proceed forward
CH3COOC2 H 5 H CH 3COOH C 2 H 5OH is
(b) Reaction proceed backward
(a) 4.33 (b) 5.33 (c) No effect on the formation of product
(c) 6.33 (d) 7.33
(d) None of these
27. Value of KP in the reaction
34. If Kc is in the range of ………………… appreciable
MgCO3(s) MgO(s) CO 2(g ) is concentrations of both reactants and products are present.
(a) KP = PCO (a) 10–4 to 104 (b) 10 – 3 to 103
2
PCO 2 PMgO (c) 10+3 to 10 – 3 (d) 10 – 5 to 103
(b) KP PCO 2 35. The reaction quotient (Q) for the reaction
PMgCO 3
N 2 ( g ) 3H 2 ( g ) 2NH 3 ( g )
PCO 2 PMgO
(c) KP
PMgCO 3 [ NH 3 ]2
is given by Q . The reaction will proceed from
PMgCO 3 [ N 2 ][ H 2 ] 3
(d) KP
PCO 2 PMgO right to left if
28. Which of the following is an example of homogeneous (a) Q = 0 (b) Q = Kc
equilibrium ? (c) Q < Kc (d) Q > Kc
where Kc is the equilibrium constant
(a) 2SO2 (g) O2 (g) 2SO3 (g)
36. The reaction quotient Q is used to
(b) (a) predict the extent of a reaction on the basis of its
C(s) H 2 O(g) CO(g) H 2 (g)
magnitude
(c) CaCO3 (s) CaO(s) CO 2 (g) (b) predict the direction of the reaction
(c) calculate equilibrium concentrations
(d) NH 4 HS(s) NH3 (g) H 2S(g) (d) calculate equilibrium constant
29. Unit of equilibrium constant for the given reaction is 37. The correct relationship between free energy change in a
Ni(s) + 4CO(g) Ni (CO)4(g) reaction and the corresponding equilibrium constant, Kc is
(a) (mol/l) –3 (b) (mol/l)3 (a) G = RT ln Kc (b) – G = RT ln Kc
(c) (mol/l) –4 (d) (mol/l)4 (c) G° = RT ln Kc (d) – G° = RT ln Kc
EBD_7207
114 EQUILIBRIUM
43. Which of the following reaction will be favoured at low
38. Usin g the equation ( K e G /RT ), the reaction
pressure ?
spontaneity can be interpreted in terms of the value of G°
is/are (a) H 2 I 2 2HI

(a) If G > 0, then – G /RT is positive, and e G /RT> 1 (b) N 2 3H 2 2NH3


making K > 1, which implies a spontaneous reaction or
(c) PCl5 PCl3 Cl 2
the reaction which proceeds in the forward direction
to such an extent that the products are present (d) N2 O2 2NO
predominantly. 44. The equilibrium which remains unaffected by pressure
(b) If G > 0, then – G /RT is negative, and e G /RT change is
< 1 making K < 1, which implies a non-spontaneous (a) N 2 (g) O2 (g) 2NO(g)
reaction or a reaction which proceeds in the forward
(b) 2SO2 (g) O2 (g) 2SO3 (g)
direction to such a small degree that only a very minute
quantity of product is formed. (c) 2O3 (g) 3O2 (g)
(c) Both (a) and (b)
(d) 2NO 2 (g) N2 O4 (g)
(d) None of the above
39. Which of the following relation represents correct relation 45. Suitable conditions for melting of ice :
between standard electrode potential and equilibrium (a) high temperature and high pressure
constant? (b) high temperature and low pressure
nFE (c) low temperature and low pressure
I. log K (d) low temperature and high pressure
2.303 RT
46. In which of the following reactions, the equilibrium remains
nFE
unaffected on addition of small amount of argon at constant
II. K e RT volume?
nFE (a) H2 (g) + I2 (g) 2HI (g)
III. log K
2.303 RT (b) PCl5 (g) PCl3 (g) + Cl2 (g)
nFE
IV. log K 0.4342 (c) N2 (g) + 3H2 (g) 2NH3 (g)
RT (d) The equilibrium will remain unaffected in all
Choose the correct statement(s). the three cases.
(a) I, II and III are correct
47. Le-Chatelier principle is not applicable to
(b) II and III are correct
(a) H 2 ( g ) I 2 ( g ) 2HI( g )
(c) I, II and IV are correct
(d) I and IV are correct (b) Fe(s) S(s) FeS(s)
40. According to Le-chatelier’s principle, adding heat to a solid (c) N 2 ( g ) 3H 2 ( g ) 2NH 3 ( g )
liquid equilibrium will cause the
(a) temperature to increase (d) N 2 ( g ) O2 ( g ) 2NO( g )
(b) temperature to decrease 48. In an equilibrium reaction if temperature increases
(c) amount of liquid to decrease (a) equilibrium constant increases
(d) amount of solid to decrease (b) equilibrium constant decreases
41. Which one of the following information can be obtained on (c) any of the above
the basis of Le Chatelier principle? (d) no effect
(a) Dissociation constant of a weak acid 49. In a two-step exothermic reaction
(b) Entropy change in a reaction A2(g) + B2(g) 3C(g) D(g)
(c) Equilibrium constant of a chemical reaction Step 1 Step 2
(d) Shift in equilibrium position on changing value of a Steps 1 and 2 are favoured respectively by
constraint (a) high pressure, high temperature and low pressure, low
42. For the manufacture of ammonia by the reaction temperature
N2 3H2 2NH3 2 kcal (b) high pressure, low temperature and low pressure, high
the favourable conditions are temperature
(a) Low temperature, low pressure and catalyst (c) low pressure, high temperature and high pressure, high
(b) Low temperature, high pressure and catalyst temperature
(c) High temperature, low pressure and catalyst (d) low pressure, low temperature and high pressure, low
(d) High temperature, high pressure and catalyst temperature
EQUILIBRIUM 115
50. What happens when an inert gas is added to an equilibrium 59. Which of the following can act as both Bronsted acid and
keeping volume unchanged? Bronsted base?
(a) More product will form (a) Na2CO3 (b) OH–
(b) Less product will form (c) HCO3 – (d) NH3
(c) More reactant will form
(d) Equilibrium will remain unchanged 60. Conjugate acid of NH 2 is :
51. In a vessel N2, H2 and NH3 are at equilibrium. Some helium (a) NH4+ (b) NH3
gas is introduced into the vessel so that total pressure (c) NH2 (d) NH
increases while temperature and volume remain constant. 61. Among boron trifluoride, stannic chloride and stannous
According to Le Chatelier’s principle, the dissociation of chloride, Lewis acid is represented by
NH3 (a) only stannic chloride
(a) increases (b) decreases (b) boron trifluoride and stannic chloride
(c) remains unchanged (d) equilibrium is disturbed (c) boron trifluoride and stannous chloride
52. Effect of a catalyst on a equilibrium reaction. (d) only boron trifluoride
(i) A catalyst increases the rate of the chemical reaction 62. Which of the following molecules acts as a Lewis acid ?
by making available a new low energy pathway for the (a) (CH3)2 O (b) (CH3)3 P
conversion of reactants to products.
(c) (CH3)3 N (d) (CH3)3 B
(ii) It increases the rate of forward and reverse reactions
63. Which one of the following molecular hydrides acts as a
that pass through the same transition state and does
Lewis acid?
not affect equilibrium.
(iii) It lowers the activation energy for the forward and (a) NH3 (b) H2O
reverse reactions by exactly the same amount. (c) B2H6 (d) CH4
Which of the above statement(s) is/are correct ? 64. Which of these is least likely to act as Lewis base?
(a) Only (i) (b) (i) and (ii) (a) F– (b) BF3
(c) (i), (ii) and (iii) (d) (ii) and (iii) (c) PF3 (d) CO
53. Which of the following is/are electrolytes? 65. Which one of the following is the correct statement ?
(i) Sugar solution (ii) Sodium chloride (a) HCO3– is the conjugate base of CO32–.
(iii) Acetic acid (iv) Starch solution (b) NH2– is the conjugate acid of NH3.
(a) (i) and (iv) (b) (ii) and (iv) (c) H2SO4 is the conjugate acid of HSO4–.
(c) (ii) and (iii) (d) (i) and (iii) (d) NH3 is the conjugate base of NH2–.
54. The geometry of hydronium ion is 66. Water is well known amphoprotic solvent. In which chemical
(a) tetrahedral (b) linear reaction water is behaving as a base?
(c) trigonal pyramidal (d) trigonal planer (a) H 2SO 4 H 2 O H 3 O + HSO 4 –
55. Which of the following statements are correct regarding
Arrhenius theory of acid and base? (b) H 2O H2O H3O + OH –
(a) This theory was applicable to only aqueous solutions (c) H2O NH2 – NH3 OH –
(b) This theory was applicable to all solutions (d) H 2 O NH 3 NH 4 OH –
(c) This theory could not explain the basicity of
substances like ammonia which do not possess a 67. An acid/ base dissociation equilibrium is dynamic involving
hydroxyl group a transfer of proton in forward and reverse directions. Now, with
(d) Both (a) and (c) passage of time in which direction equilibrium is favoured ?
56. Would gaseous HCl be considered as an Arrhenius acid ? (a) in the direction of stronger base and stronger acid
(a) Yes (b) in the direction of formation of stronger base and
(b) No weaker acid
(c) Not known (c) in the direction of formation of weaker base and weaker
(d) Gaseous HCl does not exist acid
57. A base, as defined by Bronsted theory, is a substance which (d) in the direction of formation of weaker base and
can stronger acid
(a) lose a pair of electrons 68. Three reactions involving H2PO4– are given below:
(b) donate protons (i) H3PO4 + H2 H3O+ + H2PO4–
(c) gain a pair of electrons (ii) H2PO4 + H2O HPO42– + H3O+

(d) accept protons (iii) H2PO4– + OH– H3PO4 + O2–


58. BF3 is an acid according to In which of the above does H 2 PO 4 act as an acid ?
(a) Arrhenius concept (b) Bronsted-Lowry concept (a) (ii) only (b) (i) and (ii)
(c) Lewis Concept (d) Both (b) and (c) (c) (iii) only (d) (i) only
EBD_7207
116 EQUILIBRIUM
69. The value of the ionic product of water 82. Which of the following has highest pH ?
(a) depends on volume of water
M M
(b) depends on temperature (a) KOH (b) NaOH
4 4
(c) changes by adding acid or alkali
(d) always remains constant M M
70. A base when dissolved in water yields a solution with a (c) NH 4 OH (d) Ca (OH) 2
4 4
hydroxyl ion concentration of 0.05 mol litre–1. The solution 83. A weak acid, HA, has a Ka of 1.00 × 10–5. If 0.100 mole of
is this acid dissolved in one litre of water, the percentage of
(a) basic (b) acidic acid dissociated at equilbrium is closest to
(c) neutral (d) either (b) or (c) (a) 1.00% (b) 99.9%
71. pH scale was introduced by : (c) 0.100% (d) 99.0%
(a) Arrhenius (b) Sorensen 84. A monobasic weak acid solution has a molarity of 0.005 and
(c) Lewis (d) Lowry pH of 5. What is the percentage ionization in this solution?
72. pH of solution is defined by expression (a) 2.0 (b) 0.2
1 (c) 0.5 (d) 0.25
(a) log [H ] (b) log
H 85. Calculate the pH of a solution obtained by diluting 1 mL of
0.10 M weak monoacidic base to 100 mL at constant
1 1 temperature if Kb of the base is 1 × 10–5 ?
(c) (d) (a) 8 (b) 9
log [H ] log [H ]
(c) 10 (d) 11
73. The pH of a 10–3 M HCl solution at 25°C if it is diluted 1000
86. The ionisation constant of an acid, Ka, is the measure of
times, will be –
strength of an acid. The K a values of acetic acid,
(a) 3 (b) zero hypochlorous acid and formic acid are 1.74 × 10–5, 3.0 × 10–
(c) 5.98 (d) 6.02 8 and 1.8 × 10–4 respectively. Which of the following orders
74. How many litres of water must be added to 1 litre an aqueous of pH of 0.1 mol dm–3 solutions of these acids is correct?
solution of HCl with a pH of 1 to create an aqueous solution (a) acetic acid > hypochlorous acid > formic acid
with pH of 2 ?
(b) hypochlorous acid > acetic acid > formic acid
(a) 0.1 L (b) 0.9 L
(c) formic acid > hypochlorous acid > acetic acid
(c) 2.0 L (d) 9.0 L
(d) formic acid > acetic acid > hypochlorous acid
75. What is the approximate pH of a 1 × 10–3 M NaOH solution?
87. The first and second dissociation constants of an acid H2 A
(a) 3 (b) 11
are 1.0 × 10–5 and 5.0 × 10–10 respectively. The overall
(c) 7 (d) 1 × 10–11 dissociation constant of the acid will be
76. Calculate the pOH of a solution at 25°C that contains
(a) 0.2 × 105 (b) 5.0 × 10–5
1× 10– 10 M of hydronium ions, i.e. H3O+. 15
(c) 5.0 × 10 (d) 5.0 × 10–15.
(a) 4.000 (b) 9.0000
88. Equimolar solutions of HF, HCOOH and HCN at 298 K have
(c) 1.000 (d) 7.000
the values of Ka as 6.8 × 10–4 and 4.8 × 10–9 respectively.
77. The pH value of a 10 M solution of HCl is
What is the observed trend of dissociation constants in
(a) less than 0 (b) equal to 0 successive stages ?
(c) equal to 1 (d) equal to 2
+
(a) HF > HCN > HCOOH (b) HF > HCOOH > HCN
78. What is the H ion concentration of a solution prepared by
(c) HCN > HF > HCOOH (d) HCOOH > HCN > HF
dissolving 4 g of NaOH (Atomic weight of Na = 23 amu) in
89. At 25 C, the dissociation constant of a base, BOH, is
1000 ml?
1.0 10 12 . The concentration of hydroxyl ions in
(a) 10–10 M (b) 10–4 M
–1 0.01 M aqueous solution of the base would be
(c) 10 M (d) 10–13 M
(a) 1.0 10 5 mol L 1 (b) 1.0 10 6 mol L 1
79. Calculate the pOH of a solution at 25°C that contains 1× 10– 10
(c) 2.0 10 6 mol L 1 (d) 1.0 10 7 mol L 1
M of hydronium ions, i.e. H3O+.
90. Which of the following pKa value represents the strongest
(a) 4.000 (b) 9.0000
acid ?
(c) 1.000 (d) 7.000
(a) 10–4 (b) 10–8
80. The pH of 0.005 molar solution of H 2 SO 4 is approximately: (c) 10 –5 (d) 10–2
(a) 0.010 (b) 1 91. The dissociation constant of two acids HA1 and HA2 are
(c) 2 (d) 0.005 3.14 × 10– 4 and 1.96 × 10– 5 respectively. The relative
81. Which solution has pH equal to 10 ? strength of the acids will be approximately
(a) 10–4 M KOH (b) 10–10 M KOH (a) 1 : 4 (b) 4 : 1
–10
(c) 10 M HCl (d) 10–4 M HCl (c) 1 : 16 (d) 16 : 1
EQUILIBRIUM 117
92. Given (b) solubility product to AgI is less than that of NaI
Ka (c) of common ion effect
HF H 2 O H 3O F
(d) AgI forms complex with NaI
Kb 100. When sodium acetate is added to an aqueous solution of
F H 2O HF OH
acetic acid :
Which of the following reaction is correct (a) The pH of the solution decreases
1 (b) The pH of the solution increases
(a) Kb = Kw (b) Kb
Kw (c) The pH of the solution remains unchanged
Ka (d) An acid salt is produced
(c) Ka × Kb = Kw (d) Kw 101. Which of the following statements about pH and H+ ion
Kb
concentration is incorrect?
93. At 298K a 0.1 M CH3COOH solution is 1.34% ionized. The
ionization constant Ka for acetic acid will be (a) Addition of one drop of concentrated HCl in NH4OH
(a) 1.82 × 10– 5 (b) 18.2 × 10 –5 solution decreases pH of the solution.
(c) 0.182 × 10 –5 (d) None of these (b) A solution of the mixture of one equivalent of each of
CH3COOH and NaOH has a pH of 7
94. For dibasic acid correct order is
(c) pH of pure neutral water is not zero
(a) K a1 K a 2 (b) K a1 K a 2 (d) A cold and concentrated H2SO4 has lower H+ ion
(c) K a1 K a 2 (d) not certain concentration than a dilute solution of H2SO4
95. For a polybasic acid, the dissociation constants have a 102. H2 S gas when passed through a solution of cations
different values for each step, e.g., containing HCl precipitates the cations of second group
of qualitative analysis but not those belonging to the
H 3A H H2A ; K K a1 fourth group. It is because
H 2A H HA 2 ; K Ka2 (a) presence of HCl decreases th e sulph ide ion
concentration.
HA 2 H A3 ; K Ka3 (b) solubility product of group II sulphides is more than
that of group IV sulphides.
What is the observed trend of dissociation constants in
(c) presence of HCl increases th e sulphide ion
successive stages ?
concentration.
(a) K a1 K a 2 K a 3 (b) K a1 K a 2 K a 3 (d) sulphides of group IV cations are unstable in HCl.
(c) K a1 Ka2 K a3 (d) K a1 Ka2 Ka3 103. A salt ‘X’ is dissolved in water of pH = 7. The salt is made
resulting solution becomes alkaline in nature. The salt is
96. K a1 , Ka2 and K a3 are the respective ionisation constants made
for the following reactions. (a) A strong acid and strong base
H 2S H HS (b) A strong acid and weak base
(c) A weak acid and weak base
HS H S2 (d) A weak acid and strong base
H 2S 2H S2 104. Aqueous solution of ferric chloride is acidic due to
(a) ionization (b) polarization
The correct relationship between K a1 , Ka2 and Ka3 is
(c) dissociation (d) hydrolysis
(a) K a3 K a1 K a2 (b) K a3 K a1 K a2 105. The pKa of a weak acid, HA, is 4.80. The pKb of a weak base,
BOH, is 4.78. The pH of an aqueous solution of the
(c) K a3 K a1 K a2 (d) K a3 K a1 / K a2 correspondng salt, BA, will be
97. Cationic hydrolysis gives the following solution: (a) 9.58 (b) 4.79
(a) acidic (b) basic (c) 7.01 (d) 9.22
(c) neutral (d) amphoteric 106. The pKa of a weak acid (HA) is 4.5. The pOH of an aqueous
98. In qualitative analysis, in III group NH4Cl is added before buffer solution of HA in which 50% of the acid is ionized is
NH4OH because (a) 7.0 (b) 4.5
(a) to increase the concentration of NH4+ions (c) 2.5 (d) 9.5
(b) to increase concentration of Cl– ions 107. A buffer solution is prepared in which the concentration of
(c) to reduce the concentration of OH– ions NH3 is 0.30M and the concentration of NH4+ is 0.20 M. If
(d) to increase concentration of OH– ions the equilibrium constant, Kb for NH3 equals 1.8 × 10–5,
99. The solubility of AgI in NaI solution is less than that in pure what is the pH of this solution ? (log 2.7 = 0.433).
water because : (a) 9.08 (b) 9.43
(a) the temperature of the solution decreases (c) 11.72 (d) 8.73
EBD_7207
118 EQUILIBRIUM
108. What is [H+]
in mol/L of a solution that is 0.20 M in (a) Solubility product of As2S3 is less than that of ZnS
CH3COONa and 0.10 M in CH3COOH ? Ka for CH3COOH (b) Enough As+3 are present in acidic medium
= 1.8 × 10–5. (c) Zinc salt does not ionise in acidic medium
4 5 (d) Solubility product changes in presence of an acid
(a) 3.5 × 10 (b) 1.1 × 10
119. Solid Ba(NO 3 ) 2 is gradually dissolved in a
(c) 1.8 × 10 5 (d) 9.0 × 10 6 1.0 × 10–4 M Na2CO3 solution. At which concentration of
109. Which of the following pairs constitutes a buffer? Ba2+, precipitate of BaCO3 begins to form ? (Ksp for BaCO3
(a) NaOH and NaCl (b) HNO3 and NH4NO3 = 5.1 × 10–9)
(c) HCl and KCl (d) HNO2 and NaNO2 (a) 5.1 × 10–5 M (b) 7.1 × 10–8 M
110. Buffer solutions have constant acidity and alkalinity (c) 4.1 × 10 M–5 (d) 8.1 × 1–7 M
because 120. Solubility product of silver bromide is 5.0 × 10–13. The
(a) these give unionised acid or base on reaction with quantity of potassium bromide (molar mass taken as
added acid or alkali. 120 g mol–1) to be added to 1 litre of 0.05 M solution of
(b) acids and alkalies in these solutions are shielded from silver nitrate to start the precipitation of AgBr is
attack by other ions. (a) 1.2 × 10–10 g (b) 1.2 × 10–9 g
(c) they have large excess of H+ or OH– ions (c) 6.2 × 10–5 g (d) 5.0 × 10–8 g
(d) they have fixed value of pH 121. At 25°C, the solubility product of Mg(OH) 2 is
111. The buffering action of an acidic buffer is maximum when 1.0 × 10–11. At which pH, will Mg2+ ions start precipitating
its pH is equal in the form of Mg(OH) 2 from a solution of
(a) 5 (b) 7 0.001 M Mg2+ ions?
(c) 1 (d) pKa (a) 9 (b) 10
112. When a buffer solution, sodium acetate and acetic acid is (c) 11 (d) 8
diluted with water :
(a) Acetate ion concentration increases STATEMENT TYPE QUESTIONS
(b) H+ ion concentration increases 122. Read the following statements carefully and choose the
(c) OH– ion conc. increases correct answer
(d) H+ ion concentration remains unaltered (i) Water and water vapour remain in equilibrium position
113. The product of ionic concentration in a saturated solution at atmospheric pressure (1.013 bar) and at 100°C in a
of an electrolyte at a given temperature is constant and is closed vessel.
known as (ii) The boiling point of water is 100°C at 1.013 bar pressure
(a) Ionic product of the electrolyte (iii) Boiling point of the liquid depends on the atmospheric
(b) Solubility product pressure.
(c) Ionization constant (iv) Boiling point depends on the altitude of the place; at
(d) Dissociation constant high altitude the boiling point increases.
114. The Ksp for Cr(OH)3 is 1.6 × 10–30. The solubility of this (a) (i), (ii) and (iv) are correct
compound in water is : (b) (i), (iii) and (iv)
(a) 4 30 (b) 4 30 (c) (i), (ii) and (iii) are correct
1.6 10 1.6 10 / 27
(d) only (iii) is correct
(c) 1.6 10 30/ 27 (d) 1.6 10 30 123. You must have seen that when a soda water bottle is opened,
115. At 25°C, the solubility product of Mg(OH)2 is 1.0 × 10–11. some of the carbon dioxide gas dissolved in it fizzes out
At which pH, will Mg2+ ions start precipitating in the form rapidly. There is equilibrium between the molecules in the
of Mg(OH)2 from a solution of 0.001 M Mg2+ ions? gaseous state and the molecules dissolved in the liquid
(a) 9 (b) 10 under pressure i.e.,
(c) 11 (d) 8 CO2(gas) CO2(in solution)
116. pH of a saturated solution of Ba(OH)2 is 12. The value of Which of the following statements is/are correct regarding
solubility product (Ksp) of Ba(OH)2 is : this?
(a) 3.3 × 10– 7 (b) 5.0 × 10–7 (i) The phenomenon arises due to difference in solubility
(c) 4.0 × 10 –6 (d) 5.0 × 10–6 of carbon dioxide at different pressures.
117. If s and S are respectively solubility and solubility product (ii) This equilibrium is governed by Henry's law.
of a sparingly soluble binary electrolyte then : (iii) The amount of CO2 gas dissolved in liquid increases
(a) s = S (b) s = S2 with decrease of temperature.
(iv) The amount of CO2 gas dissolved in liquid decreases
1
(c) s S1/ 2 (d) s S with increase of temperature.
2 (a) (i), (ii) and (iv) are correct
118. Why only As+3 gets precipitated as As2S3 and not Zn+2 as (b) (i) , (iii) and (iv)
ZnS when H2S is passed through an acidic solution (c) (i), (ii) and (iii) are correct
containing As+3 and Zn+2? (d) only (iii) is correct
EQUILIBRIUM 119
124. Identify the CORRECT statements below regarding chemical 128. Which of the following statement(s) is/are correct ?
equilibrium: (i) G is negative, then the reaction is spontaneous and
(i) All chemical reactions which are in equilibrium are proceeds in the forward direction.
irreversible. (ii) G is positive, then reaction is non-spontaneous
(ii) Equilibrium is achieved when the forward reaction rate (iii) G is 0, then reaction is at equilibrium
equals the reverse reaction rate. (a) (i), (ii) and (iii) are correct
(iii) Equilibrium is achieved when the concentrations of (b) (i) and (ii)
reactants and product remain constant. (c) (ii) and (iii) are correct
(iv) Equilibrium is dynamic in nature (d) only (iii) is correct
(a) (i), (ii) and (iv) are correct 129. Read the following statements and choose the correct option
(b) (i), (ii), (iii) and (iv) are correct (i) Most of the acids taste sour
(c) (i), (ii) and (iii) are correct (ii) Acids turns blue litmus paper into red
(d) only (ii) is correct (iii) Bases turns red litmus paper blue
(iv) Bases taste bitter and feel soapy
125. Nobel gas is added to a reaction at equilibrium involving
(a) (i), (ii) and (iv) are correct
gaseous reactant and gaseous product.
(b) (i), (iii) and (iv)
Which of the following statement is true for above reaction?
(c) (i), (ii) and (iii) are correct
Statement 1 : Reaction will proceed forward, as total
(d) All statements are correct
pressure has increased due to addition of Nobel gas.
130. Which of the following statements are correct ?
Statement 2 : Reaction will proceed backward, if Nobel gas
(i) Strong acids have very weak conjugate bases
react with reactant.
(ii) Weak acids have very strong conjugate bases
(a) Statement 1 and 2 are both correct. (iii) Strong bases have strong conjugate acids
(b) Statement 1 is correct but statement 2 is incorrect. (iv) Weak bases have weak conjugate acids
(c) Statement 1 is incorrect but statement 2 is correct. (a) (i) and (ii) (b) (i) and (iii)
(d) Statement 1 and 2 both are incorrect. (c) (ii) and (iv) (d) (iii) and (iv)
126 Read the following statements and choose the correct 131. Which of the following statement(s) is/are correct ?
option. (i) Water has ability to act both as an acid and a base
(i) The value of equilibrium constant is independent of (ii) In pure water one H2O molecule donate proton and
initial concentrations of the reactants and products. acts an acid and another water molecule accepts a
(ii) Equilibrium constant is temperature dependent proton and acts as a base.
(iii) The equilibrium constant for the reverse reaction is (a) Both (i) and (ii) (b) Neither (i) nor (ii)
equal to the inverse of the equilibrium constant for the (c) Only (i) (d) Only (ii)
forward reaction. 132. Which of the following statements are correct ?
(iv) The equilibrium constant for the reverse reaction is (i) Ionic product of water (Kw) = [H+] [OH–] = 10–14M2
equal to the equilibrium constant for the forward (ii) At 298K [H+] = [OH–] = 10–7
reaction. (iii) Kw does not depends upon temperature
(a) (i), (ii) and (iv) are correct (iv) Molarity of pure water = 55.55M
(b) (i), (iii) and (iv) (a) (i), (ii) and (iii) (b) (i), (ii) and (iv)
(c) (i), (ii) and (iii) are correct (c) (i) and (iv) (d) (ii) and (iii)
(d) only (iii) is correct 133. Read the following statements and choose the correct option
127. Read the following statements and choose the correct option (i) Ka (ionization constant) is a measure of the strength
(i) The numerical value of the equilibrium constant for a of the acids
reaction indicates the extent of the reaction. (ii) Smaller the value of Ka, the stronger is the acid
(ii) An equilibrium constant give information about the (iii) Ka is a dimensionless quantity
rate at which the equilibrium is reached. (a) Statements (i) and (ii) are correct
(iii) If Kc > 103, products predominate over reactants, i.e., (b) Statements (ii) and (iii) are correct
if Kc is very large, the reaction proceeds nearly to (c) Statements (i), (ii) and (iii) are correct
completion. (d) Statements (i) and (iii) are correct
(iv) If Kc < 10–3, reactants predominate over products, 134. Which of the following statement(s) is/are correct ?
i.e., if Kc is very small, the reaction proceeds rarely. (i) In a tribasic acid 2nd and 3rd (Ka , Ka ) ionization
2 3
(a) (i), (ii) and (iv) are correct constants are smaller than the first ionisation (Ka )
1
(b) (i) , (iii) and (iv) (ii) It is difficult to remove a positively charged proton
from a negative ion due to electrostatic force.
(c) (i), (ii) and (iii) are correct
(a) Both (i) and (ii) (b) Neither (i) nor (ii)
(d) only (iii) is correct
(c) Only (i) (d) Only (ii)
EBD_7207
120 EQUILIBRIUM
135. Which of the following statements are correct ? 139. Match the columns :
(i) The extent of dissociation of an acid depends on the Column-I Column-II
strength and polarity of the H –– A bond (where A is (A) N2(g) + 3H2(g) (p) n>0
an electronegative element.)
2NH3(g) (t = 300ºC)
(ii) As the strength of H–A bond increases, the energy
required to break the bond decreases. (B) PCl5(g) PCl3(g) (q) KP < KC
(iii) As the electronegativity difference between the atoms + Cl2(g) (t = 50ºC)
H and A increases, acidity increases (C) C(s) + H2O(g) (r) KP not defined
(a) (i) and (ii) (b) (ii) and (iii) CO(g) + H2(g)
(c) (i) and (iii) (d) (i), (ii) and (iii) (D) CH3COOH(l) + C2H5OH(l) (s) n=1
CH3COOC2H5(l)
MATCHING TYPE QUESTIONS
+ H2O(l)
136. Match the columns (a) A – (q), B – (p), C – (s), D – (r)
Column-I Column-II (b) A – (p), B – (q), C – (r), D – (s)
(A) H2O (l) H2O (vap) (p) rate of melting = rate of (c) A – (r), B – (p), C – (s), D – (q)
freezing (d) A – (s), B – (q), C – (p), D – (r)
(B) I2 (solid) (q) rate of evaporation 140. Match the columns :
I2 (vapour) = rate of condensation Column-I Column-II
(C) Ice water (r) rate of sublimation= rate (A) For the equilibrium NH4I(s) (p) Forward shift
of condensation NH3(g) + HI(g),
(a) A – (p), B – (q), C – (r) if pressure is increased
(b) A – (r), B – (q), C – (p) at equilibrium
(c) A – (p), B – (r), C – (q) (B) For the equilibrium (q) No change
(d) A – (q), B – (r), C – (p)
N2 + 3H2 2NH3
137. Match the Column-I with Column-II and mark the appropriate
If volume is increased
choice.
at equilibrium
Column-I Column-II
(C) For the equilibrium (r) Backward shift
(A) Liquid Vapour (p) Saturated solution
(B) Solid Liquid (q) Boiling point H2O(g) + CO(g)
(C) Solid Vapour (r) Sublimation point H2(g) + CO2 (g) inert gas is
(D) Solute (s) Solute (s) Melting point added at constant pressure
(solution) at equilibrium
(a) A – (p) ; B – (r) ; C – (q) ; D – (s) (D) For the equilibrium (s) More N2 and H2 is
(b) A – (q) ; B – (s) ; C – (r) ; D – (p) PCl5 PCl3 + Cl2 formed.
(c) A – (s) ; B – (q) ; C – (p) ; D – (r) what happens if more
(d) A – (r) ; B – (s) ; C – (q) ; D – (p) PCl5 is added
138. Match the columns. (a) A – (p), B – (q), C – (r), D – (s)
Column-I Column-II (b) A – (r), B – (s), C – (q), D – (p)
(Reactions) (Effect of increase in (c) A – (s), B – (p), C – (q), D – (r)
pressure)
(d) A – (q), B – (s), C – (r), D – (p)
(A) H2(g) + I2(g) 2HI(g) (p) Reaction proceed 141. Match the columns
backward. Column-I Column-II
1 (A) Qc < Kc, (p) Net reaction goes from
(B) CO(g) + O (g) (q) No effect on right to left.
2 2
CO2(g) reaction. (B) Qc > Kc, (q) Net reaction goes from
left to right.
(C) N2O4(g) 2NO2(g) (r) Reaction proceed
(C) Qc = Kc, (r) No net reaction occurs.
forward
(a) A – (p), B – (q), C – (r)
(a) A – (q), B – (r), C – (p)
(b) A – (r), B – (q), C – (p) (b) A – (r), B – (q), C – (p)
(c) A – (p), B – (r), C – (q) (c) A – (p), B – (r), C – (q)
(d) A – (q), B – (p), C – (r) (d) A – (q), B – (p), C – (r)
EQUILIBRIUM 121
142. Match the columns
CRITICAL THINKING TYPE QUESTIONS
Column-I Column-II
(A) Hydrochloric acid (p) Lemon and orange 148. N2(g) + 3H2(g) 2NH3(g), K1 (1)
(B) Acetic acid (q) Tamarind paste. N2(g) + O2(g) 2NO(g), K2 (2)
(C) Citric and ascorbic (r) Digestive juice
1
acids H2(g) + O (g) H2O(g), K3 (3)
2 2
(D) Tartaric acid (s) Constituent of
vinegar The equation for the equilibrium constant of the reaction
(a) A – (q), B – (r), C – (p), D – (s) 5
2NH3(g) + O2 (g) 2NO(g) + 3H2O(g), (K4) in terms
(b) A – (r), B – (s), C – (p), D – (q) 2
(c) A – (s), B – (p), C – (q), D – (r) of K1, K2 and K3 is :
(d) A – (r), B – (p), C – (s), D – (q)
143. Match the columns K1.K 2 K1.K 23
(a) K3 (b)
Column-I Column-II K2
(A) HClO4 (p) Strong base
(B) HNO2 (q) Strong acid K 2 .K 33
(C) NH2– (r) Weak base (c) K1 K2 K3 (d)
K1
(D) HSO4 – (s) Weak acid
(a) A – (s), B – (q), C – (p), D – (r) 149. Two equilibria, AB A B and
(b) A – (q), B – (s), C – (p), D – (r) AB B AB2 are simultaneously maintained in a
(c) A – (r), B – (p), C – (q), D – (s) solution with equilibrium constants, K1 and K2 respectively.
(d) A – (s), B – (q), C – (p), D – (r) The ratio of [A+] to [AB2–] in the solution is
(a) directly proportional to [B–]
ASSERTION-REASON TYPE QUESTIONS (b) inversely proportional to [B–]
Directions : Each of these questions contain two statements, (c) directly proportional to the square of [B–]
Assertion and Reason. Each of these questions also has four (d) inversely proportional to the square of [B–]
alternative choices, only one of which is the correct answer. You 150. Equilibrium constant (K) for the reaction
have to select one of the codes (a), (b), (c) and (d) given below. Ni(s) + 4CO(g) Ni(CO)4(g) can be written in terms of
(a) Assertion is correct, reason is correct; reason is a correct
(1) Ni(s) + 2CO2 (g) + 2C(s) Ni(CO)4(g);
explanation for assertion.
(b) Assertion is correct, reason is correct; reason is not a equilibrium constant = K1.
correct explanation for assertion (2) CO2(g) + C(s) 2CO(g);
(c) Assertion is correct, reason is incorrect equilibrium constant = K2.
(d) Assertion is incorrect, reason is correct. What is the relation between K, K2 and K2 ?
144. Assertion : Kp can be less than, greater than or equal to Kc. (a) K = (K1)/(K2)2 (b) K = (K1 . K2)
Reason : Relation between Kp and Kc depends on the (c) K = (K1) (K2) 2 (d) K = K1/K2
change in number of moles of gaseous reactants and
151. K1, K2 and K3 are the equilibrium constants of the following
products ( n).
reactions (I), (II) and (III) respectively:
145. Assertion : If a volume is kept constant and an inert gas
such as argon is added which does not take part in the (I) N2 + 2O2 2NO2
reaction, the equilibrium remains undisturbed. (II) 2NO2 N2 + 2O2
Reason : It is because the addition of an inert gas at constant
volume does not change the partial pressure or the molar 1
(III) NO2 N 2 + O2
concentrations of the substance involved in the reaction. 2
146. Assertion : Buffer system of carbonic acid and sodium The correct relation from the following is
bicarbonate is used for the precipitation of hydroxides of
third group elements. 1 1 1 1
(a) K1 (b) K1 2
Reason : It maintains the pH to a constant value, about 7.4. K2 K3 K2 K3
147. Assertion : Addition of silver ions to a mixture of aqueous
sodium chloride and sodium bromide solution will first 1
precipitate AgBr rather than AgCl. (c) K1 K2 K3 (d) K1 K3
K2
Reason : Ksp of AgCl > Ksp of AgBr.
EBD_7207
122 EQUILIBRIUM
152. For the following three reactions a, b and c, equilibrium are in the ratio of 9 : 1. If degree of dissociation of X and A
constants are given: be equal, then total pressure at equilibrium (1) and (2) are in
(i) CO(g) + H 2O(g) CO 2 (g) + H 2 (g); K1 the ratio :
(a) 3 : 1 (b) 1 : 9
(ii) CH4 (g) + H2O(g) CO(g) + 3H2 (g); K 2 (c) 36 : 1 (d) 1 : 1
(iii) CH4 (g) 2H2O(g) CO2 (g) 4H2 (g);K3 159. The dissociation equilibrium of a gas AB2 can be represented
as :
(a) K1 K 2 K3 (b) K 2 K3 K1
2AB2 (g) 2AB(g) B2 (g)
(c) K3 = K1 K2 (d) K3 .K 23 K12
The degree of dissociation is ‘x’ and is small compared to 1.
153. The value of equilibrium constant of the reaction The expression relating the degree of dissociation (x) with
1 1 equilibrium constant Kp and total pressure P is :
HI g H 2 (g) I 2 is 8.0
2 2 (a) (2Kp/P) (b) (2Kp/P) 1/3
The equilibrium constant of the reaction (c) (2Kp/P) 1/2 (d) (Kp/P)
H2 g I2 (g) 2HI(g) will be: 160. On increasing the pressure, the gas phase reaction proceed
forward to re-establish equilibrium, as predicted by applying
1 1 the Le Chatelier’s principle. Consider the reaction.
(a) (b)
16 64 N 2 (g) 3H 2 (g) 2NH3 (g)
1 Which of the following is correct, if the total pressure at
(c) 16 (d)
8 which the equilibrium is established, is increased without
154. For the reversible reaction, changing the temperature?
N2(g) + 3H2(g) 2NH3(g) at 500°C, the value of Kp is (a) K will remain same
(b) K will decrease
1.44 10 5 when partial pressure is measured in
(c) K will increase
atmospheres. The corresponding value of K C , with
(d) K will increase initially and decrease when
concentration in mole litre–1, is
pressure is very high
5 5
1.44 10 1.44 10 161. The exothermic formation of ClF3 is represented by the
(a) 2
(b) 2 equation :
0.082 500 8.314 773
Cl 2 (g) 3F2 (g) 2ClF3 (g) ;
5 5
1.44 10 1.44 10
(c) (d) H = – 329 kJ
0.082 773 2 0.082 773 2
Which of the following will increase the quantity of ClF3 in
155. Two moles of PCl5 were heated in a closed vessel of 2L. At
an equilibrium mixture of Cl 2 , F2 and ClF3 ?
equilibrium 40% of PCl5 is dissociated into PCl3 and Cl2.
The value of equilibrium constant is (a) Adding F2
(a) 0.53 (b) 0.267 (b) Increasing the volume of the container
(c) 2.63 (d) 5.3 (c) Removing Cl2
156. PCl5 is dissociating 50% at 250°C at a total pressure of (d) Increasing the temperature
P atm. If equilibrium constant is Kp, then which of the 162. When hydrochloric acid is added to cobalt nitrate solution
following relation is numerically correct ? at room temperature, the following reaction takes place out
(a) Kp = 3P (b) P = 3Kp the reaction mixture becomes blue. On cooling the mixture it
2K P 2P becomes pink. On the basis of this information mark the
(c) P = (d) Kp =
3 3 correct answer.
157. For the decomposition of the compound, represented as 3
CO H 2 O 6 (aq) 4Cl (aq)
NH 2 COONH 4 ( s ) 2NH 3 ( g ) CO 2 ( g )
(pink)
the Kp = 2.9 × 10–5 atm3.
2
If the reaction is started with 1 mol of the compound, the COCl4 (aq) 6H 2 O(l )
total pressure at equilibrium would be : (blue)
(a) 1.94 × 10–2 atm (b) 5.82 × 10–2 atm (a) H > 0 for the reaction
–2
(c) 7.66 × 10 atm (d) 38.8 × 10–2 atm (b) H < 0 for the reaction
158. The values of Kp1 and Kp2 for the reactions (c) H = 0 for the reaction
X Y Z ...(1) (d) The sign of H cannot be predicted on the
basis of this information.
and A 2B ...(2)
EQUILIBRIUM 123
163. In HS–, I–,
RNH2 and NH3, order of proton accepting 172. The dissociation constants for acetic acid and HCN at 25°C
tendency will be are 1.5 × 10–5 and 4.5 × 10–10 respectively. The equilibrium
(a) I– > NH3 > RNH2 > HS– constant for the equilibrium
(b) HS– > RNH2 > NH3 > I– CN– + CH3COOH HCN + CH3COO– would be:
(c) RNH2 > NH3 > HS– > I– (a) 3.0 × 10– 5 (b) 3.0 × 10– 4
(d) NH3 > RNH2 > HS– > I– (c) 3.0 × 10 4 (d) 3.0 × 105
164. Which equilibrium can be described as an acid-base reaction 173. If degree of dissociation of pure water at 100°C is
using the Lewis acid-base definition but not using the 1.8 × 10–8, then the dissociation constant of water will be
Bronsted-Lowry definition? (density of H2O = 1 gm/cc)
(a) 2NH3 + H2SO4 2NH4+ + SO42– (a) 1 × 10–12 (b) 1 × 10–14
(c) 1.8 × 10 –12 (d) 1.8 × 10–14
(b) NH3 + CH3COOH NH4+ + CH3COO–
174. Ionisation of weak acid can be calculated by the formula
(c) H2O + CH3COOH H3O+ + CH3COO–
Ka 100
(d) [Cu(H2O)4]2– + 4 NH3 [Cu(NH3)4]2+ + 4H2O (a) 100 (b) ( pK a pH )
c 1 10
165. Equal volumes of three acid solutions of pH 3, 4 and 5 are (c) Both (a) and (b) (d) None of these
mixed in a vessel. What will be the H+ ion concentration in 175. Equimolar solutions of the following were prepared in water
the mixture ? separately. Which one of the solutions will record the highest
(a) 1.11 × 10–4 M (b) 3.7 × 10–4 M pH ?
(c) 3.7 × 10– 3 M (d) 1.11× 10–3 M (a) SrCl2 (b) BaCl2
–10
166. The pH of 10 M NaOH solution is nearest to: (c) MgCl2 (d) CaCl2
(a) 6 (b) – 10 176. Solubility product constant (Ksp) of salts of types MX, MX2
(c) 4 (d) 7 and M3X at temperature T are 4.0 × 10–8, 3.2 × 10–14 and
167. 100 mL of 0.04 N HCl aqueous solution is mixed with 100 mL 2.7 × 10–15, respectively. Solubilities (mol dm–3) of the salts
of 0.02 N NaOH solution. The pH of the resulting solution at temperature 'T' are in the order –
is: (a) MX > MX2 > M3X (b) M3X > MX2 > MX
(a) 1.0 (b) 1.7 (c) MX2 > M3X > MX (d) MX > M3X > MX2
(c) 2.0 (d) 2.3 177. In qualitative analysis, the metals of Group I can be separated
168. Equal volumes of three acid solutions of pH 3, 4 and 5 are from other ions by precipitating them as chloride salts. A
mixed in a vessel. What will be the H+ ion concentration in solution initially contains Ag+ and Pb2+ at a concentration
the mixture ? of 0.10 M. Aqueous HCl is added to this solution until the
(a) 1.11 × 10–4 M (b) 3.7 × 10–4 M Cl– concentration is 0.10 M. What will the concentrations
of Ag+ and Pb2+ be at equilibrium?
(c) 3.7 × 10– 3 M (d) 1.11× 10–3 M
(Ksp for AgCl = 1.8 × 10–10, Ksp for PbCl2 = 1.7 × 10–5)
169. At 100°C the Kw of water is 55 times its value at 25°C. What
(a) [Ag+] = 1.8 × 10–7 M ; [Pb2+] = 1.7 × 10–6 M
will be the pH of neutral solution? (log 55 = 1.74)
(b) [Ag+] = 1.8 × 10–11 M ; [Pb2+] = 8.5 × 10–5 M
(a) 6.13 (b) 7.00
(c) [Ag+] = 1.8 × 10–9 M ; [Pb2+] = 1.7 × 10–3 M
(c) 7.87 (d) 5.13
(d) [Ag+] = 1.8 × 10–11 M ; [Pb2+] = 8.5 × 10–4 M
170. Ionisation constant of CH3 COOH is 1.7 × 10 –5 if
178. The solubility product (Ksp) of the following compounds
concentration of H+ ions is 3.4 × 10–4M, then find out initial
are given at 25°C.
concentration of CH3COOH molecules
Compound Ksp
(a) 3.4 × 10–4M (b) 3.4 × 10–3M
–3 AgCl 1.1 × 10–10
(c) 6.8 × 10 M (d) 6.8 × 10–4M
AgI 1.0 × 10–16
171. Values of dissociation constant, Ka are given as follows :
PbCrO4 4.0 × 10–14
Acid Ka Ag2CO3 8.0 × 10–12
HCN 6.2 × 10–10 The most soluble and least soluble compounds are
HF 7.2 × 10–4 respectively.
HNO2 4.0 × 10–4 (a) AgCl and PbCrO4 (b) AgI and Ag2CO3
Correct order of increasing base strength of the base CN–, (c) AgCl and Ag2CO3 (d) Ag2CO3 and AgI
_
F– and NO 2 will be : 179. What is the molar solubility of Fe(OH)3 if
(a)
_
(b)
_ Ksp = 1.0 ×10–38 ?
F CN NO2 NO2 CN F
_ _ (a) 3.16×10–10 (b) 1.386×10–10
(c) F NO2 CN (d) NO2 F CN –9
(c) 1.45×10 (d) 1.12×10–11
EBD_7207
124 EQUILIBRIUM

FACT / DEFINITION TYPE QUESTIONS


[AB]2
16. (c) A2 + B2 2AB Kc
1. (c) 2. (c) 3. (b) [A 2 ][B2 ]
4. (d) The time taken for complete evaporation depends on
(i) the nature of the liquid, (ii) the amount of the liquid (2.8 10 3 ) 2 (2.8) 2
and (iii) the temperature. Kc 3 3
0.62
3 10 4.2 10 3 4.2
5. (c) When the watch glass is open to the atmosphere, the
rate of evaporation remains constant but the molecules 17. (c) A+ B C +D
are dispersed into large volume of the room. As a At eqb. x x 2x 2x
consequence the rate of condensation from vapour to
liquid state is much less than the rate of evaporation. 2x.2x
Kc = 4
6. (c) Ice melts with a reduction in volume. So Increase in x.x
pressure shifts equilibrium to water side, result in
melting of ice according to Lechatelier's principle. 18. (a) For A B C, n 1 2 1
7. (a) A reaction is said to be in equilibrium when rate of
n 1
forward reaction is equal to the rate of backward mol mol
Unit of K c
reaction. litre litre
8. (c)
= Litre mole–1
9. (b) The reaction mixtures starting either with H2 or D2
reach equilibrium with the same composition, except P 2 CO 4 4
that D2 and ND3 are present instead of H2 and NH3. 19. (c) Kp = ;K p 8; C(s) 1;
PCO 2 2
10. (a) Equilibrium can be attained by either side of the
reactions of equilibrium. The concentration of solids and liquids are taken as
11. (d) According to the idea of dynamic equilibrium there is unity.
possibility of formation of all product. 20. (d) 2C(s) +O 2 ( g ) 2 CO2 ( g )
12. (b) At equilibrium, the rate of forward and backward
n =2–1=+1
reactions is equal.
Kc and Kp are not equal.
13. (a) For reation (1)
21. (b) For a gaseous phase reaction Kp and Kc are related as
[NO]2 ng
K1 = Kp K c ( RT )
[N 2 ][O2 ]
For the given reaction,
and for reaction (2) 1
CO(g) + O2 ( g ) CO2(g)
[N 2 ]½ [O 2 ]½ 1 2
K2 = therefore K1 =
[NO] K 22 1
ng = 1– (1 + 0.5) = – 0.5 or
2
14. (c) N2 + 3H2 2NH3
1
K = [NH3]2 / [N2] [H2]3 .... (i) 2
Kp = Kc ( RT )
1 3
N2 H2 NH3 1
2 2 Kp
or = ( RT ) 2
K ' [NH3 ]/[N 2 ]1/ 2 [H 2 ]3/ 2 .... (ii) Kc
Dividing equation (i) by equation (ii), we get K' = 22. (c) Using the relation KP = KC. (RT) n, we get
K KP
( RT ) n
15. (b) Given reaction, 2A B C KC
[B][C]
Kc KP
[A]2 Thus K will be highest for the reaction having
C
3 3
2 10 3 10
Kc =6 highest value of n.
(10 3 ) 2
EQUILIBRIUM 125
The n values for various reactions are Since [CaCO3 (s)] and [CaO(s)] are both constant,
therefore modified equilibrium constant for the thermal
1 1
(a) n 1– 1 – decomposition of calcium carbonate will be
2 2 Kć = [CO2(g)]
(b) n = 2 – (1 + 1) = 0 K p = [p CO2(g)]
(c) n = (1 +1) – 1= 1 31. (d) Equilibrium constant (K) is independent of
(d) n = (2 + 4) – (7 + 2) = – 3 concentrations of reactions and products.
Thus maximum value of n = 1 32. (a) Equilibrium constant is not effected by change in
n conditions like P and V. These changes can change
23. (c) As Kp = Kc RT g
only the time required to attain equilibrium.
Here ng = 1
33. (b) Reaction proceed forward according to Le-chatelier’s
So, Kp = Kc when RT = 1
principle.
Thus T = 12.5 K
34. (b)
24. (b) 3Fe(s) + 4H2O (steam) Fe3O4 (s) + 4 H2 (g) 35. (d) For reaction to proceed from right to left
(p H 2 ) 4 Q K c i.e the reaction will be fast in backward
Kp = 4 only gaseous products and reactants. direction i.e r b > rf.
(p H 2 O )
36. (b) The equilibrium constant helps in predicting the
25. (b) C(s) + CO2(g) 2CO(g) direction in which a given reaction will proceed at any
Apply law of mass action, stage. For this purpose, we calculate the reaction
quotient Q. The reaction quotient Q (Qc with molar
(PCO )2 (10PCO2 )2
KP or 63 concentration and Qp with partial pressures) is defined
PCO2 PCO2 in the same way as the equilibrium constant Kc except
that the concentrations in Qc are not necessary
(Given KP = 65) and PCO = 10PCO2 equilibrium values.
37. (d)
100(PCO2 ) 2 38. (c) Both (a) and (b) are correct for the equation,
or 63 or 63 100 PCO2
PCO2 K = e G /RT
39. (c) G = –2.303 RT log K
63
PCO 2 0.63 atm –nFE° = –2.303 RT log K
100
nFE (I)
log K
PCO 10PCO2 = 10 × 0.63 = 6.3 atm 2.303 RT
nFE
Ptotal = PCO2 PCO = 0.63 + 6.3 = 6.93 atm. 0.4342 ........ (i)
RT
26. (d) Rate constant of forward reaction (Kf) = 1.1 × 10–2 and
rate constant of backward reaction (Kb) = 1.5 × 10–3 nFE
ln K
per minute. RT
2 nFE
Equilibrium constant (Kc) Kf 1.1 10 ....... (ii)
3
7.33 K e RT
Kb 1.5 10
40. (d) Solid Liquid
27. (a) MgCO3 (s) MgO(s) CO 2 (g) It is an endothermic process. So when temperature is
MgO & MgCO3 are solid and they donot exert any raised, more liquid is formed. Hence adding heat will
pressure and hence only pressure exerted is by CO2. shift the equilbrium in the forward direction.
Therefore K P PCO2 41. (d) According to Le-chatelier's principle" whenever a
constraint is applied to a system in equilibrium, the
28. (a) All the reactants and products are in same physical
system tends to readjust so as to nullify the effect of
state.
the constraint.
1 42. (b) The most favourable conditions are :
[Ni(CO) 4 ] mol l
29. (a) K (mol l 1) 3 (i) High pressure ( n < 0)
[CO]4 (mol l 1 )4 (ii) Low temperature (Exothermic reaction)
30. (b) For the reaction (iii) Catalyst Fe is presence of Mo.
CaCO3 (s) CaO (s) + CO2 (g) 43. (c) As in this no. of moles are increasing hence low
On the basis of the stoichiometric equation, we can pressure will favour the forward direction.
write, n = (1 + 1) – 1 = 1
Kc =[CaO(s)] [CO2(g)/[CaCO3(s)] 44. (a) For n = 0, no effect of pressure.
EBD_7207
126 EQUILIBRIUM
45. (a) Melting of ice involve absorption of heat i.e 60. (b) Because NH3 after losing a proton (H+) gives NH2–
Endothermic hence high temperature favour the NH3 + H2O NH 2– H 3O
process.
(Conjugate acid-base pair differ only by a proton)
Further for a given mass volume of water is less than
61. (c) Lewis acid is that compound which have electron
ice thus high pressure favour the process. High
deficiency. eg. BF3, SnCl2.
pressure and high temperature convert ice into liquid.
62. (d) (CH3)3 B - is an electron deficient, thus behave as a
46. (d)
lewis acid.
47. (b) Le chatelier principle is not applicable to solid-solid
equilibrium. 63. (c) Boron in B2H6 is electron deficient
48. (c) Effect of increase of temperature on equilibrium 64. (b) BF3 is Lewis acid (e– pair acceptor)
constant depends on the fact that whether the reaction 65. (c) HSO4– accepts a proton to form H2SO4.
is exothermic, or endothermic. If the reaction is Thus H2SO4 is the conjugate acid of HSO4–.
exothermic, it is favoured by low temperature and if HSO4
H
H 2SO 4
the reaction is endothermic, it is favoured by high base conjugate acid
temperature. of HSO 4

49. (d) A2 (g) B2(g) 3C(g) D(g) 66. (a) Bronsted base is a substance which accepts proton.
step-1 step-2 In option (a), H2O is accepting proton, i.e., acting as a
since the steps 1 and 2 are exothermic hence low base.
temprature will favour both the reactions. In step - 1 67. (c)
moles are increasing hence low pressure will favour it. 68 (a) (i) H 3PO 4 H 2O H 3O H 2PO 4
In step 2 moles are decreasing, hence high pressure acid1 base 2 acid 2 base1
will favour it.
(ii) H 2PO4 H2O HPO 4 H 3O
50. (a) On adding inert gas at constant volume the total acid1 base 2 base1 acid 2
pressure of the system is increased, but the partial
pressure of each reactant and product remains the (iii) Hbase
2 PO 4 OH
acid 2
H 3 PO 4 O
acid1 base 2
1
same. Hence no effect on the state of equilibrium.
51. (c) The backward reaction is not favoured at high pressure. Hence only in (ii) reaction H2PO4– is acting as an acid.
52. (c) A catalyst increases the rate of the chemical reaction 69. (b) The value of ionic product of water changes with the
by making available a new low energy pathway for the temperature.
conversion of reactants to products. It increases the 70. (a) Given : Hydroxyl ion concentration
rate of forward and reverse reaction that pass through [OH–] = 0.05 mol L–1. We know that
the same transition state and does not affect 14
[H ][OH ] 1 10
equilibrium.
14
Catalyst lowers the activation energy for the forward or [H ] 1 10 2 10 13 mol L–1
and reverse reactions by exactly the same amount. 0.05
53. (c) An aqueous solution of sodium chloride is comprised We also know that
entirely of sodium ions and chloride ions, while that of 13
acetic acid mainly contains unionized acetic acid pH log[H ] log [2 10 ]
molecules and only some acetate ions and hydronium = 13
log 2 log 10 log 2 ( 13) log 10
ions. This is because there is almost 100% ionization
in case of sodium chloride which is a strong electrolyte 0.3010 13.0000 12.6990.
as compared to less than 5% ionization of acetic acid Since the value of pH > 7, therefore the solution is
which is a weak electrolyte. basic.
54. (c) The hydronium ion has a trigonal pyramidal geometry 71. (b)
and is composed of three hydrogen atoms and one
1
oxygen atom. There is a lone pair of electrons on the 72. (b) pH = – log [H+] = log
oxygen giving it this shape. The bond angle between H
the atoms is 113°. 73. (c) On dilution [H+] = 10–6 M
= 10–6 mol
55. (d) Now dissociation of water cannot be neglected,
56. (b) According to Arrhenius, acids are those substances Total [H+] = 10–6 + 10–7 = 11 × 10–7
which give proton in aqueous solution, hence gaseous pH = –log [H+]
HCl is not an Arrhenius acid. = –log (11× 10–7) = 5.98
57. (d) Base accepts protons and acid donates protons. 74. (d) pH = 1 ; H+ = 10–1 = 0.1 M
58. (c) Lewis concept. pH = 2 ; H+ = 10–2 = 0.01 M
59. (c) H2CO3 H+ + HCO3– H+ + CO3–. M1 = 0.1 V1 = 1
HCO 3 can donate and accept H+. M2 = 0.01 V2 = ?
EQUILIBRIUM 127
From 83. (a) Given Ka = 1.00×10–5,
C= 0.100 mol
M1V1 = M2V2 for a weak electrolyte,
0.1 × 1 = 0.01 × V2 degree of dissociation
V2 = 10 litres
Volume of water added = 10 – 1 = 9 litres Ka 1 10 –5
( ) 10 2 1%
75. (b) Given [OH–] = 10–3 C 0.100
pOH = 3 84. (b) HA H+ + A–
pH + pOH = 14
pH = 14 – 3 = 11 [H ][A ]
Ka , [H+] = 10–pH
76. (a) Given [H3O+] = 1 × 10–10 M [HA]
at 25º [H3O+] [OH–] = 10–14 [H+] = 10–5 ; and at equilibrium [H+] = [A– ]
14
10 4
[OH ] 10 10 5 10 5
8
10 10 Ka 2 10
0.0015
OH OH
Now, [OH ] 10 p 10 4 10 p
Ka 2 10 8 6 3
pOH = 4 4 10 2 10
C .005
77. (a) Molarity (M) = 10M. HCl is a strong acid and it is
Percentage ionization = 0.2
completely dissociated in aqueous solutions as : HCl
(10) H+(10) + Cl–. 85. (c) M1V1 = M2V2
1 × 0.10 = M2 × 100
So, for every moles of HCl, there is one H+. Therefore
[H+] = [HCl] or [H+] = 10. M2 = 0.001 = 10–3
pH = – log[H+] = – log [10] = – 1. BOH B OH
4 C 0 0
78. (d) No. of moles of NaOH = 0.1 C(1 ) C C
40
[Molecular weight of NaOH = 40] C C
Kb
No. of moles of OH– = 0.1 C(1 )

0.1 Kb = C 2 ( 1 1)
Concentration of OH– = 1 litre 0.1Mole / L

14
Kb / C
As we know that, [ H ] [OH ] 10
13 Kb
[H ] 10 ( OH – 10 –1 ) [OH ] C C K bC
C
79. (a) Given [H3 O+] = 1 × 10–10 M
at 25º [H3O+] [OH–] = 10–14 10 5
10 3
10 4

10 14 pH + pOH = 14
4
[OH ] 10 pH = 14 – 4 = 10
10
10
86. (d)
OH OH
Now, [OH ] 10 p 10 4 10 p 87. (d) H 2A H+ + HA-
pOH = 4
80. (c) H2SO4 2H+ + SO42– [H ][HA ]
K1 = 1.0 × 10–5 = (Given)
Given concentration of H2SO4 = 0.005 M [H 2 A]
[H+] = 0.005 × 2 = 0.01 = 10–2
HA H A
pH = – log [H+] = – log 10–2 = 2
81. (a) pOH = – log [OH–] 10 [H ][A ] (Given)
pH + pOH = 14 K2 5.0 10
[HA ]
For 10–4 KOH, [OH–] = 10–4
pOH = – log [OH–] = – log 10–4 = 4
[H ]2 [A 2 ]
pH = 14 – pOH = 10 K K1 K2
82. (d) Among M/4 KOH, M/4 NaOH, M/4 NH4OH and [H 2 A]
M/4 Ca(OH)2, Ca(OH)2 furnishes highest number of = (1.0 × 10–5) × (5 × 10–10) = 5 × 10–15
OH– ions ( Ca(OH)2 Ca2+ + 2OH–).
88. (b) Acidic strength Ka
So pH of M/4 Ca(OH)2 is highest.
EBD_7207
128 EQUILIBRIUM
89. (d) Given Kb = 1.0 × 10–12 101. (b) CH3COOH is weak acid while NaOH is strong base, so
[BOH] = 0.01 M [OH] = ? one equivalent of NaOH can not be neutralized with
one equivalent of CH3COOH. Hence the solution of one
BOH B OH equivalent of each does not have pH value as 7. Its pH
t o c 0 0
teq c(1 x) cx cx will be towards basic side as NaOH is a strong base
hence conc. of OH– will be more than the conc. of H+.
c 2 x2 cx 2 0.01x 2 102. (a) IVth group needs higher S2– ion concentration. In
Kb 1.0 × 10–12 = presence of HCl, the dissociation of H2 S decreases
c (1 x) (1 x ) (1 x )
hence produces less amount of sulphide ions due
On calculation, we get, x = 1.0 × 10–5 to common ion effect, thus HCl decreases the
Now, [OH–] = cx = 0.01 × 10–5 = 1 × 10–7mol L–1 solubility of H2 S which is sufficient to precipitate
90. (b) pKa = –log Ka IInd group radicals.
Smaller the value of pKa, stronger will be acid 103. (d) A salt of strong base with weak acid undergoes anionic
Acid having pKa value of 10–8 is strongest acid. hydrolysis to give basic solution.
4 104. (d) Ferric chloride is the salt of a strong acid and a weak
Ka1 3.14 10
91. (b) 1
4 :1 base, hence on hydrolysis it yields a mixture of weak
K a2 5
2 1.96 10 base and strong acid
FeCl3 3H2O Fe OH 3 3HCl
[H3O ][F ] weak base strong acid
92. (c) Ka ...(i)
[HF] Due to this, there is predominance of H+ ions in
solution, hence the solution is acidic.
[HF][OH ] 105. (c) In aqueous solution BA(salt) hydrolyses to give
Kb ...(ii)
[F ] BA + H2O BOH + HA
From (i) and (ii), KaKb = [H3O+][OH–]=Kw Base acid
(ionic product of water) Now pH is given by
2 1 1 1
1.34 pH pK w pKa pK b
93. (a) K c 2 0.1 1.8 10 5 2 2 2
100
94. (b) In polyprotic acids the loss of second proton occurs substituting given values, we get
much less readily than the first. Usually the Ka values 1
pH = (14 4.80 4.78) 7.01
for successive loss of protons from these acids differ 2
by at least a factor of 10–3 i.e., K a1 K a2 salt
106. (d) For acidic buffer pH = pKa + log
H+ + HX- (K a1 )
acid
H2 X
H+ + X2- (K a 2 )
A
HX- or pH pK a log
HA
95. (a) The values of dissociation constants for successive
stages decrease. Given pKa = 4.5 and acid is 50% ionised.
96. (a) [HA] = [A–] (when acid is 50% ionised)
97. (a) NH 4 Cl H 2 O NH 4OH HCl pH = pKa + log 1
pH = pKa = 4.5
or
pOH = 14 – pH = 14 – 4.5 = 9.5
Cl H 2O HCl H i.e., acidic 107. (b) Given [NH3] = 0.3 M, [NH4+] = 0.2 M, Kb = 1.8 × 10–5 .
98. (c) Due to common ion effect addition of NH4Cl in group
[salt]
(III) suppresses the ionisation of NH4OH with the pOH = pK b log [pKb = –log Kb;
result concentration of OH– decreases. [base]
99. (c) Solubility of weak electrolyte decreases in solvent pKb = –log 1.8 × 10–5]
having common ion. So solubility of AgI in NaI solution pKb = 4.74
is less than in pure water because of common ion effect. 0.2
100. (b) Dissociation of CH 3COOH is suppressed by the 4.74 log = 4.74 + 0.3010 – 0.4771 = 4.56
0.3
addition of sodium acetate (CH 3COONa ) due to pH = 14 – 4.56 = 9.436
common ion (CH3COO ) effect. The [ H ] decreases Salt
108. (d) pH p Ka+ log
raising the pH of the acid solution. Acid
Note : After the addition of CH3COONa to CH3COOH, Salt
a buffer solution is formed which has reserved pH log H log Ka – log
Acid
value.
EQUILIBRIUM 129
[OH–] = 10–2
Acid
log H = log Ka + log 2s = 10–2
Salt
10 2
Acid s
H = Ka 2
Salt Ksp = 4s3
0.15 3
= 1.8 × 10 = 9 × 10 6 M 10 2
0.2 = 4
2
109. (d) HNO2 is a weak acid and NaNO2 is salt of that weak
acid and strong base (NaOH). = 5 × 10–7
110. (a) Lets take an example of an acidic buffer CH3COOH 117. (c) Let binary electrolyte be AB
and CH3COONa.
AB A B
CH3COOH CH3COO – + H + ; s s s
CH3COONa CH3COO– + Na+ Hence, solubility product of AB
when few drops of HCl are added to this buffer, the H+ Ksp = [A+] [B–]
of HCl immediatly combine with CH3COO– ions to S = [s.] [s] s = S½
form undissociated acetic acid molecules. Thus there 118. (a) Ksp of As2 S3 is less than ZnS. In acid medium
will be no appreciable change in its pH value. Like ionisation of H2S is suppresed (common ion effect)
wise if few drops of NaOH are added, the OH – ions and Ksp of ZnS does not exceed.
will combine with H+ ions to form unionised water 119. (a) Given Na2CO3 = 1.0 × 10–4 M
molecule. Thus pH of solution will remain constant. [CO3– – ] = 1.0 × 10–4 M
111. (d) Buffering action is maximum when i.e. S = 1.0 × 10–4 M
[Salt] = [Acid] At equilibrium
i.e., pH = pKa [Ba++] [CO3– –] = Ksp of BaCO3
112. (d) pH or [H+] of a buffer does not change with dilution. 9
Ksp 5.1 10
113. (b) Solubility product is the product of ionic concentration [Ba++] = = 5.1 × 10–5 M
4
in a saturated solution of an electrolyte at a given [CO3 ] 1.0 10
temperature.
120. (b) AgBr Ag+ + Br-
114. (b) Cr(OH)3 (s) Cr 3 (aq.) 3OH (aq.)
s 3s Ksp = [Ag+] [Br–]
(s) (3s)3 = Ksp For precipitation to occur
Ionic product > Solubility product
27 S 4 K sp
K sp 5´10-13
[Br- ] = = = 10-11
K sp
1/ 4 30 1/ 4 [Ag + ] 0.05
1.6 10
s
27 27 i.e., precipitation just starts when 10–11 moles of KBr
is added to 1 AgNO3 solution
115. (b) Mg(OH) 2 Mg 2 OH Number of moles of Br– needed from
Ksp = [Mg++][OH–]2 KBr = 10–11
1.0 × 10–11 = 10–3 × [OH–]2 Mass of KBr = 10–11 × 120 = 1.2 × 10–9 g
121. (b) Mg(OH)2 Mg 2OH
10-11
[OH- ] = = 10-4 ++
Ksp = [Mg ][OH ] – 2
10-3
1.0 × 10–11 = 10–3 × [OH–]2
pOH = 4
pH + pOH = 14 10-11
[OH- ] = = 10-4
pH = 10 10-3
116. (b) Given pH = 12 pOH = 4
or [H+] = 10–12 pH + pOH = 14 pH = 10
Since, [H+] [OH–] = 10–14
14 STATEMENT TYPE QUESTIONS
10
[OH–] = = 10–2
10 12 122. (c) Boiling point depends on the altitude of the place; at
high altitude the boiling point decreases.
Ba(OH)2 Ba 2+ + 2OH 123. (a)
s 2s
EBD_7207
130 EQUILIBRIUM
124. (b) Chemical reactions which are in equilibrium are (B) As n < 0 therefore if V , P reaction will go in
reversible the direction in which more number of gaseous
125. (c) Since equilibrium constant is related to the partial moles are formed i.e. backward direction.
pressure of reactant and product therefore if nobel (C) As n = 0 hence no effect.
gas is added, no change is observed, Statement 1 is (D) If concentration of reactant is increased reaction
incorrect. will go in the forward direction.
If Nobel gas react with reactant, concentraction of 141. (d) 142. (b)
reactant will decrease and therefore reaction will 143. (b) HClO4 is a strong acid
proceed backword according to Le– chatelier ’s HNO2 is a weak acid.
principle. NH2– is a very good proton acceptor and thus, it is a
126. (c) Equilibrium constant is temperature dependent having base.
one unique value for a particular reaction represented H2SO4 is a strong acid hence its conjugate base (HSO4–)
by a balanced equation at a given temperature. will be a weak base.
127. (b) An equilibrium constant does not give any information
about the rate at which the equilibrium is reached. ASSERTION-REASON TYPE QUESTIONS
128. (a) 129. (d)
130. (a) As strong acid dissociate completely in water hence 144. (a) Kp = Kc(RT) n
resulting base formed would be very weak. On the 145. (a) If the volume is kept constant and an inert gas such as
other hand a weak acid is only partially dissociated in argon is added which does not take part in the reaction,
aqueous solution, hence resulting base formed would the equilibrium remains undisturbed. It is because the
be strong. addition of an inert gas at constant volume does not
131. (a) H2O(l) + H2O(l) H3O+(aq) + OH–(aq) change the partial pressure or the molar concentration
acid base conjugate acid conjugate base of the substance involved in the reaction. The reaction
132. (c) Kw depends upon temperature as it is an equilibrium quotient changes only if the added gas is reactant or
constant. product involved in the reaction.
133. (d) Larger the value of Ka, the stronger is the acid. 146. (d) In biological systems buffer system of carbonic acid
134. (a) and sodium bicarbonate is found in our blood. It
135. (c) Bond energy being directly related to bond strength maintains the pH of blood to a constant value of about
increases with increase in bond strength 7.4.
147. (a) Ionic product of AgBr is greater than that of AgCl in
MATCHING TYPE QUESTIONS comparison with there solubility product AgBr will
precipitate first rather than that of AgCl.
136. (d)
137. (b) (A) Liquid Vapour equilibrium exists at the CRITICAL THINKING TYPE QUESTIONS
boiling point.
(B) Solid Liquid equilibrium exists at the 148. (d) To calculate the value of K4 in the given equation we
melting point. should apply :
eqn. (2) + eqn.(3) × 3 – eqn. (1)
(C) Solid Vapour equilibrium exists at the
sublimation point. K 2 K33
(D) Solute Solute (solution) equilibrium exists hence K4 =
K1
in a saturated solution.
149. (d) Given,
138. (a) In case of A no. of moles of product and reactant are K1 1
same, in case of B no. of moles of reactant are greater AB A B
so reaction go forward, in case of C the no. of moles of [A ] [B ]
K1
product are greater than no. of moles of reactant. [AB]
139. (a) K2
(A) KP = KC (RT) n AB B AB 2
KP [AB2 ]
= (RT) n as n = –ve KP < KC K2
KC [AB] [B ]
(B) n > 0
Dividing K1 and K2, we get
(C) n = 2 – 1 = 1
(D) As the reaction is not containing any gaseous K1 [A ] [B ]2
K
component therefore KP is not defined for this. K2 [AB2 ]
140. (b)
(A) As n > 0 therefore if P , reaction will go in the [A ] K
backward direction. [AB2 ] [B ]2
EQUILIBRIUM 131

Ni(CO)4 [CO]2 Kp
150. (a) K1 = ; K2 = 154. (d) Kc
2
[CO 2 ] [CO2 ] (RT) n
[Ni(CO) 4 ] 1.44 10 5
K= (R in L. atm. K–1 mole–1).
[CO]4 (0.082 773) 2

[Ni(CO)4 ] [CO 2 ]
2 155. (b) PCl5 PCl3 + Cl2
K= × a (1-x) ax ax
[CO2 ]2 [CO]2
a = 2, x = 0.4, V = 2 L
K1 2 1 0.4
K PCl5 0.6 mol L 1
K 22 2
K1 2 0.4 1
151. (b) (I) N 2 2O 2 2NO 2 PCl3 Cl2 0.4 mol L
2
[NO 2 ]2
K1 ...(i) 0.4 0.4
[N 2 ][O 2 ]2 Kc 0.267
K2 0.6
(II) 2NO 2 N2 2O 2 156. (b) PCl5 PCl3 + Cl2
[N 2 ] [O2 ] 2 Moles at equilibrium
K2 ...(ii) 1 1 1
[NO 2 ]2
2 2 2
K3 1 Mole fraction at equilibrium
(III) NO2 N2 O2
2 1 1 1
[N 2 ]1/2 [O2 ] 3 3 3
K3
[NO 2 ] Partial pressure at equilibrium
2 [N 2 ][O 2 ]2 P P P
K3 ...(iii) 3 3 3
[NO2 ]2
P P
from equations (i), (ii) and (iii)
1 1 3 3 P
K1 Kp
K 2 ( K3 )2 P/3 3
152. (c) Reaction (c) can be obtained by adding reactions (a) 157. (b) NH 2 COONH 4 ( s ) 2NH 3 ( g ) CO 2 ( g )
and (b) therefore K3 = K1. K2 2
Hence (c) is the correct answer. PNH3 PCO2 2
153. (b) Given: Equilibrium constant (K1) for the reaction KP = PNH3 PCO2
PNH 2COONH 4 ( s)
K1 1 1
HI(g) H 2 (g) I 2 (g); K1 = 8; .........(i) As evident by the reaction, NH3 and CO2 are formed
2 2
in molar ratio of 2 : 1. Thus if P is the total pressure of
To find equilibrium constant for the following reaction
the system at equilibrium, then
H 2 (g) I2 (g) 2HI(g); K2 = ? .....(ii) 2 P 1 P
For this multiply (i) by 2, we get PNH3 PCO2
3 3
2HI(g) H 2 (g) I2 (g); K1 = 82 = 64 .....(iii) 2
[Note: When the equation for an equilibrium is 2P P 4 P3
KP =
multiplied by a factor, the equilibrium constant must 3 3 27
be raised to the power equal to the factor] Given KP = 2.9 × 10–5
Now reverse equation (iii), we get
4 P3
1 2.9 10 5
H 2 (g) I 2 (g) 2HI(g); K .....(iv) 27
64
[Note: For a reversible reaction, the equilibrium 5
2.9 10 27
constant of the backward reaction is inverse of the P3
4
equilibrium constant for the forward reaction.]
1
Equation (iv) is the same as the required equation (ii), 2.9 10 5
27 3
1 P = 5.82 × 10–2 atm
thus K2 for equation (ii) is i.e. option (b) is correct. 4
64
EBD_7207
132 EQUILIBRIUM
158. (c) Given reaction are Total number of moles
X Y Z ..... (i) = 2 (1–x) + 2x + x = (2 + x)
and A 2B ......(ii) 2(1 x)
PAB = P where P is the total pressure.
2 (2 x)
Let the total pressure for reaction (i) and (ii) be P1 and
P2 respectively, then 2x x
PAB = P , PB P
K P1 (2 x) 2 (2 x)
9
(given) Since x is very small so can be neglected in denominator
KP2 1 Thus, we get
After dissociation, PAB = (1 – x) × P PAB = x × P
2
X Y Z x
PB P
2 2
At equilibrium (1– )
2
[Let 1 mole of X dissociate with as degree of PAB PB2
dissociation ] Now, K P
2
Total number of moles = 1– + + PAB2
= (1+ ) x
(x)2 P 2 .P
1 = 2
Thus PX = . P1 ; PY = P1 ; (1 x) 2 P 2
1 1
x 3 .P3
= [ 1–x 1]
PZ = .P1 2 1 P2
1 1
x 3 .P 2.K p 2K p 3
= or x3 = or x =
K P1 .P1 . 2 P P
1 1 160. (a) Justification : According to Le-Chatelier’s principle, at
constant temperature, the equilibrium composition will
1 change but K will remain same.
P1 / .P1 ... (i)
1 161. (a) The reaction given is an exothermic reaction thus
Similarly for A 2B according to Lechatalier’s principle lowering of
temperature, addition of F2 and / or Cl2 favour the for
At equilibrium (1– ) 2
ward direction and hence the production of ClF3 .
We have,
162. (a)
2 P2
2
1 163. (c) Strong base has higher tendency to accept the proton.
K P2 / P2 ...(ii) Increasing order of base and hence the order of
1 1
accepting tendency of proton is
Dividing (i) by (ii), we get I HS NH3 RNH 2
2
K P1 .P1 K P1 1 P1
or . 164. (d) [Cu(H2O)4]2+ + 4NH3 [Cu(NH3)4]2+ + 4H2O
K P2 2 K P2 4 P2
4 .P2 involves lose and gain of electrons. H2O is coordinated
to Cu by donating electrons (LHS). It is then removed
1 P1 K P1 9 by withdrawing electrons.
or 9 .
4 P2 K P2 1 165. (b) [H3O]+ for a solution having pH = 3 is given by
[H3O]+ = 1×10–3 moles/litre
P1 36 [ [H3O]+ = 10–pH]
or or P1 : P2 = 36 : 1
P2 1 Similarly for solution having pH = 4,
[H3O]+ = 1 × 10–4 moles/ litre and for pH = 5
159. (b) For the reaction
[H3O+] = 1×10–5 moles/ litre
2AB2 (g) 2AB(g) B2 (g) Let the volume of each solution in mixture be IL, then
at equi 2(1 x) 2x x
total volume of mixture solution L = (1 + 1 + 1) L = 3L
2 Total [H3 O] + ion present in mixture solution
AB B2 (2x) 2 x = (10–3 + 10–4 + 10–5) moles
Kc or K c
AB2
2
{2(1 x)}2 Then [H3O]+ ion concentration of mixture solution
3
= x3 [(1–x) can be neglected in denominator (1– x) 1] 10 10 4 10 5 0.00111
= M= M
The partial pressure at equilibrium are calculated on 3 3
the basis of total number of moles at equilibrium. = 0.00037 M = 3.7 ×10–4 M.
EQUILIBRIUM 133

10 On taking log on both side


166. (d) Given concentration of NaOH = 10 M
– log [H+] = –log (55 × 10–14)1/2
NaOH Na OH
1
10 10 M
10 10 pH = – log 55 7 log10
2
10
[OH ] from NaOH = 10 pH = – 0.87 + 7
We have to consider dissociation of H2O = 6.13
7 – +
[OH ] from H 2 O = 10 170. (c) CH3COOH CH3COO + H
7 10
Total [OH ] = 10 10 [ CH 3COO ][ H ]
Ka
1001 [ CH 3COOH ]
= 10 7 (0.001 1) = 10 7
= 10 10
1001 Given that,
1000
pOH = – log [OH ] [CH 3 COO ] [H ] 3.4 10 4 M
= (log1001 10 10 ) = – 3.004 + 10 = 6.9996 Ka for CH3COOH = 1.7 × 10–5
CH3COOH is weak acid, so in it [CH3COOH] is equal
pH = 14 – pOH = 14 – 6.996 = 7.004
to initial concentration. Hence
pH of 10–10 M NaOH solution is nearest to 7.
167. (c) Number of meq. of the acid = 0.04 × 100 = 4 (3.4 10 4 )(3.4 10 4 )
1.7 10 5
Number of meq. of the base = 0.02 × 100 = 2 [CH 3 COOH ]
Number of meq. of the acid left on mixing = 4 – 2 = 2
Total volume of the solution = 200 mL 4 4
3.4 10 3.4 10
No. of meq of the acid present in 1000 mL of the [ CH 3 COOH ]
1.7 10 5
solution = 10
= 6.8 × 10–3M
or No. of eq. of the acid in 1000 mL of the solution
171. (c) Higher the value of Ka lower will be the value of pKa
10
= = 0.01 i.e. higher will be the acidic nature. Further since
1000 CN–, F– and NO2– are conjugate base of the acids
Since the acid is monobasic and completely ionises in HCN, HF and HNO2 respectively hence the correct
solution order of base strength will be
0.01 N HCl = 0.01 M HCl F– < NO2– < CN–
Thus [H ] = 0.01 ( stronger the acid weaker will be its conjugate base)
pH = – log (0.01) = – (– 2) = 2
172. (c) Given, CH3COOH CH3COO– + H+ ;
168. (b) [H3O]+ for a solution having pH = 3 is given by
[H3O]+ = 1×10–3 moles/litre Ka1 , = 1.5 × 10– 5 ....(i)
[ [H3O]+ = 10–pH]
Similarly for solution having pH = 4, HCN H++ CN–; K a 2 = 4.5 × 10–10
[H3O]+ = 1 × 10–4 moles/ litre and for pH=5 or H+ + CN– HCN;
[H3O+] = 1×10–5 moles/ litre
1 1
Let the volume of each solution in mixture be IL, then K 'a 2 ...(ii)
total volume of mixture solution L = (1 + 1 + 1) L =3L Ka 2 4.5 10 –10
Total [H3O]+ ion present in mixture solution From (i) and (ii), we find that the equilibrium constant
= (10–3 + 10–4 + 10–5) moles (Ka) for the reaction,
Then [H3O]+ ion concentration of mixture solution CN– + CH3COOH CH3COO– + HCN, is
3
10 10 4 10 5 0.00111 Ka K a1 K a' 2
= M= M
3 3
= 0.00037 M = 3.7 ×10–4 M. 1.5×10 –5 1
= –10
105 3.33 10 4
169. (a) Kw at 25°C = 1 × 10–14 4.5×10 3
At 25ºC
wt. of solute per litre of solution
Kw = [H+] [OH–] = 10–14 173. (d) As, molarity,
At 100°C (given) Mol. wt. of solute
Kw = [H+] [OH–] = 55 × 10–14 1000
Molarity of H2O = mole/litre
for a neutral solution 18
[H+] = [OH–] H2O H+ + OH–
[H ] = 55 × 10–14
+ 2
c (1 – c c
or [H+] = (55 × 10–14)1/2
pH = – log [H+] c 2
Thus, K a c 2 = 1.8 × 10–14
1
EBD_7207
134 EQUILIBRIUM
174. (c) For weak acid dissociation equilibria, degree of 1 1
é 3.2 ´10-14 ù 3
ú or éê 32 ´10-15 ùú 3
dissociation is given as :
Solubility of MX 2 = êê ú
êë 4 úû êë 4 úû
Ka Ka
% 100
c c 1
= éê8´10-15 ùú 3 or 2´10-5
ë û
[H ][A ] [H ]c [H ]
Also, K a 1
[HA] c (1 ) (1 )
é 2.7 ´10-15 ù 4
Solubility of M 3 X = êê ú
ú
log K a log H log
1 ëê 27 ûú
1
or pK a pH log
1 = éê10-16 ùú 4 or 10–4
ë û
Thus the solubilities are in the order MX > M3 X> MX2
1
pK a pH log i.e the correct answer is (d).
177. (c) Ksp = [Ag+] [Cl–]
1 1.8 × 10–10 = [Ag+] [0.1]
10pK a pH
[Ag+] = 1.8 × 10–9 M
Ksp = [Pb+2] [Cl–]2
1
or, 10pK a pH
1 1.7 × 10–5 = [Pb+2] [0.1]2
[Pb+2] = 1.7 × 10–3 M
1 178. (d) The solubility equilibrium for AgI is
pKa pH
[1 10 ] AgI (aq) Ag aq + I aq ;
175. (b) The highest pH will be recorded by the most basic Ksp = [Ag+][I–]
solution. The basic nature of hydroxides of alkaline Let solubility of AgI be S moles per litre,
earth metals increase as we move from Mg to Ba and [Ag+] = S, [I–] = S
thus the solution of BaCl2 in water will be most basic Ksp = [Ag+][I–]
and so it will have highest pH. 1 × 10–16 = (S) × (S) = S2
1
176. (d) MX M X (Where s is the solubility)
s s S = 1 10 16 2 1 10 8
On calculating solubility of all given compounds
Then Ksp s 2 or s K sp
Compound Solubility
® M 2+ + 2X -
Similarly for MX 2 ¾¾ AgCl 1 10 5

s 2s AgI 1 10 8

7
1 PbCrO4 2 10
K sp 3 Ag 2 CO3 1.26 10 4

K sp s (2 s ) 2 4 s3 or s
4
Ag2CO3 is most soluble and AgI is least soluble.
and for M 3 X 3M X 3 179. (b) Ksp = [Fe3+].[3OH–]
3s s So molar solubility of Fe3+ = S and [3OH–] = 3S
1 Fe(OH)3 Fe3 3OH
Ksp 4 [S] [3S]
Ksp (3s )3 s 27 s 4 or s 1.0 ×10–38 = [S] [3S]3
27
1.0 ×10–38 = S4 × 27
From the given values of Ksp for MX, MX2 and 1.0 10 38
M3 X, we can find the solubilities of those salts at S4 =
27
temperature, T.
S4 = 3.703×10–40
Solubility of MX = 4´10 -8 -4
= 2´10 40 1/ 4
S = 3.703 10 = 1.386 × 10–10
8
REDOX REACTIONS

6. Which of the following involves transfer of five


FACT / DEFINITION TYPE QUESTIONS
electrons ?
1. Which of the following process takes place in oxidation
(a) MnO Mn (b) CrO Cr
process?
(a) Addition of oxygen (b) Addition of hydrogen (c) MnO24 MnO 2 (d) Cr O Cr
(c) Removal of oxygen (d) Addition of chlorine
2. Given reaction, 7. Which reaction involves neither oxidation nor reduction?
2K4[Fe(CN)6] (aq) + H2O2 (aq) (a) CrO24- ¾¾ ® Cr2 O 72- (b) Cr ¾¾ ® CrCl3
2K3[Fe(CN)6](aq) + 2KOH(aq) (c) Na ¾¾ ® Na+ (d) 2S2 O 32- ¾¾
® S4 O 62-
The above given reaction is oxidation reaction due to 8. In the following reaction
(a) removal of a hydrogen from H2O2
4P 3KOH 3H 2 O 3KH 2 PO 2 PH 3
(b) addition of electropositive potassium to H2O2
(c) removal of electropositive element potassium from (a) phosphorus is both oxidised and reduced.
potassium ferrocyanide (K4[Fe(CN)6]) (b) only phosphorus is reduced.
(d) All of the above are the correct reasons. (c) phosphorus is not oxidised
3. In the reaction given below, identify the species undergoing (d) None of these
redox reaction 9. Which one of the following reaction involves oxidation-
2Na(s) + H2(g) 2NaH(s) reduction ?
(a) Na is reduced and hydrogen is oxidised (a) H2 Br2 2HBr
(b) Na is oxidised and hydrogen is reduced (b) NaBr HCl NaCl HBr
(c) Na undergoes oxidation and hydrogen undergoes (c) HBr AgNO 3 AgBr HNO 3
reduction
(d) Both (b) and (c) (d) 2 NaOH H 2 SO 4 Na 2SO 4 2H 2 O
4. The loss of electron is termed as 10. In reaction, 4Na + O2 2Na2O, sodium behaves as
(a) oxidation (b) reduction (a) oxidising agent (b) reducing agent
(c) combustion (d) neutralization (c) Both (a) and (b) (d) None of these
5. Which of the following is correct code for x and y in the 11. Zn 2 (aq.) 2e Zn(s) . This is
following reaction.
(a) oxidation (b) reduction
x (c) redox reaction (d) None of the above
12. Co(s) Cu 2 (aq) Co 2 (aq) Cu(s)
2Na(s) + S(s) Na +2 S2– (s)
The above reaction is
(a) oxidation reaction (b) reduction reaction
y
(c) redox reaction (d) None of these
(i) x = oxidation reaction, y = reduction reaction 13. One mole of N2H4 loses 10 moles of electrons to form a new
(ii) x = gain of two electrons, y = loss of two electrons, compound, y. Assuming that all nitrogen appear in the new
(iii) x = reduction reaction, y = oxidation reaction compound, what is the oxidation state of nitrogen in y (There
(iv) x = loss of two electrons, y = gain of two electrons is no change in the oxidation state of hydrogen )
(a) (i) and (ii) (b) (i) and (iv) (a) –1 (b) –3
(c) (ii) and (iii) (d) (iii) and (iv) (c) +3 (d) +5
EBD_7207
136 REDOX REACTIONS
14. When a strip of metallic zinc is placed in an aqueous 27. In which of the following compounds, iron has lowest
solution of copper nitrate the blue colour of the solution oxidation state?
disappear due to formation of (a) K 3[Fe(CN ) 6 ]
(a) Cu2+ (b) Zn 2+
(c) ZnS (d) Cus (b) K 4 [Fe(CN) 6 ]
15. The correct order of electron releasing tendency of the
(c) FeSO 4 .( NH 4 ) 2 SO 4 .6H 2 O
metals Cu, Zn and Ag is in the order:
(a) Cu > Zn > Ag (b) Zn > Ag > Cu (d) Fe(CO)5
(c) Ag > Zn > Cu (d) Zn > Cu > Ag 28. The oxidation state of osmium (Os) in OsO4 is
16. What is the oxidation number of elements in the free or (a) +7 (b) +6
in the uncombined state ? (c) +4 (d) +8
(a) +1 (b) 0
29. Which of the following transition metal has zero oxidation
(c) +2 (d) –1 state ?
17. In which of the following compounds oxygen has highest
(a) [Fe(CO)5] (b) NH2.NH2
oxidation state and in which it has lowest oxidation state?
(c) NOClO4 (d) CrO5
OF2, H2O2, KO2, O2F2
30. In which of the compounds does 'manganese' exhibit highest
(a) Highest = KO2, lowest = H2O2
oxidation number ?
(b) Highest = OF2, lowest = K2O2
(a) MnO2 (b) Mn 3O4
(c) Highest = OF2, lowest = KO2
(d) Highest = KO2, lowest = H2O2 (c) K2MnO4 (d) MnSO4
18. ‘Oxidation number of H in NaH, CaH2 and LiH, respectively 31. Among the following, identify the species with an atom in
is +6 oxidation state
(a) +1, +1, –1 (b) –1, +1, + 1 (a) MnO– (b) Cr ( CN ) –

(c) +1, + 1, + 1 (d) –1,–1, –1 (c) –


(d) CrO Cl
NiF
19. Which of the following is the correct representative of
stock notation for auric chloride? 32. In which of the following compounds the oxidation number
(a) Au(III)Cl3 (b) Au(II)Cl2 of carbon is not zero?
(c) Au(I)Cl2 (d) None of these (a) HCHO (b) CH3COOH
20. Oxidation number of N in HNO3 is (c) C12 H22 O11 (d) CH3CHO
(a) – 3.5 (b) + 3.5 33. In which of the following compounds, the oxidation number
(c) –5 (d) + 5 of iodine is fractional ?
21. In which of the following reactions, there is no change in (a) IF7 (b) I
valency ? (c) IF5 (d) IF3
(a) 4 KClO3 3KClO4 + KCl 34. A metal ion M3+ loses 3 electrons, its oxidation number will
(b) SO2 + 2H2S 2H2O + 3S be
(c) BaO2 + H2SO4 BaSO4 + H2O2 (a) +3 (b) +6
(d) 3 BaO + O2 2 BaO2. (c) 0 (d) –3
22. The oxidation number of chromium in potassium dichromate is 35. The correct name for NO2 using stock notation is
(a) + 6 (b) – 5 (a) nitrogen dioxide (b) nitrogen (iv) oxide
(c) – 2 (d) + 2 (c) nitrogen per oxide (d) All of these
23. The oxidation number of sulphur in S 8 , S2 F 2 , H2 S 36. The oxide, which cannot act as a reducing agent, is
respectively, are (a) NO2 (b) SO2
(a) 0, +1 and –2 (b) +2, +1 and –2 (c) CO2 (d) ClO2
(c) 0, +1 and +2 (d) –2, +1 and –2 37. The oxidation state of Fe in Fe3O4 is
24. Oxidation number of cobalt in K[Co(CO)4] is (a) + 3 (b) 8/3
(a) +1 (b) +3 (c) + 6 (d) + 2
(c) –1 (d) –3 38. In oxygen difluoride, the oxidation number of oxygen is
25. Oxidation number of nitrogen in (NH4)2SO4 is (a) – 2 (b) – 1
(a) –1/3 (b) –1 (c) + 2 (d) +1, – 2
(c) +1 (d) –3
39. Oxygen has an oxidation state of +2 in the compound
26. Oxidation number of carbon in CH2Cl2 is
(a) –4 (b) +4 (a) H 2 O2 (b) CO2
(c) 0 (d) –2 (c) H2 O (d) F2 O
REDOX REACTIONS 137
40. The number of electrons involved in the reduction of one 49. Which of the following do not show disproportionation
nitrate ion to hydrazine is reaction?
(a) 8 (b) 5 ClO –4 , F2, Cl2, ClO –2 , P4, S8, and ClO–
(c) 3 (d) 7
41. The average oxidation state of sulphur in Na2S4O6 is (a) ClO –2 , ClO –4 , and ClO–
(a) +2.5 (b) +2 (b) F2 only
(c) +3.0 (d) +3.5 (c) F2 and ClO –4
42. Which of the following species can function both as
oxidizing as well as reducing agent ? (d) ClO –4 only
(a) Cl– (b) ClO4– 50. Which one of the following reactions involves
(c) ClO – (d) MnO4 – disproportionation?
43. The oxidation number of an element in a compound is (a) 2H2SO4 + Cu CuSO4 + 2H2O + SO2
evaluated on the basis of certian rules. Which of the (b) As2O3 + 3H2S As2S3 + 3H2O
following rules is not correct in this respect? (c) 2KOH + Cl2 KCl + KOCl + H2O
(a) The oxidation number of hydrogen is always + 1. (d) Ca3P2 + 6H2O 3Ca(OH)2 + 2PH3
(b) The algebraic sum of all the oxidation numbers in a 51. The following species will not exhibit disproportionation
compound is zero. reaction
(c) An element in the free or the uncombined state bears
oxidation number zero. (a) ClO– (b) ClO 2
(d) In all its compounds, the oxidation number of fluorine (c) ClO3 (d) ClO 4
is – 1. 52. In the reaction
44. Nitric oxide acts as a reducing agent in the reaction
(a) 4 NH 3 5 O 2 4 NO 6 H 2O 3Br2 6CO 32 3H 2 O 5Br BrO 3– 6HCO 3
(a) Bromine is oxidised and carbonate is reduced.
(b) NO I H O NO – l– H
(b) Bromine is reduced and water is oxidised
(c) 2 NO H 2SO 3 N 2 O H 2SO 4 (c) Bromine is neither reduced nor oxidised
(d) NO H S N O S H O (d) Bromine is both reduced and oxidised
45. In the compounds KMnO4 and K2 Cr 2 O7 the highest 53. Which of the following elements does not show
oxidation state is of the element disproportionation tendency?
(a) potassium (b) manganese (a) Cl (b) Br
(c) chromium (d) oxygen (c) F (d) I
46. Atomic number of an element is 22. The highest O.S. 54. Phosphorus, sulphur and chlorine undergo disproportion
exhibited by it in its compounds is in the ...A... medium.
(a) 1 (b) 2 Here, A refers to
(c) 3 (d) 4 (a) acidic (b) alkaline
47. Why the displacement reactions of chlorine, bromine and (c) neutral (d) Both (a) and (b)
iodine using fluorine are not generally carried out in +1 -2 0 0
D
aqueous solution? 55. The reaction, 2H 2 O(l ) ¾¾® 2H 2 ( g ) + O2 ( g ) is an
(a) chlorine, bromine and iodine reacts with water and example of
displace oxygen of water (a) addition reaction (b) decomposition reaction
(b) Fluorine being very reactive attacks water and (c) displacement reaction (d) None of these
displaces oxygen of water 56. How will you balance the total ionic charge of reactant
(c) Fluorine does not react with chlorine, bromine and and products if reaction is carried out in acidic solution?
iodine in aqueous media (a) By using H+ ions
(d) None of these
(b) By using OH– ions
48. Which of the following statement is not true ?
(c) Adding H2O molecules to the reactant or product
(a) Displacement reaction of chlorine with Br– and I–
(d) Multiplying by suitable coefficients.
form the basis of identifying Br– and I– in laboratory
57. Consider the following reaction occuring in basic medium
using layer test
(b) F2, Cl2, Br2 and I2 can be recovered by halogen 2MnO –4 (aq)+Br – (aq) 2MnO2 (s) + BrO3– (aq)
displacement reactions by using their respective How the above reaction can be balanced further?
halides (a) By adding 2 OH– ions on right side
(c) F2 can be recovered from F– by oxidising it (b) By adding one H2O molecule to left side
electrolytically. (c) By adding 2H+ ions on right side
(d) None of these. (d) Both (a) and (b)
EBD_7207
138 REDOX REACTIONS

58. For the reaction : NH 3 OCl N 2 H 4 Cl in basic Out of the above given elements which is the strongest
medium, the coefficients of NH3, OCl– and N2H4 for the oxidising agent and which is the weakest oxidising agent ?
balanced equation are respectively (a) (iv) is the strong whereas (ii) is the weakest oxidising
agent
(a) 2, 2, 2 (b) 2, 2, 1
(b) (ii) is the strongest whereas (i) is the weakest
(c) 2, 1, 1 (d) 4, 4, 2
oxidising agent
59. C2H6 (g) + nO2(g) CO2(g) + H2O(l) (c) (i) is the strongest whereas (ii) is the weakest
In this equation, the ratio of the coefficients of CO2 and oxidising agent
H2O is (d) (ii) is the strongest whereas (iii) is the weakest
(a) 1 : 1 (b) 2 : 3 oxidising agent
(c) 3 : 2 (d) 1 : 3 67. Stronger is oxidising agent, more is
60. 2MnO 4 5H 2 O 2 6H 2 Z 5O 2 8H 2 O. In this (a) standard reduction potential of that species
reaction Z is (b) the tendency to get it self oxidised
(a) Mn +2 (b) Mn +4 (c) the tendency to lose electrons by that species
(d) standard oxidation potential of that species
(c) MnO2 (d) Mn
68. Standard reduction potentials of the half reactions are
61. In the redox reaction,
given below :
xKMnO4 + NH3 yKNO3 + MnO2 + KOH + H2O F2(g) + 2e– 2F– (aq); E° = + 2.85 V
(a) x = 4, y = 6 (b) x = 3, y = 8 Cl2(g) + 2e– 2Cl–(aq); E° = + 1.36 V
(c) x = 8, y = 6 (d) x = 8, y = 3 Br2(l) + 2e– 2Br–(aq); E° = + 1.06 V
62. What is ‘A’ in the following reaction I2(s) + 2e– 2I–(aq); E° = + 0.53 V
2Fe3+(aq) +Sn2+(aq) 2Fe2+(aq) + A The strongest oxidising and reducing agents respectively
(a) Sn3+ (aq) (b) Sn4+ (aq) are :
2+
(c) Sn (aq) (d) Sn (a) F2 and I– (b) Br2 and Cl–
63. Given : (c) Cl2 and Br– (d) Cl2 and I2
X Na2HAsO3 + Y NaBrO3 + Z HCl NaBr 69. Standard electrode potentials of redox couples
+ H3AsO4 + NaCl A2+/A, B2+/B, C/C2+ and D2+/D are 0.3V, – 0.5V, – 0.75V
The values of X, Y and Z in the above redox reaction are and 0.9V respectively. Which of these is best oxidising agent
respectively and reducing agent respectively –
(a) 2, 1, 2 (b) 2, 1, 3 (a) D2+/D and B2+/B (b) B2+/B and D2+/D
2+
(c) D /D and C /C 2+ (d) C2+/C and D2+/D
(c) 3, 1, 6 (d) 3, 1, 4
64. The values of x and y in the following redox reaction 70. The standard reduction potentials at 298K for the following
half reactions are given against each
x Cl 2 + 6OH ClO3 + y Cl + 3H 2O are Zn2+ (aq) + 2e Zn(s) ; –0.762 V
(a) x = 5, y = 3 (b) x = 2, y = 4 Cr3+ (aq) + 3e Cr (s); –0.740 V
(c) x = 3, y = 5 (d) x = 4, y = 2 2H+ (aq) + 2e H2 (g) ; 0.00 V
Fe3+ (aq) + e Fe2+ (aq) ; 0.770 V
65. A negative E means that redox couple is a ___A___
Which is the strongest reducing agent?
than the H+/H2 couple (a) Zn (s) (b) Cr (s)
A positive E means that the redox couple is a ____B___ (c) H2(g) (d) Fe3+ (aq)
than H+/H2 couple 71. Electrode potential data are given below :
(a) A = stronger reducing agent 3
Fe (aq e 2
Fe (aq E° = +0.77 V
) );
B = weaker reducing agent
(b) A = stronger oxidising agent Al3(aq) 3e Al (s) ; E° = – 1.66 V
B = weaker oxidising agent Br2 (aq) 2e 2Br (aq) ; E° = + 1.08V
(c) A = weaker oxidising agent Based on the data, the reducing power of Fe2+, Al and Br –
B = stronger oxidising agent will increase in the order
(d) Both (a) and (c) (a) Br– < Fe2+ < Al (b) Fe2+ < Al < Br –

(c) Al < Br < Fe 2+ (d) Al < Fe2+ < Br–
66. Given E
72. The standard reduction potentials for Cu2+/Cu; Zn2+/Zn;
(i) Mg2+/Mg(s), E = –2.36 Li +/Li; Ag+ /Ag and H+/H2 are + 0.34 V, – 0.762 V,
(ii) Ag+/Ag(s), E = 0.80 – 3.05 V, + 0.80 V and 0.00 V respectively. Choose the
strongest reducing agent among the following
(iii) Al3+/Al(s), E = –1.66
(a) Zn (b) H2
(iv) Cu2+/Cu(s), E = 0.52 (c) Ag (d) Li
REDOX REACTIONS 139
73. Given : (a) Both statements (i) and (ii) are correct.
E 1 o
1.36 V, E o
0.74 V, (b) Statement (i) is correct and (ii) is incorrect.
Cl2 / Cl Cr3 / Cr (c) Statement (ii) is correct and (i) is incorrect.
2
o (d) Both statements (i) and (ii) are incorrect.
E 1.33V, E o 1.51V
Cr2O 72 / Cr3 MnO 4 / Mn 2 80. Which of the following statement(s) is/are correct ?
The correct order of reducing power of the species (i) All alkali metals and some alkaline earth metals (Ca,
(Cr, Cr3+, Mn2+ and Cl–) will be Sr and Ba) displace hydrogen from cold water.
(a) Mn2+ < Cl– < Cr3+ < Cr (ii) Magnesium and iron react with steam as well as
(b) Mn2+ < Cl3+ < Cl– < Cr acids to produce hydrogen gas.
(c) Cr3+ < Cl– < Mn2+ < Cr (iii) Cadmium and tin do not react with steam but
(d) Cr3+ < Cl– < Cr < Mn2+ displace hydrogen from acids.
74.

E Values of some redox couples are given below. On the (a) (i) and (ii) (b) (ii) only
basis of these values choose the correct option. (c) (i) and (iii) (d) (i), (ii) and (iii)

E values : Br2/Br – = + 1.90; Ag+ /Ag(s) = + 0.80 81. Which of the following statements are correct concerning
– redox properties?
Cu2+ /Cu(s) = + 0.34; I2(s) /I = 0.54
(a) Cu will reduce Br – (b) Cu will reduce Ag (i) A metal M for which E° for the half life reaction

(c) Cu will reduce I (d) Cu will reduce Br2 Mn+ + ne– M is very negative will be a good
75. Arrange the following in the order of their decreasing reducing agent.
electrode potentials : Mg, K, Ba and Ca (ii) The oxidizing power of the halogens decreases from
(a) K, Ca, Ba, Mg (b) Ba, Ca, K, Mg chlorine to iodine.
(c) Ca, Mg, K, Ba (d) Mg, Ca, Ba, K (iii) The reducing power of hydrogen halides increases
76. The standard electrode potentials of four elements A, B, C from hydrogen chloride to hydrogen iodide
and D are –3.05, –1.66, –0.40 and +0.80. The highest chemical (a) (i), (ii) and (iii) (b) (i) and (ii)
reactivity will be exhibited by (c) (i) only (d) (ii) and (iii)
(a) A (b) B 82 Which of the following statement(s) is/are correct ?
(c) C (d) D (i) A negative value of E– means that the redox couple is
a weaker reducing agent than the H+/H2 couple.
STATEMENT TYPE QUESTIONS (ii) A positive E– means that the redox couple is weaker
77. Which of the following statement(s) is/are correct for the reducing agent than the H+/H2.
given reaction? Which of the following code is incorrect regarding above
2HgCl2 (aq) + SnCl2(aq) Hg2Cl2(s) + SnCl4(aq) statements?
(i) Mercuric chloride is reduced to Hg2Cl2 (a) Only (i) (b) only (ii)
(ii) Stannous chloride is oxidised to stannic chloride (c) Both (i) and (ii) (d) Neither (i) nor (ii)
(iii) HgCl2 is oxidised to Hg2Cl2 83. Which of the following statement(s) is/are correct ?
(iv) It is an example of redox reaction (i) Oxidation state of carbon in C3H 4 is –(4/3).
(a) (i), (ii) and (iv) (b) (i) and (ii)
(c) (iii) and (iv) (d) (iii) only (ii) Electrons are never shared in fraction.
78. Which of the following sequences of T and F is correct (a) (i) and (ii) (b) Only (i)
for given statements. Here T stands for true and F stands (c) Only (ii) (d) Neither (i) nor (ii)
for false statements
(i) Reducing agents lower the oxidation number of an MATCHING TYPE QUESTIONS
element in a given substance. These reagents are 84. Match the columns
also called as reductants Column-I Column-II
(ii) Reducing agents are acceptor of electrons (A) Addition of (p) Oxidation reaction
(iii) Loss of electron(s) by any species is called oxidation electronegative element
reaction (B) Removal of hydrogen (q) Reduction reaction
(iv) Oxidation and reduction always occur simultaneously. (C) Addition of
(a) TTTT (b) TFTT electropositive element
(c) TFFT (d) FTTT (D) Removal of oxygen
79. If aqueous solution of H2O2 is made acidic. For this which (a) (A) – (p), (B) – (q), (C) – (q), (D) – (p)
of the following statement(s) is/are correct ? (b) (A) – (p), (B) – (p), (C) – (q), (D) – (q)
(i) This aqueous solution oxidizes I– (c) (A) – (p), (B) – (q), (C) – (p), (D) – (q)
(ii) This aqueous solution oxidizes F– (d) (A) – (q), (B) – (q), (C) – (p), (D) – (p)
EBD_7207
140 REDOX REACTIONS
85. Match the columns 88. Assertion : In the reaction 2Na(s) + Cl2(g) 2NaCl(s)
Column-I Column-II sodium is oxidised.
(A) 2Mg + O2 2MgO (p) Removal of hydrogen Reason : Sodium acts as an oxidising agent in given
(B) Mg + Cl2 MgCl2 (q) Removal of reaction.
electropositive element 89. Assertion : HClO4 is a stronger acid than HClO3.
(C) 2H2S + O2 (r) Addition of oxygen Reason : Oxidation state of Cl in HClO4 is +VII and in HClO3
2S + 2H2O +V.
(D) 2KI + H2O + O3 (s) Addition of 90. Assertion : The reaction :
2KOH + I2 + O2 electronegative CaCO3 (s) CaO(s) CO 2 (g) is an example of
element, chlorine
(a) A – (s), B – (q), C – (p), D – (r) decomposition reaction
Reason : Above reaction is not a redox reaction.
(b) A – (r), B – (s), C – (p), D – (q)
(c) A – (s), B – (r), C – (q), D – (p) 91. Assertion : In a reaction
(d) A – (r), B – (p), C – (s), D – (q) Zn(s) + CuSO4 (aq) ZnSO4(aq) + Cu(s)
86. Match Column-I (compound) with Column-II (oxidation Zn is a reductant but itself get oxidized.
state of underlined element) and choose the correct Reason : In a redox reaction, oxidant is reduced by accepting
option. electrons and reductant is oxidized by losing electrons.
Column - I Column - II
CRITICAL THINKING TYPE QUESTIONS
(A) CuO (p) 4
(B) MnO2 (q) 3 92. Among NH3 , HNO3, NaN3 and Mg3N2 the number of
(C) HAuCl4 (r) 2 molecules having nitrogen in negative oxidation state is
(D) Tl2O (s) 1 (a) 1 (b) 2
(a) A – (r), B – (p), C – (q), D – (s) (c) 3 (d) 4
(b) A – (s), B – (r), C – (p), D – (q) 93. Fill up the table from the given choice.
(c) A – (r), B – (s), C – (p), D – (q) Element Oxidation number
(d) A – (s), B – (q), C – (p), D – (r) Oxygen –2 in most compounds (i) in H2O2
87. Match the columns and (ii) in OF2
Column-I Column-II Halogen –1 for (iii) in all its compounds
(A) V2O5(s) + 5 Ca(s) (p) Disproportionation Hydrogen (iv) in most of its compounds (v) in
2V(s) + 5 CaO(s) reaction binary metallic hydrides
Sulphur (vi) in all sulphides
(B) CH4(g) + 2O2(g) (q) Decomposition
(i) (ii) (iii) (iv) (v) (vi)
CO2(g) + 2 H2O(l) reaction
(a) +1 +1 Cl +1 –1 +2
(C) P4(s) + 3OH–(aq) + 3H2O(l) (r) Combination
(b) –1 +2 F +1 –1 –2
PH3(g) + 3H 2 PO 2– ( aq ) reaction (c) –1 +1 F +1 +2 +2
(d) +1 +2 Cl +1 +1 +6
(D) 2 KClO3(s) (s) Displacement
2KCl(s) + 3O2(g) reaction 94. The correct decreasing order of oxidation number of oxygen
in compounds BaF2, O3, KO2 and OF2 is
(a) A – (s), B – (q), C – (r), D – (p)
(a) BaO2 > KO2 > O3 > OF2
(b) A – (s), B – (r), C – (p), D – (q)
(b) OF2 > O3 > KO2 > BaO2
(c) A – (r), B – (s), C – (q), D – (p)
(c) KO2 > OF2 > O3 > BaO2
(d) A – (r), B – (s), C – (p), D – (q)
(d) BaO2 > O3 > OF2 > KO2
ASSERTION-REASON TYPE QUESTIONS 95. Oxidation numbers of P in PO3– 2–
4 , of S in SO 4 and that of
Directions : Each of these questions contain two statements, Cr in Cr2 O 72– are respectively
Assertion and Reason. Each of these questions also has four (a) + 3, + 6 and + 5 (b) + 5, + 3 and + 6
alternative choices, only one of which is the correct answer. You (c) – 3, + 6 and + 6 (d) + 5, + 6 and + 6
have to select one of the codes (a), (b), (c) and (d) given below. 96. When Cl2 gas reacts with hot and concentrated sodium
(a) Assertion is correct, reason is correct; reason is a correct hydroxide solution, the oxidation number of chlorine
explanation for assertion. changes from
(b) Assertion is correct, reason is correct; reason is not a (a) zero to +1 and zero to –5
correct explanation for assertion (b) zero to –1 and zero to +5
(c) Assertion is correct, reason is incorrect (c) zero to –1 and zero to +3
(d) Assertion is incorrect, reason is correct. (d) zero to +1 and zero to –3
REDOX REACTIONS 141
97. Which of the following arrangements represent increasing The value’s of x, y and z in the reaction are, respectively :
oxidation number of the central atom? (a) 5, 2 and 16 (b) 2, 5 and 8
(c) 2, 5 and 16 (d) 5, 2 and 8
(a) CrO2 , ClO3 , CrO24 , MnO 4
103. In the balanced chemical reaction
(b) ClO3 , CrO42 , MnO 4 , CrO 2 IO3 aI bH cH 2 O dI 2
a, b, c and d respectively corresponds to
(c) CrO2 , ClO3 , MnO4 , CrO42 (a) 5, 6, 3, 3 (b) 5, 3, 6, 3
(c) 3, 5, 3, 6 (d) 5, 6, 5, 5
(d) CrO24 , MnO4 , CrO2 ,ClO3
104. If equal volume of reactants are used, than no. moles of
98. Which of the following act as reducing agents ? KMnO4 (moles per liter) used in acidic medium required to
(i) PO34 (ii) SO3 completely oxidises the 0.5 M FeSO3?
(a) 0.3 (b) 0.1
(iii) PO32 (iv) NH3 (c) 0.2 (d) 0.4
(a) (i), (ii) and (iii) (b) Only (iii) 105. Acidic medium used in KMnO4 can be made from which of
(c) (i), (iii) and (iv) (d) (iii) and (iv) the following acids?
99. In the reaction shown below, oxidation state of the carbon (a) HCl (b) H2SO4
in reactant and product are (i) and (ii) respectively? Is the (c) HI (d) HBr
given reaction a redox reaction? 106. If rod of a metal (x) is put in a metal ion solution which is
Na2CO3(aq) + HCl (aq) blue in colour , solution turn colorless. The metal rod and
solution respectively are?
Na aq Cl aq H 2O CO 2 g (a) Zinc and Cu(II) (b) Zinc and Ni(II)
(a) (i) 6, (ii) 4, yes (b) (i) 6, (ii) 6, No (c) Aluminium and Cu(II) (d) Both (a) and (c)
(c) (i) 4, (ii) 4, No (d) (i) 4, (ii) 4, yes 107. What could be the X–in the system, Where X signifies
100. What products are expected from the disproportionation halogen ; formation of shown below X2 takes place, when
reaction of hypochlorous acid? F2 is purge into aqueous solution of X–?
(a) HCl and Cl2O (b) HCl and HClO3 X2(g)
(c) HClO3 and Cl2O (d) HClO2 and HClO4
101. In the disproportionation reaction
3 HClO3 HClO4 + Cl2 + 2O2 + H2O, the equivalent mass
of the oxidizing agent is (molar mass of HClO3 = 84.45) Aqueous solution of X

(a) 16.89 (b) 32.22


(c) 84.45 (d) 28.15
102. Consider the following reaction :
(a) Br – (b) Cl –
z
xMNO4 yC2 O42 zH xMn 2 2yCO 2 H 2O (c) I – (d) All of these
2
EBD_7207
142 REDOX REACTIONS

14. (b) Blue colour of the solution disappear due to


FACT / DEFINITION TYPE QUESTIONS formation of Zn 2+.
1. (a) Addition of oxygen takes place in oxidation. 15. (d) Correct order is Zn > Cu > Ag.
2. (c) Given reaction is oxidation reaction due to removal 16. (b) For elements, in the free or the uncombined state,
of electropositive element potassium from potassium each atom bears an oxidation number of zero.
ferrocyanide. 17. (c) Oxidation number of oxygen in OF2 = + 2.
D
3. (d) 2Na(s) + H 2 (g) ¾¾® 2NaH(s) 1
In KO2 =
With the careful application of the concept of 2
electronegativity only S we can find that sodium is 18. (d) Oxidation number of hydrogen when it is bonded to
oxidised and hydrogen is reduced. metals in binary compounds is –1
4. (a) Losing of electron is called oxidation. 19. (a) Auric Chloride = Au(III)Cl3
5. (b) Oxidation reaction (loss of 2e–) 20. (d) Let the oxidation no. of N in HNO3 = x
1 + x + (3 × – 2) = 0
2Na(s) + S(s)
+
(Na )2S (s) – x= +5
+2 -1 +1 +6 -2 +2 +6 -2 +1 -1
– 21. (c) Ba O2 + H 2 S O4 ¾¾
® Ba S O4 + H 2 O2
Reduction (gain of 2e )
reaction In this reaction, none of the elements undergoes a change
in oxidation number or valency.
6. (a) O.N. of Mn in MnO – is +7 and in Mn it is +2. The
22. (a) Let x = oxidation no. of Cr in K2Cr2O7.
difference is of 5 electrons. (2 × 1) + (2 × x) + 7 (– 2) = 0
7. (a) Ox. no. of Cr on both side is + 6. or 2 + 2x – 14 = 0 or x = + 6.
8. (a) 4P 3KOH 3H 2 O KH 2 PO 2 PH 3 23. (a) (i) Oxidation state of element in its free state is zero.
O.N of P = 0, In KH2PO2 it is + 1, In PH3 it is –3. (ii) Sum of oxidation states of all atoms in compound is
Hence P is oxidised and reduced. zero.
9. (a) In a redox reaction, one molecule is oxidised and other O.N. of S in S8 = 0; O.N. of S in S2F2 = + 1;
molecule is reduced i.e. oxidation number of reactants O.N. of S in H2S = –2;
are changed. 24. (c) K[Co(CO) ]
0 0 1 1
H2 Br 2 2 HBr Let O.N. of Co be x then
Here H2 is oxidised and Br2 is reduced, thus it is 1 × ( 1) x 4× (0) 0
for K for Co
oxidation-reduction reaction. for CO

+ O.N. of Co is = –1
10. (b) 4Na + O 2 ¾¾
® 2Na 2 O
25. (d) (NH4)2 SO4 is split into ions. NH 4 . Let O.N. of N be x
Loss of electrons (oxidation)
then, 1 × (x) + 4 × (+1) = 1 x = –3
In this reaction, Na converts into ion (Na+) and Na
donates electrons to oxygen atoms, So, Na behaves H 1

as reducing agent. 1 |
11. (b) Zn2+ + 2e– Zn(s) 26. (c) H – C 22 – Cl 1 O.N. of C is zero
|
Here electrons are reducing from Zn 2+ to Zn. 1
Cl
12. (c) Co(s) Cu 2 (aq) Co 2 (aq) Cu(s)
This reaction is a redox reaction as Co undergoes 27. (d) O.N. of Fe in (a), (b), (c) and (d) respectively are : +3,
oxidation whereas Cu+2 undergoes reduction. +2, +2 and 0.
loss of 10e 28. (d) OsO4
13. (c) N 24 H 44 N 2 6 Y;
N Let O.N. of Os be x then 1 × (x) + 4(–2) = 0
O.N.of N changes from – 2 to +3 x=8
REDOX REACTIONS 143
29. (a) Fe(CO)5 is metal carbonyl, hence O.N. of Fe is zero. 47. (b) Fluorine is so reactive that it attacks water and
30. (c) O.N. of Mn in K2MnO4 is +6 displaces the oxygen of water :
31. (d) MnO – (O.S. of Mn +7); Cr (CN) 36– (O.S. of Cr +3), 1 2 0 1 1 0
2H 2O( I ) 2F2 ( g ) 4HF( aq ) O 2 ( g )
NiF62 – (O.S. of Ni +4) and CrO2Cl2 (O.S. of Cr +6) 48. (b) As fluorine is the strongest oxidising agent; there is
32. (d) O.N. of carbon in CH3CHO is –1; in other cases it is no way to convert F– ions to F2 by chemical means.
zero. The only way to achieve F2 from F– is to oxidise it
electrolytically.
33. (b) O.N. of iodine in I3 is –1/3
49. (c) F2 being most electronegative element cannot exhibit
34. (b) M3+ on losing 3 elections will become M+6 and O.N. = + 6. any positive oxidation state.
35. (b) The method of representing oxidation number by a
In ClO –4 chlorine is present in its highest oxidation
Roman numeral within the paranthesis represents Stock
notation. state i.e + 7. Therefore it does not show
36. (c) Carbon has the maximum oxidation state of + 4, disproportionation reaction.
therefore carbon dioxide (CO2 ) cannot act as a 50. (c) A reaction, in which a substance undergoes
reducing agent. simultaneous oxidation and reduction, is called
disproportionation reaction. In these reactions, the
37. (b) Let the oxidation no. of Fe in Fe3O 4 x
same substance simultaneously acts as an oxidising
3x + (– 2 × 4) = 0 or 3x = 8 agent and as a reducing agent. Here Cl undergoes
8 simultaneous oxidation and reduction.
x
3
2KOH Cl 2 KCl KOCl H 2O.
38. (c) Let oxidation state of oxygen in OF2 = x 0 1 1
x + ( – 1 × 2) = 0 51. (d) In disproportionation reaction, one element of a
x= +2 compound will simultaneously get reduced and
39. (d) In H2O2 : 2 × (+1) + 2 × x = 0 x = –1 oxidised. In ClO4 , oxidation number of Cl is + 7 and it
In CO2 : 4 + 2x = 0 x = –2
can not increase it further. So, ClO4 will not get
In H2O : 2 × (+1) + x = 0 x = –2
In F2O : 2 × (–1) + x = 0 x = +2 oxidised and so will not undergo disporportionation
– reaction.
40. (d) NO3 N 2 H 4 So, for reduction of 1 mole of NO
5 2 52. (d) 3Br2 6CO32 – 3H 2 O 5Br – BrO3– 6HCO 3–
3 number of electrons required is 7. O.N. of Br 2 changes from 0 to –1 and +5
41. (a) Let the oxidation state of S be x. hence it is reduced as well as oxidised.
S4O62– 4x – 12 = – 2 4x = 10 x = 10/4 = 2.5
53. (c)
42. (c) Species O.N.
54. (b) Phosphorus, sulphur and chlorine disproportionate in
Cl – –1
the alkaline medium.
ClO4– +7
D
ClO– +1 55. (b) 2H 2 O ¾¾® 2H 2 + O 2
MnO4 – +7
There is decomposition of H2O molecule into H2 and
In ClO– chlorine is in +1 oxidation state which can be O2.
increased or decreased thus it acts as an oxidising or
56. (a) H+ ions are added to the expression on the
reducing agent.
appropriate side so that the total ionic charges of
In other given species the underlined elements are either reactants and products become equal.
in their minimum or maximum oxidation state.
57. (d) Since reaction is occuring in basic medium therefore
43. (a) 2OH– are added on right side.
44. (b) O.N. of N changes from + 2 to + 5 hence NO is reducing.
2MnO 4 (aq) + Br– (aq)
45. (b) In KMnO4 : Let O.N. of Mn be x
+1 + x + 4(–2) = 0 x = +7 2MnO2(s) + Br O3– (aq) + 2OH–(aq)
In K2Cr2O7 : Let O.N. of Cr be x
Now, hydrogen atoms balanced by adding one H2O
2(1) + 2x + 7(–2) = 0 x = +6
molecule to the left side
46. (d) The element is Ti (At. no. 22). Electronic configuration
2MnO 4 (aq) Br (aq) H 2 O( )
is 1s2, 2s2p6, 3s2p6d2, 4s2. the energy level of 3d and
4s is very close. It can have Ti4+ O.S. 2MnO2(s) + BrO 3 (aq) 2 OH (aq)
EBD_7207
144 REDOX REACTIONS
58. (c) The balanced equation : 73. (a) Lower the value of reduction potential higher will be
2 NH 3 OCl N 2H 4 Cl H 2O reducing power hence the correct order will be
Mn2+ < Cl– < Cr3+ < Cr
59. (b) The balanced equation is
74. (d)
2C2 H6 7O2 4CO 2 6H 2O.
75. (d) Order of decreasing electrode potentials of Mg, K, Ba
Ratio of the coefficients of CO2 and H2O is 4 : 6 or 2 : 3. and Ca is
60. (a) 2MnO 4– 5H 2 O 2 6H 2Mn 2 5O 2 8H 2 O. Mg > Ca > Ba > K
It can be explained by their standard reduction
61. (d) 8KMnO 4 3NH 3 8MnO 2 3KNO 3 5KOH potentials.
2H 2O E 2.925
K |K
62. (b) Reduction
E 2.90
Ba 2 |Ba
2Fe3 Sn 2 2Fe2 Sn 4

Oxidation E 2.87
Ca 2 |Ca
63. (c) On balancing the given reaction, we find
3Na2HAsO3 + NaBrO3 + 6HCl E 2.37
Mg 2 |Mg
6NaCl + 3H3AsO4 + NaBr
Highly negative value of E red shows the least value
64. (c)
o 5 –1 of electrode potential.
x Cl 2 6 H – ClO 3 yCl 3H 2 O 76. (a) Standard electrode potential i.e. reduction potential of
–1
A is minimum (–3.05V) i.e. its oxidation potential is
on balancing the eq we get maximum which implies ‘A’ is most reactive chemically.
3Cl2 6OH ClO 3 5Cl 3H 2 O
STATEMENT TYPE QUESTIONS
65. (d) Negative E Stronger reducing agent or weaker
oxidising agent 77. (a) For statement (iii), HgCl2 is reduced to Hg2Cl2
78. (b) For statement (ii) reducing agents are donor of electrons.
Positive E Weaker reducing agent or stronger
79. (b) H2O2 is strong oxidizing than I2, reduction potential
oxidising agent.
of H2O2 is greater than that of I2.
66. (b) Strongest oxidising agent = Ag+/Ag(s)
80. (d) All the given statements are correct.
Weakest oxidising agent = Mg2+/Mg(s)
67. (a) More is E°RP , more is the tendency to get itself reduced 81. (a) (i) Mn+ + ne– M, for this reaction, high
or more is oxidising power. negative value of E° indicates lower reduction
68. (a) Higher the value of reduction potential higher will be potential, that means M will be a good reducing
the oxidising power whereas the lower the value of agent.
reduction potential higher will be the reducing power.
Stronger reducing agent Easy to oxidise
69. (c) The redox couple with maximum reduction potential
will be best oxidising agent and with minimum reduction
Lower reduction potential higher oxidation potential
potential will be best reducing agent.
70. (a) Since oxidation potential of Zn is highest hence (ii) Element F Cl Br I
strongest reducing agent. Reduction potential +2.87 +1.36 +1.06 +0.54
71. (a) Fe Al Br (E° volt)
0.77 –1.66 1.08 E°Red As reduction potential decreases from fluorine to
–0.77 1.66 –1.08 E°Oxi iodine, oxidising nature also decreases from fluorine
Hence, reducing power Al > Fe2+ > Br– to iodine.
72. (d) More the negative reduction potential, more is the (iii) The size of halide ions increases from F – to I –.
tendency to lose electron. The reducing power The bigger ion can loose electron easily. Hence the
increases as the standard reduction potential becomes reducing nature increases from HF to HI.
more and more negative.
82. (a)
Thus, Li is the strongest reducing agent as the
83. (a) –(4/3) is the average oxidation state of C in C3H4.
standard reduction potential of Li+/ Li is most negative,
–3.05 V.
REDOX REACTIONS 145

MATCHING TYPE QUESTIONS Oxidation state of N in NaN3 is


1
84. (b) Oxidation is addition of electronegative or removal + 1 + 3x = 0 or x = –
3
of electroposition element to a substance or removal
Oxidation state of N in Mg3N2 is
of hydrogen from a substance.
3 × 2 + 2x = 0 or x = –3
Reduction is addition of electropositive or removal
Thus 3 molecules (i.e. NH3, NaN3 and Mg3N2 have
of electropositive element or removal of oxygen from
nitrogen in negative oxidation state.
a substance.
93. (b)
85. (b)
94. (b) Oxidation no. of O are + 2, 0, – 1/2 and – 1 respectively
86. (a) CuO +2
MnO2 +4 95. (d) PO3–
4 = x + 4 (– 2) = – 3; x – 8 = – 3; x = + 5
HAuCl4 +3
Tl2O +1 SO 2–
4 = x + 4 (– 2) = – 2; x – 8 = – 2; x = + 6
87. (b)
Cr2 O72– = 2x + 7 (– 2) = – 2; 2x – 14 = – 2;
ASSERTION-REASON TYPE QUESTIONS 2x =12; x = + 6
96. (b) On reaction with hot and concentrated alkali a mixture
88. (c) In reaction 2Na(s) + Cl2(g) 2NaCl(s) sodium is
of chloride and chlorate is formed
oxidised by loss of electrons and acts as a reducing
agent (donor of electrons). 3Cl2 + 3 NaOH(excess) Hot
89. (b) Both Assertion and Reason are true but reason is not
1 5
the correct explanation of assertion. Greater the number
5NaCl NaClO3 3H 2O
of negative atoms present in the oxy-acid make the
acid stronger. In general, the strengths of acids that 97. (a)
have general formula (HO)m ZOn can be related to the 98. (d) In (i) and (ii) both P and S are in highest oxidation
value of n. As the value of n increases, acidic character state. In (iii) and (iv) ; P has oxidation state of +4
also increases. The negative atoms draw electrons which can be oxidized to +5 state, while in case of
away from the Z-atom and make it more positive. The NH3 nitrogen has oxidation state of –3 which can be
Z-atom, therefore, becomes more effective in oxidised.
withdrawing electron density away from the oxygen 99. (c) The redox reaction involve loss or gain of electron(s)
atom that bonded to hydrogen. In turn, the electrons i.e. change in oxidation state. Given reaction is not a
of H – O bond are drawn more strongly away from the redox reaction as this reaction involves no change in
H-atom. The net effect makes it easier from the proton oxidation state of reactant or product.
release and increases the acid a strength. 100. (b) During disproportionation same compound undergo
90. (b) Decomposition of calcium carbonate is not a redox simultaneous oxidation reduction.
reaction. Oxidation
Oxidation loss of 2e– 1 5
91. (a) 3HO Cl 2HCl HClO 3
Zn(s) + Cu2+(aq) Zn2+ (aq) + Cu(s)
Reduction
101. (a) ClO3 Cl20
Reduction gain of 2e–
x 6 1 x 0
x 5 x 0 x = oxidation number
CRITICAL THINKING TYPE QUESTIONS
Molecular mass 84.45
92. (c) Calculating the oxidation state of nitrogen in given Equivalent mass = = 16.89
Oxidation number 5
molecules;
102. (c) On balancing the given equations, we get
Oxidation state of N in NH3 is
x + 3 × (+ 1) = 0 or x = – 3 2MnO 4 5C 2 O 4 2 16H 2Mn
Oxidation state on N in NaNO3 is 10CO 2 8H 2O
1 + x + 3 × (– 2) = 0 or x = + 5 So, x = 2, y = 5 & z = 16
EBD_7207
146 REDOX REACTIONS
103. (a) Given reaction is 3
MnO4 5e 8H Mn 2 4H 2O
IO3 aI bH cH 2 O dI 2 5
st
I half reaction Fe2+ Fe3+ + le–
I I2 ...(i) SO32 SO 24 2e
–1 0 (oxidation)
IInd half reaction 8 24
MnO 4 H Fe2 SO32
IO 3 I2 ...(ii) 5 5
`
+5 0 (reduction) 3Mn 2 4H 2 O Fe3 SO 24
On balancing equation (ii) we have
10e 2IO3 12H I2 6H 2 O ...(iii) 105. (b) If one uses HCl, HBr or HI, to make acidic medium for
Now, balance equation (i) KMnO4 than all the halide ion can be oxidized as the
2I I2 2e ....(iv) reduction potential of KMnO4 is very high in acidic
medium, while in case of H2SO4, sulphur is already in
Multiply eqn (iv) by 5 and add it to eqn (iii), we get
its highest oxidation state cannot be further oxidized.
2IO 3 10I 12H 6I 2 6H 2 O
106. (d) Reduction potential of Cu(II) is greater than that of
or, IO3 5I 6H 3I 2 3H 2 O Zn(II) and Al(III) thus can be easily replaced by these
Hence a = 5, b = 6, c = 3, d = 3 ions. Moreover solution of copper is blue in color.
107. (d) F2 is strongest oxididing agent among halogens thus
104. (a) Both Fe(ii) and S(iv) in SO32 can be oxidised to Fe(iii) X– can be possibly Br–, Cl– or I–.
and (SO4)2– respectively hence (3/5)×0.5 =0.3 moles /
litre.
9
HYDROGEN

FACT / DEFINITION TYPE QUESTIONS (a) halogen (b) alkali metals


(c) chalcogens (d) alkaline earth metals
1. Following are some properties of hydrogen which of the 6. Which of the following statements is correct ?
following properties resemble with alkali metals and which (a) Hydrogen has same IP as alkali metals
with halogens (b) Hydrogen has same electronegativity as halogens
(i) Hydrogen lose one electron to form unipositive ions (c) It has oxidation number of –1 and +1
(ii) Hydrogen gain one electron to form uninegative ions (d) It will not be liberated at anode
(iii) Hydrogen forms oxides, halides and sulphides 7. Why does H+ ion always get associated with other atoms
(iv) Hydrogen has a very high ionization enthalpy or molecules?
(v) Hydrogen forms a diatomic molecule, combines with (a) Ionisation enthalpy of hydrogen resembles that of
elements to form hydrides and covalent compounds. alkali metals.
(a) Alkali metals resemble (i), (iii) and (iv) (b) Its reactivity is similar to halogens.
Halogens resemble (ii) and (v) (c) It resembles both alkali metals and halogens.
(b) Alkali metals resemble (i) and (iii) (d) Loss of an electron from hydrogen atom results in a
Halogens resemble (ii), (iii) and (v) nucleus of very small size as compared to other atoms
(c) Alkali metals resemble (i) and (iii) or ions. Due to small size it cannot exist free.
Halogens resemble (ii), (iv) and (v) 8. Which one of the following is not an isotope of hydrogen ?
(d) Alkali metals resemble (i) only (a) Deuterium (b) Tritium
Halogens resemble (iv) and (v) (c) Ortho hydrogen (d) None of these
2. Hydrogen molecules differs from chlorine molecule in the 9. Number of neutrons in three isotopes of hydrogen,
following respect protium, deuterium and tritium respectively is
(a) Hydrogen molecule is non-polar but chlorine molecule (a) 0, 1, 2 (b) 1, 1,1
is polar (c) 2, 1, 0 (d) 2, 0, 1
(b) Hydrogen molecule is polar while chlorine molecule is 10. Which isotope(s) of hydrogen is/are radioactive and
non-polar emits low energy – particles?
(c) Hydrogen molecule can form intermolecular hydrogen (i) Protium (ii) Tritium
bonds but chlorine molecule does not (iii) Deuterium
(d) Hydrogen molecule cannot participate in coordination (a) (i) and (ii) (b) (iii) only
bond formation but chlorine molecule can (c) (ii) only (d) (ii) and (iii)
3. Hydrogen can behave as a metal 11. Hydrogen bond energy is equal to :
(a) at very high temperature (a) 3-7 cals (b) 30-70 cals
(b) at very low temperature (c) 3-10 kcals (d) 30-70 kcals
(c) at very high pressure 12. Which of the following reaction(s) represents commercial
(d) at very low pressure method for production of dihydrogen?
4. The property of hydrogen which distinguishes it from alkali (i) CO(g) + H2O(g)
673K
CO2(g) + H2(g)
catalyst
metals is
electrolysis
(a) its electropositive character (ii) 2H2O(l) traces of acid/base 2H2(g) + O2(g)
(b) its affinity for non metal (iii) Zn + 2H + Zn2+ + H2
(c) its reducing character
(d) its non-metallic character (iv) CH4(g) + H2O(g) 1270 Ni
CO(g) + 3H2(g)
5. Hydrogen accepts an electron to form inert gas (a) (i), (ii) and (iii) (b) (iii) only
configuration. In this it resembles (c) (i), (ii) and (iv) (d) (ii), (iii) and (iv)
EBD_7207
148 HYDROGEN
13. Which of the following is formed when zinc reacts with 23. Very pure hydrogen (99.9) can be made by which of the
sodium hydroxide? following processes ?
(a) Hydrogen gas (b) Sodium zincate (a) Reaction of methane with steam
(c) Zinc oxide (d) Both (a) and (b) (b) Mixing natural hydrocarbons of high molecular weight
14. Identify x and y in following reaction. What is the mixture (c) Electrolysis of water
of x and y called? (d) Reaction of salts like hydrides with water
1270 K 24. Which of the following is formed on reaction of carbon
CH4(g) + H2O(g) x+y
Ni
monoxide gas with dihydrogen in presence of cobalt as
(a) x = CO2, y = H2O, water gas a catalyst?
(b) x = CO, y = H2O, syn gas (a) Methanal (b) Methanol
(c) x = CO, y = H2, water gas (c) Methane (d) Formic acid
(d) x = CO2, y = H2, syn gas 25. Which of the following is not a use of dihydrogen ?
15. Why is water gas (mixture of CO and H2) also called ‘syn (a) It used in fuel cells for generating electrical energy.
gas’? (b) Atomic hydrogen and oxy-hydrogen torches are
(a) Because it is synthesised from sewage, saw – dust, used for cutting and welding purposes.
scrap wood etc. (c) It used in the synthesis of hydroquinone and tartaric
(b) Because it is synthesised from methane gas acid.
(c) Because it is used in the synthesis of methanol and (d) Both (b) and (c)
a number of hydrocarbons. 26. Elements of which of the following group do not form
(d) None of these hydrides?
16. Which of the following statements is correct? (a) Alkali metals (b) Halogens
(a) Production of syngas from coal is called coal (c) Alkaline earth metals (d) Noble gases
gasification. 27. Which of the following statements is incorrect?
(b) CO(g) + H2O (g)
673K
CO2(g) + H2 (g) (a) Ionic hydrides are stoichiometric compounds of
catalyst
dihydrogen formed with most of s-block elements
represents water gas shift reaction. (b) Ionic hydrides are crystalline, non-volatile and non-
(c) CO2 formed in water gas shift reaction is removed by conducting in solid state.
scrubbing with sodium zincate solution. (c) Melts of ionic hydrides conduct electricity and
(d) Both (a) and (b) liberate dihydrogen gas at cathode.
17. Which one of the following pairs of substances on reaction (d) Both (a) and (c)
will not evolve H2 gas? 28. Saline hydrides react explosively with water, such fires can
(a) Iron and H2SO4 (aqueous) be extinguished by
(b) Iron and steam (a) water (b) carbon dioxide
(c) Copper and HCl (aqueous) (c) sand (d) None of these
(d) Sodium and ethyl alcohol 29. Choose the correct option for following hydrides.
18. Which of the following metal evolves hydrogen on reacting B2H6, CH4, NH3 and HF
with cold dilute HNO3 ? (a) Electron deficient hydride = B2H6 and HF
(a) Mg (b) Al Electron precise hydride = CH4
(c) Fe (d) Cu Electron rich hydride = NH3
19. Hydrogen is evolved by the action of cold dil. HNO3 on (b) Electron deficient hydride = B2H6
(a) Fe (b) Mn Electron precise hydride = CH4
(c) Cu (d) Al Electron rich hydride = NH3 and HF
20. In Bosch’s process which gas is utilised for the production (c) Electron deficient hydride = CH4
of hydrogen gas ? Electron precise hydride = B2H6
(a) Producer gas (b) Water gas Electron rich hydride = NH3 and HF
(c) Coal gas (d) None of these (d) Electron deficient hydride = CH4 and HF
21. Hydrogen is not obtained when zinc reacts with Electron precise = B2H6
(a) Cold water (b) dil. HCl Electron rich hydride = NH3,
(c) dil. H2SO4 (d) Hot NaOH (20%) 30. Elements of which of the following group(s) of periodic
22. Which one of the following pairs of substances will not table do not form hydrides.
produce hydrogen when reacted together? (a) Groups 7, 8, 9 (b) Group 13
(a) Copper and conc. nitric acid (c) Groups 15, 16, 17 (d) Group 14
(b) Ethanol and metallic sodium 31. Which hydride is an ionic hydride ?
(c) Magnesium and steam (a) H2S (b) TiH1.73
(d) Phenol and metallic sodium (c) NH3 (d) NaH
HYDROGEN 149
32. Metal hydride on treatment with water gives 43. The boiling point of water is exceptionally high because
(a) H2O2 (b) H2O (a) there is covalent bond between H and O
(c) Acid (d) Hydrogen (b) water molecule is linear
33. The polymeric hydride is (c) water molecules associate due to hydrogen bonding
(a) CaH2 (b) NaH (d) water molecule is not linear
(c) BaH2 (d) MgH2 44. Water possesses a high dielectric constant, therefore :
34. Ionic hydrides reacts with water to give (a) it always contains ions
(a) acidic solutions (b) hydride ions (b) it is a universal solvent
(c) basic solutions (d) electorns (c) can dissolve covalent compounds
35. Which of the following in incorrect statement? (d) can conduct electricity
(a) s-block elements, except Be and Mg, form ionic hydride 45. At its melting point ice is lighter than water because
(b) BeH4 , MgH2, CuH2, ZnH2 , CaH2 and HgH2 are (a) H2O molecules are more closely packed in solid state
intermediate hydride (b) ice crystals have hollow hexagonal arrangement of H2O
(c) p-block elements form covalent hydride molecules.
(d) d-and f-block elements form ionic hydride (c) on melting of ice the H2O molecule shrinks in size
36. Metal hydrides are ionic, covalent or molecular in nature. (d) ice froms mostly heavy water on first melting.
Among LiH, NaH, KH, RbH, CsH, the correct order of 46. The low density of ice compared to water is due to
increasing ionic character is (a) hydrogen-bonding interactions
(a) LiH > NaH > CsH > KH > RbH (b) dipole-dipole interactions
(b) LiH < NaH < KH < RbH < CsH (c) dipole-induced dipole interactions
(c) RbH > CsH > NaH > KH > LiH (d) induced dipole-induced dipole interactions
(d) NaH > CsH > RbH > LiH > KH 47. When two ice cubes are pressed over each other, they unite
37. LiAlH4 is used as : to form one cube. Which of the following forces is
(a) An oxidizing agent (b) A reducing agent responsible to hold them together ?
(a) Hydrogen bond formation
(c) A mordant (d) A water softener
(b) Van der Waals forces
38. Water is : (c) Covalent attraction
(a) more polar than H2S (d) Ionic interaction
(b) more or less identical in polarity with H2S 48. Which of the following reactions is an example of use of
(c) less polar than H2S water gas in the synthesis of other compounds?
(d) None of these
1270K
39. In gas phase water is A molecule with a bond angle (a) CH4(g) + H2O(g) CO(g) + H2(g)
Ni
of B and O–H bond length of C d
673K
(a) A = Bent, B = 100.5°, C = 95.7 pm (b) CO(g) + H2O(g) CO2(g) + H2(g)
Catalyst
(b) A = Bent, B = 104.5°, C = 95.7 pm
1270K
(c) A = Bent, B = 109.5°,C = 99.7 pm (c) CnH2n+2 + nH2O (g) nCO + (2n + 1)H2
Ni
(d) A = Bent, B = 104.5°, C = 99.7 pm
Cobalt
40. The H–O–H angle in water molecule is about (d) CO(g) + 2H2(g) CH3OH(l)
Catalyst
(a) 90º (b) 180º
(c) 102.5° (d) 104.5º 49. Which of the following metals reacts with H 2 O at room
41. Identify the structuer of water in the gaseous phase. temp?
+ (a) Ag (b) Fe
(a) H–O–H (b) H – O – H
(c) Al (d) Na
H
50. Which of the following statements do not define the
2 –
O 95.
7p
characteristic property of water “Water is a universal
(c) m (d) None of these solvent”
+ 104.5° +
H H
(a) It can dissolve maximum number of compounds
42. The unusual properties of water in the condensed phase (b) It has very low dielectric constant
(liquid and solid states) are due to the (c) It has high liquid range
(a) presence of hydrogen and covalent bonding between (d) None of these
the water molecules 51. Which of the following groups of ions makes the water
(b) presence of covalent bonding between the water hard?
molecules (a) Sodium and bicarbonate
(c) presence of extensive hydrogen bonding between (b) Magnesium and chloride
water molecules (c) Potassium and sulphate
(d) presence of ionic bonding (d) Ammonium and chloride.
EBD_7207
150 HYDROGEN
52. The process used for the removal of hardness of water is 65. The structure of H2O2 is
(a) Calgon (b) Baeyer (a) planar (b) non planar
(c) Serpeck (d) Hoope (c) spherical (d) linear
53. When zeolite (hydrated sodium aluminium silicate) is treated 66. The O – O – H bond angle in H2O2 is
with hard water the sodium ions are exchanged with (a) 106° (b) 109 28'
(a) H+ ions (b) Ca2+ ions (c) 120° (d) 94.8°
(c) 2 ions (d) OH– ions 67. Which of the following is the true structure of H2O2 ?
SO 4
54. Calgon used as a water softener is H
|
(a) Na 2 [Na 4 (PO 3 )6 ] (b) Na 4 [ Na 2 (PO 3 ) 6 ] (a) H– O – O – H (b) O O
|
(c) Na 4 [Na 4 (PO 4 )5 ] (d) Na 4 [ Na 2 ( PO 4 ) 6 ] H
55. Polyphosphates are used as water softening agents because
H H
they (c) O=O (d) O O.
(a) form soluble complexes with anionic specise H H
(b) precipitate anionic species 68. In the hydrogen peroxide molecule :
(c) forms soluble complexes with cationic species (a) O – H bonds are polar but molecule is
(d) precipitate cationic species non-polar.
56. Permanent hardness of water can be removed by adding (b) The four atoms are arranged in a non-linear and non-
Calgon (NaPO3)n. This is an example of planar manner.
(a) Adsorption (b) Exchange of ion (c) All the four atoms are in same plane.
(c) Precipitation (d) None (d) Two hydrogen atoms are connected to one of the
oxygen.
57. Which one the following removes temporary hardness of
69. H2O2 is a
water ? (a) Weak acid (b) Weak base
(a) Slaked lime (b) Plaster of Paris (c) Neutral (d) None of these
(c) Epsom (d) Hydrolith 70. When H2O2 is oxidised the product is
58. Permanent hardness of water is due to the presence of (a) OH– (b) O2
(a) bicarbonates of sodium and potassium (c) O2– (d) HO2 –
71. Which of the following is false about H2O2
(b) chlorides and sulphates of sodium and potassium
(a) Act as both oxidising and reducing agent
(c) chlorides and sulphates of calcium and magnesium (b) Two OH bonds lies in the same plane
(d) bicarbonates of calcium and magnesium (c) Pale blue liquid
59. In lab H2O2 is prepared by (d) Can be oxidised by ozone
(a) Cold H2SO4 + BaO2 (b) HCl + BaO2 72. In which of the following reactions, H2O2 acts as a reducing
(c) Conc. H2SO4 + Na2O2 (d) H2 + O2 agent
60. HCl is added to following oxides. Which one would give (a) PbO2(s) + H2O2 (aq) PbO(s) + H2O( ) + O2 (g)
H2O2 (b) Na2SO3 (aq) + H2O2(aq) Na2SO4(aq) + H2O( )
(a) MnO2 (b) PbO2 (c) 2KI(aq) + H2O2 (aq) 2KOH (aq) + I2(s)
(c) BaO (d) None (d) KNO2(aq) + H2O2 (aq) KNO3 (aq) + H2O( )
61. The oxide that gives H2O2 on treatment with dilute H2SO4 73. H2O2 2H+ + O2 + 2e– ; E° = –0.68 V. This equation
is– represents which of the following behaviour of H2O2.
(a) PbO2 (b) BaO2 (a) Reducing (b) Oxidising
(c) MnO2 (d) TiO2 (c) Acidic (d) Catalytic
62. 30 volume hydrogen peroxide means 74. The reaction
(a) 30% of H 2 O 2 solution
H 2S H 2 O 2 S 2H 2 O manifests
(b) 30 cm3 solution contains 1g of H 2 O 2
(a) Acidic nature of H2O2
(c) 1 cm3 of solution liberates 30 cm3 of O2 at STP
(b) Alkaline nature of H2O2
(d) 30 cm3 of solution contains 1 mole of H2O2
(c) Oxidising action of H2O2
63. The volume strength of 1.5 N H 2 O 2 solution is : (d) Reducing action of H2O2.
(a) 8.4 (b) 8.0 75. Which of the following statements is incorrect ?
(c) 4.8 (d) 3.0
(a) H2O2 can act as an oxidising agent
64. Commercial 10 volume H2O2 is a solution with a strength of
approximately (b) H2O2 can act as a reducing agent
(a) 15% (b) 3% (c) H2O2 has acidic properties
(c) 1% (d) 10% (d) H2O2 has basic properties
HYDROGEN 151
76. Consider the reactions 86. What is formed when calcium carbide reacts with heavy
(A) H2O2 + 2HI I2 + 2H2O water?
(B) HOCl + H2O2 H3O+ + Cl– + O2 (a) C2D2 (b) CaD2
Which of the following statements is correct about H2O2 (c) Ca2D2O (d) CD2
with reference to these reactions? Hydrogen peroxide is 87. D2O is used in
______ . (a) motor vehicles (b) nuclear reactor
(a) an oxidising agent in both (A) and (B) (c) medicine (d) insecticide
(b) an oxidising agent in (A) and reducing agent in (B) 88. Complete the following reaction.
(c) a reducing agent in (A) and oxidising agent in (B) Al4C3 + D2O x+y
(d) a reducing agent in both (A) and (B)
(a) x = C2D2 and y = Al(OD)3
77. Which of the following equations depict the oxidising nature
(b) x = CD4 and y = Al(OD)3
of H2O2?
(c) x = CO2 and y = Al2D3
(a) 2MnO4 6H 5H 2 O2 2Mn2+ + 8H2O + 5O2 (d) x = CD4 and y = Al2D3
(b) 2Fe3+ + 2H+ + H2O2 2Fe2+ + 2H2O + O2 89. Which of the following is correct about heavy water ?
(c) 2I– + 2H+ + H2O2 I2 + 2H2O (a) Water at 4°C having maximum density is known as
(d) KIO4 + H2O2 KIO3 + H2O + O2 heavy water
78. Which one of the following undergoes reduction with (b) It is heavier than water (H2O)
hydrogen peroxide in an alkaline medium ? (c) It is formed by the combination of heavier isotope of
(a) Mn 2+ (b) HOCl hydrogen with oxygen
(c) PbS (d) I2 (d) None of these
79. Which of the following does not represent reducing 90. D2O is preferred to H2O, as a moderator, in nuclear reactors
action of H2O2? because
(a) PbS(s) + 4H2O2(aq) PbSO4(s) + 4H2O(l) (a) D2O slows down fast neutrons better
(b) HOCl + H2O2 H3O + + Cl¯ + O2 (b) D2O has high specific heat
(c) 2MnO4– + 3H2O2 2MnO2 + 3O2 +2H2O+2OH¯ (c) D2O is cheaper
(d) I2 + H2O2 + 2OH¯ 2I¯ + 2H2O + O2
(d) None of these
80. Which of the following is not true for hydrogen peroxide?
91. The numbers of protons, electrons and neutrons in a
(a) H2O2 decomposes slowly on exposure to light.
molecule of heavy water are respectively :
(b) It is kept away from dust because dust can induce
(a) 8, 10, 11 (b) 10, 10, 10
explosive decomposition of the compound.
(c) H2O2 is used as bleaching agent for textiles, paper (c) 10, 11, 10 (d) 11, 10, 10
pulp etc. 92. Choose the incorrect statement
(d) It is used as a moderator in nuclear reactor. (a) Dihydrogen can release more energy than petrol.
81. The decomposition of H2O2 is accelerated by – (b) The only pollutant in combustion of dihydrogen is
(a) glycerine (b) alcohol carbon dioxide.
(c) phosphoric acid (d) Pt powder (c) Hydrogen economy is based on the principle of
82. H2O2 is always stored in black bottles because transportation and storage of energy in the form of
(a) It is highly unstable liquid or gaseous dihydrogen
(b) Its enthalpy of decomposition is high (d) Hydrogen economy has advantage that energy is
(c) It undergo auto-oxidation on prolonged standing transmitted in the form of dihydrogen and not as
(d) None of these electric power.
83. H2O2 is 93. Which of the following fuel is used for runnning the
(a) Poor polar solvent than water automobiles first time in the history of India during October
(b) Better polar solvent than H2O 2005?
(c) Both have equal polarity (a) D2O (b) H2O2
(d) Better polar solvent but its strong auto-oxidising ability (c) D2 (d) H2
limits its use as such
84. Which of the following is wrong about H2O2? It is used STATEMENT TYPE QUESTIONS
(a) As aerating agent in production of spong rubber
94. The storage tanks used for H2 are made up of which metal
(b) As an antichlor
alloy(s)
(c) For restoring white colour of blackened lead painting
(d) None of these (i) NaNi5 (ii) B2H6
85.. Heavy water is represented as (iii) Ti–TiH2 (iv) Mg–MgH2
(a) H2 18O (b) D2O (a) (iii) and (iv) (b) (i) and (ii)
(c) D2 18 O (d) H2O at 4°C (c) (i), (iii) and (iv) (d) (ii), (iii) and (iv)
EBD_7207
152 HYDROGEN
95. Which of the following sequence of T and F is correct for
MATCHING TYPE QUESTIONS
given statements? Here T stands for true and F stands for
false statement 100. Match the columns
(i) The H–H bond dissociation enthalpy is highest for Column-I Column-II
a single bond between two atoms of any element (A) Ionic hydrides (p) NiH0.6-0.7
(ii) H2 is relatively inert at room temperature. (B) Molecular hydrides (q) LiH
(iii) Hydrogen combines with almost all the elements due (C) Metallic hydrides (r) HF
to its incomplete orbital (a) A – (q), B – (r), C – (p)
(iv) The atomic hydrogen is produced at high temperature (b) A – (r), B – (q), C – (p)
in an electric arc or under UV radiations. (c) A – (q), B – (p), C – (r)
(a) TTTT (b) FTFT (d) A – (r), B – (p), C – (q)
(c) FTTT (d) FTTF 101. Match the columns
96. Which of the following statement(s) is/are incorrect? Column - I Column - II
(i) Dihydrogen reduces copper (II) oxide to copper (Chemical property (Chemical equation)
(ii) Reaction of dihydrogen with sodium gives sodium of water)
hydride. (A) Basic nature (p) 2H2O(l) + 2Na(s)
(iii) Hydroformylation of olefins yields aldehydes which 2NaOH(aq) + H2(g)
further undergo reduction to give alcohols. (B) Auto-protolysis (q) H2O(l) + H2O(l)
(iv) Hydrogenation of vegetable oils using iron as H 3O (aq)+ OH–(aq)
catalyst gives edible fats.
(a) (i), (ii) and (iii) (b) (i) and (iv) (C) Oxidising nature (r) 2F2(g) + 2H2O(l)
(c) (iv) only (d) (iii) and (iv) 4H+(aq) + 4F–(aq) + O2(g)
97. Choose the correct sequence of T and F for following (D) Reducing nature (s) H2O(l) + H2S(aq)
statements. Here T stands for true and F stands for false H3O+(aq) + HS–(aq)
statement. (a) A – (s), B – (q), C – (r), D – (p)
(i) At atmospheric pressure ice crystallises in the (b) A – (s), B – (q), C – (p), D – (r)
hexagonal form, but at very low temperatures it (c) A – (r), B – (q), C – (s), D – (p)
condenses to cubic form. (d) A – (p), B – (q), C – (s), D – (r)
(ii) Density of ice is less than that of water. Therefore, 102. Match the columns
an ice cube floats on water. Column-I Column-II
(iii) In winter season ice formed on the surface of a lake (A) Clark’s method (p) Mg(HCO3)2 + 2Ca(OH)2
makes survival of the aquatic life difficult. 2CaCO3 + Mg(OH)2
(iv) Hydrogen bonding gives ice a open type structure + 2H2O
with wide holes. (B) Calgon’s method (q) 2NaZ(s) + M2 +(aq)
(a) TTFT (b) FTFT MZ2(s)+ 2Na+(aq)
(c) FTTT (d) TFTT (C) Boiling (r) Ca(HCO3)2
98. Which of the following statements are correct ? CaCO3 + H2O + CO2
(i) Hydrogen peroxide is industrially prepared by the (D) Ion exchange (s) M2+ + Na4P6O18 2–

auto-oxidation of 2-alkylanthraquinols method [Na2MP6O18]2– + 2Na+


(ii) One millilitre of 30% H2O2 means that solution will (a) A – (s), B – (q), C – (r), D – (p)
give 100 V of oxygen at STP (b) A – (q), B – (p), C – (r), D – (s)
(iii) Dihedral angle of H2O2 in gas phase is 90.2° and in (c) A – (p), B – (s), C – (r), D – (q)
solid phase dihedral angle is 111.5° (d) A – (r), B – (q), C – (p), D – (s)
(a) (i), (ii) and (iii) (b) (i) and (iii) 103. Match the columns
Column-I Column-II
(c) (ii) and (iii) (d) (i) and (ii)
(A) Coordinated water (p) [Cu(H2O)4]4+SO2– 4. H2O
99. Some statements about heavy water are given below:
(B) Interstitial water (q) C17H35COONa
(i) Heavy water is used as a moderator in nuclear reactors.
(C) Hydrogen-bonded (r) BaCl2.2H2O
(ii) Heavy water is more associated than ordinary water. water
(iii) Heavy water is more effective solvent than ordinary (s) [Cr(H2O)6]3+ 3Cl–
water. (a) A – (r), B – (s), C – (q)
Which of the above statements are correct? (b) A – (q), B – (r), C – (s)
(a) (i) and (iii) (b) (i) and (ii) (c) A – (r), B – (q), C – (p)
(c) (i), (ii) and (iii) (d) (ii) and (iii) (d) A – (s), B – (r), C – (p)
HYDROGEN 153
104. Match the columns 111. Assertion : Decomposition of H2O2 is a disproportionation
Column-I Column-II reaction.
(A) Heavy water (p) Bicarbonates of Reason : H2 O2 molecule simultaneously undergoes
Mg and Ca in water oxidation and reduction.
(B) Temporary (q) No foreign ions 112. Assertion : H2O2 is not stored in glass bottles.
hard water in water Reason : Alkali oxides present in glass catalyse the
(C) Soft water (r) D2O decomposition of H2O2
(D) Permanent hard (s) Sulphates & chlorides of
water Mg & Ca in water CRITICAL THINKING TYPE QUESTIONS
(a) A – (r), B – (s), C – (q), D – (p)
113. Hydrogen will not reduce
(b) A – (q), B – (r), C – (s), D – (s)
(a) heated cupric oxide (d) heated ferric oxide
(c) A – (q), B – (s), C – (r), D – (p)
(c) heated stannic oxide (d) heated aluminium oxide
(d) A – (r), B – (p), C – (q), D – (s)
114. Which of the following terms is not correct for hydrogen ?
105. Match the Column-I with Column-II and mark the appropriate
(a) Its molecule is diatomic
choice.
(b) It exists both as H+ and H– in different chemical
Column-I Column-II
compounds
(A) Syngas (p) Na6P6O18
(c) It is the only species which has no neutrons in the
(B) Calgon (q) NaAlSiO4
nucleus
(C) Permutit (r) CO + H2
(d) Heavy water is unstable because hydrogen is
(D) Producer gas (s) CO + N2
substituted by its isotope deuterium
(a) (A) – (p), (B) – (q), (C) – (r), (D) – (s)
115. The sum of the number of neutrons and protons in all the
(b) (A) – (r), (B) – (p), (C) – (q), (D) – (s)
three isotopes of hydrogen is
(c) (A) – (r), (B) – (q), (C) – (s), (D) – (p)
(a) 6 (b) 5
(d) (A) – (r), (B) – (q), (C) – (p), (D) – (s)
(c) 4 (d) 3
ASSERTION-REASON TYPE QUESTIONS 116. The hydride ion, H–, is a stronger base than the hydroxide
ion, OH–. Which one of the following reactions will occur if
Directions : Each of these questions contain two statements, sodium hydride (NaH) is dissolved in water?
Assertion and Reason. Each of these questions also has four (a) H (aq) H 2 O(l) H 3O (aq)
alternative choices, only one of which is the correct answer. You (b) H (aq) H 2O(l) OH (aq) H 2 (g)
have to select one of the codes (a), (b), (c) and (d) given below. (c) H (aq ) H 2 O( l) OH (aq ) 2 H ( aq ) 2e
(a) Assertion is correct, reason is correct; reason is a correct (d) H ( aq ) H 2 O ( l) No reaction
explanation for assertion. 117. The reaction of H2S + H2O2 S + 2H2O manifests
(b) Assertion is correct, reason is correct; reason is not a (a) Acidic nature of H2O2
correct explanation for assertion (b) Alkaline nature of H2O2
(c) Assertion is correct, reason is incorrect (c) Oxidising nature of H2O2
(d) Assertion is incorrect, reason is correct. (d) Reducing action of H2O2
106. Assertion : H+ does not exist freely and is always 118. Which of the following is not true?
associated with other atoms or molecules. (a) D2O freezes at lower temperature than H2O
Reason : Loss of the electron from hydrogen atom (b) Reaction between H2 and Cl2 is much faster than D2
results in nucleus (H+) of ~ 1.5 × 10–3 pm size. This is and Cl2
extremely small as compared to normal atomic and ionic (c) Ordinary water gets electrolysed more rapidly than D2O
sizes of 50 to 200 pm. (d) Bond dissociation energy of D2 is greater than H2
107. Assertion : Hydrogen combines with other elements by 119. Heavy water reacts respectively with CO2, SO3, P2O5 and
losing, gaining or sharing of electrons. N2O5 to give the compounds :
Reason : Hydrogen forms electrovalent and covalent bonds (a) D2CO3, D2SO4, D3PO2, DNO2
with other elements. (b) D2CO3, D2SO4, D3PO4, DNO2
108. Assertion : Temporary hardness can be removed by boiling. (c) D2CO3, D2SO3, D3PO4, DNO2
Reason : On boiling the soluble bicarbonates change to (d) D2CO3, D2SO4, D3PO4, DNO3
carbonates which being insoluble, get precipitated. 120. Identify x and y in following reaction:
109. Assertion : Calgon is used for removing permanent electrolysis hydrolysis
hardness of water. 2HSO4 (aq) x
Reason : Calgon forms precipitates with Ca2+ and Mg2+. y + 2H+(aq) + H2O2(aq)
110. Assertion : Hard water is not suitable for laundary. (a) x = H2SO4 (aq), y = 2HSO 4 (aq)
Reason : Soap containing sodium stearate reacts with (b) x = HO3SOOSO3H(aq), y = 2HSO 4 (aq)
hard water to precipitate out as calcium or magnesium (c) x = HO3SOOSO3H (aq), y = H2SO4(aq)
stearate. (d) x = H2SO4(aq) , y = HO3SOOSO3H(aq)
EBD_7207
154 HYDROGEN

19. (b) Mn + 2HNO3 (dil.) Mn(NO3)2 + H2


FACT / DEFINITION TYPE QUESTIONS
catalyst
1. (c) (i) and (iii) are properties of hydrogen which shows 20. (b) CO + H 2+ H2O CO 2+ 2H2
its resemblance with alkali metals whereas (ii) , (iv) water gas
and (v) shows resemblance with halogens. 21. (a) Zinc has no action on cold water.
2. (d) Chlorine has lone pair which it can donate to form 22. (a) Cu + 4HNO3(conc.) Cu(NO3)2 + 2NO2 + 2H2O
coordinate bond while hydrogen cannot.
3. (c) Hydrogen behaves as a metal at very high pressure. C2 H 5OH Na C2 H 5O Na 1 / 2H 2
4. (d) Hydrogen is a non-metal while all other members of
group 1 (alkali metals) are metals. Mg 2H 2 O(steam) Mg(OH) 2 H2
5. (a) H + e– (1s1) H– (1s2 or [He])
C6 H 5OH Na C6 H 5O Na 1 / 2H 2
F + e–([He]2s22p5) F– ( [He] 2s2 2p6 or [Ne])
6. (c) In metal hydrides the O.S. of hydrogen –1 otherwise it NaH H 2 O NaOH H 2
is +1.
7. (d) 23. (d) Very pure hydrogen can be prepared by the action of
water on sodium hydride.
8. (c) (i) Protium, deuterium and tritium are isotopes of
hydrogen. NaH + H2O NaOH + H2
(ii) Ortho and para hydrogens are allotropes of (very pure Hydrogen)
hydrogen. In ortho hydrogen, protons are cobalt
24. (b) CO(g) + 2H2(g) CH3OH(l)
spinning in same direction (parallel spin), while in catalyst
para hydrogen, protons spin in opposite direction 25. (c) Hydrogen is not used in the synthesis of
(antiparallel). hydroquinone and tartaric acid.
9. (a) Number of neutrons in protium, deuterium and 26. (d) Almost all elements except noble gases, forms
tritium respectively is = 0, 1 and 2 hydrides.
10. (c) Tritium is radioactive and emits low energy – particles. 27. (c) Melts of ionic hydrides conduct electricity and
11. (c) Hydrogen bond is weak force of attraction existing liberate dihydrogen gas at anode.
between molecules. Its energy is equal to 3-10 k cal 28. (c) Fire due to action of water on saline hydrides cannot
12. (c) Except method given in statement (iii) all other are be extinguished with water or CO2. These hydrides
commercial methods for production of dihydrogen. can reduce CO2 at high temperature to produce O2.
13. (d) Zn+ 2NaOH Na2ZnO2 + H2 29. (b) Electron deficient hydride = B2H6
Electron precise = CH4
(Sodium zincate)
Electron rich = NH3 and HF
1270 K
14. (c) CH4(g) + H2O(g) Ni
CO + 3H2 30. (a)
Mixture of CO and H2 is called water gas. 31. (d) All metal hydrides are ionic in nature.
15. (c) Mixture of CO and H2 is used in synthesis of 32. (d) Metal hydride +H2O Metal hydroxide + H2
methanol and a number of hydrocarbons due to this 33. (d) Due to its covalent nature MgH2 is Polymeric in nature.
reason it is also called syn gas. 34. (c) Ionic hydrides give the basic solution when it reacts
16. (d) Carbon dioxide formed in water gas shift reaction is with water, e.g.,
removed by scrubbing with sodium arsenite solution. LiH + H2O LiOH + H2
35. (d) d- and f-block elements form metallic hydride.
17. (c) Fe dil. H 2SO 4 FeSO 4 H2 While p-block elements form covalent hydrides, s-block
3Fe 4H 2 O Fe 3O 4 4H 2 elements except Be and Mg form ionic hydrides.
Steam Hydrides of Be, Mg, Cu, Zn, Ca and Hg are intermediate
Cu + dil. HCl No reaction hydride.
Copper does not evolve H2 from acid as it is below 36. (b)
hydrogen in electrochemical series. 37. (b) LiH + AlCl3 (AlH3)n excess Li[AlH 4 ]
LiH
2Na + 2C 2 H 5OH ® 2C 2H 5ONa + H 2 - Lithuim aluminium hydride is a most useful organic
18. (a) Mg dil. HNO 3 Mg( NO 3 ) 2 H 2 (Mg and Mn reducing agent. It reduces functional groups but does
give H2 with dil HNO3) not attack double bonds.
HYDROGEN 155
38. (a) Polarity of bond depends on difference in 52. (a) Calgon process is used to remove permanent hardness
electronegativity of the two concerned atoms. H2O is of water
more polar than H2S because oxygen (in O–H) is more 53. (b) Na zeolite + CaCl 2 Ca zeolite 2NaCl
electronegative than sulphur (in S–H).
54. (a) The complex salt of metaphosphoric acid sodium
39. (b) In gas phase water is a bent molecule with a bond
hexametaphosphate (NaPO3)6, is known as calgon. It
angle of 104.5° and O–H bond length of 95.7 pm.
is represented as Na2[Na4(PO3)6]
40. (d) The hybridisation in water is sp3 and bond angle 104.5º
55. (c) Polyphosphates (sodium hexametaphosphates,
2 –
41. (c) O 95.7 pm O sodium tripolyphosphate or STPP) form soluble
104.5° + + complexes with Ca+2, Mg+2 present in hard water.
H H H H
(i) (ii) 56. (b)
57. (a) This method is known as Clark's process. In this
method temporary hardness is removed by adding lime
water or milk of lime.
Ca(OH)2 Ca(HCO3 )2 2CaCO3 2H 2O
O ppt.
+ +
H H H 58. (c) Permanent hardness of water is due to chlorides and
H sulphates of calcium and magnesium.
(iii) 59. (a) H2SO4 + BaO2 BaSO4 + H2O2
42. (c) The unusual properties of water in the condensed 60. (d) MnO2 , PbO2 and BaO will not give H2O2 with HCl.
phase (liquid an solid states) are due to the presence MnO2 and PbO2 will give Cl2 and BaO will react with
of extensive hydrogen bonding between the water HCl to give BaCl2 and water.
molecules. 61. (b)
43. (c) The high boiling point of water is due to H-bonding. 62. (c) 30 vol of H2O2 means one volume of H2O2 on
44. (b) Due to high dielectric constant, water acts as a good decomposition will give 30 volume of oxygen.
solvent therefore it is also called a universal solvent.
vol. strength
45. (b) In the structure of ice each molecule of H2O is 63. (a) Normality of H2O2 =
surrounded by three H2O molecules in hexagonal 5.6
Volume of normal (1N) H2O2 solution = 5.6 volumes.
honey comb manner which results an open cage like
structure. As a result there are a number of 'hole' or Volume of strength of 1.5 N H2O2
= 1.5 × 5.6 = 8.4 volumes.
open spaces. In such a structure lesser number of
molecules are packed per ml. When ice melts a large 68 10
64. (b) Strength of 10V H2O2 = 100 3.035%
no. of hydrogen bonds are broken. The molecules 22400
therefore move into the holes or open spaces and come 65. (b) Structure of H2O2 is nonplanar
closer to each other than they were in solid state. This 66. (d) O – O – H bond angle in H2O2 is 94.8°.
result sharp increase in the density. Therefore ice has H
lower density than water. 67. (b) O–O is the true structure of H2O2.
46. (a) We know that due to polar nature, water molecules are H
held together by intermolecular hydrogen bonds. The 68. (b)
structure of ice is open with large number of vacant
69. (a) H 2O 2 H 2 O [O]
spaces, therfore the density of ice is less than water. weak acid
47. (a) Two ice cubes stick to each other due to H-bonding.
48. (d) 70. (b) H 2 O2 [O] Oxidation H 2 O O 2
49. (d) Sodium is most electropositive element among those 71. (b) The value of dipole moment of H2O2 is 2.1 D, which
suggest the structure of H2O2 cannot be planar.
given.
2Na 2H 2O 2NaOH H 2 An open-book structure is suggested for H2 O2 in
Room temp. which O – H bonds lie in different plane.
50. (b) Water has high dielectric constant i.e., 78.39 C2/Nm2, 72. (a) In the following reaction H2O2 acts as a reducing agent.
high liquid range and can dissolve maximum number PbO2(s) + H2O2 (aq) PbO(s) + H2O( ) + O2 (g)
of compounds. That is why it is used as universal 73. (a) As H2O2 is loosing electrons so it is acting as reducing
solvent. agent.
51. (b) Temporary hardness is due to presence of bicarbonates
74. (c) H2S is oxidised to S by H2O2.
of calcium and magnesium and permanent hardness is
due to the sulphates and chlorides of both of calcium 75. (d) H2O2 does not have basic properties.
and magnesium. 76. (b) 77. (c)
EBD_7207
156 HYDROGEN
78. (b, d) HOCl(aq) H 2 O 2 (aq)
MATCHING TYPE QUESTIONS
H 3O (aq) Cl (aq) O 2 (g)
100. (a) 101. (b) 102. (c)
I 2 (s) H 2 O 2 (aq) 2OH (aq) 103. (d) Many salts can be crystallised as hydrated salts from
2I (aq) 2H 2 O(l ) O 2 (g) an aqueous solutions such an association of water is
79. (a) Option (a) represents oxidising action of H2O2 in of different types viz.,
acidic medium. (i) Coordinated water e.g., [Cr(H2O)6]3+ 3Cl–
80. (d) H2O2 is not used as a moderator in nuclear reactors (ii) Interstitial water e.g., BaCl2 . 2H2O
81. (d) Decomposition of H2O2 can be accelerated by finely (iii) Hydrogen-bonded water
divided metals such as Ag, Au, Pt, Co, Fe etc. e.g., [Cu(H2O)4]4+ SO2– 4. H2O in CuSO4.5H2O
82. (c) H2O2 is unstable liquid and decomposes into water 104. (d) Heavy water is D2O (1 – C); Temporary hard water
and oxygen either on standing or on heating. contains the bi-carbonates of Mg and Ca (2 – A); Soft
83. (d) Although H2O2 is a better polar solvent than H2O. water contains no foreign ions (3 – B); Permanent
However it cannot be used as such because of the hard water contains the sulphates and chlorides of
strong auto-oxidation ability. Mg and Ca (4 – D) therefore the answer is D.
84. (d) H2O2 show all these properties. 105. (b)
85. (b) The formula of heavy water (deuterium oxide) is D2O.
86. (a) CaC2 2D 2 O C 2 D 2 Ca(OD) 2 ASSERTION-REASON TYPE QUESTIONS
87. (b) D2O is used in nuclear reactors as moderator. 106. (a) Due to extremely small size of H+ as compared to
88. (b) Al4C3 + D2O 3CD4 + 4Al(OD)3 normal atomic and ionic size H+ does not exist freely.
89. (c) Heavy water is formed by the combination of heavier 107. (a) 108. (a)
isotope (1H2 or D) with oxygen. 109. (c) Both assertion is correct reason is not true.
2D 2 O 2 2D 2 O Correct reason : Calgon forms soluble complexes with
Heavy water
Ca2+ and Mg2+ in which properties of these ions are
90. (d) H2 O absorbs neutrons more than D2 O and this masked.
decreases the number of neutrons for the fission 110. (a) 2C17H35COONa(aq) + M2+ (aq)
process.
(C17H35COO)2M (M = Ca or Mg) + 2Na+(aq)
91. (b) Heavy water is D2O hence
111. (a) Both assertion and reason are true and reason is the
number of electrons = 2 + 8 = 10 correct explanation of assertion.
number of protons = 10 112. (a)
Atomic mass of D2O = 4 + 16 = 20
hence number of neutron CRITICAL THINKING TYPE QUESTIONS
= Atomic mass – number of protons
= 20 – 10 = 10 113. (d) H2 will not reduce heated Al2O3. As Al is more electro-
92. (b) The only pollutant in combustion of dihydrogen is positive than hydrogen. therefore, its oxide will not be
oxides of dinitrogen (due to the presence of reduced by hydrogen.
dinitrogen as impurity with dihydrogen). 114. (d) Heavy water is stable.
93. (d) It is for the first time in the history of India that a pilot 115. (a) 1H1 1D2 1T3
project using dihydrogen as fuel was launched in no. of neutrons respectively are 0 , 1 , 2
October 2005 for running automobiles. Initially 5% H2 no. of protons respectively are 1 , 1 , 1
has been mixed in CNG for use in four wheeler vehicles. Hence the sum of protons + neutrons = 1 + 2 + 3 = 6
116. (b) H (aq) H 2 O(l) OH (aq) H 2 (g)
STATEMENT TYPE QUESTIONS base 1 acid 1 base 2 acid2

94. (c) Tanks of metal alloy like NaNi5, Ti – TiH2, Mg – MgH2 In this reaction H–acts as bronsted base as it accepts
etc are used for storage of dihydrogen in small one proton (H+) from H2O and for H2.
quantities. 2 0
95. (a) 117. (c) H 2 S H 2 O2 S 2H 2O
96. (c) Hydrogenation occurs in presence of nickel as a In this reaction H2O2 shows oxidising nature.
catalyst. 118. (a) D2O actually has higher freezing point (3.8°C) than
97. (a) In winter seasons ice formed on the surface of a lake water H2O (0°C)
provides thermal insulation which ensures the 119. (d)
survival of the aquatic life.
electrolysis
98. (d) Dihedral angle of H2O2 in gas phase is 111.5° and 120. (b) 2HSO4– (aq) HO3SOOSO3H(aq)
in solid phase it is 90.2° Hydrolysis
99. (b) 2HSO 4 (aq) + 2H+(aq) + H2O2(aq)
10
THE s-BLOCK ELEMENTS

FACT / DEFINITION TYPE QUESTIONS 8. Which of the following has density greater than water?
(a) Li (b) Na
1. Group 2 elements are called alkaline earth metals why? (c) K (d) Rb
Choose the correct reason(s).
9. The elements of group 1 provide a colour to the flame of
(i) Hydroxides formed by group 2 elements are alkaline
Bunsen burner due to
in nature.
(a) low ionization potential
(ii) Their metal oxides are found in the earth’s crust.
(iii) Their oxides are alkaline in nature (b) low melting point
(iv) Group 2 elements react with alkalies. (c) softness
(a) (i) and (ii) (b) (ii) and (iv) (d) presence of one electron in the outermost orbit
(c) (i), (ii) and (iii) (d) (ii) and (iii) 10. The metal that produces red-violet colour in the non-
2. Which of the following alkali metal is highly radioactive? luminous flame is
(a) Rubidium (b) Caesium (a) Ba (b) Ag
(c) Francium (d) Both (a) and (c) (c) Rb (d) Pb
3. Which of the following are found in biological fluids Na+, 11. The alkali metals have low melting point. Which of the
Mg2+, Ca2+, K+, Sr2+, Li+ and Ba2+ following alkali metal is expected to melt if the room
(a) Mg2+, Ca2+, and Sr2+ temperature rises to 30°C?
(b) Na2 +and K+ (a) Na (b) K
(c) Na+, K+, Mg2+and Ca2+ (c) Rb (d) Cs
(d) Sr+, Li and Ba2+
12. In the case of the alkali metals
4. Which of the following statements is not correct for alkali
metals? (a) the cation is less stable than the atom
(a) Alkali metals are the most electropositive metals. (b) the cation is smaller than the atom
(b) Alkali metals exist in free state in nature. (c) the cation and the atom have about the same size
(c) These metals have the largest size in a particular
(d) the cation is larger than the atom
period of the periodic table.
(d) Both (b) and (c) 13. Which of the following is not correct ?
5. Which of the following has largest size ? 2Li 2 O
heat
Li 2 O 2 2Li
(a)
(a) Na (b) Na+ 673K

(c) Na – (d) Can’t be predicted heat


(b) 2K 2O K 2O2 2K
6. Ionization potential of Na would be numerically the same as 673K
(a) electron affinity of Na+ heat
(b) electronegativity of Na+ (c) 2Na 2 O Na 2O 2 2Na
673K
(c) electron affinity of He
heat
(d) ionization potential of Mg (d) 2Rb 2O Rb 2 O 2 2Rb
673K
7. Which one of the following properties of alkali metals
14. The element which on burning in air gives peroxide is
increases in magnitude as the atomic number rises ?
(a) Ionic radius (b) Melting point (a) lithium (b) sodium
(c) Electronegativity (d) First ionization energy. (c) rubidium (d) caesium
EBD_7207
158 THE s-BLOCK ELEMENTS
15. Which one of the alkali metals, forms only, the normal oxide, 26. Na metal is stored in
M2O on heating in air ?
(a) C6H6 (b) kerosene
(a) Rb (b) K
(c) alcohol (d) toluene
(c) Li (d) Na
16. Which of the following is used as a source of oxygen in 27. Which of the following metal is used along with lithium
space capsules, submarines and breathing masks ? to make the alloy named ‘white metal’ ?
(a) Nickel (b) Aluminium
(a) Li 2O (b) Na 2O 2
(c) Silver (d) Lead
(c) KO2 (d) K 2O 2 28. Which of the following metal is used as a coolant in
17. The ionic mobility of alkali metal ions in aqueous solution is breeder nuclear reactors?
maximum for (a) Potassium (b) Sodium
(a) Li+ (b) Na+ (c) Caesium (d) Rubidium
(c) K + (d) Rb+ 29. Which is most basic in character ?
18. For an aqueous solution under an electric field which of (a) CsOH (b) KOH
the following have lowest mobility ?
(c) NaOH (d) LiOH
(a) Li+ (b) Na+
+ 30. Which compound will show the highest lattice energy ?
(c) K (d) Rb+
(a) RbF (b) CsF
19. Which of the following pairs of substances would give
same gaseous product on reaction with water? (c) NaF (d) KF
(a) Na and Na2O2 (b) Ca and CaH2 31. In crystals which one of the following ionic compounds
(c) Ca and CaO (d) Ba and BaO2 would you expect maximum distance between centres of
20. Which is the most basic of the following? cations and anions?
(a) LiF (b) CsF
(a) Na 2O (b) BaO
(c) CsI (d) LiI
(c) As2O3 (d) Al2O3 32. Among LiI, NaI, KI, the one which is more ionic and more
21. Which hydride is most stable soluble in water is
(a) NaH (b) KH (a) KI (b) NaI
(c) CsH (d) LiH (c) LiI (d) None of these
22. The most stable compound is 33. The products obtained on heating LiNO2 will be
(a) LiF (b) LiCl
(a) Li 2 O NO2 O2 (b) Li 3 N O2
(c) LiBr (d) LiI
23. Which of the following represents a correct sequence of (c) Li 2 O NO O 2 (d) LiNO3 O2
reducing power of the following elements?
34. On heating anhydrous Na2CO3,.......is evolved
(a) Li > Cs > Rb (b) Rb > Cs > Li
(a) CO2 (b) water vapour
(c) Cs > Li > Rb (d) Li > Rb > Cs
(c) CO (d) no gas
24. What is the colour of solution of alkali metals in liquid
ammonia? 35. Complete the following two reactions.
(a) Bronze (b) Blue (i) 4LiNO3 x + O2
(c) Green (d) Violet (ii) 2NaNO3 y + O2
25. The alkali metals dissolve in liquid ammonia giving deep (a) x = LiNO2, y = NaNO2
blue solution. The solution is __x ___. In concentrated (b) x = Li2O + NO2, y = Na2O + NO2
solution, the blue colour changes to ___y__ and becomes (c) x = Li2O + NO2, y = NaNO2
__z___ (d) x = LiNO2, y = Na2O + NO2
(a) x = paramagnetic y = colourless 36. Which of the following does not illustrate the anomalous
z = diamagnetic properties of lithium?
(b) x = diamagnetic y = colourless
(a) The melting point and boiling point of Li are
z = paramagnetic comparatively high
(c) x = paramagnetic y = bronze
(b) Li is much softer than the other group I metals
z = diamagnetic
(c) Li forms a nitride Li3N unlike group I metals
(d) x = paramagnetic y = black
(d) The ion of Li and its compounds are more heavily
z = diamagnetic
hydrated than those of the rest of the group
THE s-BLOCK ELEMENTS 159
37. Why lithium react less vigorously with water than other 45. Which of the following is/are present as impurity in crude
alkali metals? sodium chloride, obtained by crystallisation of brine
(a) Lithium has most negative E value solution?
(i) Sodium sulphate (ii) Calcium chloride
(b) Lithium has small size and very high hydration
energy. (iii) Magnesium chloride (iv) Potassium chloride
(a) (i), (ii) and (iv) (b) (ii) and (iii)
(c) Lithium has least negative E value
(c) (iii) and (iv) (d) (i), (ii) and (iii)
(d) Both (a) and (b)
46. Which is manufactured by electrolysis of fused NaCl ?
38. Identify the correct statement
(a) NaOH (b) Na
(a) Elemental sodium can be prepared and isolated by
electrolysing an aqueous solution of sodium chloride (c) NaClO (d) NaClO3.
(b) Elemental sodium is a strong oxidising agent 47. Baking soda is
(a) NaHCO3 (b) K2CO3
(c) Elemental sodium is insoluble in ammonia
(c) Na2CO3 (d) NaOH
(d) Elemental sodium is easily oxidised
48. Baking powder contains :
39. Washing soda has formula
(a) NaHCO3, Ca(H2PO2)2 and starch
(a) Na2CO3.7H2O (b) Na2CO3.10H2O (b) NaHCO3, Ca(H2PO2)2
(c) Na2CO3.3H2O (d) Na2CO3 (c) NaHCO3, starch
40. The process associated with sodium carbonate manufacture (d) NaHCO3
is known as 49. Which of the following is the most abundant ion within
(a) Chamber (b) Haber cell fluids?
(a) Sodium ions (b) Potassium ions
(c) LeBlanc (d) Castner
(c) Calcium ions (d) None of these
41. In Solvay ammonia process, sodium bicarbonate is 50. Which of the following is non-metallic?
precipitate due to (a) B (b) Be
(a) presence of NH3 (c) Mg (d) Al
(b) reaction with CO2 51. Electronic configuration of calcium atom may be written as
(a) [Ne], 4p2 (b) [Ar], 4s2
(c) reaction with brine solution 2
(c) [Ne], 4s (d) [Ar], 4p2
(d) reaction with NaOH 52. The outer electronic configuration of alkaline earth metal is
42. Sodium carbonate is manufactured by Solvay process. The (a) ns 2 (b) ns 1
products which can be recycled are (c) np 6 (d) nd10
(a) CO2 and NH3 (b) CO2 and NH4Cl 53. Which of the following atoms will have the smallest size ?
(c) NaCl and CaO (d) CaCl2 and CaO. (a) Mg (b) Na
43. How NH3 is recovered in Solvay process? (c) Be (d) Li
(a) By reaction of NH4Cl and Ca(OH)2 54. The first ionization energy of magnesium is lower than the
first ionization energy of
(b) By reaction of NH4HCO3 and NaCl
(a) Lithium (b) Sodium
(c) By reaction of (NH4)2CO3 with H2O
(c) Calcium (d) Beryllium
(d) By any of the above
55. Which of the following relations is correct with respect to
44. Why Solvay process cannot be extended to the first (I) and second (II) ionization potentials of sodium and
manufacture of potassium carbonate? magnesium?
(a) Ammonium hydrogen carbonate does not react with
(a) I Mg = II Na (b) I Mg II Na
potassium chloride.
(b) Potassium hydrogen carbonate is too soluble to be (c) I Na I Mg (d) II Na IIMg
precipitated by the addition of ammonium 56. The first ionization energies of alkaline earth metals are
hydrogencarbonate to a saturated solution of higher than those of alkali metals. This is because
potassium chloride. (a) there is no change in the nuclear charge
(c) Ammonium carbonate is precipitated out instead of (b) there is decrease in the nuclear charge of alkaline earth
potassium hydrogen carbonate on reaction of metals
ammonium hydrogen carbonate with potassium (c) there is increase in the nuclear charge of alkaline earth
chloride metals
(d) None of the above (d) none of these
EBD_7207
160 THE s-BLOCK ELEMENTS
57. Which of the following has maximum ionization energy 67. Arrange the following compounds in order of increasing
(a) Ba Ba e (b) Be solubility
Be e
(i) MgF2 (ii) CaF2,
(c) Ca Ca 2 2e (d) Mg Mg 2 2e (iii) BaF2
58. The most electropositive amongst the alkaline earth metals (a) (i) < (ii) < (iii) (b) (ii) < (i) < (iii)
is (c) (ii) < (iii) < (ii) (d) (iii) < (ii) < (i)
(a) beryllium (b) magnesium 68. Alkaline earth metals are
(c) calcium (d) barium (a) reducing agent (b) amphoteric
59. Alkaline earth metals are not found free in nature because (c) dehydrating agent (d) oxidizing agent
of 69. The oxidation state shown by alkaline earth metals is
(a) their thermal instability (a) +2 (b) +1, +2
(b) their low melting points (c) –2 (d) –1, –2
(c) their high boiling points 70. Which one of the following is the most soluble in water?
(d) their greater chemical reactivity
(a) Mg(OH)2 (b) Sr(OH)2
60. A firework gives out crimson coloured light. It contains a
(c) Ca(OH) 2 (d) Ba(OH) 2
salt of
(a) Ca (b) Na 71. Which of the following alkaline earth metal hydroxides is
amphoteric in character
(c) Sr (d) Ba
(a) Be(OH)2 (b) Ca(OH)2
61. Following are colours shown by some alkaline earth
(c) Sr(OH)2 (d) Ba(OH)2
metals in flame test. Which of the following are not
correctly matched? 72. Of the metals Be, Mg, Ca and Sr of group 2 A. In the periodic
table the least ionic chloride would be formed by
Metal Colour
(a) Be (b) Mg
(i) Calcium Apple green
(c) Ca (d) Sr
(ii) Strontium Crimson
73. The order of solubility of sulphates of alkaline earth metals
(iii) Barium Brick red
in water is
(a) (i) and (iii) (b) (i) only
(a) Be > Mg > Ca > Sr > Ba
(c) (ii) only (d) (ii) and (iii)
(b) Mg > Be >> Ba > Ca > Sr
62. Which one of the following properties of alkali metals
(c) Be > Ca > Mg > Ba >> Sr
increases in magnitude as the atomic number rises ?
(d) Mg > Ca > Ba >> Be > Sr
(a) Ionic radius (b) Melting point
74. The solubilities of carbonates decrease down the magnesium
(c) Electronegativity (d) First ionization energy group due to a decrease in
63. Out of the following elements which one do you expect to (a) hydration energies of cations
be most reactive, chemically ?
(b) inter-ionic attraction
(a) Mg (b) Ca
(c) entropy of solution formation
(c) Sr (d) Ba
(d) lattice energies of solids
64. In the reaction Mg H2O X H 2 ; X is 75. The correct order of increasing thermal stability of K2CO3,
(steam) MgCO3, CaCO3 and BeCO3 is
(a) MgO (b) Mg(OH)2 (a) BeCO3< MgCO3 < CaCO3 < K2CO3
(c) MgH2 (d) None of these (b) MgCO3 < BeCO3 < CaCO3 < K2CO3
65. The metals A and B form oxide but B also forms nitride when (c) K2CO3 < MgCO3 < CaCO3 < BeCO3
both burn in air. The A and B are (d) BeCO3 < MgCO3 < K2CO3 < CaCO3
(a) Cs, K (b) Mg, Ca 76. In which of the following the hydration energy is higher
(c) Li, Na (d) K, Mg than the lattice energy?
66. Which of the following is the best method for preparation (a) MgSO4 (b) RaSO4
of BeF2? (c) SrSO4 (d) BaSO4
(a) Reaction of Be with F2 77. Which of the following alkaline earth metal sulphates has
(b) Thermal decomposition of (NH4)2BeF4 hydration enthalpy higher than the lattice enthalpy?
(c) Reaction of Be with HF (a) CaSO 4 (b) BeSO 4
(d) All of the above are equally effective (c) BaSO 4 (d) SrSO 4
THE s-BLOCK ELEMENTS 161
78. Beryllium shows diagonal relationship with aluminium. 90. For a good quality cement what should be the ratio of
Which of the following similarity is incorrect ? following :
(a) Be forms beryllates and Al forms aluminates I. Silica to alumina
(b) Be(OH)2 like Al(OH)3 is basic. II. CaO to the total of oxides of SiO2, Al2O3 and Fe2O3
(c) Be like Al is rendered passive by HNO 3 . (a) I = 2.5 to 4
II = Greater than 2
(d) Be2C like Al4C3 yields methane on hydrolysis.
(b) I = Nearly 4
79. The substance not likely to contain CaCO3 is
II = Less than 2
(a) gypsum (b) sea shells
(c) I = 2.5
(c) dolomite (d) a marble statue
II = Closer to 2
80. Plaster of Paris is
(d) I = 2.5 to 4
(a) CaSO 4 .2H 2 O (b) CaSO 4 .H 2 O II = Closer to 2
91. Calcitonin and parathyroid hormone regulate concentration
1
(c) CaSO 4 . H 2 O (d) CaSO 4 .4H 2 O of which of the following element in plasma?
2
(a) Calcium (b) Magnesium
81. Gypsum on heating at 120 – 130°C gives (c) Sodium (d) Potassium
(a) anhydrous salt (b) hemihydrate 92. Which of the following metal is found in green colouring
(c) monohydrate (d) dihydrate pigment chlorophyll of plants?
82. Plaster of Paris on making paste with little water sets to (a) Fe (b) Mg
hard mass due to formation of (c) Na (d) Al
(a) CaSO4 (b) CaSO4.1/2H2O
(c) CaSO4.H2O (d) CaSO4.2H2O STATEMENT TYPE QUESTIONS
83. The chemical which is used for plastering the broken bones
93. Select the correct statements
is
(i) Cs+ is more highly hydrated that the other alkali metal
(a) (CaSO4)2H2O (b) MgSO4.7H2O
ions
(c) FeSO4. 7H2O (d) CuSO4. 5H2O
(ii) Among the alkali metals Li, Na, K and Rb, lithium has
84. Dead burn plaster is
the highest melting point
(a) CaSO4.2H2O (b) MgSO4. 7H2O
(iii) Among the alkali metals only lithium forms a stable
(c) CaSO4.½ H2O (d) CaSO4 nitride by direct combination with nitrogen
85. The formula for calcium chlorite is (a) (i), (ii) and (iii) (b) (i) and (ii)
(a) Ca(ClO4)2 (b) Ca(ClO3)2 (c) (i) and (iii) (d) (ii) and (iii)
(c) CaClO2 (d) Ca(ClO2)2 94. Which of the following sequence of T and F is correct for
86. Bone ash contains alkali metals ? Here T represents True and F represents
(a) CaO (b) CaSO4 False statement.
(c) Ca3 (PO4)2 (d) Ca(H2PO4)2 (i) Alkali metal hydrides are ionic solids with high
87. Mortar is a mixture of melting point.
(a) CaCO3, sand and water (ii) All alkali halides are ionic in nature.
(b) slaked lime and water (iii) Li is the least powerful reducing agent and Na is the
(c) slaked lime, sand and water most powerful reducing agent.
(d) CaCO3 and CaO (a) TTT (b) TFT
88. Which gas is released when CaCO3 reacts with dilute (c) FTF (d) TFF
HCl? 95. Which of the following statement(s) is/are correct regarding
(a) H2 (b) CO2 Li2CO3 and Na2CO3 ?
(c) O2 (d) Cl2 (i) Sodium salt evolve CO2 at higher temperature.
89. Setting of cement is an (ii) Polarization of Na+ is lesser than that of Li+.
(a) exothermic reaction (a) Both statements (i) and (ii) are correct
(b) endothermic reaction (b) Both statements (i) and (ii) are incorrect
(c) neither endothermic nor exothermic (c) Statement (ii) is correct explanation for statement (i)
(d) example of neutralisation reaction (d) Statement (i) is correct explanation for statement (ii)
EBD_7207
162 THE s-BLOCK ELEMENTS
96. Which of the following sequence of T and F is correct for
MATCHING TYPE QUESTIONS
given statements?
(i) The alkali metal hydroxides are the strongest of all 101. Match the columns
bases. Column-I Column-II
(ii) All alkali metal halides have high negative enthalpies (Alkali metal) (Colour imparted to an
of formation. oxidizing flame)
(iii) The stability of the carbonates and hydrogen (A) Cs (p) Yellow
carbonates of alkali metals decrease with increase in (B) Rb (q) Blue
electropositive character down the group. (C) K (r) Violet
(iv) Only LiHCO3 exist as solid. (D) Na (s) Red violet
(a) TTFF (b) TTTT (E) Li (t) Crimson red
(c) FTFT (d) TFFT (a) A – (q), B – (s), C – (r), D – (p), E – (t)
97. Which of the following statement(s) is/are correct? (b) A – (s), B – (q), C – (r), D – (p), E – (t)
(i) The atomic and ionic radii of alkaline earth metals are (c) A – (t), B – (r), C – (s), D – (p), E – (q)
smaller than those of the corresponding alkali metals (d) A – (q), B – (p), C – (r), D – (p), E – (t)
in the same periods. 102. Match the columns
(ii) Second ionisation enthalpies of the alkaline earth Column-I Column-II
metals are smaller than those of the corresponding (Metal) (Oxide formed on burning)
alkali metals. (A) Caesium (p) Superoxide
(iii) Compounds of alkaline earth metals are more (B) Lithium (q) Peroxide
extensively hydrated than those of alkali metals
(C) Sodium (r) Monoxide
(a) (i) and (ii) (b) (ii) and (iii)
(a) A – (q), B – (p), C – (r)
(c) (i) and (iii) (d) (i), (ii) and (iii)
(b) A – (r), B – (q), C – (p)
98. Which of the following statements are correct ? (c) A – (p), B – (r), C – (q)
(i) Copper - beryllium alloys are used in the preparation (d) A – (q), B – (r), C – (p)
of high strength springs
103. Match the columns. Here Column-I shows the names of
(ii) Metallic beryllium is used for making window X-ray the metals used with lithium to make useful alloys and
tubes. Column-II shows the uses of these alloys
(iii) Magnesium powder is used in incendiary bombs and Column-I Column-II
singnals.
(A) Aluminium (p) Armour plates
(iv) Barium is used in treatment of cancer.
(B) Magnesium (q) Aircraft parts
(a) (i), (ii) and (iv) (b) (i) and (iii) (C) Lead (r) Bearings for motor
(c) (i), (ii) and (iii) (d) (i), (ii), (iii) and (iv) engines.
99. Which of the following is/are not characteristic (a) A – (q), B – (p), C – (r)
property(ies) of alkaline earth metals ? (b) A – (q), B – (r), C – (p)
(i) All alkaline earth metal oxides are basic in nature and (c) A – (p), B – (q), C – (r)
forms sparingly soluble hydroxides with water.
(d) A – (p), B – (r), C – (q)
(ii) The hydrated chlorides, bromides and iodies of Ca,
104. Match the columns.
Sr and Ba on heating undergoes hydrolysis while
Column-I Column-II
corresponding hydrated halides of Be and Mg on
heating undergo dehydration. (Sodium compound) (Uses)
(iii) Nitrates of alkaline earth metals decompose on (A) Sodium carbonate (p) In fire extinguisher
heating as below (B) Sodium chloride (q) In manufacture of
2M(NO3)2 2MO + 4NO2 + O2 glass, soap, borax
(a) (i) only (b) (ii) only and caustic soda.
(c) (i) and (iii) (d) (i) and (ii) (C) Sodium hydroxide (r) In preparation of Na2O2,
NaOH and Na2CO3
100. Which of the following statement(s) is/are correct regarding
Al and Be ? (D) Sodium hydrogen (s) In petroleum refining
(i) Both of these react with alkali. carbonate
(a) A – (q), B – (r), C – (s), D – (p)
(ii) There is diagonal relationship among these elements.
(b) A – (s), B – (q), C – (r), D – (p)
(a) Both (i) and (ii) (b) Only (i)
(c) A – (p), B – (s), C – (r), D – (q)
(c) Only (ii) (d) Neither (i) nor (ii)
(d) A – (s), B – (r), C – (p), D – (q)
THE s-BLOCK ELEMENTS 163

105. Match the columns CRITICAL THINKING TYPE QUESTIONS


Column-I Column-II
111. The melting point of lithium (181°C) is just double the melting
(A) Quick lime (p) Ca(OH)2 point of sodium (98°C) because
(B) Slaked lime (q) CaO (a) down the group, the hydration energy decreases
(C) Bleaching powder (r) Ca(OCl)2 (b) down the group, the ionization energy decreases
(D) Plaster of Paris (s) CaSO4. H2O (c) down the group the cohesive energy decreases
(a) A – (p), B – (q), C – (r), D – (s) (d) None of these
(b) A – (s), B – (r), C – (q), D – (p) 112. Li has the maximum value of ionisation potential among
alkali metals i.e. lithium has the minimum tendency to ionise
(c) A – (q), B – (p), C – (r), D – (s) to give LI+ ion. Thus, in aq. solution lithium is
(d) A – (q), B – (p), C – (s), D – (r) (a) strongest reducing agent
106. Match the columns (b) poorest reducing agent
(A) Quick lime (p) Setting fractured bones (c) strongest oxidising agent
(B) Plaster of Paris (q) A constituent of chewing gum (d) poorest oxidising agent
(C) Slaked lime (r) Manufacture of bleaching 113. Lithium is strongest reducing agent among alkali metals
due to which of the following factor?
powder
(a) Ionization energy (b) Electron affinity
(D) Limestone (s) Manufacture of dyestuffs
(c) Hydration energy (d) Lattice energy
(a) A – (p), B – (s), C – (q), D – (r) 114. Which of the following statements is incorrect?
(b) A – (s), B – (p), C – (r), D – (q) (a) Alkali metal hydroxide are hygroscopic
(c) A – (q), B – (r), C – (p), D – (s) (b) Dissolution of alkali metal hydroxide is endothermic
(d) A – (r), B – (q), C – (s), D – (p) (c) Aqueous solution of alkali metal hydroxides are
strongly basic
ASSERTION-REASON TYPE QUESTIONS (d) Alkali metal hydroxides form ionic crystals
115. Which of the following on thermal decomposition yields a
Directions : Each of these questions contain two statements, basic as well as acidic oxide ?
Assertion and Reason. Each of these questions also has four (a) NaNO3 (b) KClO3
alternative choices, only one of which is the correct answer. You (c) CaCO3 (d) NH4NO3
have to select one of the codes (a), (b), (c) and (d) given below. 116. Which one of the following on hydrolysis, gives the
(a) Assertion is correct, reason is correct; reason is a correct corresponding metallic hydroxide, H2O2 and O2?
explanation for assertion. (a) Li2O (b) Na2O2
(b) Assertion is correct, reason is correct; reason is not a (c) NaO2 (d) Na2O
correct explanation for assertion 117. Which of the following oxides of potassium is not known ?
(c) Assertion is correct, reason is incorrect (a) K 2O (b) K 2O 4
(d) Assertion is incorrect, reason is correct. (c) KO3 (d) K 2O 3
107. Assertion: Lithium salts are mostly hydrated. 118. Suppose an element is kept in air chamber, than air content
Reason : The hydration enthalpy of alkali metal ions was evaluated after sometime , oxygen and nitrogen content
decreases with increase in ionic sizes. was found to be low comparitively. The given element will
108. Assertion : Lithium carbonate is not so stable to heat. be
(a) Li (b) Rb
Reason : Lithium being very small in size polarizes large
(c) Na (d) K
CO32 ion leading to the formation of more stable Li2O 119. Suppose metal react with the oxygen to form oxide, than
aqueous solution of this oxide when added to a solution of
and CO2
HI, solution turn yellowish brown in colour. This compound
109. Assertion : Compounds of beryllium is largely covalent is
and get hydrolysed easily. (a) Na2O (b) Li2O
Reason : This is due to high value of ionisation potential (c) NaOH (d) Na2O2
and small size of Be. 120. Which of the following salt of lithium is most soluble in
110. Assertion : Radium is most abundant s-block element. organic solvent ?
(a) LiF (b) LiCl
Reason : s-block elements are non-radioactive in nature.
(c) LiBr (d) LiI
EBD_7207
164 THE s-BLOCK ELEMENTS
121. Arrange the following in increasing order of their melting (a) A < B < C < D (b) A < C < B < D
point ? (c) C < A < B < D (d) C < B < A < D
(A) LiCl, (B) NaCl, (C) KCl 128. Which of the following will precipitate first when aqueous
(a) A < B < C (b) B < A < C solution containing sulphate ions are added?
(c) C < A < B (d) A < B C (a) Mg2+ (b) Ca2+
122. The raw materials in Solvay Process are (c) Sr 2+ (d) Ba2+
(a) Na 2 CO 3 , CaCO 3 and NH 3 129. If the fluoride salts of group 2 metals are dissolved in
water,than which of the following will show high solubility?
(b) Na 2SO 4 , CaCO3 and NH 3 (a) BaF2 (b) RbF2
(c) NaCl , NH3 and CaCO3. (c) CaF2 (d) BeF2
130. Aqueous solution of group 2 is precipitated by adding
(d) NaOH, CaO and NH 3 .
Na2CO3, then this precipitate is tested on flame, no light in
123. Compared with the alkaline earth metals, the alkali metals visible region is observed , this element can be
exhibit
(a) Ba (b) Mg
(a) smaller ionic radii (b) highest boiling points
(c) Ca (d) Sr
(c) greater hardness (d) lower ionization energies.
131. Which of the following statement is false ?
124. Property of the alkaline earth metals that increases with
(a) Strontium decomposes water readily than beryllium
their atomic number is
(a) solubility of their hydroxides in water (b) Barium carbonate melts at a higher temperature than
calcium carbonate
(b) solubility of their sulphates in water
(c) Barium hydroxide is more soluble in water than
(c) ionization energy
magnesium hydroxide
(d) electronegativity
(d) Beryllium hydroxide is more basic than barium
125. Which one of the following does not react with water even
hydroxide.
under red hot condition?
(a) Na (b) Be 132. Bleaching powder is obtained by the interaction of chlorine
with
(c) Ca (d) K
126. Magnesium burns in CO2 to form (a) dil. solution of Ca(OH) 2
(a) MgO + C (b) MgO + CO (b) dry CaO
(c) MgCO3 (d) MgO. (c) conc. solution of Ca(OH) 2
127. Arrange the following in increasing order of their solubility? (d) dry slaked lime
MgCO3(A), CaCO3(B), SrCO3(C), Na2CO3(D)
THE s-BLOCK ELEMENTS 165

FACT / DEFINITION TYPE QUESTIONS 15. (c) All the alkali metals when heated with oxygen form
different types of oxides for example lithium forms
1. (c) Group 2 elements are called alkaline earth metals as lithium oxide (Li2O), sodium forms sodium peroxide
their oxides and hydroxides are alkaline in nature and (Na2O2), while K, Rb and Cs form their respective
these metal oxides are found in the earth’s crust. superoxides.
2. (c) Francium is highly radioactive. 1
2Li O Li 2O
3. (c) Monovalent sodium and potassium ions and divalent 2 2
magnesium and calcium ions are found in large 16. (c) Because KO2 not only provides O2 but also removes
proportions in biological fluids. 1CO2 as follows
4. (b) Alkali metals readily lose electron to give monovalent 4KO2 2CO2 2K 2 CO3 3O 2
M+ ion. Hence they are never found in free state in nature. 4KO 2 4CO 2 2H 2 O KHCO3 3O 2
5. (c) A cation is always much smaller than the
17. (d) Smaller the size of cation higher is its hydration energy
corresponding atom, whereas an anion is always larger and lesser is its ionic mobility hence the correct order
than the corresponding atom. is Li+ < Na+ < K+ < Rb+
Hence, correct order of the size is 18. (a) In aqueous solution because of high charge density
Na– > Na > Na+ of Li+ it is heavily hydrated, therefore due to its
6. (a) Na Na+ + e– ; IE of Na = + ve extensive hydration which increases its size to highest
Na+ + e– Na ; E.A. of Na+ = – ve the mobility of Li+ ion will be lowest.
19. (b) Both Ca and CaH2 produce H2 gas with water.
Both are equal but opposite in nature
7. (a) Within a group, ionic radius increases with increase in Ca 2H 2 O Ca(OH) 2 H2
atomic number. The melting points decrease down the CaH 2 2H 2 O Ca(OH) 2 2H 2
group due to weakening of metallic bond. The
20. (a)
electronegativity and the 1st ionization energy also
21. (d) The basic character and stability of hydrides decrease
decreases down the group. down the group.
8. (d) Li, Na, K are lighter than water but Rb is heavier than 22. (a) For a given metal, order of stability of halides is
water. MF > MCl > MBr > MI
9. (a) 23. (a) A reducing agent is a substance which can loose
10. (c) Alkali metals have large size. When they are heated in electron and hence a reducing agent should have low
the flame of Bunsen burner, the electrons present in ionisation energy. Now since ionisation energy
the valence shell move from lower energy level to decreases from Li to Cs, the reducing property should
higher energy level by absorption of heat from the increase from Li to Cs. The only exception to this is
flame. When they come back to the ground state, they lithium. This is because the net process of converting
emit the extra energy in the form of visible light to an atom to an ion takes place in 3 steps.
provide colour to the flame. (i) M(s) M(g) H = Sublimation energy
(ii) M(g) M (g) + e–
+ H = Ionisation energy
11. (d)
(iii) M+(g)+H2O M+ (aq) H = Hydration energy
12. (b) Gp 1 metals form cations M+ by loss of electron from
The large amount of energy liberated in hydration of
outermost shell. Electronic configuration of Gp 1 metals
Li (because of its small size) makes the overall H
is ns1. When the outer electron is removed to give a negative. This accounts for the higher oxidation
positive ion, the size decreases because the outermost potential of lithium i.e., its high reducing power.
shell is completely removed. After removal of an 24. (b) The alkali metals dissolve in liquid ammonia giving
electron, the positive charge of the nucleus is greater deep blue solution.
on the remaining electrons so that each of the remaining 25. (c) x = paramagnetic y = bronze
electrons are attracted more strongly towards the z = diamagnetic
nucleus. This further reduces the size. 26. (b) Na reacts violently and may catch fire on exposure to
13. (a) Lithium does not form peroxide. moisture (air + water). So it is always stored in
14. (b) Sodium metal on burning in air gives sodium peroxide. kerosene. Na reacts with alcohol to produce H2.
EBD_7207
166 THE s-BLOCK ELEMENTS
27. (d) Lithium with lead is used to make white metal. 47. (a) NaHCO3 (baking soda) is one of the major constituents
28. (b) Liquid sodium is used as a coolant in fast breeder of baking powder.
nuclear reactors. 48. (a) Baking powder has starch, NaHCO3 and Ca(H2PO2)2.
29. (a) Since the ionization energies of alkali metals decrease 49. (b) Potassium ions are the most abundant cations within
down the group, the ionic character and consequently cell fluids.
basic property of their hydroxides increases in the same 50. (a) Metallic character decreases, as we go to the right
order, i.e. from LiOH to CsOH. side in a period and increases when we move
30. (c) With the same anion, smaller the size of the cation, downwards in a group.
higher is the lattice energy. The correct order of size of 51. (b) Ca (20) = 1s2 2s2 2p6 3s2 3p6 4s2 = [Ar], 4s2.
cations is – 52. (a)
Na+ < K+ < Rb+ < Cs+ 53. (c) Within a period, the size decreases from left to right,
Hence, the lattice energy of NaF will be maximum. i.e., Na > Mg > Li > Be. Atomic size increases down the
i.e., NaF. group.
31. (c) As Cs+ ion has larger size than Li+ and I– has larger 54. (d)
size than F–, therefore maximum distance between
55. (d) The IInd ionisation potential of Na is higher than Mg
centres of cations and anions is in CsI.
because it requires more energy to remove an electron
32. (a) Larger cation (K+) develops less polarisation in anion from a saturated shell or stable (fully filled) orbital.
and thus KI has more ionic nature and more soluble in
I
water. 11 Na 1s 2 , 2s 2 2p 6 , 3s1
33. (a) 4LiNO3 2Li 2 O 4NO 2 O 2 II
1s 2 , 2s 2 2 p 6 , 3s 0
34. (d) Anhydrous form of Na2CO3 does not decompose on
heating even to redness. It is a amorphous powder 1s 2 , 2s 2 2p5 , 3s 0
called soda ash.
I
35. (c) 4LiNO3 2Li2O + 4NO2 + O2 12 Mg 1s 2 , 2s 2 2p 6 , 3s 2
2 NaNO3 2NaNO2 + O2 II
36. (b) Li is much softer than the other group I metals. Actually 1s 2 , 2s 2 2p 6 , 3s1 1s 2 , 2s 2 2p 6 , 3s 0
Li is harder then other alkali metals. Here Na-I < Mg-I and Na-II > Mg-II.
37. (b) Lithium although has most negative E value reacts 56. (c) As we go from grp I element to grp II element in a
less vigorously with water than other alkali metals period, an extra electron is added in same shell which
due to its small size and very high hydration energy. results in increase in nuclear charge due to which
38. (d) Elemental sodium is easily oxidised ( has low I.P.) and force of attraction by the nucleus increases and hence
acts as reductant. ionic radii decreases and consequently I.E. increases.
39. (b) Washing soda is Na2CO3.10 H2O. 57. (d) 58. (d) 59. (d) 60. (c)
40. (c) 61. (a) Calcium gives brick red colour and barium gives
apple green colour in flame test.
41. (c) NH 4 HCO3 NaCl NaHCO3 NH 4 Cl
Brine Sod. bicarbonat e ppt .
62. (a) Within a group, ionic radius increases with increase in
atomic number. The melting points decrease down the
42. (a) CO2 and NH3 formed are reused (See Solvay process)
group due to weakning of metallic bond. The
43. (a) NH3 is recovered when the solution containing
electronegativity and the 1st ionization energy also
NH4Cl is treated with Ca(OH)2
decreases down the group.
2NH4Cl + Ca(OH)2 2NH3 + CaCl2 + H2O
63. (d) Barium is most electropostive element among those
44. (b)
given. Hence it is most reactive.
45. (d) Sodium sulphate, calcium chloride and magnesium
64. (a) Mg(OH)2 is not formed because of poor solubility of
chloride are present as impurities in crude sodium
MgO in H2O.
chloride.
65. (d) K and Mg, both form oxides
46. (b) Na metal is manufactured by electrolysis of fused NaCl
by two methods. K O2 KO 2 ;2Mg O 2 2MgO
(i) Castner's process
Mg form nitride also 3Mg N 2 Mg 3 N 2
(ii) Down's process
In both the above processes electrolysis of fused K does not form nitride.
sodium chloride produces Na at cathode. 66. (b) Thermal decomposition of (NH4)2BeF4 is the best
method for preparation of BeF2.
2NaOH 2Na OH
67. (b) BaF2 > MgF2 > CaF2
at cathode 2Na 2e 2Na 68. (a) Alkaline earth metals have a fairly strong tendency to
at anode 4OH 2H 2 O O 2 4e lose their outermost electrons due to which they act
as reducing agent.
THE s-BLOCK ELEMENTS 167
69. (a) 84. (d) 85. (d) 86. (c) 87. (c)
70. (d) For a compoud to be soluble, the hydration energy
88. (b) CaCO 3 2HCl 2CaCl 2 H 2 O CO 2
must exceed lattice energy. For Gp.II hydroxides
(Mg(OH)2, Sr(OH)2, Ca(OH)2, Ba(OH)2, lattice energy 89. (a) During setting of cement, silicates and aluminates of
decrease more rapidly than the hydration energy & so calcium are hydrated. Hydration is an exothermic
their solubility increases down the group. Ba(OH)2> process. Therefore setting of cement is exothermic
Sr(OH)2 > Ca(OH)2 > Mg(OH)2 process.
71. (a) Be(OH)2 is amphoteric while Ca(OH)2, Sr(OH)2 and 90. (d) For a good quality cement, the ratio of silica (SiO2)
Ba(OH)2 are all basic. to alumina (Al2O3) should be between 2.5 and 4 and
72. (a) Because of small atomic size and high I.E. Be forms the ratio of lime (CaO) to the total of the oxides of
covalent chloride. silicon (SiO2), aluminium (Al2O3) and iron (Fe2O3)
73. (a) should be as close as possible to 2.
74. (a) As we move down the group, the lattice energies of 91. (a) The calcium concentration in plasma is regulated at
carbonates remain approximately the same. However about 100 mgL–1. It is maintained by two hormones :
the hydration energies of the metal cation decreases calcitonin and parathyroid hormone.
from Be++ to Ba++, hence the solubilities of carbonates 92. (b)
of the alkaline earth metal decrease down the group
mainly due to decreasing hydration energies of the STATEMENT TYPE QUESTIONS
cations from Be++ to Ba++.
75. (a) As the basicity of metal hydroxides increases down 93. (d) Amongst alkali metal Li ions are highly hydrated.
the group from Be to Ba, the thermal stability of their 94. (d) Lithium halides are some what covalent in nature.
carbonates also increases in the same order. Further Li is the most powerful reducing agent and Na is the
group 1 compounds are more thermally stable than least powerful reducing agent.
group 2 because their hydroxide are much basic than 95. (c)
group 2 hydroxides therefore, the order of thermal 96. (a) For statement (iii), stability of the carbonates and
stability is hydrogen carbonates of alkali metals increases with
BeCO3 < MgCO3< CaCO3< K2CO3. increase in electropositive character down the group.
76. (a) In alkaline earth metals ionic size increases down the Hydrogen carbonate of lithium does not exist as a
group. The lattice energy remains constant because solid.
sulphate ion is so large, so that small change in cationic 97. (d) All the given statements are correct.
sizes do not make any difference. On moving down 98. (c) Radium is used in treatment of cancer.
the group the degree of hydration of metal ions 99. (d) All alkaline earth metal oxides except BeO are basic
decreases very much leading to decrease in solubility in nature. BeO is amphoteric in nature.
BeSO 4 MgSO 4 CaSO 4 SrSO 4 BaSO 4 Hydrated halides of Ca, Sr and Ba on heating
undergo dehydration while corresponding hydrated
77. (b) Be 2+ is very small, hence its hydration enthalpy is halides of Be and Mg on heating suffer hydrolysis.
greater than its lattice enthalpy 100. (a)
78. (b) The Be(OH)2 and Al(OH)3 are amphoteric in nature.
79. (a) Gypsum is CaSO4.2H2O MATCHING TYPE QUESTIONS
80. (c) Chemically plaster of Paris is CaSO4.1/2H2O.
101. (a)
120 C 1 102. (c) Cs + O2 CsO2 (Superoxide)
81. (b) CaSO 4 2H 2 O CaSO4 H 2O
2 4Li + O2 2Li2O (Oxide)
Plaster of Paris is hemihydrate. 2Na + O2 Na2O2 (Peroxide)
1 103. (a) Lithium metal is used to make useful alloys, for
82. (d) Plaster of Paris (CaSO4 . H O) on making paste with example with lead to make ‘white metal’ bearings for
2 2
motor engines, with aluminium to make aircraft parts,
little water sets to a hard mass due to formation of
and with magnesium to make armour plates.
gypsum (CaSO4.2H2O).
104. (a) 105. (c)
1 3 106. (b) Quick lime is used for the manufacture of dyestuffs.
CaSO4 . H2O + H2O CaSO4.2H2O + Heat
2 2 Plaster of Paris is used for setting of fractured bones.
Plaster of Paris Gypsum Slaked lime is used for the manufacture of bleaching
83. (a) (CaSO4)2.H2O – Plaster of paris is used for plastering powder.
the broken bones. Limestone is a constituent of chewing gum.
EBD_7207
168 THE s-BLOCK ELEMENTS

ASSERTION-REASON TYPE QUESTIONS 120. (d) LiI is more soluble as the degree of covalent character
is high due to larger size of anion i.e., iodide ion by
107. (a) Li+ has maximum degree of hydration among other greater polarization of lithium cation.
alkali metals. 121. (a)
108. (a) Lithium carbonate is unstable to heat; lithium being
122. (c) NaCl (brine), NH 3 and CO 2 are raw materials. CaCO3
very small in size polarises a large CO32 ion leading
is source of CO 2 .
to the formation of more stable Li2O and CO2.
109. (a) Because of high value of ionisation enthalpy and small 123. (d) Because of larger size and smaller nuclear charge, alkali
size it forms compound which are highly covalent in metals have low ionization potential relative to alkaline
nature, hence, it get hydrolysed easily. earth metals.
110. (d) Both assertion and reason are false. 124. (a) Lattice energy decreases more rapidly than hydration
Radium is the rarest of all s-block elements comprising energy for alkaline earth metal hydroxides. On moving
only 10–10 percent of igneous rocks. Francium (s-block down a group solubility of their hydroxides
member) is radioactive; its long lived isotope 223Fr increases.
ahs a half life of only 21 minutes. 125. (b) 2Na 2H 2 O 2NaOH H 2
2K 2H 2O 2KOH H 2
CRITICAL THINKING TYPE QUESTIONS
All alkali metals decompose water with the evolution
111. (c) The atom becomes larger on descending the group, so of hydrogen.
the bonds becomes weaker (metallic bond), the
cohesive force/energy decreases and accordingly Ca 2H 2 O C a(OH) 2 H2
melting point also decreases. Sr 2H 2O Sr(OH) 2 H2
112. (a) The ionisation potential value of lithium is maximum
among alkali metals i.e., its tendency to ionise to give Be + 2H2O No reaction
Li+ ions should be the minimum i.e. Li should be the Ca, Sr, Ba and Ra decompose cold water readily with
poorest reducing agent. But, lithium is the strongest evolution of hydrogen. Mg decomposes boiling water
reducing agent in aq. solution. This is due to the largest but Be is not attacked by water even at high
value of hydration energy of Li+ ions. temperatures as its oxidation potential is lower than
113. (c) Li due to highest hydration energy among the alkali the other members.
metals is the strongest reducing agent. 126. (a) Mg burns in CO2 to give MgO and C.
114. (b) During the dissolution of alkali metal hydrides energy 127. (d) Group1 carbonates are more soluble than group 2
is released in large amount, i.e., it is exothermic in nature. which are sparingly soluble, and also in case of group
115. (c) Calcium carbonate on thermal decomposition gives 2, down the group the solubility of carbonates
CaO (Basic oxide) and CO2 (Acidic oxide) decreases.
CaCO 3 CaO + CO 2 128. (d) Down the group solubility of sulphate decreases. Thus
Basic oxide Acidic oxide
Ba2+ ions will precipitate out most easily.
116. (c) 2NaO2 + 2H2O 2NaOH + H2O2 + O 129. (a) BeF2 is highly soluble in water due to the high
hydration enthalpy of the small Be2+ ion.
117. (b) O 24 ion is not possible and K 2O4 is unknown . 130. (b) Electrons in Mg due to its small size are tightly bound
118. (a) All the given elements react with oxygen to form so they cannot be excited by the flame.
oxides but only Li also react with nitrogen to form 131. (d) Be(OH)2 is amphoteric, but the hydroxides of other
Li3N. alkaline earth metals are basic. The basic strength
119. (d) (a) and (b) forms corresponding hydroxides (NaOH increases gradually.
and LiOH) in aqueous solution 132. (d) When cold calcium hydroxide reacts with chlorine, then
M 2 O H 2O 2M 2OH (M Na or Li) bleaching powder is obtained.
Therefore reaction of HI with (a), (b) and (c) is simply 3Ca(OH) 2 2Cl 2
a neutralization reaction, while aqueous solution of slaked lim e
(d) form H2O2 which act as oxidizing agent, hence Ca(OCl)2 .Ca(OH) 2 .CaCl 2 .2H 2 O
convert Iodide to Iodine( I2). Bleaching powder
Na 2O 2 2H 2 O 2Na 2OH H 2O 2
11
THE p-BLOCK ELEMENTS
(GROUP 13 AND 14)
FACT / DEFINITION TYPE QUESTIONS 9. Which of the following does not form M3+ ion?
(a) Boron (b) Aluminium
1. The non-metal oxides are ___x_____ whereas metal oxides (c) Indium (d) Gallium
y
are _______ in nature. 10. The group 13 element that is liquid during summer and used
(a) x = acidic or neutral, y = basic for measuring high temperature is
(b) x = acidic, y = neutral (a) Boron (b) Aluminium
(c) x = basic, y = acidic (c) Gallium (d) Indium
(d) x = neutral, y = basic 11. Thallium shows different oxidation states because
2. Which of the following is most abundant in the earth crust ? (a) it is transition element
(a) Boron (b) Aluminium (b) of inert pair effect
(c) of its amphoteric character
(c) Gallium (d) Thallium
(d) of its higher reactivity
3. Ionisation enthalpy ( iH1kJ mol–1) for the elements of Group
13 follows the order. 12. The exhibition of highest co-ordination number depends
on the availability of vacant orbitals in the central atom.
(a) B > Al > Ga > In > Tl (b) B < Al < Ga < In < Tl
Which of the following elements is not likely to act as central
(c) B < Al > Ga < In > Tl (d) B > Al < Ga > In < Tl
4. The relationship between first, second and third ionisation atom in MF63 ?
enthalpies of each group-13 element is (a) B (b) Al
(a) (c) Ga (d) In
iH1 > iH2 > iH3 (b) iH1 < iH2 < iH3
13. Which out of the following compounds does not exist?
(c) iH1 = iH2 > iH3 (d) iH3 > iH1 > iH2
(a) BF3 (b) TlCl3
5. Which of the following properties of aluminium makes it (c) TlCl5 (d) Both (b) and (c)
useful for food packaging ? 14. Aluminium chloride is a/an
(a) Good electrical conductivity (a) Bronsted - Lowery acid (b) Arhenius acid
(b) Good thermal conductivity (c) Lewis acid (d) Lewis base
(c) Low density 15. The strongest Lewis acid is
(d) Non toxicity (a) BF3 (b) BCl3
6. Which of the following is/are true regarding gallium? (c) BBr3 (d) BI3
(i) It has unusually low melting point (303 K). 16. AlCl3 on hydrolysis gives
(ii) It exist in liquid state during summer. (a) Al2O3. H2O (b) Al(OH)3
(iii) It has a high boiling point (2676 K). (c) Al2O3 (d) AlCl3.6H2O
17. Which metal is protected by a layer of its own oxide?
The correct option is
(a) Al (b) Ag
(a) (i) and (ii) (b) (i) and (iii)
(c) Au (d) Fe
(c) (i), (ii) and (iii) (d) (ii) and (iii) 18. Aluminium vessels should not be washed with materials
7. The element which shows least metallic character is containing washing soda because
(a) Indium (b) Boron (a) washing soda is expensive
(c) Aluminium (d) Gallium (b) washing soda is easily decomposed
8. Which one of the following has the lowest m.p.? (c) washing soda reacts with aluminium to form soluble
aluminate
(a) B (b) Al
(d) washing soda reacts with aluminium to form insoluble
(c) Ga (d) Tl aluminium oxide
EBD_7207
170 THE p-BLOCK ELEMENTS (GROUP 13 AND 14)
19. When Al is added to KOH solution 33. Which is false in case of boric acid H3BO3?
(a) no action takes place (b) oxygen is evolved (a) It acts as a tribasic acid.
(c) water is produced (d) hydrogen is evolved (b) It has a planar structure.
20. Which of the following does not react with aqueous (c) It acts as a monobasic acid.
NaOH ? (d) It is soluble in hot water.
(a) B (b) Al 34. BCl3 does not exist as dimer but BH3 exists as dimer (B2H6)
(c) Ga (d) Tl because
21. Amphoteric oxide among the following is (a) chlorine is more electronegative than hydrogen
(a) B2O3 (b) Ga2O3 (b) there is p -p back bonding in BCl3 but BH3 does not
(c) In2O3 (d) Tl2O3 contain such multiple bonding
22. Boron forms covalent compound due to (c) large sized chlorine atoms do not fit in between the
(a) higher ionization energy small boron atoms where as small sized hydrogen
(b) lower ionization energy atoms get fitted in between boron atoms
(c) small size (d) None of the above
(d) Both (a) and (c) 35. In reaction
23. NH3 and BF3 form an adduct readily because they form BF3 + 3LiBH4 3LiF + X ; X is
(a) a coordinate bond (b) a hydrogen bond (a) B4 H10 (b) B2H6
(c) an ionic bond (d) a covalent bond (c) BH3 (d) B3H8
24. The factor responsible for weak acidic nature of B–F bonds 36. Inorganic benzene is
in BF3 is (a) B3 H 3 N 3 (b) BH 3 NH 3
(a) large electronegativity of fluorine
(c) B3 H 6 N 3 (d) H 3B3 N 6
(b) three centred two electron bonds in BF3
(c) p - d back bonding 37. The structure of diborane ( B 2 H 6 ) contains
(d) p - p back bonding (a) four 2c-2e bonds and four 3c-2e bonds
25. In borax bead test which compound is formed? (b) two 2c-2e bonds and two 3c-3e bonds
(a) Ortho-borate (b) Meta-borate (c) two 2c-2e bonds and four 3c-2e bonds
(c) Double oxide (d) Tetra-borate (d) four 2c-2e bonds and two 3c-2e bonds
26. The formula of mineral borax is 38. In diborane
(a) Na 2 B 4 O 7 (b) Na 2 B4 O 7 .4H 2 O (a) 4–bridged hydrogens and two terminal hydrogens are
present
(c) Na 2 B4 O 7 .5H 2O (d) Na 2 B 4O 7 .10H 2O (b) 2– bridged hydrogens and four terminal hydrogens
27. Which of the following hydroxide is acidic ? are present
(a) Al(OH)3 (b) Ca(OH)3 (c) 3–bridged and three terminal hydrogens are present
(c) Tl(OH)3 (d) B(OH)3 (d) None of these
28. Orthoboric acid 39. Diborane upon hydrolysis gives
(a) donate proton to form H2BO3– (a) boric anhydride (b) metaboric acid
(b) accept proton of form H4BO3+ (c) orthoboric acid (d) boron oxide
(c) donate OH– to form H2BO2+ 40. Borazole is known as
(d) accept OH– to form [B(OH)4]– (a) organic benzene (b) organic xylene
29. H3BO3 on heating up to 373 K yields: (c) inorganic benzene (d) inorganic xylene
(a) boric anhydride (b) orthoboric acid 41. The compounds of boron and hydrogen are collectively
(c) metaboric acid (d) tetraboric acid called
30. Boric acid is polymeric due to (a) diboranes (b) borazoles
(a) its acidic nature (c) boracits (d) boranes
(b) the presence of hydrogen bonds 42. The bonds present in borazole or inorganic benzene are
(c) its monobasic nature (a) 9 , 6 (b) 12 , 3
(d) its geometry (c) 6 , 9 (d) 15 only
31. B(OH)3 is 43. The two type of bonds present in B2H6 are covalent and
(a) monobasic acid (b) dibasic acid (a) ionic (b) co-ordinate
(c) tribasic acid (d) triacidic base (c) hydrogen bridge bond (d) None of these
32. Orthoboric acid when heated to red hot gives 44. Reaction of diborane with ammonia gives initially
(a) metaboric acid (b) pyroboric acid (a) B2H6 . NH3 (b) Borazole
(c) boron and water (d) boric anhydride (c) B2H6 . 3NH3 (d) [BH2(NH3)2]+[BH4]–
THE p-BLOCK ELEMENTS (GROUP 13 AND 14) 171
45. Which of the following compounds is not matched correctly 54. Red lead is
with its structure? (a) Pb3O4 (b) Pb2O3
(c) Pb2 O (d) PbO
H
55. The oxide of lead used in lead accumulators is
B (a) PbO (b) Pb 2 O3
H–N N–H
(a) – Borazine (c) Pb3O 4 (d) PbO 2
H–B B–H
N 56. Which of the following is/are not correctly matched ?
(i) GeO2 – Acidic (ii) PbO2– Amphoteric
H (iii) CO – Neutral (iv) SiO2 – Amphoteric
(a) (i) and (iv) (b) (iv) only
H H H
(b) (c) (ii) only (d) (iii) only
B B – Diborane
H H 57. Least thermally stable is
H
(a) CCl4 (b) SiCl4
Cl Cl Cl (c) GeCl4 (d) GeBr4
(c) Al Cl – Aluminium chloride
Cl Al Al 58. Unlike the other elements of its group carbon and silicon
does not form MX2 type molecules because
(a) energetically this is not possible
(d) Cl B – Cl – Boron trichloride (b) carbon undergoes catenation
Cl
(c) it is non-metallic
(d) carbon does not contain d-orbital
46. The electronic configuration of four different elements is
given below. Identify the group 14 element among these 59. Which of the following halides is the most stable?
(a) CF4 (b) CI4
(a) [He] 2s1 (b) [Ne] 3s 2 (c) CBr4 (d) CCl4
(c) [Ne] 3s 2 3 p 2 (d) [Ne] 3s 2 3 p 5 60. The stability of dihalides of Si, Ge, Sn and Pb increases
steadily in the sequence
47. Which of the following is most electronegative?
(a) Pb (b) Si (a) PbX 2 SnX 2 GeX 2 SiX 2
(c) C (d) Sn (b) GeX2 << SiX2 << SnX2 << PbX2
48. Which of the following isotope of carbon is radioactive? (c) SiX2 << GeX2 << PbX2 << SnX2
(a) 12 C (b) 13 C (d) SiX2 << GeX2 << SnX2 << PbX2.
(c) 14 C (d) All of these 61. Which of the following is not correct?
49. Carbon and silicon belong to group 14. The maximum (a) Ge(OH)2 is amphoteric
coordination number of carbon in commonly occurring (b) GeCl2 is more stable than GeCl4
compounds is 4, whereas that of silicon is 6. This is due to
(a) large size of silicon (c) GeO2 is weakly acidic
(b) more electropositive nature of silicon (d) GeCl4 in HCl forms [ GeCl2]2– ion
(c) availability of d-orbitals in silicon 62. The main reason that SiCl4 is easily hydrolysed as compared
(d) Both (a) and (b) to CCl4 is that
50. The inert pair effect is most prominent in (a) Si-Si bond is weaker
(a) C (b) Pb (b) SiCl4 can form hydrogen bonds
(c) Ge (d) Si (c) SiCl4 is covalent
51. The most stable +2 oxidation state is exhibited by (d) Si can extend its coordination number beyond four
(a) Fe (b) Sn 63. Which halide is least stable and has doubtful existence
(c) Pb (d) Si (a) CI4 (b) GeI4
52. Which of the following lead oxides is present in ‘Sindhur’? (c) SnI4 (d) PbI4
(a) PbO (b) PbO 2 64. PbF4, PbCl4 exist but PbBr4 and PbI4 do not exist because
(c) Pb2 O3 (d) Pb3O4 of
53. Mark the oxide which is amphoteric in character (a) large size of Br – and I–
(a) CO2 (b) SiO 2 (b) strong oxidising character of Pb4+
(c) strong reducing character of Pb4+
(c) SnO2 (d) CaO (d) low electronegativity of Br – and I–.
EBD_7207
172 THE p-BLOCK ELEMENTS (GROUP 13 AND 14)
65. Catenation i.e., linking of similar atoms depends on size and 77. The hybridisation state of carbon in fullerene is
electronic configuration of atoms. The tendency of (a) sp (b) sp 2
catenation in Group 14 elements follows the order : (c) sp 3 (d) sp 3 d
(a) C > Si > Ge > Sn (b) C >> Si > Ge Sn 78. The number of carbon atoms in Buckminsterfullerene is
(c) Si > C > Sn > Ge (d) Ge > Sn > Si > C (a) 50 (b) 350
66. The catenation tendency of C,Si and Ge is in the order (c) 60 (d) 70
1
Ge < Si < C.The bond energies (in kJ mol ) of C-C,Si-Si 79. Graphite is a soft solid lubricant extremely difficult to melt.
and Ge-Ge bonds, respectively are The reason for this anomalous behaviour is that graphite
(a) 167, 180, 348 (b) 180, 167, 348 (a) is an allotropic form of diamond
(c) 348, 167, 180 (d) 348, 180, 167 (b) has molecules of variable molecular masses like
67. Lead pipes are readily corroded by polymers
(c) has carbon atoms arranged in large plates of rings of
(a) H 2SO 4 (b) HCl strongly bound carbon atoms with weak inter plate
(c) CH 3COOH (d) pure water bonds
(d) is a non-crystalline substance
68. Lead pipes are not suitable for drinking water because
80. In graphite, electrons are
(a) lead forms basic lead carbonate
(a) localised on every third C-atom
(b) lead reacts with water containing air to form
(b) present in anti-bonding orbital
Pb(OH)2
(c) localised on each C-atom
(c) a layer of lead dioxide is deposited over pipes
(d) spread out between the structure
(d) lead reacts with air to form litharge
81. The elements commonly used for making transistors are
69. The reducing power of divalent species decreases in the
(a) C and Si (b) Ga and In
order
(c) P and As (d) Si and Ge
(a) Ge > Sn > Pb (b) Sn > Ge > Pb
(c) Pb > Sn > Ge (d) None of these 82. The element which is exclusively applied as semi-conductor
70. The element that does not show catenation among the (a) Au (b) Ge
following p-block elements is (c) Pt (d) Si
(a) carbon (b) silicon 83. Glass is a
(c) germanium (d) lead (a) liquid
71. How many six membered and five membered rings are (b) solid
present in fullerene? (c) supercooled liquid
(a) Six membered = 20, five membered = 10 (d) transparent organic polymer
(b) Six membered = 20, five membered = 12 84. Glass reacts with HF to produce
(c) Six membered = 25, five membered = 10 (a) SiF4 (b) H2SiF6
(d) Six membered = 12, five membered = 25 (c) H2SiO3 (d) Na3AlF6
72. Which of the following is the pure form of carbon ? 85. Producer gas is the mixture of
(a) Diamond (a) CO + N2 (b) CO + H2
(b) Fullerene (c) CO + water vapours (d) N2 + CH4
(c) Graphite 86. Coal gas is a mixture of
(d) All three forms are equally pure (a) H 2 O and CO (b) H 2 ,CO, N 2 and CH 4
73. Which one of the following is not an allotrope of carbon ?
(c) H 2 and CO (d) CH 4 and CO
(a) Carborundum (b) Diamond
(c) Soot (d) Graphite 87. Crystalline form of silica is called
74. Which of the following types of forces bind together the (a) crystalline silicon (b) quartz
carbon atoms in diamond ? (c) rock (d) talc
(a) Ionic (b) Covalent 88. Dry ice is
(c) Dipolar (d) van der Waal’s (a) solid SO2 (b) solid NH3
75. Carborundum is (c) solid O2 (d) solid CO2
(a) SiC (b) CaC2 89. In silica (SiO2), each silicon atom is bonded to
(c) Mg2C3 (d) None of these (a) two oxygen atoms
76. Buckminster fullerene is (b) four oxygen atoms
(a) pure graphite (b) C-60 (c) one silicon and two oxygen atoms
(c) diamond (d) C-90 (d) one silicon and three oxygen atoms
THE p-BLOCK ELEMENTS (GROUP 13 AND 14) 173

90. R3SiCl on hydrolysis forms 101. Which of the following is not a man-made silicate ?
(a) Glass (b) Cement
(a) R3SiOH (b) R 3Si O SiR 3
(c) Zeolites (d) All are man-made silicates
(c) R 2Si O (d) None of these 102. Which type of zeolite is used to convert alcohols directly
91. Which of the following statements is false? into gasoline ?
(a) Water gas is a mixture of hydrogen and carbon (a) ZSM – 3 (b) ZSM – 5
monoxide (c) ZSM – 2 (d) All of these
(b) Producer gas is a mixture of CO and nitrogen
STATEMENT TYPE QUESTIONS
(c) Water gas is a mixture of water vapour and hydrogen
(d) Natural gas consists of methane, ethane and gaseous 103. Which of the following statement(s) is/are not correct ?
hydrocarbons. (i) Valence shell electronic configuration of p-block
92. Which gas is essential constituent of almost all fuel gases ? elements is ns2 np1-6
(a) CO2 (b) N2 (ii) Non metals and metalloids exist only in the p-block
of the periodic table.
(c) CO (d) H2O
(iii) In boron, carbon and nitrogen families the group
93. CO2 is used for extinguishing fire because oxidation state is the most stable state for the lighter
(a) it has a relatively high critical temperature elements in the group.
(b) in solid state, it is called dry ice (iv) For heavier elements in each group oxidation state
(c) it is neither combustible nor a supporter of combustion two unit less than the group oxidation state becomes
(d) it is a colourless gas more stable due to inert pair effect
94. The correct statement with respect to CO is (a) (ii) only
(a) it combines with H2O to give carbonic acid (b) (ii), (iii) and (iv)
(b) it reacts with haemoglobin in RBC (c) (iii) and (iv)
(c) it is powerful oxidising agent (d) All given statements are correct
(d) it is used to prepare aerated drinks 104. Which of the following sequence of T and F is correct for
given statements. Here T represents ‘True’ and F
95. Producer gas, a fuel and also a source of nitrogen is obtained
represents ‘False’ statement.
by
(i) Aluminium forms [AlF6]3– ion while boron forms
(a) passing a mixture of steam and air over incandescent
only [BF4]– ion due to presence of d-orbitals in
coke.
aluminium.
(b) spraying oil into hot retorts.
(ii) The first member of a group differs from the heavier
(c) restricted supply of air through a bed of incandescent members in its ability to form p -p multiple bonds
coke. to itself and to other second row elements. While
(d) passing steam over incandescent coke. heavier member forms d -p bonds.
96. Which of the following shows bond in silicone : (iii) d-orbitals contribute more to the overall stability of
(a) Si – Si – Si – Si (b) – Si – O – Si – O – Si molecules than p -p bonding of second row
(c) Si – C – Si – C – Si (d) Si – C – Si – O – Si elements.
97. Which of the following is formed on dehydration of (a) TTT (b) FTF
formic acid with concentrated H2SO4 ? (c) TTF (d) FTT
(a) CO (b) CO2 105. Which of the following statement(s) is/are incorrect ?
(c) CH4 (d) H2 (i) Trichlorides on hydrolysis in water form tetrahedral
98. _____ helps to maintain pH of blood between 7.26 to 7.42 [M(OH)4]– species.
(a) CO2 (b) H2CO3 (ii) Hybridisation state of metal in tetrahedral species is
sp3 .
(c) CO32 (d) H2CO3/HCO3–
(iii) Aluminium chloride in acidified aqueous solution
99. Which of the following is not the crystalline form of forms [Al(OH)4]– ion.
silica? (a) (i) and (ii) (b) (ii) only
(a) Quartz (c) (iii) only (d) (i) and (iii)
(b) Cristobalite 106. Which of the following statement(s) regarding BCl3 and
(c) Tridymite AlCl3 is/are correct ?
(d) All are crystalline form of silica. (i) BCl3 possess lower melting point than AlCl3.
100. Which of the following is used in surgical and cosmetic (ii) BCl3 is more covalent in character than AlCl3.
plants? (a) Statement (i) is correct explanation for statement (ii).
(a) Silicones (b) Silicates (b) Statement (i) and (ii) both are incorrect
(c) Silica (d) None of these (c) Statement (i) and (ii) both are correct
(d) Statement (ii) is correct explanation for statement (i)
EBD_7207
174 THE p-BLOCK ELEMENTS (GROUP 13 AND 14)
107. Which of the following statement(s) is/are incorrect ? MATCHING TYPE QUESTIONS
(i) Higher boranes are not flammable.
(ii) Boranes are hydrolysed by water to give orthoboric 112. Match the columns
acid. Column-I Column-II
(iii) Boranes undergoes cleavage reactions with Lewis (A) Borax-bead (p) Alum
bases to give borane adducts. (B) Inorganic benzene (q) Diborane
(a) (i) only (b) (ii) and (iii) (C) Antiseptic (r) Metaborate
(c) (iii) only (d) (i) and (ii) (D) Bridged hydrogens (s) Borazole
(a) A – (p), B – (r), C – (q), D – (s)
108. Select the correct statements for diborane :
(b) A – (r), B – (s), C – (p), D – (q)
(i) Boron is approximately sp3 hybridized
(c) A – (s), B – (r), C – (p), D – (q)
(ii) B – H – B angle is 180°
(d) A – (q), B – (r), C – (s), D – (p)
(iii) There are two terminal B – H bonds for each boron
113. Identify (i) to (v) in reactions (1) and (2) on the basis of
atom
your identification choose the correct code for matching
(iv) There are only 12 bonding electrons Column-I with Column-II.
(a) (i), (ii) and (iv) (b) (i), (ii) and (iii)
(c) (ii), (iii) and (iv) (d) (i), (iii) and (iv) 1. Na2B4O7.10H2O (i) (ii) + (iii)
109. Which of the following sequence of T and F is correct for 2. Na2B4O7.7H2O (iv) + (v)
given statements. Here T stands for the true and F stands Column-I Column-II
for false statement. (A) (i) (p) H3BO3
(i) The tendency to show +2 oxidation state increase in (B) (ii) (q) B2O3
the sequence Ge < Sn < Pb. (C) (iii) (r) NaBO2
(ii) Tin in +2 state is a reducing agent. (D) (iv) (s) NaOH
(iii) Lead compounds in +2 state are strong oxidising (E) (v) (t) Na2B4O7
agents. (a) A – (t), B – (s), C – (p), D – (q), E – (r)
(iv) In tetravalent state molecules of group 13 elements (b) A – (r), B – (q), C – (s), D – (p), E – (t)
act as electrons donor species. (c) A – (t), B – (r), C – (q), D – (p), E – (s)
(a) TTTT (b) TTFF (d) A – (t), B – (r), C – (s), D – (q), E – (p)
(c) TTFT (d) TFFT 114. Match Column-I (Compound of boron) with Column-II
110. Which of the following statement(s) is / are incorrect for (Use) and choose the correct option.
CO2? Column-I Column-II
(i) In laboratory CO2 is prepared by the action of dilute (A) Metal borides (p) Flux for soldering metals
HCl on calcium carbonate (B) Boron fibres (q) Bullet-proof vest
(ii) Carbon dioxide is a poisonous gas (C) Borax (r) As a mild antiseptic
(iii) Increase in carbon dioxide content in atmosphere (D) Boric acid (s) As control rods in
lead to increase in green house effect. nuclear industry
(iv) CO2 as dry ice is used as a refrigerant for ice cream (a) A – (q), B – (s), C – (r), D – (p)
and frozen food. (b) A – (q), B – (s), C – (p), D – (r)
(a) (i) and (ii) (b) Only (ii) (c) A – (s), B – (q), C – (r), D – (p)
(c) (i), (ii) and (iii) (d) (ii) and (iii) (d) A – (s), B – (q), C – (p), D – (r)
111. Which of the following sequence of T and F is correct for 115. Match the columns
given statements. Here T stands for true and F stands for Column-I Column-II
false statement. (A) Carbon (p) Metal
(i) Quartz is extensively used as a piezoelectric material. (B) Silicon (q) Non-metal
(ii) Kieselghur is an amorphous form of silica which is (C) Germanium (r) Metalloid
used in filteration plants. (D) Tin
(iii) Silica does not react with halogens, dihydrogen and (E) Lead
most of the acids and metals even at elevated (a) A – (q), B – (q), C – (r), D – (p), E – (p)
temperature. (b) A – (q), B – (r), C – (r), D – (p), E – (p)
(a) TTT (b) TFF (c) A – (q), B – (r), C – (r), D – (p), E – (q)
(b) TFT (d) FFT (d) A – (q), B – (q), C – (q), D – (r), E – (p)
THE p-BLOCK ELEMENTS (GROUP 13 AND 14) 175
116. Match columns 122. Assertion : PbI4 of lead does not exist.
Column-I Column-II Reason : Pb–I bond initially formed during the reaction
(A) Graphite fibres (p) Abrasive for sharpening does not release enough energy to unpair 6s2 electrons.
hard tools 123. Assertion : Graphite is thermodynamically most stable
(B) Carbon black (q) Formation of light allotrope of carbon.
weight composites. Reason : of graphite is taken as zero.
fH
(C) Charcoal (r) Used in water filters to
remove organic CRITICAL THINKING TYPE QUESTIONS
contaminators
(D) Diamond (s) As filler in automobile 124. The liquefied metal which expands on solidification is :
tyres (a) Ga (b) Al
(a) A – (s), B – (q), C – (r), D – (p) (c) Zn (d) In
(b) A – (q), B – (s), C – (r), D – (p) 125. What is x in the following reaction?
(c) A – (q), B – (r), C – (s), D – (p) Al(s) + NaOH (aq) + H2O (l) x + H2(g)
(d) A – (p), B – (r), C – (s), D – (q) (a) Na2[Al(OH)4] – (b) Na+[Al(OH)4]–
117. Match the columns (c) Na2[Al(OH)6] – (d) Na+ [Al(OH)6]–
Column-I Column-II 126. Which among the following oxides react with alkali?
1
(A) Borazole (p) CaSO4. H2O B2O3, Al2O3 and Tl2O
2 (a) B2O3 and Al2O3 (b) Al2O3 and Tl2O
(B) Plaster of Paris (q) C60
(c) Only B2O3 (d) B2O3 and Tl2O
(C) Boric acid (r) SiO2
127. White fumes appeared around the bottle of anhydrous
(D) Quartz (s) B3N3H6
aluminium chloride is due to _____
(E) Buckminsterfullerene (t) H3BO3
(a) Cl2 gas
(a) A – (r); B – (p); C – (q); D – (t); E – (s)
(b) A – (p); B – (t); C – (r); D – (s); E – (q) (b) moist HCl
(c) A – (t); B – (q); C – (p); D – (r); E – (s) (c) condensation of aluminium chloride vapours
(d) A – (s); B – (p); C – (t); D – (r); E – (q) (d) None of these
128. What is the oxidation state and hybridisation of boron in
ASSERTION-REASON TYPE QUESTIONS compound formed when BCl3 undergoes reaction with the
water?
Directions : Each of these questions contain two statements, (a) 3, sp2d (b) 3, sp3
Assertion and Reason. Each of these questions also has four 3
(c) 4, sp (d) 3, sp2d
alternative choices, only one of which is the correct answer. You
have to select one of the codes (a), (b), (c) and (d) given below. 129. Which is not correct?
(a) Assertion is correct, reason is correct; reason is a correct (a) Al acts as a reducing agent
explanation for assertion. (b) Al does not react with steam even at higher
(b) Assertion is correct, reason is correct; reason is not a temperature
correct explanation for assertion (c) Al forms a number of alloys with other metals
(c) Assertion is correct, reason is incorrect (d) Al is ionic in all its compounds
(d) Assertion is incorrect, reason is correct. 130. Which one of the following is the correct statement?
118. Assertion : Atomic radius of gallium is higher than that (a) Boric acid is a protonic acid
of aluminium (b) Beryllium exhibits coordination number of six
Reason : The presence of additional d-electron offer poor (c) Chlorides of both beryllium and aluminium have
screening effect for the outer electrons from increased bridged structures in solid phase
nuclear charge. (d) B2H6.2NH3 is known as ‘inorganic benzene’
119. Assertion : Boron is metalloid. 131. BF3 is used as a catalyst in several industrial processes
Reason : Boron shows metallic nature. due to its
120. Assertion : The use of aluminium and its compounds for
(a) strong reducing nature
domestic purposes is now reduced considerably.
(b) weak reducing action
Reason : The highly toxic nature of aluminium is the
responsible factor. (c) strong Lewis acid nature
121. Assertion : Pb4+ compounds are stronger oxidizing agents (d) weak Lewis acid character
than Sn4+ compounds. 132. What is the colour obtained when borax is heated in a
Reason : The higher oxidation states for the group 14 Bunsen burner flame with CoO?
elements are more stable for the heavier members of the (a) Blue (b) Black
group due to ‘inert pair effect’. (c) Green (d) Violet
EBD_7207
176 THE p-BLOCK ELEMENTS (GROUP 13 AND 14)
133. Which of the following statements about H3BO3 is not (c) ionic radii of Pb2+ and Pb4+ are larger than those of
correct? Ge2+ and Ge4+
(a) It is a strong tribasic acid (d) of more pronounced inert pair effect in lead than
(b) It is prepared by acidifying an aqueous solution of in Ge
borax 143. Which of the following statements is not correct ?
(c) It has a layer structure in which planar BO3 units are (a) Fullerene is formed by condensation of vapourised
joined by hydrogen bonds Cn small molecules consists of mainly C60.
(d) It does not act as proton donor but acts as a Lewis (b) In fullerene a six membered ring can only fuse with
acid by accepting a lone pair of electrons five membered ring and a five membered ring can
134. The hybridisation of boron atom in orthoboric acid is only fuse with six membered rings.
(a) sp (b) sp 2 (c) All carbon atoms are sp2 hybridised in fullerene
(d) All the above are correct.
(c) sp 3 (d) sp3d
144. The element that does not form a monoxide is
135. Which is not the use of orthoboric acid?
(a) lead (b) tin
(a) As an antiseptic and eye wash. (c) germanium (d) silicon
(b) In glass industry. 145. A group 14 element is oxidised to form corresponding oxide
(c) In glazes for pottery. which is gaseous in nature, when dissolved in water pH of
(d) In borax - bead test. the water decreases further addition of group 2 hydroxides
136. Which of the following reaction shows production of leads to precipitation. This oxide can be
diborane on industrial scale ? (a) GeO2 (b) CO
(a) 4BF3 + 3LiAlH4 2B2H6 + 3LiF + 3AlF3 (c) CO2 (d) SnO2
146. Which among the following can act as reducing agent
(b) 2NaBH4 + I2 B2H6 + 2NaI + H2
(A)SnCl2, (B)CO and (C)PbCl2 ?
450K (a) (A) and (B) (b) (B) and (C)
(c) 2BF3 + 6NaH B2H6 + 6NaF
(d) Both (b) and (c) (c) (C) and (A) (d) Only (B)
137. Identify the statement that is not correct as far as structure 147. Lead is not affected by dil. HCl in cold because
of diborane is concerned (a) Pb is less electronegative than H
(b) PbO film is formed which resists chemical attack by
(a) There are two bridging hydrogen atoms and four
acid
terminal hydrogen atoms in diborane
(b) Each boron atom forms four bonds in diborane (c) PbCl2 protective coating gets formed on Pb surface
(c) The hydrogen atoms are not in the same plane in (d) PbO2 film is always present on Pb surface, which resist
diborane chemical attack
(d) All, B – H bonds in diborane are similar 148. The percentage of s-character of the hybrid orbitals of
138. Which of the following structure is similar to graphite? carbon in graphite and diamond are respectively
(a) B (b) B4C (a) 33, 25 (b) 50, 50
(c) B2H6 (d) BN (c) 67, 25 (d) 33, 67
139. A compound X, of boron reacts with NH3 on heating to 149. What is the hybridisations of carbon atoms present in
give another compound Y which is called inorganic benzene. diamond, graphite and fullerene respectively ?
The compound X can be prepared by treating BF3 with (a) sp3,sp2 and sp2 (b) sp2,sp3 and sp2
2 2
(c) sp ,sp and sp 3 (d) sp3,sp3 and sp2
lithium aluminium hydride. The compounds X and Y are
represented by the formulas. 150. Which one of the following allotropic forms of carbon is
(a) B2H6, B3N3H6 (b) B2O3, B3N3H6 isomorphous with crystalline silicon?
(a) Graphite (b) Coal
(c) BF3, B3N3H6 (d) B3N3H6, B2H6
(c) Coke (d) Diamond
140. The product/s formed when diborane is hydrolysed is/are
151. Which one of the following statements about the zeolites is
(a) B2O3 and H3BO3 (b) B2O3 only
false ?
(c) H3BO3 and H2 (d) H3BO3 only
(a) They are used as cation exchangers
141. Which of the following species exists (A) [SiF 6 ]2– , (b) They have open structure which enables them to take
(B) [GeCl6]2– and (C) [CCl6]2– ? up small molecules
(a) (A) and (B) (b) (B) and (C) (c) Zeolites are aluminosilicates having three dimensional
(c) Only (C) (d) (A) and (C) network
142. Ge(II)compounds are powerful reducing agents (d) None of the above
whereas Pb(IV)compounds are strong oxidants .It is because 152. Which of the following attacks glass
(a) Pb is more electropositive than Ge (a) HCl (b) HF
(b) ionization potential of lead is less than that of Ge (c) HI (d) HBr
THE p-BLOCK ELEMENTS (GROUP 13 AND 14) 177

FACT / DEFINITION TYPE QUESTIONS 27. (d) B(OH)3 is acid because it can take OH– ions.
H3BO3 or B(OH)3 + OH– B(OH) 4
1. (a) The non – metal oxides are acidic or neutral whereas
metal oxides are basic in nature. 28. (d) H3BO3 acts as a Lewis acid and accepts OH– ions to
2. (b) Aluminium does not occur in the free state in nature form [B(OH)4]–
29. (c) H3BO3 on heating at 373K yields metaboric acid
but is most abundant metal in the earth’s crust.
(HBO2)
3. (d)
373K
4. (b) The order of ionisation enthalpies, as expected, is H3BO3 HBO2 H 2O
metaboricacid
iH1 < iH2 < iH3. (orthorombic form)
5. (c) Due to the low density of aluminium it is useful for
30. (b) In Boric acid each B atom is sp2 hybridized and
food packaging.
6. (c) 7. (b) contains BO 33 units which are held together by
8. (c) The m.p decreases from B to Ga , hence gallium (Ga) hydrogen bonds.
has least m.p. (303 K) among group of 13 element. 31. (a)
9. (a) Due to its small size and high ionization energy boron 100 C 160 C
32. (b) H 3 BO 3 HBO 2 H
does not form B3+ ion.
10. (c) Gallium is remarkable for its unusually low M.P.
H 2 B4 O 7 H 2O 2B2 O3 H 2O
(29.7°C).
33. (a) H3BO3 is monobasic acid.
11. (b) Thallium shows different (+1 and +3) oxidation states
34. (c) 35. (b)
because of inert pair effect.
36. (c) H
12. (a) |
13. (c) Because Tl +5 does not exist B
|
||

14. (c) H N N H
|
|

| ||
15. (d) The order of strength of Lewis acid character for boron
H B B H
|

halides is, BF3 < BCl3 < BBr3 < BI3 (due to back
N
||
|

bonding) |
16. (b) 17. (a) 18. (c) H
Inorganic benzene, B 3N 3H6
19. (d) 2KOH 2Al 2H 2 O 2KAlO 2 3H 2
It is isoelectronic with benzene.
20. (a) 37. (d) In diborane (B2H6) structure there are four 2c-2e bonds
21. (b) Down the group basic character of oxides increases. and two 3c–2e bonds (see structure of diborane).
B2O3 - Acidc Structure of B 2H6 :
Al2O3 - Amphoteric
Ga2O3 - Amphoteric Hb
In2O3 - Basic Ht •• Ht
Tl2O3 - Basic
B B
22. (d) 23. (a)
24. (d) It is p p back bonding involving B and F. The Ht •• Ht
smaller atoms show more back bonding. Hb
38. (b) 39. (c) 40. (c) 41. (d) 42. (b)
25. (b) Na2B4O7. 10 H2O Na2B4O7
10H 2O 43. (c) B2H6 contains hydrogen bridge bonds. These are one
electron bonds also known as banana bonds.
Na 2 B4O7 2 NaBO2 B2 O3
anhydrous sod.metaborate Boric anhydride excess NH3
44. (d) B2H6 + NH3 low temperature B2H6.2NH3
CuO B2 O3 Cu(BO 2 )2
cupric meta borate(Blue beed) Diborane with ammonia gives B2H6 .2NH3 that is
26. (d) formulated as [BH2(NH3)]+[BH4]– which when heated
to 473K decomposes to give borazole.
EBD_7207
178 THE p-BLOCK ELEMENTS (GROUP 13 AND 14)

Cl Cl Cl 64. (b) F and Cl are more oxidising in nature and can achieve
45. (c) Al Cl AlCl3 (dimer)
Cl Al Al Pb in (IV) O.S. but Br2 and I 2 can not achieve Pb in
46. (c) Valence shell electronic configuration of group 14 (IV) O.S. secondly Pb 4 is strong in oxidising nature
2 2.
elements is ns p and in its presence, Br and I can not exist.
65. (b)
47. (c) Electronegativity decreases down the group. 66. (d) The more the bond energy, the more is the
48. (c) 14C is a radioactive isotope with half life of 5770 catenation.
years and used for radiocarbon dating. 67. (c) Lead pipes are readily corroded by water containing
49. (c) Due to non-availability of vacant d-orbitals, it cannot organic acids.
exceed its coordination number more than four. Thus
68. (b) 2Pb 2H 2O O 2 2Pb(OH) 2
carbon never forms complexes e.g., [CCl6]2– deos not
exist but [SiCl6]2– exists. 69. (a) The stability of +2 O.S. follows the order
50. (b) The inert pair effect is most prominent in the heavier Pb 2 Sn 2 Ge 2
members of the group. Inert pair effect increases as we
Hence reducing power Ge Sn Pb
move the group down the group.
70. (d) The order of tendency of catenation for elements of C
51. (c) Inert pair effect increases down the group. Hence for
family is
Pb2 ,O.S. is most stable. C >> Si > Ge Sn > Pb
71. (b) Fullerene contains twenty six membered rings and
52. (d) Pb3O4 is also known as Sindhur.. twelve five membered rings.
53. (c) CO2, SiO2 are acidic, CaO is basic and SnO 2 is 72. (b) Fullerenes are the only pure form of carbon because
amphoteric. they have smooth structure without having dangling
bonds.
54. (a) Red lead is Pb3O4. It is a mixed oxide of Pb (II) and
Pb (IV). It acts as a powerful oxidising agent. 73. (a) Carborundum is chemically SiC. It is not an allotrope
of carbon.
55. (d) PbO2 is a strong oxidising agent and is produced in
74. (b) In diamond each carbon atom is sp3 hybridized and
situ in lead storage batteries. The anode is oxidized to thus forms covalent bonds with four other carbon atoms
PbO2 and cathode is reduced to spongy Pb. lying at the corners of a regular tetrahedron.
56. (b) SiO2 is acidic oxide. 75. (a)
57. (d) The thermal stability of tetrahalides decreases in order 76. (b) Buckminster fullerene is C60. The molecule has shape
CX4 > SiX4 > GeX4 > SnX4 and in terms of same metal of soccer ball.
with different halides is in order of 77. (b) In fullerene each carbon atom is bonded to three other
MF4 > MCl4 > MBr4 > MI4. carbon atoms and is sp2 hybridised.
58. (a) The stability of dihalides (MX2) increases down the 78. (c) Buckminster fullerene has the formula C60 and is made
group. Except C and Si, the other members form from interlocking hexagonal and pentagonal rings of
dihalides. C-atoms.
59. (a) Since bond energy of C-F >C-Cl > C-Br > C-I 79. (c)
80. (d) In graphite, each carbon is sp2 -hybridized and the
Hence CF4 is most stable.
single occupied unhybridized p-orbitals of C-atoms
60. (d) Reluctance of valence shell electrons to participate in
bonding is called inert pair effect. The stability of lower overlap side wise to give -electron cloud which is
oxidation state (+2 for group 14 element) increases on delocalized and thus the electrons are spread out
going down the group. So the correct order is between the structure.
SiX2 < GeX2 < SnX2 < PbX2 81. (d) Si and Ge are semiconductors and are used in making
transistors.
61. (b) Ge4 is more stable than Ge2+ . Hence GeCl 4 is more
82. (b) Both Ge and Si are extensively used as semiconductors.
stable than GeCl 2 Semiconductors are solids where there is only a small
62. (d) Carbon halides are not hydrolysed due to absence of difference in energy, called band gap, between the filled
d-orbitals. On the other hand SiCl4 is easily hydrolysed
valency band of electrons and a conduction band since
due to the availability of d-orbitals in Si.
the band gap of Ge is less than Si, it is a better element
SiX4 + 2H2O SiO2 + 4HX
to be used as semiconductor.
63. (d) In nature Pb 4 is strong oxidant and I is strong 83. (c) Glass is a super cooled liquid.
reductant. Hence PbI 4 cannot exist. 84. (b) 6 HF SiO 2 H 2SiF6 2 H2O
THE p-BLOCK ELEMENTS (GROUP 13 AND 14) 179
85. (a) Producer gas is a fuel gas and is mixture of CO and N2. 106. (d) BCl3 is a covalent compound hence lower melting
86. (b) Coal gas is a mixture of H2 + CO + N2 + CH4 point.
87. (b) Quartz is crystalline form of silica. 107. (a) Higher boranes are also spontaneously flammable in
88. (d) air.
89. (b) In silica (SiO2); each Si atom is surrounded by four H H H
oxygen atom. 108. (d) B 197° B
H H H
— —

— —

— —

— —
3
B is sp hybridized
— Si — O —Si — O — Si — O —Si —
Only 12 bonding electrons available
O O O O BHB angle is 97° not 180°.


— Si — O —Si — O — Si — O —Si — 109. (b) Lead compounds in +4 state are strong oxidising
agents. In tetravalent state the number of electrons


O O O O around the central atom in a molecule is eight. Being
— —

— —

— —

— —

electron precise molecules, they are normally not


— Si — O —Si — O — Si — O —Si — expected to act as electron acceptor or electron
donor species.
Structure of SiO2 110. (b) Carbon dioxide is not a poisonous gas.
Only Si – O bonds exist and no Si = O. 111. (a) All the given statements are true.
90. (b) R 3SiCl HOH R 3SiOH HCl MATCHING TYPE QUESTIONS
R 3SiOH HOSiR 3 R 3Si O SiR 3 H 2O
112. (b)
91. (a) Water gas is CO H 2
113. (c) Na2B4O7.10H2O Na2B4O7
92. (c) CO is essential constituent of almost all fuel gases. (i)
93. (c) CO 2 is incombustible and non supporter of 2NaBO2 + B2O3
combustion. (ii) (iii)
94. (b) CO react with haemoglobin, forms carboxy
haemoglobin and stopes the supply of O2 Na2B4O7 + 7H2O 4H3BO3 + 2NaOH
(iv) (v)
95. (c) Producer gas is mixture of CO + N 2 . It is prepared by
114. (d)
incomplete combustion of coal in limited supply of air. 115. (a) Carbon and silicon are non-metals. Germanium is a
96. (b) Silicone has Si – O – Si – O – Si linkage. metalloid. Tin and lead are metals.
373K 116. (b) 117. (d)
97. (a) HCOOH conc. H2SO4 H2O + CO
98. (d) H2CO3/HCO3– buffer system help to maintain pH of ASSERTION-REASON TYPE QUESTIONS
blood between 7.26 to 7.42. 118. (c) Atomic radius of gallium is less than that of
99. (d) Quartz, Cristobalite and Tridymite are crystalline aluminium.
form of silica. 119. (c) Boron is metalloid. Thus assertion is correct. Metalloids
100. (a) Being biocompatible silicones are used in surgical possess, metallic as well as non-metallic nature. Hence,
and cosmetic plants. reason is false.
101. (c) Zeolite is not a man-made silicate. 120. (a) The use of aluminium and its compounds for domestic
102. (b) ZSM – 5 type of zeolite is used to convert alcohols purposes is now reduced considerably because of their
directly into gasoline. toxic nature.
121. (c) Assertion is true because lower oxidation state
STATEMENT TYPE QUESTIONS
becomes more & more stable for heavier elements in p-
103. (d) All the given statements are correct. block due to inert pair effect. Hence Reason is false.
104. (c) d-orbitals are of higher energy than the p-orbitals, 122. (a) PbI4 does not exist because Pb–I bond initially
they contribute less to the overall stability of formed during the reaction does not release enough
molecules than p -p bonding of the second row energy to unpair 6s2 electrons and excite one of
elements. them to higher orbital to have four unpaired electrons
105. (c) Aluminium chloride in acidified aqueous solution around lead atom.
forms octahedral [Al(H2O)6]3+ ion. 123. (a)
EBD_7207
180 THE p-BLOCK ELEMENTS (GROUP 13 AND 14)
138. (d) Boron nitride (BN) is known as inorganic graphite. The
CRITICAL THINKING TYPE QUESTIONS
most stable form is hexagonal one. It has layered
124. (a) Gallium (Ga) is soft , silvery metal. Its melting point is structure similar to graphite.
30ºC. This metal expands by 3.1% when it solidifies
and hence, it should not be stored in glass or metal N N+

containers. + B
B– –B
125. (b) 2Al(s) + 2NaOH(aq) + 6H2O(l)
2Na+[Al(OH)4]–(aq) + 3H2 (g) N+ +N N
– +
126. (a) B2O3 is acidic and Al2O3 is amphoteric. –B B B– –B
127. (b) Anhydrous aluminium chloride gets partially
hydrolysed with atmospheric moisture to liberate +N N N+ – + N
HCl gas. Moist HCl appears white in colour. +
B– –B B
128. (b) BCl3 forms [B(OH)4]– in which B is sp3 hybridized
and have +3 oxidation state.
N+ +N N
129. (d) Al in its compounds forms covalent bonds. – +
B B–
130. (c) The correct formula of inorganic benzene is B3N3H6
so (d) is incorrect statement 139. (a)
140. (c) When diborane is hydrolysed one can get both
OH orthoboric acid and H2.
|
Boric acid (H3BO3 or B OH ) is a lewis acid so (a) B2H6 + 6H2O 2H3BO3 + 6H2
| 141. (a) Carbon does not contain d-orbital hence it cannot
OH expand its octet.
is incorrect statement. 142. (d) Ge(II) tends to acquire Ge (IV) state by loss of electrons.
The coordination number exhibited by beryllium is 4 Hence it is reducing in nature. Pb (IV) tends to acquire
and not 6 so statement (b) is incorrect. Pb (II) O.S. by gain of electrons. Hence it is oxidising
Both BeCl2 and AlCl3 exhibit bridged structures in solid in nature. This is due to inert pair effect.
state so (c) is correct statement. 143. (b) In fullerene a six membered ring can fuse with five
as well as with six membered ring while a five
Cl m Cl Cl Cl Cl
2p membered ring can only fuse with a six membered
20
Be 98° Be 263 pm Be Be ring.
82° 144. (d) Silicon does not form mono oxide.
Cl Cl Cl Cl Cl 145. (c) CO2 forms carbonic acid H2CO3, when dissolved in
water, CO is neutral, whereas other two GeO2 and
Cl Cl Cl
SnO2 are solids.
Al Al 146. (a) Lead in +2 oxidation is stable while Sn and C are both
stable in +4 oxidation.
Cl Cl Cl 147. (c) Pb with dil HCl forms protective coating of PbCl 2
131. (d) 148. (a) Graphite sp3, % s character = 33%
132. (a) When borax is heated in a Bunsen burner flame with Diamond sp3, % s character = 25%
CoO on a loop of platinum wire a blue coloured 149. (a)
Co(BO2)2 bead is formed. 150. (d) Diamond and crystalline silicon are isomorphous.
133. (a) H3BO3 is a weak monobasic acid. 151. (d)
134. (b) The hybridizations of B in H3BO3 is sp2 152. (b) Glass being a mixture of sodium and calcium silicates
135. (d) Borax on heating gives B2O3 and NaBO2 which is reacts with hydrofluoric acid forming sodium and
glassy mass and used for borax-bead test. calcium fluorosilicates respectively.
136. (c) Diborane is produced on industrial scale by the Na 2SiO 3 6HF Na 2SiF6 3H 2 O
reaction of BF3 with sodium hydride.
137. (d) CaSiO3 6HF CaSiF6 3H 2 O
The etching of glass is based on these reactions.
12
ORGANIC CHEMISTRY–SOME BASIC
PRINCIPLES & TECHNIQUES
FACT / DEFINITION TYPE QUESTIONS The state of hybrization of carbons 1, 3 and 5 are in the
following sequence
1. Which of the following scientist proposed that a ‘vital (a) sp2, sp, sp3 (b) sp, sp3, sp2
force’ was responsible for the formation of organic (c) sp, sp2, sp3 (d) sp3, sp2, sp
compounds ?
9. The percentage of s- character of the hybrid orbitals in
(a) Berzilius (b) Wohler ethane, ethene and ethyne are respectively.
(c) Berthelot (d) Kolbe (a) 50, 75, 100 (b) 10, 20, 40
2. First organic compound to be synthesised was (c) 25, 33, 50 (d) 25, 50, 75
(a) methane (b) cane sugar 10. Select the molecule which has only one -bond
(c) acetic acid (d) urea (a) CH CH (b) CH2 = CHCHO
3. Which of the following organic compound was (c) CH3CH = CH2 (d) CH3CH=CHCOOH
synthesised by F. Wohler from an inorganic compound? 11. 2- Pentene contains
(a) Methane (b) Urea (a) 15 - and one - bond (b) 14 -and one - bond
(c) Acetic acid (d) Chloroform (c) 15 - and two - bonds (d) 14 - and two - bonds
4. The discovery that shook the belief in the vital force theory 12. Which of the following does not represent the 2 – bromo
was pentane ?
(a) Stereoisomerism H H H Br H
(b) Synthesis of indigo (i) H C C C C C H
(c) Wholer’s synthesis of urea from ammonium cyanate H H H H
(d) Fermentation of sugars
5. In laboratory, first organic compound was synthesised by Br
(a) Kekule (b) Hennel (ii)
(c) Wohler (d) Liebig
6. Who is known as the “Father of Chemistry”? Br
(a) Faraday (b) Priestley
(iii)
(c) Rutherford (d) Lavoisier
7. The hybridisation of carbon atom in C — C single bond of
H2C = CH — CH = CH2 is Br
(a) sp3 — sp (b) sp2 — sp (iv)
(c) sp2 — sp2 (b) sp3 — sp3
8. In the hydrocarbon (v) CH3CH2 CH2CHBrCH3
CH3 – CH = CH – CH2 – C CH (a) (ii), (iii) and (v) (b) Only (ii)
6 5 4 3 2 1 (c) (ii) and (iii) (d) (iii) and (v)
EBD_7207
182 ORGANIC CHEMISTRY – SOME BASIC PRINCIPLES & TECHNIQUES
13. Which of the following correctly represents the expanded 1 2 3 4 5 6
form of following organic compound ? (ii) CH3 CH CH 2 CH 2 CH 2 CH CH 2 CH 2 CH 3
| |
CH3 7CH 2 C H3
8
H H H
H 1 2 3 4 5 6 7 8 9
C C
(iii) CH3 CH CH 2 CH 2 CH 2 CH CH 2 CH2 CH3
(a) H C C C | |
H C
CH3 CH 2 CH3

H
H H H 8 7 6 5 4 3
H H (iv) CH3 CH CH 2 CH 2 CH 2 CH CH 2 CH 2 CH3
| |
C C CH3 CH 2 CH3
(b) H C C C 2 1
C (a) (i) and (ii) (b) (iv) only
H
H
(c) (i), (ii) and (iv) (d) (ii) only
H 18. The correct decreasing order of priority of functional
H H
H
H H
H H
groups is
H C C
(a) – SO3H , – OH, – COCl, C=C
C
(c) C C
(b) – COOH, – SO3H, – COOR, – OH
C (c) – C C, – NH2, – OH, C=O
H H H H (d) – CN, – CONH2, C = O, – OH
H H H H 19. Which of the following is incorrectly matched –
H C C
H
(a) vinegar carboxylic acid
C C C (b) C2H6 alkane
(d) (c) ethanol alcohol
C
(d) methanol ketone
H H H H
20. The functional group present in organic, acid is –
14. Structural formula of benzene is (a) – OH (b) – CHO
H H (c) –COOH (d) > C = O
H C H 21. Which of these contains the carbonyl group?
C H C C H
H C C H H H (a) ketones (b) aldehydes
(a) H (b) H C C H (c) esters (d) all of these
C C H C
H C 22. Butanone is a four-carbon compound with the functional
H H group –
H
H (a) carboxylic acid (b) aldehyde.
C C H (c) ketone (d) alcohol.
H C C H 23. The functional group present in CH3COOC2H5 is –
H C C H
(c) H C C H (d) H C C H (a) ketonic (b) aldehydic
C H (c) ester (d) carboxylic
C
H H 24. Which of the following compounds contains 1°, 2°, 3° as
15. The successive members in a homologues series differ well as 4° carbon atoms ?
from each other by ________ (a) Neopentane (b) 2-methyl pentane
(a) – CH2CH2– unit (b) – CH2 unit (c) 2,3-dimethyl butane (d) 2,2,3-trimethyl pentane
(c) – OCH3 unit (d) – CH3 unit 25. The number of secondary hydrogens in 2, 2-dimethylbutane
is
16. Which of the following have incorrect molecular formula?
(a) 8 (b) 6
A. Icosane – C10H22
(c) 4 (d) 2
B. Triacontane – C30H62
26. The compound which has one isopropyl group is
C. Nonane – C9H20 (a) 2, 2, 3, 3 - Tetramethylpentane
D. Heptane – C7H14 (b) 2, 2 - Dimethylpentane
(a) (A) and (D) (b) Only (D) (c) 2, 2, 3- Trimethylpentane
(c) (B) and (D) (d) Only (B) (d) 2- Methypentane
17. Which of the following are incorrect methods of selecting 27. Which of the following statements is false for isopentane ?
parent chain ? (a) It has three CH3 groups
1 2 3 4 5 6 (b) It has one CH2 group
(i) CH3 CH CH 2 CH 2 CH 2 CH CH 2 CH 2 CH3 (c) It has one CH group
| |
CH3 7CH 2 CH 3 (d) It has a carbon which is not bonded to hydrogen
8
ORGANIC CHEMISTRY – SOME BASIC PRINCIPLES & TECHNIQUES 183
28. The number of primary, secondary and tertiary carbons in 3, (a) 3-bromo-1-chlorocyclohexene
4-dimethylheptane are respectively (b) 1-bromo-3-chlorocyclohexene
(a) 4, 3 and 2 (b) 2, 3 and 4 (c) 2-bromo-6-chlorocyclohex-1-ene
(c) 4, 2 and 3 (d) 3, 4 and 2 (d) 6-bromo-2-chlorocyclohexene
29. The number of primary, secondary, tertiary and quaternary 37. Name of the following compound is
carbons in neopentane are respectively
CH 3CH 2 CH 3
(a) 4, 3, 2 and 1 (b) 5, 0, 0 and 1
C
(c) 4, 0, 0 and 1 (d) 4, 0, 1 and 1 CH 3 CH 2 OH
30. What is the IUPAC name of t-butyl alcohol.
(a) Butanol–2 (b) 2–Methyl-propan–2-ol (a) 2-ethylbutan-2-ol
(c) Butanol–1 (d) Propanol-2 (b) 1-ethyl-1-methylpropan-1-ol
31. The IUPAC name of CH3COCH (CH3)2 is - (c) 3-methyl pentan-3-ol
(a) isopropyl methyl ketone (d) diethylethanol
(b) 2-methyl-3-butanone 38. The IUPAC name for
(c) 4-methylisopropyl ketone C1
(d) 3-methyl-2-butanone |
CH3 C CH 2 CH CH CH3 is
32. CH3CH2– CH CH – CH 2CH3 has the IUPAC name – |
| | H
CH3 CHO (a) 5–chlorohex–2–ene
(b) 2–chlorohex–5–ene
(a) 2–sec butylbutanal
(c) 1–chloro–1–methylpent–3–ene
(b) 2, 3–diethylbutanal
(d) 5–chloro–5–methylpent–2–ene
(c) 2–ethyl–3–methylpentanal
39. IUPAC name of following compound is :
(d) 3–methyl–2–ethylpentanal
33. Which of the following statements is false for isopentane– H
(a) It has three CH3 groups CH
CH33 - C - CH
CH22- -CH
CH
3
(b) It has one CH2 group
(c) It has one CH group
(d) It has a carbon which is not bonded to hydrogen (a) 2 - cyclohexylbutane (b) 2 - phenylbutane
34. The IUPAC name of the compound (c) 3 - cyclohexylbutane (d) 3 - phenylbutane
CH 3OCH 2 CH 2 CH 2 OCH 2 CH 3 is 40. What is the IUPAC name of the following compound ?
(a) 3-ethoxy-1-methoxypropane NH 2
(b) 1-ethoxy-3-methoxypropane
(c) 2, 5-dioxyhexane
(d) ethoxypropane oxymethane
(a) 2-methyl-4-hexanamine
35. Which of the following compounds has wrong IUPAC
(b) 5-methyl-3-hexanamine
name?
(c) 2-methyl-4-amino hexane
(a) CH3–CH2–CH2 –COO–CH2CH3 ethyl butanoate
(d) 5-methyl-3-amino hexane
(b) CH 3 CH CH 2 CHO 3-methyl-butanal 41. Which one of the following is ethyl-4-(dimethyl amino)
| butanoate ?
CH3

(c) CH3 CH CH CH3 2-methyl-3-butanol


| | (a) (MeNH) 2 OEt
OH CH 3

O
|| (b)
OEt
(d) CH3 CH C CH2 CH3 2-methyl-3-pentanone
| NH 2
CH3
36. The IUPAC name of the compound shown below is
(c) H2N
Cl OEt

Br (d) Me 2 N
OEt
EBD_7207
184 ORGANIC CHEMISTRY – SOME BASIC PRINCIPLES & TECHNIQUES
42. Identify the correct IUPAC name of the compound given 49. The IUPAC name of neopentane is
below (a) 2, 2-dimethylpropane (b) 2-methylpropane
(c) 2, 2-dimethylbutane (d) 2-methylbutane
50. The IUPAC name for
H Cl
NO2

(a) 4 – benzyl – 5 – methyl hexanal


(b) 2 – methyl – 3 – phenyl hexanal CH3
(c) 5 – isopropyl – 5 – phenyl butanal (a) 1-Chloro-2-nitro-4-methyl benzene
(d) 5 – methyl – 4 – phenyl hexanal (b) 1-Chloro-4-methyl-2-nitrobenzene
43. IUPAC name of (CH3)3 CCl is (c) 2-Chloro-1-nitro-5-methyl benzene
(a) 1-butyl chloride (d) m-Nitro-p-chlorotoluene
(b) 3-chloro butane 51. What is the IUPAC name of the following compound ?
(c) 2-chloro-2-methylpropane
(d) 2-butyl chloride COOH
44. IUPAC name of the following compound Br
O
CH3
C N
CH3 CH 2 CH 3
(a) N, N-dimethylcyclopropane carboxamide
(a) 6 – bromo – 4 – ethylbenzene carboxylic acid
(b) N-methylcyclopropanamide'
(b) 2 – bromo – 4 – ethylbenzene carboxylic acid
(c) cyclopropionamide
(c) Ortho – bromo – paraethyl benzoic acid
(d) None of these
(d) 4 – bromo – 3 – ethyl benzoic acid
45. Which of the following is a 3-methylbutyl group?
52. Total number of structural isomers possible for C3H6 are :
(a) CH3CH2CH2CH2CH2–
(a) 2 (b) 1
(b) (CH3CH2)2CH–
(c) 4 (d) 3
(c) (CH3)3CCH2–
53. An aromatic compound of formula C7H7Cl has in all .....
(d) (CH3)2CHCH2CH2–
isomers :
46. The IUPAC name of the following compound
(a) 5 (d) 2
H3C CH2 CH CH2 CH CH2CH3 (c) 4 (d) 3
54. CH3CH2OH and CH3OCH3 are the examples of
CH2CH3 CH3 (a) chain isomerism (b) functional isomerism
is (c) position isomerism (d) metamerism
(a) 3-ethyl-5-methylheptane 55. Which organic structure among the following is not an
(b) 5-ethyl-3-methylheptane isomer of the compound
(c) 3,5-diethylhexane CH3–CO–CH2CH2CH2CH3 ?
(d) 1,1-diethyl-3-methylpentane (a) CH3CH2OCH =CHCH2CH3
47. The IUPAC name of the following compound is (b) CH3CH = CHCH2CH2CHO
(c) (CH3)2CH–CO–CH2CH3
(CH3 )2 CH CH 2CH CH CH CH CHCH3
| (d) CH3CH2COCH2CH2CH3
C2 H5 56. The least number of carbon atoms in alkane showing
(a) 1,1,7,7-tetramethyl-2,5-octadiene isomerism is
(b) 2,8-dimethyl-3,6-decadiene (a) 3 (b) 1
(c) 1,5-di-iso-propyl-1,4-hexadiene (c) 2 (d) 4
(d) 2,8-dimethyl-4,6-decadiene 57. The number of possible alkynes with molecular formula
48. The IUPAC name of the compound C5H8 is
CH3 — CH(CH3) — CO – CH3, is (a) 2 (b) 3
(a) 3-methyl 2-butanone (c) 4 (d) 5
(b) 2-methyl 3-butanone 58. The total number of isomers for C4H8 is
(c) isopropyl methyl ketone (a) 5 (b) 6
(d) methyl isopropyl ketone (c) 7 (d) 8
ORGANIC CHEMISTRY – SOME BASIC PRINCIPLES & TECHNIQUES 185
59. Which of the following compounds is isomeric with 2, 2, 4, 71. Which of the following ions is most stable ?
4- tetramethylhexane?
(a) 3-ethyl -2, 2- dimethylpentane (a) CH 3 C CH 3 (b) CH 3 CH 2 CH 2
(b) 4-isopropylheptane |
(c) 4-ethyl-3-methyl-4-n propyloctane CH 3
(d) 4, 4-diethyl-3-methylheptane
60. Which are isomers ? (c) CH 3CHCH 2 CH 3 (d) None of these
(a) ethyl alcohol and dimethyl ether 72. The order of stability of the following carbocations :
(b) acetone and acetaldehyde
(c) propionic acid and propanone CH 2
(d) methyl alcohol and dimethyl ether
61. Methoxyethane and propanol are the examples of isomerism CH 2 CH C H 2 ; CH3 CH 2 CH 2 ; is :
of the type I II
(a) structural (b) position III
(c) functional (d) tautomerism (a) III > II > I (b) II > III > I
(c) I > II > III (d) III > I > II
62. Isomers of propionic acid are
73. Select the most stable carbocation amongst the following
(a) HCOOC2H5 and CH3COOCH3
+
(b) HCOOC2H5 and C3H7COOH (a)
(c) CH3COOCH3 and C3H7OH
(b)
(d) C3H7OH and CH3COCH3 +
63. C6H5C N and C6H5N C are which type of isomers?
(a) Position (b) Functional
(c) +
(c) Tautomerism (d) Linkage
64. A functional isomer of 1-butyne is
(a) 2-butyne (b) 1-butene
(d) +
(c) 2-butene (d) 1, 3-butadiene
65. In which of the following, functional group isomerism is not 74. What is the correct order of decreasing stability of the
possible? following cations.
(a) Alcohols (b) Aldehydes
(c) Alkyl halides (d) Cyanides I. CH3 — CH— CH3
66. The compounds CH3CH == CHCH3 and
CH3CH2CH == CH2 II. CH3 — CH— OCH 3
(a) are tautomers
(b) are position isomers
(c) contain same number of sp3– sp3, sp3– sp2 and sp2– sp2 III. CH3 — CH— CH 2 — OCH3
carbon-carbon bonds (a) II > I > III (b) II > III > I
(d) exist together in dynamic equilibrium (c) III > I > II (d) I > II > III
67. Heterolytic fission of a covalent bond in organic molecules 75. The most stable carbonium ion among the following is
gives +
(a) C6 H5 CHC6 H5 (b) C6 H5CH 2
(a) free radicals (b) cations and anions
(c) only cations (d) only anions
68. Which of the following statements is not correct ? (c) CH3 CH 2 (d) C6 H5CH 2CH 2
(a) Carbocation posses sextet of electrons. 76. The organic reaction which proceed through heterolytic
(b) The order of carbocation stability is : bond cleavage are called ________
(a) ionic (b) polar
CH3 (CH3 )2 CH (CH3 )3 C (c) nonpolar (d) Both (a) and (b)
77. Among the following, the true property about
(c) Carbocations have trigonal planar shape
(d) Carbocations are formed by heterolytic cleavage CH3
+
69. Heterolytic fission of C – Br bond results in the formation of C – CH3 is
(a) free radical (b) carbanion
(c) carbocation (d) Both (b) and (c) CH3
70. Which of the following carbocations is least stable? (a) it is non-planar
(a) tert-Alkyl (b) sec-Alkyl (b) its C+ is sp2-hybridized
(c) pri-Alkyl (d) Methyl (c) an electrophile can attack on its C+
(d) it does not undergo hydrolysis
EBD_7207
186 ORGANIC CHEMISTRY – SOME BASIC PRINCIPLES & TECHNIQUES
78. The shape of methyl carbanion is similar to that of – 90. Which of the following is strongest nucleophile
(a) BF3 (b) NH3 (a) Br– (b) : OH–
(c) methyl free radical (d) methyl carbocation
79. Arrange the carbanions, (c) : CN (d) C2 H 5O :
91. Which of the following represents a set of nucleophiles?
(CH3 )3 C , C Cl3 , (CH3 ) 2 C H , C6 H5 CH 2 (a) BF3, H2O, NH2– (b) AlCl3, BF3, NH3
in order of their decreasing stability : (c) CN–, RCH2–, ROH (d) All of these
(a) (CH3 ) 2 C H > C Cl3 > C6 H5 C H2 > (CH3 )3 C 92. Which of the following species does not acts as a
nucleophile?
(b) C Cl3 > C6 H5 CH 2 > (CH 3 )2 CH > (CH3 )3 C (a) ROH (b) ROR
(c) (c) PCl3 (d) BF3
(CH3 )3 C > (CH3 )2 CH > C6 H 5 CH 2 > C Cl3
93. Which of the following is an electrophile ?
(d) C6 H5 CH 2 > CCl3 > (CH3 )3 C > (CH3 ) 2 CH (a) Lewis acid
80. The homolytic fission of a covalent bond liberates (b) Lewis base
(a) Carbonium ions (b) Carbanions (c) Negatively charged species
(c) Free radicals (d) Carbenes (d) None of the above
81. Homolytic fission of C–C bond in ethane gives an 94. Which of the following pairs represent electrophiles?
intermediate in which carbon is (a) AlCl3, H2O (b) SO3, NO2+
(a) sp3-hybridised (b) sp2-hybridised (c) BF3, H2O (d) NH3, SO3
(c) sp-hybridised (d) sp2d-hybridised 95. Which out of A, B, C and D is/are not correctly
82. Geometry of methyl free-radical is categorised.
(a) pyramidal (b) planar Nucleophile Electrophile
(c) tetrahedral (d) linear A. HS Cl
83. In which of the following homolytic bond fission takes place ? (CH3 )3N
B. BF3
(a) Alkaline hydrolysis of ethyl chloride
(b) Addition of HBr to double bond C. H 2 N C O
(c) Photochlorination of methane D. R 3C X C2 H5O
(d) Nitration of benzene (X H alogen)
84. On exciting, Cl2 molecules by UV light, we get
(a) B, C and D (b) C and D
(a) Cl (b) Cl+ (c) C only (d) B and D
(c) Cl – (d) all of these 96. Arrangement of (CH3)3 – C –, (CH3)2 – CH –, CH3 – CH2 –
85. The increasing order of stability of the following free radicals when attached to benzyl or an unsaturated group in
is increasing order of inductive effect is
• • •
(a) (C6H5)2 C H < (C6H5)3 C < (CH3)3 C < (CH3)2 C H
• (a) (CH3)3 –C – < (CH3)2 – CH – < CH3 – CH2–
• • • • (b) CH3 –CH2– < (CH3)2– CH – < (CH3)3 –C –
(b) (CH3)2 C H < (CH3)3 C < (C6H5)2 C H < (C6H5)3C (c) (CH3)2 – CH– < (CH3)3 –C – < CH3—CH2–
• • • •
(c) (CH3)2 C H < (CH3)3 C < (C6H5)2 C H < (C6H5)3 C (d) (CH3)3 – C– < CH3 –CH2 – < (CH3)2 –CH –
• • • • 97. Polarization of electrons in acrolein may be written as
(d) (C6H5)3C < (C6H5)2 C H < (CH3)3 C < (CH3)2 C H
86. Which of the following orders regarding relative stability of
free radicals is correct? (a) CH 2 CH CH O (b) CH 2 CH CH O
(a) 3° < 2° < 1° (b) 3° > 2° > 1°
(c) 1° < 2° > 3° (d) 3° > 2° < 1° (c) CH 2 CH CH O (d) CH 2 CH CH O
87. The most stable free radical among the following is 98. Point out the incorrect statement about resonance?
(a) C 6 H5 CH 2CH 2 (b) (a) Resonance structures should have equal energy
C6 H 5 CHCH 3
(b) In resonating structures, the constituent atoms must
(c) CH 3CH 2 (d) CH 3 CHCH 3 be in the same position
(c) In resonating structures, there should not be same
88. For the reaction of phenol with CHCl3 in presence of KOH,
number of electron pairs
the electrophile is
(d) Resonating structures should differ only in the location
(a) CHCl 2 (b) : CCl2 of electrons around the constituent atoms
(c) CHCl 2 (d) CCl4 99. : CH 2 C CH 3 and CH 2 C CH 3 are
|| |
89. The least stable free radical is O
. .: : .O. :
(a) CH3 CH 2 (b) CH3CH2 CH 2 (a) resonating structures (b) tautomers
(c) (CH3)2 CH (d) CH 3 (c) geometrical isomers (d) optical isomers
ORGANIC CHEMISTRY – SOME BASIC PRINCIPLES & TECHNIQUES 187
100. In which of the following, resonance will be possible? 109. Which of the following is an example of elimination reaction?
(a) CH3 CH 2 CH 2 CHO (a) Chlorination of methane
(b) Dehydration of ethanol
(b) CH 2 CH CH O (c) Nitration of benzene
(c) CH 3COCH 3 (d) Hydroxylation of ethylene
110. CH3 – Br + NH3 CH 3 – NH 2 HBr
(d) CH 2 CH CH 2 CH CH 2
The above reaction is classified as
101. Which of the following statements regarding the resonance
(a) substitution (b) addition
energy of benzene is correct?
(c) elimination (d) rearrangement
(a) Resonance energy is the energy required to break the
111. Which of the following method is not used for determining
C–H bond in benzene
purity of a compound ?
(b) Resonance energy is the energy required to break the
(a) Chromatographic techniques
C–C bond in benzene
(b) Spectroscopic techniques
(c) Resonance energy is a measure of stability of benzene
(c) Melting point
(d) Resonance energy is the energy required to convert
(d) All of the above parameters are used for determining
the purity of a compound.
112. Which of the following is not the criteria of purity of a
substance?
102. Which of the following is not correctly matched ? (a) solubility (b) melting point
Group showing + R effect Group showing – R effect (c) boiling point (d) density
(a) – NHCOR – COOH 113. In crystallisation process impurities which impart colour
to the solution are removed by which of the following ?
(b) C=O – OH
(a) Repeated crystallisation
(c) – OR – CHO (b) Activated charcoal
(d) – OCOR – NO2 (c) Bleaching agent
103. The polarity is produced in the molecule by the interaction (d) Both (a) and (b)
of two – bonds or between a – bond and lone pair 114. Aniline is purified by
of electrons present on an adjacent atom. (a) steam distillation (b) simple distillation
The above statement is true for which of the following ? (c) vacuum distillation (d) extraction with a solvent
(a) Inductive effect (b) Electromeric effect 115. Which is purified by steam distillation
(c) Resonance effect (d) Hyperconjugation (a) Aniline (b) Benzoic acid
104. Electromeric effect is a (c) Petroleum (d) Naphthalene
(a) permanent effect (b) temporary effect 116. The best method for the separation of naphthalene and
(c) resonance effect (d) inductive effect benzoic acid from their mixture is:
105. The kind of delocalization involving sigma bond orbitals is (a) distillation (b) sublimation
called (c) chromatography (d) crystallisation
(a) inductive effect (b) hyperconjugation effect 117. In steam distillation the vapour pressure of volatile organic
(c) electromeric effect (d) mesomeric effect compound is
106. Hyperconjugation involves overlap of the following orbitals (a) equal to atmospheric pressure
(a) - (b) - (b) double the atmospheric pressure
(c) p- p (d) - (c) less than atmospheric pressure
107. Choose the correct order of stability of carbocation using (d) more than atmospheric pressure
concept of hyperconjugation. 118. Fractional distillation is used when
(a) there is a large difference in the boiling point of liquids
CH3 CH3
| | (b) there is a small difference in the boiling points of liq-
CH3 C CH3 C CH 3CH 2 CH 3 uids
| | III IV (c) boiling points of liquids are same
CH3 CH3 (d) liquids form a constant boiling mixture
I II 119. Purification of petroleum is carried out by
(a) I < II < III < IV (b) IV < III < II < I (a) fractional distillation (b) steam distillation
(c) III < IV < II < I (d) None of these (c) vacuum distillation (d) simple distillation
108. Hyperconjugation is most useful for stabilizing which of 120. A liquid decomposes at its boiling point. It can be purified
the following carbocations ? by
(a) neo-Pentyl (b) tert-Butyl (a) steam distillation (b) fractional distillation
(c) iso-Propyl (d) Ethyl (c) vacuum distillation (d) sublimation
EBD_7207
188 ORGANIC CHEMISTRY – SOME BASIC PRINCIPLES & TECHNIQUES
121. Distillation under reduced pressure is employed for 134. In quantitative analysis of carbon and hydrogen, the
(a) C6H6 mass of water produced is determined by passing the
(b) petrol mixture through a weighed U – tube containing ___X___
(c) CH2OHCHOHCH2OH and carbon dioxide is absorbed in concentrated solution
(d) organic compounds used in medicine of ___Y___
122. Impure glycerine is purified by (a) X = CaCl2 , Y = NaOH
(a) steam distillation (b) simple distillation (b) X = Ca(OH)2 , Y = CuSO4
(c) vacuum distillation (d) None of these (c) X = CuSO4 , Y = Ca(OH)2
123. Glycerol can be separated from spent lye in soap industry (d) X = CaCl2 , Y = KOH
by which of the following method ? 135. Kjeldahl method is not applicable to which of the
(a) Steam distillation following ?
(b) Fractional distillation (a) Nitro compounds (b) Azo compounds
(c) Distillation under reduced pressure (c) Pyridine (d) All of these.
(d) Differential extraction 136. Nitrogen in an organic compound can be estimated by
124. The latest technique for the purification of organic (a) Kjeldahl’s method only(b) Duma’s method only
compounds is
(c) Both (a) and (b) (d) Neither (a) nor (b)
(a) chromatography (b) fractional distillation
137. Duma's method involves the determination of nitrogen
(c) crystallization (d) vacuum distillation
content in the organic compound in form of
125. Which of the following is used as an adsorbent in
(a) NH3 (b) N2
adsorption chromatography ?
(a) Silica gel (b) Alumina (c) NaCN (d) (NH4)2SO4
(c) Zeolite (d) Both (a) and (b) 138. In Kjeldahl’s method nitrogen present is estimated as
126. Which of the following acts as the stationary phase in (a) N2 (b) NH3
paper chromatography ? (c) NO2 (d) None of these
(a) Water (b) Alumina 139. In kjeldahl's method of estimation of nitrogen, K2SO4 acts
(c) Silica gel (d) None of these as
127. The most satisfactory method to separate mixture of sugars (a) oxidising agent (b) catalytic agent
is (c) hydrolysing agent (d) boiling point elevator
(a) fractional crystallisation (b) sublimation 140. 0.5g of an organic compound containing nitrogen on
(c) chromatography (d) benedict reagent Kjeldahlising required 29 mL of N/5 H2SO4 for complete
128. Chromatography is a valuable method for the separation, neutralization of ammonia. The percentage of nitrogen in
isolation, purification and identification of the constituents the compound is
of a mixture and it is based on general principle of (a) 34.3 (b) 16.2
(a) phase rule (b) phase distribution (c) 21.6 (d) 14.8
(c) interphase separation (d) phase operation 141. The percentage of sulphur in an organic compound whose
129. In paper chromatography 0.32 g produces 0.233 g of BaSO4 [At. wt. Ba = 137, S = 32]
(a) moving phase is liquid and stationary phase in solid is
(b) moving phase is liquid and stationary phase is liquid (a) 1.0 (b) 10.0
(c) moving phase is solid and stationary phase is solid (c) 23.5 (d) 32.1
(d) moving phase is solid and stationary phase is liquid 142. An organic compound contains C = 40%, H = 13.33% and
130. Which of the following is used for detection of carbon N = 46.67%. Its empirical formula would be
and hydrogen ? (a) CHN (b) C2H2N
(a) Ca(OH)2 (b) CuO (c) CH4N (d) C3H7N
(c) CaCl2 (d) KOH 143. 2.79 g of an organic compound when heated in Carius tube
131. In sodium fusion test of organic compounds, the nitrogen
of the organic compound is converted into with conc. HNO 3 and H 3PO 4 formed converted into
(a) sodamide (b) sodium cyanide MgNH 4 .PO 4 ppt. The ppt. on heating gave 1.332 g of
(c) sodium nitrite (d) sodium nitrate
132. Which of the following compounds does not show Mg 2 P2O 7 . The percentage of P in the compound is
Lassaigne’s test for nitrogen ? (a) 23.33% (b) 13.33%
(a) Urea (b) Hydrazine (c) 33.33% (d) 26.66%
(c) Phenylhydrazine (d) Azobenzene 144. A compound contains 38.8% C, 16% H and 45.2% N. The
133. The compound formed in the positive test for nitrogen with formula of compound would be :
the Lassaigne solution of an organic compound is (a) CH 3 NH 2 (b) CH 3CN
(a) Fe4[Fe(CN)6]3 (b) Na3[Fe(CN)6]
(c) Fe(CN)3 (d) Na4[Fe(CN)5NOS] (c) C 2 H 5CN (d) C H2(NH2)2
ORGANIC CHEMISTRY – SOME BASIC PRINCIPLES & TECHNIQUES 189
145. In estimation of percentage of oxygen. The mixture of 150. Which of the following statements are correct for
gaseous products containing oxygen is passed over red fractional distillation ?
hot coke. All oxygen is converted to A . This mixture (i) Fractional distillation method is used if the two
is passed through B when A is converted to liquids have sufficiently large difference in their
_ C . boiling points.
What is A, B and C in above statement. (ii) A fractionating column provides many surfaces for
(a) A = CO2 , B = KOH , C = pure O2 heat exchange between the ascending vapours and
(b) A = CO , B = I2O5 , C = CO2 the descending condensed liquid.
(c) A = CO , B = I2 , C = CO2 (iii) Each successive condensation and vaporisation unit
(d) A = CO2 , B = Ca(OH)2 , C = CaCO3 in the fractionating column is called a theoretical
plate.
STATEMENT TYPE QUESTIONS (iv) Fractional distillation method is used to separate
146. Which of the following statement(s) is/are correct ? different fractions of crude oil in petroleum industry.
(i) A carbon atom having an sp hybrid orbital is less (a) (i), (ii) and (iv) (b) (ii), (iii) and (iv)
electronegative than carbon atoms possessing sp2 (c) (i), (ii) and (iii) (d) (i), (ii), (iii) and (iv)
or sp3 hybridised orbitals. 151. Which of the following sequence of T and F is currect for
(ii) -bonds provide the most reactive centres in the given statements. Here ‘T’ stands for True and ‘F’ stands
molecules containing multiple bonds for False statement.
(iii) The number of and bonds in compound (i) The relative adsorption of each component of
CH2= C= CHCH3 are 7 and 2 respectively. mixture is expressed in terms of its retardation factor
(a) (i) and (iii) (b) (ii) and (iii) (RF)
(c) (ii) only (d) (i) only (ii) Retardation factor is given as :
147. Which of the following sequence of T and F is correct.
Here T stands for true statement and F stands for false Distance moved by the solvent from base line
statement RF =
Distance moved by the substance from baseline
(i) In heterolytic cleavage the bond breaks in such a
fashion that the shared pair of electrons remain with (iii) In TLC the spots of colourless compounds can be
one of the fragment. detected by ultraviolet light.
(ii) In homolytic cleavage, one of the electrons of the (iv) Spots of amino acids may be detected by iodine.
shared pair in a covalent bond goes with each of the (a) TTTF (b) TFFF
bonded atoms. (c) TTTT (d) TFTF
heat or
(iii) R – X R +X 152. In Kjeldahl’s method for the estimation of N 2 , potassium
light
This equation represents heterolytic cleavage sulphate and copper sulphate are used. On the basis of
(a) TTF (b) FFT their functions which of the following statement(s) is/are
(c) FFF (d) TTT correct?
148. Which of the following is/are correct for inductive effect ? (i) Potassium sulphate raises the bpt. and ensures
(i) In inductive effect polarisation of sigma bond is complete reaction.
caused by the adjacent bond. (ii) Copper sulphate acts as catalyst.
(ii) Halogens, –NO2 , –CN, and –CH3 are electron withdrawing (iii) Potassium sulphate acts as catalyst and copper
groups. sulphate raises the bpt.
(iii) –CH2CH3and –OC6H5 are electron donating groups. (a) Only (iii) is correct (b) (i) and (ii) are correct
(a) (i) only (b) (ii) only (c) Only (ii) is correct (d) None is correct
(c) (i) and (iii) (d) (i), (ii) and (iii) 153. In the estimation of carbon and hydrogen by combustion
149. Which of the following sequence of T and F is correct for method which of the following is/are correct ?
given statements. Here T stands for correct and F stands
(i) A spiral of copper is introduced at the right extreme of
for false statement
combustion tube if the organic compound contains
(i) The more the number of contributing structures, the
nitrogen.
more is the resonance energy.
(ii) A spiral of silver is introduced if the organic compound
(ii) The resonance structures have different positions of
contains halogens.
nuclei but same number of unpaired electrons
(iii) The energy of actual structure of the molecule (the (iii) The copper oxide in the combustion tube is replaced
resonance hybrid) is lower than that of any of the by lead chromate if the organic compound contains
canonical structures. sulphur.
(a) TTT (b) TFT (a) (i) and (ii) are correct (b) (i) and (iii) are correct
(c) FFF (d) TFF (c) (ii) and (iii) are correct (d) All are correct
EBD_7207
190 ORGANIC CHEMISTRY – SOME BASIC PRINCIPLES & TECHNIQUES
(a) A – (q), B – (p), C – (s), D – (r)
MATCHING TYPE QUESTIONS (b) A – (p), B – (s), C – (q), D – (r)
154. Match the columns (c) A – (q), B – (p), C – (p), D – (r)
Column-I Column-II (d) A – (q), B – (p), C – (q), D – (r)
158. Match the columns.
(A) Non – benzenoid compound (p) Column-I Column--II
O CH3
|
(B) Alicyclic compound (q) (A) CH3 — C— CH 2 (p) Isobutyl
|
CH3
(C) Benzenoid compound (r) (B) CH3 CH 2 CH (q) sec – Butyl
|
S CH3
(D) Heterocyclic aromatic compound (s) (C) CH3 CH CH 2 (r) Neopentyl
|
CH3
O
(a) A – (r), B – (p), C – (s), D – (q) CH3
(b) A – (s), B – (p), C – (q), D – (r) |
(D) CH3 C (s) tert – Butyl
(c) A – (p), B – (r), C – (s), D – (q) |
(d) A – (r), B – (p), C – (q), D – (s) CH3
155. Match Column-I (organic compound) with Column-II
(a) A – (r), B – (q), C – (p), D –(s)
(common name of the compound) and choose the correct
(b) A – (s), B – (p), C – (r), D –(q)
option.
(c) A – (s), B – (p), C – (q), D –(r)
Column-I Column-II
(d) A – (s), B – (q), C – (p), D –(r)
(Organic compound) (Common name of compound) 159. Column-II give formula for compounds given in Column-I,
(A) C6H5OCH3 (p) Neopentane match them correctly.
(B) H3CCH2CH2 OH (q) Anisole Column-I Column-II
(C) (H3C)4C (r) Acetophenone (A) Propane (p) C2H5OH
(D) C6H5COCH3 (s) n – propyl alcohol. (B) ethyl alcohol (q) C3H8
(a) A – (r), B – (s), C – (p), D – (q) (C) carboxylic acid (r) CH3COOH
(b) A – (r), B – (p), C – (s), D – (q) (D) ethyl ethanoate (s) CH3COOC2H5
(c) A – (q), B – (s), C – (p), D – (r) (a) A – (q), B – (p), C – (r), D – (s)
(d) A – (q), B – (s), C – (r), D – (p) (b) A – (p), B – (q), C – (s), D – (r)
156. Match the columns (c) A – (q), B – (s), C – (p), D – (r)
Column-I Column-II (d) A – (q), B – (p), C – (s), D – (r)
(A) Aldehyde (p) Chloro 160. Match the columns
(B) Ketone (q) ol Column-I Column-II
(C) Alcohol (r) one (Organic compounds) (Type of isomerism)
(D) Halogen (s) al
(A) CH3CH2CH2CH2CH3 (p) Functional group
(a) A – (s), B – (r), C – (q), D – (p) isomerism
(b) A – (s), B – (q), C – (p), D – (r) CH3
|
(c) A – (p), B – (q), C – (r), D – (s) & CH3 CH CH 2CH3
(d) A – (r), B – (s), C – (q), D – (p)
157. Identify (i), (ii), (iii) and (iv) in the structure of given (B) CH3CH2CH2OH (q) Chain isomerism
organic compound. On the basis of your identification OH
match the columns. (iii) |
& CH 3 CH CH 3
OH
O H
||
CH3 – CH – CH2 – CH – CH – CH3 (iv) (C) CH3 C CH3 & CH3 (r) CH2Metamerism
C O
O H
H3C Br | |
CH 3 C CH 3 & CH 3 CH 2 C O
(i) (ii)
(D) CH3OC3H7 & C2H5OC2H5 (s) Position isomerism
Column-I Column-II
A (i) (p) Functional group (a) A – (s), B – (q), C – (r), D – (p)
B (ii) (q) Branch chain (b) A – (s), B – (q), C – (p), D – (r)
C (iii) (r) Parent chain (c) A – (q), B – (s), C – (r), D – (p)
D (iv) (s) Homologues unit (d) A – (q), B – (s), C – (p), D – (r)
ORGANIC CHEMISTRY – SOME BASIC PRINCIPLES & TECHNIQUES 191
161. Match the columns 165. Match the columns
Column-I Column-II Column - I Column - II
(A) CH3COOH & (p) Functional isomers
80 m1 100
HCOOCH3 (A) Duma’s method (p)
(B) 1 butene & (q) Metamers 188 m
2-butene 31 m1 100
(C) diethyl ether & (r) Position isomers (B) Kjeldahl’s method (q) %
methyl propyl ether 1877 m
(D) dimethyl ether (s) Chain isomers and ethanol
v1
and ethanol 1.4 M 2 v
(C) Carius method (r) 2
(a) A – (p), B – (r), C – (q), D – (s) %
(b) A – (q), B – (r), C – (s), D – (p) for bromine m
(c) A – (q), B – (s), C – (p), D – (r)
(d) A – (q), B – (p), C – (s), D – (r) (D) Percentage of phosphorus
162. Match the columns
28 V 100
Column-I Column-II (s) %
(A) Free radical (p) Trigonal planar 22400 m
(B) Carbocation (q Pyramidal (a) A – (s), B – (r), C – (p), D – (q)
(C) Carbanion (r) Linear (b) A – (r), B – (s), C – (q), D – (p)
(a) A – (p), B – (q), C – (r) (c) A – (s), B – (p), C – (q), D – (r)
(b) A – (p), B – (p), C – (q) (d) A – (p), B – (r), C – (q), D – (s)
(c) A – (r), B – (p), C – (q)
(d) A – (p), B – (p), C – (r) ASSERTION-REASON TYPE QUESTIONS
163. Match the columns
Directions : Each of these questions contain two statements,
Column - I Column - II
Assertion and Reason. Each of these questions also has four
(A) Separation of sublimable (p) Steam distillation
alternative choices, only one of which is the correct answer. You
compounds from non
have to select one of the codes (a), (b), (c) and (d) given below.
sublimable
(a) Assertion is correct, reason is correct; reason is a correct
(B) Method based on the (q) Sublimation
explanation for assertion.
difference in the solubilities
(b) Assertion is correct, reason is correct; reason is not a
of the compound and the
correct explanation for assertion
impurities in a suitable
solvent (c) Assertion is correct, reason is incorrect
(C) Separation of liquids (r) Distillation (d) Assertion is incorrect, reason is correct.
having sufficient difference 166. Assertion : A primary suffix indicates the type of linkage in
in their boiling points. the carbon atom.
(D) Separation of substances (s) Crystallisation Reason : CN is a Primary suffix
which are steam volatile 167. Assertion : The general formula for a dihydric alcohol is
and are immiscible with water. CnH2n(OH)2
(a) A – (q), B – (s), C – (r), D – (p) Reason : Ethylene glycol is a dihydric alcohol.
(b) A – (q), B – (r), C – (p), D – (s) 168. Assertion : IUPAC name of the following organic
(c) A – (s), B – (q), C – (r), D – (p) compound is 3, 4, 7 – trimethyloctane
(d) A – (q), B – (s), C – (p), D – (r)
164. Match the columns CH3 CH CH 2 CH 2 CH CH CH 2 CH3
| | |
Column - I Column - II CH3 CH3 CH 3
(Elements) (Colour of precipitate formed
in Lassaigne’s test) Reason : The numbering is done in such a way that the
(A) Nitrogen (p) Yellow branched carbon atoms get the lowest possible numbers.
(B) Sulphur (q) Prussian blue 169. Assertion : Chain isomerism is observed in compounds
(C) Chlorine (r) Violet containining four or more than four carbon atoms
(D) Phosphorus (s) White Reason : Only alkanes show chain isomerism
(a) A – (q), B – (r), C – (p), D – (s) 170. Assertion : But-1-ene and 2-methylprop-1-ene are position
(b) A – (r), B – (q), C – (p), D – (s) isomers.
(c) A – (q), B – (r), C – (s), D – (p) Reason : Position isomers have same molecular formula but
(d) A – (r), B – (q), C – (s), D – (p) differ in position of functional group or C = C.
EBD_7207
192 ORGANIC CHEMISTRY – SOME BASIC PRINCIPLES & TECHNIQUES

171. Assertion : Benzene exhibit two different bond lengths,


due to C – C single and C = C double bonds. OH

Reason : Actual structure of benzene is a hybrid of 179 IUPAC name of is :


following two structures.
OH

(a) But – 2 ene – 2, 3– diol


(b) Pent – 2 –ene – 2, 3 – diol
172. Assertion : Aniline is better nucleophile than anilium ion. (c) 2 – methylbut – 2 – ene – 2, 3 – diol
Reason : Anilium ion have +ve charge. (d) Hex – 2 – ene – 2, 3 – diol
173. Assertion : Different number of electron pairs are present 180. The state of hybridization of C2, C3, C5 and C6 of the
in resonance structures. hydrocarbon,
Reason : Resonance structures differ in the location of
CH3 CH3
electrons around the constituent atoms. | |
174. Assertion : Energy of resonance hybrid is equal to the CH3 C CH = CH CH C CH
7 6| 5 4 3 2 1
average of energies of all canonical forms.
CH3
Reason : Resonance hybrid cannot be presented by a single
structure. is in the following sequence
175. Assertion : Simple distillation can help in separating a (a) sp3, sp2, sp2 and sp (b) sp, sp2, sp2 and sp3
mixture of propan-1-ol (boiling point 97°C) and propanone (c) sp, sp2, sp3 and sp2 (d) sp, sp3, sp2 and sp3
(boiling point 56°C).
181. Which of the following numberings is correct ?
Reason : Liquids with a difference of more thatn 20°C in
their boiling points can be separated by simple distillation. FF 1 2 FF 3 2
A. 3 B. 1
176. Assertion : Components of a mixture of red and blue inks Br Br
5 4 4 5
can be separated by distributing the components between
stationary and mobile phases in paper chromatography. F 5 4 FF 5 1
Reason : The coloured components of inks migrate at C. 3 D. 2
1 Br Br
different rates because paper selectively retains different 2 4 3
components according to the difference in their partition
(a) A (b) B (c) C (d) D
between the two phases.
182. The ratio of - to - bonds in benzene is
177. Assertion : Sulphur present in an organic compound can
be estimated quantitatively by Carius method. (a) 1 : 4 (b) 1 : 2
Reason : Sulphur is separated easily from other atoms in (c) 3 : 1 (d) 1 : 6
the molecule and gets precipitated as light yellow solid. 183. In which of the compounds given below there is more than
one kind of hybridization (sp, sp2, sp3) for carbon ?
CRITICAL THINKING TYPE QUESTIONS (i) CH3CH2CH2CH3 (ii) CH3CH = CHCH3
178. The IUPAC name of the following compounds is (iii) CH2=CH–CH=CH2 (iv) H –C C – H
(a) (ii) (b) (iii) and (iv)
N
| (c) (i) and (iv) (d) (ii) and (iii)
H O 184. Which of the following represents the given mode of
hybridisation sp2–sp2 – sp – sp from left to right?
(a) N – phenyl ethanamide
(a) H2C = CH – C N (b) CH C – C CH
(b) N – phenyl ethanone
(c) N – phenyl methanamide CH2

(d) None of these (c) H2C = C = C = CH2 (d) CH2


ORGANIC CHEMISTRY – SOME BASIC PRINCIPLES & TECHNIQUES 193
x
185. The compound in which C uses its sp3 - hybrid orbitals for | |
bond formation is (a) F3 C F3 C C CH3 (b) H 3C F3C C F3C
| |
X X
(a) HCOOH (b) (H 2 N)2CO
| |
X X (c) F3C C F3C H3C (d) F3C C H 3C F3C
(c) (CH3 )3COH (d) CH 3 C HO | |
186. The IUPAC name of the compound is
193. The most stable carbanion among the following is

CH2 – CH2– CH2–


HO

(a) 3, 3-dimethyl - 1- cyclohexanol (a) (b)


(b) 1, 1-dimethyl-3-hydroxy cyclohexane
(c) 3, 3-dimethyl-1-hydroxy cyclohexane
(d) 1, 1-dimethyl-3-cyclohexanol CH2– CH2–

O (c) (d)
CH2– C – OH
OH
187. The IUPAC name of compound C is: OCH3 NO2
COOH
CH2– COOH
194. Which one of the following is a free-radical substitution
reaction?
(a) 1, 2, 3 - tricarboxy - 2, 1 - propane
(b) 3 - carboxy - 3 hydroxy - 1 , 5 - pentanedioic acid (a) CH 3CHO HCN CH 3CH ( OH ) CN
(c) 3 - hydroxy - 3 - carboxy - 1, 5 - pentanedioic acid
(d) 2 - hydroxy propane -1, 2, 3 - tricarboxylic acid. CH3 CH2Cl
(b) +Cl2
Boiling

188. The number of possible open chain (acyclic) isomeric


compounds for molecular formula C5H10 would be
(a) 8 (b) 7 Anh. AlCl3 CH3
(c) +CH3Cl
(c) 6 (d) 5
189. Which of the following compounds will show metamerism?
CH2Cl CH2NO2
(a) CH3–CO–C2H5 (b) C2H5–S–C2H5 (d) + AgNO2
(c) CH3–O–CH3 (d) CH3–O–C2H5
190. The compound C4H10O can show 195. Rate of the reaction
(a) metamerism (b) functional isomerism
O O –
(c) position isomerism (d) All of these R – C + Nu R – C + Z is fastest when Z is
191. Which pair of isomerism is not possible together? Z Nu

(a) Ring-chain and functional (a) OC2H5 (b) NH2


(b) Geometrical and optical (c) Cl (d) OCOCH3
(c) Metamerism and functional 196. For (i) I–,(ii) Cl –,
(iii) Br – , the increasing order of
nucleophilicity would be
(d) Metamerism and chain
(a) Cl– < Br– < I– (b) I– < Cl– < Br–
192. Which of the following represents the correct order of
stability of the given carbocations ? (c) Br– < Cl– < I– (d) I– < Br– < Cl–
EBD_7207
194 ORGANIC CHEMISTRY – SOME BASIC PRINCIPLES & TECHNIQUES

197. Which of the following is least reactive in a nucleophilic 200. The most suitable method for separtion of a 1 : 1 mixture of ortho
substitution reaction. and para nitrophenols is
(a) Sublimation (b) Chromatography
(a) (CH 3 )3 C Cl
(c) Crystallization (d) Steam distillation
(b) CH 2 CHCl 201. The Lassaigne’s extract is boiled with dil. HNO3 before
testing for halogens because
(c) CH 3CH 2Cl (a) silver halides are soluble in HNO3
(b) Na2S and NaCN are decomposed by HNO3
(d) CH 2 CHCH 2 Cl
(c) Ag2S is soluble in HNO3
198. Which of the following does not represent formation of
(d) AgCN is soluble is HNO3
reactive intermediate correctly ?
202. The molecular mass of an organic compound which contains
only one nitrogen atom can be
(i) CH3 CN C H3 CN
(a) 152 (b) 146
(c) 76 (d) 73
(ii) CH 3 C u CH3 Cu 203. 0.25 g of an organic compound on Kjeldahl's analysis gave
enough ammonia to just neutralize 10cm3 of 0.5 M H2SO4.
(iii) CH 3 B r CH3 Br The percentage of nitrogen in the compound is
(a) 28 (b) 56
(iv) CH 3 Cl CH3 Cl (c) 14 (d) 112
204. During hearing of a court case, the judge suspected that
(a) (ii) only (b) (ii) and (iii)
some changes in the documents had been carried out. He
(c) (ii) and (iv) (d) (iii) and (iv) asked the forensic department to check the ink used at two
199. In Lassaigne’s test, the organic compound is fused with a different places. According to you which technique can
piece of sodium metal in order to give the best results?
(a) increase the ionisation of the compound (a) Column chromatography
(b) decrease the melting point of the compound (b) Solvent extraction
(c) increase the reactivity of the compound (c) Distillation
(d) convert the covalent compound into a mixture of ionic (d) Thin layer chromatography
compounds
ORGANIC CHEMISTRY – SOME BASIC PRINCIPLES & TECHNIQUES 195

FACT / DEFINITION TYPE QUESTIONS 8. (b) C – 1 is sp hybridized C C


C – 3 is sp3 hybridized (C– C)
1. (a) Berzilius, a Swedish chemist proposed the concept
C – 5 is sp2 hybridized (C = C)
of ‘vital force’.
Thus the correct sequence is sp, sp3, sp2.
2. (d) Urea was first discovered in human urine by H.M. 9. (c) 10. (c)
Rouelle in 1773. It was synthesised in 1828 by Friedrich
Wohler and was the first organic compound to be H H H
| | |
synthesised from inorganic starting materials. It was H C C C C C H
found when Wohler attempted to synthesize ammo- 11. (b) | | | | | ;
nium cyanate, to continue a study of cyanates which H H H H H
he had be carrying out for several years. On treating 2 Pentene
silver cyanate with ammonium chloride solution he No. of bonds = 14, No. of bonds = 1
obtained a white crystalline material which proved iden- 12. (c) (ii) and (iii) do not represent 2-bromopentane
tical to urea obtained from urine. 13. (c) Correct expanded form of given structure is shown
3. (b) F. Wohler synthesised urea from an inorganic in option (c).
compound ammonium cyanate 14. (c)
Heat 15. (b) The successive members of a homologous series
NH4CNO ¾¾¾
® NH2CONH2
Ammonium Urea
differ by a – CH2 unit.
cyanate 16. (a) Correct molecular formula of icosane is C20H42
4. (c) According to vital force theory, organic compounds Correct molecular formula of heptane is C7H16
could only be produced by living matter by a vital 17. (c) (iii) is the only correct method of selecting parent
chain.
force. It was in 1828, Friedrich Wholer heated NH4CNO
18. (b) Correct order of decreasing priority is
(derived from inorganic substance) and obtained urea – COOH, – SO3H, – COOR, – OH.
(an organic compound). 19. (d) 20. (c) 21. (d) 22. (c) 23. (c)
5. (c) Wholer synthesized urea from ammonium cyanate in 24. (d) 2, 2,3-trimethyl pentane
1828. Kekule proposed catenation and structure of
benzene. Liebig is a history maker in sports science 1 1
(energy metabolism). CH3 CH3
1 2 | | 1
6. (d) Antoine-Laurent de Lavoisier (August 26, 1743 – May H3C H 2 C CH C CH 3
8, 1794) is known as the “father of modern chemistry.” 3 |4
He was a French nobleman prominent in the histories CH3
of chemistry, finance, biology, and economics. He 1
stated the first version of the Law of conservation of
1
mass, co-discovered, recognized and named oxygen CH 3
(1778) as well as hydrogen, disproved the phlogiston 1 | 2 1
4
theory, introduced the Metric system, invented the first 25. (d) CH 3 C CH 2 CH 3
|
periodic table including 33 elements, and helped to CH 3
reform chemical nomenclature. 1

7. (c) Hybridisation on the particular carbon can be Thus number of secondary hydrogens is two.
established by number of and bonds attached to it. CH3 CH3
Bond Bond Hybridisation | |
4 – sp3 26. (d) CH 3 C C CH 2 CH 3
| |
3 1 sp2 CH3 CH3
2 2 sp
(a )
1 2 3 4
C H2 CH CH C H2 CH3
3 3 3 3 |
1 1 1 1 CH 3 – C – CH 2 CH 3
|
sp 2 sp 2 sp 2 sp 2 CH3
Both carbon atoms forming C—C single bond (C2 and ( b)
C3) are sp2 hybridised
EBD_7207
196 ORGANIC CHEMISTRY – SOME BASIC PRINCIPLES & TECHNIQUES

CH3 CH3 Cl
1 | 3
| | CH3 — C — CH3
2
43. (c)
CH 3 C C HCH 2 CH 3 |
| CH3
CH3
2-chloro-2-methyl propane
(c )
44. (a)

CH 3 45. (d) (CH 3 ) 2 CHCH 2CH 2


3-methylbutyl group
|
CH 3 C HCH 2 CH 2CH 3 46. (a)
(d )
27. (d) In isopentane, (CH3)2CH CH2 CH3, every carbon is CH3
having hydrogen atom(s). 1 2| 3 4 5 6 7 8
47. (d) CH3CH CH 2CH CH CH CH CH CH3
H H H H H 2,8-Dimethyl-4,6-decadiene |
9 10
1° | 2° | 2° | 3° |3° |2° 1°
28. (a) H3C – C – C – C – C – C – CH3 CH2 — CH3
| | 1°| 1°| |
H H CH3 CH3 H
Ketonic group
3, 4-dimethylheptane
4 3 2 1
There are four 1° C-atoms, three 2° C-atoms and two 48. (a) CH3 CH CO CH3
3° C-atoms
29. (c) The structure of neopentane is CH3 methyl group
CH 3 3-methyl-2-butanone

H3C — C— CH3 CH3


49. (a)
|1
CH3 2 3
H3C C CH3
|
It has 1 quaternary and 4 primary carbons. CH3
30. (b) 31. (d) 32. (c) 33. (d) Neopentane
or 2, 2- Dimethylpropane
1 2 3
34. (a) CH 3 O CH 2 CH 2 CH 2 O CH 2 CH 3 50. (b)
3-ethoxy-1-methoxypropane 51. (b) The compound is a derivative of benzoic acid. The
35. (c) The correct name is 3 - methylbutan - 2 - ol positions of substituents attached to benzene nucleus
are represented by number of C-atoms and not by
Cl
1
ortho, meta and para.
6 2 52. (a) C3H6 has 2 structural isomers.
36. (a)
5 3 Br CH 3 CH CH 2 and H 2 C CH 2
4 propene
3-bromo-1chlorocyclohexene CH 2
cyclopropane
OH
1 2 3| 53. (c) C7 H7 Cl has 4 isomers
37. (c) CH3 CH 2 C CH 3
5 |4 CH3
CH3 CH 2 |
CH3 CH3 CH 2 Cl
3 Methyl pentan 3 ol | | |
Cl
|

38. (a) The IUPAC name of the given compound is |


5-chlorohex-2-ene. Cl Cl
|

39. (b) The compound is a derivative of butane. o-Chlorotoluene m-Chlorotoluene p-Chlorotoluene benzyl chloride
40. (b) The compound contains longest chain of 6C atoms 54. (b) Alcohols and ethers are functional isomers.
and amino group. Hence it is an alkanamine. 55. (b) Structures (a), (c) and (d) have the same molecular
41. (d) The compound is an ester. Its IUPAC name is derived formula (C6H12O) while (b) has C6H10O as molecular
from alkyl alkanoate. formula
42. (d) The compound is an aldehyde containing longest 56. (d)
chain of 6 C-atoms and side chains.
ORGANIC CHEMISTRY – SOME BASIC PRINCIPLES & TECHNIQUES 197
57. (b) CH3CH2CH2C CH,(CH3)2CHC CH, CH3CH2C CCH3 +
I II III CH2 CH2 CH2 CH2
58. (b) CH3CH2CH=CH2 CH3CH=CHCH3 (CH3)2C=CH2
1-butene (i) 2- butene (ii), (iii) 2-methylpropene
(cis,- trans) (iv)

CH3
Resonating structures of benzyl carbocation

whereas in alkyl carbocations dispersal of positive


charge on different hydrogen atoms is due to
hyper conjugation hence the correct order of stability
cyclobutane (v) methylcyclopropa ne (vi) will be
59. (b) 2, 2, 4, 4 - Tetramethylhexane has 10 carbon atoms,
only 4-isopropylheptane has also 10 carbon atoms so CH2
these two are isomers.
60. (a) 61. (c) 62. (a) 63. (b) 64. (d) > CH 2 CH C H 2 CH3 CH 2 CH 2
65. (c) Allyl Propyl
66. (b) The two isomers differ in the position of the double bond so Benzyl
they are called position isomers. 73. (b) Structure (b) is a 3º carbocation, while (a) is 2º and (c)
67. (b) and (d) are 1º carbocations; thus (b) is the most stable.
68. (b) The order of stability of carbocations is : 74. (a)
75. (a) Higher the possibility of delocalisation, greater is its
(CH3 )3 C (CH 3 )2 CH CH 3
stability; in C6 H5CHC6 H5 , +ve charge can delocalise
69. (c)
70. (d) Greater the number of alkyl groups attached to a over two benzene rings.
positively charged C atom, greater is the 76. (d) The organic reaction which proceed through
hyperconjugation (no bond resonance) and stable is heterolytic bond cleavage are called ionic or
the cation. heteropolar or just polar reactions.
77. (b) In carbocations, carbon bearing positive charge is
Thus order of decreasing stability of carbocation is,
always sp2-hybridised
tert – Alkyl > Sec-Alky > Pri-Alkyl > Methyl.
78. (b) Methyl carbanion is sp3 hybridised, with three bond
71. (a) Carbonium ions are electron deficient species. More
pairs and one lone pair same is the case with NH3.
the number of alkyl groups attached to it, more will be
stability due to + I effect. Cl
– – – –
79. (b) C Cl > C6H5CH2 > (CH3)2 CH > (CH3)3C
CH3 C CH3 > Cl
| –ve charge –M effect +I effect of CH 3 group
highly dispersed delocalises intensifies the –ve charge
CH3 due to – I effect –ve charge
3°carbonium ion
(+ve charge dispersed to maximum extent) 80. (c) In homolytic fission each of the atoms acquires one
(9 hyperconjugative H's) of the bonding electrons producing free radicals
(species having one unpaired electron).
CH 3 C– H > CH3CH 2 C H2 A B A + B
| 1° carbonium ion
(+ve charge least dispersed) 81. (b) Homolytic fission of the C – C bond gives free radicals
CH 2CH3 (2 hyper. H's) in which carbon is sp2- hybridised.
2° carbonium ion 82. (b) The carbon atom of alkyl free radicals which is bonded
(5 hyper. H's) to only three atoms or groups of atoms is sp 2 -
Alternatively, above order of stability order can be hybridized. Thus free radicals have a planar structure
explained in terms of hyperconjugation. with odd electrons situated in the unused p-orbital at
72. (d) Higher stability of allyl and aryl substituted methyl right angles to the plane of hybrid orbitals.
carbocation is due to dispersal of positive charge due
to resonance R

R 120°
CH 2 CH C H 2 CH 2 CH CH 2
Resonating structures of allyl carbocation
R
EBD_7207
198 ORGANIC CHEMISTRY – SOME BASIC PRINCIPLES & TECHNIQUES
83. (c) 103. (c) Resonance effect is the polarity produced in the
84. (a) On exposure to UV light, Cl2 molecule undergoes molecule by the interactions of two – bonds or
homolytic fission, to form chlorine free radicals. between a – bond and a lone pair of electrons
U.V. . present on an adjacent atom.
Cl Cl 2Cl 104. (b) Electromeric effect is purely a temporary effect and is
(Chlorine free radicals) brought into play only at the requirement of attacking
85. (b) The order of stability of free radicals reagent, it vanishes out as soon as the attacking
• • • • reagent is removed from reaction mixture.
(C 6 H 5 ) 3 C (C 6 H 5 ) 2 CH (CH 3 ) 3 C (CH 3 ) 2 C H
105. (b)
The stabilisation of first two is due to resonance and 106. (b) Alkyl groups with at least one hydrogen atom on the
last two is due to inductive effect. -carbon atom, attached to an unsaturated carbon
86. (d) Free radicals are stabilized by hyperconjugation, thus atom, are able to release electrons in the following
3° free radicals having maximum number of way.
hyperconjugative structures are the most stable, and
primary free radical the least.
.
87. (b) C6 H5CHCH3 is a 2º benzylic free radical, hence
stabilized most due to resonance. Note that the delocalisation involves and bond
88. (b) Dichlorocarbene, : CCl2 (a carbene) is the electrophile orbitals (or p orbitals in case of free radicals) ; thus it
formed as an intermediate in Reimer-Tiemann reaction. is also known as – conjugation. This type of
89. (d) Order of stability of free radicals is electron release due to the presence of the system
H—C—C = C is known as hyperconjugation
3° > 2° > 1° > C H3 107. (b) The stability of carbocation on the basis of hyper-
90. (c) The strength of nucleophile depends upon the nature conjugation can be explained as hyperconjugation
of alkyl group R on which nucleophile has to attack stabilises the carbocation because electron density
and also on the nature of solvent. The order of strength from the adjacent -bond helps in dispersing the
of nucleophiles follows the order : positive charge.
CN– > I– > C6H5O– > OH– > Br– > Cl–
91. (c) 92. (d)
H H H H H H H H
93. (a) Electrophile is positivly charged or electron deficient | | | | | | | |
species. Lewis acids are electron acceptors that is H C C H C C HC C H C C
| | | | | | |
electron deficient species. H H H
H H H H H
94. (b) Electrophiles are electron deficient or positively
charged species. In general greater the number of alkyl groups attached
95. (d) BF3 and R3C – X are electrophile while (CH3)3N and to a positively charged carbon atom, the greater is the
C2H5O– are nucleophile hyperconjugation interaction and stabilisation of the
96. (b) –CH3 group has +I effect, as number of –CH3 group cation. Thus, we have the following relative stability
increases, the inductive effect increases. of carbocation.
97. (d) Due to – I effect of the – CHO group, oxygen acquires-
CH3
- charge and the terminal carbon acquires + charge. |
CH3 C (CH3 ) 2 CH CH3 CH 2 CH3
CH2 = CH – C = O |
CH3
H
98. (c) All resonating structures should have same number Hence, stability of carbocation is directly proportional
of electron pairs. to number of alkyl group directly attached to
99. (a) The two structures involve only movement of carbocations.
electrons and not of atoms or groups, hence these are 108. (b) Stability order of different alkyl carbocations on the basis
resonating structures. of hyperconjugation is :
100. (b) Only structure (b) has a conjugated system, which is 3° > 2° > 1° > methyl
necessary for resonance. In t-butyl cation, the C-atom bearing the positive charge
101. (c) is attached to three methyl groups therefore it possess
nine -hydrogens. It will give maximum nine
102. (b) – OH shows + R effect while C = O shows – R hyperconjugative structures leading to maximum
effect. stability.
ORGANIC CHEMISTRY – SOME BASIC PRINCIPLES & TECHNIQUES 199
109. (b) In elimination reactions one or two molecules are lost contact with a “two phase system”. One phase is
from the substrate to form a multiple bond. Dehydration stationary and the other is moving or mobile. The
of ethanol is an example of elimination reaction. stationary phase may be a solid packed in a tube or a
Conc piece of paper. The mobile phase may be liquid of
C2 H5 OH CH 2 CH 2 H2O . gaseous.
H 2SO4
110. (a) 111. (d) 112. (a) 128. (b) 129. (b)
113. (b) Coloured impurities are removed by adsorbing over 130. (b) Carbon and hydrogen are detected by heating the
activated charcoal. compound with copper (II) oxide. Carbon present in
114. (a) This method is applied for the purification of the compound is oxidised to CO2 and hydrogen to
substances which (i) are insoluble in water, (ii) are H2O.
volatile in steam, (iii) are associated with non steam 131. (b)
volatile impurities, (iv) have high molecular weights 132. (b) Hydrazine (NH2NH2) does not contain carbon and
and (v) possess a fairly high vapour pressure at about hence on fusion with Na metal, it cannot form NaCN;
the boiling point of water e.g. Aniline. consequently hydrazine does not show Lassaigne’s
115. (a) Aniline is purified by steam distillation. test for nitrogen.
A mixture of water and aniline boils at 371 K and 760 133. (a) Prussian blue Fe 4 [Fe(CN) 6 ]3 is formed in lassaigne
mm pressure which is less than boiling point of water. test for nitrogen.
116. (b) Among the given compounds naphthelene is volatile
but benzoic acid is non-volatile (it forms a dimer). So, 3Na 4 [Fe(CN)6 Fe 3
the best method for their separation is sublimation,
which is applicable to compounds which can be Fe 4 [Fe(CN) 4 ]3 12Na
converted directly into the vapour phase from its solid Prussian blue
state on heating and back to the solid state on cooling. 134. (d)
Hence it is the most appropriate method. 135. (d) Kjeldahl method is not applicable to any of the given
117. (d) compounds. As nitrogen of these compounds does
118. (b) If there is a small difference (10 or less) in the boiling not change to ammonium sulphate on heating with
points of liquids fractional distillation is used e.g. ac- conc. H2SO4.
etone b.p. 333 K and methanol b.p. 338 K. 136. (c) 137. (b)
119. (a) Fractional distillation is used for the distillation of pe- 138. (b) In Kjeldahl’s method nitrogen is converted into
troleum. This method is used for separating a mixture (NH4)2 SO4, then to NH3
of two or more miscible, volatile liquids having close 139. (d) To increase the bpt of H2SO4,K2SO4 is added
(less than 40 degrees) boiling points. (For example, a
mixture of acetone, b.p., 56°C and methanol, b.p. 65°C) 1.4 N V
140. (b) N%
120. (c) If any liquid decomposes at its boiling point, it can be wt.of organic compound
purified by vacuum distillation.
121. (c) Glycerol decomposes at its boiling point, hence it 1.4 29 1/ 5
16.24%
should be purified by distillation under reduced 0.5
pressure.
32 0.233
122. (c) Vaccum distillation means distillation under reduced 141. (b) % of S 100 10 %
pressure. 233 0.32
123. (c) 142. (c) As in above question,
124. (a) The latest technique for the purification of organic 40 13.33 46.67
compounds is chromatography. These are of various C 3.33; H 13.33; N 3.34
12 1 14
types like column, paper and gas-chromatography.
125. (d) Both silica gel and alumina are used as adsorbents Relative No. of atoms,
in adsorption chromatography. 3.33 13.33 3.34
126. (a) Chromatography paper contains water trapped in it, C 1; H 4; N 1
3.33 3.33 3.33
which acts as the stationary phase.
Empirical formula = CH4N
127. (c) The mixture of sugars is a homogenous one.
Homogeneous mixtures of a solvent and one or more 62 wt . of Mg 2 P2 O 7
solutes (dissolved substances) are often separated by 143. (b) Percentage of P = × × 100
222 wt. of compound
chromatography. Chromatography works to separate
a mixture because the components of a mixture 62 1.332
distribute themselves differently when they are in = × × 100 = 13.33%
222 2.79
EBD_7207
200 ORGANIC CHEMISTRY – SOME BASIC PRINCIPLES & TECHNIQUES
144. (a) % of element Relative Simple ratio
MATCHING TYPE QUESTIONS
no. of atoms
38.8 154. (b) 155. (c) 156. (a) 157. (c) 158. (a)
C 38.8 3.2 1
12 159. (a) 160. (d) 161. (a) 162. (b) 163. (d)
16 164. (c) 165. (a)
H 16.0 16.0 5
1
ASSERTION-REASON TYPE QUESTIONS
45.28
N 45.28 3.2 1
14 166. (c) – CN is a secondary suffix.
145. (b) Compound heat O2 + Other gaseous products 167. (b)

1373K 168. (d) The correct name of the given compound is


2C O 2 2CO
(A) 2, 5, 6 -trimethyloctane
169. (c) 170. (d)
I 2O 5 5CO I 2 5CO 2
(B) (A) (C) 171. (c) Benzene has a uniform C – C bond distance of 139
pm, a value intermediate between the C – C single.
STATEMENT TYPE QUESTIONS (154 pm) and C = C double (134 pm) bonds.
146. (c) A carbon having an sp hybrid orbital with 50% 172. (a) It is fact that aniline is better nucleophile than anilium
s-character is more electronegative than carbon ion. Anilium ion contain +ve charge, which reduces
atoms having sp2 and sp3 hybrid orbitals with 33% the tendency to donate lone pair of electron
and 25% s-character respectively.
In CH2 = C = CHCH3 C6 H5 NH3+ .
Number of bonds : Anilium ion

C – C = 3, C – H = 6, total = 6 + 3 = 9
Number of bonds = 2 173. (d) Resonance structures contain the same number of
unpaired electrons. However, they differ in the way of
heat or
147. (a) R–X R +X distribution of electrons.
light

Above equation is an example of homolytic cleavge 174. (d) 175. (a) 176. (a) 177. (c)
148. (c) – CH3 is an electron donating group.
149. (b) The resonance structures have same positions of CRITICAL THINKING TYPE QUESTIONS
nuclei and same number of unpaired electrons.
178. (a) It is derivative of ethanamide having N-phenyl group.
150. (b) Fractional distillation method is used if the difference
in boiling points of two liquids is not much. 179. (b) The compound contains longest chain of 5C - atoms
151. (d) For statement (ii), and e of ene is retained as the suffix name starts with
Distance moved by the substance from baseline constant
RF = Distance moved by the solvent from base line
CH3 CH3
3|
For statement (iv), amino acids sports may be
180. (d) CH3 sp sp 2 sp3 | sp
detected by spraying the TLC plate with ninhydrin C CH CH CH — C C H
7 6| 5 4 3 2 1
solution. CH3
152. (b) K 2SO 4 raises bpt. and CuSO 4 acts as catalyst.
153. (d) When organic compound contains nitrogen, upon 181. (d) F *1
combustion it will produce oxides of nitrogen soluble 5
in KOH solution. The copper will convert them into Br
2
**
N2 4 3

2NO + 2Cu 2CuO + N 2 * **


The numbering of C-atom starts from C or C . But
2 NO 2 + 4Cu 4CuO + N 2 etc.
*
Halogens will be removed as AgX. In case of sulphur numbering from C give minimum locant (2) to Br which
SO 2 formed will be removed as PbSO 4 . is correct.
ORGANIC CHEMISTRY – SOME BASIC PRINCIPLES & TECHNIQUES 201

191. (c) Metamerism shown among compounds of the same


182. (a) functional group.
192. (b) – I group destablises carbocation and since inductive
No. of bonds = 12 ; No. of bonds = 3 effect decreases with increasing length of carbon
chain. Therefore (b) is the correct option.
Ratio of : bonds = 3 : 12 = 1 : 4
193. (d) –NO2 group, being strong electron-withdrawing,
183. (a) In compounds (i), (iii) and (iv), all carbon atoms are
disperses the –ve charge, hence stabilizes the
sp3, sp2 and sp hybridised, respectivley. However,
concerned carbanion.
compound (ii) has sp2 and sp3 hybridised carbon atoms;
194. (b) In the presence of UV rays or energy, by boiling chlorine,
sp 3
sp 2
sp 2
sp 3 free radical is generated which attack the methyl carbon
CH 3 CH CH CH 3 atom of the toluene.
184. (a) CH3 CH2
x +H
185. (c) See the number of bonds formed by C in each case.
benzyl free
radical
x x x x
In HCOOH , (H2 N)2CO and CH3C HO,C forms 3
2Cl•
h
Cl 2
bonds an d 1 bond, hybridisation is sp 2. In
•CH CH2Cl
x x 2 •
(CH3 )3COH , C forms 4 bonds, hence hybridisation + Cl
is sp3
195. (c) Cl– is the best leaving group among the given option.
196. (a) Nucleophilicity increases down the periodic table.
186. (a) 1 3 IUPAC name – 3, 3-Dimethyl -1 I Br Cl F
2
HO 197. (b) H2C CHCl is capable of showing resonance which
cyclohexanol develops a partial double bond character on C–Cl bond,
187. (d) The compound contains longest chain of 3 C - atoms thereby making it less reactive toward nucleophilic
and three -COOH groups and one -OH group attached substitution.
to it (latest convention). .. _
H 2C CH Cl
..
: H 2 C CH Cl
188. (c) C5H10 has 1º degree of unsaturation since the isomers
are acyclic, all of these are alkenes. For writing the
isomers, first introduce the double bond at different 198. (d) CH3 Br CH3 Br
possible positions, and then consider the possibility
of branching in the alkyl group. CH 3 Cl CH 3 Cl
CH3CH2CH2CH =CH2 CH3CH2CH=CHCH3
199. (d) To convert covalent compounds into ionic compounds
1-pentene (i) 2- pentene, (cis,- trans) (ii), (iii) such as NaCN, Na2S, NaX, etc.

CH3 CH3 200. (d) The boiling point of o-nitrophenol is less than para-
| | nitrophenol due to presence of intramolecular hydrogen
CH 3 CH CH CH 2 CH 3 CH 2 C CH 2 bonding. Since p-nitrophenol is less volatile in than o-
nitrophenol due to presence of inter molecular hydrogen
3-methyl-1-butene, (iv) 2-methyl-1-butene, (v)
bonding hence they can be separated by steam
CH3 distillation.
|
CH 3 C CHCH 3
201. (b) Na2S and NaCN, formed during fusion with metallic
sodium, must be removed before adding AgNO3,
2-methyl-2-butene, (vi) otherwise black ppt. due to Na2S or white precipitate
189. (b) 190. (d) due to AgCN will be formed and thus white precipitate
of AgCl will not be identified easily.
EBD_7207
202 ORGANIC CHEMISTRY – SOME BASIC PRINCIPLES & TECHNIQUES
203. (b) Percentage of N in a compound
Na 2S 2AgNO3 2NaNO3 Ag 2S
Black 1.4 Normality of acid Volume of acid used
=
Mass of the substance taken
NaCN AgNO 3 NaNO 3 AgCN
White Given, 0.5 M H2SO4 is used.
Normality = Molarity × n
NaCl AgNO 3 NaNO 3 AgCl
Mol.mass 98
where n 2
white Eq.mass 49

boil Normality = 0.5 × 2 = 1 N H2SO4


Na 2S 2HNO 3 2 NaNO 3 H 2S
Volume of acid used to neutralise
boil NH3 = 10 cm3
NaCN HNO 3 NaNO 3 HCN
Mass of organic compound taken = 0.25 g
202. (d) The compounds with odd number of N-atoms have
odd masses and with even number of N-atoms have 1.4 1 10
%N 56.
even masses. This is “nitrogen rule”. 0.25

204. (d)
13
HYDROCARBONS

10. The IUPAC n ame of the following compound


FACT / DEFINITION TYPE QUESTIONS
H3C CH2 CH CH2 CH CH2CH3
1. Which of the following fuel cause the least pollution ?
(a) Petrol (b) CNG
CH2CH3 CH3
(c) Kerosene (d) LPG
2. LPG mainly contains : is
(a) ethyne (b) butane (a) 3-ethyl-5-methylheptane
(c) methane (d) ethane (b) 5-ethyl-3-methylheptane
3. Natural gas is a mixture of : (c) 3,5-diethylhexane
(a) CH4 + C2H6 + C3H8 (b) CO + H2 + CH4 (d) 1,1-diethyl-3-methylpentane
(c) CO + H2 (d) H2O + CO2 11. The number of chain isomers possible for the hydrocarbon
4. Which of the following gas is find in coal mines and marshy C5H12 is
places? (a) 1 (b) 2
(a) Methane (b) Ethane (c) 3 (d) 4
(c) Benzene (d) Propane 12. The number of primary, secondary and tertiary carbons in 3,
5. Which of the following represents the correct general 4-dimethylheptane are respectively
formula of alkanes ? (a) 4, 3 and 2 (b) 2, 3 and 4
(a) CnH2n (b) CnH2n + 2 (c) 4, 2 and 3 (d) 3, 4 and 2
(c) CnH2n – 2 (d) CnHn 13. Name of the given compound -
6. Two adjacent members of a homologous series have CH3
(a) a difference of CH2 in their structure CH3
(b) a different of 14 amu in molecular mass
(c) same general method of preparation CH3
(d) All the above
7. Methane, ethane and propane are said to form a homologous CH3
series because all are (a) 2, 3-diethyl heptane (b) 5-ethyl-6-methyl octane
(a) hydrocarbons (c) 4-ethyl-3-methyl octane (d) 3-methyl-4-ethyl octane
(b) saturated compounds 14. Which of the following statements is false for isopentane–
(c) aliphatic compounds (a) It has three CH3 groups
(d) differ from each other by a CH2 group (b) It has one CH2 group
8. Which of the following does not belong to the same (c) It has one CH group
homologous series? (d) It has a carbon which is not bonded to hydrogen
(a) CH4 (b) C2H6 15. Molecular formula of which of the following alkane can
(c) C3H8 (d) C4H8 exist in more than one structure ?
9. In which of the following compounds only primary carbon (a) CH4 (b) C2H6
atoms are present? (c) C3H8 (d) C4H10
(a) CH3 CH CH2 CH3 (b) CH3 CH CH3 16. How many isomers are possible for the C5H12 ?
(a) 2 (b) 3
CH3 CH3
(c) 4 (d) 5
CH3 17. The number of 4° carbon atoms in 2,2,4,4-tetramethyl
pentane is –
(c) CH3 C CH3 (d) CH3 – CH3
(a) 1 (b) 2
CH3 (c) 3 (d) 4
EBD_7207
204 HYDROCARBONS
18. Which one of the following cannot be prepared by Wurtz 30. Aromatisation of n-hexane gives :
reaction ? (a) cyclohexane (b) benzene
(a) CH4 (b) C2H6 (c) cycloheptane (d) toluene
(c) C3H8 (d) C4H10 31. Liquid hydrocarbons can be converted to a mixture of
19. The reaction, gaseous hydrocarbons by :
CH 3 Br + 2Na + Br – CH 3 the product, is called (a) oxidation
(a) Wurtz reaction (b) Perkin’s reaction (b) cracking
(c) Aldol condensation (d) Levit reaction (c) distillation under reduced pressure
20. Pure methane can be produced by (d) hydrolysis
(a) Wurtz reaction 32. n-Hexane isomerises in presence of anhydrous aluminium
(b) Kolbe’s electrolytic method chloride and hydrogen chloride gas to give
(c) Soda-lime decarboxylation
(a) 2-Methyl pentane (b) 3-Methyl pentane
(d) Reduction with H2
21. Sodium salts of carboxylic acids on heating with soda lime (c) Both (a) and (b) (d) Neither (a) nor (b)
give alkanes containing _______ than the carboxylic acid. 33. Which of the following represents the correct reaction ?
(a) one carbon more (b) one carbon less Ni
(a) CH 4 2H 2 O CO 2 4H 2
(c) two carbon less (d) Either (a) or (b)
22. Which one of the following has the least boiling point? Ni
(b) CH 4 H 2O CO 3H 2
(a) 2, 2– dimethylpropane (b) n-butane
(c) 2-methylpropane (d) n-pentane Ni
(c) CH 4 H 2O CH 3OH H 2
23. Which one of the following has highest boiling point?
(a) n-Octane (b) 2,2 dimethyl pentane Ni
(d) CH 4 H 2O HCHO 2H 2
(c) Iso-octan (d) All have equal values
24. Which of the following reactions of methane is incomplete 34. How many conformations are possible for ethane ?
combustion ? (a) 2 (b) 3
(a) 2CH4 + O2 Cu / 523K /100 atm 2CH3OH (c) infinite (d) one
35. Spatial arrangements of atoms which can be converted into
(b) CH4 + O2 Mo 2O3 HCHO + H2O one another by rotation around a C–C single bond are called
(c) CH4 + O2 C(s) + 2H2O(l) (a) Stereoisomers (b) Tautomers
(d) CH4 + 2O2 CO2(g) + 2H2O(l) (c) Optical isomers (d) Conformers
25. In the free radical chlorination of methane, the chain initiating 36. General formula of alkenes and alkyl radicals are
step involves the formation of respectively:
(a) chlorine free radical (a) CnH2n and CnH2n+1 (b) CnH2n and CnH2n+2
(b) hydrogen chloride (c) CnH2n–1 and CnH2n (d) CnH2n+1 and CnH2n+2
(c) methyl radical
37. The restricted rotation about carbon-carbon double
(d) chloromethyl radical.
bond in 2- butene is due to
26. Which one of the following gives only one monochloro
derivative? (a) overlap of one s- and one sp2-hybridized orbitals
(a) n-hexane (b) 2-methylpentane (b) overlap of two sp2-hybridized orbitals
(c) 2, 3-dimethylpentane (d) neo-pentane (c) overlap of one p-and one sp2-hybridized orbitals
27. Photochemical halogenation of alkane is an example of (d) sideways overlap of two p-orbitals
(a) electrophilic substitution 38. Bond angle in alkenes is equal to
(b) electrophilic addition (a) 120° (b) 109°28'
(c) nucleophilic substitution (c) 180° (d) 60°
(d) free radical substitution 39. The molecular formula of a compound in which double bond
28. 2-Methylbutane on reacting with bromine in the presence is present between C & C :
of sunlight gives mainly
(a) CnH2n + 2 (b) CnHn
(a) 1-bromo-3-methylbutane
(b) 2-bromo-3-methylbutane (c) CnH2n (d) CnH2n–2
(c) 2-bromo-2-methylbutane 40. IUPAC name of the following compound is
(d) 1-bromo-2-methylbutane H3C CH CH 2 CH CH CH3
29. Complete combustion of CH 4 gives : |
Cl
(a) CO 2 H 2 O (b) CO 2 H 2
(a) 5-chloroheptene (b) 4-chloropent-1-ene
(c) COCl2 (d) CO + CO2 + H2O (c) 5-chloropent-3-ene (d) 5-chlorohex-2-ene
HYDROCARBONS 205
41. IUPAC name of the following compound will be 50. Geometrical isomerism is not shown by
CH3 CH C CH 2 CH3 CH3
|
CH 2 CH 2 CH3 |
(a) CH3CH 2C C CH 2CH 3
(a) 3–Ethyl–2–hexene (b) 3–Propyl–2–hexene |
CH3
(c) 3–Propyl–3–hexene (d) 4–Ethyl–4–hexene
42. Which of the following represents the correct IUPAC name
(b) C2H 5 C C CH 2 I
of the compound | |
CH2 = CH – CH2 Cl? H H
(a) Allyl chloride (b) 1- chloro -3- propene (c) CH 2 C(Cl)CH3
(c) 3- chloro-1- propene (d) Vinyl chloride
(d) CH 3 CH CH CH CH 2
43. The name of ClCH 2 C C CH 2 Cl according to
| | 51. Hex-2-ene and 2-methylpent-2-ene exhibit
Br Br (a) chain isomerism (b) position isomerism
IUPAC nomenclature system is (c) geometrical isomerism (d) optical isomerism
(a) 2, 3- dibromo -1, 4- dichlorobutene-2 52. Ethyl bromide gives ethylene when reacted with –
(b) 1, 4- dichloro-2, 3- dibromobutene-2 (a) ethyl alcohol (b) dilute H2SO4
(c) Dichlorodibromobutene (c) aqueous KOH (d) alcoholic KOH
53. By which reaction ethene is obtained from ethyne -
(d) Dichlorodibromobutane
(a) oxidation (b) polymerisation
44. The IUPAC name of
(c) hydrogenation (d) dehydrogenation
CH 3 CH CH C CHO 54. The major product formed when 2-bromobutane is treated
| | with alcoholic KOH is
OH CH 3 (a) 2-Butanol (b) 1-Butene
(a) 4-Hydroxy-1-methylpentanal (c) 1-Butanol (d) Trans-2-butene
(b) 4-Hydroxy-2-methylpent-2-en-1-al 55. Ethyl alcohol is heated with conc. H 2SO4 . The product
(c) 2-Hydroxy-4-methylpent-3-en-5-al formed is :
(d) 2-Hydroxy-3-methylpent-2-en-5-al
(a) H3C C OC2 H5 (b) C2 H 6
45. The alkene that exhibits geometrical isomerism is ||
(a) 2- methyl propene (b) 2-butene O
(c) 2- methyl -2- butene (d) propene
(c) C2 H 4 (d) C2 H 2
46. Which one of the following exhibits geometrical isomerism?
56. Alcoholic solution of KOH is used for
(a) 1, 2-dibromopropene (b) 2, 3-dimethylbut-2-ene
(a) Dehydrogenation (b) Dehalogenation
(c) 2, 3-dibromobut-2-ene (d) Both (a) and (c) (c) Dehydration (d) Dehydrohalogenation
47. The compounds CH3CH == CHCH3 and 57. Paraffins are soluble in
CH3CH2CH == CH2 (a) Distilled water (b) Benzene
(a) are tautomers (c) Methanol (d) Sea water
58. When hydrochloric acid gas is treated with propene in
(b) are position isomers
presence of benzoyl peroxide, it gives
(c) contain same number of sp3 – sp3, sp3 – sp2 and sp2 (a) 2-Chloropropane (b) Allyl chloride
– sp2 carbon-carbon bonds (c) No reaction (d) n-Propyl chloride.
(d) exist together in dynamic equilibrium 59. "The addition of unsymmetrical reagents to unsymmetrical
48. The total number of isomers for C4H8 is alkenes occurs in such a way that the negative part of the
(a) 5 (b) 6 addendum goes to that carbon atom of the double bond
(c) 7 (d) 8 which carries lesser number of hydrogen atoms" is called
49. Consider the following statements : A hydrocarbon of by :
molecular formula C5H10 is a (a) Saytzeff rule (b) Markownikoff's rule
(c) Kharasch effect (d) Anti-Saytzeff rule
I. monosubstituted alkene
60. When one mole of an alkene on ozonolysis produces 2
II. disubstituted alkene
moles of propanone, the alkene is
III. trisubstituted alkene (a) 3-methyl-1-butene
Which of the following statement(s) is(are) correct? (b) 2, 3-dimethyl-1-butene
(a) I, II and III (b) I and II (c) 2, 3-dimethyl-2-pentene
(c) II and III (d) I and III (d) 2, 3-dimethyl-2-butene
EBD_7207
206 HYDROCARBONS
61. Which alkene on ozonolysis gives CH3 CH2CHO and 69. Ethyl hydrogen sulphate is obtained by reaction of H2SO4
on
CH 3CCH 3
|| (a) Ethylene (b) Ethane
O (c) Ethyl chloride (d) Ethanol
CH3 70. The negative part of an addendum adds on to the carbon
(a) CH3CH2CH = C atom joined to the least number of hydrogen atoms. This
CH3 statement is called
(b) CH3CH2CH = CHCH2CH3 (a) Thiele’s theory (b) Peroxide effect
(c) CH3 CH2 CH = CH CH3 (c) Markownikoff’s rule (d) Baeyer’s strain theory
71. Which of the following compounds does not follow
(d) CH 3 C CHCH 3
| Markownikoff’s law ?
CH3 (a) CH3CH = CH2 (b) CH2CHCl
62. Reaction of HBr with propene in the presence of peroxide (c) CH3CH = CHCH3 (d) None of these
gives 72. In the following sequence of reactions, the alkene affords
(a) isopropyl bromide (b) 3-bromo propane the compound ‘B’
(c) allyl bromide (d) n-propyl bromide CH 3 CH CH CH 3
O3
A
H2O
B.
Zn
63. H 3 C CH CH CH 2 HBr A
| The compound B is
CH 3 (a) CH3CH2CHO (b) CH3COCH3
A(predominantly) is : (c) CH3CH2COCH3 (d) CH3CHO
(a) CH 3 CH CH 2 CH 2 Br 73. One mole of a symmetrical alkene on ozonolysis gives two
|
moles of an aldehyde having a molecular mass of 44 u. The
CH 3 alkene is
Br (a) propene (b) 1-butene
| (c) 2-butene (d) ethene
(b) CH3 C CH 2 CH3 74. The alkene that will give the same product with HBr in the
| absence as well as in the presence of peroxide is
CH3
(a) 2-butene (b) 1-butene
(c) CH3 CH CH CH 3 (c) propene (d) 1-hexene
| | 75. Ethylene reacts with alkaline KMnO4 to form
Br CH3
(a) Oxalic acid (b) HCHO
(d) CH 3 CH CH CH 3 (c) Ethyl alcohol (d) Glycol
| |
76. The reaction of HI with CH3 – CH = CH2 at 400°C yields :
CH 3 Br
(a) CH2 I – CH = CH2 (b) CH3 – CHI – CH3
64. Butene-1 may be converted to butane by reaction with (c) CH3 – CH2 – CH2 I (d) CH2I – CH2 – CH2I
(a) Sn – HCl (b) Zn – Hg
77. Ethene when treated with Br2 in the presence of CCl4 which
(c) Pd/H2 (d) Zn – HCl compound is formed
65. Alkenes usually show which type of reaction –
(a) 1, 2-dibromoethane (b) 1-bromo-2-chloroethane
(a) addition (b) substitution
(c) Both (a) and (b) (d) 1, 1, 1-tribromoethane
(c) elimination (d) superposition
78. In a reaction
66. A reagent used to test unsaturation in alkene is -
(a) ammonical Cu2Cl2 (b) ammonical AgNO3 CH 2 OH
Hypochlorous R
(c) solution of Br2 in CCl4 (d) conc. H2SO4 CH 2 CH 2 M |
acid
67. In the given reaction CH 2 OH
X Where M = molecule; R = reagent; M and R are
CH3CH 2 CH CHCH3
(a) CH3CH2Cl and NaOH
CH3CH 2COOH CH3COOH (b) CH3Cl – CH2OH and aq. NaHCO3
The X is (c) CH3CH2OH and HCl
(a) C2 H5ONa (b) Conc. HCl +Anhy.ZnCl2 (d) CH2 = CH2 and heat
79. The test for unsaturation is confirmed by the decolourisation
(c) Anh. AlCl3 (d) KMnO4/OH–
of which of the following
68. Polythene is a resin obtained by polymerisation of
(a) Iodine water (b) CuSO4 solution
(a) Butadiene (b) Ethylene
(c) Bromine water (d) All of these
(c) Methane (d) Ethyne
HYDROCARBONS 207
80. Isopropyl alcohol is obtained by reacting which of the 93. Which C-atom is the most electronegative in this structure?
following alkenes with conc. H2SO4 and H2O III II I
(a) Ethylene (b) Propylene CH 3 CH 2 C CH
(c) 2-methyl propene (d) Isoprene (a) I
81. Which one of the following is the strongest bond? (b) II
(a) > C = C < (b) — C C — (c) III
| | | (d) all are equal electronegative
(c) — C — C = (d) — C — C — Reagent
| | | | 94. R CH 2 CCl 2 R R C C R
82. An alkyne has general formula : The reagent is
(a) Na (b) HCl in H2O
(a) Cn H 2n (b) Cn H 2n 1
(c) KOH in C2H5OH (d) Zn in alcohol.
(c) Cn H 2n 2 (d) Cn H 2n 2 95. Calcium carbide when treated with water gives :
83. The IUPAC name of the compound (a) ethylene (b) methane
(c) acetylene (d) ethane
CH3CH = CHC CH is
96. Which one of the following has the minimum boiling
(a) Pent-l-yn-3-ene (b) Pent-4-yn-2-ene
point ?
(c) Pent-3-en-1-yne (d) Pent-2-en-4-yne
(a) 1-Butene (b) 1-Butyne
84. Number of alkynes for formula C5H8 is – (c) n- Butane (d) Isobutane
(a) 2 (b) 3 97. Ammonical silver nitrate forms a white precipitate easily
(c) 4 (d) 5 with
85. The IUPAC name of the compound having the formula (a) CH3C CH (b) CH3C CCH3
CH C – CH = CH2 is :
(c) CH3CH CH 2 (d) CH2 = CH2
(a) 1-butyn-3-ene (b) but-1-yne-3-ene
(c) 1-butene-3-yne (d) 3-butene-1-yne 98. When acetylene is passed through dil. H2SO4 in presence
of HgSO4, the compound formed is
86. The homologue of ethyne is
(a) ether (b) acetaldehyde
(a) C2H4 (b) C2H6
(c) acetic acid (d) ketone
(c) C3H8 (d) C3H6 99. Which of the following will be the final product when C2H2
87. The C - H bond length is minimum in the bond formed by reacts with HCl
(a) sp - s overlapping (as in alkynes)
(b) sp2 – s overlapping (as in alkenes) CH CH3
(c) sp3 – s overlapping (as in alkanes) (a) (b) |
CHCl CHCl 2
(d) None of these
88. Triple bond of ethyne is made of CHCl
(a) Three – bonds (c) (d) None of these
(b) Three – bonds CHCl
(c) Two and one – bond 100. The hydrocarbon which can react with sodium in liquid
(d) Two and one – bond ammonia is
89. Maximum carbon-carbon bond distance is found in – (a) CH 3CH 2 CH 2 C CCH 2CH 2 CH3
(a) ethyne (b) ethene (b) CH 3CH 2C CH
(c) ethane (d) benzene
90. The acetylene molecule contains : (c) CH 3CH CHCH3
(a) 5 sigma bonds (b) 4 sigma and 1 pi bonds (d) CH 3CH 2C CCH 2CH3
(c) 3 sigma and 2 pi bonds (d) 2 sigma and 3 pi bonds 101.Which of these will not react with acetylene?
91. Butyne-2 contains : (a) NaOH (b) Ammonical AgNO3
(a) sp hybridised carbon atoms only (c) Na (d) HCl.
102. When acetylene is passed over heated iron tube, the product
(b) sp3 hybridised carbon atoms only
obtained is –
(c) both sp and sp 2 hybridised carbon atoms (a) C2H2 (b) C4H4
(c) C6H6 (d) C8H8
(d) both sp and sp3 hybridised carbon atoms 103. But–2–yne on chlorination gives
(a) 1 –chlorobutane
92. The correct order towards bond length is
(b) 1, 2 –dichlorobutane
(a) C C C C C C (b) C C C C C C (c) 1, 1, 2, 2 –tetrachlorobutane
(c) C C C C C C (d) C C C C C C (d) 2, 2, 3, 3 –tetrachlorobutane
EBD_7207
208 HYDROCARBONS
104. When propyne reacts with aqueous H2SO4 in the presence 113. Benzene was discovered by
of HgSO4, the major product is (a) Ramsay (b) Dalton
(a) Propanal (c) Faraday (d) Priestley
(b) Propyl hydrogen sulphate 114.The ring structure of benzene was proposed by
(c) Acetone (a) Faraday (b) Davy
(d) Propanol (c) Kekule (d) Wohler
105. Propyne on polymerisation gives 115. Six carbon atoms f benzene are of
(a) Mesitylene (b) Benzene (a) one type (b) two types
(c) Ethyl benzene (d) Propyl benzene (c) three types (d) six types
106. What happens when a mixture of acetylene and hydrogen 116. Select the true statement about benzene amongst the
is passed over heated Lindlar’s catalyst ? following
(a) Ethane and water are formed
(a) Because of unsaturation benzene easily undergoes
(b) Ethylene is formed
addition
(c) Acetylene and ethane are formed
(b) There are two types of C – C bonds in benzene molecule
(d) None of these
(c) There is cyclic delocalisation of pi-electrons in benzene
107. Which of the following reaction is shown by alkynes ?
(a) Addition (b) Substitution (d) Monosubstitution of benzene gives three isomeric
(c) Polymerization (d) All of these products.
108. Which of the following reactions will yield 2, 117. The benzene molecule contains
2-dibromopropane ? (a) 6 sp2 hybrid carbons (b) 3 sp2 hybrid carbons
(a) HC CH 2HBr (c) 6 sp3 hybrid carbons (d) 3 sp3 hybrid carbons
118. Aromatic compounds burn with sooty flame because
(b) CH3C CH 2HBr (a) they have a ring structure of carbon atoms
(c) CH3CH CH 2 HBr (b) they have a relatively high percentage of hydrogen
(c) they have a relatively high percentage of carbon
(d) CH3CH CHBr HBr
(d) they resist reaction with oxygen of air
109. In the given reactions 119. Carbon atom in benzene molecule is inclined at an angle of
CH 3C CH
A
CH 3CBr2 CHBr2 (a) 120° (b) 180°
(c) 109° 28' (d) 60°
B
CH 3C CH CH 3CBr2 CH 3 120. The conditions for aromaticity is :
(a) molecule must have cyclic clouds of delocalised
Hg 2 H electrons
CH3C CH C
333 K
(b) molecule must contain (4n + 2) electrons
Hg 2 H
(c) Both (a) and (b)
HC CH D (d) None of the above
333 K
121. The chemical system that is non-aromatic is
A, B, C and D are respectively
(a) HBr, Br2, CH3COCH3, CH3CHO
(b) Br2, HBr, CH3COCH3, CH3CHO (a) (b)
(c) HBr, HBr, CH3COCH3, CH3CHO
(d) Br2, HBr, CH3CH2CHO, CH3CHO
110. Which of the following polymer can be used as electrodes +
in batteries ?
+ –
(a) Polypropene (b) Polyacetylene (c) (d)
(c) Polyethene (d) Polyisoprene
111. Which of the following catalyst is used for the following 122. Benzene can be directly obtained from
conversion ? (a) Acetylene (b) Phenol
(c) Chlorobenzene (d) All the above
3CH CH
873 K COONa
(a) Platinized Asbestos (b) Red hot iron tube
123. NaOH + CaO
(c) Platinized Nickel (d) Iron-molybdenum A
112. Which one of the following is a non-benzenoid aromatic
compound? The product A is
(a) Aniline (b) Benzoic acid (a) Benzene (b) Benzaldehyde
(c) Naphthalene (d) Tropolone (c) Toluene (d) Benzoic acid
HYDROCARBONS 209
124. In a reaction of C6H5Y, the major product (> 60%) is 134. Benzene on reaction with ozone forms __________.
m-isomer, so the group Y is (a) 2 molecules of aldehyde and 1 molecule of ketone
(a) –COOH (b) –NH2 (b) 2 molecules of ketone and 1 molecule of aldehyde
(c) –OH (d) –Cl (c) triozonide
(d) hexaozonide
HNO3 Br2 135. AlCl3 acts as ________ in Friedel-Crafts reaction
125. A B. The compound B is
H 2SO4 FeBr2
(a) nucleophile (b) electrophile
(c) free radical (d) intermediate
NO2 NO2
Br STATEMENT TYPE QUESTIONS
(a) (b)
136. The electrophilic substitutions reactions of benzene takes
Br place via
NO2 (i) generation of electrophile
NO2 (ii) generation of nucleophile
(iii) formation of carbocation intermediate
(c) (d) (iv) removal of proton from the carbocation intermediate
(a) (i), (iii) and (iv) (b) (ii), (iii) and (iv)
Br Br Br
(c) (i) and (iv) (d) (ii) and (iv)
126. Chlorobenzene is o, p-directing in electrophilic substitution 137. During the nitration of benzene. In the process of generation
reaction. The directing influence is explained by of nitronium ion sulphuric acid behaves as a/an ______
(a) + M of Ph (b) +I of Cl and nitric acid behave as a/an _________.
(c) + M of Cl (d) + I of Ph (a) base, acid (b) acid, base
(c) strong acid, weak acid (d) weak acid, strong acid
127. Catalytic hydrogenation of benzene gives
138. Benzene is highly unsaturated but it does not undergo
(a) xylene (b) cyclohexane
addition reaction because
(c) benzoic acid (d) toluene (a) -electrons of benzene are delocalised.
128. The strongest ortho - para and strongest meta - directing (b) cyclic structures do not show addition reaction
groups respectively are (c) benzene is a non-reactive compound
(a) –NO2 and –NH2 (b) –CONH2 and –NH2 (d) All of the above
(c) –NH2 and –CONH2 (d) –NH2 and –NO2 139. Which of the following statements are correct ?
129. For the formation of toluene by Friedal Craft reaction, (i) LNG is obtained by liquefaction of natural gas.
reactants used in presence of anhydrous AlCl 3 are (ii) Petrol is obtained by fractional distillation of petroleum.
(a) C2H2 and CCl4 (b) CH4 and CaCN2 (iii) Coal gas is obtained by destructive distillation of coal.
(c) C6H6 and CH3Cl (d) C2H5 OH and Zn (iv) CNG is found in upper strata during drilling of oil wells.
130. Benzene can be obtained in the reaction (a) (i), (ii) and (iv) (b) (i), (ii) and (iii)
(a) Ethene + 1, 3-butadiene (c) (i) and (iii) (d) (ii) and (iv)
(b) Trimerisation of ethyne 140. Which of the following statements are correct ?
(c) Reduction of PhCHO (i) Saturated hydrocarbons contain only carbon-carbon
(d) All of these single bonds.
131. Nitration of benzene by nitric acid and sulphuric acid is (ii) Saturated hydrocarbons contain both carbon-carbon
and carbon-hydrogen single bond.
(a) Electrophilic substitution
(iii) Unsaturated hydrocarbons contain carbon-carbon
(b) Electrophilic addition
double bonds.
(c) Nucleophilic substitution (iv) Unsaturated hydrocarbons contain carbon-carbon
(d) Free radical substitution double and triple bonds both.
BHC, anhydrous (a) (i) and (iii) (b) (ii) and (iv)
132. C6 H6 CH3Cl AlCl3
C6 H5CH3 HCl
(c) (i) and (ii) (d) (i) and (iv)
is an example of 141. Which of the following statements are correct regarding
(a) Friedel - Craft’s reaction structure of methane ?
(b) Kolbe’s synthesis (i) Methane has tetrahedral structure.
(c) Wurtz reaction (ii) The bond angle between all H – C – H bonds is 109.5°.
(iii) The carbon atom is sp2 hybridized.
(d) Grignard reaction
(iv) C – C and C – H bond lengths are 154 pm and 112 pm
133. Benzene reacts with CH3COCl + AlCl3 to give
respectively.
(a) chlorobenzene (b) toluene
(a) (i), (ii) and (iii) (b) (i), (iii) and (iv)
(c) benzyl chloride (d) acetophenone (c) (i), (ii) and (iv) (d) (i), (ii), (iii) and (iv)
EBD_7207
210 HYDROCARBONS
142. In the preparation of alkanes from hydrogenation of alkenes 148. Which of the following statements are correct ?
and alkynes. Finely divided catalysts are used which of the (i) Cis form of alkene is polar whereas trans form is
following statement(s) is/are correct regarding these non-polar
catalysts (ii) Cis form of alkene is non-polar whereas trans form is
(i) Platinum and palladium catalyse the reaction at room polar.
temperature. (iii) In case of solid alkenes the trans isomer has higher
(ii) Nickel catalyse the reaction at relatively higher melting point than the cis isomer.
temperature and pressure. (iv) Cis and trans both form have same properties.
(iii) Platinum and palladium catalyse the reaction at higher (a) (i) and (iii) (b) (ii) and (iii)
temperature. (c) (i), (iii) and (iv) (d) (i) and (iv)
(a) (i) and (iii) (b) (i) and (ii) 149. Which of the following statements are correct ?
(c) (ii) and (iii) (d) (i) only (i) Alkynes on reduction with palladised charcoal form
143. Which of the following statements are correct ? cis alkenes.
(i) The rate of reactivity of alkanes with halogens is (ii) Alkynes on reduction with palladised charcoal form
F2 > Cl2 > Br2 > I2. trans alkenes.
(ii) Rate of replacement of hydrogens of alkanes is (iii) Alkynes on reduction with sodium in liquid ammonia
form trans alkenes.
3° > 2º > 1°
(iv) Propyne on reduction with palladised charcoal form a
(iii) Fluorination of alkanes is a very slow process.
mixture of cis and trans propene.
(iv) Iodination of alkanes is too violent to be controlled.
(a) (i) and (iv) (b) (i) and (iii)
(a) (i), (ii) and (iii) (b) (i) and (ii)
(c) (ii) and (iv) (d) (i), (iii) and (iv)
(c) (ii) and (iii) (d) (i) and (iv)
150. Which of the following statements are correct ?
144. Which of the following statements are correct ? (i) Polynuclear hydrocarbons contain two or more
(i) Decomposition reaction of higher alkanes into smaller benzene rings fused together.
fragments by the application of heat is called pyrolysis. (ii) Polynuclear hydrocarbons have carcinogenic property.
(ii) Pyrolysis and cracking are different processes. (iii) Polynuclear hydrocarbons are formed on incomplete
(iii) Dodecane on pyrolysis gives a mixture of heptane and combustion of organic materials like tobacco, coal and
pentene. petroleum.
(iv) Pyrolysis follow free radical mechanism. (iv) They are also produced in human body due to various
(a) (i), (ii) and (iii) (b) (i), (ii) and (iv) biochemical reactions.
(c) (i), (iii) and (iv) (d) (ii) and (iv) (a) (i), (ii) and (iv) (b) (i), (iii) and (iv)
145. Which of the following statement(s) is/are correct ? (c) (ii), (iii) and (iv) (d) (i), (ii) and (iii)
(i) Alkanes can have infinite number of conformations
by rotation around a C – C single bonds. MATCHING TYPE QUESTIONS
(ii) Rotation around C – C single bond is completely free.
(iii) Rotation is hindered by a small energy barrier of 151. Match the columns
1-20 kJ mol–1 due to torsional strain. Column-I Column-II
(a) (i) and (ii) (b) (i) and (iii) H
(c) (ii) and (iii) (d) Only (iii) H H
146. Which of the following statements are correct ?
(i) Stability of conformation is affected due to torsional
(A) Eclipsed (p)
strain. H H
(ii) Magnitude of torsional strain depends upon the angle
of rotation about C – C bond. H
HH
(iii) Eclipsed form has least torsional strain.
(iv) Staggered form has maximum torsional strain.
(a) (i) and (iii) (b) (i) and (ii)
(c) (iii) and (iv) (d) (i) and (iv) (B) Staggered (q) H
147. Which of the following statements are correct ? H H
(i) The general formula of alkenes is CnH2n. H
(ii) Alkenes are also known as paraffins. HH
(iii) Bond length of C–C double bond in alkene is shorter
than C–C single bond in alkane.
(iv) Carbon–Carbon double bond in alkene consists of two (C) Skew (r) H
sigma bonds.
(v) Alkenes are easily attacked by electrophilic reagent. H H
H
(a) (i) and (iv) (b) (i), (iii) and (v) (a) A – (r), B – (p), C – (q) (b) A – (r), B – (q), C – (p)
(c) (i) and (iii) (d) (i), (ii), (iv) and (v) (c) A – (p), B – (q), C – (r) (d) A – (q), B – ( p), C – (r)
HYDROCARBONS 211
152. Match the columns (a) A – (s), B – (r), C – (q), D – (p)
Column-I Column-II (b) A – (q), B – (r), C – (s), D – (p)
(A) CH2 = CH2 CH3 – CH3 (p) H2, Zn, H+
(B) CH3Cl CH4 (q) NaOH, CaO (c) A – (r), B – (p), C – (q), D – (s)
(C) CH3Br CH3CH3 (r) H2, Pt/Pd (d) A – (q), B – (p), C – (s), D – (r)
(D) CH3COONa CH4 (s) Na, dry ether 156. Match the following reactants in Column I with the
(a) A – (r), B – (p), C – (s), D – (q) corresponding reaction products in Column II and choose
(b) A – (p), B – (s), C – (r), D – (q) the correct option from the codes given below.
(c) A – (s), B – (q), C – (p), D – (r)
(d) A – (q), B – (p), C – (s), D – (r) Column - I Column - II
153. Match the columns (A) Benzene + Cl2 AlCl3 (p) Benzoic acid
Column-I Column-II
(A) CH4 + O2 Cu/523K/100 atm (B) Benzene + CH3Cl (q) Methyl phenyl
(p) HCHO
Mo 2O3 AlCl3 ketone
(B) CH4 + O2 (q) (CH3)3COH
(C) Benzene + CH3COCl (r) Toluene
(CH3COO) 2 Mn
(C) C2H6 + O2 (r) CH3OH AlCl3
KMnO 4
(D) (CH3)3CH oxidation
(s) CH3COOH KMnO4 / NaOH
(D) Toluene (s) Chlorobenzene
(a) A – (s), B – (p), C – (r), D – (s)
(b) A – (q), B – (p), C – (s), D – (r) (a) A – (s), B – (r), C – (q), D – (p)
(c) A – (r), B – (p), C – (s), D – (q) (b) A – (s), B – (r), C – (p), D – (q)
(d) A – (p), B – (q), C – (r), D – (s) (c) A – (r), B – (s), C – (p), D – (q)
154. Match the columns
Column-I Column-II (d) A – (r), B – (s), C – (q), D – (p)
(A) CH CH H 2 (p) Zn 157. Match the columns
CH 2 CH 2 Column - I Column - II
(B) CH 3CH 2 Br (q) Conc. H2SO4 (A) Alkyl + Acid halide (p) Sulphonation
in presence of dry ether
CH 2 CH2
(B) Arene + Acid halide (q) Wurtz reaction
(C) CH 2 BrCH 2 Br (r) Pd/C
in presence of AlCl3
CH 2 CH2 (C) Arene + Fuming sulphuric (r) Catalytic
(D) CH3CH2OH (s) Alc. KOH in presence of AlCl3 hydrogenation
CH2 = CH2 (D) Arene + Hydrogen (s) Friedel-Crafts
(a) A – (r), B – (s), C – (p), D – (q)
in presence of Ni reaction
(b) A – (s), B – (r), C – (q), D – (p)
(c) A – (q), B – (p), C – (s), D – (r) (a) A - (p), B - (r), C - (q); D - (s)
(d) A – (r), B – (s), C – (q), D – (p) (b) A - (s), B - (q), C - (r); D - (p)
155. Match the columns (c) A - (r), B - (p), C - (s); D - (q)
Column-I Column-II
(d) A - (q), B - (s), C - (p); D - (r)
Cl
(A) (p) Cl2, uv, 500 K 158. Match the columns
Column - I Column - II
CH2Cl
(A) Aromatic (p) Planar
(B) (q) anhy. AlCl3
(B) Antiaromatic (q) Non-planar
Cl (C) Huckel rule (r) 4 n localised
Cl Cl electrons
(C) (r) CH2Cl2, anhy. AlCl3 (D) Cyclo-octatetraene (s) (4 n + 2) delocalised
Cl Cl electrons
Cl (a) A - (p, s), B - (p, r), C - (s), D - (q, r)
Cl
Cl Cl (b) A - (p, r), B - (p, s), C - (s), D - (q, r)
(c) A - (p, s), B - (s), C - (p, r), D - (q, r)
(D) (s) Cl2, anhy. AlCl3,
Cl Cl (d) A - (q, r), B - (p, r), C - (s), D - (p, s)
Cl
EBD_7207
212 HYDROCARBONS

159. Match the columns CRITICAL THINKING TYPE QUESTIONS


Column - I Column - II
165. In cyclopropane, cyclobutane and cyclohexane, the
(Reactants) (No. of chlorinated
common group is
products)
| |
Cl2 , light
(A) Benzene p. Three compounds (a) C (b) CH
| |
(B) Toluene Cl2 , light q. Four compounds
|
Cl2 , light (c) CH3 (d) CH2
(C) Methane r. Single monochloro |
derivative 166. The number of primary, secondary, tertiary and quaternary
Cl2 , AlCl3 carbons in neopentane are respectively
(D) Benzene s. Six isomeric
(a) 4, 3, 2 and 1 (b) 5, 0, 0 and 1
compounds (c) 4, 0, 0 and 1 (d) 4, 0, 1 and 1
(a) A – (r), B – (p, r), C – (q, r), D – (s)
(b) A – (s), B – (p, r), C – (q, r), D – (r) H C 4 H9
(c) A – (p, r), B – (s), C – (q, r), D – (r) | |
167. The IUPAC name of CH3 CH 2 C C CH3 : –
(d) A – (s), B – (p, r), C – (r), D – (q, r) | |
CH3CH3
ASSERTION-REASON TYPE QUESTIONS
(a) 3, 4, 4–Trimethyl octane
Directions : Each of these questions contain two statements,
(b) 3, 4, 4–Trimethyl heptane
Assertion and Reason. Each of these questions also has four
(c) 2–Ethyl, 3,3–dimethyl heptane
alternative choices, only one of which is the correct answer. You
(d) 2–Butyl, 2 methyl,3–ethyl butane
have to select one of the codes (a), (b), (c) and (d) given below.
168. Which one of the following has the lowest boiling point?
(a) Assertion is correct, reason is correct; reason is a correct
(a) 2-methylbutane (b) 2-methyl propane
explanation for assertion.
(c) 2, 2-dimethyl propane (d) n-pentane
(b) Assertion is correct, reason is correct; reason is not a 169. Arrange the following in decreasing order of their boiling
correct explanation for assertion points.
(c) Assertion is correct, reason is incorrect (A) n–butane (B) 2-methylbutane
(d) Assertion is incorrect, reason is correct. (C) n-pentane (D) 2, 2–dimethylpropane
(a) A > B > C > D (b) B > C > D > A
160. Statement-1 : 1-Butene on reaction with HBr in the presence
of a peroxide produces 1-bromobutane. (c) D > C > B > A (d) C > B > D > A
170. When neo-pentyl bromide is subjected to Wurtz reaction,
Statement-2 : It involves the free radical mechanism.
the product formed is
161. Statement-1 : CH4 does not react with Cl2 in dark.
(a) 2,2,4,4-tetramethylhexane
Statement-2 : Chlorination of CH4 takes place in sunlight. (b) 2,2,4,4-tetramethylpentane
162. Statement-1 : Iodination of alkanes is reversible. (c) 2,2,5,5-tetramethylhexane
Statement-2 : Iodination is carried out in presence of iodic (d) 2,2,3,3-tetramethylhexane
acid. 171. Which one of the following reactions is expected to readily
163. Statement-1 : All the hydrogen atoms in CH2 = C = CH2 lie give a hydrocarbon product in good yields ?
in one plane. Electrolyt ic
(a) RCOOK
Statement-2 : Carbon atoms are sp2 and sp hybridized. oxidation
164. Statement-1 : Tropylium cation is aromatic in nature
(b) Br2
RCOO Ag

Cl2
(c) CH 3 CH 3
h
Statement-2 : The only property that determines its aromatic C 2 H 5OH
(d) (CH 3 ) 3 CCl
behaviour is its planar structure.
HYDROCARBONS 213
172. A hydrocarbon A on chlorination gives B which on heating Dipole moment B.P. M.P. Stability
with alcoholic potassium hydroxide changes into another (a) I > II I > II II > I I > II
hydrocarbon C. The latter decolourises Baeyer's reagent (b) II > I II > I II > I II > I
and on ozonolysis forms formaldehyde only. A is (c) I > II I > II I > II I > II
(a) Ethane (b) Butane (d) II > I II > I I > II I > II
(c) Methane (d) Ethene 181. But-2-ene exhibits cis-trans-isomerism due to
173. Which of the following compounds can yield only one (a) rotation around C3 – C4 sigma bond
monochlorinated product upon free radical chlorination? (b) restricted rotation around C = C bond
(a) Propane (b) 2, 2-Dimethylpropane (c) rotation around C1 – C2 bond
(c) 2-Methylpropane (d) n-Butane (d) rotation around C2 – C3 double bond
174. In the eclipsed conformation of ethane, the dihedral angle 182. In the following reactions,
between the hydrogen atoms of adjacent methyl groups
CH3
is H + /Heat
A + B
(i) CH –CH–CH–CH
CH3–CH–CH–CH33
(a) 60° (b) 120° Major Minor
(c) 0° (d) 180° OH products products

175. The nodal plane in the -bond of ethene is located in (ii) A


A
HBr,dark
CC D
D
(a) the molecular plane inabsenceof
in absenceofperoxide
peroxide Major Minor
product product
(b) a plane parallel to the molecular plane
the major products (A) and (C) are respectively :
(c) a plane perpendicular to the molecular plane which bisects
the carbon - carbon -bond at right angle CH3 CH3
(d) a plane perpendicular to the molecular plane which (a) CH2 2= C – CH2–2 CH33 and CH22– CH– CH22– CH33
contains the carbon - carbon -bond. Br
176. The IUPAC name of the compound having the formula CH3
CH3
(CH3)3CCH = CH2 is –
(a) 3, 3, 3-trimethyl-1-propane (b) CH3–3 C = CH–CH33 and CH3– C – CH2– CH3
(b) 1, 1, 1-trimethyl-1-butene Br
(c) 3, 3-dimethyl-1-butene CH3 CH 3
(d) 1, 1-dimethyl-1, 3-butene (c) CH33– C = CH – CH33 and CH3 – CH – CH – CH 3
177. The IUPAC name of the following compound is
Br
Cl CH2CH3
CH3 CH3
C C (d) CH22= C – CH22– CH33 and CH33– C – CH22 – CH33
CH3 I Br
183. When 3, 3-dimethyl 2-butanol is heated with H2SO4, the
(a) trans-2-chloro-3-iodo-2-pentene
major product obtained is
(b) cis-3-iodo-4-chloro-3-pentene
(a) 2,3-dimethyl 2-butene
(c) trans-3-iodo-4-chloro-3-pentene
(b) 3, 3-dimethyl 1- butene
(d) cis-2-chloro-3-iodo-2-pentene
(c) 2, 3-dimethyl 1- butene
178. The number of possible open chain (acyclic) isomeric
(d) cis & trans isomers of 2, 3-dimethyl 2-butene
compounds for molecular formula C5H10 would be
184. An alkene having molecular formula C7H14 was subjected
(a) 8 (b) 7 to ozonolysis in the presence of zinc dust. An equimolar
(c) 6 (d) 5 amount of the following two compounds was obtained
179. Correct order of stability is :
CH3 CH3
(a) cis -2- butene > 1-butene > trans -2-butene C = O and C=O
CH3 CH3CH2
(b) trans-2-butene > cis-2-butene > 1-butene
(c) 1-butene > cis-2-butene > trans-2- butene The IUPAC name of the alkene is
(d) cis-2-butene > trans-2-butene > 1-butene (a) 3, 4-dimethyl-3-pentene (b) 3, 4-dimethyl-2-pentene
180. Which of the following is correct set of physical properties (c) 2, 3-dimethyl-3-pentene (d) 2, 3-dimethyl-2-pentene
of the geometrical isomers – 185. Reaction of hydrogen bromide with propene in the absence
of peroxide is a/an
CH3 CH3 (a) free radical addition
(b) nucleophilic addition
(c) electrophilic substitution
(d) electrophilic addition
EBD_7207
214 HYDROCARBONS
186.Which of the following types of reaction occur when a 195. Acetylenic hydrogens are acidic because
reactant has got a double bond ? (a) Sigma electron density of C – H bond in acetylene is
(i) Addition nearer to carbon, which has 50% s-character
(ii) Photolysis (b) Acetylene has only open hydrogen in each carbon
(iii) Nucleophilic substitution (c) Acetylene contains least number of hydrogens among
(iv) Polymerization the possible hydrocarbons having two carbons
(a) (i) and (iv) (b) (i), (ii) and (iii) (d) Acetylene belongs to the class of alkynes with
(c) (iii) and (iv) (d) (ii) and (iii) molecular formula, CnH2n – 2.
187. The disappearance of the characteristic purple colour of 196. Propyne can be prepared by dehydrohalogenation of
KMnO4 in its reaction with an alkene is the test for (a) 1–chloropropane
unsaturation. It is known as (b) 1, 2–dichloropropane
(a) Markownikoff test (b) Baeyer test (c) 1, 2–dichloroethane
(c) Wurtz test (d) Grignard test (d) 1, 1, 2, 2–tetrachloroethane
188. CH2 = CHCl reacts with HCl to form 197. Which is the most suitable reagent among the following to
distinguish compound (3) from rest of the compounds ?
(a) CH 2 Cl CH 2 Cl (b) CH3 CHCl2
1. CH 3 C C CH 3
(c) CH 2 CHCl.HCl (d) None of these
2. CH 3 CH 2 CH 2 CH 3
189. The only alcohol that can be prepared by the indirect
hydration of alkene is 3. CH 3 CH 2C CH
(a) Ethyl alcohol (b) Propyl alcohol 4. CH 3 CH CH 2.
(c) Isobutyl alcohol (d) Methyl alcohol
(a) Bromine in carbon tetrachloride
190. Which reactions are most common in alkenes
(b) Bromine in acetic acid
(a) Electrophilic substitution reactions
(c) Alk KMnO4
(b) Nucleophillic substitution reactions
(d) Ammonical silver nitrate.
(c) Electrophilic addition reactions 198. Predict the product C obtained in the following reaction of
(d) Nucleophilic addition reactions butyne-1.
191. In the presence of peroxide, hydrogen chloride and hydrogen HI
iodide do not give anti-Markownikov’s addition to alkenes CH 3CH 2 C CH HCl B C
because
I
(a) Both are highly ionic |
(a) CH3 CH2 CH 2 C H
(b) One is oxidising and the other is reducing
|
(c) One of the steps is endothermic in both the cases
Cl
(d) All the steps are exothermic in both the cases
192. Which of the following statements is incorrect regarding I
|
dehydrohalogenation of alkenes ? (b) CH 3 CH 2 CH CH 2 Cl
(a) During the reaction hydrogen atom is eliminated from
the - carbon atom. I
|
(b) Rate of reaction for same alkyl group; (c) CH3CH 2 C CH3
Iodine > Bromine > Chlorine |
(c) Rate of reaction; (CH3)3C – > (CH3)2CH – > CH3CH2 – Cl
(d) Only nature of halogen atom determine rate of the (d) CH3 CH CH2CH 2 I
reaction. |
193. How many structural isomers are possible for the alkyne Cl
C6H10 ? 199. The correct increasing order of acidity of the following
alkynes
(a) 7 (b) 6
(c) 8 (d) 5 (1) CH3 C C CH3
194. Which of the following will have least hindered rotation (2) CH3 C CH
around carbon - carbon bond ?
(3) CH CH
(a) Ethane (b) Ethylene
(a) 1 < 2 < 3 (b) 2 < 3 < 1
(c) Acetylene (d) Hexachloroethane (c) 3 < 2 < 1 (d) 1 < 3 < 2
HYDROCARBONS 215
200. Identify the alkyne in the following sequence of reactions. (a) It forms only one monosubstitution product
H2 Ozonolysis Wacker (b) The C - C bond distance in benzene is uniformly 1.397Å
Alkyne A B (c) It is a resonance hybrid of a number of canonical forms
Lindlar's catalyst only Process
(d) It has three delocalised -molecular orbitals
CH2 = CH2
206. The ratio of to bonds in benzene is :
(a) H3C C C CH3 (a) 2 (b) 3
(b) H3C CH 2 C CH (c) 4 (d) 8
.
(c) H 2C CH C CH 207. The radical, CH2 is aromatic because it has :
(d) HC C CH 2 C CH (a) 7 p-orbitals and 6 unpaired electrons
201. Which of the following represent the correct order of acidic (b) 7 p-orbitals and 7 unpaired electrons
strength ? (c) 6 p-orbitals and 7 unpaired electrons
(i) HC CH H 2C CH 2 CH3 CH3 (d) 6 p-orbitals and 6 unpaired electrons
208. (i) Chlorobenzene and (ii) benzene hexachloride are
(ii) HC CH CH3 CH3 H 2C CH 2
obtained from benzene by the reaction of chlorine, in the
(iii) CH3C CH HC CH CH3 C C CH3 pesence of
(a) (i) Direct sunlight and (ii) anhydrous AlCl3
(iv) HC CH CH3 C CH CH3 C C CH3
(b) (i) Sodium hydroxide and (ii) sulphuric acid
(a) (i) and (iii) (b) (ii) and (iv)
(c) (i) Ultraviolet light and (ii) anhydrous FeCl3
(c) (i) and (iv) (d) (i) and (iv)
202. Which one of these is not compatible with arenes? (d) (i) Anhydrous AlCl3 and (ii) direct sunlight
(a) Greater stability 209. A group which deactivates the benzene ring towards
(b) Delocalisation of -electrons electrophilic substitution but which directs the incoming
group principally to the o- and p-positions is
(c) Electrophilic additions
(d) Resonance (a) –NH2 (b) –Cl
(c) –NO2 (d) –C2H5
COOH COOH
210. Benzene can be obtained by heating either benzoic acid
CHO with X or phenol with Y. X and Y are respectively.
203. and are
(a) Zinc dust and soda lime
(b) Soda lime and zinc dust
(a) Position isomer (b) Chain isomer
(c) Functional isomer (d) Stereoisomer (c) Zinc dust and sodium hydroxide
204. The carbon-carbon bond length in benzene is (d) Soda lime and copper
(a) Same as in C2H4 211. Which of the following chemical system is non aromatic?
(b) In between C2H6 and C2H2
(c) In between C2H4 and C2H2 (a) (b)
(d) In between C2H6 and C2H4
205. Point out the wrong statement in relation to the structure of
benzene (c) (d)
S
EBD_7207
216 HYDROCARBONS

FACT / DEFINITION TYPE QUESTIONS H


H–C–H
1. (d) LPG is used as a domestic fuel with the least pollution. H H

–— —–
H H H H
2. (b) LPG mainly contains butane.

– –

– –
– –

– – –

– –
– –
H –C—C —C –C –H H – C –— C –— C – H
3. (a) Natural gas is a mixture of CH4, C2H6 and C3H8.
4. (a) 5. (b) 6. (d) 7. (d) 8. (d) H H– C –H H H H H
H H–C–H
1° 1°


9. (d) CH3 CH3 H
17. (b)
Both carbon atoms in ethane are primary. 18. (a) CH4 has only one carbon atom, hence it can’t be
10. (a) prepared by Wurtz reaction, which involves two
11. (c) Pentane (C5H12) exists as three chain isomers molecules of alkyl halide.
CH3CH 2 CH 2CH 2 CH3 CH3 CH CH2 CH3 19. (a) When alkyl halide is treated with sodium metal in
n -pentane | presence of ether, alkane is obtained, this reaction is
CH3 called as Wurtz reaction.
2 methylbutane (iso pentane)
dry ether
R X 2Na X R ' R R ' 2 NaBr
dry ether
CH3 CH3–Br + 2Na+Br–CH3
|
H3C C CH3 CH 3 CH 3 2 NaBr
| 20. (c) Other three methods can be used for the preparation
CH3 of alkane having at least two carbon atoms.
2,3-dimethylpropane (neo pentane)
21. (b) Sodium salts of carboxylic acids on heating with soda
lime (mixture of sodium hydroxide and calcium oxide)
H H H H H give alkanes containing one carbon atom less than the
1° | 2° | 2° | 3° |3° |2° 1°
12. (a) H3C – C – C – C – C – C – CH3 carboxylic acid. This process of elimination of carbon
| | 1°| 1°| | dioxide from a carboxylic acid is known as
H H CH3 CH3 H
decarboxylation
3, 4-dimethylheptane
CaO
CH3COO Na NaOH CH 4 Na 2CO3
There are four 1° C-atoms, three 2° C-atoms and two
3° C-atoms Sodium ethanoate Methane
13. (c) 14. (d) 22. (a) Higher is the branching lesser will be the boiling point
15. (d) CH4, C2H6 and C3H8 can have only one structure but further increase in molecular weight increases boiling
C4H10 can have more than one structure. Possible point in alkane. Hence 2, 2– dimethyl propane will have
structures of C4H10 are following least boiling point.
H H H H CH3
2 1
3 4
|
H C C C C H CH3 C CH 3
|
H H H H CH3
Butane (n- butane), (b.p. 273 K)
23. (a) n-octane has highest boiling point due to unbranched
H H H chain and maximum carbon atoms. It has max. Van der
1 2 3
H C C C H Waal forces.
HH C HH 24. (c)
hv
H 25. (a) Cl2 2Cl•
Chain initiation
2-Methylpropane (isobutane)
(b.p.261 K) CH3
|
16. (b) Possible isomers of C5H12 are 26. (d) Neo-pentane, H3C C CH3 , has only 1° hydrogen
H H H H H |
CH3
– –
– –
– –
– –
– –

H–C–C–C–C–C–H
and hence gives only one monochloro derivative.
H H H H H
HYDROCARBONS 217
27. (d) 5 4 3 2
44. (b) CH 3 CH CH C 1CHO
CH3 CH3 | |
|
Br2
| OH CH 3
28. (c) CH3 CH CH2 CH3 CH3 C CH2CH3
4-Hydroxy-2-methylpent-2-en-1-al
sunlight |
Br 45. (b) H3C CH3 H3C
2 bromo 2 methyl butane H
C=C C=C
Ease of replacement of H-atom 3° > 2° > 1°.
29. (a) Complete combustion of all organic compounds leads H H H CH3
to formation of CO2 + H2O. 46. (d) Alkenes having double bonds with two different
30. (b) CH 3 CH 2 CH 2 CH 2 CH 2 CH 3 groups on each end of the double bond show
geometrical isomerism. A2b2c2, A2b2cd, A2bcde.
Aromatisation
Br Br
C C 1,2-dibromopropene
Benzene H CH3
Aromatisation is a process in which aromatic
compounds are formed from open chain compounds. Br Br
C C 2,3-dibromobut-2-ene
31. (b) During cracking higher hydrocarbons (liquid) are H3C CH3
converted to lower gaseous hydrocarbons. 47. (b) The two isomers differ in the position of the double
32. (c) n-Alkanes on heating with anhydrous aluminium bond so they are called position isomers.
chloride and hydrogen chloride gas isomerise to 48. (b) CH3CH2CH=CH2 CH3CH=CHCH3 (CH3)2C=CH2
branched chain alkanes. 1-butene (i) 2- butene (ii), (iii) 2-methylpropene
33. (b) Methane reacts with steam at 1273 K in the presence (cis,- trans) (iv)
of nickel catalyst to form carbon monoxide and CH3
dihydrogen. This method is used for industrial
preparation of dihydrogen gas.
34. (c) A conformation is defined as the relative arrangement
of atoms or groups around a central atom, obtained by
the free rotation of one part of the molecule with respect cyclobutane (v) methylcyclopropa ne (vi)
to rest of the molecule. For a complete rotation of 360º, 49. (a) As sketched in the above question , C5H10 may be
one part may rotate through any degree say 0.1º, 0.5º, monosubstituted (i) and (iv), disubstituted as in (ii),
1º etc. giving rise to infinite number of relative (iii) and (v) and trisubstituted as in (vi)
arrangements of group (atom) around a central atom, 50. (c) The condition for geometrical isomerism is
keeping other part fixed. a a a e
C=C or C = C
35. (d) Spatial arrangements of atoms which can be converted
b b b d
around a C – C single bond are called conformations 51. (a)
or conformers or rotamers.
36. (a) 52. (d) CH3 CH2 Br KOH CH 2 CH 2 KBr H 2O
(alc)
37. (d) 53. (c)
38. (a) As predicted by the VSEPR model of electron pair
repulsion, the molecular geometry of alkenes Br
includes bond angles about each carbon in a double |
bond of about 120°. 54. (d) CH3– CH – CH2 – CH3 alc. KOH
2-Bromobutane
39. (c) Double bond in between carbon-carbon is present in +
alkenes whose general formula is CnH2n. CH3–CH–CH2–CH3
+
6 5 4 3 2 1 –H
40. (d) H3C CH CH 2 CH CH CH3 H3C H
| C=C
H CH3
Cl trans-2-butene
IUPAC name : 5- chlorohex-2-ene conc. H 2SO 4
41. (a) 55. (c) C2 H5 OH C2H 4 H 2O
KMnO4
1 2 3 Note : If ethyl alcohol is taken in excess and the reaction
42. (c) CH2= CH–CH2Cl is carried out at a temperature of 433-443 K diethyl
43. (a) Since b (from bromo) comes earlier in alphabetical order ether is formed.
than c (from chloro), the correct name should be 2, 3- conc. H2SO4
dibromo-1, 4-dichlorobutene-2 and not 1,4-dichloro- 2C2 H5OH C2H5OC2H5 H2O
433 443 K
2, 3-dibromobutene-2. (excess)
EBD_7207
218 HYDROCARBONS
56. (d) Alcoholic KOH is used for dehydrohalogenation e.g. H3C
| CH – CH = CH2

|
alc. H3C 3– methyl-1-butene
CH 3 CH 2 CH 2 CH 2 Cl
KOH
CH3 CH 2 CH CH 2 CH3 +
| CH – CH2 – CH2 (1°)

|
57. (b) Paraffins or alkanes are non-polar compounds. Hence + CH3
soluble in benzene. +H
CH3 +
58. (a) Peroxide effect is observed only in case of HBr. | CH – CH – CH3

|
CH3 (2°)
Therefore, addition of HCl to propene even in the
presence of benzyoyl peroxide occurs according to of the two possibilities 2° carbocation is more stable
Markovnikov’s rule : so the product of the reaction expected was
HCl predominantly one formed by 2° carbocation i.e.
CH3 CH CH 2
(C6 H5CO)2 O 2 CH3 – C – CH – CH3
| |
CH 3 CHCl CH 3 CH3 Br
59. (b) Markonikov’s way of addition : i.e. 2– Bromo-3-Methylbutane
CH 3 CH CH 2
A B However some electrophilic addition reaction form
CH3 CH CH 2
| | products that are clearly not the result of the addition
B A of electrophile to the sp2 carbon bonded to the most
60. (d) Since given alkene on ozonolysis gives 2 moles of hydrogens and the addition of a nucleophile to the
propanone hence alkene should have a double bond other sp2 carbon.
between two equivalent C atoms i.e. the formula should In the above cases the addition of HBr to 3-methyl-1-
be butene the two products formed are shown below.
H 3C CH3 CH3
C C i.e. |
H 3C CH3 CH3 – C – CH – CH = CH2 + HBr
H3C CH3 CH3 CH3
C C +O3 Zn/H 2O | |
H3C CH3 CH3 – CH – CH – CH3 + CH3 – C – CH2 – CH3
2, 3-dimethyl but-2-ene
| |
Br Br
2-Bromo-3-methylbutane 2-Bromo-2-methylbutane
O
In this case the major product formed is 2- Bromo-2-
2CH3 —C—CH3
Propanone methylbutane i.e. option (b) is correct answer.
(Note: The unexpected product results from a
H CH3 H O CH3 rearrangement of carbocation intermediate. Please note
O3
61. (a) CH3– CH2– C C CH3– CH 2– C– C that all carbocation do not rearrange.
CH3
CH3 O O 64. (c) Alkenes combine with hydrogen under pressure and
O O
in presence of a catalyst (Ni, Pt or Pd) and form alkanes.
CH3– C – CH 3+ CH3 – CH2 – CHO (–H2O)
2 H / Pd
62. (d) In presence of peroxide, HBr adds on alkenes in Butene - 1 Butane
anti–markovnikov’s way, thus 65. (a) Alkenes are unsaturated hydrocarbon having double
Peroxide
bond so generally gives addition reaction.
H3CCH CH 2 HBr H3CCH 2CH 2 Br 66. (c)
Pr opene n propyl bromide
67. (d) A doubly bonded carbon atom having an alkyl group
Kharasch observed that the addition of HBr to is oxidised to aldehyde which is further oxidised to
unsymmetrical alkene in the presence of organic carboxylic acid.
peroxides follows an opposite course to that suggested
by Markownikoff . This is termed anti-Markownikoff (i) KMnO 4 ,OH
CH3CH 2CH CH CH3
or peroxide effect. (ii) H
63. (b) We know that in case of an unsymmetrical alkene there
is the possibility of forming two products. In such CH3CHO CH3CH 2CHO
cases the formation of major product is decided on the
basis of Markownikoffs rule which is rationalized in CH3COOH CH3CH 2COOH
terms of stability of the intermediate carbocation. Also
remember that 3° carbocation is more stable than 2° 68. (b) Polythene is manufactured by heating ethylene to
carbocation and 2° carbocation is more stable than 1° 473K under a pressure of 1500 atmosphere and in the
carbocation. presence of a trace of oxygen.
HYDROCARBONS 219

473K,1500atm
(–CH 2 – CH 2 –)n CH3CH CHCH3
n(CH 2 CH 2 )
Trace of oxygen 2 butene

The polythene manufactured in this way is called low H O


O3 H
density of polythene. C C

69. (a) CH 2 = CH 2 + H - O - SO2 OH O O


H 2SO 4

Zn / H 2O 2CH3CHO
CH 3 CH 2 OSO 2 OH CH 3CH 2 OSO 2 OH
Ethyl hydrogen sulphate 74. (a) The addition of HBr takes place according to anti-
Addition of sulphuric acid takes place according to Markovnikoff’s rule in presence of peroxide for
Markownikoff’s rule. Alkanes do not absorb cold conc. unsymmetrical alkenes.
The addition of HBr to symmetrical alkenes is not
H 2SO4 .
affected by the presence or absence of peroxide.
70. (c) According to Markownikoff’s rule, "in case of addition
of an unsymmetrical reagent (H– X), the positive part 75. (d) CH 2 CH 2 KMnO4 OH
get attached to the C which is least substituted or
which bears larger number of hydrogen atoms." CH2 CH 2
| | + MnO 2 KOH
H H OH OH
R H | | Glycol
C=C +H X R C C H 76. (b) When unsymmetrical unsaturated hydrocarbon reacts
H H with unsymmetrical reagent, then negative part of
|
H reagents attacks that carbon which has less H-atom.
[Markownikoff's rule]
Markownikoff’s rule is based on the stability of
I
carbocations (3° > 2° > 1° > methyl). |
400 C
71. (c) As per Markovnikoff’s law, the positive part (e.g. H CH 3 CH CH 2 HI CH 3 CH CH 3
of HX) or the less negative part of the reagent adds Propene-2 2-Iodopropane
to that carbon atom of alkene which has more
number of hydrogen atoms (the rich gets richer). So CCl4
77. (a) CH 2 CH 2 Br2 CH 2 CH 2
(c) is the correct option as the two carbons | |
containing the double bond have one H atom each Br Br
i.e. symmetric. 1, 2-dibromo ethane

72. (d) Completing the sequence of given reactions,


HOCl aq. NaHCO3
O3
78. (b) CH 2 CH 2 CH 2 CH 2
CH 3 – CH CH CH 3 | |
Cl OH
O
CH 2OH
Zn / H 2O
|
CH3– CH CH – CH3 CH 2OH
Glycol
O O
79. (c) By adding bromine water to a solution, if the colour of
‘A’ bromine water decolourise then the compound is
(ozonide)
unsaturated. This is a confirmatory test for unsaturation.

2CH3CHO H 2 O ZnO Conc. H 2 SO4


80. (b) CH 3 CH CH 2 H 2O
Markonikoff's rule
'B '
Thus ‘B’ is CH3CHO CH 3 CH CH3
Hence (d) is correct answer. |
OH
73. (c) The given molecular formula suggests that the aldehyde Isopropyl alcohol
formed will be acetaldehyde hence the alkene will be
EBD_7207
220 HYDROCARBONS
81. (b) Greater the s-character of C, higher is its
95. (c) CaC2 + 2H2O HC CH + Ca (OH)2
electronegativity, shorter and stronger will be the bond Calcium Acetylene
formed by it. Thus C C is the strongest bond. carbide
82. (b) General formula for alkynes is CnH2n–2 96. (d) Among isomeric alkanes, the straight chain isomer
83. (c) When both double and triple bonds are present, then has higher boiling point than the branched chain
triple bond is considered as the principal group. isomer. The greater the branching of the chain, the
CH3 CH CH C CH lower is the boiling point. Further due to the presence
5 4 3 2 1 of electrons, these molecules are slightly polar and
84. (b) Three alkynes are possible for the formula C5H8. hence have higher boiling points than the
CH3CH2CH2C CH corrosponding alkanes.
97. (a) Terminal alkyenes give a white precipitate easily on
CH3
reaction with ammonical silver nitrate solution.
CH3 98. (b) dil H 2SO 4
CH CH H 2 O
HgSO 4 .60 C
CH3CH2C CCH3
85. (c) If both the double and triple bonds are present, the [CH 2 CHOH] CH3CHO
compound is regarded as derivative of alkyne. Further unstable
if double and triple bonds are at equidistance from 99. (b) HCl
CH CH HCl CH 2 CH Cl
either side, the preference is given to double bond.
86. (d) C2H2 and C3H4 are homologue because they differ by Cl
– CH2 group. Both have triple bond in their molecule.
CH3 – CH
Cl
87. (a) 1, 1-dichloroethane
88. (d) In CH CH triple bond consists of one and two 100. (b) Alkynes having terminal –C H react with Na in
bonds. liquid ammonia to yield H2 gas. CH3CH2C CH can
89. (c) In C2H6, C – C bond length is 1.54 Å. react with Na in liquid NH3 so the correct answer is (b).
90. (c) Acetylene molecule can be represented as, Na in
CH3CH 2 C CH
liquid NH3
H C C H
1
CH3CH 2C C – NaH2 (g )
So, it contains 3 and 2 bonds. 2
91. (d) We know that carbon having 101. (a) Acetylene reacts with the other three as:
(i) 4 bonds correspond to sp3
CH2
(ii) 3 and 1 bond correspond to sp2 CH CNa Na
CH CH +HCl
(iii) 2 and 2 bonds correspond to sp liq. NH3
CHCl
(1) (2) (3) (4) CH3
H3 C C C CH3 +HCl
(1) (2) (3) (4) CHCl CHCl2
No. of bonds : 4 2 2 4 [AgNO3+NH4OH]
– 2 2 – CH CH AgC CAg + NH4NO3
white ppt.
Hybridisation : sp3 sp sp sp3
Thus, butyne-2 has sp and sp3 hybridised carbon 102. (c) 103. (d)
atoms. 104. (c) 40% H 2SO4
CH3 C CH H 2O
1% HgSO4
92. (b) C C C C C C
1.20A 1.34A 1.54A
Rearrangement
93. (a) As the number of bonds between carbon atoms CH3 C CH 2 CH3 C CH3
increases, electronegativity of that carbon also |
OH O
increases due to increasing active power of electrons. Acetone
Also sp hybrid is more electronegative than sp2 which
is more electronegative than sp 3 ( s character CH3 CH CH3
decreases) Hence, option (a) is correct.
C C
94. (c) On heating ethylene chloride (1, 1 dichloro ethane)
105. (a) 3CH 3 C CH
with alcoholic potash followed by sodamide alkyne is
CH CH
obtained
alc.KOH C
R CH 2 CCl 2 R R CH CCl R
NaNH 2 CH3
R C C R Mesitylene or 1, 3, 5-trimethyl benzene
HYDROCARBONS 221

106. (b) CH CH H 2
Lindlar's
CH 2 CH 2 Y Y
Acetyle Catalyst
Pd. BaSO4 Ethylene +
X
124. (a)
Ni
107. (d) Addition – CH CH 3H 2 CH 3 CH 3 X
m-isomer
Substitution – (> 60%)
1 Y = –COOH because it is meta directing group while
CH CH Na CH C Na H2
2 –NH2, – OH and –Cl are o and p directing groups.
Polymerization – 125. (a) – NO2 is a meta-directing group. As it is also a
deactivating group so no chance of introduction of
hot Cu tube
3CH CH C6 H 6 second – Br atom.
Polymerization
Benzene 126. (c) Cl exhibits –I effect and +M effect.

Br + 3H2
Ni
|
108. (b) CH3C CH 2HBr CH3 C CH3 127. (b)
| Benzene Cyclohexane
Br 128. (d)
109. (b) CH3
110. (b) Thin film of polyacetylene can be used as electrodes
anhydrous AlCl3
in batteries. These films are good conductors, lighter 129. (c) + CH3Cl + HCl
and cheaper than the metal conductors.
111. (b) 130. (b) Benzene can be obtained by polymerisation of
112. (d) Amongst all tropolone is a non-benzenoid aromatic acetylene.
compound.
113. (c) Red hot tube
3HC CH
114. (c) Kekule in 1865 suggested a ring structure of benzene 500 C
in which the ring was composed of six carbon atoms, 131. (a) In electrophilic substitution reaction an electrophile
each of which carries one atom of hydrogen. To satisfy
(in this case NO2 ) replaces another atom (in this case
the fourth valency of the carbon atom, he suggested
three alternate double bonds. H) from the substrate (benzene).
H NO 2
CH
HC CH HNO3 H 2SO 4

HC CH Nitrobenzene
C 132. (a) This is an example of Friedel - Craft alkylation.
H 133. (d)

115. (a) O O
O
116. (c) Benzene do not show addition reactions like other O
unsaturated hydrocarbons. However it show 134. (c) + O3 O
O
substitution reactions. Due to resonance all the C – C O
bonds have the same nature, which is possible because O O
of the cyclic delocalisation of -electrons in benzene. Benzene triozonide
Monosubstitution will give only a single product. 135. (b) Friedel- Craft reaction occurs in presence of an
117. (a) In the benzene molecule all the six carbons are sp2 attacking reagent which is an electrophile (AlCl3).
hybridised as each C has one double bond.
118. (c) They have a relatively high percentage of carbon. STATEMENT TYPE QUESTIONS
119. (a) 120. (c) 121. (c)
122. (d) Benzene can be obtained by all the compounds given. 136. (a) According to experimental evidences, electrophilic
123. (a) This is an example of decarboxylation reaction. substitution reactions are supposed to proceed via the
following three steps:
COONa
(1) Generation of the electrophile
NaOH + CaO
(2) Formation of carbocation intermediate
+ Na2CO3 (3) Removal of proton from the carbocation intermediate
EBD_7207
222 HYDROCARBONS
137. (b) 149. (b) For statement (ii), alkynes on reduction with Pd/C form
H cis-alkenes. For statement (iv), Propyne on reduction
.. + -
.. - NO 2
HO3SO - H + H - O H-O
.. - NO2 + HSO 4 with Pd/C form propene. Propene does not show
geometrical isomerism. Only those compounds show
H geometrical isomerism which have two different atoms
+
H O
.. NO 2 H 2O NO 2 or groups attached to each carbon atom.

Protonated Nitronium ion CH3 H


nitric acid C C
In the process of generation of nitronium ion, sulphuric H H
Propene
acid serves as an acid and nitric acid as a base.
138. (a) -electrons of benzene rings are delocalised 150. (d) Polynuclear hydrocarbons are not produced in human
throughout the molecule. This makes the molecule very body by any biochemical reaction as when they enter
stable. The stability resists breaking of double bonds
into human body they undergo various biochemical
for addition.
reactions which finally damage DNA and cause cancer.
139. (b) Natural gas is found in upper strata during drilling of
oil wells. The gas after compression is known as
MATCHING TYPE QUESTIONS
Compressed Natural Gas (CNG).
140. (b) 151. (a) Among the infinite number of conformations in the
141. (c) In methane carbon atom is sp3 hybridized. staggered conformation hydrogen atoms are as far as
142. (b) Dihydrogen gas adds to alkenes and alkynes in the apart as possible. While in eclipse conformation
presence of finely divided catalysts like platinum, hydrogen atoms are perfectly eclipsed.
palladium or nickel to form alkanes. These metals
In skew conformation, hydrogen atoms are closer than
adsorb dihydrogen gas on their surfaces and activate
the hydrogen-hydrogen bond. Platinum and palladium in staggered but away than in eclipsed conformation.
catalyse the reaction at room temperature but relatively Pt/Pd/ Ni
higher temperature and pressure are required with 152. (a) CH 2 CH 2 H2 CH3 CH3
nickel catalyst. Ethene Ethane
143. (b) For statement (iii), Fluorination is too violent to be
controlled. For statement (iv), Iodination is very slow Zn, H
and a irreversible reaction. It can be carried out in the CH3 Cl H 2 CH 4 HCl
Chloromethane Methane
presence of oxidizing agents like HIO3 or HNO3

CH 4 I2 CH 3 I HI dry ether
CH3Br 2Na BrCH3 CH3 CH3 2NaBr
5 HI HIO3 3I 2 3H 2O Bromomethane Ethane
144. (c) Pyrolysis and cracking are same process.
145. (b) Rotation around a C-C single bond is not completely CaO
CH 3COO Na NaOH CH 4 Na 2 CO3
free. It is hindered by a small energy barrier of 1-20 kJ
mol-1 due to weak repulsive interaction between the 153. (c)
adjacent bonds. Such a type of repulsive interaction is
154. (a) Pd/C
called torsional strain. CH CH H 2 CH 2 CH 2
146. (b) Eclipsed form has maximum torsional strain while Ethyne Ethene
staggered form has the least.
H H H H
147. (b) For statement (ii), Alkenes are also known as olefins. alc.KOH
For statement (iv), Carbon–Carbon double bond in H C C H C =C
alkene consists of one sigma and one pi bond. H X H H
H H (X Cl, Br, I)
C C
H H
CH 2 Br CH 2 Br Zn CH 2 CH 2 ZnBr2
148. (a) For statement (i), cis form of alkenes have significant
dipole moment whereas dipole moment of trans form is
almost zero. For statement (iv), due to different H H
Conc.H 2SO 4
arrangements of atoms or groups in space cis and trans H C C H CH 2 CH 2 H2 O
isomers differ in their properties like melting point, H OH Ethene
boiling point, dipole moment, solubility etc. Ethanol
HYDROCARBONS 223

Cl 161. (b) The assertion that chlorination of CH4 does not take
place in dark is correct because it is a free radical
Anhy. AlCl3 reaction and free radicals are obtained in presence of
155. (b) + Cl2 + HCl
sun light.
Chlorobenzene 162. (b) Iodination is reversible since formed HI is a strong
reducing agent and reduces the alkyl iodide back to
CH2Cl
alkane. CH4 + I2 CH3I + HI
Anhy. AlCl3 Iodination can be done only in presence of strong
+ CH2Cl2 + H2O
oxidising agents like iodic acid which destroys the
Benzyl hydriodic acid.
chloride 163. (d) The two hydrogen atoms on first carbon and the two
Cl H-atoms on the third carbon atom lie in perpendicular
Cl Cl planes. The central carbon atom is sp-hybridized while
Anhy. AlCl3 terminal carbon atoms are sp2-hybridized
+ 6Cl2 + 6HCl
dark, cold 164. (c) (4n + 2) electrons and planar structure are the
Cl Cl essential conditions for aromaticity.
Cl
Hexachlorobenzene CRITICAL THINKING TYPE QUESTIONS
(C6Cl6)

Cl 165. (d) ; ;
Cl Cl
uv
+ 3Cl2
500 K |
Cl Cl In all these we find CH 2 group i.e. correct answer
|
Cl
Benzene hexachloride. is (d).
(BHC) 166. (c) The structure of neopentane is
AlCl3
156. (a) (A) C6H 6 Cl 2 C6H5Cl CH 3
(Benzene) (Chlorobenzene)
H3C — C— CH3
AlCl3
(B) C6 H6 CH3Cl C6H5CH3 HCl
CH3
(Toluene)
It has 1 quaternary and 4 primary carbons.
(C) C6 H6 CH3COCl C6H 5COCH3 HCl 167. (a)
methyl phenyl 168. (b) For the compounds with same molecular mass, boiling
ketone
point decreases with increase in branching. The boiling
KMnO4 point also increases steadily with increase in molecular
(D) C6 H5 CH3 C6H5COOH
NaOH mass. Hence 2-methyl propane will have lowest boiling
(Toulene)
point
157. (d) 158. (a) 159. (b)
CH3 —CH—CH3
ASSERTION-REASON TYPE QUESTIONS
CH3
2-methyl propane
160. (a) This reaction is followed by anti Markownikoff rule
H H H H 169. (d)
Peroxide CH3 CH3
H – C – C – C = C + HBr | |
170. (c) H3C – C – CH2Br + 2Na + Br – CH2– C – CH3
ether
H H H | |
–2NaBr
1 Butene CH3 CH3
H H H H
neo-pentyl bromide
H–C–C–C–C–H
H H H Br CH3 CH3
| |
1-Bromobutane H3C – C – CH 2 – CH2 – C – CH3
In this reaction anti Markownikoff’s addition is | |
CH3 CH3
explained on the basis of the fact that in the presence
2,2,5,5-tetramethylhexane
peroxides, addition takes place via free radical
mechanism rather than the ionic mechanism.
EBD_7207
224 HYDROCARBONS
171. (a) Electrolysis of a concentrated aqueous solution of CH3CH2CH2CH =CH2 CH3CH2CH=CHCH3
either sodium or potassium salts of saturated mon- 1-pentene (i) 2- pentene, (cis,- trans) (ii), (iii)
carboxylic acids yields higher alkane at anode. CH3 CH3
Electrolyt ic | |
2RCOOK 2RCOO 2K
Oxidation Anode Cathode CH 3 CH CH CH 2 CH 3 CH 2 C CH 2
3-methyl-1-butene, (iv) 2-methyl-1-butene, (v)
At anode 2RCOO 2RCOO 2e –
R — R 2CO 2 CH3
|
At Cathode 2K 2e – 2K CH 3 C CHCH 3
2K H2O 2KOH H2 2-methyl-2-butene, (vi)
(Kolbe's Method) 179. (b) Stability of an alkene depends upon the heat of
172. (a) Given hydrogenation of an alkene. The heat of hydration is
Cl 2 alc. / KOH O3 / H 2 O the standard enthalpy change in hydrogenation of on
A B C CH 2 O
alkene. The lower the heat of hydrogenation of an
Hydrocarbon alkene higher will be stability.
Since hydrocarbon C give only CH2O, on ozonolysis, Order of stability Heat of hydrogenation
C should be CH2 = CH2 hence going backward A (kJ/mol)
should be ethane. Thus the reactions are trans-2-butene – 115.5
CH 3CH 3
Cl 2 / h
CH 3CH 2Cl alc. cis-2-butene – 119.6 and
KOH ... 1-butene – 126.8 respectively.
(A) (B)
180. (c) In compounds
CH 2 CH 2 O3 / H 2 O HCHO
(C ) ( D) CH3 CH3
CH 3

173. (b) CH 3 — C— CH 3 first has more dipole moment than second.


Therefore its boiling point will be higher. Melting point
CH 3
2,2-dimethyl Propane depends on symmetry therefore I has higher melting
point than II. Steric crowding in I is more than II
All hydrogen atoms are equivalent in 2, therefore I is more stable than II.
2-dimethylpropan e. So it forms only one
181. (b)
monochlorinated product.
174. (c) In the eclipsed conformation of ethane, the dihedral CH3
H+/ Heat
angle between hydrogen atoms of adjacent methyl 182. (b) CH3 — CH — CH — CH3
groups is 0º.
OH
HH H
CH3 CH3
H
H CHH— CH — CH — CH H / Heat
CH3 — C = CH — CH3 + CH3 — CH — CH = CH2
H H H H H 2 methyl 3 methyl
H butene-2 butene-1 (20%)
(80%) (B)
175. (a) The bond is formed by the sideways overlapping of
two p-orbitals of the two carbon atoms. (A)
The molecular plane does not have any electron In this case dehydration is governed by Saytzeff’s
density as the p-orbitals are perpendicular to the plane rule according to which hydrogen is preferentially
containing the ethene molecule. The nodal plane in eliminated from the carbon atom with fewer number of
the -bond of ethene is located in the molecular plane. hydrogen atoms i.e., poor becomes poorer. Thus, 2-
176. (c) methyl butene-2 is the major product.
4 5 CH3
177. (a) Cl CH2CH3 HBr, dark
C=C CH3 — C = CH — CH3
2 3 in absence
CH3 I (A) of peroxide
1
Correct IUPAC name of above compound is trans-2-
chloro-3-iodo-2-pentene
178. (c) C5H10 has 1º degree of unsaturation since the isomers CH3
are acyclic, all of these are alkenes. For writing the (CH3)2 — CH — CH — CH3 + CH3 — C — CH2 – CH3
isomers, first introduce the double bond at different
possible positions, and then consider the possibility Br Br
of branching in the alkyl group. (Minor) (Major)
HYDROCARBONS 225
This reaction is governed by Markownikoff’s rule
according to which when an unsymmetrical reagent OH | |
C C KMnO 4 C C + MnO 2 KOH
e.g. HBr adds to an unsymmetrical alkene, then the | |
OH OH
negative part of the reagent is added to that carbon 1,2 diols
atom of the double bond which bears the least number
of hydrogen atom. Thus, in above case. 2-methyl 2- This test is known as Baeyer test.
bromo butane will be the major product. 188. (b) CH 2 CH 2 Cl HCl CH 3CHCl 2
183. (a) When 3, 3 dimethyl 2-butanol is heated with H2SO4 Addition of HCl takes place according to
the major product obtained is 2, 3 dimethyl 2-butene. Markownikoff’s rule. (H goes to C which is least
substituted)
CH3
|
Conc.H 2SO 4 CH 2 CH3 H 2O
CH3 CH C CH3 189. (a) H 2SO 4 |
| | CH 2 CH 2 H 2SO4
OH CH3
3,3 dimethyl 2-butanol CH3
| H 2SO 4
CH3 CH3 CH 2OH
+
CH3 – CH – C – CH3 CH3 – CH – C – CH3 Except ethyl alcohol, no other primary alcohol can be
OH2 CH3 prepared by this method as the addition of H2SO4
+
CH3
follows Markownikoff’s rule. Generally secondary and
CH3 CH3 tertiary alcohols are obtained.
+
CH3 – CH – C – CH3 190. (c) Electrophilic addition reactions are shown by alkenes
CH3 – C = C – CH3
or alkynes, in these reactions attacking species is
H CH3 CH3 electrophile
2, 3 dimethyl -2-butene Slow
CH 3 CH CH 2 H CH 3 CH CH 3
184. (d) CH3 CH3 2° carbonium ion
C=O+O=C
CH3 C2 H5 CH 3 CH CH3 Br
Fast
CH 3 CH CH 3
|
Br
CH3 CH3 2-Bromopropane
C=C 191. (c)
CH3 CH2 CH3
192. (d) Nature of halogen atom and the alkyl group both
2,3-Dimethyl-2-pentene
determine rate of reaction.
185. (d) Addition of hydrogen halide to alkene is an example of
193. (a) (i) HC C CH 2 CH 2 CH 2 CH3
electrophilic addition involving carbocations as
intermediates. Hex 1 yne

HBr (ii) CH3 C C CH 2 CH 2 CH3


CH3 CH CH 2 CH3 —CH—CH3
in absence of Hex 2 yne
peroxide
Br
(Markownikoff addition)
(iii) CH3 CH 2 C C CH 2 CH3
186. (a) Addition reaction occurs on a double bond.
Hex 3 yne
The compound containing double bonds are also
undergo polymerisation. So, the correct option are both (iv) HC C CH CH 2 CH3
(a) and (d) e.g. |
CH3
CH 2 CH 2 HOCl CH 2OH CH 2 Cl 3- Methylpent -1-yne
(addition reaction)
(v) HC C CH 2 CH CH3
Polymerisation
CH 2 CH 2 n
|
nCH 2 CH 2 CH3
Polyethene
4- Methylpent -1-yne
187. (b) Alkenes react with cold alkaline KMnO4 solution to
form glycols. During this reaction KMnO4 (violet) gets (vi) CH3 C C CH CH3
|
converted to MnO2 (brown ppt.) and therefore the CH3
colour of KMnO4 is disappeared.
4- Methylpent -2-yne
EBD_7207
226 HYDROCARBONS

CH3 201. (c) Due to the maximum percentage of s character (50%),


| the sp hybridised orbitals of carbon atoms in ethyne
(vii) HC C C CH 3 molecules have highest electronegativity; hence, these
| attract the shared electron pair of the C-H bond of
CH3 ethyne to a greater extent than that of the sp 2
3,3- Dimethylbut-1-yne hybridised orbitals of carbon in ethene and the sp3
194. (a) Ethylene has restricted rotation, acetylene has no hybridised orbital of carbon in ethane. Thus in ethyne,
rotation, hexachloroethane has more rotation than hydrogen atoms can be liberated as protons more easily
ethylene but less than ethane because of greater size as compared to ethene and ethane.
of the substituent (chlorine) in hexachloroethane than 202. (c) In arenes, electrons are delocalised, hence arenes do
in ethane (substituent is hydrogen). not undergo addition reactions easily. Aromatic
195. (a) The acidity of acetylene or 1–alkyne can be explained compounds (Arenes) are highly stable and show
on the basis of molecular orbital concept according to resonance. eg. Benzene is the simplest example.
which formation of C—H bond in acetylene involves 203. (c) Compounds having same molecular formula but
sp-hybridised carbon atom. Now since s electrons are different functional groups in their molecules are called
closer to the nucleus than p electrons, the electrons functional isomers.
present in a bond having more s character will be 204. (d) In benzene due to delocalisation of - electrons, all
correspondingly more closer to the nucleus. the C – C bond lengths are equal as each C– C bond
Thus owing to high s character of the C—H bond in has some double bond character and thus the bond
alkynes (s = 50%), the electrons constituting this bond length is between single and double bond, i.e., between
are more strongly held by the carbon nucleus i.e., the C2H6 and C2H4.
acetylenic carbon atom or the sp orbital acts as more 205. (a)
electronegative species than the sp2 and sp3 with the 206. (c) Benzene has 12 and 3 bonds.
result the hydrogen present on such a carbon atom Ratio of bonds to bonds = 12/3 = 4
( C—H) can be easily removed as a proton. 207. (d) Presence of 6p orbitals, each containing one
196. (b)
unpaired electron, in a six membered cyclic structure
197. (d) Br2 in CCl4 (a), Br2 in CH3 COOH (b) and alk. KMnO4
is in accordance with Huckel rule of aromaticity.
(c) will react with all unsaturated compounds, i.e., 1, 3
and 4 while ammonical AgNO3 (d) reacts only with 208. (a)
terminal alkynes, i.e., 3 and hence 3 can be distinguished Cl
from 1, 2 and 4 by. ammonical AgNO3 (d).
198. (c) This reaction occurs according to Markownikoff’s rule + HCl
Subtitution, Cl2, anly AlCl3
which states that when an unsymmetrical alkene
undergo hydrohalogenation, the negative part goes Cl
Addition, 3Cl2, sunlight Cl
to that C-atom which contain lesser no. of H-atom. Cl
Benzene
CH3 CH 2 C CH HCl
Cl Cl
CH3 CH 2 C CH 2 Cl
|
Cl 209. (b) –Cl group is o-, p-directing due to +R effect ; however
it is deactivating due to strong –I effect of Cl (difference
I
| from other o-, p-directing groups which are activating).
HI
CH3 CH 2 C CH3 The net result is that chlorobenzene undergoes o, p-
| substitution, but with difficulty
Cl
210. (b) C6 H5COOH NaOH C6 H 5COONa H 2O
199. (a) Only terminal alkynes show acidic nature. Ethyne is (X)
more acidic than propyne. But-2-yne is not acidic as it
distill
does not contain any hydrogen attached to sp C6 H5OH Zn C6 H 6 ZnO
(Y)
hybridised carbon.
200. (a) 211. (a) Huckel rule is not obeyed. It has only four electrons.
Further it does not have continous conjugation.
14
ENVIRONMENTAL CHEMISTRY

FACT / DEFINITION TYPE QUESTIONS 9. The quantity of CO2 in atmosphere is


(a) 3.34% (d) 6.5%
1. The type of pollution caused by spraying of DDT is (c) 0.034% (d) 0.34%
(a) air and soil (b) air and water
10. The substance which is not regarded as a pollutant?
(c) air (d) air, water and soil
(a) NO2 (b) CO2
2. What is DDT among the following ?
(c) O3 (d) Hydrocarbons
(a) Greenhouse gas
11. Which of the following is/are the hazardous pollutant(s)
(b) A fertilizer
present in automobile exhaust gases?
(c) Biodegradable pollutant
(i) N2 (ii) CO
(d) Non-biodegradable pollutant
(iii) CH4 (iv) Oxides of nitrogen
3. The uppermost region of the atmosphere is called
(a) (ii) and (iii) (b) (i) and (ii)
(a) Ionosphere (b) Stratosphere
(c) (ii) and (iv) (d) (i) and (iii)
(c) Troposphere (d) Exosphere
12. The gas emitted by supersonic jet planes that slowly
4. Which of the following is the coldest region of atmosphere ?
depletes the concentration of ozone layer is
(a) Thermosphere (b) Mesosphere
(a) CO (b) NO
(c) Troposphere (d) Stratosphere
5. The region which is greatly affected by air pollution is (c) SO2 (d) O2
(a) Thermosphere (b) Stratosphere 13. Carbon monoxide (CO) is harmful to man because
(c) Troposphere (d) Mesosphere (a) it forms carbolic acid
6. The region containing water vapour is (b) it generates excess CO2
(a) thermosphere (b) stratosphere (c) it is carcinogenic
(c) troposphere (d) mesosphere (d) it competes with O2 for haemoglobin
7. High concentration of which of the following in atmosphere 14. Increase in global temperature increases the incidence of
leads to stiffness of flower buds which eventually fall off which of the following infectious disease(s)
from plants? (i) Sleeping sickness (ii) Yellow fever
(a) NO2 (b) SO2 (iii) Malaria (iv) Dengue
(c) CFC (d) Smog (a) (ii) only (b) (i) and (ii)
8. The irritant red haze in the traffic and congested places (c) (iii) and (iv) (d) (i), (ii), (iii) and (iv)
is due to presence of which of the following ? 15. The green house effect is caused by
(i) Oxides of sulphur (a) CO2 (b) NO2
(ii) Oxides of nitrogen (c) NO (d) CO
(iii) Carbon dioxide 16. Which is related to ‘Green House Effect’?
(iv) Mists, smoke and dust
(a) Farming of Green plants
(v) Smog
(b) Farming of Vegetables in Houses
(a) (i), (iv) and (v) (b) (iii) only
(c) Global Warming
(c) (ii) only (d) (ii) and (v)
(d) Biodegradable pollutant
EBD_7207
228 ENVIRONMENTAL CHEMISTRY
17. Green house gases 25. Acid rain is due to
(a) allow shorter wavelength to enter earth's atmosphere (a) CH3 (b) N2O5
while doesn't allow longer wavelength to leave the (c) SO2 and NO2 (d) C2H5OH
earth's atmosphere. 26. The pH of normal rain water is
(b) allow longer wavelength to enter earth atmosphere (a) 6.5 (b) 7.5
while doesn't allow shorter wavelength to leave the (c) 5.6 (d) 3.5
surface 27. Which of the following statements is incorrect ?
(c) don't have wavelength specific character. (a) Smoke particulates consist of solid or mixture of
(d) show wavelength specific behaviour near the earth solid and liquid particles formed during combustion
while far from earth these have wavelength indepen- of organic matter.
dent behaviour. (b) Herbicides and insecticides that miss their target and
18. Today the concentration of green house gases is very high travel through air form mists.
because of (c) Organic solvents, metals and metallic oxides form
(a) use of refrigerator fume particles
(b) increased combustion of oils and coal (d) None of these
(c) deforestation 28. Which of the following green house gas is released in paddy
(d) All of the above field ?
19. The greenhouse effect is because of the I. CFCs II. CH4
(a) presence of gases, which in general are strong infrared III. SO2
absorbers, in the atmosphere (a) Only I (b) Only II
(b) presence of CO2 only in the atmosphere (c) Only III (d) I and II
(c) pressure of O3 and CH4 in the atmosphere 29. Photochemical smog is due to the presence of
(d) N2O and chlorofluorohydrocarbons in the atmosphere (a) oxides of sulphur (b) oxides of nitrogen
20. The greenhouse gas is (c) oxides of carbon (d) lead
(a) CO2 (b) SO2 30. The secondary precursors of photochemical smog are
(c) N2 (d) H2S (a) SO2 and NO2
21. Which of the following gases is not a green house gas? (b) SO2 and hydrocarbons
(a) CO (b) O3 (c) NO2 and hydrocarbons
(c) CH4 (d) H2O vapour (d) O3 and PAN
22. Which of the following strategy is not a correct approach 31. The main element of smog is
to reduce global warming ? (a) O3 and PAN (b) O3
(a) Reducing the green house gas emission by limiting (c) PAN (d) PPN and PBN
the use of fossil fuels 32. Classical smog occurs in places of
(b) Increase the vegetation cover particularly the forest (a) excess SO2 (b) low temperature
for photosynthetic utilization of CO2 (c) high temperature (d) excess NH3
(c) Minimizing the use of nitrogen fertilizers in agriculture 33. The smog is essentially caused by the presence of
for reducing N2O emission (a) Oxides of sulphur and nitrogen
(d) Increasing the use of air conditioners, refrigeration (b) O2 and N2
unit and production of plastic foams and propellants (c) O2 and O3
in aerosal spray cans (d) O3 and N2
23. The substance having the largest concentration in acid 34. Air pollution causing photochemical oxidants production
rain include
(a) H2CO3 (b) HNO3 (a) Carbon monoxide, sulphur dioxide
(c) HCl (d) H2SO4 (b) Nitrous oxide, nitric acid fumes, nitric oxide
24. When rain is accompanied by a thunderstorm, the collected (c) Ozone, peroxyacetyl nitrate, aldehydes
rain water will have a pH value (d) Oxygen, chlorine, fuming nitric acid
(a) slightly lower than that of rain water without 35. Photochemical smog formed in congested metropolitan cities
thunderstorm mainly consists of
(b) slightly higher than that when the thunderstorm is not (a) ozone, peroxyacetyl nitrate and NOx
there (b) smoke, peroxyacetyl nitrate and SO2
(c) uninfluenced by occurrence of thunderstorm (c) hydrocarbons, SO2 and CO2
(d) which depends upon the amount of dust in air (d) hydrocarbons, ozone and SOx
ENVIRONMENTAL CHEMISTRY 229
36. In almost all Indian metropolitan cities like Delhi, the major 46. The aromatic compounds present as particulates are
atmospheric pollutant(s) is/are (a) Polycyclic aromatic hydrocarbons
(a) suspended particulate matter (SPM) (b) Benzene
(b) oxides of sulphur (c) Toluene
(c) carbon dioxide and carbon monoxide (d) Nitrobenzene
(d) oxides of nitrogen 47. Which of the following can control the photochemical
37. The non-viable particulate among the following is smog ?
(a) Dust (b) Bacteria (A) Use of catalytic converters in automobiles.
(c) Moulds (d) Fungi (B) Plantation of trees like pinus, pyrus vitis etc.
38. Photochemical smog occurs in warm, dry and sunny climate. (C) Using less sulphur containing fossil fuels.
One of the following is not amongst the components of (a) A and C (b) B
photochemical smog, identify it.
(c) A and B (d) A, B and C
(a) NO2
48. The gas responsible for ozone depletion :
(b) O3
(a) NO and freons (b) SO2
(c) SO2
(c) CO2 (d) CO
(d) Unsaturated hydrocarbon
49. Identify the incorrect statement from the following :
39. The pollutants which came directly in the air from sources
are called primary pollutants. Primary pollutants are (a) Ozone absorbs the intense ultraviolet radiation of the
sometimes converted into secondary pollutants. Which of sun.
the following belongs to secondary air pollutants? (b) Depletion of ozone layer is because of its chemical
(a) C O (b) Hydrocarbon reactions with chlorofluoro alkanes.
(c) Peroxyacetyl nitrate (d) NO (c) Ozone absorbs infrared radiation.
40. The main element of smog is (d) Oxides of nitrogen in the atmosphere can cause the
(a) O3 and PAN (b) O3 depletion of ozone layer.
(c) PAN (d) Both (a) and (b) 50. Identify the wrong statement in the following:
41. Which of the following statements is not true about classical (a) Chlorofluorocarbons are responsible for ozone layer
smog? depletion.
(a) Its main components are produced by the action of (b) Greenhouse effect is responsible for global warming.
sunlight on emissions of automobiles and factories. (c) Acid rain is mostly because of oxides of nitrogen and
(b) Produced in cold and humid climate. sulphur.
(c) It contains compounds of reducing nature. (d) Ozone layer does not permit infrared radiation from
(d) It contains smoke fog and sulphur dioxide the sun to reach the earth.
42. Which of the following statements about photochemical 51. Which of the following chemical, harmful to ozone, is
smog is wrong? released by chlorofluoro carbon?
(a) It has high concentration of oxidising agents (a) Sulphur dioxide (b) Fluorine
(b) It has low concentration of oxidising agent (c) Chlorine (d) Nitrogen dioxide
(c) It can be controlled by controlling the release of NO2, 52. In Antarctica ozone depletion is due to the formation of
hydrocarbons ozone, etc. following compound
(d) Plantation of some plants like pinus helps in controlling (a) acrolein (b) peroxyacetyl nitrate
photochemical smog. (c) SO2 and SO3 (d) chlorine nitrate
43. Select the process that does not add particulate materials to 53. Depletion of ozone layer causes
air. (a) breast cancer (b) blood cancer
(a) Use of air conditioner (c) lung cancer (d) skin cancer
(b) Burning of fosssil fuels 54. Select the one that has an adverse effect on ozone layer.
(c) Paper industry (a) Carbon dioxide (b) Chlorofluorocarbons
(d) Incomplete combustion of coal (c) Soil (d) Dust particles
44. The biggest particulate matter is 55. Ozone hole refers to
(a) HNO3 droplets (b) Soot (a) Increase in concentration of ozone
(c) H2SO4 droplets (d) Fly ash (b) Hole in ozone layer
45. The viable particulate among the following is
(c) Reduction in thickness of ozone layer in troposphere
(a) Fumes (b) Algae
(d) Reduction in thickness of ozone layer in stratsophere
(c) Smoke (d) Mist
EBD_7207
230 ENVIRONMENTAL CHEMISTRY
56. Which of the following statements is wrong? 65. Which of the following does not occur when the sewage is
(a) Ozone is not responsible for green house effect. discharged into water ?
(b) Ozone can oxidise sulphur dioxide present in the (a) Increase in O2
atmosphere to sulphur trioxide. (b) Cyanophycean blooms occur
(c) Ozone hole is thinning of ozone layer present in (c) Depletion of O2 layers
stratosphere. (d) Eutrophication
(d) Ozone is produced in upper stratosphere by the action 66. Which of the following metal is a water pollutant and causes
of UV rays on oxygen. sterility in human being
57. Which of the following statements is correct? (a) As (b) Mn
(a) Ozone hole is a hole formed in stratosphere from which (c) Mg (d) Hg
ozone oozes out. 67. Sewage mostly constitutes
(b) Ozone hole is a hole formed in the troposphere from (a) Non-biodegradable pollutants
which ozone oozes out. (b) Biodegradable pollutants
(c) Ozone hole is thinning of ozone layer of stratosphere (c) Effluents
at some places. (d) Air pollutants
(d) Ozone hole means vanishing of ozone layer around 68. Sewage containing organic waste should not be disposed
the earth completely. in water bodies because it causes major water pollution.
58. Ozone is an important constituent of stratosphere because Fishes in such a polluted water die because of
it (a) large number of mosquitoes
(a) Destroys bacteria which are harmful to human life (b) increase in the amount of dissolved oxygen
(b) Prevents the formation of smog over large cities (c) decrease in the amount of dissolved oxygen in water
(c) Removes poisonous gases of the atmosphere by (d) clogging of gills by mud
reacting with them 69. Sewage water is purified by
(d) Absorbs ultraviolet radiation which is harmful to (a) aquatic plants (b) microoganisms
human life (c) light (d) fishes
59. The gas(es) not responsible for ozone depletion : 70. Water is often treated with chlorine to
(a) NO and freons (b) SO2 (a) remove hardness
(c) CO2 (d) Both (b) and (c) (b) increase oxygen content
60. What is the concentration of dissolved oxygen in cold (c) kill germs
water ? (d) remove suspended particles
(a) 5 ppm (b) 10 ppm 71. Which causes death of fish in water bodies polluted by
(c) 200, 000 ppm (d) 100 ppm sewage?
61. Water pollution is caused by (a) Foul smell (b) Pathogens
(c) Herbicides (d) Decrease in D.O.
(a) pesticides (b) SO2
72. B.O.D. test or biochemical oxygen demand test is made for
(c) O2 (d) CO2
measuring
62. Minamata disease of Japan is due to pollution of
(a) air pollution (b) water pollution
(a) Aresenic (b) Lead
(c) noise pollution (d) soil pollution
(c) Cynide (d) Mercury
73. Brewery and sugar factory waste alters the quality of a
63. The high amount of E. coli in water is the indicator of water body by increasing
(a) hardness of water (a) temperature (b) turbidity
(b) industrial pollution (c) pH (d) COD and BOD
(c) sewage pollution 74. Which one of the following statement is not true ?
(d) presence of chlorine in water (a) pH of drinking water should be between 5.5 – 9.5.
64. A lake with an inflow of domestic sewage rich in organic (b) Concentration of DO below 6 ppm is good for the
waste may result in growth of fish.
(a) drying of the lake very soon due to algal bloom (c) Clean water would have a BOD value of less than 5
(b) an increase production of fish due to lot of nutrients ppm.
(c) death of fish due to lack of oxygen (d) Oxides of sulphur, nitrogen and carbon are the most
(d) increased population of aquatic food web organisms widespread air pollutant.
ENVIRONMENTAL CHEMISTRY 231

75. Limit of BOD prescribed by Central pollution Control Board 86. The quantity of DDT in food chain
for the discharge of industrial and municipal waste waters (a) decreases (b) remains same
into natural surface waters, is (c) increases (d) changes
(a) < 100 ppm (b) < 30 ppm 87. The effect of polluted water on soil is, that
(c) < 3.0 ppm (d) < 10 ppm (a) it decreases fertility
76. Biochemical Oxygen Demand, (BOD) is a measure of organic (b) it contaminates ground water
material present in water. BOD value less than 5 ppm
(c) it renders soil acidic or basic
indicates a water sample to be __________.
(d) all of the above
(a) rich in dissolved oxygen
88. Soil is polluted by
(b) poor in dissolved oxygen
I. pesticides
(c) highly polluted
II. synthetic fertilizers
(d) not suitable for aquatic life
III. green manure
77. Phosphate fertilizers when added to water leads to
(a) Increased growth of decomposers Choose the correct option.
(b) Reduced algal growth (a) I and III (b) I and II
(c) Increased algal growth (c) II and III (d) I, II and III
(d) Nutrient enrichment (eutrophication) 89. Which of the following trophic level has least concentration
78. BOD of pond is connected with of toxins deposition ?
(a) Microbes & organic matter (a) Aquatic plant (b) Small fish
(b) Organic matter (c) Human being (d) Largest fish
(c) Microbes 90. Green chemistry means such reactions which :
(d) None of these (a) produce colour during reactions
79. The maximum prescribed concentration of cadmium in (b) reduce the use and production of hazardous chemicals
drinking water in ppm is (c) are related to the depletion of ozone layer
(a) 0.05 (b) 3 (d) study the reactions in plants
(c) 2 (d) 0.005 91. Which of the following practices will not come under green
80. Excess nitrate in drinking water can cause chemistry?
(a) methemoglobinemia (b) kidney damage (a) If possible, making use of soap made of vegetable oils
(c) liver damage (d) laxative effect instead of using synthetic detergents.
81. Eutrophication causes reduction in (b) Using H2O2 for bleaching purpose instead of using
(a) dissolved oxygen (b) nutrients chlorine based bleaching agents.
(c) dissolved salts (d) All of the above (c) Using bicycle for travelling small distances instead of
82. Water pollution is caused by using petrol/ diesel based vehicles.
(a) pesticides (b) fly ash (d) Using plastic cans for neatly storing substances.
(c) auto exhausts (d) aeroplanes 92. “Reducing potentially hazardous waste through smarter
83. Which causes death of fishes in water bodies polluted by production”.
sewage? This represents a great step forward for
(a) Foul smell (b) Pathogens (a) green revolution (b) green chemistry
(c) Clogging of gills by silt (d) Decrease in D.O. (c) industrial revolution (d) green biotechnology
84. Chief source of soil and water pollution is 93. Use of which of the following solvent in dry cleaning will
(a) mining (b) agro industry result in less harm to ground water ?
(c) thermal power plant (d) All of the above (a) Cl2C = CCl2 (b) Liquid CO2
85. What is DDT among the following ? (c) H2O2 (d) None of these
(a) Greenhouse gas 94. Synthesis of ethanal commercially from which of the
(b) A fertilizer following reagent is the part of green chemistry ?
(c) Biodegradable pollutant (a) CH3 CH2OH (b) CH2 = CH2
(d) Non-biodegradable pollutant (c) HC CH (d) All of these
EBD_7207
232 ENVIRONMENTAL CHEMISTRY
(iii) Technology has now been developed to produce
STATEMENT TYPE QUESTIONS
electricity from the garbage.
95. Which of the following sequence of T and F is correct for (a) (ii) only (b) (ii) and (iii)
given statements. Here T stands for True statement and (c) (iii) only (d) All of these
F stands for False statement.
(i) Troposphere is the lowest region of atmosphere in MATCHING TYPE QUESTIONS
which the human beings along with other organisms
live. 100. Match the columns
(ii) Troposphere extends up to the height of 10 km from Column - I Column - II
sea level. (A) Concentration of (p) 6 ppm
(iii) Stratosphere lies above troposphere, between 10 dissolved oxygen in
and 20 km above sea level. cold water
(iv) Troposphere contains much little water vapour, (B) Concentration of (q) 17 ppm
dinitrogen, dioxygen and ozone dissolved oxygen
(v) Stratosphere contains ozone, and cloud formation below which growth
also takes place in this region. of fish gets inhibited
(a) TTTTT (b) TFTFF
(C) BOD value of clean (r) 5 ppm
(c) TTFFF (d) TFTFT
water
96. Which of the following statement(s) is / are correct ?
(D) BOD value of (s) 10 ppm
(i) Sulphuric acid, nitric acid as well as ammonium salts
polluted water.
are components of acid rain.
(ii) Formation of acid rain can be reduced by using less (a) A – (s), B – (s), C – (q), D – (p)
sulphur content fossil fuels for power plants and (b) A – (p), B – (q), C – (r), D – (s)
industries. (c) A – (s), B – (p), C – (r), D – (q)
(iii) Catalytic converters must be used in cars to reduce (d) A – (p), B – (s), C – (q), D – (r)
the harmful effect of exhaust. 101. Match the columns
(iv) Main component of catalytic converter is ceramic Column I Column II
honey comb coated with metals like – Au, Ag, Pt etc. (A) Acid rain (p) CHCl2 – CHF2
(a) (i), (ii) and (iii) (b) (ii) and (iii) (B) Photochemical smog (q) CO
(c) (ii), (iii) and (iv) (d) (i), (ii), (iii) and (iv) (C) Combination with (r) CO2
97. Which of the following statement(s) is/are correct ? haemoglobin
(i) Classical smog is a mixture of smoke, fog and (D) Depletion of ozone (s) SO2
sulphur dioxide. layer
(ii) Classical smog is also called oxidising smog (t) Unsaturated
(iii) Hydrocarbons, NO2 and PAN are components of hydrocarbons
photochemical smog. (a) A – (r, s), B – (t, s), C – (q), D – (p)
(a) (i) and (iii) (b) (i) and (ii) (b) A – (r), B – (s), C – (q), D – (p)
(c) (iii) only (d) (i), (ii) and (iii)
(c) A – (t,s), B – (s), C – (q), D – (r)
98. Which of the following statements are not correct?
(d) A – (r), B – (t), C – (q), D – (p)
(i) F– ion concentration above 2ppm causes brown
102. Match the columns
mottling in teeth.
Column-I Column-II
(ii) Excessive F– (over 10 ppm) causes harmful effect to
bones and teeth. (A) Oxides of sulphur (p) Global warming
(iii) Excessive lead in drinking water causes disease (B) Nitrogen dioxide (q) Damage to kidney
methemoglobinemia (C) Carbon dioxide (r) ‘Blue baby’ syndrome
(iv) Deficiency of sulphate in drinking water causes (D) Nitrate in drinking (s) Respiratory diseases
laxative effect. water
(a) (ii) and (iv) (b) (ii) and (iii) (E) Lead (t) Red haze in traffic and
(c) (ii), (iii) and (iv) (d) (iii) and (iv) congested areas
99. Which of the following statement(s) is/are true about (a) A – (t), B – (p), C – (r), D – (s), E – (q)
waste recycling ? (b) A – (s), B – (t), C – (p), D – (r), E – (q)
(i) Clothes can be made from recycled plastic waste. (c) A – (s), B – (q), C – (p), D – (t), E – (r)
(ii) Fuel that has high octane rating and contains no (d) A – (q), B – (s), C – (t), D – (r), E – (p)
lead can be obtained from plastic waste.
ENVIRONMENTAL CHEMISTRY 233
103. Match the columns (a) A – (p,s), B – (r), C – (p), D – (q)
Column-I Column-II (b) A – (p), B – (s), C – (r), D – (q)
(A) Nitrous oxide (p) Secondary pollutant (c) A – (s), B – (r), C – (q), D – (p)
from car exhausts (d) A – (p), B – (q), C – (s), D – (r)
(B) Chlorofluorocarbon (q) Combustion of fossil
(CFCs) fuels, wood, etc ASSERTION-REASON TYPE QUESTIONS
(C) Methane (r) Denitrification Directions : Each of these questions contain two statements,
(D) Ozone (O3) (s) Refrigerators, aerosol, Assertion and Reason. Each of these questions also has four
sprays alternative choices, only one of which is the correct answer. You
(E) Carbon dioxide (t) Cattle, rice fields, toilets. have to select one of the codes (a), (b), (c) and (d) given below.
(a) A – (r), B – (s), C – (t), D – (p), E – (q) (a) Assertion is correct, reason is correct; reason is a correct
(b) A – (t), B – (p), C – (r), D – (s), E – (q) explanation for assertion.
(c) A – (s), B – (t), C – (p), D – (q), E – (r) (b) Assertion is correct, reason is correct; reason is not a
(d) A – (p), B – (r), C – (s), D – (t), E – (q) correct explanation for assertion
104. Match the columns (c) Assertion is correct, reason is incorrect
Column-I Column-II (d) Assertion is incorrect, reason is correct.
(A) Releasing gases to the (p) Water pollution 106. Assertion : Uncatalysed oxidation of sulphur dioxide is a
atmosphere after slow process.
burning waste material Reason : Particulate matter in polluted air catalyses the
containing sulphur oxidation of sulphur dioxide.
(B) Using carbamates as (q) Photochemical smog, 107. Assertion : Dinitrogen and dioxygen do not react with
pesticides damage to plant life, each other at a normal temperature.
corrosion to building Reason : At high altitudes dinitrogen combines with
material, induce dioxygen to form oxides of nitrogen
breathing problems, 108. Assertion : CO2 causes green house effect.
water pollution Reason : Other gases do not show such effect.
(C) Using synthetic (r) Damaging ozone layer 109. Assertion : Green house effect was observed in houses
detergents for washing used to grow plants and these are made of green glass.
clothes Reason : Green house name has been given because glass
(D) Releasing gases (s) May cause nerve houses are made of green glass.
produced by diseases in human 110. Assertion : The pH of acid rain is less than 5.6.
automobiles and Reason : Carbon dioxide present in the atmosphere
factories in the dissolves in rain water and forms carbonic acid.
atmosphere.
111. Assertion : Photochemical smog is oxidising in nature.
(E) Using chlorofluoro- (t) Classical smog, acid rain,
Reason : Photochemical smog contains NO2 and O3, which
carbon compounds water pollution, induce
are formed during the sequence of reactions.
for cleaning computer breathing problems,
parts damage to buildings, 112. Assertion : Suspended particulate matter (SPM) is an
important pollutant released by diesel vehicles.
corrosion of metals.
(a) A – (t), B – (s), C – (p), D – q, E – (r) Reason : Catalytic converters greatly reduce pollution
caused by automobiles.
(b) A – (s), B – (t), C – (q), D – (p), E – (r)
113. Assertion : Carbon dioxide is one of the important
(c) A – (q), B – (t), C – (r), D – (p), E – (s)
greenhouse gases.
(d) A – (r), B – (s), C – (p), D – (q), E – (t)
Reason : It is largely produced by respiratory function of
105. Match the columns animals and plants.
Column I Column II 114. Assertion : Ozone is destroyed by solar radiation in upper
(A) Phosphate fertilisers (p) BOD level of water stratosphere.
in water increases Reason : Thinning of the ozone layer allows excessive UV
(B) Methane in air (q) Acid rain radiations to reach the surface of earth.
(C) Synthetic detergents (r) Global warming 115. Assertion : Excessive use of chlorinated synthetic
in water pesticides causes soil and water pollution.
(D) Nitrogen oxides in air (s) Eutrophication Reason : Such pesticides are non-biodegradables.
EBD_7207
234 ENVIRONMENTAL CHEMISTRY
116. Assertion : If BOD level of water in a reservoir is less than 126. Which pollutant is harmful for ‘Taj Mahal’?
5 ppm it is highly polluted. (a) Hydrogen (b) O2
Reason : High biological oxygen demand means low activity (c) SO2 (d) Chlorine
of bacteria in water. 127. The beauty of Taj Mahal is endangered due to
117. Assertion : Eutrophication shows increase in productivity (a) degradation of marble due to high temperature
in water. (b) discharge of industrial waste in Yamuna river
Reason : With increasing eutrophication, the diversity of (c) air pollutants released from oil refinery
the phytoplankton increases. (d) riparian erosion
118. Assertion : The F – ions make the enamel on teeth much 128. Acid rain is caused by or recent reports of acid rain in some
harder. industrial cities are due to the effect of atmospheric pollution
Reason : F– ions converts hydroxyapatite by
[3(Ca(PO4)2 Ca(OH)2] into fluorapatite [3(Ca3(PO4)2. CaF2]. (a) excessive release of CO2 by burning of fuels like wood
and charcoal, cutting of forests and increased animal
CRITICAL THINKING TYPE QUESTIONS population
(b) excessive release of NO2 and SO2 in atmosphere by
119. In which of the following regions hydrogen and helium are burning of fossil fuel
found ? (c) excessive release of NH3 by industrial plants and coal
(a) Stratosphere (b) Mesosphere gas
(c) Exosphere (d) Troposphere (d) excessive release of CO in atmosphere by incomplete
120. Which one of the following pairs is mismatched ? combustion of coke, charcoal and other carbonaceous
(a) Fossil fuel burning - release of CO2 fuel in paucity of oxygen.
129. Which of the following is the major cause of global
(b) Nuclear power - radioactive wastes
warming?
(c) Solar energy - Greenhouse effect
(a) re-radiation of U.V. rays by CO2 and H2O
(d) Biomass burning - release of CO2
(b) re-radiation of I.R. rays by CO2 and H2O
121. Which of the following acts as a sink for CO? (c) re-radiation of I.R. rays by O2 and N2
(a) Plants (d) re-radiation of U.V. rays by O2 and N2
(b) Haemoglobin 130. Formation of London smog takes place in
(c) Microorganisms present in the soil (a) winter during day time
(d) Oceans (b) summer during day time
122. How many time oxyhaemoglobin is less stable than (c) summer during morning time
carboxyhaemoglobin? (d) winter during morning time
(a) 50 (b) 200 131. The false statement among the followings :
(c) 500 (d) 300 (a) The average residence time of NO is one month
123. Dinitrogen and dioxygen are main constituents of air but (b) Limestone acts as a sink for SOx
these do not react with each other to form oxides of nitrogen (c) SOx can be removed from flue gases by passing
because __________. through a solution of citrate ions
(a) the reaction is endothermic and requires very high (d) Ammonia acts as a sink for NOx
temperature. 132. Which of the following statements about polar stratosphere
(b) the reaction can be initiated only in presence of a clouds (PSCs) is not correct?
catalyst. (a) PSCs do not react with chlorine nitrate and HCl
(c) oxides of nitrogen are unstable. (b) Type I clouds are formed at about –77ºC and contain
(d) N2 and O2 are unreactive solid HNO3 . 3H2O
(c) Type II clouds are formed at about –85ºC and contain
124. SO2 is one of the air pollutants. SO2
some ice
(a) is a lung irritant
(d) A tight whirlpool of wind called Polar Vortex is formed
(b) dissolves in water to form acid rain which surrounds Antarctica
(c) both (a) and (b) 133. Which of the following is/are formed when ozone reacts
(d) none of the above with the unburnt hydrocarbons in polluted air ?
125 The greatest affinity for haemoglobin is shown by which (i) Formaldehyde (ii) Acrolein
of the following : (iii) Peroxyacetyl nitrate (iv) Formic acid
(a) NO (b) CO (a) (i) and (iv) (b) (ii) only
(c) O2 (d) CO2 (c) (iii) only (d) (i), (ii) and (iii)
ENVIRONMENTAL CHEMISTRY 235

134. Thermal pollution affects mainly 138. Lichens do not like to grow in cities
(a) vegetation (b) aquatic creature (a) because of absence of the right type of algae and fungi
(c) rocks (d) air (b) because of lack of moisture
135. A dental disease characterised by mottling of teeth is due (c) because of SO2 pollution
to presence of a certain chemical element in drinking water. (d) because natural habitat is missing
Which is that element? 139. BOD of pond is connected with
(a) Boron (b) Chlorine (a) microbes & organic matter
(c) Fluorine (d) Mercury (b) organic matter
136. Frequent occurrence of water blooms in a lake indicates (c) microbes
(d) None of these
(a) nutrient deficiency
140. Which is known as ‘Third poison of environment’ and also
(b) oxygen deficiency
creates ‘Blue baby syndrome’
(c) excessive nutrient availability (a) Nitrate present in water
(d) absence of herbivores in the lake (b) Phosphate and detergents found in water
137. Which one of the following statements is correct ? (c) Cynide
(a) Extensive use of chemical fertilizers may lead to (d) Pesticides
eutrophication of nearby water bodies 141. Negative soil pollution is
(b) Both Azotobacter and Rhizobium fix atmospheric (a) reduction in soil productivity due to erosion and over
nitrogen in root nodules of plants use
(c) Cyanobacteria such as Anabaena and Nostoc are (b) reduction in soil productivity due to addition of
important mobilizers of phosphates and potassium for pesticides and industrial wastes
plant nutrition in soil (c) converting fertile land into barren land by dumping
(d) At present it is not possible to grow maize without ash, sludge and garbage
chemical fertilizers (d) None of the above
EBD_7207
236 ENVIRONMENTAL CHEMISTRY

FACT / DEFINITION TYPE QUESTIONS caused by human emissions of sulfur and nitrogen
compounds which react in the atmosphere to produce
1. (d) DDT causes air, water and soil pollution. acids. In recent years, many governments have
2. (d) DDT is a non-biodegradable pollutant. introduced laws to reduce these emissions.
3. (d) The uppermost region of atmosphere is exosphere. 26. (c) pH of normal rain water is 5.6 as CO2 present in
4. (b) The coldest region is mesosphere (temp. –27ºC to atmosphere combines with moisture to form H2CO3.
– 92ºC) 27. (d)
5. (c) Air pollution greatly affect the troposphere. 28. (b) Large amounts of CH4 are released in paddy fields,
6. (c) Troposphere contains water vapour. coal mines and by fossil fuels.
7. (b) High concentration of SO2 leads to stiffness of 29. (b) The oxidised hydrocarbons and ozone in presence of
flower buds. humidity cause photochemical smog.
8. (c) The irritant red haze in the traffic and congested Hydrocarbons + O2, NO2, NO, O, O3 Peroxides
places is due to presence of oxides of nitrogen.
30. (d) 31. (a) 32. (b)
9. (c)
33. (a) Smog is caused by oxides of sulphur and nitrogen.
10. (b) CO2 is generally not regarded as pollutant.
34. (c) 35. (a) 36. (a) 37. (a) 38. (c)
11. (c) CO and oxides of Nitrogen are poisnous gases present
39. (c) 40. (a) 41. (a) 42. (b) 43. (a)
in automobile exhaust gases.
44. (d) 45. (b)
12. (b) Nitric oxide (NO) which may be produced at the ground
level due to human activity or natural sources or is 46. (a) PAH (Poly Aromatic Hydrocarbon)
produced in large amounts in the exhaust gases by the 47. (c) Usually catalytic converters are used in the
engine of supersonic transport planes and introduced automobiles, which prevent the release of nitrogen
directly into the strateosphere. oxide and hydrocarbons to the atmosphere. Certain
plants e.g., Pinus, Juniparus. Quercus, Pyrus and
NO O3 NO 2 O2
Vitis can metabolise nitrogen oxide and therefore,
13. (d) CO is highly toxic and impairs respiration. CO combine their plantation could help in this matter.
with haemoglobin of blood and reduces its O2 carry
48. (a) NO and freons are responsible for ozone depletion.
capacity.
49. (c) The ozone layer, existing between 20 to 35 km above
14. (d)
the earth’s surface, shield the earth from the harmful
15. (a) CO2 causes green house effect.
U. V. radiations from the sun.
16. (c)
Depletion of ozone is caused by oxides of nitrogen
17. (a) Radiation coming from sun or outerspace have high
energy or short wavelength, which are allowed to enter N 2O + h NO + N
by green house gases. However, radiation emitted by reactive nitric oxide
earth is in infrared region, having long wavelength, NO + O 3 NO 2 + O2
are reflected back by the envelope of green house O3 + h O2 + O
gases.
18. (d) NO 2 + O NO + O 2
19. (a) Green house gases such as CO2, ozone, methane, the
2 O3 + h 3 O 2 (Net reaction)
chlorofluorocarbon compounds and water vapour form
a thick cover around the earth which prevents the IR The presence of oxides of nitrogen increase the
rays emitted by the earth to escape. It gradually leads decomposition of O3.
to increase in temperature of atmosphere. 50. (d) Ozone layer acts as a shield and does not allow
20. (a) CO2 is a green house gas. ultraviolet radiation from sun to reach earth. It does
21. (a) 22. (d) not prevent infra-red radiation from sun to reach earth,
23. (d) Acid rain contains H2SO4 > HNO3> HCl. thus option (d) is wrong statement and so it is the
24. (a) Normal rain water has pH 5.6. Thunderstorm results in correct answer.
the formation of NO and HNO3 which lowers the pH. h •
51. (c) CF2 Cl 2 CF2 Cl Cl
25. (c) Acid rain is rain or any other form of precipitation that
is unusually acidic. It has harmful effects on plants, • •
Cl O3 Cl O O2
aquatic animals, and infastructure. Acid rain is mostly
ENVIRONMENTAL CHEMISTRY 237
52. (a) In antarctica ozone depletion is due to formation of 91. (d)
acrolein. 92. (b) This represents a great step forward for green
53. (d) Depletion of ozone layer causes skin cancer. chemistry.
54. (b) They create holes in ozone layer. 93. (b) Replacement of earlier used tetra-chloroethene as
55. (d) Ozone hole is reduction in ozone layer in stratosphere. solvent for dry cleaning by liquid CO2 results in less
56. (a) 57. (c) harm to ground water.
58. (d) Ozone absorbs U.V. radiations harmful to human life. 94. (b) Ethanal is commerically prepared by one step
59. (d) oxidation of ethene in the presence of ionic catalyst
60. (b) In cold water, dissolved oxygen can reach a in aqueous medium with an yield of 90%.
concentration upto 10 ppm, whereas oxygen in air is Catalyst
about 200, 000 ppm. CH2 = CH2 + O2 Pb(II)|Cu(II)
CH3CHO
61. (a) Pesticides cause water pollution. (in water)
62. (d) Minamata is caused by Hg poisoning.
63. (c) 64. (c) 65. (a) 66. (b) STATEMENT TYPE QUESTIONS
67. (b) Domestic sewage constitute biodegradable pollutants.
68. (c) 95. (c) For statement (iii), Stratosphere lies above
69. (b) Sewage water is purified by micro-organisms. troposphere between 10 and 50 km above sea level
70. (c) Water is often treated with Cl2 to kill germs. cloud formation takes place in troposphere.
71. (d) Decrease in D.O causes death of fish. For statement (iv), Troposphere is a turbulent, dusty
72. (b) Strength of sewage or degree of water pollution is zone containing air, much water vapour and clouds.
measured in terms of BOD (Biochemical oxygen For statement (v), Stratosphere contains dinitrogen,
demand) value. dioxygen, ozone and little water vapour.
73. (d) 96. (d)
74. (b) The ideal value of D.O for growth of fishes is 8 mg/ . 97. (a) Classical smog is also called reducing smog.
98. (d) For statement (iii), Methemoglobinemia (blue baby
7mg / is desirable range, below this value fishes get
syndrome) is caused due to excess of nitrate in
susceptible to disease. A value of 2 mg/ or below is
drinking water.
lethal for fishes.
For statement (iv), Excessive sulphate (> 500 ppm) in
75. (b) Water pollution is mainly caused by industrial wastes,
drinking water causes laxative effect, otherwise at
sewage, insecticide, herbicides, etc.
moderate levels it is harmless.
76. (a)
99. (d) All the given statements are true for about waste
77. (d) Addition of phosphate fertilizers to water leads to
recycling.
nutrient enrichment (eutrophication).
78. (a) BOD of pond is connected with microbes and organic MATCHING TYPE QUESTIONS
matter.
79. (d) 100. (c) 101. (a) 102. (b)
80. (a) Excessive concentration of nitrate in drinking water is 103. (a) 104. (a) 105. (a)
harmful and can cause methemoglobinemia (blue baby
syndrome). ASSERTION-REASON TYPE QUESTIONS
81. (a) Eutrophication causes reduction in D.O
106. (a) The presence of particulate matter in polluted air
82. (a) Pesticides cause water pollution.
catalyses the oxidation of SO2 to SO3
83. (d) Decrease in D.O causes death of fish
107. (b) At high altitudes when lightening strikes dinitrogen
84. (d)
and dioxygen combine to form oxides of nitrogen.
85. (d) DDT is a non-biodegradable pollutant.
108. (c) Other gases like CFCs, ozone, water vapour and nitrous
86. (c) 87. (d)
oxide also show green house effect.
88. (b) Pesticides and synthetic fertilizers pollute the soil.
109. (c) 110. (b) 111. (a)
89. (a) Lower trophic level has lower toxins deposition than
112. (b) SPM (Suspended Particulate Matter) is defined as
higher trophic level.
particles floating in the air with a diameter below 10
90. (b) Green chemistry may be defined as the programme of
m. Studies have shown that high SPM concentrations
developing new chemical products and chemical
in the air can have a detrimental impact on respiratory
processes or making improvements in the already
organs. SPM is generated from natural sources (e.g.,
existing compounds and processes so as to make less
volcanoes or dust storms) and human activities
harmful to human health and environment. This means
(vehicles, incinerators and industrial plants).
the same as to reduce the use and production of
hazardous chemicals.
EBD_7207
238 ENVIRONMENTAL CHEMISTRY

SPM Other aerosols 122. (d) Carboxyhaemoglobin is 300 times more stable than
oxyhaemoglobin.
Less than 10 m Less than 100 m 123. (a) 124. (c)
Tend to float longer in Tend to settle fairly 125. (a) Haemoglobin has great affinity for NO.
air due to small size quickly due to comparative 126. (c)
heaviness 127. (c) The beauty of Taj Mahal is endangered due to air
pollutants like SO2 released from oil refinery.
Catalytic converters is a device designed to reduce
128. (b) When SO2 pollution in air is much higher. Sometimes,
the amount of emissions from automobiles. The current
SO2 mixes in the air with small particles of metals near
(so-called three-way) systems use a heated metal
the factories and gets oxidised into sulphur trioxide
catalyst to reduce the emissions of carbon monoxide
SO3. These gases are harmful and they react with water
(CO), hydrocarbons, and nitric oxide (NO), all of which
to form sulphuric acid (H2SO4) or sulphurous acid
contribute to the formation of photochemical smog. In
(H2SO3) and come down to earth with rain water, it is
an automobile’s exhaust system, a catalytic converter called acid rain or acid precipitation.
provides an environment for a chemical reaction where 129. (b)
unburned hydrocarbons completely combust. 130. (d) London smog is formed in morning during winter.
113. (b) 114. (d) 115. (a) 116. (c) 131. (a) The average residence time of NO is 4 days.
117. (b) Eutrophication is a natural process which literally 132. (a) PSCs react with chlorine nitrate and HCl to give HOCl
means well nourished or enriched. It is a natural state and Cl2.
in many lakes and ponds which have a rich supply of 133. (d) 3CH4 + 2O3 3CH2= O + 3H2O
nutrients. Eutrophication become excessive, however Formaldehyde
when abnormally high amount of nutrient from sewage, CH2 == CHCH == OCH3COONO2
fertilizers, animal wastage and detergent, enter streams ||
and lakes causes excessive growth or blooms of O
microorganisms. With increasing eutrophication, the Acrolein Peroxyacetyl nitrate (PAN)
diversity of the phytoplankton community of a lake 134. (b) Thermal pollution is caused by power plants. Power
increases and the lake finally becomes dominated by plant requires a larger quantity of water for cooling.
blue - green algae. The water after cooling is left in the water body. The
118. (a) The F– ions make the enamel on teeth much harder temperature of left water is generally very high and
by converting hydroxyapatite, [3(Ca 3 (PO4)] 2 . affects aquatic life.
Ca(OH)2]. the enamel on the surface of the teeth, 135. (c) The excess of fluorine in water causes fluorosis. The
into much harder fluorapatite, [3(Ca3(PO4)2. CaF2]. symptoms of fluorosis are mottling of teeth (yellowish
streaks) and abnormal bones liable to fracture etc. It is
CRITICAL THINKING TYPE QUESTIONS an example of endemic disease.
119. (c) H2, He and ionic oxygen are present in exosphere. 136. (b) 137. (a)
120. (c) Solar energy is not responsible for green house effect 138. (c) Because they are very sensitive to sulphur dioxide
instead it is a source of energy for the plants and and in cities the amount of SO2 is high so lichen do
not grow in cities.
animals.
139. (a) BOD of pond is connected with microbes and organic
121. (c) CO is converted into CO2 by microorganism present
matter.
in soil.
140. (b) 141. (a)
15
THE SOLID STATE

FACT / DEFINITION TYPE QUESTIONS 7. Which of the following statements about amorphous solids
is incorrect ?
1. Which of the following is not a characteristic property of (a) They melt over a range of temperature
solids? (b) They are anisotropic
(a) Intermolecular distances are short. (c) There is no orderly arrangement of particles
(b) Intermolecular forces are weak. (d) They are rigid and incompressible
(c) Constituent particles have fixed positions. 8. Which of the following is not a crystalline solid?
(d) Solids oscillate about their mean positions. (a) KCl (b) CsCl
2. Most crystals show good cleavage because their atoms, ions (c) Glass (d) Rhombic S
or molecules are 9. Which of the following is an amorphous solid ?
(a) weakly bonded together (a) Graphite (C) (b) Quartz glass (SiO2)
(b) strongly bonded together (c) Chrome alum (d) Silicon carbide (SiC)
(c) spherically symmetrical 10. Which of the following statement is not true about
amorphous solids ?
(d) arranged in planes
(a) On heating they may become crystalline at certain
3. “Crystalline solids are anisotropic in nature. What is the
temperature.
meaning of anisotropic in the given statement?
(b) They may become crystalline on keeping for long time.
(a) A regular pattern of arrangement of particles which
(c) Amorphous solids can be moulded by heating.
repeats itself periodically over the entire crystal.
(d) They are anisotropic in nature.
(b) Different values of some of physical properties are
11. The sharp melting point of crystalline solids is due to _____.
shown when measured along different directions in
(a) a regular arrangement of constituent particles observed
the same crystals.
over a short distance in the crystal lattice.
(c) An irregular arrangement of particles over the entire (b) a regular arrangement of constituent particles observed
crystal. over a long distance in the crystal lattice.
(d) Same values of some of physical properties are shown (c) same arrangement of constituent particles in different
when measured along different directions in the same directions.
crystals. (d) different arrangement of constituent particles in
4. A crystalline solid different directions.
(a) changes abruptly from solid to liquid when heated 12. Why some glass objects from ancient civilisations are
(b) has no definite melting point found to become milky in appearance?
(c) undergoes deformation of its geometry easily (a) Glass is a crystalline solid, milky appearance is due to
(d) has an irregular 3-dimensional arrangements its crystalline nature.
5. Which of the following is not a characteristic of a crystalline (b) Glass is amorphous but on heating it become
solid ? crystalline at some temperature.
(a) Definite and characteristic heat of fusion. (c) Because of reaction of glass with impurities present
(b) Isotropic nature. in the atmosphere.
(c) A regular periodically repeated pattern of arrangement (d) None of these.
of constituent particles in the entire crystal. 13. Which of the following amorphous solid is used as
(d) A true solid photovoltaic material for conversion of sunlight into
6. Which of the following is not a crystalline solid? electricity?
(a) KCl (b) CsCl (a) Quartz glass (b) Quartz
(c) Glass (d) Rhombic S (c) Silicon (d) Both (a) and (b)
EBD_7207
240 THE SOLID STATE
14. Solid CH4 is 28. Crystals can be classified into basic crystal units, equal to
(a) ionic solid (b) covalent solid (a) 7 (b) 4
(c) molecular solid (d) does not exist (c) 14 (d) 2
15. An example of a covalent crystalline solid is: 29. How many three dimensional crystal lattice are possible?
(a) Si (b) Al (a) 20 (b) 7
(c) NaF (d) Ar (c) 14 (d) 10
16. Among solids, the highest melting point is exhibited by 30. Which of the following represents monoclinic crystal
(a) Covalent solids (b) Ionic solids system?
(c) Pseudo solids (d) Molecular solids (a) a b c 90°
17. Which of the following exists as covalent crystals in the (b) a = b = c , = °
solid state ? (c) a b c , = 90° 90°
(a) Iodine (b) Silicon (d) a = b = c , = = 90°
(c) Sulphur (d) Phosphorus 31. In face-centred cubic lattice, a unit cell is shared equally
18. The major binding force of diamond, silicon and quartz is by how many unit cells
(a) electrostatic force (b) electrical attraction (a) 2 (b) 4
(c) covalent bond force (d) non-covalent bond force (c) 6 (d) 8
19. In graphite electrons are :
(a) localised on each carbon atom 32. For orthorhombic system axial ratios are a b c and the
(b) spread out between the sheets axial angles are
(c) localised on every third carbon atom (a) 90 (b) 90
(d) present in antibonding orbital.
(c) 90 , 90 (d) 90
20. Which one of the following forms a molecular solid when
solidified? 33. The unit cell dimensions of a cubic lattice (edges a, b, c and
(a) Silicon carbide (b) Calcium fluoride the angles between them, , and ) are
(c) Rock salt (d) Methane (a) a = b = c, = = = 900
21. Which of the following is a network solid ? (b) a = b c, = = = 900
(a) SO2 (solid) (b) I2 (c) a = b = c, = = 900, 900
(d) a b c, = = 90 , 0 900
(c) Diamond (d) H2O (Ice)
22. Which of the following solids is not an electrical conductor? 34. Tetragonal crystal system has the following unit cell
(a) Mg (s) (b) TiO (s) dimensions
(c) I2 (s) (d) H2O (s) (a) a b c, 90
23. Iodine molecules are held in the crystals lattice by ______.
(a) london forces (b) a b c, 90
(b) dipole-dipole interactions (c) a b c, 90
(c) covalent bonds
(d) coulombic forces (d) a b c, 90 120
24. Which of the following is not the characteristic of ionic 35. The number of atoms contained in a fcc unit cell of a
solids? monoatomic substance is
(a) Very low value of electrical conductivity in the molten (a) 1 (b) 2
state. (c) 4 (d) 6
(b) Brittle nature. 36. In face-centred cubic lattice, a unit cell is shared equally
(c) Very strong forces of interactions. by how many unit cells
(d) Anisotropic nature. (a) 2 (b) 4
25. Graphite is a good conductor of electricity due to the (c) 6 (d) 8
presence of ______. 37. The number of atoms per unit cell of bcc structure is
(a) lone pair of electrons (b) free valence electrons (a) 1 (b) 2
(c) cations (d) anions (c) 4 (d) 6
26. Graphite cannot be classified as ______. 38. When molten zinc is converted into solid state, it acquires
(a) conducting solid (b) network solid hcp structure. The number of nearest neighbours of Zn will
(c) covalent solid (d) ionic solid be
27. Which of the following cannot be regarded as molecular (a) 6 (b) 12
solid ? (c) 8 (d) 4
(i) SiC (Silicon carbide) (ii) AlN 39. Hexagonal close packed arrangement of ions is described
(iii) Diamond (iv) I2 as
(a) (i), (ii) and (iii) (b) (ii) and (iii) (a) ABC ABA (b) ABC ABC
(c) (iv) (d) (ii) and (iv) (c) ABABA (d) ABBAB
THE SOLID STATE 241
40. In which of the following crystals alternate tetrahedral voids 50. In which of the following arrangements octahedral voids
are occupied? are formed ?
(a) NaCl (b) ZnS (i) hcp (ii) bcc
(c) CaF2 (d) Na2O (iii) simple cubic (iv) fcc
41. Which of the following metal(s) show(s) hexagonal close (a) (i), (ii) (b) (i), (iv)
packed structure (hcp) and which show face centred (c) (iii) (d) (ii), (iv)
cubic (fcc) structure? 51. Which of the following is the correct increasing order of
hcp fcc packing efficiency for hcp, bcc and simple cubic lattice?
(a) Ag, Zn Mg, Cu (a) hcp < bcc < simple cubic
(b) bcc < hcp < simple cubic
(b) Mg, Zn Ag, Cu
(c) simple cubic < bcc < hcp
(c) Cu, Fe Al, Sn
(d) simple cubic < hcp < bcc
(d) Na, Li Zn, Cu
52. Total volume of atoms present in bcc unit cell is.
42. The number of octahedral voids present in a lattice is A .
The number of closed packed particles, the number of 16 3 4 3
(a) r (b) r
tetrahedral voids generated is B the number of closed 3 3
packed particles 8 3 12 3
(c) r (d) r
(a) A- equal, B- half (b) A- twice, B- equal 3 3
(c) A- twice , B- half (d) A- equal, B- twice 53. Total volume of atoms present in a face-centred cubic unit
43. In the hexagonal close packed structure of a metallic lattice, cell of a metal is (r is atomic radius)
the number of nearest neighbours of a metallic atom is 12 3 16 3
(a) twelve (b) four (a) r (b) r
3 3
(c) eight (d) six 24 3
20 3
44. The Ca2+ and F– are located in CaF2 crystal, respectively at (c) r (d) r
3 3
face centred cubic lattice points and in
54. The interionic distance for cesium chloride crystal will be
(a) tetrahedral voids (b) half of tetrahedral voids a
(c) octahedral voids (d) half of octahedral voids (a) a (b)
2
45. If Germanium crystallises in the same way as diamond, then
which of the following statement is not correct? 3a 2a
(c) (d)
(a) Every atom in the structure is tetrahedrally bonded to 2 3
4 atoms. 55. The fraction of total volume occupied by the atoms present
(b) Unit cell consists of 8 Ge atoms and co-ordination in a simple cube is
number is 4.
(a) (b)
(c) All the octahedral voids are occupied. 3 2 4 2
(d) All the octahedral voids and 50% tetrahedral voids
remain unoccupied. (c) (d)
4 6
46. The arrangement ABC ABC .......... is referred to as
56. Percentages of free space in cubic close packed structure
(a) Octahedral close packing
and in body centered packed structure are respectively
(b) Hexagonal close packing (a) 30% and 26% (b) 26% and 32%
(c) Tetrahedral close packing (c) 32% and 48% (d) 48% and 26%
(d) Cubic close packing 57. The empty space in the body centred cubic lattice is
47. The total number of tetrahedral voids in the face centred (a) 68% (b) 52.4%
unit cell is ______. (c) 47.6% (d) 32%
(a) 6 (b) 8 (e) 74%
(c) 10 (d) 12 58. Which one of the following statements about packing in
48. What is the coordination number in a square close packed solids is incorrect ?
structure in two dimensions ? (a) Coordination number in bcc mode of packing is 8.
(a) 2 (b) 3 (b) Coordination number in hcp mode of packing is 12.
(c) 4 (d) 6 (c) Void space in hcp mode of packing is 32%.
49. In the cubic close packing, the unit cell has ______. (d) Void space is ccp mode of packing is 26%.
(a) 4 tetrahedral voids each of which is shared by four 59. A metallic crystal crystallizes into a lattice containing a
adjacent unit cells. sequence of layers AB AB AB......Any packing of spheres
(b) 4 tetrahedral voids within the unit cell. leaves out voids in the lattice. What percentage of volume
(c) 8 tetrahedral voids each of the which is shared by four of this lattice is empty space?
adjacent unit cells. (a) 74% (b) 26%
(c) 50% (d) none of these.
(d) 8 tetrahedral voids within the unit cells.
EBD_7207
242 THE SOLID STATE
60. In NaCl, the centre-to-centre nearest-neighbour distance of 70. Which of the following statements is not correct ?
ions is (a) Vacancy defect results in decrease in density of
substance.
1 3
(a) a (b) a (b) Vacancy defect can develop when a substance is
4 2 heated.
1 1 (c) Interstitial defect increases the density of the
(c) a 2 (d) a substance.
2 2
61. The edge lengths of the unit cells in terms of the radius of (d) Ionic solids show interstitial defects only.
spheres constituting fcc, bcc and simple cubic unit cell are 71. Which defect is shown in the given figure?
respectively ________.
+ – + –
4r 4r Na Cl Na Cl
(a) 2 2r, , 2r (b) , 2 2r, 2r
3 3
– 2+ – +
4r 4r Cl Sr Cl Na
(c) 2r, 2 2r, (d) 2r, , 2 2r
3 3 – –
+
62. CsBr crystallises in a body centered cubic lattice. The unit Na Cl Cl
cell length is 436.6 pm. Given that the atomic mass of Cs =
133 and that of Br = 80 amu and Avogadro number being – + – +
Cl Na Cl Na
6.02 × 1023 mol–1, the density of CsBr is
(a) 0.425 g/cm3 (b) 8.5 g/cm3
(c) 4.25 g/cm 3 (d) 82.5 g/cm3 (a) Frenkel defect (b) Impurity defect
63. An element occuring in the bcc structure has 12.08 × 1023 unit (c) Schottky defect (d) Vacancy defect
cells. The total number of atoms of the element in these 72. Each of the following solids show, the Frenkel defect except
cells will be (a) ZnS (b) AgBr
(a) 24.16 × 1023 (b) 36.18 × 1023 (c) AgI (d) KCl
(c) 6.04 × 10 23 (d) 12.08 × 1023 73. Schottky defect defines imperfection in the lattice structure
64. Pottasium has a bcc structure with nearest neighbour of
distance 4.52 Å. Its atomic weight is 39. Its density (in kg m –3) (a) solid (b) gas
will be (c) liquid (d) plasma
(a) 454 (b) 804 74. When electrons are trapped into the crystal in anion
(c) 852 (d) 910 vacancy, the defect is known as :
65. The cubic unit cell of a metal (molar mass = 63.55g mol–1) (a) Schottky defect (b) Frenkel defect
has an edge length of 362 pm. Its density is 8.92g cm–3. (c) Stoichiometric defect (d) F-centre
The type of unit cell is 75. Schottky defect in crystals is observed when
(a) primitive (b) face centered (a) an ion leaves its normal site and occupies an interstitial
(c) body centered (d) end centered site
66. AB; crystallizes in a body centred cubic lattice with edge
(b) unequal number of cations and anions are missing from
length ‘a’ equal to 387 pm. The distance between two
the lattice
oppositely charged ions in the lattice is :
(c) density of the crystal increases
(a) 335 pm (b) 250 pm
(c) 200 pm (d) 300 pm (d) equal number of cations and anions are missing from
67. Iron crystallizes in a b.c.c. system with a lattice parameter of the lattice
2.861 Å. Calculate the density of iron in the b.c.c. system 76. The appearance of colour in solid alkali metal halides is
(Atomic weight of Fe = 56, NA = 6.02 × 1023 mol–1) generally due to
(a) 7.92 g ml–1 (b) 8.96 g ml–1 (a) Schottky defect (b) Frenkel defect
(c) 2.78 g ml –1 (d) 6.72 g ml–1 (c) Interstitial positions (d) F-centre
68. An element (atomic mass = 100 g / mol) having bcc structure 77. Crystal defect indicated in the diagram below is
has unit cell edge 400 pm. Then, density of the element is Na Cl Na Cl Na Cl
(a) 10.376 g/cm3 (b) 5.188 g/cm3 Cl– Cl– Na+ Na+
(c) 7.289 g/cm 3 (d) 2.144 g/cm36 + –
Na Cl Cl Na Cl–
– +
69. A metal crystallizes in 2 cubic phases fcc and bcc whose Cl Na Cl Na+
– + – Na+
unit cell lengths are 3.5 Å and 3.0Å respectively. The ratio (a) Interstitial defect
of their densities is
(b) Schottky defect
(a) 0.72 (b) 2.04
(c) Frenkel defect
(c) 1.46 (d) 3.12
(d) Frenkel and Schottky defects
THE SOLID STATE 243
78. Schottky defect generally appears in : 92. Which of the following is non stoichiometric defect?
(a) NaCl (b) KCl (i) Metal excess defect (ii) Impurity defect
(c) CsCl (d) all of these (iii) F-centre (iv) Metal deficiency defect
79. Which defect causes decrease in the density of crystal (a) (i) and (iv) (b) (i), (iii) and (iv)
(a) Frenkel (b) Schottky (c) (iii) and (ii) (d) All of these
(c) Interstitial (d) F – centre 93. Which kind of defects are introduced by doping ?
80. Which statement does not make sense? (a) Dislocation defect (b) Schottky defect
(a) Frenkel defect is not found in alkali metal halides (c) Frenkel defects (d) Electronic defects
94. Silicon doped with electron – rich impurity forms _______.
(b) Schottky defect is very common in alkali metal halides
(a) p-type semiconductor
(c) Schottky defect lowers the density of the crystal
(b) n-type semiconductor
(d) Frenkel defect lowers the density of the crystal. (c) intrinsic semiconductor
81. Frenkel and Schottky defects are : (d) insulator
(a) nucleus defects (b) non-crystal defects 95. Which of the following defects is also known as dislocation
(c) crystal defects (d) nuclear defects defect ?
82. Equal number of atoms or ion missing from normal lattice (a) Frenkel defect
point creating a vacancy due to (b) Schottky defect
(a) Frenkel defect (b) Mass defect (c) Non – stoichiometric defect
(c) Schottky defect (d) Interstitial defect (d) Simple interstitial defect
83. In stoichiometric defects, the types of compound exhibit 96. Doping of silicon (Si) with boron (B) leads to :
Frenkel defects have/has (a) n-type semiconductor (b) p-type semiconductor
(a) Low co-ordination nos. (c) metal (d) insulator
(b) High co-ordination 97. On doping Ge metal with a little of In or Ga, one gets
(c) Small difference in the size of cations and anions (a) p-type semi conductor (b) n-type semi conductor
(d) None of these (c) insulator (d) rectifier
84. The crystal with metal deficiency defect is 98. With which one of the following elements silicon should be
(a) NaCl (b) FeO doped so as to give p-type of semiconductor ?
(c) KCl (d) ZnO (a) Germanium (b) Arsenic
(c) Selenium (d) Boron
85. Which of the following has Frenkel defects?
99. To get a n- type semiconductor, the impurity to be added to
(a) Sodium chloride (b) Graphite
silicon should have which of the following number of
(c) Silver bromide (d) Diamond
valence electrons
86. Which of the following crystals does not exhibit Frenkel (a) 1 (b) 2
defect? (c) 3 (d) 5
(a) AgBr (b) AgCl 100. If we mix a pentavalent impurity in a crystal lattice of
(c) KBr (d) ZnS germanium, what type of semiconductor formation will
87. Due to Frenkel defect, the density of ionic solids occur?
(a) decreases (b) increases (a) p-type (b) n-type
(c) neither (a) nor (b) (d) does not change (c) both (a) and (b) (d) None of the two.
88. In a solid lattice the cation has left a lattice site and is located 101. The addition of arsenic to germanium makes the latter a
at an interstitial position, the lattice defect is : (a) metallic conductor (b) intrinsic semiconductor
(a) Interstitial defect (b) Valency defect (c) mixed conductor (d) extrinsic semiconductor
(c) Frenkel defect (d) Schottky defect 102. Pure silicon doped with phosphorus is a
(a) metallic conductor (b) insulator
89. Doping of AgCl crystals with CdCl 2 results in
(c) n-type semiconductor (d) p-type semiconductor
(a) Frenkel defect 103. Which of the folliowing metal oxides is anti-ferromagnetic
(b) Schottky defect in nature?
(c) Substitutional cation vacancy (a) MnO2 (b) TiO2
(d) Formation of F - centres (c) VO2 (d) CrO2
90. Cations are present in the interstitial sites in ________. 104. Which of the following compound is like metallic copper in
(a) Frenkel defect (b) Schottky defect its conductivity and appearance?
(c) Vacancy defect (d) Metal deficiency defect (a) VO3 (b) TiO3
91. What is the energy gap between valence band and (c) ReO3 (d) CrO2
conduction band in crystal of insulators ? 105. Magnetic moment of electron is due to which of the
(a) Both the bands are overlapped with each other following reason?
(b) Very small (a) Due to its orbital motion around the nucleus.
(c) Infinite (b) Due to its spin around its own axis.
(d) Very large (c) Due to negative charge on electron.
(d) Both (a) and (b).
EBD_7207
244 THE SOLID STATE
106. What is the value of Bohr magneton B in A m2? (i)
(a) 9.27 × 10–24 Am2 (b) 9.27 × 10–20 Am2 : : : :
(c) 9.00 × 10–30 Am2 (d) 9.27 × 10–25 Am2

:
107. Which of the following type of substances can be
: : As : :

:
permanently magnetised? : : : :
(a) Diamagnetic (b) Ferromagnetic

:
(c) Ferrimagnetic (d) Antiferromagnetic : : : :

:
108. Which of the following structure represents

:
ferrimagnetism? (ii)
: : : :
(a) (b)

:
(c) (d) : : B : :

:
109. Which of the following statement is not correct? : : : :
(a) Paramagnetic substances lose their magnetism in the

:
absence of magnetic field. : : : :

:
(b) Diamagnetic substances are weakly magnetised in
magnetic field in opposite direction. (iii)
(c) Ferromagnetic substances becomes paramagnetic on
: : : :

:
heating. : : P : :
(d) In antiferromagnetism domains are oppositely oriented

:
and cancel out each other’s magnetic moment. : : : :
110. An element containing an odd number of electrons is:

:
(a) Paramagnetic (b) Diamagnetic : : : :

:
(c) Antiferromagnetic (d) None of these
111. Which of the following oxides behaves as conductor or (iv)
: : : :
insulator depending upon temperature ?

:
(a) TiO (b) SiO2 : : Al : :
(c) TiO3 (d) MgO :

:
112. Which of the following oxides shows electrical properties : : : :
:

:
like metals ? :
(a) SiO2 (b) MgO
: : :
:

:
(c) SO2 (s) (d) CrO2 (a) (i) and (iii) (b) (i)
113. Which of the following statements is not true ? (c) (ii) and (iv) (d) (iii)
(a) Paramagnetic substances are weakly attracted by
magnetic field. STATEMENT TYPE QUESTIONS
(b) Ferromagnetic substances cannot be magnetised
permanently 116. Which of the following statement(s) is/are correct?
(c) The domains in antiferromagnetic substances are (i) Crystallin e solids have definite characteristic
oppositely oriented with respect to each other. geometrical shape.
(d) Pairing of electrons cancels their magnetic moment in (ii) Crystalline solids have long range order.
the diamagnetic substances. (iii) Sodium chloride and quartz glass are examples of
114. A ferromagnetic substance becomes a permanent magnet crystalline solids.
when it is placed in a magnetic field because ________. (iv) Crystalline solids are isotropic in nature.
(a) all the domains get oriented in the direction of magnetic (a) (i), (ii) and (iii) (b) (i), (ii) and (iv)
field. (c) (i) and (ii) (d) (i) only
(b) all the domains get oriented in the direction opposite 117. Which of the following sequence of T and F is correct for
to the direction of magnetic field. given statements. Here T stands for true statement and F
(c) domains get oriented randomly. stands for false statement?
(d) domains are not affected by magnetic field. (i) Ionic solids are electrical insulators in the solid state
115. A perfect crystal of silicon interchange is doped with some but conduct electricity in molten state.
elements as given in the options. Which of these options (ii) Graphite is a covalent solid.
show n-type semiconductors ? (iii) Covalent solids are conductor of electricity.
(iv) Non polar molecular solids are held by weak
Silicon atom : : : : dispersion forces or London forces while polar
:
:

molecular solids are held by stronger dipole - dipole


:

: : : :
interactions.
:

: : : :
(a) TTTF (b) FTTF
:

: : : : (c) TFTT (d) TTFT


:

:
THE SOLID STATE 245
118. Which of the following statements(s) is/are incorrect? 123. White crystal of zinc oxide is heated
(i) Only 1/8th portion of an atom located at corner of a (i) Metal excess defect is created.
cubic unit cell is its neighbouring unit cell. (ii) Crystal become p-type semiconductor
(ii) Total number of atoms per unit cell for a face centered (iii) Crystal become yellow in color.
cubic unit cell is 3 . (iv) Free electron are created.
(iii) Atom located at the body center is shared between (a) All statement(s) are correct. (b) (i), (ii) and (iv)
two adjacent unit cells. (c) (i), (ii) and (iii) (d) (ii) and (iv)
(a) (iii) only (b) (ii) only 124. Consider the Oxygen and chromium dioxide, both are placed
(c) (i) and (ii) (d) (ii) and (iii) in magnetic field:
119. Which of the following is/are not true about the voids formed (i) Oxygen is attracted strongly in a magnetic field.
in 3 dimensional hexagonal close packed structure ? (ii) Magnetic field persist in chromium dioxide while in
(i) A tetrahedral void is formed when a sphere of the Oxygen not.
second layer is present above triangular void in the (a) Both Statements are correct.
first layer. (b) Statements (i) is correct only.
(ii) All the triangular voids are not covered by the spheres (c) Statements (ii) is correct only.
of the second layer. (d) Both Statements are incorrect.
(iii) Tetrahedral voids are fomed when the triangular voids
in the second layer lie above the triangular voids in MATCHING TYPE QUESTIONS
the first layer and the triangular shapes of these voids
do not overlap. 125. Match the columns
(iv) Octahedral voids are formed when the triangular voids Column-I Column-II
in the second layer exactly overlap with similar voids (Type of solid) (Example of solid)
in the first layer. (A) Molecular solid (p) Ag
(a) (i) and (iv) (b) (iii) and (iv) (B) Ionic solid (q) SiC
(c) (ii) and (iii) (d) (iv) only (C) Metallic solid (r) CCl4
120. Which of the following sequence of T and F is true for (D) Covalent solid (s) MgO
given statements. Here T stands for true statement and F (a) A – (s), B – (r), C – (p), D – (q)
stands for false statement? (b) A – (q), B – (s), C – (p), D – (r)
(i) Frenkel defect results in increase in density of the (c) A – (r), B – (q), C – (p), D – (s)
solid. (d) A – (r), B – (s), C – (p), D – (q)
(ii) ZnS, AgCl, AgBr and AgI shows Frenkel defect. 126. Match the columns
(iii) Schottky defect results in decrease in density of the Column-I Column-II
solid. (Type of unit cell) (Characteristic feature)
(iv) AgBr shows Schottky defect only. (A) Primitive cubic unit (p) Each of the three
(v) For NaCl there is one Schottky defect per 1016 ions. cell perpendicular edges
(a) TTTFT (b) TTTFF compulsorily have the
(c) FTTFT (d) FTTFF different edge length i.e.;
121. Which of the following statements is /are correct? a b c.
(i) LiCl crystals are pink due to metal excess defect due (B) Body centered cubic (q) Number of atoms per unit
to presence of extra L+i ion at interstitial sites. unit cell cell is one
(ii) Zinc oxide on heating turns yellow because its anionic (C) Face centered cubic (r) Each of the three
sites are occupied by unpaired electrons. unit cell perpendicular edges
(iii) In FeO crystals some Fe2+ are missing and the loss of compulsorily have the
positive charge is made up by the presence of required same edge length i.e.;
number of Fe3+ ions. a = b = c.
(a) (i) and (ii) (b) (i), (ii) and (iii) (D) End centered (s) In addition to the
(c) (ii) and (iii) (d) (iii) only orthorhombic unit contribution from the
122. Which of the following sequence of T and F is correct? cell corner atoms the number
Here ‘T’ stands for true and ‘F’ stands for false statement. of atoms present in a unit
(i) Solids have conductivities in the order of 10–20 to 107 cell is one.
ohm–1 m–1. (t) In addition to the
(ii) In semiconductors the gap between filled valence band contribution from the
and conduction band is small. corner atoms the number
(iii) Electrical conductivity of insulators increases with rise of atoms present in a unit
in temperature. cell is three.
(iv) Insulators have conductivities ranging between 10–6 (a) A – (q), B – (s), C – (r, t), D – (p)
to 104 ohm–1 m–1 . (b ) A – (q, r), B – (r, s), C – (r, t), D – (p,s)
(a) TTFF (b) TTFT (c) A – (r, s), B – (q, r), C – (r), D – (p)
(c) FTFF (d) FTTF (d) A – (t), B – (r, s), C – (p, s), D – (q)
EBD_7207
246 THE SOLID STATE
127. Match the columns 131. Match the columns
Column-I Column-II Column-I Column-II
(Crystal system) (Compounds) (A) Mg in solid state (p) p-Type semiconductor
(A) Rhombohedral (p) KNO3 (B) MgCl2 in molten state (q) n-Type semiconductor
(B) Orthorhombic (q) Zinc blende (C) Silicon with (r) Electrolytic conductors
(C) Cubic (r) CdS phosphorus
(D) Hexagonal (s) Calcite (D) Germanium with boron (s) Electronic conductors
(a) A – (p), B – (q), C – (s), D – (r) (a) A – (q), B – (p), C – (r), D – (s)
(b) A – (r), B – (p), C – (q), D – (s) (b) A – (p), B – (q), C – (s), D – (r)
(c) A – (s), B – (p), C – (q), D – (r) (c) A – (s), B – (r), C – (q), D – (p)
(d) A – (r), B – (s), C – (p), D – (q)
(d) A – (q), B – (r), C – (s), D – (p)
132. Match the columns
128. Match the columns
Column-I Column-II
Column-I Column-II
(Molecule/ion) (Magnetic property)
(A) Square close packing (p) Triangular voids
(A) C6H6 (p) Antiferromagnetic
in two dimensions
(B) CrO2 (q) Ferrimagnetic
(B) Hexagonal close (q) Pattern of spheres is (C) MnO (r) Ferromagnetic
packing in two repeated every fourth (D) Fe3O4 (s) Paramagnetic
dimensions layer (E) Fe3+ (t) Diamagnetic
(C) Hexagonal close (q) Coordination number 4 (a) A – (t), B – (r), C – (q), D – (p), E – (s)
packing in three (b) A – (r), B – (t), C – (p), D – (s), E – (q)
dimensions (c) A – (t), B – (r), C – (p), D – (q), E – (s)
(D) Cubic close packing (s) Pattern of sphere is (d) A – (t), B – (r), C – (p), D – (s), E – (q)
in three dimensions repeated alternate layers 133. Match the columns
(a) A – (r), B – (p), C – (s), D – (q) Column-I Column-II
(b) A – (p), B – (s), C – (q), D – (r) (Compound) (Magnetic Property)
(c) A – (s), B – (p), C – (q), D – (s) (A) NaCl (p) Ferrimagnetic
(d) A – (r), B – (p), C – (s), D – (q) (B) MnO (q) Paramagnetic
129. Match Column-I (Type of close packed structure) with (C) CrCl3 (r) Ferromagnetic
Column-II (Coordination number) and choose the correct (D) CrO2 (s) Diamagnetic
option. (E) MgFe2O4 (t) Antiferromagnetic
Column-I Column-II (a) A – (p), B – (r), C – (q), D – (t), E – (s)
(A) One dimensional close (p) 12 (b) A – (t), B – (q), C – (r), D – (p), E – (s)
packed arrangement. (c) A – (r), B – (t), C – (q), D – (p), E – (s)
(B) Square close packing in (q) 6 (d) A – (s), B – (t), C – (q), D – (r), E – (p)
two dimensions.
(C) Two dimensional (r) 2 ASSERTION-REASON TYPE QUESTIONS
hexagonal close packing. Directions : Each of these questions contain two statements,
(D) Cubic close packed (s) 4 Assertion and Reason. Each of these questions also has four
arrangement. alternative choices, only one of which is the correct answer. You
(a) A – (r), B – (s), C – (q), D – (p) have to select one of the codes (a), (b), (c) and (d) given below.
(b) A – (r), B – (s), C – (p), D – (q) (a) Assertion is correct, reason is correct; reason is a correct
(c) A – (s), B – (r), C – (q), D – (p) explanation for assertion.
(d) A – (s), B – (q), C – (p), D – (r) (b) Assertion is correct, reason is correct; reason is not a
130. Match the columns correct explanation for assertion
Column-I Column-II (c) Assertion is correct, reason is incorrect
(A) Impurity defect (p) NaCl with anionic sites (d) Assertion is incorrect, reason is correct.
F-centres 134. Assertion : Crystalline solids have long range order.
(B) Metal excess defect (q) FeO with Fe3+ Reason : Amorphous solids have short range order.
(C) Metal deficiency (r) NaCl with Sr2+ and some 135. Assertion: Glass panes fixed to windows or panes of old
defect cationic sites vacant buildings are found to be slightly thicker at the bottom.
Reason: Amorphous solids have a tendency to flow.
(a) A – (r), B – (p), C – (q)
136. Assertion : In crystal lattice, the size of the tetrahedral hole
(b) A – (p), B – (q), C – (r)
is larger than an octahedral hole.
(c) A – (r), B – (q), C – (p)
Reason : The cations occupy less space than anions in
(d) A – (q), B – (p), C – (r) crystal packing.
THE SOLID STATE 247
137. Assertion : In close packing of spheres, a tetrahedral void is 149. In a cubic lattice A atom occupy all the corners. If B atom
surrounded by four spheres whereas an octahedral void is occupy one of the opposite face, and atom C occupy the
surrounded by six spheres. remaining faces. The simplest formulae of the compound is
Reason : A tetrahedral void has a tetrahedral shape whereas (a) ABC3 (b) ABC2
an octahedral void has an octahedral shape. (c) ABC (d) AB2C
138. Assertion : The packing efficiency is maximum for the fcc 150. A solid has a structure in which ‘W’ atoms are located at the
structure. corners of a cubic lattice ‘O’ atoms at the centre of edges
Reason : The cordination number is 12 in fcc structures. and Na atoms at the centre of the cube. The formula for the
139. Assertion : In any ionic solid (MX) with Schottky defects, compound is
the number of positive and negative ions are same. (a) Na2WO3 (b) Na2WO2
Reason : Equal number of cation and anion vacancies are (c) NaWO2 (d) NaWO3
present. 151. Potassium crystallizes with a
140. Assertion : Electrical conductivity of semiconductors (a) body-centred cubic lattice
increases with increasing temperature. (b) face-centred cubic lattice
Reason : With increase in temperature, large number of (c) simple cubic lattice
electrons from the valence band can jump to the conduction
(d) orthorhombic lattice
band.
152. In a compound, atoms of element Y form ccp lattice and
141. Assertion : On heating ferromagnetic or ferrimagnetic
those of element X occupy 2/3rd of tetrahedral voids. The
substances, they become paramagnetic.
formula of the compound will be
Reason : The electrons change their spin on heating.
(a) X4Y3 (b) X2Y3 (c) X2Y
CRITICAL THINKING TYPE QUESTIONS (d) X3Y4
153. A compound MpXq has cubic close packing (ccp) arrangement
142. Which of the following type of solid has high melting point of X. Its unit cell structure is shown below. The empirical
and do not conduct electricity but its aqueous solution formula of the compound is
and melt conduct electricity ?
(a) Covalent (b) Ionic
(c) Molecular (d) Metallic
143. A group 1 hydride crystal when heated in presence of its
constituent metal vapour shows pink color. This metal can M
be
(a) Na (b) K X
(c) Rb (d) Li
144. A solid with high electrical and thermal conductivity is
(a) Si (b) Li
(c) NaCl (d) Ice
145. Which of the following is true about the value of refractive
index of quartz glass ?
(a) Same in all directions (a) MX (b) MX2
(b) Different in different directions (c) M2X (d) M5 X14
(c) Cannot be measured 154. A substance AxBy crystallizes in a face centred cubic (fcc)
(d) Always zero lattice in which atoms ‘A’ occupy each corner of the cube
146. Which of the following features are not shown by quartz and atoms ‘B’ occupy the centres of each face of the cube.
glass ? Identify the correct composition of the substance AxBy
(i) This is a crystalline solid. (a) AB3
(ii) Refractive index is same in all the directions. (b) A4B3
(iii) This has definite heat of fusion.
(c) A3 B
(iv) This is also called super cooled liquid.
(a) (i) and (iii) (b) (iii) and (iv) (d) Composition can’t be specified
(c) (i), (ii) and (iv) (d) (iii) only 155. In which of the following structures coordination number
147. The number of carbon atoms per unit cell of diamond unit for cations and anions in the packed structure will be same?
cell is : (a) Cl – ion form fcc lattice and Na + ions occupy all
(a) 8 (b) 6 octahedral voids of the unit cell.
(c) 1 (d) 4 (b) Ca2+ ions form fcc lattice and F– ions occupy all the
148. Na and Mg crystallize in bcc and fcc type crystals respectively, eight tetrahedral voids of the unit cell.
then the number of atoms of Na and Mg present in the unit (c) O2– ions form fcc lattice and Na+ ions occupy all the
cell of their respective crystal is eight tetrahedral voids of the unit cell.
(a) 4 and 2 (b) 9 and 14 (d) S2– ions form fcc lattice and Zn2+ ions go into alternate
(c) 14 and 9 (d) 2 and 4 tetrahedral voids of the unit cell.
EBD_7207
248 THE SOLID STATE
156. If ‘a’ stands for the edge length of the cubic systems : simple 163. A metal has a fcc lattice. The edge length of the unit cell is
cubic, body centred cubic and face centred cubic, then the 404 pm. The density of the metal is 2.72 g cm-3. The molar
ratio of radii of the spheres in these systems will be mass of the metal is :
respectively, (NA Avogadro’s constant = 6.02 × 1023 mol–1)
(a) 30 g mol–1 (b) 27 g mol–1
1 3 1 1 1 (c) 20 g mol –1 (d) 40 g mol–1
(a) a: a: a (b) a : 3a : a
2 4 2 2 2 2 164. Al (at. wt 27) crystallizes in the cubic system with a cell

1 3 3 edge of 4.05 Å . Its density is 2.7 g per cm 3 . Determine the


(c) a: a: a (d) 1a : 3a : 2a unit cell type calculate the radius of the Al atom
2 2 2
(a) fcc, 2.432 Å (b) bcc, 2.432 Å
157. Packing efficiency by arrangement of atoms in two
dimensional hexagonal close packing is (c) bcc, 1.432 Å (d) fcc, 1.432 Å
165. A compound is formed by elements A and B. The crystalline
(a) 60.43 (b) 65.78
cubic structure has the A atoms at the corners of the cube
(c) 59.78 (d) 68.76
and B atoms at the body centre. The simplest formula of the
158. The packing efficiency of the two-dimensional square unit compound is
cell shown below is : (a) AB (b) A6 B
(c) AB6 (d) A8B4
166. What type of semiconductors respectively are formed when
the group 14 are doped with the group 13 and group 15?
(a) p,n (b) n,p
(c) p,p (d) n,n
167. Which of the following is ferroelectric compound?
L (a) BaTlO3 (b) K4[Fe(CN)6]
(a) 39.27% (b) 68.02% (c) Pb2O3 (d) None of these
(c) 74.05% (d) 78.54% 168. Substance which is weakly repelled by a magnetic field is
159. Edge length of unit cell is 3.608 × 10–8 cm, which crystallizes (a) O2 (b) H2O
in fcc and is determined to have a density of 8.92 g/cm 3. (c) CrO2 (d) Fe3O4
The mass of four atoms is (e) ZnFe2O4
(a) 4.18 × 10–22 (b) 1.67 × 10–21 169. Which one of the following statements is correct?
(c) 2.09 × 10 –22 (d) 8.37 × 10–22 (a) NaCl is a paramagnetic salt
160. The edge length of unit cell of a metal having molecular (b) CuSO4 is a diamagnetic salt
weight 75 g/mol is 5Å which crystallizes in cubic lattice. If (c) MnO is an example of ferromagnetic substance
the density is 2g/cc then find the radius of metal atom. (d) Ferrimagnetic substance like ZnFe2 O4 becomes
(NA = 6 × 1023). Give the answer in pm. paramagentic on heating
(a) 217 pm (b) 210 pm 170. Which of the following is true about the charge acquired by
p-type semiconductors ?
(c) 220 pm (d) 205 pm
(a) positive
161. The number of atoms in 100 g of an fcc crystal with density,
(b) neutral
d = 10 g/cm3 and cell edge equal to 100 pm, is equal to
(c) negative
(a) 1 × 1025 (b) 2 × 1025
25
(d) depends on concentration of p impurity
(c) 3 × 10 (d) 4 × 1025 171. Which of the following represents correct order of
162. A metallic element exists as cubic lattice. Each edge of the conductivity in solids ?
unit cell is 2.88 Å. The density of the metal is 7.20 g cm–3. (a) Kmetals > > Kinsulators < Ksemiconductors
How many unit cell will be present in 100 g of the metal? (b) Kmetals < < Kinsulators < Ksemiconductors
(a) 6.85 × 102 (b) 5.82 × 1023 (c) Kmetals ; Kinsulators > Ksemiconductors = zero
(c) 4.37 × 10 5 (d) 2.12 × 106 (d) Kmetals < Ksemiconductors > Kinsulators zero
THE SOLID STATE 249

FACT / DEFINITION TYPE QUESTIONS 31. (c)


1. (b) Intermolecular forces are strong in solids.
2. (d) Crystals show good cleavage because their constituent
particles are arranged in planes.
3. (b) Crystalline solids are anisotropic in nature that is some
of their physical properties like electrical resistance
or refractive index show different values when
measured along different directions in the same
crystals.
4. (a) In crystalline solid there is perfect arrangement of the
constituent particles only at 0 K. As the temperature An isolated fcc cell is shown here. Each face of the
increases the chance that a lattice site may be cell is common to two adjacent cells. Therefore, each
unoccupied by an ion increases. As the number of face centre atom contributes only half of its volume
defects increases with temperature solid changes into and mass to one cell. Arranging six cells each
liquid. sharing the remaining half of the face centred atoms,
constitutes fcc cubic lattice. e.g., Cu and Al.
5. (b)
32. (b) For orthorhombic system 90
6. (c) Glass is amorphous solid. 33. (a) It is based on the definition of the cubic lattice.
7. (b) Amorphous solids are isotropic, because these 34. (b) For tetragonal a b c , 90
substances show same properties in all directions.
35. (c) The no. of atoms is a unit cell may be calculated by the
8. (c) Glass is amorphous solid. the formula
9. (b) 10. (d) 11. (b) nc nb n f ne
Z
12. (b) On heating, amorphous solids become crystalline at 8 1 2 4
some temperature. The milky appearance of glass is Where nc = no. of atom at the corner
because of some crystallisation. nb = no. of atoms at body centre
13. (c) Amorphous silicon is used as best photovoltaic nf = no. of atoms at face centre
material available for conversion of sunlight into ne = no. of atoms at edge centre.
An fcc crystal contains
electricity.
14. (c) Solid CH4 is a molecular solid. In this, the constituent 8 6
= + = 4 atoms in a unit cell.
molecules are held together by van der Waal’s forces. 8 2
15. (a) Si is an example of covalent crystalline solid among 36. (c) H
the given choices. Si atoms are covalently linked in G
E
tetrahedral manner.
16. (a) Covalent as in case of diamond. F
a
17. (b) Among the given crystals, only silicon exists as a
covalent solid. It has diamond like structure.
C
18. (c) Covalent bond force A
19. (b) In graphite, the electrons are spread out between the a a
sheets. B
20. (d) 21. (c) 22. (c) 23. (a) 24. (a) An isolated fcc cell is shown here. Each face of the
25. (b) 26. (d) 27. (a) 28. (a) cell is common to two adjacent cells. Therefore, each
29. (c) There are only 14 possible three dimensional lattice. face centre atom contributes only half of its volume
These are called Bravais lattices. and mass to one cell. Arranging six cells each
sharing the remaining half of the face centred atoms,
30. (b) constitutes fcc cubic lattice. e.g., Cu and Al.
EBD_7207
250 THE SOLID STATE
37. (b) In bcc structure, 57. (d) Packing fraction of bcc = 68%
no. of atoms at corner = (1/8) × 8 = 1 Empty space = 100 – 68 = 32%
no. of atoms at body centre = 1 58. (c) The hcp arrangement of atoms occupies 74% of the
Total no. of atoms per unit cell = 1 + 1 = 2. available space and thus has 26% vacant space.
38. (b) hcp is a closed packed arrangement in which the unit 59. (b) In AB AB packing spheres occupy 74%. 26% is empty.
cell is hexagonal and coordination number is 12. 60. (d) In NaCl the Cl– and Na+ touch along edge of cube the
39. (c) ABAB...... is hexagonal close packing.
40. (b) In ZnS structure, sulphide ions occupy all fcc lattice a
distance between ions is
points while Zn 2+ ions are present in alternate 2
tetrahedral voids. 61. (a)
41. (b) Metals such as copper and silver crystallise in fcc 62. (b) For body centred cubic lattice Z = 2
structure while metals Mg and Zn crystallise in hcp Atomic mass of unit cell = 133 + 80 = 213 a.m.u
structure. Volume of cell = (436.6 × 10–10)3 cm3
42. (d)
ZM
43. (a) Co-ordination number in hcp and ccp arrangement Density, =
is 12. a3 NA
44. (a) 45. (c) 2 213
46. (d) It represents ccp arrangement. (436.6 10 10 )3 6.02 1023
47. (b) 48. (c) 49. (d) 50. (b) = 8.50 g/cm3
51. (c) hcp and ccp structures have maximum packing 63. (a) There are two atoms in a bcc unit cell.
efficiency = 74%.
So, number of atoms in 12.08 × 1023 unit cells
For bcc = 68%
= 2 × 12.08 × 1023 = 24.16 × 1023 atoms.
For simple cubic = 52.4%
52. Total volume of atoms present in bcc unit cell is. 3 2d 2 4.52
64. (d) For bcc, d a or a 5.219Å
16 3 4 3 2 3 1.732
(a) r (b) r = 522 pm
3 3
z M
8 3 12 3
(c) r (d) r a 3
N A 10 30
3 3
52. (c) For bcc structure total number of atoms = 2. 2 39
3
4 3 (522) (6.023 1023 ) 10 30
Total volume = 2 r
3
0.91g / cm3 910 kg m 3
8 3
= r ZM
3 65. (b)
53. (b) The face centered cubic unit cell contains 4 atom NA V
4 3 16 3 NA V 8.92 6.02 1023 (362)3 10 30
Total volume of atoms 4 r r Z
3 3 M 63.55
54. (c) As CsCl is body-centred, d 3a / 2. =4
55. (d) Number of atoms per unit cell = 1 It has fcc unit cell
a
Atoms touch each other along edges. Hence r = 66. (a) For bcc lattice body diagonal = a 3 .
2
The distance between the two oppositely charged ions
( r = radius of atom and a = edge length)
a
4 = 3
r 2
3
Therefore % fraction = 387 1.732
(2 r )3 6 = 335pm
2
56. (b) Packing fraction is defined as the ratio of the volume
ZM
of the unit cell that is occupied by the spheres to the 67. (a) For b.c.c., Z = 2, Now, d = and V = a3
total volume of the unit cell. NV
P.F. for cpp and bcc are 0.74 and 0.68 respectively. 2 56
d= = 7.92 g ml–1
So, the free space in ccp and bcc are 26% & 32% 23
(6.02 10 ) (2.861 10 8 )3
respectively.
THE SOLID STATE 251
82. (c) The vacancy created due to missing of equal no. of
68. (b) Z M 2 100
atoms or ions form normal lattice point is called
NA a 3 6.023 10 23 (400 10 10 ) 3 Schottky defect. In this type of defect electrical
neutrality of ionic crystal is maintained.
5.188 g / cm 3 83. (a) In stoichiometric Frenkel defects occurs in those
3 compound which have
d1 (a 2 )3 z1 3 4 (i) Low C.N.
69. (c) 1.46
d2 3 z2 3.5 2
(a1 ) (ii) Large difference in size of cations and anions
70. (d) Ionic solids must always maintain electrical neutrality. 84. (b) Transition metals exhibit this defect due to metal
Ionic solids show vacancy or interstitial defects as deficiency, the compound obtained are non
Frenkel and Schottky defect. stoichiometric e.g. It is difficult to prepare ferrous oxide
with the ideal composition of FeO what we actually
71. (b)
obtain is Fe0.95O or FexO with x = 0.93 to 0.96
72. (d) In KCl, co-ordination number of cation and anion is 6
85. (c) AgBr exhibit Frenkel defect.
and 6 respectively. KCl is highly ionic so Schottky
86. (c) KBr does not exhibit Frenkel defect.
defect is common.
87. (d) No change in density
Note : Schottky defect is common in compounds having
88. (c) Frenkel defect is due to dislocation of ion from its usual
high coordination nummber while Frenkel defect is
lattice site to interstitial position.
common in compounds with low coordination number.
89. (c) 90. (a)
73. (a) Schottky defects are found in solid.
91. (d) When insulators (non metal atoms) interact to form a
74. (d) When electrons are trapped in anion vacancies, these solid, their atomic orbitals mix to form two bunch of
are called F-centre. orbitals, separated by a large band gap. Electrons
cannot therefore be promoted to an empty level, where
they could move freely.
+ve –ve
e– ion ion 92. (b) 93. (d) 94. (b) 95. (a) 96. (b)
97. (a) p-type of semiconductors are produced
F- centre in crystal (i) due to metal deficiency defects
(ii) by adding impurity containing less electrons (i.e.,
75. (d) If in an ionic crystal of the type A+, B–, equal number atoms of group 13)
of cations and anions are missing from their lattice
Ge belongs to Group 14 and In to Group 13. Hence on
sites so that the electrical neutrality is maintained. The doping, p-type semicondutor is obtained.
defect is called Schottky defect.
98. (d) The semiconductors formed by the introduction of
76. (d) The appearance of colour in solid alkali metal halide is impurity atoms containing one elecron less than the
due to presence of F-centre found as defect in the parent atoms of insulators are termed as p-type
crystal structure. semiconductors. Therefore silicon containing 14
77. (b) When equal number of cations (Na+) and anions (Cl–) electrons is to be doped with boron containing 13
are missing from their regular lattice positions, we have electrons to give a p-type semi-conductor.
Schottky defect. 99. (d) For n-type, impurity added to silicon should have more
78. (d) Schottky defect occurs in ionic crystals of type A+ B, than 4 valence electrons.
when equal number of cations and anions are missing 100. (b) n-type, since electron is set free.
from their lattice sites so that the electrical neutrality is 101. (d) Extrinsic semiconductor
maintained. This defect generally appears in highly 102. (c) Pure silicon doped with phosphorus is a n-type
ionic compounds which have high coordination semiconductor, as n-type extrinsic semiconductor (Si)
number. NaCl, KCl and CsCl all have high coordination is made by doping the semiconductor with pentavalent
numbers i.e., 6, 6 and 8 respectively. So, Schottky defect element.
appear in all of the given compounds. 103. (a) MnO2
79. (b) More is the Schottky defect in crystal more is the 104. (c) Rhenium oxide ReO3 is like metallic copper in
decrease in density. conductivity.
80. (d) Frenkel defect does not lower the density of the crystal 105. (d) 106. (a) 107. (b)
since the ions do not leave the crystal lattice. 108. (d) Ferrimagnetism is observed when the magentic
81. (c) Frenkel and Schottky defects are crystal defects. It moments of the domains in the substance are aligned
arises due to dislodgement of cation or anion from in parallel and antiparallel directions in unequal
their places in the crystal lattice. numbers.
EBD_7207
252 THE SOLID STATE
109. (c) Ferrimagnetic substances lose ferrimagnetism on 137. (c) Tetrahedral void is so called because it is surrounded
heating and become paramagnetic. by four spheres tetrahedrally while octahedral void is
110. (a) An element containing an odd number of electrons is so called because it is surrounded by six spheres
paramagnetic. octahedrally.
111. (c) 112. (d) 113. (b) 114. (a) 115. (a) 138. (b)
139. (a) Schottky defect is due to missing of equal number of
STATEMENT TYPE QUESTIONS cations and anions.
140. (a) In case of semiconductors, the gap between valence
116. (c) Quartz glass is an example of amorphous solid and band and the conduction band is small and therefore
crystalline solids are anisotropic in nature. some of the electrons may jump from valence band to
117. (d) Covalent solids are insulator of electricity. Graphite is conduction band and thus on increasing temperature
a covalent solid but it is a conductor of electricity due conductivity is also increased.
to its structure. 141. (a) All magnetically ordered solids (ferromagnetic,
ferrimagnetic and antiferromagnetic solids) transform
118. (d) Total number of atoms per unit cell for a face centered
to the paramagnetic state at high temperature due to
cubic unit is 4.
the randomisation of spins.
The atom at the body center completely belongs to
the unit cell in which it is present. CRITICAL THINKING TYPE QUESTIONS
119. (b)
142. (b)
120. (c) Frenkel defect does not change the density of the
143. (d) Excess of lithium makes LiCl crystal pink.
solid.
144. (b) Out of the given substances, only Li has high electrical
AgBr shows both Frenkel and Schottky defects. and thermal conductivity as Li is a metallic solid.
121. (d) LiCl crystals are pink because its anionic sites are 145. (a) 146. (a)
147. (a) Diamond is like ZnS. In diamond cubic unit cell, there
occupied by unpaired eonline . Zinc oxide shows metal
are eight corner atoms, six face centered atoms and
excess defect due to presence of extra cations at four more atoms inside the structure.
interstitial sites. Number of atoms present in a diamond cubic cell
122. (a) Electrical conductivity of semiconductors increases
1 1
with rise in temperature. Insulators have = 8´ + 6´ + 4 = 8
8 2
conductivities ranging between 10–20 to 10–10 ohm–
1 m–1 (corners) (face (inside
. centered) body)
123. (b) Due to release of electrons, the crystal can conducts 148. (d) The bcc cell consists of 8 atoms at the corners and one
electricity but conductivity is not as high as that of atom at centre. Contribution of each atom at each corner
metals. Its conductivity is very low and because
conduction is due to electrons so it is n-type 1
is equal to .
semiconductor, also excitation of these electron give 8
rise to yellow color in crystal. 1
124. (c) CrO2 is a ferromagnetic whereas O2 is paramagnetic. n 8 1 2
8
The fcc cell consists of 8 atoms at the eight corners
MATCHING TYPE QUESTIONS
and one atom at each of the six faces. This atom at the
125. (d) 126. (b) 127. (c) 128. (a) 129. (a) face is shared by two unit cells.
130. (a) 131. (c) 132. (c) 133. (d) 1 1
n 8 6 4
8 2
ASSERTION-REASON TYPE QUESTIONS
1 1 1
134. (b) In crystalline solids constituents are arranged in 149. (b) A : B : C = 8: 2: 4 =1:1:2
8 2 2
definite orderly arrangement. This regular arrangement
of constituents extends throughout the three 1
150. (d) In a unit cell, W atoms at the corner 8 1
dimensional network of crystal. Thus crystalline 8
substances said to have long range order. Whereas 1
O-atoms at the centre of edges 12 3
amorphous solids have no regular arrangement. 4
135. (a) Na-atoms at the centre of the cube = 1
136. (d) Tetrahedral holes are smaller in size than octahedral W : O : Na = 1 : 3 : 1
Hence, formula = NaWO3
holes. Cations usually occupy less space than anions.
THE SOLID STATE 253
151. (a) Potassium crystallises in bcc lattice.
3
152. (a) From the given data, we have Area of hexagonal 6 (2r)2
Number of Y atoms in a unit cell = 4 4

2 16
Number of X atoms in a unit cell 8 3
3 3 =6 4r 2
4
From the above we get the formula of the compound
as X16 / 3Y4 or X 4 Y3 6 3 r2

1 2 r2
153. (b) No. of M atoms = 4+1=1+1=2 % occupied by 100
4 6 3 r2
1 1 2 3.14
No. of X atoms = ×6+ ×8=3+1=4 100 60.43%
2 8 6 3
So, formula = M2X4 = MX2 158. (d) Packing efficiency
1
154. (a) Number of A atoms = 8 1 = Area occupied by circles within the square
8 Area of square
1 2 r2 2 r2
Number of B atoms = 6 3 100 100 =
2 = 100 78.54%
L2 2( 2 r ) 2
4
Formula AB3
159. (a) In fcc structure one unit cell consist 4 atoms, hence
155. (a)
density × a3 = The mass of four atoms
156. (a) Following generalization can be easily derived for
= 8.92 × (3.608 × 10–8)3
various types of lattice arrangements in cubic cells
= 4.18 × 10–22
between the edge length (a) of the cell and r the radius
of the sphere. ZM
160. (a)
a NA V
For simple cubic : a = 2r or r
2 NA V 2 6 1023 (5 10 8 )3
For body centred cubic : Z
M 75
4 3 Z = 2, which represents bcc structure
a r or r a
3 4 3 3
r a 5 2.165Å 216.5 pm
For face centred cubic : 4 4
1 217 pm
a 2 2r or r a
2 2
a3 N A 10 30
Thus the ratio of radii of spheres for these will be 161. (d) M
Z
simple : bcc : fcc
a 3 1 10 (100)3 6.02 1023 10 30
= : a: a 15.05
2 4 2 2 4
i.e. option (a) is correct answer.
6.02 1023
Number of atoms in 100 g = 100
157. (a) 15.05
= 4 × 1025
162. (b) The volume of the unit cell
= (2.88 Å)3 = 23.9 × 10–24 cm3.
The volume of 100 g of the metal
m 100
Let radius of the sphere = r = = = 13.9 cm3
7.20
Area occupied by sphere in hexagonal close packing
Number of unit cells in this volume
1
r2 6 r2 2 r2 13.9 cm3
6 = 24 = 5.82 × 1023
23.9 10 cm3
EBD_7207
254 THE SOLID STATE
163. (b) Density is given by 165. (a) Given: Atoms are present in the corners of cube = A
and atom present at body centre = B. We know that a
Z M
d ; where Z = number of formula units present cubic unit cell has 8 corners. Therefore contribution of
NA a 3
1
in unit cell, which is 4 for fcc each atom at the corner = . Since number of atoms
a = edge length of unit cell. M = Molecular mass 8
per unit cell is 8, therefore total contribution =
4 M 10
2.72 = ( 1pm 10 cm) 1
23 10 3 8 1 . We also know that atoms in the body centre,
6.02 10 404 10
8
therefore number of atoms per unit cell = 1. Thus
2.72 ´ 6.02 ´ (404)3
M= = 26.99 = 27 gm mole–1 formula of the compound is AB.
4 ´107
166. (a) In first case conduction is due to hole,while in second
Z M case it is due electron.
164. (d) = , 167. (a) BaTlO3
No a3
168. (b) Substances which are weakly repelled by external
Z 27 magnetic field are called diamagnetic substances, e.g.,
2.7 = 23
Z=4 H2O.
6.02 10 ( 4.05) 3 10 24
169. (d) Ferrimagnetic substance become para-magnetic on
Hence it is face centred cubic unit lattice. heating. This is due to randomisation of spins on
Again 4r = a 2 = 5.727 Å heating.
r = 1.432 Å 170. (b) 171. (a)
16
SOLUTIONS

FACT / DEFINITION TYPE QUESTIONS 9. 1 M, 2.5 litre NaOH solution is mixed with another 0.5 M, 3
litre NaOH solution. Then find out the molarity of resultant
1. “The importance of many pure substance in life depends solution
on their composition.” (a) 0.80 M (b) 1.0 M
Which of the following statement justify the above fact? (c) 0.73 M (d) 0.50 M
(a) 1 ppm of fluoride ions in water prevents tooth decay. 10. An X molal solution of a compound in benzene has mole
(b) 1.5 ppm of fluoride ions causes tooth decay. fraction of solute equal to 0.2. The value of X is
(c) Concentration above 1.5 ppm can be poisonous. (a) 14 (b) 3.2
(d) All of the above. (c) 1.4 (d) 2
2. Which of the following fluoride is used as rat poison? 11. The molarity of the solution containing 7.1 g of Na2SO4 in
(a) CaF2 (b) KF 100 ml of aqueous solution is
(c) NaF (d) MgF2 (a) 2 M (b) 0.5 M
3. Most of the processes in our body occur in (c) 1 M (d) 0.05 M
(a) solid solution (b) liquid solution 12. The vapour pressure of pure benzene at 25°C is 640 mm Hg
(c) gaseous solution (d) colloidal solution and that of solution of solute A is 630 mm Hg. The molality
4. The term homogenous mixtures signifies that of solution is
(a) its composition is uniform throughout the mixture. (a) 0.2 m (b) 0.4 m
(b) its properties are uniform throughout the mixture. (c) 0.5 m (d) 0.1 m
(c) both composition an d properties are un iform 13. 4.0 g of NaOH is dissolved in 100 ml solution. The normality
throughout the mixture. of the solution is
(d) neither composition nor properties are uniform (a) 0.1 N (b) 0.5 N
throughout the mixture. (c) 4.0 N (d) 1.0 N
5. Which of the following mixture is(are) called solution? 14. The molarity of pure water is
(i) water + ammonia (ii) water + acetone (a) 50 M (b) 18 M
(iii) acetone + alcohol (iv) hexane + water (c) 55.6 M (d) 100 M
(a) (i), (ii) and (iii) (b) (i), (iii) and (iv) 15. An aqueous solution of glucose is 10% in strength. The
(c) (i) and (iv) (d) (ii) and (iii) volume in which 1 g mole of it is dissolved, will be
6. Which of the following is a quantitative description of the (a) 9 litre (b) 1.8 litre
solution? (c) 8 litre (d) 0.9 litre
(a) Dilute (b) Concentrated
(c) Saturated (d) Molar 16. 10 g of NaCl is dissolved in 10 6 g of the solution. Its
7. When a solute is present in trace quantities the following concentration is
expression is used (a) 100 ppm (b) 0.1 ppm
(a) Gram per million (b) Milligram percent (c) 1 ppm (d) 10 ppm
(c) Microgram percent (d) Parts per million 17. On adding a solute to a solvent having vapour pressure
8. Molarity of liquid HCl will be, if density of solution is 0.80 atm, vapour pressure reduces to 0.60 atm. Mole fraction
1.17 gm/cc of solute is
(a) 36.5 (b) 32.05 (a) 0.25 (b) 0.75
(c) 18.25 (d) 42.10 (c) 0.50 (d) 0.33
EBD_7207
256 SOLUTIONS
18. 2.5 litres of NaCl solution contain 5 moles of the solute. N N N
What is the molarity? 29. If 50 ml H2SO4, 30 ml HNO3, 10 ml HCl is mixed
10 3 2
(a) 5 molar (b) 2 molar and solution is made to 1L. Then normality of resultant
(c) 2.5 molar (d) 12.5 molar solution is
19. The mole fraction of the solute in one molal aqueous
N N
solution i (a) (b)
20 40
(a) 0.009 (b) 0.018
(c) 0.027 (d) 0.036 N
(c) (d) N
20. 5 ml of N HCl, 20 ml of N/2 H2SO4 and 30 ml of N/3 HNO3 50
are mixed together and volume made to one litre. The 30. A solution made by dissolving 40 g NaOH in 1000 g of water
normality of the resulting solution is is
N N (a) 1 molar (b) 1 normal
(a) (b) (c) 1 molal (d) None of these
5 10
31. Which of the following concentration terms is/are
N N independent of temperature?
(c) (d)
20 40 (a) Molality only
21. 25ml of a solution of barium hydroxide on titration with a 0.1 (b) Molality and mole fraction
molar solution of hydrochloric acid gave a titre value of 35 (c) Molarity and mole fraction
ml. The molarity of barium hydroxide solution was (d) Molality and normality
(a) 0.07 (b) 0.14 32. A solution is prepared by dissolving 10 g NaOH in 1250 mL
(c) 0.28 (d) 0.35 of a solvent of density 0.8 mL/g. The molality of the solution
22. Mole fraction of the solute in a 1.00 molal aqueous solution in mol kg–1 is
is (a) 0.25 (b) 0.2
(a) 0.1770 (b) 0.0177 (c) 0.008 (d) 0.0064
(c) 0.0344 (d) 1.7700 33. Which of the following units is useful in relating
23. What is the normality of a 1 M solution of H3PO4 ? concentration of solution with its vapour pressure?
(a) 0.5 N (b) 1.0 N (a) mole fraction (b) parts per million
(c) 2.0 N (d) 3.0 N (c) mass percentage (d) molality
24. The volume of 4 N HCl and 10 N HCl required to make 1 litre 34. For mixture containing “four” components which of the
of 6 N HCl are following is correct in term of mole fraction?
(a) 0.75 litre of 10 N HCl and 0.25 litre of 4 N HCl (a) x1+ x2+ x3+ x4
(b) 0.50 litre of 4 N HCl and 0.50 litre of 10 N HCl n3
(b) x3
(c) 0.67 litre of 4 N HCl and 0.33 litre of 10 N HCl n1 n 2 n 3
(d) 0.80 litre of 4 N HCl and 0.20 litre of 10 N HCl
n1 n1
25. Molarity of H2SO4 is 18 M. Its density is 1.8 g/ml. Hence (c x1
molality is n1 n 2 n 3 n 4 n
(a) 36 (b) 200 (d) n1 + n2 + n3 + n4 = 1
35. Which of the following concentration unit is independent
(c) 500 (d) 18
of temperature ?
26. 200 ml of water is added to 500 ml of 0.2 M solution. What is
(a) Normality (b) Molarity
the molarity of this diluted solution ?
(c) Formality (d) Molality
(a) 0.5010 M (b) 0.2897 M
36. Which of the following factor do not affect solubility of
(c) 0.7093 M (d) 0.1428 M
solid solute in liquid ?
27. How many grams of concentrated nitric acid solution should
(a) Temperature (b) Pressure
be used to prepare 250 mL of 2.0M HNO3 ? The concentrated
(c) Nature of solute (d) All of these
acid is 70% HNO3
37. When a solid solute is added to the solvent, some solute
(a) 90.0 g conc. HNO3 (b) 70.0 g conc. HNO3
dissolves and its concentration increases in solution. This
(c) 54.0 g conc. HNO3 (d) 45.0 g conc. HNO3 process is known as ______. Some solute particles in
28. For preparing 0.1 N solution of a compound from its impure solution collide with the solid solute particles and get
sample of which the percentage purity is known, the weight separated out of solution. This process is known as
of the substance required will be ______.
(a) Less than the theoretical weight (a) Crystallization, dissolution.
(b) More than the theoretical weight (b) Dissolution, saturation.
(c) Same as the theoretical weight (c) Saturation, crystallization.
(d) None of these (d) Dissolution, crystallization.
SOLUTIONS 257
38. At the state of dynamic equilibrium, for 43. The statement “If 0.003 moles of a gas are dissolved in 900
solute + solvent solution. g of water under a pressure of 1 atmosphere, 0.006 moles
(a) Rate of dissolution = Rate of unsaturation. will be dissolved under a pressure of 2 atmospheres”,
(b) Rate of dissolution = Rate of unsaturation. illustrates
(c) Rate of dissolution = Rate of saturation (a) Dalton’s law of partial pressure
(d) Rate of crystallization = Rate of saturation. (b) Graham’s law
39. Which of the following statements is incorrect? (c) Raoult’s law
(a) A solution in which no more solute can be dissolved (d) Henry’s law
at the same temperature and pressure is called a 44. According to Henry’s law, the amount of gas that will
saturated solution. dissolve in blood plasma or any other liquid is determined
(b) An unsaturated solution is one in which more solute by which of these factor?
can be dissolved at the same temperature. (a) Solubility of the gas in the liquid.
(c) The solution which is in dynamic equilibrium with (b) The total pressure of the gas mixture .
undissolved solute is the saturated solution.
(c) pH of the liquid.
(d) The minimum amount of solute dissolved in a given
(d) The osmotic pressure of the gas mixture.
amount of solvent is its solubility.
40. On dissolving sugar in water at room temperature solution 45. Henry’s law constant of oxygen is 1.4 × 10–3 mol. lit–1. atm–1
feels cool to touch. Under which of the following cases at 298 K. How much of oxygen is dissolved in 100 ml at 298
dissolution of sugar will be most rapid ? K when the partial pressure of oxygen is 0.5 atm?
(a) Sugar crystals in cold water. (a) 1.4 g (b) 3.2 g
(b) Sugar crystals in hot water. (c) 22.4 mg (d) 2.24 mg
(c) Powdered sugar in cold water. 46. At equillibrium the rate of dissolution of a solid solute in a
(d) Powdered sugar in hot water. volatile liquid solvent is ______.
41. The solubility of a solid in a liquid is significantly affected (a) less than the rate of crystallisation.
by temperature changes. (b) greater than the rate of crystallisation.
Solute + Solvent Solution. (c) equal to the rate of crystallisation.
The system being in a dynamic equilibrium must follow (d) zero
Le-chatelier’s principle. Considering the Le-chatelier’s 47. A beaker contains a solution of substance ‘A’. Precipitation
principle which of the following is correct? of substance ‘A’ takes place when small amount of ‘A’ is
(a) added to the solution. The solution is ______.
sol > 0; solubility ; temperature
(a) saturated (b) supersaturated
(b) sol < 0; solubility ; temperature
(c) unsaturated (d) concentrated
(c) sol > 0; solubility ; temperature 48. Maximum amount of a solid solute that can be dissolved in
(d) a specified amount of a given liquid solvent does not depend
sol < 0; solubility ; temperature
upon __________.
42. W1 (a) Temperature (b) Nature of solute
Piston W1 W2 W3 (c) Pressure (d) Nature of solvent
49. Low concentration of oxygen in the blood and tissues of
people living at high altitude is due to _________.
(a) low temperature
(a) (b) (b) low atmospheric pressure
On the basis of the figure given above which of the (c) high atmospheric pressure
following is not true? (d) both low temperature and high atmospheric pressure
(a) In figure (a) assuming the state of dynamic equilibrium 50. Value of Henry's constant KH _______.
rate of gaseous particles entering and leaving the (a) increases with increase in temperature.
solution phase is same. (b) decreases with increase in temperature.
(b) In figure (b) on compressing the gas number of (c) remains constant.
gaseous particles per unit volume over the solution
(d) first increases then decreases.
increases.
51. The value of Henry's constant KH is _______.
(c) Rate at which gaseous particles are striking the
solution to enter it, decreases. (a) greater for gases with higher solubility.
(d) Rate at which gaseous particles are striking the (b) greater for gases with lower solubility.
solution to enter it, increases. (c) constant for all gases.
(d) not related to the solubility of gases.
EBD_7207
258 SOLUTIONS
52. Which of the followingfactor(s) affect the solubility of a 59. Iodine and sulphur dissolve in
gaseous solute in the fixed volume of liquid solvent ? (a) water (b) benzene
(i) Nature of solute (ii) Temperature (iii) Pressure (c) carbon disulphide (d) ethanol
(a) (i) and (iii) at constant T 60. The liquids at a given temperature vapourise and under
(b) (i) and (ii) at constant P equilibrium conditions the pressure exerted by the vapours
(c) (ii) and (iii) only of the liquid over the liquid phase is called
(d) (iii) only (a) osmotic pressure (b) atmospheric pressure
53. Which of the following graph is a correct representation of (c) hydrostatic pressure (d) vapour pressure
Henry’s law? 61. The vapour pressure of the solution at a given temperature
(a) is found to be …………… than the vapour pressure of the
(b) Partial pressure of gas
Partial pressure of gas

pure solvent at the same temperature.


(a) higher (b) lower
(c) equal (d) can't calculate
62. The decrease in the vapour pressure of solvent depends
on the
Mole fraction of gas
(a) quantity of non-volatile solute present in the solution
Mole fraction of gas
in solution in solution (b) nature of non-volatile solute present in the solution
(c) molar mass of non-volatile solute present in the
(c) (d)
Partial pressure of gas
Partial pressure of gas

solution
(d) physical state of non-volatile solute present in the
solution
63. A plot of p1 or p2 vs the mole fractions x1 and x2 is given
as.

Mole fraction of gas Mole fraction of gas


in solution in solution
p2 III p2°
54. Which is an application of Henry’s law? = p1 +
Vapour pressure

p Total II
(a) Spray paint (b) Bottled water p1° p2
(c) Filling up atire (d) Soft drinks (soda)
55. Scuba divers may experience a condition called ______.
p
To avoids this, the tanks used by scuba divers are filled 1
I
with air diluted with _____ .
(a) Migrains, Hydrogen (b) Cramps, Nitrogen x1 = 1 Mole fraction x1 = 0
(c) Nausea, Oxygen (d) Bends, Helium x2 = 0 x2 x2 = 1
56. People living at high attitudes often reported with a problem
In this figure, lines I and II pass through the point for
of feeling weak and inability to think clearly. The reason for
which.
this is.
(a) x1 1; x2 = 1 (b) x1 = x2 1
(a) at high altitudes the partial pressure of oxygen is less
(c) x1 = 1; x2 d) x1 = x2 = 1
than at the ground level.
64. The vapour pressure of two liquids ‘P’ and ‘Q’ are 80
(b) at high altitudes the partial pressure of oxygen is more
and 60 torr, respectively. The total vapour pressure of
than at the ground level.
solution obtained by mixing 3 mole of P and 2 mole of Q
(c) at high altitudes the partial pressure of oxygen is equal
would be
to at the ground level.
(a) 72 torr (b) 140 torr
(d) None of these.
(c) 68 torr (d) 20 torr
57. ____ a contemporary of Henry concluded independently
65. 18 g of glucose (C6H12O6) is added to 178.2 g of water. The
that solubility of a gas in a liquid solution is a function of
vapour pressure of water for this aqueous solution is
____ of the gas.
(a) 76.00 torr (b) 752.40 torr
(a) Mosley, temperature
(c) 759.00 torr (d) 7.60 torr
(b) Dalton, temperature
66. PA and PB are the vapour pressure of pure liquid components,
(c) Dalton, partial pressure
A and B, respectively of an ideal binary solution. If XA
(d) Mosley, partial pressure represents the mole fraction of component A, the total
58. Raoult’s law becomes a special case of Henry’s law when pressure of the solution will be.
(a) K H = p ° (b) K H > p ° (a) PA + XA (PB – PA) (b) PA + XA (PA – PB)
1 1
(c) PB + XA (PB – PA) (d) PB + XA (PA – PB)
(c) K H < p1° (d) K H ³ p1°
SOLUTIONS 259
67. A mixture of ethyl alcohol and propyl alcohol has a vapour 76. Which one of the following is non-ideal solution
pressure of 290 mm Hg at 300 K. The vapour pressure of (a) Benzene + toluene
propyl alcohol is 200 mm Hg. If the mole fraction of ethyl (b) n-hexane + n-heptane
alcohol is 0.6, its vapour pressure (in mm Hg) at the same (c) Ethyl bromide + ethyl iodide
temperature will be (d) CCl4 + CHCl3
(a) 360 (b) 350 77. Mixtures of ethanol and acetone show positive deviation.
The reason is
(c) 300 (d) 700
(a) In pure ethanol, molecules are hydrogen bonded.
68. Two liquids X and Y form an ideal solution. At 300 K, vapour
(b) In pure acetone, molecules are hydrogen bonded
pressure of the solution containing 1 mol of X and 3 mol of (c) In both molecules are hydrogen bonded
Y is 550 mm Hg. At the same temperature, if 1 mol of Y is (d) None of these
further added to this solution, vapour pressure of the 78. A mixture of components A and B will show –ve deviation
solution increases by 10 mm Hg. Vapour pressure ( in mm when
Hg) of X and Y in their pure states will be, respectively (a) Vmix > 0
(a) 300 and 400 (b) 400 and 600 (b) Hmix < 0
(c) 500 and 600 (d) 200 and 300 (c) A – B interaction is weaker than A – A and B – B
69. The vapour pressure of two liquids X and Y are 80 and 60 interactions
torr respectively. The total vapour pressure of the ideal (d) A–B interaction is stronger than A–A and B–B
solution obtained by mixing 3 moles of X and 2 moles of Y interactions.
would be 79. Which of the following liquid pairs shows a positive
(a) 68 Torr (b) 140 Torr deviation from Raoult’s law ?
(c) 48 Torr (d) 72 Torr (a) Water - Nitric acid
70. The vapour pressure of pure benzene and toluene at a (b) Benzene - Methanol
particular temperature are 100 mm and 50 mm respectively. (c) Water - Hydrochloric acid
(d) Acetone - Chloroform
Then the mole fraction of benzene in vapour phase in
80. A solution of acetone in ethanol
contact with equimolar solution of benzene and toluene is
(a) shows a positive deviation from Raoult’s law
(a) 0.67 (b) 0.75 (b) behaves like a non ideal solution
(c) 0.33 (d) 0.50 (c) obeys Raoult’s law
71. A solution containing components A and B follows Raoult's (d) shows a negative deviation from Raoult’s law
law when 81. Negative deviation from Raoult’s law is observed in
(a) A – B attraction force is greater than A – A and B – B which one of the following binary liquid mixtures?
(b) A – B attraction force is less than A – A and B – B (a) Ethanol and acetone
(c) A – B attraction force remains same as A–A and B –B (b) Benzene and toluene
(c) Acetone and chloroform
(d) volume of solution is different from sum of volume of
(d) Chloroethane and bromoethane
solute and solvent
82. Which one of the following binary liquid systems shows
72. Relation between partial pressure and mole fraction is stated positive deviation from Raoult’s law?
by (a) Benzene-toluene
(a) Graham’s law (b) Raoult’s law (b) Carbon disulphide-acetone
(c) Le-Chatelier (d) Avogadro law (c) Phenol-aniline
73. Which one of the following is not correct for an ideal (d) Chloroform-acetone
solution? 83. A binary liquid solution is prepared by mixing n-heptane
(a) It must obey Raoult’s law and ethanol. Which one of the following statements is
correct regarding the behaviour of the solution?
(b) H 0
(a) The solution is non-ideal, showing negative deviation
(c) H V 0 from Raoult’s Law.
(d) All are correct (b) The solution is non-ideal, showing positive deviation
74. An ideal solution is formed when its components from Raoult’s Law.
(a) have no volume change on mixing (c) n-heptane shows positive deviation while ethanol
(b) have no enthalpy change on mixing shows negative deviation from Raoult’s Law.
(d) The solution formed is an ideal solution.
(c) Both (a) and (b) are correct 84. Which one is not equal to zero for an ideal solution:
(d) Neither (a) nor (b) is correct (a) Smix
75. All form ideal solution except (b) Vmix
(a) C6H6 and C6H5CH3 (b) C2H6 and C2H5I (c) P = Pobserved - PRaoult
(c) C6H5Cl and C6H5Br (d) C2H5 I and C2H5OH. (d) Hmix
EBD_7207
260 SOLUTIONS
85. A mixture of two completely miscible non-ideal liquids which 93. The value of P° for benzene is 640 mm of Hg. The vapour
distill as such without change in its composition at a pressure of solution containing 2.5gm substance in 39gm.
constant temperature as though it were a pure liquid. This benzene is 600mm of Hg the molecular mass of X is
mixture is known as (a) 65.25 (b) 130
(a) binary liquid mixture (b) azeotropic mixture (c) 40 (d) 75
(c) eutectic mixture (d) ideal mixture 94. The vapour pressure at a given temperature of an ideal
86. The azeotropic mixture of water (b.p.100°C) and HCl solution containing 0.2 mol of a non-volatile solute and 0.8
(b.p.85°C) boils at 108.5°C. When this mixture is distilled it mol of solvent is 60 mm of Hg. The vapour pressure of the
is possible to obtain pure solvent at the same temperature is
(a) pure HCl (a) 150 mm of Hg (b) 60 mm of Hg
(b) pure water (c) 75 mm of Hg (d) 120 mm of Hg
(c) pure water as well as pure HCl 95. 12 g of a nonvolatile solute dissolved in 108 g of water
(d) neither HCl nor H2O in their pure states produces the relative lowering of vapour pressure of 0.1.
87. The system that forms maximum boiling azeotrope is The molecular mass of the solute is
(a) carbondisulphide – acetone (a) 80 (b) 60
(b) benzene – toluene (c) 20 (d) 40
(c) acetone – chloroform 96. The amount of solute (molar mass 60 g.mol –1) that must be
(d) n-hexane – n-heptane added to 180 g of water so that the vapour pressure of water
88. Which one of the following binary mixtures forms an is lowered by 10% is
azeotrope with minimum boiling point type? (a) 30 g (b) 60 g
(a) acetone-ethanol (b) H2O-HNO3 (c) 120 g (d) 12 g
(c) benzene-toluene (d) n-hexane-n-heptane 97. The vapour pressure of water at 20°C is 17.5 mm Hg. If 18 g
89. On the basis of information given below mark the correct of glucose (C6H12O6) is added to 178.2 g of water at 20°C,
option. the vapour pressure of the resulting solution will be
(a) 17.325 mm Hg (b) 15.750 mm Hg
Information: On adding acetone to methanol some of the
(c) 16.500 mm Hg (d) 17.500 mm Hg
hydrogen bonds between methanol molecules break.
98. Which one of the following is a colligative property ?
(a) At specific composition methanol-acetone mixture will
form boiling azeotrope and will show positive deviation (a) Boiling point (b) Vapour pressure
from Raoult’s law. (c) Osmotic pressure (d) Freezing point
(b) At specific composition methanol-acetone mixture 99. Which one of the following aqueous solutions will exihibit
forms boiling azeotrope and will show positive highest boiling point ?
deviation from Raoult’s law. (a) 0.015 M urea (b) 0.01 M KNO3
(c) At specific composition methanol-acetone mixture will (c) 0.01 M Na2SO4 (d) 0.015 M glucose
form minimum boiling azeotrope and will show negative 100. The rise in the boiling point of a solution containing 1.8 g of
deviation from Raoult’s law. glucose in 100 g of solvent is 0.1°C. The molal elevation
constant of the liquid is
(d) At specific composition methanol-acetone mixture will
form boiling azeotrope and will show negative (a) 0.01 K/m (b) 0.1 K/m
deviation from Raoult’s law. (c) 1 K/m (d) 10 K/m
90. According to Raoult's law, relative lowering of vapour 101. For an electrolyte, elevation of B.P. is directly proportional to
pressure for a solution is equal to (a) molarity (b) molality
(a) moles of solute (b) moles of solvent (c) mole fraction (d) All of these
(c) mole fraction of solute (d) mole fraction of solvent 102. Which of the following aqueous solution has minimum
freezing point ?
91. The relative lowering of the vapour pressure is equal to the
(a) 0.01 m NaCl (b) 0.005 m C2H5OH
ratio between the number of
(c) 0.005 m MgI2 (d) 0.005 m MgSO4.
(a) solute molecules to the solvent molecules
103. 1.00 g of a non-electrolyte solute (molar mass 250 g mol–1)
(b) solute molecules to the total molecules in the solution
was dissolved in 51.2 g of benzene. If the freezing point
(c) solvent molecules to the total molecules in the solution
depression constant, Kf of benzene is 5.12 K kg mol –1, the
(d) solvent molecules to the total number of ions of the
freezing point of benzene will be lowered by
solute.
(a) 0.3 K (b) 0.5 K
92. Vapour pressure of benzene at 30°C is 121.8 mm Hg. When
(c) 0.4 K (d) 0.2
15 g of a non volatile solute is dissolved in 250 g of benzene
104. In a 0.2 molal aqueous solution of a weak acid HX the degree
its vapour pressure decreased to 120.2 mm Hg. The
of ionization is 0.3. Taking kf for water as 1.85, the freezing
molecular weight of the solute (Mo. wt. of solvent = 78)
point of the solution will be nearest to
(a) 356.2 (b) 456.8
(a) – 0.360ºC (b) – 0.260ºC
(c) 530.1 (d) 656.7
(c) + 0.481ºC (d) – 0.481ºC
SOLUTIONS 261
105. The molecular weight of benzoic acid in benzene as 116. Osmotic pressure of a solution at a given temperature
determined by depression in freezing point method (a) increases with concentration
corresponds to (b) decreases with concentration
(a) ionization of benzoic acid. (c) remains same
(b) dimerization of benzoic acid. (d) initially increases and then decreases
(c) trimerization of benzoic acid. 117. At 25°C, at 5 % aqueous solution of glucose (molecular
(d) solvation of benzoic acid. weight = 180 g mol–1) is isotonic with a 2% aqueous solution
106. A 0.5 molal solution of ethylene glycol in water is used as containing an unknown solute. What is the molecular weight
coolant in a car. If the freezing point constant of water be of the unknown solute?
1.86°C per mole, the mixture shall freeze at (a) 60 (b) 80
(a) 0.93°C (b) –0.93°C (c) 72 (d) 63
(c) 1.86°C (d) –1.86°C 118. Which one of the following statements is false?
107. A solution of urea (mol. mass 56 g mol 1) boils at 100.18 C (a) Raoult’s law states that the vapour pressure of a
at the atmospheric pressure. If Kf and Kb for water are component over a binary solution of volatile liquids is
1.86 and 0.512 K kg mol 1 respectively, the above solution directly proportional to its mole fraction
will freeze at (b) Two sucrose solutions of the same molality prepared
(a) 0.654 C (b) 0.654 C in different solvents will have the same depression of
(c) 6.54 C (d) 6.54 C freezing point
108. The freezing point of 1% solution of lead nitrate in water will (c) The correct order of osmotic pressures of 0.01 M
be solution of each compound is
(a) 2°C (b) 1°C BaCl2 > KCl > CH3COOH > glucose
(c) 0°C (d) below 0°C (d) In the equation osmotic pressure = MRT, M is the
109. A solution of sucrose (molar mass = 342 g mol–1) has been molarity of the solution
prepared by dissolving 68.5 g of sucrose in 1000 g of water. 119. Which of the following statements is false?
The freezing point of the solution obtained will be (a) Units of atmospheric pressure and osmotic pressure
(Kf for water = 1.86 K kg mol–1). are the same.
(a) – 0.372°C (b) – 0.520°C (b) In reverse osmosis, solvent molecules move through
(d) + 0.372°C (d) – 0.570°C a semipermeable membrane from a region of lower
110. A solution containing 1.8 g of a compound (empirical formula concentration of solute to a region of higher
CH2O) in 40 g of water is observed to freeze at –0.465° C. concentration.
The molecular formula of the compound is (c) The value of molal depression constant depends on
(Kf of water = 1.86 kg K mol–1) nature of solvent.
(a) C2H4O2 (b) C3H6O3 (d) Relative lowering of vapour pressure, is a
(c) C4H8O4 (d) C6H12O6 dimensionless quantity.
111. Blood cells retain their normal shape in solution which are 120. During osmosis, flow of water through a semipermeable
(a) hypotonic to blood (b) isotonic to blood membrane is
(c) hypertonic to blood (d) equinormal to blood. (a) from both sides of semipermeable membrane with equal
112. Isotonic solutions have same flow rates
(a) molar concentration (b) molality (b) from both sides of semipermeable membrane with
(c) normality (d) None of these unequal flow rates
113. As a result of osmosis, the volume of more concentrated (c) from solution having lower concentration only
solution (d) from solution having higher concentration only
(a) gradually decreases (b) gradually increases 121. If 0.1 M solution of glucose and 0.1 M solution of urea are
(c) is not affected (d) suddenly increases placed on two sides of the semipermeable membrane to
114. Which of the following pairs of solution are isotonic at the equal heights, then it will be correct to say that
same temperature ? (a) there will be no net movement across the membrane
(a) 0.1 M Ca(NO3)2 and 0.1 M Na2SO4 (b) glucose will flow towards urea solution
(b) 0.1 M NaCl and 0.1 M Na2SO4 (c) urea will flow towards glucose solution
(c) 0.1 M urea and 0.1 M MgCl2 (d) water will flow from urea solution to glucose
(d) 0.2 M urea and 0.1 M NaCl 122. The van’t Hoff factor i for a compound which undergoes
115. Osmotic pressure of 0.4% urea solution is 1.64 atm and that dissociation in one solvent and association in other solvent
of 3.42% cane sugar is 2.46 atm. When the above two is respectively
solutions are mixed, the osmotic pressure of the resulting (a) less than one and greater than one.
solution is : (b) less than one and less than one.
(a) 0.82 atm (b) 2.46 atm (c) greater than one and less than one.
(c) 1.64 atm (d) 4.10 atm (d) greater than one and greater than one.
EBD_7207
262 SOLUTIONS
123. If the various terms in the given below expressions have 132. The elevation in boiling point of a solution of 13.44 g of
usual meanings, the van’t Hoff factor (i) cannot be calculated CuCl2 in 1 kg of water using the following information will
by which one of the following expressions be (Molecular weight of CuCl2 = 134.4 g and Kb= 0.52 K
(a) V kg mol 1)
inRT (a) 0.16 (b) 0.05
(b) T f iK f .m (c) 0.1 (d) 0.2
(c) Tb iK b .m 133. Freezing point of an aqueous solution is
Psolvent Psolution – 0.186°C. If the values of Kb and Kf of water are respectively
n
(d) i 0.52 K kg mol–1 and 1.86 K kg mol–1, then the elevation of
Psolvent N n boiling point of the solution in K is
124. Van’t Hoff factor is given by the expression ________. (a) 0.52 (b) 1.04
(c) 1.34 (d) 0.052
Normal molar mass
(a) i
Abnormal molar mass STATEMENT TYPE QUESTIONS
Abnormal molar mass 134. Study the given statements and choose the correct option.
(b) i (i) 3.62 mass percentage of sodium hypochlorite in water
Normal molar mass
is used as commercial bleaching solution.
Observed colligative property (ii) 35% volume percentage of ethylene glycol is used as
(c) i an antifreeze (as coolent in car engines).
Calculated colligative property
(iii) Concentration of dissolved oxygen in a litre of sea
(d) Both (a) and (c) water is 5.8 ppm.
125. We have three aqueous solutions of NaCl labelled as 'A', 'B' (a) Statements (i) and (ii) are correct
and 'C' with concentrations 0.1M, 0.01M and 0.001M, (b) Statements (i) and (iii) are correct
respectively. The value of van't Hoff factor for these (c) Statements (ii) and (iii) are correct
solutions will be in the order ________. (d) Statements (i),(ii) and (iii) are correct
(a) iA < iB < iC (b) iA > iB > iC 135. Molarity and molality of a solution of NaOH is calculated.
(c) iA = iB = iC (d) iA < iB > iC If now temperature of the solution is increased then which
126. If is the degree of dissociation of Na 2SO4, the Vant Hoff’s of the following statement(s) is/are correct ?
factor (i ) used for calculating the molecular mass is (i) Molarity of solution decreases
(a) 1 + (b) 1 – (ii) Molality of the solution increases
(c) 1 + 2 (d) 1 – 2 (a) Both statements are correct
127. The freezing point of equimolal aqueous solutions will be (b) Statement (i) is correct only
highest for (c) Statement (ii) is correct only
(d) Both statements are incorrect.
(a) C6H5NH3+Cl– (aniline hydrochloride)
136. “If temperature increases solubility of gas decreases”. For
(b) Ca(NO3)2 this situation which of the following statement(s) is/are
(c) La(NO3)3 correct ?
(d) C6H12O6 (glucose) (i) Reaction is endothermic
128. The correct relationship between the boiling points of very (ii) Le-chatelier’s principle can be applied
dilute solutions of AlCl3 (t1) and CaCl2 (t2), having the (a) Statement (i) and (ii) both are correct
same molar concentration is (b) Statement (i) is correct only
(a) t1 = t2 (b) t1 > t2 (c) Statement (ii) is correct only
(c) t2 > t1 (d) t2 t1 (d) Both statement(s) (i) and (ii) are incorrect
129. At 25°C, the highest osmotic pressure is exhibited by 0.1 M 137. Read the following statements carefully and choose the
solution of correct option.
(a) CaCl2 (b) KCl (i) Different gases have different KH values at the same
(c) glucose (d) urea temperature.
(ii) Higher the value of KH at a given temperature, lower
130. Which one of the following salts will have the same value of
is the solubility of the nature of gas in the liquid.
van’t Hoff factor (i) as that of K4[Fe(CN)6].
(iii) KH is a function of the nature of the gas.
(a) Al2(SO4)3 (b) NaCl (c)
(iv) Solubility of gases increases with increase of
Al(NO3)3 (d) Na2SO4. temperature.
131. Which of the following 0.10 m aqueous solutions will have (a) (i), (ii) and (iv) are correct.
the lowest freezing point ? (b) (ii) and (iv) are correct.
(a) Al2(SO4)3 (b) C6H12O6 (c) (i), (ii) and (iii) are correct.
(c) KCl (d) C12 H22 O11 (d) (i) and (iv) are correct.
SOLUTIONS 263
138. Read the following statements and choose the correct 142. Read the following statements carefully and choose the
option. correct option
(i) Polar solutes dissolve in a polar solvent. (i) Osmotic pressure is not a colligative property.
(ii) Polar solutes dissolve in a non-polar solvent. (ii) For dilute solutions, osmotic pressure is proportional
(iii) Non-polar solutes dissolve in a non-polar solvent. to the molarity, C of the solution at a given temperature
(iv) Non-polar solutes dissolve in a polar solvent. T.
(a) (i) and (ii) are correct. (iii) During osmosis ,solvent molecules always flow from
higher concentration to lower concentration of
(b) (i), (ii) and (iii) are correct.
solution.
(c) (i) and (iii) are correct. (iv) The osmotic pressure has been found to depend on
(d) (ii) and (iv) are correct. the concentration of the solution
139. Read the following statements carefully and choose the (a) (i), (ii) and (iv) are correct
correct option (b) (ii) and (iv) are correct
(i) The vapour pressure of a liquid decreases with (c) (iii), and (iv) are correct
increase of temperature. (d) (i), (ii) and (iii) are correct
(ii) The liquid boils at the temperature at which its vapour
pressure is equal to the atmospheric pressure. MATCHING TYPE QUESTIONS
(iii) Vapour pressure of the solvent decreases in the
presence of non-volatile solute. 143. Match the columns
(iv) Vapour pressure of the pure solvent and solution is a Column -I Column-II
function of temperature. (A) Mass percentage (p) Medicine and pharmacy
(a) (i), (ii) and (iv) are correct (B) Mass by volume (q) Concentration of
(b) (i), (iii), and (iv) are correct pollutants in water
(c) (ii), (iii), and (iv) are correct (C) ppm (r) Industrial chemical
application
(d) (i), (ii) and (iii) are correct
(D) Volume percentage (s) Liquid solutions
140. On the basis of information given below mark the correct
(a) A – (q), B – (p), C – (s), D – (r)
option.
(b) A – (s), B – (r), C – (p), D – (q)
(i) In bromoethane and chloroethane mixture
intermolecular interactions of A-A and B-B type are (c) A – (r), B – (q), C – (s), D – (p)
nearly same as A-B type interactions. (d) A – (r), B – (p), C – (q), D – (s)
(ii) In ethanol and acetone mixture A-A or B-B type 144. Match the columns
intermolecular interactions are stronger than A-B type Column-I Column-II
interactions. (A) Na-Hg Amalgam (p) gas - solid
(iii) In chloroform and acetone mixture A-A or B-B type (B) H2 in Pd (q) gas - liquid
intermolecular interactions are weaker than A-B type (C) Camphor in nitrogen gas (r) liquid - solid
interactions. (D) Oxygen dissolved in water (s) solid - gas
(a) Solution (ii) and (iii) will follow Raoult's law. (a) A – (q), B – (s), C – (r), D – (p)
(b) Solution (i) will follow Raoult's law. (b) A – (t), B – (p), C – (q), D – (s)
(c) Solution (ii) will show negative deviation from Raoult's (c) A – (r), B – (p), C – (s), D – (q)
law. (d) A – (s), B – (q), C – (p), D – (p)
(d) Solution (iii) will show positive deviation from Raoult's 145. Match the Column I, II & III and choose the correct option.
law. Column-I Column-II Column-III
141. Which observation(s) reflect(s) colligative properties? (A) Gaseous (p) Solid-liquid (h) Copper dissolved
(i) A 0.5 m NaBr solution has a higher vapour pressure solutions in gold
than a 0.5 m BaCl2 solution at the same temperature (B) Liquid (q) Solid-solid (i) Chloroform mixed
(ii) Pure water freezes at the higher temperature than pure solutions with nitrogen
methanol (C) Solid (r) Liquid-gas (j) Common salt
(iii) a 0.1 m NaOH solution freezes at a lower temperature solutions dissolved in water
than pure water (a) (A) – (r) – (h), (B) – (r) – (i), (C) – (p) – (j)
Choose the correct answer from the codes given below (b) (A) – (r) – (i), (B) – (p) – (j), (C) – (q) – (h)
(a) (i), (ii) and (iii) (b) (i) and (ii) (c) (A) – (r) – (j), (B) – (p) – (h), (C) – (q) – (i)
(c) (ii) and (iii) (d) (i) and (iii) (d) (A) – (r) – (j), (B) – (q) – (i), (C) – (p) – (h)
EBD_7207
264 SOLUTIONS
146. Match the columns 151. Assertion : Azeotropic mixtures are formed only by
Column-I Column-II non-ideal solutions and they may have boiling points either
(A) Saturated solution (p) Solution having same greater than both the components or less than both the
osmotic pressure at a given components.
temperature as that of given Reason : The composition of the vapour phase is same as
solution. that of the liquid phase of an azeotropic mixture.
(B) Binary solution (q) A solution whose osmotic 152. Assertion : When methyl alcohol is added to water, boiling
pressure is less than that point of water increases.
of another. Reason : When a volatile solute is added to a volatile solvent
(C) Isotonic solution (r) Solution with two elevation in boiling point is observed.
components 153. Assertion : When NaCl is added to water a depression in
(D) Hypotonic solution (s) A solution which contains freezing point is observed.
maximum amount of solute Reason : The lowering of vapour pressure of a solution
that can be dissolved in a causes depression in the freezing point.
given amount of solvent at 154. Assertion : When a solution is separated from the pure
a given temperature. solvent by a semi- permeable membrane, the solvent
(a) A – (s), B – (r), C – (p), D – (q) molecules pass through it from pure solvent side to the
(b) A – (r), B – (p), C – (s), D – (q) solution side
(c) A – (r), B – (s), C – (q), D – (p) Reason : Diffusion of solvent occurs from a region of high
(d) A – (q), B – (p), C – (r), D – (s) concentration solution to a region of low concentration
147. Match the laws given in the Column-I with expression given solution.
in Column-II.
Column-I Column-II CRITICAL THINKING TYPE QUESTIONS
(A) Raoult’s law (p) Tf = Kfm 155. The normality of orthophosphoric acid having purity of 70
(B) Henry’s law (q) = CRT % by weight and specific gravity 1.54 is
o o (a) 11 N (b) 22 N
(C) Elevation of boiling point (r) p x1p1 x 2 p 2
(D) Depression in freezing point (s) Tb = Kbm (c) 33 N (d) 44 N
156. Which of the following statements, regarding the mole
(E) Osmotic pressure (t) p = KH.x fraction (x) of a component in solution, is incorrect?
(a) A – (r), B – (t), C – (s), D – (p), E – (q)
(a) 0 x 1
(b) A – (t), B – (r), C – (q), D – (s), E – (p)
(c) A – (p), B – (t), C – (r), D – (q), E – (s) (b) x 1
(d) A – (s), B – (p), C – (q), D – (r), E – (t) (c) x is always non-negative
(d) None of these
ASSERTION-REASON TYPE QUESTIONS 157. Which one of the following gases has the lowest value of
Henry’s law constant?
Directions : Each of these questions contain two statements, (a) N2 (b) He
Assertion and Reason. Each of these questions also has four (c) H2 (d) CO2
alternative choices, only one of which is the correct answer. You 158. Equal masses of methane and oxygen are mixed in an empty
have to select one of the codes (a), (b), (c) and (d) given below. container at 25°C. The fraction of the total pressure exerted
(a) Assertion is correct, reason is correct; reason is a correct by oxygen is
explanation for assertion. (a) 1/2 (b) 2/3
(b) Assertion is correct, reason is correct; reason is not a 1 273
correct explanation for assertion (c) (d) 1/3
3 298
(c) Assertion is correct, reason is incorrect 159. When a gas is bubbled through water at 298 K, a very dilute
(d) Assertion is incorrect, reason is correct. solution of the gas is obtained. Henry’s law constant for
148. Assertion : Molarity of a solution in liquid state changes the gas at 298 K is 100 kbar. If the gas exerts a partial pressure
with temperature. of 1 bar, the number of millimoles of the gas dissolved in
Reason : The volume of a solution changes with change in one litre of water is
temperature. (a) 0.555 (b) 5.55
149. Assertion : If a liquid solute more volatile than the solvent (c) 0.0555 (d) 55.5
is added to the solvent, the vapour pressure of the solution 160. KH value for Ar(g), CO2(g), HCHO (g) and CH4(g) are 40.39,
may increase i.e., ps > po. 1.67, 1.83 × 10–5 and 0.413 respectively.
Reason : In the presence of a more volatile liquid solute, Arrange these gases in the order of their increasing
only the solute will form the vapours and solvent will not. solubility.
150. Assertion : If one component of a solution obeys Raoult’s (a) HCHO < CH4 < CO2 < Ar
law over a certain range of composition, the other component (b) HCHO < CO2 < CH4 < Ar
will not obey Henry’s law in that range. (c) Ar < CO2 < CH4 < HCHO
Reason : Raoult’s law is a special case of Henry’s law. (d) Ar < CH4 < CO2 < HCHO
SOLUTIONS 265
161. What is the ratio of no. of moles of nitrogen to that of (a) 35.67 g (b) 356.7 g
oxygen in a container of 5 litre at atmospheric pressure? (c) 432.8 g (d) 502.7 g
(a) 1 : 1.71 (b) 1 : 2 170. For a dilute solution containing 2.5 g of a non-volatile non-
(c) 2 : 1 (d) 1 : 24 electrolyte solute in 100 g of water, the elevation
162. Consider a and b are two components of a liquid mixture, in boiling point at 1 atm pressure is 2°C. Assuming
their corresponding vapour pressure (mmHg) are concentration of solute is much lower than the concentration
respectively 450 and 700 in pure states and total pressure of solvent, the vapour pressure (mm of Hg) of the solution
given is 600. Then corresponding composition in liquid is (take Kb = 0.76 K kg mol–1)
phase will be (a) 724 (b) 740
(a) 0.4, 0.6 (b) 0.5, 0.5 (c) 736 (d) 718
(a) 0.6, 0.4 (d) 0.3, 0.7 171. The difference between the boiling point and freezing point
163. Which will form maximum boiling point azeotrope of an aqueous solution containing sucrose (molecular wt =
(a) HNO3 + H2O solution (b) C2H5OH + H2O solution 342 g mole–1 ) in 100 g of water is 105°C. If Kf and
(c) C6H6 + C6H5CH3 solution (d) None of these Kb of water are 1.86 and 0.51 K kg mol –1 respectively, the
164. If two liquids A and B form minimum boiling azeotrope at weight of sucrose in the solution is about
some specific composition then _______. (a) 34.2 g (b) 342 g
(a) A – B interactions are stronger than those between
(c) 7.2 g (d) 72 g
A – A or B – B
172. If the elevation in boiling point of a solution of non-volatile,
(b) vapour pressure of solution increases because more
non-electrolytic and non-associating solute in a solvent
number of molecules of liquids A and B can escape
(Kb = x K kg mol–1) is y K, then the depression in freezing
from the solution.
point of solution of same concentration would be (Kf of the
(c) vapour pressure of solution decreases because less
solvent = z K kg mol–1)
number of molecules of only one of the liquids escape
from the solution 2xz yz
(d) A – B interactions are weaker than those between A – (a) (b)
y x
A or B – B
165. Chloroform and acetone are added to each other, Raoult’s xz yz
law shows negative deviation.what does this suggests ? (c) (d)
y 2x
(a) Exothermic reaction
(b) Endothermic reaction 173. 1 g of a non-volatile, non-electrolyte solute of molar mass
(c) Zero change in enthalpy 250 g/mol was dissolved in 51.2 g of benzene. If the
(d) None of these freezing point depression constant Kf of benzene is 5.12
166. At 300 K the vapour pressure of an ideal solution containing kg K mol–1. The freezing point of benzene is lowered by
1 mole of liquid A and 2 moles of liquid B is 500 mm of Hg. (a) 0.3 K (b) 0.5 K
The vapour pressure of the solution increases by 25 mm of (c) 0.2 K (d) 0.4 K
Hg, if one more mole of B is added to the above ideal solution 174. The boiling point of 0.2 mol kg–1 solution of X in water is
at 300 K. Then the vapour pressure of A in its pure state is greater than equimolal solution of Y in water. Which one of
(a) 300 mm of Hg (b) 400 mm of Hg the following statements is true in this case ?
(c) 500 mm of Hg (d) 600 mm of Hg (a) Molecular mass of X is greater than the molecular mass
167. Someone has added a non electrolyte solid to the pure of Y.
liquid but forgot that among which of the two beakers he (b) Molecular mass of X is less than the molecular mass of
has added that solid. This problem can be solved by Y.
checking (c) Y is undergoing dissociation in water while X
(a) relative lower in vapour pressure undergoes no change.
(b) elevation in boiling point (d) X is undergoing dissociation in water.
(c) depression in Freezing point 175. What is the freezing point of a solution containing 8.1 g
(d) all above HBr in 100 g water assuming the acid to be 90% ionised ?
168. The vapour pressure of a solvent decreases by 10 mm of Hg (Kf for water = 1.86 K kg mol–1) :
when a non-volatile solute was added to the solvent. The (a) 0.85°K (b) – 3.53°K
mole fraction of the solute in the solution is 0.2. What should
(c) 0°K (d) – 0.35°K
be the mole fraction of the solvent if the decrease in the
176. An 1% solution of KCl (I), NaCl (II), BaCl2 (III) and urea (IV)
vapour pressure is to be 20 mm of Hg ?
have their osmotic pressure at the same temperature in the
(a) 0.8 (b) 0.6
ascending order (molar masses of NaCl, KCl,
(c) 0.4 (d) 0.2
169. Vapour pressure of benzene at 30°C is 121.8 mm. When 15g BaCl2 and urea are respectively 58.5, 74.5, 208.4 and
of a non-volatile solute is dissolved in 250 g of benzene, its 60 g mole–1). Assume 100% ionization of the electrolytes at
vapour pressure is decreased to 120.2 mm. The molecular this temperature
weight of the solute is (a) I < III < II < IV (b) III < I < II < IV
(c) I < II < III < IV (d) III < IV < I < II
EBD_7207
266 SOLUTIONS

FACT / DEFINITION TYPE QUESTIONS Number of moles


14. (c) Molarity
Volume of solution (L)
1. (d)
2. (c) Sodium fluoride is used as rat poison. Mass 1000
Moles of water 55.6
3. (b) Almost all the processes in our body occur in liquid Molar mass 18
solution.
4. (c) In homogeneous mixtures composition and properties 55.6
Molarity = = 55
both are uniform throughout the mixture. 1
5. (a) Hexane is not water soluble, hence solution is not 15. (b) 10 g glucose is dissolved in = 100 ml solution.
formed. 180 g (g mole) is dissolved in
6. (d) Dilute, concentrated and saturated terms are 100
qualitative methods of description of concentration = 180 = 1800 ml = 1.8 L
of solution whereas molar or molarity is quantitative 10
method. Mass of solute
16. (d) ppm 106
7. (d) Mass of solution
8. (b) Density = 1.17 gm/cc (Given)
Mass 10
As d = ppm 106 10 ppm
Volume 106
volume = 1cc mass = d = 1.17g p p
17. (a) xsolute
No. of moles 1.17 ´1000 p
Molarity = =
Volume in litre 36.5 ´1 Given, p° = 0.8 atm, p = 0.6 atm, xsolute = ?
1170 0.8 0.6 0.2
= =32.05M xsolute or xsolute ,
36.5 0.8 0.8
9. (c) From molarity equation or xsolute = 0.25
M1V1 + M2V2 = M3(V1 + V2)
1× 2.5 + 0.5 × 3 = M3 × 5.5 No. of moles of solute 5
18. (b) Molarity 2M
4 Volume in litres 2.5
M3 = = 0.73M 19. (b) One molal solution means one mole of solute is present
5.5
in 1 kg (1000 g) solvent
10. (b) Relation between molality and mole fraction is
i.e., mole of solute = 1
1000 x2 1000 .2
m 3.2 1000g 1000
x1M1 0.8 78 Mole of solvent (H2O) = =
Thus, X (m) = 3.2 18g 18
Wt 1000 7.1 1000 1
11. (b) M ; M 0.5M 18
M.Wt . V 142 100 Mole fraction of solute = = = 0.018.
1000 1008
p n2 n2 1
640 630 0.0156 18
12. (a) or
p n1 n2 n1 n2 640 1
20. (d) NV = N1V1 + N2V2 + N3V3
or 1 mole i.e., 78g benzene contains solute = 0.0156 mol
1 1 1
or, 1000N 1 5 20 30 or N .
0.0156 103 2 3 40
Molality of solution = 0.2 m
78 21. (a) For HCl
Moles of solute M = N = 0.1
13. (d) Normality N1V1 = N2V2 ; 25 × N1 = 0.1 × 35
Volume of solution in litre
0.1 35 0.1 35
Given mass of solute = 4.0 g N1 ; M 0.07 .
No. of moles of NaOH = 4/40 25 25 2
Volume of solution = 100 ml = 100/1000 L 22. (b) 1 molal solution means 1 mole of solute dissolved in
1000 gm solvent.
4 / 40 nsolute = 1 wsolvent = 1000 gm
Normality 1N
100 /1000
SOLUTIONS 267

1000 33. (a)


nsolvent = = 55.56 34. (c) In a given solution sum of all the mole fraction is unity
18
i.e.,
1 (a) x1 + x2 + x3 + x4 = 1
xsolute = = 0.0177
1 55.56
n3
23. (d) H3PO4 is tribasic so N = 3M = 3 × 1N = 3N. (b) n n x3
24. (c) N1V1 + N2V2 = NV 1 2 n3 n 4
4x + 10 (1 – x) = 6 × 1; –6x = –4 ; x = 0.67 35. (d) Volume is temperature dependent, hence expression
Thus 0.67 litre of 4N HCl involving volume term (normality, molarity and
1 – x = 1 – 0.67 = 0.33 litre of 10 N HCl formality) varies with temperature
Molarity
25. (c) Molality (m) geq of solute
Molarity Molecular mass Normality =
Density vol. of solution in L
1000
moles of solute
18 Molarity =
5000 vol. of solution in L
18 98
1.8
1000 formula mass
26. (d) No. of millimoles = 500 × 0.2 = 100 Formality =
vol. of solution in L
Thus, molarity of diluted solution
moles of solute
100 Molality =
mass of solvent in kg
700
(Molarity = No. of moles L–1 = No. of millimoles mL–1) Since molality does not include the volume term, it is
independent of temperature.
0.1428 M
36. (b) An increase in temperature of the solution increases
wt 1000 the solubility of a solid solute.
27. (d) Molarity (M) =
mol. wt. vol (ml) The amount of solute that dissolve depends on what
type of solute it is.
wt. 1000 For solids and liquid solutes, changes in pressure have
2
63 250 practically no effect on solubility.
63 37. (d)
wt. = gm 38. (b) Number of solute particles going into solution will be
2
equal to the solute particles separating out and a state
100 of dynamic equilibrium is reached.
wt. of 70% acid = 31.5 = 45 gm
70 solute + solvent solution.
28. (b) More than theoretical weight since impurity will not i.e., rate of dissolution = rate of crystallization.
contribute. 39. (d) The maximum amount of solute dissolved in a
29. (c) Applying the law of equivalence, given amount of solvent is its solubility.
N1V1 N 2V2 N3V3 N RVR 40. (d)
N N N 41. (b) According to Le-chateliers principle, for an exother
50 30 10 N R 1000
10 3 2 mic reaction ( < 0) increase in temperature decreases
5 N 10 N 5 N = 1000 N R the solubility.
N 42. (c) On increasing the pressure over the solution phase
NR
50 by compressing the gas to a smaller volume (in fig b)
30. (c) 40 g NaOH = 1 mole increase the number of gaseous particles per unit
moles of the solute volume over the solution and also the rate at which
1
molality = = 1 molal the gaseous particles are striking the surface of
mass of the solvent in kg 1
solution to enter it. The solubility of the gas will
31. (b) Both molality and mole fraction are not related to the increase until a new equilibrium is reached resulting
volume of solution, thus they are both independent of in an increase in the pressure of a gas above the
temperature. solution and thus its solubility increases.
32. (a) Given w = 10 g Mol. mass = 40
43. (d)
Weight of solvent = 1250 × 0.8 g = 10000 g = 1 kg
44. (a) According to Henry’s law at a constant temperature,
10 the solubility of a gas in a liquid is directly proportional
molality 0.25
40 1 to the pressure of the gas.
EBD_7207
268 SOLUTIONS
45. (d) According to Henry’s law, 62. (a) For example, decrease in the vapour pressure of water
m=k×p by adding 1.0 mol of sucrose to one kg of water is
given KH = 1.4 × 10–3 nearly similar to that produced by adding 1.0 mol of
pO2 = 0.5 or urea to the same quantity of water at the same
temperature.
pO2 KH x O2
63. (d)
0.5 64. (a) Given V.PP = 80 torr
x O2 3
1.4 10 V.PQ = 60 torr
m Ptotal = V·PP × xp + V·Pq × xq
No. of moles; n
M
0.7 10 4
m 3 2
32 = 80 60 = 16 × 3 + 12 × 2
5 5
4
m 22.4 10 g 2.24 mg Ptotal = 48 + 24 = 72 torr
46. (c)
47. (b) [ Hint : If added substance dissolves, the solution is 65. (b) Moles of glucose 18
0. 1
unsaturated. If it does not dissolve solution is 180
saturated. If precipitation occurs solution is
supersaturated.] Moles of water 78.2
9.9
48. (c) 18
49. (b) Body temperature of human beings remains constant. Total moles = 0.1 + 9.9 = 10
50. (a) 51. (b) 52. (a)
53. (c) The partial pressure of the gas in vapour phase (p) is pH2O = Mole fraction × Total pressure 9.9
760
proportional to the mole fraction of the gas (x). 10
p = KH .x = 752.4 Torr
Where KH is Henry’s constant. 66. (d) p = pAxA + pBxB
54. (d) To increase the solubility of CO2 in soft drinks and = pAxA + pB (1 – xA)
soda water, the bottle is sealed under high pressure. pAxA + pB – pBxA
55. (d) Scuba divers must cope with high concentrations of pB+ xA (pA – pB)
dissolved gases while breathing air at high pressure
underwater. Increased pressure increases the solubility 67. (b) pA ? , Given p B 200 mm of Hg, xA = 0.6,
of atmospheric gases in blood. When the divers come
towards surface, the pressure gradually decreases. xB = 1 – 0.6 = 0.4, P = 290 of Hg
This releases the dissolved gases and leads to the P = PA + PB = PA x A PB xB
formation of bubbles of nitrogen in the blood. This
blocks capillaries and creates a medical condition 290 = PA × 0.6 + 200 × 0.4 p A = 350 mm of Hg.
known as bends, which are painful and dangerous to
life. To avoid bends, as well as, the toxic effects of 68. (b) ptotal pA xA pB x B
high concentrations of nitrogen in the blood, the tanks
1 3
used by scuba divers are filled with air diluted with 550 pA p
helium (11.7% helium, 56.2% nitrogen and 32.1% 4 B 4
oxygen). pA 3 pB 550 4 ...(i)
56. (a) At high altitudes the partial pressure of oxygen is In second case
less than that at the ground level. This leads to low
1 4
concentrations of oxygen in the blood and tissues of ptotal pA pB
people living at high altitudes or climbers. Low blood 5 5
oxygen causes climbers to become weak and unable pA 4 pB 560 5 ...(ii)
to think clearly, symptoms of a condition known as Subtract (i) from (ii)
anoxia.
pB 560 5 550 4 600
57. (c)
58. (a) Raoult’s law becomes special case of Henry’s law when pA 400
KH become equal to p °. 69. (d) According to given information
1
59. (c) 60. (d) pX = 80 Torr
61. (b) In the solution, the surface has both solute and solvent pY = 60 Torr
molecules; thereby the fraction of the surface covered nX = 3 moles
by the solvent molecules gets reduced. Consequently, nY = 2 moles
the number of solvent molecules escaping from the
surface is correspondingly reduced, thus, the vapour nX 3 3
mole fraction of X(xX)
pressure is also reduced. nX nY 3 2 5
SOLUTIONS 269
On adding benzene, its molecules come between
nY 2 2
mole fraction of Y(xY) ethanol molecules there by breaking H-bonds which
nX nY 3 2 5 weaken intermolecular forces. This results in increase
Total Pressure, P = pXxX + pYxY in vapour pressure.
3 2 80. (a) A solution of acetone in ethanol shows positive
80 60 = 48 + 24 = 72 Torr.. deviation from Raoult's law. It is because ethanol
5 5
molecules are strongly hydrogen bonded. When
70. (a) Total vapour pressure = vapour pressure of pure acetone is added, these molecules break the hydrogen
benzene + vapour pressure of toluene bonds and ethanol becomes more volatile. Therefore
= 100 + 50 = 150 mm its vapour pressure is increased.
We know, 81. (a) Acetone and chloroform shows negative deviation
PC6H6 P xC6 H6 from Raoult's law when these are mixed, the hydrogen
bonding takes place between the two molecular species
100 150 xC6 H 6 due to which escaping tendency of either liquid
molecules becomes less and boiling point of solution
100 increases.
xC6 H 6 0.67
150 82. (b)
71. (c) These two components A and B follows the condition 83. (b) For this solution intermolecular interactions between
of Raoult’s law if the force of attraction between A and n-heptane and ethanol are weaker than n-heptane-n-
B is equal to the force of attraction between A and A or heptane & ethanol-ethanol interactions hence the
B and B. solution of n-heptane and ethanol is non-ideal and
72. (b) According to Raoult’s law “The partial pressure of a shows positive deviation from Raoult’s law.
volatile component of a solution is directly proportional 84. (a) For an ideal solution Smix > 0
to its mole fraction in solution at any temperature”. 85. (b)
p = Px 86. (d) Azeotropic mixture is constant boiling mixture, it is not
where, p = Partial pressure of component possible to separate the components of azeotropic
P° = Vapour pressure of component in pure form mixture by boiling.
x = mole fraction of component in solution. 87. (c)
73. (c) For an ideal solution, H 0 , V 0 88. (a) Minimum boiling azeotrope is formed by solution
Hence, option (c) is incorrect. showing positive deviation. e.g. acetone – ethanol.
74. (c) For ideal solution, 89. (b)
Vmixing 0 and H mixing 0. 90. (c) Relative lowering of vapour pressure depends upon
the mole fraction of solute.
75. (d) C2H5I and C2H5OH form non-ideal solution.
P P
76. (d) i.e., mole fraction of solute
77. (a) On adding acetone, its molecules get in between the P
host molecules and break some of the hydrogen bonds 91. (b) According to Raoult's law, the relative lowering in
between them. Due to weakening of interactions, the vapour pressure of a dilute solution is equal to the mole
solution shows positive deviation from Raoult's law. fraction of the solute present in the solution.
78. (d) A solution containing A and B components shows p p n
negative deviation when A–A and B–B interactions Mole fraction of solute
p n N
are weaker than that of A–B interactions. For such
92. (a) Given vapour pressure of pure solvent
solutions.
H = –ve and V = –ve (P°) = 121.8 mm Hg; Weight of solute (w) = 15 g
79. (b) Positive deviations are shown by such solutions in Weight of solvent (W) = 250 g; Vapour pressure of
which solvent-solvent and solute-solute interactions solution (P) = 120.2 mm Hg and Molecular weight of
are stronger than the solute-solvent interactions. In solvent (M) = 78
such solution, the interactions among molecules From Raoult’s law
becomes weaker. Therefore their escaping tendency Po P w M
increases which results in the increase in their partial o
P m W
vapour pressures.
In pure methanol there exists intermolecular 121.8 120.2 15 78
H–bonding. 121.8 m 250
---O – H--- O—H --- O—H--- 15 78 121.8
or m 356.2
| | | 250 1.6
CH3 CH3 CH3
EBD_7207
270 SOLUTIONS

P P n2 0.1 180 100


93. (d) 100. (c) Kb 1K/m
P n1 n2 1.8 1000
101. (b) Tb = Kb × i × m
640 600 2.5 x
Where Tb = Elevation in boiling point
640 39 78 Kb = molal elevation constant
640 78 2.5 i = vant Hoff factor
x 80
39 40 Tb molality..
94. (c) According to Raoult's law 102. (a) Tf = i × Kf × m
p p Van't Hoff factor, i = 2 for NaCl, m = 0.01
xB
p hence Tf = 0.02 Kf which is maximum in the present
case.
.2 1 Hence Tf is maximum or freezing point is minimum.
xB Mole fraction of solute
.2 .8 5 103. (c)
p 60 mm of Hg K f W2 1000 1 1000
T Kfm 5.12 0.4K
p 1 p M 2W1 250 51.2
or 4 p ( p) 5
p 5
104. (d) ( HX H X , i = 1.3); Tf = Kf × m × i
60´5
Þ p° = = 75mm of Hg 1 0.3 0. 3 0.3
4 Tf = 1.85 × 0.2 × 1.3 = 0.481º C
o n w M
P Ps Tf = T f T f = 0 – 0.481ºC = – 0.481ºC
95. (c) =
Po N m W 105. (b) Benzoic acid exists as dimer in benzene.
12 18 106. (b) T f K f m 1.86 0.5 0.93 C; T f 0.93 C
0.1 =
m 108
107. (b) As Tf = Kf. m
12 18 Tb = Kb. m
m= 20
0.1 108
Tf Tb
P P w2 M1 Hence, we have m
96. (b) 100 100 10 Kf Kb
P M2 w1
Kf
w 2 18 or Tf Tb
100 10 or w2 = 60 g Kb
60 180
Thus, 60 g of the solute must be added to 180 g of [ Tb 100.18 100 0.18 C]
water so that the vapour pressure of water is lowered
1.86
by 10%. = 0.18 × = 0.654°C
97. (a) The vapour pressure of a solution of glucose in water 0.512
can be calculated using the relation As the Freezing Point of pure water is 0°C,
p ps Tf = 0 –Tf
Moles of glucose in solution
0.654 = 0 – Tf
ps Moles of water in solution Tf = – 0.654
17.5 ps 18/180 Thus the freezing point of solution will be – 0.654°C.
or [ p 17.5 ] 108. (d) Addition of solute to water decreases the freezing point
ps 178.2/18
of water (pure solvent).
0.1 ps When 1% lead nitrate (solute) is added to water, the
or 17.5 – ps or ps = 17.325 mm Hg. freezing point of water will be below 0°C.
9.9
Hence (a) is correct answer.
1000W2 1.86×1000×68.5
98. (c) Osmotic pressure is a colligative property. 109. (a) Tf Kf = = 0.372
M 2W1 342×1000
99. (c) Tb Kb m
Elevation in boiling point is a colligative property, Tf = T °f – Tf
which depends upon the no. of particles Tf = – 0.372°C
(concentration of solution). Thus greater the number 110. (d) Tf = Kf × m
of particles, greater is the elevation in boiling point 1000 K f w 2 solute
and hence greater will be its boiling point. M
Tf w1 solvent
Na2SO4 2Na+ + SO42–
Since Na2SO4 has maximum number of particles (c) 1000 1.86 1.8
M = 180
hence has maximum boiling point. 0.465 40
SOLUTIONS 271
Molecular formula = (empirical formula)n 124. (d) 125. (c)
Molecular mass 180 126. (c) Na2SO4 2Na+ + SO 24
n 6
Empirical formula mass 30 Mol. before dissociation 1 0 0
Molecular formula = (CH2O)6 = C6H12O6. Mol. after dissociation 1 – 2 1
111. (b) Blood cells neither swell nor shrink in isotonic solution. i 1 2 1 2
As isotonic solutions have equal concentration 127. (d) The salt that ionises to least extent will have highest
therefore there is no flow of solvent occurs and hence freezing point. [ i.e., minimum Tf ]
solvent neither enters nor flow out of the blood cells. 128. (b) AlCl3 furnishes more ions than CaCl2 and thus possess
112. (a) Isotonic solutions have same molar concentration at higher boiling point i.e., t1 > t2.
given temperature provided the Van't Hoff factor (i) is 129. (a) Concentration of particles in CaCl2 solution will be
same. maximum as i = 3 for CaCl2 and i = 2 for KCl.
113. (b) Glucose and Urea do not dissociate into ions, as they
114. (a) The solution which provide same number of ions are are nonelectrolytes.
isotonic.
130. (a) K 4 [Fe(CN) 6 ] and Al 2(SO 4 )3 both dissociates to
Ca (NO3)2 Ca2+ + 2 NO3–
give 5 ions or i = 5
Total ions produced = 3
K4 [Fe(CN)6] 4K+ + [Fe(CN)6]4–
Na2SO4 2 Na+ + SO42–
Total ions produced = 3 and Al2 (SO4 )3 2Al3+ + 3SO 4--
0.1 M Ca (NO3)2 and 0.1 M Na2SO4 are isotonic. 131. (a) Depression in freezing point No. of particles.
115. (d) Osmotic pressure is a colligative property. Hence (when concentration of different solutions is equal)
resulting osmotic pressure of the solution is given by Al 2 (SO4 )3 provides five ions on ionisation
T = 1 + 2 + 3 ..........
Al 2 (SO 4 ) 3 2 Al 3 3SO 42 –
T = 1.64 + 2.46 = 4.10 atm.
116. (a) According to Boyle-van't Hoff law, while KCl provides two ions
C (at constant temp) KCl ¾¾ ® K + + Cl-
117. (c) Isotonic solutions have same osmotic pressure C6 H12 O6 and C12 H22 O11 are not ionised so they
glucose = unknown solute
have single particle in solution.
Hence, Al2(SO4)3 have maximum value of depression
m1 m2 5 2
or M2 72 in freezing point or lowest freezing point.
M1 M2 180 M2
118. (b ) Tf = Kf × m No. of particles after ionisation
132. (a) (i) i
Kf is a characteristic of a particular solvent i.e., it will No. of particles before ionisation
be different for different solvents. (ii) Tb = i × Kb × m
119. (b)
120. (c) During osmosis water flows through semipermeable CuCl2 Cu2+ + 2Cl
1 0 0
membrane from lower concentration to higher
concentration. (1 ) 2
121. (a) As both the solutions are isotonic hence there is no 1 2
net movement of the solvent occurs through the i , i=1+2
1
semipermeable membrane between two solutions.
Assuming 100% ionization
122. (c) If compound dissociates in solvent i > 1 and on
association i < 1. So, i = 1 + 2 = 3
123. (a) Van’t Hoff equation is 13.44
Tb = 3 0.52 0.1 = 0.156 0.16 [m 0.1]
V = inRT 134.4
For depression in freezing point.
133. (d) Tf i.k f .m ; Tb i.k b .m
Tf = i × Kf × m
For elevation in boiling point. Tf kf
Tb = i × Kb × m Tb kb
For lowering of vapour pressure,
Tf 0 ( 0.186 C) 0.186 C
Psolvent Psolution n
i . 0.186 1.86 0.52 0.186
N n Tb 0.052
Psolvent Tb 0.52 1.86
EBD_7207
272 SOLUTIONS

STATEMENT TYPE QUESTIONS 156. (a) Mole fraction of any component A in solution

134. (d) No. of moles of A


x
135. (b) Molarity include volume thus with increase Total No. of moles of solution
intemperature increases’ volume increases, hence As total no. of moles of solution > No. of moles of A
molarity decreases while in case of molality mass of
Thus x can never be equal to one or zero.
solvent is taken, which is not effected by temperature.
157. (d) According to Henry’s law the mass of a gas dissolved
136. (c) As temperature increases solubility of gas decreases,
per unit volume of solvent is proportional to the
so dissolution of gas can be considered as exothermic
process. pressure of the gas at constant temperature m = K p
i.e. as the solubility increases, value of Henry’s law
137. (c) Solubility of gases increase with decrease of
temperature. constant decreases. Since CO2 is most soluble in water
among the given set of gases. Therefore CO2 has the
138. (c) A solute dissolves in a solvent if intermolecular
lowest value of Henry’s law constant.
interactions are similar in the two or we may say like
dissolves like. 158. (d) Let the mass of methane and oxygen = m gm.
139. (c) The vapour pressure of a liquid increases with Mole fraction of O2
increase of temperature.
Moles of O2
140. (b) =
Moles of O 2 + Moles of CH 4
141. (d) Colligative properties depends upon the no. of
particles. Since methanol is non electrolyte hence m / 32 1
m / 32
cannot be considered. = = =
m / 32 m /16 3m / 32 3
142. (b) Osmotic pressure is a colligative property as it
depends on the number of solute molecules and not Partial pressure of O2 = Total pressure × mole fraction
on their identity. For dilute solutions, during osmosis 1 1
solvent molecules always flow from lower of O2 , PO2 = P × = P
3 3
concentration to higher concentration of solution.
159. (a) kH = 100 kbar = 105 bar, p = 1 bar
MATCHING TYPE QUESTIONS p = k H × xA
143. (d) 144. (c) 145. (b) 146. (a) 147. (a) p 1 5
xA = 10
kH 100 103
ASSERTION-REASON TYPE QUESTIONS
148. (a) 1000
Moles of water = 55.5
149. (c) Both the solute and solvent will form the vapours but 18
vapour phase will become richer in the more volatile Weight of water = 1000 g ( 1000 mL = 1000 g)
component.
150. (b) 151. (b) 152. (d) 153. (a) 154. (b) x
Mole fraction = 10–5 =
55.5 x
CRITICAL THINKING TYPE QUESTIONS As 55.5 >>> x, thus neglecting x from denominator
155. (c) Equivalent weight of orthophosphoric acid 5 x
10 x 55.5 10 5 moles
3 31 64 98 55.5
(H3PO4) =
3 3 or 0.555 millimoles.
Now 100 gm solution contains 70 gm H3PO4 160. (c)
161. (a) % of N2 in atmosphere = 78.9%
100 70
litre of solution contains gm % of O2 in atmosphere = 20.95%
1000 1.54 98 / 3
Partial pressure of N2 = 0.789 atm = 0.799 bar
equivalent of H3PO4
Partial pressure of O2 = 0.2095 atm = 0.212 bar
Normality of solution
According to Henry’s law,
70 3
70 3 0.212 6
98 10 1.54 33 N P (K H )O2 x x O2 6.08 10
= (34.86 1000)
1 98
10 1.54
SOLUTIONS 273
167. (d)
0.799 5
P (K H ) N 2 x x N2 1.0447 10 168. (b) According to Raoult's law
(76.48 1000)
p n
n O2 n N2 (mole fraction of solute)
: p n N
n O2 n H 2O n N2 n O2 n H 2O n N2
10
x O2 : x N 2 0.2 p° = 50 mm of Hg
p
6.08 10 6 :1.04 10 5 For other solution of same solvent
= 1 : 1.71
20 n
162. (a) According to idea of Raoult’s law “partial pressure of (Mole fraction of solute)
one of the component is proportional to mole fraction p n N
of that component in the solution.” 20
Mole fraction of solute
P = P1° x1 + P2° x 2 50
600 = 450 x1 + 700 x2 Mole fraction of solute = 0.4
4.5 x1 + 7x2 = 6 As mole fraction of solute + mole fraction of solvent = 1
x1 + x2 = 1 Hence, mole fraction of solvent = 1 – 0.4 = 0.6
x1 = 0.6 x2 = 0.4 169 (b) Relative lowering of vapour pressure is given by :
x1 = 0.4, x2 = 0.6
P – Ps w/m
163. (a) The solutions (liquid mixture) which boils at constant
P w/ m W / M
temperature and can distil as such without any change
in composition are called azeotropes. where, P° = vapour pressure of pure solvent
Solution of HNO3 and H2O will form maximum boiling Ps = vapour pressure of solution
point azeotrope. Maximum boiling azeotropes show w = mass of solute
negative deviation from Raoult’s law. m = molecular mass of solute
Composition (%) Boiling Point W = mass of solvent
HNO3 68.0 359 K M = molecular mass of solvent
H2O 32.0 373 K For dilute solution
Boiling point of the azeotrope of these two solutions 121.8 120.2 15/ m 15 78 2
is 393.5 K. = 1.3 10
121.8 250 / 78 m 250
164. (a) m = 356.265
165. (a) Since formation of hydrogen bonding takes place, due
170. (a) From Raoult law
to this bond energy is supposed to be released and
hence change in enthalpy is negative,so reaction is p –p
exothermic. p
166. (a) According to Raoult’s law,
PT = xAp°A + xBp°B No.of moles of solute
=
Given, = PT1 500 mm Hg No. of moles of solvent+ No. of moles of solute
nA = 1 and nB = 2 xA = 1/3 and xB = 2/3 When the concentration of solute is much lower than
the concentration of solvent,
1 2
500 pA pB p p No. of moles of solute
3 3
p No. of moles of solvent
1500 pA 2pB ...(i)
Tb = Kb × m
Also given Qn that one more mole of B is added to the
Number of moles of the solute
solution, the pressure of the ideal solution increases m 1000
by 25 mm Hg. Mass of solvent in grams
PT2 = 500 + 25 = 525 mm Hg Number of moles of the solute
Tb Kb × 1000
Also, nB = 3 xA = 1/4 and xB = 3/4 Mass of solvent in grams

1 3 Number of moles of solute


525 pA pB ...(ii)
4 4 Tb Mass of solvent in grams
Subtract (i) and (ii), K b 1000
2100 PA 3PB
p°B = 600 mm Hg 2 100
0.26,
p°A + 2p°B = 1500 p°A = 300 mm Hg. 0.76 1000
EBD_7207
274 SOLUTIONS
100 For HBr
Number of moles of solvent 5.56
18 HBr H Br
760 p 0.26 1 0 0
From equation (i) we get, (1 )
760 5.56 = 90% = 0.9
On solving, p = 724.46 724 i = 1 + = 1 + 0.9 = 1.9
171. (d) (100 + Tb) – (0 – Tf) = 105 T f = Kf × m × i
Tb + Tf = 5
m (kb + kf) = 5 moles of solute
= 1.86 × × 1.9
mass of solvent in kg
5 5
m i.e., moles in 1000 g water
2.37 2.37 8.1/ 81
= 1.86 × 1.9
5 100 /1000
(or) moles in 100 g water = 1.86 × 1 × 1.9 = 3.534K
2.37 10
Tf Tf Tf
5
Wt. of sucrose 342 = 72g or Tf = 0 – 3.534 K
2.37 10
172. (b) Given kb = x K kg mol–1 Tf = – 3.534K
176. (d) 1% solution contains 1 g of the solute in 100 g of
Tb = kb × m solution.
y= x × m Osmotic pressure, = CRT
y The value of R and T is same for all the solute however,
m all of them undergo 100% dissociation
x
i×C
We know
iKCl = 2, iNaCl = 2, iBaCl = 3 and iurea = 1.
Tf = kf × m nKCl = 1/74.5
2
On substituting value of m,
1/ 74.5
CKCl 1000 0.13
yz 100
Tt =
x or KCl = 2 × 0.13 = 0.26
173. (d) Mass of non-volatile solute = 1g 1
Molar mass of solute = 250g mol–1 nNaCl=
58.5
Mass of benzene = 51.2g, Kf = 5.12 K kg mol–1
1/ 58.5
Kf 1000 w 2 C NaCl 1000 0.17
Tf 100
M 2 w1
NaCl = 2 × 0.17 = 0.34
where, w2= mass of the solute 1
n =
M2 = molar mass of solute BaCl2 208.4
w1 = mass of the solvent
1/ 208.4
5.12 1000 1 CBaCl2 1000 0.048
On substituting given values, Tf 100
51.2 250
or BaCl2 3 0.048 0.14
Tf 0.4K
1
174. (d) Tb = iKb m n urea
60
Given, ( Tb)x > ( Tb)y
ix Kb m > iyKb m (Kb is same for same solvent) Curea
1/ 60
1000 0.16
ix > iy 100
So, x is undergoing dissociation in water. urea = 1 × 0.16 = 0.16
175. (b) Given mass of solute = 8.1 g BaCl2 urea KCl NaCl
Mass of solvent = 100 g or III < IV < I < II
17
ELECTROCHEMISTRY

7. Which one is not called a anode reaction from the following?


FACT / DEFINITION TYPE QUESTIONS
1
1. Which of the following statements is incorrect regarding (a) Cl Cl2 e (b) Cu Cu 2e
2
electrochemistry?
(a) It is the study of production of electricity from energy (c) Hg Hg e (d) Zn2 2e Zn
released during spontaneous chemical reactions. 8. The cell reaction Cu 2Ag Cu 2
Ag is best
(b) NaOH, Cl2, alkali and alkaline earth metals are prepared represented by
by electrochemical methods.
(c) The demerit associated with electrochemical methods (a) Cu(s ) | Cu 2 (aq) | | Ag (aq) | Ag( s )
is that they are more polluting. Thus they are eco- 2
destructive. (b) Pt | Cu | | Ag ( aq ) | Ag( s)
(d) Electrochemical reactions are more energy efficient 2
(c) Cu | Cu | | Pt | Ag
and less polluting.
2. What flows in the internal circuit of a galvanic cell? (d) None of the above representations
(a) Ions (b) Electrons
9. Zn (s ) | Zn 2 ( aq) | | Cu 2 ( aq) |Cu( s) is
(c) Electricity (d) Atoms anode cathode
3. Which of the following statements about galvanic cell is
incorrect (a) Weston cell (b) Daniel cell
(a) anode is positive (c) Calomel cell (d) Faraday cell
(b) oxidation occurs at the electrode with lower reduction 10. The tendency of an electrode to lose electrons is known as
potential (a) electrode potential (b) reduction potential
(c) cathode is positive (c) oxidation potential (d) e.m.f.
(d) reduction occurs at cathode 11. The chemical reaction,
4. Reaction that takes place at graphite anode in dry cell is
2AgCl(s) H 2 (g) 2HCl(aq) 2Ag(s)
(a) Zn 2 2e Zn(s)
2
taking place in a galvanic cell is represented by the notation
(b) Zn(s) Zn 2e
(a) Pt(s) | H 2 (g),1 bar |1M KCl(aq) | AgCl(s) | Ag(s)
(c) Mn 2 2e Mn(s)
(d) Mn(s) Mn e 1.5V (b) Pt(s) | H 2 (g),1 bar |1M HCl(aq) |1MAg (aq) | Ag(s)
5. In which of the following conditions salt bridge is not
required in a galvanic cell? (c) Pt(s) | H 2 (g),1 bar |1M HCl(aq) | AgCl(s) | Ag(s)
(a) When galvanic cell is used in geyser. (d) Pt(s) | H 2 (g),1 bar |1M HCl(aq) | Ag(s) | AgCl(s)
(b) When distance between oxidation half cell and
reduction half cell is negligible. 12. Given that the standard reduction potentials for M+/M and
(c) Electrolytic solutions used in both the half cells are of N+/N electrodes at 298 K are 0.52 V and 0.25 V respectively.
same concentration. Which of the following is correct in respect of the following
(d) When both the electrodes are dipped in the same electrochemical cell ?
electrolytic solution. M/M+ | | N+/N
6. Which device converts chemical energy of a spontaneous (a) The overall cell reaction is a spontaneous reaction.
redox reaction into electrical energy? (b) The standard EMF of the cell is – 0.27 V.
(a) Galvanic cell (b) Electrolytic cell (c) The standard EMF of the cell is 0.77 V.
(c) Daniell cell (d) Both (a) and (c) (d) The standard EMF of the cell is – 0.77 V.
EBD_7207
276 ELECTROCHEMISTRY
13. The difference between the electrode potentials of two 22. Standard reduction potentials of the half reactions are given
electrodes when no current is drawn through the cell is below :
called _________. F2(g) + 2e– 2F– (aq); E° = + 2.85 V
(a) Cell potentials (b) Cell emf Cl2(g) + 2e– 2Cl–(aq); E° = + 1.36 V
(c) Potential difference (d) Cell voltage Br2(l) + 2e– 2Br–(aq); E° = + 1.06 V
14. For the given Nernst equation I2(s) + 2e– 2I–(aq); E° = + 0.53 V
The strongest oxidising and reducing agents respectively
RT Mg 2 are
E cell E cell ln
2F 2 (a) F2 and I– (b) Br2 and Cl–
Ag –
(c) Cl2 and Br (d) Cl2 and I2
Which of the following representation is correct? 23. A button cell used in watches functions as following
(a) Ag+|Ag||Mg2+|Mg (b) Mg2+|Mg||Ag|Ag+ Zn(s) + Ag2O(s) + H2O(l)
(c) Mg|Mg2+||Ag+|Ag (d) Mg|Mg2+||Ag|Ag+ 2Ag(s) + Zn2+(aq) + 2OH–(aq)
15. For cell representation: If half cell potentials are :
Cu(s)|Cu2+(aq)||Ag+(aq)|Ag(s) Zn2+(aq) + 2e– Zn(s); Eo = – 0.76 V
Which of the following is correct? Ag2O(s) + H2O (l) + 2e– 2Ag(s) + 2OH–(aq); Eo = 0.34 V
(i) Cu is reducing agent. The cell potential will be :
(ii) Overall cell reaction is (a) 0.42 V (b) 0.84 V
(c) 1.34 V (d) 1.10 V
Cu s 2Ag aq Cu 2 aq 2Ag s 24. The oxidation potentials of A and B are +2.37 and +1.66 V
(iii) Cu is cathode respectively. In chemical reactions
(iv) Ag is anode (a) A will be replaced by B
(a) (ii), (iii) and (iv) (b) (ii), (iii) and (iv) (b) A will replace B
(c) (iii) and (iv) (d) (i) and (ii) (c) A will not replace B
16. The reference electrode is made by using (d) A and B will not replace each other
(a) ZnCl2 (b) CuSO4 25. A smuggler could not carry gold by depositing iron on the
(c) HgCl2 (d) Hg2Cl2 gold surface since
17. The standard hydrogen electrode potential is zero, because (a) gold is denser
(a) hydrogen oxidized easily (b) iron rusts
(b) electrode potential is considered as zero (c) gold has higher reduction potential than iron
(c) hydrogen atom has only one electron (d) gold has lower reduction potential than iron
(d) hydrogen is a very light element 26. Which cell will measure standard electrode potential of
copper electrode ?
18. Without losing its concentration ZnCl2 solution cannot be
(a) Pt (s) |H2 (g, 0.1 bar) |H+ (aq., 1 M) ||Cu2+ (aq., 1 M) |
kept in contact with
Cu
(a) Au (b) Al
(b) Pt (s) |H2 (g, 1 bar) |H+ (aq., 1 M) ||Cu2+ (aq., 2 M) |
(c) Pb (d) Ag Cu
19. On the basis of the following E° values, the strongest (c) Pt (s) |H2 (g, 1 bar) |H+ (aq., 1 M) ||Cu2+ (aq., 1 M) |
oxidizing agent is : Cu (d) Pt (s) |H2 (g, 1 bar) |H+ (aq., 0.1
[Fe(CN)6]4– [Fe(CN)6]3– + e– ; E° = – 0.35 V 2+
M) ||Cu (aq., 1 M) | Cu
Fe2+ Fe3+ + e–; E° = – 0.77 V 27. Which of the following statement is not correct about an
(a) [Fe(CN)6]4– (b) Fe2+ inert electrode in a cell ?
(c) Fe3+ (d) [Fe(CN)6]3– (a) It does not participate in the cell reaction.
20. Standard electrode potential of three metals X, Y and Z are (b) It provides surface either for oxidation or for reduction
– 1.2 V, + 0.5 V and – 3.0 V, respectively. The reducing power reaction.
of these metals will be : (c) It provides surface for conduction of electrons.
(a) Y > Z > X (b) X > Y > Z (d) It provides surface for redox reaction.
(c) Z > X > Y (d) X > Y > Z 28. In the electrochemical reaction
21. Standard electrode potential for Sn4+ / Sn2+ couple is + 0.15 V 2Fe 3 Zn Zn 2 2Fe 2 ,
and that for the Cr3+ / Cr couple is – 0.74 V. These two couples on increasing the concentration of Fe2+
in their standard state are connected to make a cell. The cell
(a) increases cell emf
potential will be
(b) increases the current flow
(a) + 1.19 V (b) + 0.89 V
(c) decreases the cell emf
(c) + 0.18 V (d) + 1.83 V (d) alters the pH of the solution
ELECTROCHEMISTRY 277
29. The standard e.m.f. of a galvanic cell involving cell reaction 37. The electrode potential E of a zinc electrode at
with n = 2 is found to be 0.295 V at 25°C. The equilibrium (Zn 2 Zn)
constant of the reaction would be 25°C with an aqueous solution of 0.1 M ZnSO 4 is
(Given F = 96500 C mol–1; R = 8.314JK–1mol–1) 2.303RT
(a) 2.0 1011 (b) 4.0 1012 [E = – 0.76 V. Assume = 0.06 at 298 K].
(Zn 2 Zn) F
(c) 1 .0 10 2 (d) 1 .0 1010 (a) + 0.73 (b) – 0.79
(c) – 0.82 (d) – 0.70
30. What will be the emf for the given cell
38. In the cell reaction
Pt | H2 (P1) | H+ (aq) | | H2 (P2) | Pt
Cu(s) 2Ag (aq) Cu 2 (aq) 2Ag(s) ,
RT P RT P
(a) log e 1 (b) log e 1 o
E cell = 0.46 V. By doubling the concentration of Cu2+, Eocell
F P2 2F P2 will become
RT P (a) doubled
(c) log e 2 (d) None of these (b) halved
F P1
(c) increases but less than double
31. The value of electrode potential (10–4 M) H+ | H2(1 atm) | Pt (d) decreases by a small fraction
at 298 K would be 39. E° of a cell aA bB cC dD is
(a) – 0.236 V (b) + 0.404 V
(c) + 0.236 V (d) – 0.476 V [a ]A [b]B RT [C]c [D]d
(a) E RT ln (b) E ln
32. According to Nernst equation, which is not correct if [c]C [d]D nF [A]a [B]b
Q = Kc :
RT RT [C]c [d]D RT [a ]A [B]B
(a) Ecell = 0 (b) ln Q E cell (c) E ln (d) E ln
nF nF [A]A [B]B nF [C]C [d]D
nFEcell 40. E° for the cell,
(c) Kc e RT (d) E cell E cell Zn | Zn 2 (aq) | | Cu 2 (aq)| Cu is 1.10 V at 25°C. The
33. The standard emf of a cell, involving one electron change is
equilibrium constant for the cell reaction
found to be 0.591 V at 25°C. The equilibrium constant of the
reaction is (F = 96500 C mol–1) Zn Cu 2 (aq) Cu Zn 2 (aq)
(a) 1.0 × 101 (b) 1.0 × 105 is of the order of
(c) 1.0 × 1010 (d) 1.0 ×1030 (a) 10–37 (b) 1037
34. For the galvanic cell (c) 10 –17 (d) 1017
Zn | Zn2+ (0.1M) || Cu2+ (1.0M)|Cu the cell potential increase 41. What is the standard cell potential E° for an electrochemical
if: cell in which the following reaction takes place
(a) [Zn2+] is increased spontaneously ?
(b) [Cu2+] is increased Cl2 (g) 2Br Br2 (aq) 2Cl G 50.6 kJ
(c) [Cu2+] is decreased (a) 1.2 V (b) 0.53 V
(d) surface area of anode is increased (c) 0.26 V (d) –0.53 V
35. Consider the following cell reaction: 42. If 0.01 M solution of an electrolyte has a resistance of 40
2Fe( s) O2 ( g) 4H (aq) 2Fe2 (aq) 2H2O(l);E 1.67V ohms in a cell having a cell constant of 0.4 cm–1, then its
2+ –3
At [Fe ] = 10 M, p(O2) = 0.1 atm and pH = 3, the cell molar conductance in ohm–1 cm2 mol–1 is
(a) 10 2 (b) 104
potential at 25ºC is
(a) 1.47 V (b) 1.77 V (c) 10 (d) 103
43. Specific conductance of a 0.1 N KCl solution at 23ºC is
(c) 1.87 V (d) 1.57 V
0.012 ohm–1 cm–1. Resistance of cell containing the solution
36. The e.m.f. of a Daniell cell at 298 K is E1. at same temperature was found to be 55 ohm. The cell
ZnSO4 CuSO4 constant is
Zn Cu (a) 0.0616 cm–1 (b) 0.66 cm–1
(0.01 M) (1.0 M)
(c) 6.60 cm–1 (d) 660 cm–1
When the concentration of ZnSO4 is 1.0 M and that of 44. The unit of equivalent conductivity is
CuSO4 is 0.01 M, the e.m.f. changed to E2. What is the (a) ohm cm
relationship between E1 and E2? (b) ohm–1 cm2 (g equivalent)–1
(a) E 2 0 E1 (b) E1 E 2 (c) ohm cm2 (g equivalent)
(c) E1 E 2 (d) E1 E 2 (d) S cm–2
EBD_7207
278 ELECTROCHEMISTRY
45. The resistance of 0.01 N solution of an electrolyte was found (iii) (iv)

m/(S cm mol )
–1
to be 220 ohm at 298 K using a conductivity cell with a cell

m/(S cm mol )
–1
constant of 0.88cm–1. The value of equivalent conductance

2
of solution is –

2
(a) 400 mho cm2 g eq–1 (b) 295 mho cm2 g eq–1
(c) 419 mho cm g eq 2 –1 (d) 425 mho cm2 g eq–1
46. Specific conductance of 0.1 M HNO3 is 6.3×10–2 ohm–1 cm–1. 1
1
The molar conductance of the solution is c / (mol L) 2
c / (mol L) 2
(a) 100 ohm–1 cm2 (b) 515 ohm–1 cm2
(c) 630 ohm cm –1 2 (d) 6300 ohm–1 cm2 (v)

m/(S cm mol )
–1
47. The specific conductance of a 0.1 N KCl solution at 23°C is
0.012 ohm–1cm–1. The resistance of cell containing the

2
solution at the same temperature was found to be 55 ohm.
The cell constant will be
(a) 0.142 cm–1 (b) 0.66 cm–1
1
(c) 0.918 cm –1 (d) 1.12 cm–1
c / (mol L) 2
48. The unit of specific conductivity is
(a) ohm cm–1 (b) ohm cm–2 Weak acid Strong acid
(c) ohm cm –1 (d) ohm–1 cm–1 (a) (iv) (v)
49. Which of the following solutions of KCl will have the highest (b) (ii) (iv)
value of specific conductance? (c) (i) (ii)
(a) 1.0 N (b) 0.1 N (d) (iii) (ii)
(c) 1.0 ×10–2N (d) 1.0 ×10–3N Electrolyte: KCl KNO 3 HCl NaOAc NaCl
50. The cell constant of a conductivity cell ___________. 55.
2 –1
(a) changes with change of electrolyte. (S cm mol ) : 149.9 145 426.2 91 126.5
(b) changes with change of concentration of electrolyte. Calculate HOAc using appropriate molar conductances
(c) changes with temperature of electrolyte.
(d) remains constant for a cell. of the electrolytes listed above at infinite dilution in H 2 O
51. Which of the following pair(s) is/are incorrectly matched? at 25°C
(i) R (resistance) – ohm ( ) (a) 217.5 (b) 390.7
(ii) (resistivity) – ohm metre ( m) (c) 552.7 (d) 517.2
(iii) G (conductance) – seimens or ohm (S)
56. Kohlrausch’s law states that at
(iv) (conductivity) – seimens metre–1 (Sm–1)
(a) (i), (ii) and (iii) (b) (ii) and (iii) (a) finite dilution, each ion makes definite contribution to
(c) (i), (ii) and (iv) (d) (iii) only equivalent conductance of an electrolyte, whatever
52. On which of the following magnitude of conductivity does be the nature of the other ion of the electrolyte.
not depends? (b) infinite dilution each ion makes definite contribution
(a) Nature of material (b) Temperature to equivalent conductance of an electrolyte depending
(c) Pressure (d) Mass of the material on the nature of the other ion of the electrolyte.
53. Which of the following expression correctly represents (c) infinite dilution, each ion makes definite contribution
molar conductivity? to conductance of an electrolyte whatever be the nature
K KA of the other ion of the electrolyte.
(a) m (b) m (d) infinite dilution, each ion makes definite contribution
C l
to equivalent conductance of an electrolyte, whatever
(c) m KV (d) All of these be the nature of the other ion of the electrolyte.
54. Which of the following represents variation of molar 57. Which of the following expressions correctly represents
conductance of electrolyte with (concentration) ½ the equivalent conductance at infinite dilution of Al2(SO4)3,
respectively for weak and strong electrolyte ?
(i) (ii) Given that and are the equivalent
SO24
m/(S cm mol )
m/(S cm mol )

Al3
–1

–1

conductances at infinite dilution of the respective ions?


2

1 1 2 3
(a) (b) Al3 SO24
3 Al3 2 SO42

1 1
(c) Al3 SO42 (d) 6
c / (mol L) 2 c / (mol L) 2 Al3 SO24
ELECTROCHEMISTRY 279
58. Limiting molar conductivity of NH4OH 65. Molar ionic conductivities of a two-bivalent electrolytes

i.e., m(NH 4OH is equal to : x 2 and y2 are 57 and 73 respectively. The molar
conductivity of the solution formed by them will be
(a) m NH 4Cl m NaCl m NaOH (a) 130 S cm2 mol–1 (b) 65 S cm2 mol–1
2
(c) 260 S cm mol –1 (d) 187 S cm2 mol–1
(b) m NaOH m NaCl m NH 4Cl 66. On electrolysis of dilute sulphuric acid using platinum
electrodes, the product obtained at the anode will be
(c) m NH 4OH m NH 4Cl m HCl (a) hydrogen (b) oxygen
(d) m NH 4Cl m NaOH m NaCl
(c) hydrogen sulphide (d) Sulphur dioxide
67. If 0.5 amp current is passed through acidified silver nitrate
59. Molar conductivities ( m ) at infinite dilution of NaCl, HCl solution for 100 minutes. The mass of silver deposited on
and CH3COONa are 126.4, 425.9 and 91.0 S cm2 mol–1 cathode, is (eq.wt.of silver nitrate = 108)
respectively. m for CH3COOH will be (a) 2.3523 g (b) 3.3575 g
(a) 425.5 S cm2 mol–1 (b) 180.5 S cm2 mol–1 (c) 5.3578 g (d) 6.3575 g
(c) 290.8 S cm2 mol–1 (d) 390.5 S cm2 mol–1 68. Aluminium oxide may be electrolysed at 1000°C to furnish
60. At 25°C, the molar conductance at infinite dilution for the aluminium metal (At. Mass = 27 amu; 1 Faraday = 96,500
strong electrolytes NaOH, NaCl and BaCl2 are 248 × 10–4, 3+
126 × 10 –4 and 280 × 10 – 4 Sm 2 mol –1 respectively. Coulombs). The cathode reaction is– Al + 3e Al
0 To prepare 5.12 kg of aluminium metal by this method we
m Ba(OH)2 in S m2 mol –1 is require electricity of
(a) 52.4 × 10–4 (b) 524 × 10–4
(c) 402 × 10 –4 (d) 262 × 10–4 (a) 5.49 × 101 C (b) 5.49 × 10 4 C

61. CICH2 COONa 224 ohm 1cm 2gm eq 1, (c) 1.83 × 10 7 C (d) 5.49 × 10 7 C
69. Which of the following is the use of electrolysis?
NaCl 38.2 ohm 1cm 2gm eq 1, (a) Electrorefining (b) Electroplating
(c) Both (a) & (b) (d) None of these
HCl 203 ohm 1cm 2 gm eq 1,
70. An electrolytic cell contains a solution of Ag2SO4 and has
What is the value of CICH2COOH platinum electrodes. A current is passed until 1.6 gm of O2
has been liberated at anode. The amount of silver deposited
(a) 288.5 ohm–1cm2gmeq–1
at cathode would be
(b) 289.5 ohm–1cm2gmeq–1
(a) 107.88 gm (b) 1.6 gm
(c) 388.8 ohm–1cm2gmeq–1
(c) 0.8 gm (d) 21.60 gm
(d) 59.5 ohm–1cm2gmeq–1 71. When 9650 coulombs of electricity is passed through a
62. At 25°C molar conductance of 0.1 molar aqueous solution solution of copper sulphate, the amount of copper deposited
of ammonium hydroxide is 9.54 ohm-1 cm2mol-1 and at is (given at. wt. of Cu = 63.6)
infinite dilution its molar conductance is 238 ohm-1 cm2 mol-1. (a) 0318g (b) 3.18 g
The degree or ionisation of ammonium hydroxide at the
(c) 31.8g (d) 63.6g
same concentration and temperature is:
72. Find the charge in coulombs required to convert 0.2 mole
(a) 20.800% (b) 4.008% VO3–2 into VO4–3 –
(c) 40.800% (d) 2.080% (a) 1.93 × 104 (b) 9.65 × 104
63. The electrical properties and their r espective (c) 1.93 × 105 (d) 9.65 × 105
SI units are given below. Identify the wrongly matched pair. 73. A silver cup is plated with silver by passing 965 coulombs
Electrical property SI unit of electricity. The amount of Ag deposited is :
(a) Specific conductance S m–1 (a) 107.89 g (b) 9.89 g
(b) Conductance S (c) 1.0002 g (d) 1.08 g
(c) Equivalent conductance S m2 gequiv–1 74. The number of coulombs required to reduce 12.3 g of
(d) Cell constant m nitrobenzene to aniline is :
64. The ion of least limiting molar conductivity among the (a) 115800 C (b) 5790 C
following is (c) 28950 C (d) 57900 C
75. The amount of electricity that can deposit 108 g of Ag from
(a) SO 24 (b) H+ AgNO3 solution is:
(a) 1 F (b) 2 A
(c) Ca 2 (d) CH 3COO (c) 1 C (d) 1 A
EBD_7207
280 ELECTROCHEMISTRY
76. To deposit one equivalent weight of silver at cathode, the (a) 1.0 mol (b) 0.20 mol
charge required will be (c) 0.40 mol (d) 0.80 mol
(a) 9.65 × 104 C (b) 9.65 × 103 C 87. A current strength of 9.65 amperes is passed through excess
(c) 9.65 × 10 C5 (d) 9.65 × 107 C fused AlCl3 for 5 hours. How many litres of chlorine will be
77. The volume of oxygen gas liberated at NTP by passing a liberated at STP? (F = 96500 C)
current of 9650 coulombs through acidified water is (a) 2.016 (b) 1.008
(a) 1.12 litre (b) 2.24 litre (c) 11.2 (d) 20.16
(c) 11.2 litre (d) 22.4 litre
88. A solution of copper sulphate (CuSO4) is electrolysed for
78. Three faradays electricity was passed through an aqueous
10 minutes with a current of 1.5 amperes. The mass of copper
solution of iron (II) bromide. The weight of iron metal
deposited at th e cathode (at. mass of Cu = 63u)
(at. wt = 65) deposited at the cathode (in gm) is
is
(a) 56 (b) 84
(c) 112 (d) 168 (a) 0.3892g (b) 0.2938g
79. On passing C ampere of electricity through a electrolyte (c) 0.2398g (d) 0.3928g
solution for t second. m gram metal deposits on cathode. 89. When 0.1 mol MnO42– is oxidised the quantity of electricity
The equivalent weight E of the metal is required to completely oxidise MnO42– to MnO4– is
C t C m (a) 96500 C (b) 2 × 96500 C
(a) E (b) E
m 96500 t 96500 (c) 9650 C (d) 96.50 C
90. The weight of silver (at wt. = 108) displaced by a quantity of
96500 m C t 96500
(c) E (d) E electricity which displaces 5600 mL of O2 at STP will be
C t m
(a) 5.4 g (b) 10.8 g
80. The number of electrons passing per second through a
cross-section of copper wire carrying 10–6 amperes of (c) 54.9 g (d) 108.0 g
current per second is found to be 91. Electrolysis of a salt solution was carried out, after some
(a) 1.6 × 10–19 (b) 6 × 10–35 time solution turned yellow than salt can be
(c) 6 × 10 –16 (d) 6 × 1012 (i) NaCl (ii) KCl
81. Faraday’s laws of electrolysis will fail when (iii) RbCl (iv) KBr
(a) temperature is increased
(a) (i), (ii) and (iii) (b) (ii), (ii) and (iv)
(b) inert electrodes are used
(c) (i), (ii) and (iv) (d) (i), (iii) and (iv)
(c) a mixture of electrolytes is used
(d) None of these cases 92. Which of the following statements is incorrect?
82. The electric charge for electrode decomposition of one gram (a) Both electronic and electrolytic conductance depends
equivalent of a substance is on the nature of conducting material.
(a) one ampere per second (b) Both electronic and electrolytic conductance varies
(b) 96500 coulombs per second similarly with temperature.
(c) one ampere for one hour (c) Electronic conductance is independent but electrolytic
(d) charge on one mole of electrons conductance depends on the amount of the
83. In electrolysis of dilute H2SO4 using platinum electrodes conducting substance.
(a) H2 is evolved at cathode (d) All the above statements are incorrect.
(b) NH2 is produced at anode 93. Which of the following statements is incorrect?
(c) Cl2 is obtained at cathode
(a) Electrodes made up of gold participates in the chemical
(d) O2 is produced
reaction.
84. In electrolysis of NaCl when Pt electrode is taken then H2 is
liberated at cathode while with Hg cathode it forms sodium (b) Electrolytic products of NaCl are Na and Cl2 whereas
amalgam. This is because of aqueous NaCl are NaOH, Cl2 and H2.
(a) Hg is more inert than Pt (c) During electrolysis at cathode, reaction with higher
(b) more voltage is required to reduce H+ at Hg than at Pt value of E is preferred.
(c) Na is dissolved in Hg while it does not dissolve in Pt
(d) All of the above statements are incorrect.
(d) conc. of H+ ions is larger when Pt electrode is taken
85. Electrolysis of fused NaCl will give 94. During electrolysis of sulphuric acid, which of the following
(a) Na (b) NaOH processes is possible at anode?
(c) NaClO (d) None of these A. 2H 2 O(l) O 2 (g) 4H (aq) 4e Ecell
= + 1.23 V
86. How many moles of Pt may be deposited on the cathode
when 0.80 F of electricity is passed through a 1.0 M B. 2SO 24 (a q) S2O82 (aq) 2e Ecell
= 1.96 V
solution of Pt4+?
ELECTROCHEMISTRY 281
Choose the correct option based on following statements. 104. Hydrogen-Oxygen fuel cells are used in space craft to supply
(i) Process A is preferred at higher concentration of (a) power for heat and light
sulphuric acid. (b) power for pressure
(ii) Process B is preferred at higher concentration of (c) oxygen
sulphuric acid. (d) water
(iii) Process A is preferred for dilute sulphuric acid. 105. Prevention of corrosion of iron by zinc coating is called
(iv) Process B is preferred for dilute sulphuric acid. (a) electrolysis (b) photoelectrolysis
(v) Both A and B are equally possible at higher (c) cathodic protection (d) galvanization
concentration. 106. The best way to prevent rusting of iron is
(a) (v) and (iii) (b) (iii) and (ii) (a) making it cathode (b) putting in saline water
(c) (i) and (iv) (d) (v) and (iv)
95. Which of the following metals is not produced by (c) Both of these (d) None of these
electrochemical reduction? 107. Several blocks of magnesium are fixed to the bottom of a ship
(a) Na (b) Fe to
(c) Mg (d) Al (a) make the ship lighter
96. As lead storage battery is charged (b) prevent action of water and salt
(a) lead dioxide dissolves (c) prevent puncturing by under-sea rocks
(b) sulphuric acid is regenerated (d) keep away the sharks
(c) lead electrode becomes coated with lead sulphate 108. Which of the following batteries cannot be reused?
(d) the concentration of sulphuric acid decreases (a) Lead storage battery (b) Ni-Cd cell
97. During the charging of lead storage battery, the reaction at (c) Mercury cell (d) Both (b) and (c)
anode is represented by 109. Which of the following is a merit of Ni–Cd cell over lead
(a) Pb2 SO24 PbSO 4 storage battery?
(a) Ni–Cd cell can be re-used.
(b) PbSO 4 2H 2 O PbO2 SO42 4H 2e (b) Ni–Cd cell is comparatively economical to manufacture
(c) Pb 2 (c) Ni–Cd cell has comparatively longer life
Pb 2e
(d) Pb2 2e Pb (d) All the above are the merits of Ni–Cd cell over lead
98. Which colourless gas evolves, when NH4Cl reacts with storage battery.
zinc in a dry cell battery 110. Which of the following statements regarding fuel cell is
(a) NH4 (b) N2 incorrect?
(c) H2 (d) Cl2 (a) These cells are eco-friendly.
99. In a hydrogen-oxygen fuel cell, combustion of hydrogen (b) These cells convert energy of combustion of fuels
occurs to like H2, CH4, CH3OH etc., directly into electrical energy.
(a) produce high purity water (c) H2 – O2 fuel cell is used in Apollo space programme.
(b) create potential difference between two electrodes (d) Fuel cells produce electricity with an efficiency of
(c) generate heat about 100%.
(d) remove adsorbed oxygen from elctrode surfaces
100. Among the following cells: STATEMENT TYPE QUESTIONS
Leclanche cell (i)
Nickel-Cadmium cell (ii) <1.1 V =1.1 V
Lead storage battery (iii) 111.
e–
Mercury cell (iv) anode cathode
current I=0
primary cells are Zn Cu Zn Cu
salt
(a) (i) and (ii) (b) (i) and (iii) –ve bridge +ve
(c) (ii) and (iii) (d) (i) and (iv)
101. The electrolyte used in Leclanche cell is
(a) paste of KOH and ZnO
(b) 38% solution of H2SO4 ZnSO4 CuSO4 ZnSO4 CuSO4
(c) moist paste of NH4Cl and ZnCl2 Figure -1 Figure -2
(d) moist sodium hydroxide >1.1 V
102. A device that converts energy of combustion of fuels like
e–
hydrogen and methane, directly into electrical energy is cathode current anode
known as : +ve –ve
Zn Cu
(a) Electrolytic cell (b) Dynamo
(c) Ni-Cd cell (d) Fuel Cell
103. Which one of the following cells can convert chemical
energy of H2 and O2 directly into electrical energy?
(a) Mercury cell (b) Daniell cell ZnSO4 CuSO4
(c) Fuel cell (d) Lead storage cell Figure -3
EBD_7207
282 ELECTROCHEMISTRY
(i) Figure 1 represents electrochemical and Figure 3 Which of the following is the correct coding for the
represents electrolytic cell. statements above.
(ii) Figure 2 represents electrolytic an d Figure 3 (a) TTTTT (b) TTFTF
represents electrochemical cell. (c) FFTTT (d) FFFTT
(iii) Figure 2 represents a cell which is not working i.e. no 115. Which of the following statement(s) is/are correct?
current flows through the cell. (i) Molar conductivity for strong electrolytes increases
(iv) Energy conversion shown in Figure 1 is chemical to gradually and of weak electrolytes increases rapidly
electrical whereas energy conversion shown in Figure on dilution.
2 is electrical to chemical. (ii) If is the degree of dissociation of weak electrolytes.
Which of the following is the correct coding for the
statements above. m
Then,
(a) TFTT (b) TTTT m

(c) TFFT (d) FTFF (iii) Molar conductivity of CaX2 increases rapidly on
112. Which of the following statements regarding given cell dilution.
representation is/are correct? (a) (i) and (iii) (b) (ii) only
Cd(s)/Cd2+(aq)//Ag+(aq)/Ag(s) (c) (i) only (d) (i) and (ii)
(i) In the given cell Cd electrode act as an anode whereas 116. Read the following statements.
Ag electrode acts as a cathode. (i) According to Faraday’s second law amounts of
(ii) In the given cell Cd electrode acts as a cathode whereas different substances liberated by same quantity of
Ag electrode acts as a annode. electricity passing through the electrolytic solution
are proportional to their chemical equivalent weights.
(iii) E cell E
Ag /Ag
E
Cd 2 /Cd
(ii) 1 F = 96487 Cmol–1 96500 Cmol–1 (for more accurate
(a) (i) and (ii) (b) Only (ii) calculation).
(c) Only (i) (d) (i) and (iii) (iii) As per electrode reactions
113. Which of the following is/are correct statement(s) for the
addition of Li, K, Rb to the aqueous solution of Na+. K e K
(i) The correct order of metals in which they reduce the
Na+ ion is Rb < K < Li. Al3 3e Al
(ii) Reduction of metal ions would not take place. one mole of K+ and Al3+ require 1(1F) and 3(3F) mol of
(a) Statement (i) and (ii) are correct. electrons respectively.
(b) Statement (i) is correct only. Which of the following is the correct coding for the above
(c) Statement (ii) is correct only. statements?
(d) Neither (i) nor (ii) is correct. (a) TTT (b) FFT
114. Read the following statements carefully. (c) TFT (d) FTF
(i) According to a convention cell potential of hydrogen 117. Which of the following statement(s) is/ are incorrect for
electrode (S.H.E.) is considered to be zero at all corrosion of iron?
temperatures.
(i) Reaction occurring at anode is
(ii) e.m.f. of the cell Pt(s)/H2(g, 1 bar)/H+(aq, 1 M)
|| Zn 2 (aq,1M) / Z n is – 0.76. This negative value O2 g 4H aq 4e 2H 2 O l
indicates that Zn 2+ ion reduces less easily then H+ (ii) Reaction occurring at cathode is
ions.
(iii) Copper does not dissolve in HCl but dissolves in HNO3 2Fe s 2Fe2 4e
as in nitric acid it gets oxidised by nitrate ion.
(iv) Inert metals like Pt or Au are used in certain electrodes (iii) Rust is Fe2O3.xH2O
i.e., these metals does not participate in reaction but (iv) H+ involved in corrosion reaction is provided from
provide surface for oxidation and reduction reactions. H2CO3 which is formed due to dissolution of carbon
(v) Fluorine has the highest electrode potential thereby dioxide from air in to water.
making it strongest oxidising agent whereas lithium (a) (iv) only (b) (i) only
with lowest electrode potential is the weakest oxidising (c) (i) and (ii) (d) (i), (ii) and (iv)
and strongest reducing agent.
ELECTROCHEMISTRY 283

MATCHING TYPE QUESTIONS 121. Match the columns.


Column-I Column-II
118. Match the Column-I (functioning of Daniel cell) with (A) (p) I × t
Column-II (value of Eext) and choose the correct option. o
(B) m (q) m / m
Column-I Column-II
(A) Flow of electrons from (p) E = 1.1 V
Cu to Zn and current (C) (r)
c
flows from Zn to Cu
(B) No flow of electrons (q) E < 1.1 V G*
(D) Q (s)
or current R
(C) Zn dissolves at anode (r) E > 1.1 V (a) (A) – (p), (B) – (r), (C) – (q), (D) – (s)
and copper deposits at (b) (A) – (s), (B) – (q), (C) – (r), (D) – (p)
cathode (c) (A) – (r), (B) – (s), (C) – (q), (D) – (p)
(a) (A) – (q), (B) – (p), (C) – (r) (d) (A) – (s), (B) – (r), (C) – (q), (D) – (p)
(b) (A) – (r), (B) – (p), (C) – (q) 122. Match the columns
(c) (A) – (p), (B) – (r), (C) – (q) Column-I Column-II
(d) (A) – (r), (B) – (q), (C) – (p) (A) Cell in which electrolyte (p) H2 – O2 fuel cell
is a paste of KOH and
119. Match the items of Column I and Column II on the basis of
ZnO. This cell is used in
data given below : low current devices like
O O hearing aids, watches, etc.
EO = 2.87 V, E Li /Li
= –3.5 V, E Au 3 /Au
=1.4 V,
(B) Cell in which 38% H2SO4 (q) Mercury cell
F2 /F
solution is used as an
EO =1.09 V electrolyte.
Br2 /Br
(C) Cell in which vapours (r) Lead storage battery
Column-I Column-II produced during electro-
(A) F2 (p) metal is the strongest chemical reaction were
condensed and added
reducing agent.
to drinking water
(B) Li (q) anion that can be oxidised (D) Cell having longer life (s) Nickel – Cadmium cell
by Au3+ than lead storage cell
(C) Au3+ (r) non metal which is the and is expensive to
best oxidising agent manufacture
(D) Br– (s) metal ion which is an (a) (A) – (r), (B) – (q), (C) – (p), (D) – (s)
oxidising agent (b) (A) – (q), (B) – (r), (C) – (p), (D) – (s)
(a) (A) – (r), (B) – (p), (C) – (s), (D) – (q) (c) (A) – (q), (B) – (r), (C) – (p), D) – (s)
(d) (A) – (q), (B) – (r), (C) – (s), (D) – (p)
(b) (A) – (p), (B) – (r), (C) – (s), (D) – (q)
(c) (A) – (q), (B) – (p), (C) – (s), (D) – (r) ASSERTION-REASON TYPE QUESTIONS
(d) (A) – (r), (B) – (s), (C) – (p), (D) – (q)
Directions : Each of these questions contain two statements,
120. Match the columns.
Assertion and Reason. Each of these questions also has four
Column-I Column-II alternative choices, only one of which is the correct answer. You
(A) m (p) intensive property have to select one of the codes (a), (b), (c) and (d) given below.
(a) Assertion is correct, reason is correct; reason is a correct
(B) EO
cell (q) Depends on number of explanation for assertion.
ions/ volume (b) Assertion is correct, reason is correct; reason is not a
correct explanation for assertion
(C) (r) Extensive property
(c) Assertion is correct, reason is incorrect
(D) G
r cell
(s) Increases with dilution (d) Assertion is incorrect, reason is correct.
(a) (A) – (p), (B) – (s), (C) – (q), (D) – (r) 123. Assertion : The resistivity for a substance is its resistance
(b) (A) – (s), (B) – (p), (C) – (q), (D) – (r) when it is one meter long and its area of cross section is one
(c) (A) – (s), (B) – (q), (C) – (p), (D) – (r) square meter.
(d) (A) – (s), (B) – (p), (C) – (r), (D) – (q) Reason : The SI units of resistivity is ohm metre ( m).
EBD_7207
284 ELECTROCHEMISTRY
124. Assertion : On increasing dilution, the specific conductance If the standard reduction potential of Cu2+/Cu is +0.34 V,
keep on increasing. the reduction potentials in volts of the above electrodes
Reason : On increasing dilution, degree of ionisation of follow the order.
weak electrolyte increases and molality of ions also (a) P > S > R > Q (b) S > R > Q > P
increases. (c) R > S > Q > P (d) Q > R > S > P
125. Assertion : Galvanised iron does not rust. 132. At 298K the standard free energy of formation of H2O ( ) is
Reason : Zinc has a more negative electrode potential than –237.20 kJ/mole while that of its ionisation into H+ ion
iron. and hydroxyl ions is 80 kJ/mole, then the emf of the following
cell at 298 K will be
CRITICAL THINKING TYPE QUESTIONS [Take Faraday constant F = 96500C]
H2 (g , 1 bar) | H+ (1M) | | OH– (1M) | O2(g, 1 bar)
126. If salt bridge is removed from two half-cells the voltage (a) 0.40V (b) 0.81V
(a) drops to zero (b) does not change (c) 1.23 V (d) – 0.40 V
(c) increases gradually (d) increases rapidly 133. If the following half cells have E° values as
127. In the electrolytic cell, flow of electrons is from A3+ + e– –––– A2+, E° = y2V
(a) cathode to anode in solution A2+ + 2e– –––– A, E° = –y1V
(b) cathode to anode through external supply The E° of the half cell A3+ + 3e –––– A will be
(c) cathode to anode through internal supply
2 y1 y2 y2 2 y1
(d) anode to cathode through internal supply (a) (b)
3 3
128. Standard potentials (Eº) for some half-reactions are given
below : (c) 2y1 – 3y2 (d) y2 – 2y1
134. Cu + (aq ) is unstable in solution and undergoes
(i) Sn 4 2e Sn 2 ; E º 0.15 V
simultaneous oxidation and reduction according to the
(ii) 2 Hg 2 2e Hg 2 2 ; E º 0.92 V reaction :
(iii) PbO2 4H 2e Pb 2 2H 2O ; E º 1.45 V 2Cu (aq) Cu 2 (aq) Cu( s )
Based on the above, which one of the following statements Choose correct Eº for given reaction if Eº Cu2+/Cu = 0.34 V
is correct ? and Eº Cu2+/Cu+ = 0.15 V
(a) Sn4+ is a stronger oxidising agent than Pb4+ (a) –0.38 V (b) +0.49 V
(b) Sn2+ is a stronger reducing agent than Hg22+ (c) +0.38 V (d) –0.19 V
(c) Hg2+ is a stronger oxidising agent than Pb4+ 135. In a cell that utilises the reaction
(d) Pb2+ is a stronger reducing agent than Sn 2+
Zn( s) 2H (aq) Zn 2 (aq) H 2 ( g ) addition of H2SO4
129. Consider the following relations for emf of a electrochemical
cell: to cathode compartment, will
(i) emf of cell = (Oxidation potential of anode) (a) increase the E and shift equilibrium to the right
– (Reduction potential of cathode) (b) lower the E and shift equilibrium to the right
(c) lower the E and shift equlibrium to the left
(ii) emf of cell = (Oxidation potential of anode)
(d) increase the E and shift equilibrium to the left
+ (Reduction potential of cathode)
136. For a cell reaction involving two electron change, the
(iii) emf of cell = (Reduction potential of anode)
standard EMF of the cell is 0.295 V at 2°C. The equilibrium
+ (Reduction potential of cathode) constant of the reaction at 25°C will be:
(iv) emf of cell = (Oxidation potential of anode) (a) 29.5 × 10–2 (b) 10
– (Oxidation potential of cathode) (c) 1 × 1010 (d) 2.95 × 10–10
Which of the above relations are correct? 137. Standard cell voltage for the cell Pb | Pb2+ || Sn2+ | Sn is
(a) (ii) and (iv) (b) (iii) and (i) – 0.01 V. If the cell is to exhibit Ecell = 0, the value of
(c) (i) and (ii) (d) (iii) and (iv) [Sn 2+] / [Pb2+] should be antilog of –
130. The correct order of E 2 values with negative sign (a) + 0.3 (b) 0.5
M /M
(c) 1.5 (d) – 0.5
for the four successive elements Cr, Mn, Fe and Co is
(a) Mn > Cr > Fe > Co (b) Cr < Fe > Mn > Co 138. The cell, Zn | Zn 2 (1 M) || Cu 2 (1 M) | Cu ( E cell 1.10 V)
(c) Fe > Mn > Cr > Co (d) Cr > Mn > Fe > Co was allowed to be completely discharged at 298 K. The
131. Consider the following four electrodes:
P = Cu2+ (0.0001 M)/Cu(s) [Zn 2 ]
relative concentration of Zn2+ to Cu2+ is
Q = Cu2+ (0.1 M)/Cu(s) [Cu 2 ]
R = Cu2+ (0.01 M)/Cu(s) (a) 9.65 × 104 (b) antilog (24.08)
S = Cu2+ (0.001 M)/Cu(s) (c) 37.3 (d) 1037.3.
ELECTROCHEMISTRY 285
139. What is the potential of half-cell consisting of 146. A weak electrolyte having the limiting equivalent
zinc electrode in 0.01 M ZnSO 4 solution at 25°C conductance of 400 S cm2. equivalent–1 at 298 K is 2%
ionized in its 0.1 N solution. The resistance of this solution
Eox 0.763 V (in ohms) in an electrolytic cell of cell constant 0.4 cm–1 at
(a) 0.8221 V (b) 8.221 V this temperature is
(a) 200 (b) 300
(c) 0.5282 V (d) 9.282 V
(c) 400 (d) 500
140. The oxidation potential of 0.05 M H2SO4 is
147. Conductivity , is equal to _________.
(a) –2 × 0.0591 (b) –0.01 × 0.0591
(c) –2.321 × 0.0591 (d) +1 × 0.0591 1 l G
(i) (ii)
141. For a relation RA R
rG nFE cell l
(iii) m (iv)
E cell E cell in which of the following condition? A
(a) Concentration of any one of the reacting species (a) (i) and (iii) (b) (i) and (ii)
should be unity (c) (i), (ii) and (iii) (d) (ii), (iii) and (iv)
(b) Concentration of all the product species should be 148. Arrange the following in increasing order of their
unity. conductivity Na+ (A), K+ (B), Ca2+ (C), Mg2+(D)
(c) Concentration of all the reacting species should be (a) A, B, C, D (b) B, A, C, D
unity. (c) C, A, D, B (d) A, B, D, C
(d) Concentration of all reacting and product species 149. The conductivity of electrolytic solutions depends upon
should be unity. which of the following?
142. A 0.5 M NaOH solution offers a resistance of 31.6 ohm in a (i) Size of ions produced
conductivity cell at room temperature. What shall be the (ii) Viscosity of the solvent
approximate molar conductance of this NaOH solution if (iii) Concentration of electrolyte
cell constant of the cell is 0.367 cm–1 . (iv) Solvation of ions produced
(a) 234 S cm2 mole–1 (b) 23.2 S cm2 mole–1 (a) (i) and (iii) (b) (i), (ii) and (iii)
2
(c) 4645 S cm mole –1 (d) 5464 S cm2 mole–1 (c) (i), (iii) and (iv) (d) All of these
150. Mark the false statement?
143. The equivalent conductances of two strong electrolytes at
(a) A salt bridge is used to eliminate liquid junction
infinite dilution in H2O (where ions move freely through a
potential
solution) at 25°C are given below :
(b) The Gibbs free energy change, G is related with
CH3COONa 91.0 S cm 2 / equiv. electromotive force E as G = –nFE
(c) Nernst equation for single electrode potential is
HCl 426.2 S cm 2 / equiv. RT
What additional information/ quantity one needs to calcu- E Eo log a M n
nF
late of an aqueous solution of acetic acid? (d) The efficiency of a hydrogen-oxygen fuel cell is 23%
(a) of chloroacetic acid (ClCH2COOH) 151. When electric current is passed through acidified water,
112 ml of hydrogen gas at STP collected at the cathode in
(b) of NaCl
965 seconds. The current passed in amperes is
(c) of CH3COOK (a) 1.0 (b) 0.5
(d) the limiting equivalent coductance of H ( ). (c) 0.1 (d) 2.0
H
144. Resistance of 0.2 M solution of an electrolyte is 50 . The 152. On passing current through two cells, connected in series
specific conductance of the solution is 1.3 S m –1 . If containing solution of AgNO3 and CuSO4, 0.18 g of Ag is
resistance of the 0.4 M solution of the same electrolyte is deposited. The amount of the Cu deposited is:
260 , its molar conductivity is : (a) 0.529 g (b) 10.623 g
(a) 6.25 × 10–4 S m2 mol–1 (b) 625 × 10–4 S m2 mol–1 (c) 0.0529 g (d) 1.2708 g
(c) 62.5 S m2 mol–1 (d) 6250 S m2 mol–1 153. In the electrolysis of water, one faraday of electrical energy
would liberate
145. The limiting molar conductivities of HCl, CH3COONa and
(a) one mole of oxygen (b) one gram atom of oxygen
NaCl are respectively 425, 90 and 125 mho cm2 mol–1 at
(c) 8 g oxygen (d) 22.4 lit. of oxygen
25°C. The molar conductivity of 0.1 M CH3COOH solutions
154. Electrolysis of dilute aqueous NaCl solution was carried
is 7.8 mho cm2 mol–1 at the same temperature. The degree of
out by passing 10 milli ampere current. The time required to
dissociation of 0.1 M acetic acid solution at the same
liberate 0.01 mol of H2 gas at the cathode is
temperature is
(1 Faraday = 96500 C mol–1)
(a) 0.10 (b) 0.02
(a) 9.65 × 104 sec (b) 19.3 × 104 sec
(c) 0.15 (d) 0.03 4
(c) 28.95 × 10 sec (d) 38.6 × 104 sec
EBD_7207
286 ELECTROCHEMISTRY
155. What is the amount of chlorine evolved when 2 amperes of (i) Copper will deposit at cathode.
current is passed for 30 minutes in an aqueous solution of (ii) Copper will deposit at anode.
NaCl ?
(iii) Oxygen will be released at anode.
(a) 66 g (b) 1.32 g
(iv) Copper will dissolve at anode.
(c) 33 g (d) 99 g
(a) (i) and (iii) (b) (ii) and (iv)
156. On passing a current of 1.0 ampere for 16 min and 5 sec
(c) (i) and (ii) (d) (ii) and (iii)
through one litre solution of CuCl2, all copper of the solution
was deposited at cathode. The strength of CuCl2 solution 160. How much charge is required, when 1 mole of Cr2O72
was (Molar mass of Cu= 63.5; Faraday constant = 96,500 reduce to form 1 mole of Cr 3+ ?
Cmol–1) (a) 6F (b) 3F
(a) 0.01 N (b) 0.01 M (c) 1F (d) 2F
(c) 0.02 M (d) 0.2 N 161. When a lead storage battery is discharged
(a) SO2 is evolved
157. 0.2964 g of copper was deposited on passage of a current of
(b) Lead sulphate is consumed
0.5 amp for 30 mins through a solution of copper sulphate.
(c) Lead is formed
Calculate the oxidation state of Cu (At. mass 63.56).
(d) Sulphuric acid is consumed
(a) +1 (b) +2
162. The most durable metal plating on iron to protect against
(c) +3 (d) +4 corrosion is
158. One Faraday of electricity is passed through molten Al 2O3, (a) nickel plating (b) copper plating
aqueous solution of CuSO4 and molten NaCl taken in three (c) tin plating (d) zinc plating
different electrolytic cells connected in series. The mole 163. Which of the following statements is incorrect regarding
ratio of Al, Cu and Na deposited at the respective cathode dry (Leclanche) cell?
is (a) Cathode used in the cell is coated by powdered
(a) 2 : 3 : 6 (b) 6 : 2 : 3 manganese dioxide and carbon.
(c) 6 : 3 : 2 (d) 1 : 2 : 3 (b) Most common application of this cell is in our
transistors and clocks.
159. What will happen during the electrolysis of aqueous
(c) At cathode, Mn is oxidised from + 3 to + 4.
solution of CuSO4 by using platinum electrodes ?
(d) At anode Zn is oxidised from 0 to + 2.
ELECTROCHEMISTRY 287

FACT / DEFINITION TYPE QUESTIONS 20. (c) As the value of standard reduction potential decreases
the reducing power increases i.e.,
1. (c) Electrochemical reactions are more energy efficient Z X Y
and less polluting. Thus study of electrochemistry is ( 3.0) ( 1.2) ( 0.5)
important to create new eco-friendly technologies. 21. (b) Given ESn 4 2 = + 0.15 V
Sn
2. (a) We know that in the internal circuit of a galvanic cell
ions flow whereas in the external circuit, the electrons E Cr 3
Cr = – 0.74 V
flow from one electrode to another.
3. (a) Anode has negative polarity.
4. (b) E cell E ox E red
5. (d) When both the electrodes are kept in the same solution = 0.74 + 0.15
there will be no requirement of salt bridge. = 0.89 V
6. (d) 22. (a) Higher the value of reduction potential higher will be
Reduction the oxidising power whereas the lower the value of
reduction potential higher will be the reducing power.
7. (d) It shows reduction reaction.
Zn 2+ 2e- Zn 23. (d) E°Cell = E°OP + E°RP
= 0.76 + 0.314 = 1.10 V
8. (a) 24. (b) Follow ECS, A will replace B.
9. (b) The cell in which Cu and Zn rods are dipped in its
solution is called Daniel cell. 25. (c) Gold having higher E oRe d and oxidises Fe Fe .
10. (c) The magnitude of the electrode potential of a metal is 26. (c) 27. (d)
a measure of its relative tendency to loose or gain
electrons. i.e., it is a measure of the relative tendency RT ln[Fe 2 ]2
28. (c) Nernst equation Ecell = E ocell Zn 2
3
nF [Fe ] 2
to undergo oxidation (loss of electrons) or reduction
(gain of electrons). increasing [Fe2+] will decrease the Ecell.
M M n
ne (oxidation potential) 0.0591
29. (d) E log K
M n
ne M (reduction potential) n
11. (b) 2AgCl(s) H 2 (g) 2HCl(aq) 2Ag(s) Here, n 2, E 0.295
The activities of solids and liquids are taken as unity 2 0.295
and at low concentrations, the activity of a solute is log K = 9.98 10 or K 1010
0.0591
approximated to its molarity.
The cell reaction will be 30. (b) RHS : 2H+ + 2e– H2 (P2)
LHS : H2(P1) 2H+ + 2e–
Pt(s) | H 2 (g),1bar | H (aq)1M | AgCl(aq)1M | Ag(s) overall reaction : H2 (P1) H2(P2)
12. (b) Eocell o
Ecathode o
Eanode Eoright o
E left RT P2 RT P2 RT P1
E = E°- n = 0- n = n
nF P1 nF P1 nF P2
Eocell 0.25 0.52 0.27V
13. (b) 14. (c) 0.059 1
31. (a) E E log
15. (d) Cu is anode and Ag+ is cathode. n [H ]
16. (d) Calomel electrode is used as reference electrode.
17. (b) Electrode potential is considered as zero. 0.059 1
=0 log 0.236V
18. (b) Without losing its concentration ZnCl 2 solution 1 10 4
cannot kept in contact with Al because Al is more RT
reactive than Zn due to its highly negative electrode 32. (d) Ecell Ecell ln Q
nF
reduction potential.
At equilibrium,
19. (c) From the given data we find Fe3+ is strongest oxidising
Ecell = 0 and Q = Kc
agent. More the positive value of E°, more is the
tendency to get oxidized. Thus correct option is (c).
Ecell Ecell
EBD_7207
288 ELECTROCHEMISTRY
33. (c) The E°cell is given by o 0.059 1.10 2
40. (b) E cell log K C or log K C
0.0591 2 0.059
E°cell log Keq
n KC 1.9 1037
0.0591 G
0.591 log Keq 41. (c) G nFE ; E ;
1 nF
0.591 ( 50.61kJ)
or log Keq = = 10 E 0.26V
0.0591 2 96500 10 3
or Keq = 1 × 1010 42. (d) Molarity = 0.01 M ; Resistance = 40 ohm;
34. (b) For the given cell l -1
Cell constant = 0.4cm .
0.059V [Zn 2 ( aq )] A
Ecell = E°cell log
2 [Cu 2 ( aq)] Specific conductivity ( )
The cell potential will decreases with increase in
cell constant 0.4
[Zn 2+ (aq)] and will increases with increase in = = 0.01 ohm 1 cm 1
[Cu2+(aq)]. resistance 40
35. (d) Here n = 4, and [H+] = 10– 3 (as pH = 3) 1000
Applying Nernst equation Molar conductance ( m)
Molarity

0.059 [Fe 2 ]2 1000 0.01


E = Eº – log 103 ohm 1 cm 2 mol 1
n [H ]4 ( pO2 ) .01
43. (b) Specific conductance of the solution ( ) = 0.012 ohm–1
0.059 (10 3 )2 cm–1 and resistance (R) = 55 ohm.
1.67 log
4 (10 3 ) 4 0.1 Cell constant = Specific conductance × Observed

0.059 resistance
= 1.67 - log107 = 1.67 – 0.103 = 1.567
4 = 0.012 × 55= 0.66 cm–1.
44. (b) ohm–1 cm2 (geq)–1
36. (b) Cell reaction is, Zn Cu 2 Zn 2 Cu
1000 1 l 1000
° RT [Zn 2 ] 45. (a) eq =
Ecell Ecell n N R a N
nF [Cu 2 ]
1 1000 1 1000
= cell constant = 0.88
2 R N 220 0.01
( Zn )
Greater the factor 2 , less is the EMF = 400 mho cm2 g eq–1
(Cu )
46. (c) Molar conductance of solution is related to specific
Hence E1 > E2 conductance as follows :
37. (b) For Zn2+ Zn 1000
m = ....(a)
2.303RT Zn C
E E log
Zn 2 /Zn Zn 2 /Zn where C is molar concentration.
nF Zn 2
Putting = 6.3 × 10–2 ohm–1 cm–1 and
C = 0.1M
0.06 1 1000
0.76 log 0.76 0.03 = (6.3 × 10–2 ohm–1 cm–1) ×
2 0.1 m
(0.1mol / cm3 )
E 0.79V = 6.3 × 10–2 × 104 ohm–1cm2 mol–1
Zn 2 /Zn = 630 ohm–1 cm2 mol–1
38. (d) emf will decrease. 1
47. (b) Cell constant
o RT [C]c [D]d R
39. (b) E cell E cell ln .
nF [A]a [B]b Cell constant = × R; 0.012 × 55 = 0.66 cm–1.
48. (d) ohm–1 cm–1
RT [C]c [D]d 49. (a) The specific conductance increases with concentration.
Hence E ocell E cell ln
nF [A]a [B]b The number of ions per cm–3 increase with increase of
concentration.
ELECTROCHEMISTRY 289
50. (d)
60. (b) 248 10 4 Sm 2 mol 1
51. (c) Correct matching for pair (iii) will be Na OH
[G (conductance) – siemens or ohm–1(S).]
52. (d) Conductivity does not depend upon mass or weight 126 10 4 Sm 2 mol 1
Na Cl
of material.
4
53. (d) 280 10 S m 2 mol 1
Ba 2 2Cl
54. (c) (i) represents weak electrolyte
Now, Ba(OH) 2 BaCl2 2 NaOH 2 NaCl
(ii) represents strong electrolyte.
55. (b) 426.2 (i) Ba(OH)2 280 10 4 2 248 10 4 2 126 10 4
HCl

(ii) Ba(OH) 2 524 10 4 Sm 2 mol 1.


AcONa 91.0
61. (c) ClCH 2COONa HCl ClCH 2COOH NaCl
NaCl 126.5 (iii)
LClCH 2COONa + LHCl = LClCH 2COOH + L NaCl
AcOH (i) (ii) (iii)
224 + 203 = LClCH2COOH + 38.2
[426.2 91.0 126.5] 390.7
56. (d) Kohlrausch’s Law states that at infinite dilution, each LClCH 2COOH = 427 - 38.2
ion migrates independent of its co-ion and contributes
to the total equivalent conductance of an electrolyte a = 388.8 ohm-1cm 2gm eq-1
definite share which depends only on its own nature. 9.54
M
From this definition we can see that option (d) is the 62. (b) = = = 0.04008 = 4.008 %.
238
correct answer. M
63. (d) Cell constant = l/a
57. (c) Conductivity of an electrolyte depends on the mobility Unit = m/m2 = m–1.
of ions and concentration of ions. The motion of an
64. (d) Larger the size, lower the speed.
ionic species in an electric field is retarded by the
oppositely charged ions due to their interionic 2 1
65. (a) m 57 73 130 Scm mol
attraction. On dilution, concentration of electrolyte 66. (b) At anode :
decreases and the retarding influence of oppositely
1
charged ions decreases. Therefore mobility of ions 2HO– H 2O + O2
2
increases.
67. (b) Given current (i) = 0.5 amp;
Time (t) = 100 minutes × 60 = 6000 sec
58. (d) m NH 4Cl m NH m Cl
4 Equivalent weight of silver nitrate (E) = 108.
According to Faraday's first law of electrolysis
m NaOH m Na mOH
Eit 108 0.5 6000
W 3.3575 g.
96500 96500
m NaCl m Na m Cl
68. (d) 1 mole of e– = 1F = 96500 C
27g of Al is deposited by 3 × 96500 C
m NH m OH 5120 g of Al will be deposited by
4
3 96500 5120
= 5.49 10 7 C
27
m NH m Cl m Na
4
69. (c) Electrorefining and electroplating are done by
electrolysis.

m OH m Na m Cl 70. (d) WA WB 1.6 Wt. of Ag


;
EA EB 8 108
Wt . of Ag 21.6 g
m NH 4OH m NH 4Cl m NaOH m NaCl
71. (b) Cu+2 + 2e– Cu (s)
2 × 96500 C 63.6g
59. (d) CH 3COOH CH3COONa HCl NaCl
63.6
= 91 + 425.9 – 126.4 = 390.5 S cm2mol–1 9650 C will deposit = 9650 = 3.18 g
2 96500
EBD_7207
290 ELECTROCHEMISTRY
72. (a) Charge = 0.2 × 1 Faraday NaCl (s) Na+ (l) + Cl– (l)
= 0.2 × 96500 coulombs Na+ + e– Na (l) (at cathode)
= 19300 = 1.93 × 104 coulombs Cl– (l) Cl (g) + e– (at anode)
73. (d) Ag e Ag Cl (g) + Cl(g) Cl2 (g)
96500 coulombs deposit = 108 g of Ag 86. (b) Pt4+ + 4e–— Pt
108 4F electricity is required to deposit 1 mole of Pt.
965 coulombs deposit = 965 1.08 g Ag 0.80 F of electricity will deposit
96500
= 1/4 × 0.80 moles of Pt = 0.20 mol.
74. (d) C 6 H 5 NO 2 + 6H + + 6e- ¾ ¾
® C 6 H 5 NH 2 + 2H 2 O
87. (d) 1F 11.2 L Cl2 at STP
123
E C6 H 5 NO 2 (eq.wt) = = 20.5 No. of Faradays =
9.65 5 60 60
1.8
6 96500
w ´96500 Vol. of Cl2 = 1.8 × 11.2 L = 20.16
Number of coulombs required = 88. (b) W = Zit
Eq. wt
where Z = Electrochemical equivalent
12.3´ 96500
= = 57900 C 63
20.5 Eq. wt. of copper = 31.5
2
75. (a) According to Faraday law's of electrolysis, amount of
31.5
electricity required to deposit 1 mole of metal Z=
96500
= 96500 C = 1 F
i.e., for deposition of 108g Ag electricity required = 1 F 31.5
W = Zit = 1.5 10 60 0.2938g
76. (a) For deposition of silver, reaction is 96500
Ag+ + e– Ag ( 6) ( 7)
1 mol of Ag will be deposited by 89. (c) MnO 24 MnO4 e
= 1 F = 96500 C = 9.65 × 104 C 0.1 mole
Since 1 equivalent weight of Ag is also equal to the Quantity of electricity required
weight of its 1 mol, hence 1 equivalent weight of Ag = 0.1F = 0.1 × 96500 = 9650 C
will be deposited by = 9.65 × 104 C 90. (d) wO = nO × 32
77. (b) A current of 96500 coulombs liberate 1 mole of O2. 2 2
96500 C liberates = 22.4 L of O2 at NTP 5600
wO = 32 8g
22.4
2 22400
9650 C liberates = ´ 9650 = 1 equivalent of O2
96500
= 2.24 L of O2 at NTP = 1 equivalent of Ag = 108
91. (a) Electrolysis of these (i), (ii) and (iii) salt release Chlorine
2 56 which is yellowish in colour while Br2 is reddish brown
78. (b) Fe 2e Fe; E Fe 28
2 in colour
1 Faraday liberates = 28 g of Fe 92. (b) Electronic conductance decreases with increase in
3 Faraday liberates = 3 × 28 = 84 gm temperature whereas electrolytic conductance
79. (c) increases with increase in temperature as no. of ions
80. (d) Charge (Coulombs) pass per second = 10–6 or charge carriers increases with increase in
number of electrons passed per second temperature.
10 6 93. (a) Gold is an inert metal. Electrodes made up of inert
19
6.24 1012 metals does not participate in chemical reaction.
1.602 10
81. (d) 94. (b) Reaction A is preferred for electrolysis of dilute
sulphuric acid and B is preferred for electrolysis of
82. (d) Charge on one mole of electrons = 96500 C.
concentrated sulphuric acid.
83. (a) When platinum electrodes are dipped in dilute solution
H2SO4 than H2 is evolved at cathode. 95. (b) Iron can be easily reduced with carbon it does not
require electrochemical reduction.
84. (b) In electrolysis of NaCl when Pt electrode is taken then
H2 liberated at cathode while with Hg cathode it forms 96. (b) H2SO4 is regenerated.
sodium amalgam because more voltage is required to 97. (b) During charging, the lead storage battery behaves like
reduce H+ at Hg than Pt. an electrolytic cell. So, at anode the reaction is
85. (a) When molten or fused NaCl is electrolysed, it yields PbSO 4 2H 2 O PbO 2 4H SO 42 2e
metallic sodium and gaseous chlorine. Reactions
98. (c) 2NH 4 Cl Zn 2NH 3 ZnCl 2 H2 .
involved are as follows:
ELECTROCHEMISTRY 291

99. (b) In H 2 O 2 fuel cell, the combustion of H2 occurs to MATCHING TYPE QUESTIONS
create potential difference between the two
118. (b) 119. (a) 120. (b) 121. (d) 122. (b)
electrodes.
100. (d) Primary cells are those cells, in which the reaction ASSERTION-REASON TYPE QUESTIONS
occurs only once and after use over a period of time, it
becomes dead and cannot be reused again. e.g.,
Leclanche cell and mercury cell. 123. (b) We know, R or R , where proportionality
A A
101. (c) The electrolyte used in Leclanche cell is moist paste of
NH4Cl and ZnCl2. constant is called resistivity. If = 1m and A = 1m2,
102. (d) A device that converts energy of combustion of fuels, then R = i.e., Resistance = Resistivity..
directly into electrical energy is known as fuel cell. 124. (d) The specific conductivity decreases while equivalent
103. (c) and molar conductivities increase with dilution.
104. (b) H2 – O2 fuel cell supply power for pressure. 125. (a) Zinc metal which has a more negative electrode
105. (d) Prevention of corrosion by zinc coating is called potential than iron will provide electrons in preference
galvanization. of the iron, and therefore corrodes first. Only when all
106. (a) Cathodic protection is best method to prevent iron the zinc has been oxidised, the iron start to rust.
from rusting. In this method iron is made cathode by
application of external current. CRITICAL THINKING TYPE QUESTIONS
Saline water is highly conducting and hence
126. (a) Salt bridge allows the flow of current by completing
accelerates the formation of rust.
circuit. No current will flow and voltage will drop to
107. (b) Magnesium provides cathodic protection and prevent zero, if salt bridge is removed.
rusting or corrosion.
127. (d) In electrolytic cell the flow of electrons is from anode
108. (c) Mercury cell being primary in nature can be used only
to cathode through internal supply.
once.
109. (c) 128. (b) E° = -0.15V > E° = -0.92
110. (d) Fuel cells produce electricity with an efficiency of Sn 2+ / Sn 4+ Hg 22+ / Hg 2+
about 70% compared to thermal plants whose Hence, Sn 2+ is stronger reducing agent than Hg 22 .
efficiency is about 40%.
129. (a) Option (ii) and (iv) are correct.
STATEMENT TYPE QUESTIONS
130. (a) The value of E ° for given metal ions are
M 2+ M
111. (a) Statement (ii) is false as in Figure 2 equal potential is
applied from either side thereby making his cell
E° = -1.18 V,
non-functional. Figure 3 represents electrolytic cell Mn 2+ Mn
as current flows from anode to cathode.
112. (d) According to an accepted convention anode is written E° = -0.9 V,
on the left side and cathode on the right while Cr 2+ Cr
representing the galvanic cell.
113. (c) Because reduction potential of water is higher than E° = -0.44 V and
Fe2+ Fe
that of Na+ so H2 will be evolved and no reduction of
metal ions occurs.
E° = -0.28 V.
114. (a) All statements given above are correct. Co 2+ Co
115. (d) CaX2 is a strong electrolyte thus its molar conductivity
increases slowly on dilution. Th e correct order of E ° 2+ values without
M M
116. (c) We consider,
considering negative sign would be
96487Cmol 1 96500Cmol 1 for approximate Mn2+ > Cr2+ > Fe2+ > Co2+.
calculations not for accurate calculations.
0.591
117. (c) Reaction occurring at anode 131. (d) E red E ored log[M n ]
n
2Fe s 2Fe2 4e Lower the concentration of Mn+, lower is the reduction
Reaction occuring at cathode potential.
O2 g 4H aq 4e 2H 2 O Hence order of reduction potential is :
Q> R>S >P
EBD_7207
292 ELECTROCHEMISTRY
132. (a) Cell reaction 136. (c) Using the relation,
1 2.303RT 0.0591
cathode : H 2O( ) O2 ( g ) 2e 2OH (aq) E°cell = log K c log K c
2 nF n

anode : 0.0591
H2 ( g ) 2H ( aq ) 2e 0.295 V = log K c
–––––––––––––––––––––––––––––––––––––––– 2
1 2 0.295
H 2 O( ) O2 ( g ) H2 ( g ) 2H (aq) 2OH (aq ) or log Kc = 10
2 0.0591
Also we have or Kc = 1 × 1010
1 137. (a) Apply Nernst equation to the reaction
H2 ( g ) O2 (g ) H 2O( ) ; Pb + Sn2+ Pb2+ + Sn
2
0.059 [Sn 2 ]
Gf 237.2 kJ / mole or E° + log Ecell
2 [Pb 2 ]
H 2O( ) H ( aq ) OH ( aq ); G 80 kJ / mole [Sn 2 ] 0.01 2
Hence for cell reaction or log 2
0.3 ( Ecell 0)
[Pb ] 0.059
G 237.2 (2 80) 77.20 kJ / mole
[Sn 2 ]
G 77200 or antilog (0.3)
E 0.40V [Pb 2 ]
nF 2 96500
133. (b) (i) A3+ + e– –––– A2+, G1 = – 1 F y2 138. (d) Ecell = 0; when cell is completely discharged.
(ii) A2+ + 2e– –––– A, G2 = –2F(–y1) = 2Fy1
Add, (i) and (ii), we get 0.059 Zn 2
Ecell = E°cell log
A3+ + 3e– –––– A ; 2 Cu 2
G3 = G1 + G2
–3FE° = –Fy2 + 2Fy1
–3FE° = –F (y2 – 2y1)
0.059 Zn 2
y 2 y1 or 0 = 1.1 log
E° = 2 2 Cu 2
3
134. (c) 2
2 Cu Cu Cu
Zn 2 2 1.1
º
2e Cu 2
Cu ; E1 0.34V; ... (i) log
2
37.3
Cu 0.059
2 º
e Cu Cu ; E2 0.15V; ... (ii)

Cu e Cu; E3º ? ... (iii) Zn 2


1037.3
2
Now, G1º nFE1º 2 0.34F Cu

G2º 1 0.15F , G3º 1 E3º F 139. (a) The Half cell reaction is Zn 2e Zn .
Again, G1º G2º G3º o 0.059 1
E cell E cell log
2 [Zn ]
0.68F 0.15F E3º F
= 0.059 1
E3º 0.68 0.15 0.53V 0.763 log 0.822V
2 0.01
º
Ecell º
Ecathode º
(Cu / Cu) Eanode (Cu 2
/ Cu ) Eoxi = 0.822 V
= 0.53 – 0.15 = 0.38 V. 140. (d) 2H 2e H2
+ 2+
135. (a) Zn( s) + 2H (aq) Zn (aq) + H 2 ( g ) 0.059 1
o ;
E Re d E Red log
0.059 [Zn 2 ][H 2 ] n [H ]2
Ecell E cell log
2 [H ]2 0.059 1
E Re d 0 log ; ERed = –0.059 V,,
Addition of H2SO4 will increase [H+]and Ecell will also 2 (0.1) 2
increase and the equilibrium will shift towards RHS. Eoxi = 0.059 V.
ELECTROCHEMISTRY 293
141. (c) When the concentration of all reacting species kept 146. (d) C 0 8
unity, then Ecell E cell and the given relation will 1 1000
become C
R A N
rG nFE cell 0.4 1000
R = 500 Ohms
142. (b) Here, R = 31.6 ohm 8 0.1
1 1 147. ( b)
Conductance = ohm 1 = 0.0316 ohm–1 148. (d) Charge on Mg and Ca ion is greater than that of Na
R 31.6
Specific conductance = conductance × cell constant. and K, so Mg and Ca ions possess higher conductivity,
= 0.0316 ohm–1 × 0.367 cm–1 also solvation of metal ion decreases as we move down
the group, hence conductivity increases)
= 0.0116 ohm–1 cm–1
149. (d) The conductivity of electrolytic solution depends
Now, molar concentration = 0.5M (given)
upon all of the given factors.
= 0.5 × 10–3 mole cm–3
150. (c) Correct Nernst equation is
0.0116
Molar conductance = 2.303RT
0.5 10 3C E Eo log a M n .
nF
= 23.2 S cm2 mol–1 151. (a) 2H+ + 2e– ¾ ¾® H2
143. (b) According to Kohlrausch’s law, molar conductivity of
2 22400
weak electrolyte acetic acid (CH3COOH) can be EH (Eq. wt) = =1 g = = 11200 ml (STP)
calculated as follows: 2 2
96500´112
CH3COOH CH3COONa HCl NaCl Total charge passed = = 965
11200
Value of NaCl should also be known for
Q = It = 965
calculating value of CH 3COOH . 965
I= = 1amp.
965
1
144. (a) 152. (c) Using Faraday’s second law of electrolysis,
R A
Weight of Cu deposited Equ. wt. of Cu
1
1.3 Weight of Ag deposited Equ. wt. of Ag
50 A
w Cu 63.5 1
1
65m 0.18 2 108
A
63.5 18
1000 wCu = = 0.0529 g.
2 108 100
molarity
153. (c) According to the definition 1 F or 96500 C is the charge
[molarity is in moles/litre but 1000 is used to convert carried by 1 mol of electrons when water is electrolysed
liter into cm3] 2H2O 4H+ + O2 + 4e–
1 So, 4 Faraday of electricity liberate = 32 g of O2.
65 m 1 1000 cm 3
260 Thus 1 Faraday of electricity liberate
0.4 moles 32
= g of O2 = 8 g of O2
4
650 m 1 1
m3 154. (b) H 2O H+ + OH-
260 4 mol 1000
1
= 6.25 × 10–4 S m2 mol–1 H+ + e- ¾¾ ® H2
2
145. (b) for CH3COOH
0.5 mole of H2 is liberated by 1 F = 96500 C
= CH3COO Na H Cl Na Cl 0.01 mole of H2 will be liberated by
96500
= CH3COO H = ´0.01 = 1930 C
0.5
= 90 + 425 – 125 = 390 mho cm2 mol–1.
Q=I ×t
c
m 7.8 Q 1930 C
Degree of dissociation ( ) = 0.02 t= = = 19.3´10 4 sec
m 390
I 10 ´10-3 A
EBD_7207
294 ELECTROCHEMISTRY
1 158. (a) The charge carried by 1 mole of electrons is one faraday.
155. (b) At Anode, Cl Cl e Thus for a reaction
2 2
Mn ne M
35.5 2
Equivalent wt. of chlorine (ECl2 ) 35.5 nF = 1 mole of M
2
Al3 3e Al
ECl2 I t 35.5 2 30 60 3F 1 mol
WCl2 1.32 gm. 1F 1/3 mol
96500 96500
Cu 2 2e Cu
W Q 2F 1 mol
156. (a) By Faraday's 1st Law of electrolysis, = 1F 1/2 mol
E 96500
Na e Na
(where Q it = charge of ion ) 1F 1 mol
We know that no. of gram equivalent The mole ratio of Al, Cu and Na deposited at the
W it 1 965 1 1 1
respective cathode is : :1 or 2 : 3 : 6.
E 96500 96500 100 3 2
(where i= 1 A, t = 16×60+5 = 965 sec.) 159. (a)
Since, we know that 160. (b) Total of 6 electrons are required to form 2 moles of
Cr3+ therefore to form 1 mole 3F of charge is required.
1
Discharge
Normality No. of gram equivalent 100 = 0.01 N 161. (d) Pb PbO 2 2H 2SO 4 2PbSO 4 2H 2 O.
Recharge
Volume (in litre) 1
Sulphuric acid is consumed on discharging.
157. (b) Quantity of charge passed = 0.5 × 30 × 60 = 900 coulomb 162. (d) This is because zinc has higher oxidation potential
900 coulomb will deposit = 0.2964g of copper than Ni, Cu and Sn. The process of coating of iron
96500 coulomb will deposit surface with zinc is known as galvanization. Galvanized
0.2964 iron sheets maintain their lustre due to the formation
= 96500 = 31.75 g of copper of protective layer of basic zinc carbonate.
900
Thus, 31.75 is the eq. mass of copper 163. (c) At cathode reduction occurs according to following
At. mass = Eq. mass × Valency reaction.
63.56 = 31.75 × x 4 3
x = + 2. MnO 2 NH 4 e MnO OH NH3
18
CHEMICAL KINETICS

FACT / DEFINITION TYPE QUESTIONS d[ NH 3 ] 3 d[H 2 ]


(b)
dt 2 dt
1. The term – dc/dt in a rate equation refers to :
(a) the conc. of a reactant d[ NH 3 ] d[H 2 ]
(c)
(b) the decrease in conc. of the reactant with time dt dt
(c) the velocity constant of reaction d[ NH 3 ] 1 d[ H 2 ]
(d) None of these (d)
dt 3 dt
2. The rate law for the single- step reaction 2A B 2C, 8. For the reaction 2 A + B 3C + D
is given by: which of the following does not express the reaction rate ?
(a) rate = k [A].[B] (b) rate = k [A]2.[B]
d[ B] d[ D]
(c) rate = k [2A].[B] (d) rate = k [A]2.[B]o (a) (b)
3. Rate of which reaction increases with temperature : dt dt
(a) of any type of reactions 1 d[A] 1 d[C]
(c) – (d) -
(b) of exothemic reactions 2 dt 3 dt
(c) of endothemic reactions 9. Which of the following reaction does not occur fastly ?
(d) of none (a) Precipitation of AgCl by mixing aqueous solutions of
4. In a slow reaction, rate of reaction generally .......... with AgNO3 and NaCl.
time: (b) Burning of gasoline
(a) decreases (c) Rusting of iron
(b) increases (d) Burning of LPG for cooking
(c) sometimes increases and sometimes decreases. 10. Chemical kinetics is a study to find out
(d) remains constant (a) the feasibility of a chemical reaction
5. The rate of a chemical reaction tells us about, (b) extent to which a reaction will proceed
(a) the reactants taking part in reaction (c) speed of a reaction
(b) the products formed in the reaction (d) All of the above
(c) how slow or fast the reaction is taking place 11. Rate of a reaction can be defined as
(d) None of the above (a) the rate of decrease in concentration of any one of the
6. For the reaction 2 A + B 3C + D reactants
which of the following does not express the reaction rate ? (b) the rate of increase in concentration of any one of the
d[ B] d[D] products
(a) (b)
dt dt (c) the rate of decrease in concentration of any one of the
1 d[A] 1 d[C] reactants or the rate of increase in concentration of
(c) – (d) - any one of the products
2 dt 3 dt
7. Consider the reaction (d) the sum of rate of decrease in concentration of all the
N2 (g) + 3H2 (g) 2 NH3 (g) reactants or the rate of increase in concentration of all
the products
d[ NH 3 ] d[H 2 ]
The equality relationship between and 12. The rate of reaction
dt dt (a) increases as the reaction proceeds
is (b) decreases as the reaction proceeds
d[ NH 3 ] 2 d[ H 2 ] (c) remains the same as the reaction proceeds
(a) (d) may decrease or increase as the reaction proceeds
dt 3 dt
EBD_7207
296 CHEMICAL KINETICS
13. The unit of rate of reaction is 23. Order of reaction can be
(a) mole/dm3 (b) mole/pound (a) 0 (b) fraction
(c) mole/dm3 sec (d) mole/cm3 (c) whole number (d) integer, fraction, zero
14. In the rate equation, when the conc. of reactants is unity 24. Units of rate constant of first and zero order reactions in
then rate is equal to terms of molarity M unit are respectively
(a) specific rate constant (a) sec–1 , Msec–1 (b) sec–1, M
–1
(c) Msec , sec –1 (d) M, sec–1.
(b) average rate constant
25. A reaction involving two different reactants can never
(c) instantaneous rate constant
be
(d) None of above (a) bimolecular reaction (b) second order reaction
15. The rate of reaction between two specific time intervals is (c) first order reaction (d) unimolecular reaction
called 26. , it would be a zero order reaction when
3A B C
(a) instantaneous rate (b) average rate (a) the rate of reaction is proportional to square of
(c) specific rate (d) ordinary rate concentration of A
16. Instantaneous rate of a chemical reaction is (b) the rate of reaction remains same at any concentration
(a) rate of reaction in the beginning of A
(b) rate of reaction at the end (c) the rate remains unchanged at any concentration of B
(c) rate of reaction at a given instant and C
(d) rate of reaction between two specific time intervals (d) the rate of reaction doubles if concentration of B is
17. At the beginning the decrease in the conc. of reactants is increased to double
(a) slow (b) moderate 27. For the following homogeneous reaction,
(c) rapid (d) None of above A B
k
C
18. The average rate and instantaneous rate of a reaction are the unit of rate constant is
equal (a) sec –1 (b) sec–1 mol L–1
(a) at the start (c) sec–1 mol–1 L (d) sec–1 mol–2 L2
(b) at the end 28. Order of reaction is decided by
(c) in the middle (a) temperature
(d) when two rate have time interval equal to zero (b) mechanism of reaction as well as relative concentration
19. The rate of reaction depends upon the of reactants
(a) volume (b) force (c) molecularity
(c) pressure (d) conc. of reactants (d) pressure
20. For the following reaction: NO2(g) + CO(g) NO(g) + 29. Velocity constant k of a reaction is affected by
(a) change in the concentration of the reactant
CO2(g), the rate law is: Rate = k [NO2]2. If 0.1 mole of
(b) change of temperature
gaseous carbon monoxide is added at constant temperature
(c) change in the concentration of the product
to the reaction mixture which of the following statements is
(d) None of the above
true?
30. The rate constant for the reaction
(a) Both k and the reaction rate remain the same
2N 2O5 4 NO 2 O 2 is 3.10 × 10–5 sec–1. If the rate is
(b) Both k and the reaction rate increase
(c) Both k and the reaction rate decrease 2.4 × 10–5 mol litre–1 sec–1 then the concentration of N 2 O 5
(d) Only k increases, the reaction rate remain the same (in mol litre–1) is :
21. Which one of the following statements for the order of a (a) 0.04 (b) 0.8
reaction is incorrect ? (c) 0.07 (d) 1.4
(a) Order can be determined only experimentally. 31. A zero order reaction is one whose rate is independent of
(b) Order is not influenced by stoichiometric coefficient (a) the concentration of the reactants
of the reactants. (b) the temperature of reaction
(c) the concentration of the product
(c) Order of reaction is sum of power to the concentration
(d) the material of the vessel in which reaction is carried
terms of reactants to express the rate of reaction.
out
(d) Order of reaction is always whole number. 32. The rate law for a reaction between the substances A and B
22. The rate of the reaction 2NO Cl2 2NOCl is is given by Rate = k [A]n [B]m
given by the rate equation rate = k [NO]2 [Cl2] On doubling the concentration of A and halving the
The value of the rate constant can be increased by: concentration of B, the ratio of the new rate to the earlier
(a) increasing the concentration of NO. rate of the reaction will be as
(b) increasing the temperature. (a) (m + n) (b) (n – m)
(c) increasing the concentration of the Cl 2 1
(c) 2(n – m) (d) (m n )
(d) doing all of the above 2
CHEMICAL KINETICS 297

33. In the reaction 2A B A 2 B, if the concentration of A is 43. What is order with respect to A, B, C, respectively
[A] [B] [C] rate (M/sec.)
doubled and that of B is halved, then the rate of the reaction will:
(a) increase 2 times (b) increase 4 times 0.2 0.1 0.02 8.08 × 10–3
(c) decrease 2 times (d) remain the same 0.1 0.2 0.02 2.01 × 10–3
34. The order of a reaction, with respect to one of the reacting 0.1 1.8 0.18 6.03 × 10–3
component Y, is zero. It implies that: 0.2 0.1 0.08 6.464 × 10–2
(a) the reaction is going on at a constant rate (a) –1, 1, 3/2 (b) –1, 1, 1/2
(b) the rate of reaction does not vary with temperature (c) 1, 3/2, –1 (d) 1, –1, 3/2
(c) the reaction rate is independent of the concentration 44. Select the rate law that corresponds to the data shown for
of Y the following reaction :
(d) the rate of formation of the activated complex is zero A B C
35. If the rate of a gaseous reaction is independent of pressure,
Expt. No. (A) (B) Initial Rate
the order of reaction is:
1 0.012 0.035 0.10
(a) 0 (b) 1
2 0.024 0.070 0.80
(c) 2 (d) 3
36. If the rate of the reaction is equal to the rate constant, the 3 0.024 0.035 0.10
order of the reaction is 4 0.012 0.070 0.80
(a) 3 (b) 0 (a) Rate = k[B]3 (b) Rate = k [B]4
(c) 1 (d) 2 (c) Rate = k [A] [B] 3 (d) Rate = k [A]2 [B]2
37. In a reaction, when the concentration of reactant is increased 45. The order of a reaction with rate equal to k[A]3/2 [B]–1/2 is
two times, the increase in rate of reaction was four times. :
Order of reaction is : 1
(a) zero (b) 1 (a) 1 (b)
2
(c) 2 (d) 3
3
38. For the reaction A + 2B C, rate is given by R = [A] [B]2 (c) (d) 2
then the order of the reaction is 2
(a) 3 (b) 6 46. For the reaction,
(c) 5 (d) 7 H
CH3COCH3 + I2 products
39. The unit of rate constant for a zero order reaction is
The rate is governed by expression
(a) mol L–1 s–1 (b) L mol–1 s–1
2
(c) L mol s –2 –1 (d) s – 1 dx
k[acetone][H ]
40. Which one of the following reactions is a true first order dt
reaction? The order w.r.t. I2 is:
(a) Alkaline hydrolysis of ethyl acetate (a) 1 (b) 0
(b) Acid catalyst hydrolysis of ethyl acetate (c) 3 (d) 2
(c) Decomposition of N2O 47. The rate constant of a reaction is 3.00 × 103 L mol–1 sec–1.
(d) Decomposition of gaseous ammonia on a hot platinum The order of this reaction will be:
surface (a) 0 (b) 1
41. For a reaction A + B C + 2D, experimental results (c) 2 (d) 3
were collected for three trials and the data obtained are 48. During the kinetic study of the reaction, 2A + B C + D,
given below: following results were obtained:
Trial [A], M [B], M Initial Rate, M s–1 Run [A ](mol L–1 ) [B](mol L– 1 ) Initial rate of
–4 formation of
1 0.40 0.20 5.5 × 10 –1 –1
D (mol L min )
2 0.80 0.20 5.5 × 10–4 –3
I 0.1 0.1 6.0 × 10
3 0.40 0.40 2.2 × 10–3 II 0.3 0.2 7.2 × 10
–2

The correct rate law of the reaction is III 0.3 0.4 2.88 × 10
–1

(a) rate = k[A]0 [B]2 (b) rate = k[A] [B]2 IV 0.4 0.1 2.40 × 10
–2

(c) rate = k[A] [B] (d) rate = k[A] [B]0


Based on the above data which one of the following is
42. The rate law for the reaction
correct?
xA + yB mP + nQ is Rate = k [A]c [B]d. What is the
total order of the reaction? (a) rate = k [A]2 [B] (b) rate = k[A] [B]
(a) (x + y) (b) (m + n) 2
(c) rate = k [A] [B] 2
(d) rate = k [A] [B]2
(c) (c + d) (d) x/y
EBD_7207
298 CHEMICAL KINETICS
49. For the reaction H2(g) + Br2 (g) 2HBr (g), the experimental 57. Half life of a first order reaction is 4s and the initial
data suggest, rate = k[H2][Br2]1/2. The molecularity and concentration of the reactant is 0.12 M. The concentration
order of the reaction are respectively of the reactant left after 16 s is
3 3 3 (a) 0.0075 M (b) 0.06 M
(a) 2, (b) , (c) 0.03 M (d) 0.015 M
2 2 2
1 58. The reaction A B follows first order kinetics. The time
(c) 1, 1 (d) 1, taken for 0.8 mole of A to produce 0.6 mole of B is 1 hour.
2
50. The chemical reaction 2O 3 3O 2 proceeds as follows: What is the time taken for conversion of 0.9 mole of A to
produce 0.675 mole of B?
Fast Slow
O3 O2 O ; O O 3 2 O 2 the rate law (a) 2 hours (b) 1 hour
expression should be (c) 0.5 hour (d) 0.25 hour
(a) r = k[O3]2 (b) r = k [O3]2[O2]–1 59. The rate of a first order reaction is 1.5 × 10–2 mol L–1 min–1 at
3
(c) r = k [O3][O2] 2 (d) r = [O3][O2]2 0.5 M concentration of the reactant. The half life of the reaction
51. Nitrogen monoxide, NO, reacts with hydrogen, H2, according is
to the following equation: (a) 0.383 min (b) 23.1 min
2NO(g) + 2H2(g) N2(g) + 2H2O(g) (c) 8.73 min (d) 7.53 min
If the mechanism for this reaction were, 60. The rate constant for a first order reaction whose half-life, is
2NO(g) + H2(g) N2(g) + H2O2(g) ; slow 480 seconds is :
H2O2(g) + H2(g) 2H2O(g) ; fast (a) 2.88 × 10–3 sec–1 (b) 2.72 × 10–3 sec–1
Which of the following rate laws would we expect to obtain (c) 1.44 × 10 sec –3 –1 (d) 1.44 sec–1
experimentally?
61. The rate constant of a first order reaction is 6.9 10 3 s 1 .
(a) Rate = k[H2O2][H2] (b) Rate = k[NO]2[H2]
2 2 How much time will it take to reduce the initial concentration
(c) Rate = k[NO] [H2] (d) Rate = k[NO][H2]
to its 1/8th value?
52. Which of the following is not a first order reaction ?
(a) 100 s (b) 200 s
(a) Hydrogenation of ethene
(c) 300 s (d) 400 s
(b) Natural radioactive decay of unstable nuclei
(c) Decomposition of HI on gold surface 62. A reaction proceeds by first order, 75% of this reaction
(d) Decomposition of N2O was completed in 32 min. The time required for 50%
53. The plot that represents the zero order reaction is : completion is
(a) 8 min (b) 16 min
(c) 20 min (d) 24 min
[R] [R] 63. Point out the wrong statement:
(a) (b)
t t For a first order reaction
(a) time for half-change (t 1/2) is independent of initial
concentration
[R] (b) change in the concentration unit does not change the
(c) (d) In[R]
t t rate constant (k)
54. The plot of concentration of the reactant vs time for a (c) time for half-change × rate constant = 0.693
reaction is a straight line with a negative slope. The reaction (d) the unit of k is mole–1 min–1
follows a rate equation 64. t 1 can be taken as the time taken for the concentration of a
(a) zero order (b) first order 4
(c) second order (d) third order 3
reactant to drop to of its initial value. If the rate constant
55. The half-life of a reaction is inversely proportional to the 4
square of the initial concentration of the reactant. Then the t
order of the reaction is for a first order reaction is k, the 1 can be written as
4
(a) 0 (b) 1 (a) 0.75/k (b) 0.69/k
(c) 2 (d) 3 (c) 0.29/k (d) 0.10/k
56. The rate equation for a reaction,
65. In a first-order reaction A B, if k is rate constant and
N2O N2 + 1/2O2
inital concentration of the reactant A is 0.5 M, then the half-
is Rate = k[N2O]0 = k. If the initial concentration of the
life is
reactant is a mol Lit–1, the half-life period of the reaction is
a log 2 log 2
(a) t 1 (b) t 1 ka (a) (b)
2 k 2
k k 0.5
2
a k ln 2 0.693
(c) t1 (d) t1 (c) (d)
k a k 0.5k
2 2
CHEMICAL KINETICS 299
66. Consider the reaction, 2A + B products. When 74. A reaction having equal energies of activation for forward
concentration of B alone was doubled, the half-life did not and reverse reaction has :
change. When the concentration of A alone was doubled, (a) G= 0 (b) H = 0
the rate increased by two times. The unit of rate constant (c) H= G= S= 0 (d) S = 0
for this reaction is 75. In an exothermic reaction if H is the enthalpy then activation
(a) s – 1 (b) L mol–1 s–1 energy is
(c) no unit (d) mol L–1 s–1. (a) more than H (b) less than H
67. The decomposition of N2O5 occurs as (c) equal to H (d) none of the above
2N2O5 4NO2 + O2 and follows Ist order kinetics, 1
hence: 76. In the Arrhenius plot of ln k vs , a linear plot is obtained
T
(a) the reaction is unimolecular
with a slope of –2 × 104 K. The energy of activation of the
(b) the reaction is bimolecular
reaction (in kJ mole–1) is (R value is 8.3 J K–1 mol–1)
(c) t1/2 a0
(a) 83 (b) 166
(d) None of these
(c) 249 (d) 332
68. In a first-order reaction A B, if k is rate constant and
inital concentration of the reactant A is 0.5 M, then the half- 77. The rate of reaction is doubled for every 10°C rise in
life is temperature. The increase in reaction rate as a result of
temperature rise from 10°C to 100°C is
log 2 log 2 (a) 112 (b) 512
(a) (b)
k k 0.5 (c) 400 (d) 614
78. Plots showing the variation of the rate constant (k) with
ln 2 0.693
(c) (d) temperature (T) are given below. The plot that follows
k 0.5k Arrhenius equation is
69. For a first order reaction, a plot of log (a – x) against time is
a straight line with a negative slope equal to
k
(a) (b) – 2.303 k
2.303 (a) (b)
2.303 Ea
(c) (d)
k 2 .303 R
70. In a reaction A Products, when start is made from
8.0 × 10–2 M of A, half-life is found to be 120 minute. For the
initial concentration 4.0 × 10–2 M, the half-life of the reaction
(c) (d)
becomes 240 minute. The order of the reaction is :
(a) zero (b) one
(c) two (d) 0.5
79. If the activation energy for the forward reaction is 150 kJ
71. The value of rate constant of a pseudo first order reaction
mol–1 and that of the reverse reaction is 260 kJ mol–1, what
_________.
is the enthalpy change for the reaction ?
(a) depends on the concentration of reactants present in
(a) 410 kJ mol–1 (b) –110 kJ mol–1
small amount.
(c) 110 kJ mol–1 (d) – 410 kJ mol–1
(b) depends on the concentration of reactants present in
excess. 80. Activation energy of a chemical reaction can be determined
(c) is independent on the concentration of reactants. by
(d) depends only on temperature. (a) evaluating rate constant at standard temperature
72. In the Haber process for the manufacture of ammonia the (b) evaluating velocities of reaction at two different
following catalyst is used temperatures
(a) Platinized asbestos (c) evaluating rate constants at two different temperatures
(b) Iron with molybdenum as promoter (d) changing concentration of reactants
(c) Copper oxide 81. In respect of the equation k Ae Ea / RT in chemical
(d) Alumina kinetics, which one of the following statements is correct ?
73. What is the activation energy for a reaction if its rate doubles (a) A is adsorption factor
when the temperature is raised from 20°C to 35°C?
(b) Ea is energy of activation
(R = 8.314 J mol–1 K–1)
(a) 269 kJ mol–1 (b) 34.7 kJ mol–1 (c) R is Rydberg’s constant
(c) 15.1 kJ mol –1 (d) 342 kJ mol–1 (d) k is equilibrium constant
EBD_7207
300 CHEMICAL KINETICS
82. Rate of a reaction can be expressed by Arrhenius equation (a) A catalyst changes the potential energies of the
as : k Ae Ea / RT reactants and products.
(b) A catalyst decreases the temperature of the reaction
In this equation, Ea represents which leads to a faster rate.
(a) the total energy of the reacting molecules at a (c) A catalyst lowers the activation energy for the reaction
temperature, T by providing a different reaction mechanism.
(b) the fraction of molecules with energy greater than the (d) A catalyst destroys some of the reactants, which lowers
activation energy of the reaction the concentration of the reactants.
(c) the energy below which all the colliding molecules will 90. In terms of the ‘Collision Theory of Chemical Kinetics’, the
react rate of a chemical reaction is proportional to
(d) the energy below which colliding molecules will not (a) the change in free energy per second
react
(b) the change in temperature per second
83. The minimum energy required for the reacting molecules to
(c) the number of collisions per second
undergo reaction is :
(d) the number of products molecules
(a) potential energy (b) kinetic energy
91. According to collision theory, which of the following is
(c) thermal energy (d) activation energy
NOT a true statement concerning a catalyst?
84. The reason for almost doubling the rate of reaction on
(a) A catalyst changes the temperature of reaction.
increasing the temperature of the reaction system by 10°C
(b) The mechanism of a reaction will change when a
is
catalyst is added.
(a) the value of threshold energy increases
(c) A catalyst provides a different activation energy for a
(b) collision frequency increases
reaction.
(c) the fraction of the molecule having energy equal to
(d) A catalyst changes the speed of a reaction, but not the
threshold energy or more increases
equilibrium constant.
(d) activation energy decreases
92. Which of the following influences the rate of a chemical
85. The slope in Arrhenius plot, is equal to:
reaction performed in solution?
Ea Ea (a) Temperature
(a) (b)
2.303 R R (b) Activation energy
R (c) Presence of a catalyst
(c) (d) None of these (d) All of the above influence the rate
2.303 Ea
93. How can be activation energy for a reaction be determined
86. The activation energy for a reaction is 9.0 kcal/mol. The graphically?
increase in the rate constant when its temperature is (a) Plot k versus T, the slope of the line will be equal to Ea
increased from 298K to 308K is
(b) Plot 1/[A]t versus t, the slope of the line will be equal
(a) 63% (b) 50% to Ea
(c) 100% (d) 10% (c) Plot ln [A]t versus t, the slope of the line will be equal
87. In a reversible reaction the energy of activation of the to – Ea
forward reaction is 50 kcal. The energy of activation for the (d) Plot ln k versus 1/T, the slope of the line will be equal
reverse reaction will be to – Ea / R
(a) < 50 kcal 94. The Arrhenius equation expressing the effect of temperature
(b) either greater than or less than 50 kcal on the rate constant of the reaction is
(c) 50 kcal
Ea
(d) > 50 kcal (a) k e E a / RT (b) k
RT
88. A catalyst
(a) increases the rate of reaction by decreasing G of a Ea
(c) k log e (d) k Ae E a / RT
reaction. RT
(b) increases the rate of reaction by increasing G of a 95. In Arrhenius plot, intercept is equal to
reaction. Ea
(c) increases the rate of reaction by decreasing activation (a) (b) ln A
R
energy of the forward reaction.
(c) ln k (b) log10 a
(d) increases the rate of reaction by providing an
96. A chemical reaction was carried out at 300 K and 280 K. The
alternative pathway via an intermediate with lower
rate constants were found to be k1 and k2 respectively.
activation energy.
then
89. Which of the following statements best describes how a
(a) k1 = 4k1 (b) k2 = 2k1
catalyst works?
(c) k2 = 0.25 k1 (d) k2 = 0.5 k1
CHEMICAL KINETICS 301
97. For a first order reaction, the plot of log K against 1/T is a (b) Eb Ef
straight line. The slope of the line is equal to
2.303 (c) Eb Ef
Ea
(a) (b)
R Ea R (d) Eb Ef
Ea Ea 106. The energies of activation for forward and reverse reactions
(c) (d) 2.303 R for A2 + B2 2AB are 180 kJ mol–1 and 200 kJ mol–1
2.303
respectively. The presence of a catalyst lowers the activation
98. Collision theory is applicable to
energy of both (forward and reverse) reactions by 100 kJ
(a) first order reactions (b) zero order reactions
mol–1. The enthalpy change of the reaction (A2 + B2 ®
(c) bimolecular reactions (d) intra-molecular reactions
2AB) in the presence of a catalyst will be (in kJ mol–1)
99. According to the collision theory of reaction rates, the rate
of reaction increases with temperature due to (a) 20 (b) 300
(a) greater number of collision (c) 120 (d) 280
(b) higher velocity of reacting molecules 107. For the exothermic reaction A B C D, H is the heat
(c) greater number of molecules having the activation of reaction and Ea is the energy of activation. The energy of
energy activation for the formation of A + B will be
(d) decrease in the activation energy (a) Ea (b) H
100. Which of the following has been used to explain the subject (c) Ea + H (d) H – Ea
of chemical kinetics
108. In most cases, for a rise of 10K temperature the rate constant
(a) Collision theory of bimolecular reactions
is doubled to tribled. This is due to the reason that
(b) The activated complex theory
(a) collision frequency increases by a factor of 2 to 3.
(c) Arrhenius equation
(b) fraction of molecules possessing threshold energy
(d) All of these
increases by a factor of 2 to 3
101. A catalyst increases rate of reaction by
(a) decreasing enthalpy (c) Activation energy is lowered by a factor of 2 to 3.
(b) decreasing internal energy (d) none of these
(c) decreasing activation energy 109. Consider Fig. and mark the correct option.
(d) increasing activation energy Activated complex
102. Activation energy of the reaction is
(a) the energy released during the reaction
(b) the energy evolved when activated complex is formed
E1
(c) minimum amount of energy needed to overcome the
Products
Energy

potential barrier
E2
(d) the energy needed to form one mole of the product
103. In a reaction, the threshold energy is equal to
Reactants
(a) activation energy + normal energy of reactants
(b) activation energy - normal energy of reactants Reaction coordinate
(c) normal energy of reactants - activation energy
(d) average kinetic energy of molecules of reactants (a) Activation energy of forward reaction is E1 + E2 and
104. The temperature dependence of rate constant (k) of a product is less stable than reactant.
chemical reaction is written in terms of Arrhenius equation, (b) Activation energy of forward reaction is E 1 + E2 and
product is more stable than reactant.
k = A . e E a Activation energy (Ea) of the reaction can be
(c) Activation energy of both forward and backward
calculated by plotting
reaction is E1 + E2 and reactant is more stable than
1 1 product.
(a) k vs. (b) log k vs
log T T (d) Activation energy of backward reaction is E 1 and
product is more stable than reactant.
1 110. Activation energy of a chemical reaction can be determined
(c) log k vs. (d) k vs. T
log T by ___________
105. Consider an endothermic reaction X Y with the (a) determining the rate constant at standard temperature.
activation energies E b and E f for the backward and (b) determining the rate constants at two temperatures.
forward reactions, respectively. In general (c) determining probability of collision.
(d) using catalyst.
(a) there is no definite relation between E b and E f
EBD_7207
302 CHEMICAL KINETICS
111. According to which theory activation energy and proper (ii) in fig. b, D represents average rate and C represents
orientation of the molecules together determine the criteria instantaneous rate
for an effective collision ? (iii) fig. a, A represents instantaneous rate and B represents
(a) Arrhenius theory average rate
(b) Activated complex theory (iv) fig. b, C represents average rate and D represents
(c) Collision theory instantaneous rate
(d) Both (a) and (c) (a) (i) and (ii) are correct (b) (ii) and (iv) are correct
(c) (i) and (iv) are correct (d) (ii) and (iii) are correct
STATEMENT TYPE QUESTIONS 115. Choose correct option based on following statements. Here
T stands for true statement and F for false statement.
112. Consider the following reaction : (i) Molecularity is defined as the number of reacting
Pt(s) species taking part in a complex reaction,
4NH 3 (g) 5O 2 (g) 4NO(g) 6H 2O(g)
(ii) Molecularity helps in understanding the mechanism
(i) Rate of reaction with respect to NH 3 will be
of reaction.
1 [NH3 ] (iii) Reactions with the molecularity three are very rare and
4 t slow to proceed.
1 [O2 ] 1 [H 2O] (iv) Complex reactions involving more than three molecules
(ii) For the given reaction take place in more than one step.
5 t 6 t
(a) TTTF (b) TFTF
1 [NH3 ] 1 [NO] (c) FTTF (d) FTTT
(iii) For the given reaction
4 t 4 t 116. Read the following statements
(iv) For the given reaction, (i) Order of reaction can be fractional or zero.
1 [NH3 ] 1 [O2 ] (ii) Molecularity of a reaction can be fractional but cannot
Rate = be zero.
4 t 5 t
4 [NO] 6 [H 2O] (iii) Slowest step in the complex reaction is considered as
= a rate determining step.
t t
(iv) Units of rate constant for second order reaction are
Which of the following is the correct code for the statements
mol L s–1.
above.
(v) Order is applicable to elementary as well as complex
(a) TTTT (b) TFTF
reactions whereas molecularity is applicable only for
(c) FTFT (d) TFFT
elementary reactions.
113. Which of the following statement(s) is/are correct?
Which of the following is the correct code for the statements
(i) Rate of reaction decreases with passage of time as the
above ?
concentration of reactants decrease. (a) TTFFT (b) TFTFT
(ii) For a reaction (c) FFFTT (d) FTTFF
pP qQ rR sS 117. Consider the following statements with respect of zero order
x y
Rate = k[P] [Q] where x = p and y = q reaction
(i) The rate of the reaction is independent of reactant
(iii) Rate law is the expression in which reaction rate is
concentration
given in terms of molar concentration of reactants with
(ii) The rate of the reaction is independent of temperature
each term raised to some power, which may or may not
(iii) The rate constant of the reaction is independent of
be same as the stichiometric coefficient of the reacting
temperature
species in a balanced chemical equation.
(iv) The rate constant of the reaction is independent of
(a) (i) and (iii) (b) (i) and (ii)
reactant concentration
(c) (ii) and (iii) (d) (i) only
Choose the correct statement(s).
114. Study the following graphs and choose the correct option
(a) (i) only (b) (i) and (ii) only
[R]0 (c) (iii) and (iv) only (d) (i) and (iv)only
(P) 118. Which of the following statement(s) is/are correct ?
(i) For a zero order reaction concentration [R] vs time (t)
Concentration

Concentration

c1 A c2 C gives a straight line plot


c2 B [P]
c1 [R]0
t D (ii) For a first order reaction log does not vary linearly
[R]
t4 t2 t Time t4 t2 t Time with time.
(a) (b) (iii) Inversion of cane sugar is a pseudo first order reaction.
(i) in fig. a, A represents average rate and B represents (a) (i) and (iii) (b) (i) only
instantaneous rate (c) (ii) and (iii) (d) (iii) only
CHEMICAL KINETICS 303
119. At high pressure the following reaction is of zero order. 123. Read the following statements.
1130 K (i) e E a /RT corresponds to the fraction of molecules that
2NH3 g N2 g 3H 2 g
Platinum catalyst have kinetic energy greater than Ea.
Which of the following statements are correct for above (ii) Ea can be calculated as follows
reaction?
k Ea T2 T1
(i) Rate of reaction = Rate constant log 1
k2 2.303R T1T2
(ii) Rate of reaction depends on concentration of ammonia.
(iii) Rate of decomposition of ammonia will remain constant (iii) Catalyst can alter a reaction both ways means it can
until ammonia disappears completely. either decrease on increase rate of reaction
(iv) Further increase in pressure will change the rate of (iv) A catalyst always decreases the activation energy of
reaction. the reaction but does not alter Gibb’s energy.
(a) (i), (iii) and (iv) (b) (i), (ii) and (iii) (v) A catalyst does not alter equilibrium constant rather, it
(c) (ii) and (iv) (d) (i), (ii) and (iv) helps in attaining the equilibrium faster.
120. Consider the following statements: Which of the following is the correct codes for above
(i) Increase in concentration of reactant increases the statements ?
rate of a zero order reaction. (a) TTTFF (b) TFFTT
(ii) Rate constant k is equal to collision frequency A if (c) TFFTF (d) FTFTT
Ea = 0. 124. The following statement(s) is (are) correct :
(iii) Rate constant k is equal to collision frequency A if (i) A plot of log kp versus 1/T is linear
Ea = . (ii) A plot of log [X] versus time is linear for a first order
(iv) lnk vs T is a straight line. reaction, X P
(v) lnk vs 1/T is a straight line. (iii) A plot of log p versus 1/T is linear at constant volume
Correct statements are (iv) A plot of p versus 1/V is linear at constant temperature
(a) (i) and (iv) (b) (ii) and (v) (a) (i) only (b) (ii) only
(c) (iii) and (iv) (d) (ii) and (iii) (c) (i) and (iv) (d) (i), (ii) and (iv)
121. According to collision theory, not all collisions between
molecules lead to reaction. Which of the following MATCHING TYPE QUESTIONS
statements provide reasons for the same ?
125. Match the columns
(i) The total energy of the two colliding molecules is less
than some minimum amount of energy. Column-I Column-II
(ii) Molecules cannot react with each other unless a (A) Mathematical expression for rate (p) rate constant
catalyst is present. of reaction
(iii) Molecules that are improperly oriented during collision (B) Rate of reaction for zero order (q) rate law
will not react. reaction is equal to
(iv) Molecules in different states of matter cannot react (C) Units of rate constant for zero (r) order of slowest
with each other. order reaction is same as that of step
(a) (i) and (ii) (b) (i) and (iii) (D) Order of a complex reaction is (s) rate of reaction
(c) (ii) and (iii) (d) (i) and (iv) determined by
122. Consider the following statements (a) A – (q), B – (p), C – (s), D – (r)
(i) Rate constant for every physical and chemical change (b) A – (r), B – (p), C – (s), D – (q)
gets doubled with 10°C rise in temperature (c) A – (q), B – (s), C – (p), D – (r)
(ii) On taking log both side Arrhenius equation will (d) A – (p), B – (q), C – (s), D – (r)
become 126. Match the columns
Ea Column-1 Column-II
log k log A (A) Zero order reaction (p) L mole–1 sec–1
RT
(iii) The energy required to form activated complex is known (B) First order reaction (q) mole L–1 sec–1
as activation energy (C) Second order reaction (r) sec –1
Which of the following is the correct code for statements (a) A – (q), B – (r) ,C – (p)
above? (b) A – (q), B – (p) ,C – (r)
(a) TTT (b) FTT (c) A – (p), B – (q) ,C – (r)
(c) FTF (d) TFT (d) A – (p), B – (r) ,C – (q)
EBD_7207
304 CHEMICAL KINETICS
127. Match the columns 130. Match the columns
Column-I Column-II Column - I Column - II
(A) The decomposition (p) Zero order reaction (A) Number of collisions per (p) Effective collisions.
of gaseous ammonia second per unit volume
on a hot platinum of the reaction mixture.
surface (B) Fraction of molecules (q) Collision frequency
(B) The thermal (q) Pseudo first order with energies equal to
decomposition of HI reaction. or greater than Ea
on gold surface
(C) Molecules for which (r) e E a /RT
(C) All natural and (r) Zero order reaction at
artificial radioactive high pressure Ea /RT
Rate = Z ABe
decay of unstable
nuclei shows significant deviations
(D) Inversion of cane sugar (s) First order reaction. (D) Collision in which molecules (s) Complex molecules
(a) A – (r), B – (p), C – (s), D – (q) collide with sufficient K.E.
(b) A – (r), B – (s), C – (q), D – (p) and proper orientation.
(c) A – (q), B – (s), C – (p), D – (r) (a) A – (q), B – (r), C – (s), D – (p)
(d) A – (q), B – (p), C – (s), D – (p) (b) A – (r), B – (q), C – (s), D – (p)
128. Match the columns. (c) A – (q), B – (s), C – (r), D – (p)
Column-I Column-II (d) A – (q), B – (r), C – (p), D – (s)
(A) Catalyst alters the rate (p) cannot be fraction or zero 131. Consider the energy diagram of a reaction : B A, on the
of reaction basis of given diagram select the correct code for matching
(B) Molecularity (q) proper orientation is not Column-I and Column-II.
there always. X
(C) Second half life of first (r) by lowering the activation
order reaction enrgy Energy
(D) Energetically favourable (s) is same as the first
reactions are sometimes B
slow
A
(a) A – (q), B – (r), C – (s), D – (p)
(b) A – (r), B – (s), C – (p), D – (q)
(c) A – (r), B – (p), C – (s), D – (q) Progress of reaction
(d) A – (p), B – (r), C – (s), D – (q) Column-I Column-II
129. Match the columns (A) X – A (p) Enthalpy of reaction
Column - I Column - II (B) X – B (q) Energy of transition state
[R ] [R] (C) A – B (r) Activation energy of
(A) k = 0 (p) k = 2.303 log 2
t t forward reaction
12
2.303 [R ] (D) X (s) Activation energy of
(B) k = log 0 (q) Rate constant for zero backward reaction
t [R]
(a) A – (s), B – (r), C – (q), D – (p)
order reaction
(b) A – (q), B – (r), C – (p), D – (s)
(C) Value of k for first (r) k = [R]0 (c) A – (r), B – (s), C – (p), D – (q)
2t1 2 (d) A – (s), B – (r), C – (p), D – (q)
order reaction when
[R]0
CRITICAL THINKING TYPE QUESTIONS
t = t 1 and[R]
2 132. In the following reaction, how is the rate of appearance of
2
underlined product related to the rate of disappearance of
(D) Value of k for zero (s) Rate constant for first
the underlined reactant ?
order reaction when order reaction
BrO 3 (aq) 5Br (aq) 6H 3Br 2 (l) 3H 2 O(l)
[R]0
t = t1 2 and [R]
2 d[Br2 ] d[Br ] d[Br2 ] 3 d[Br ]
(a) (b)
(a) A – (s), B – (q), C – (p), D – (r) dt dt dt 5 dt
(b) A – (q), B – (s), C – (p), D – (r)
d[Br2 ] 3 d[Br ] d[Br2 ] 5 d[Br ]
(c) A – (q), B – (p), C – (s), D – (r) (c) (d)
(d) A – (q), B – (s), C – (p), D – (t) dt 5 dt dt 3 dt
CHEMICAL KINETICS 305
133. For the reaction A + B C + D. The variation of the Which of the above mechanism(s) can be consistent with
concentration of the products is given by the curve the given information about the reaction?
(a) A and B both (b) Neither A nor B
Y (c) A only (d) B only
Z 137. The hypothetical reaction A 2 B 2 2 AB ; follows
the following mechanism A 2 Fast
Conc A A,
Slow Fast
W A B2 AB B, A B AB .
X The order of the overall reaction is
Time (a) 0 (b) 1
(a) Z (b) Y (c) 2 (d) 3/2
138. The initial rates of reaction
(c) W (d) X
3A + 2B + C Products, at different initial
134. The rate of the reaction 2N2O5 4NO2 + O2 can be written
concentrations are given below:
in three ways :
Initial rate, [A]0, M [B]0, M [C]0, M
d[N 2O5 ]
k [N 2O5 ] Ms–1
dt 5.0 × 10–3 0.010 0.005 0.010
d[NO 2 ]
5.0 × 10–3 0.010 0.005 0.015
dt
k [N 2O5 ] 1.0 × 10–2 0.010 0.010 0.010
1.25 × 10–3 0.005 0.005 0.010
d[O 2 ] The order with respect to the reactants, A, B and C are
k [N 2O 5 ]
dt respectively
The relationship between k and k' and between k and k (a) 3, 2, 0 (b) 3, 2, 1
are: (c) 2, 2, 0 (d) 2, 1, 0
(a) k = 2k ; k = k (b) k = 2k ; k = k / 2 139. The rate law for the reaction 2X + Y Z is Rate =
(c) k = 2k ; k = 2k (d) k = k ; k = k k[X][Y]. The correct statement with regard to this relation
is
135. CHCl3 Cl2 CCl 4 HCl (a) the rate of the reaction is independent of [X] and [Y]
(b) for this reaction t1/2 is independent of initial
Rate law for above reaction will be
concentrations of reactant
1 (c) the rate of formation of Z is twice the rate of
Rate = k[CHCl3 ][Cl2 ]2 disappearance of X
On the basis of information provided which of the following (d) the rate of disappearance of X is equal to rate of
option will be correct ? disappearance of Y
140. The bromination of acetone that occurs in acid solution is
(a) Rate law for any chemical reaction can be predicted
represented by this equation.
accurately by looking at balanced chemical equation.
CH3COCH3 (aq) + Br2 (aq) CH3COCH2Br (aq) + H+ (aq)
(b) Rate law for a chemical reaction has to determine
+ Br– (aq)
experimentally.
These kinetic data were obtained for given reaction
(c) Either determined experimentally or obtained from
concentrations.
balanced chemical reaction, rate law will be same.
(d) None of the above is correct. Initial Initial rate,
136. The reaction of hydrogen and iodine monochloride is given Concentrations, M disappearance of
as: Br2, Ms–1
[CH3 COCH3] [Br2] [H+]
H 2 (g) 2ICl(g) 2HCl(g) I 2 (g) 0.30 0.05 0.05 5.7×10–5
The reaction is of first order with respect to H2(g) and ICI(g), 0.30 0.10 0.05 5.7 × 10–5
following mechanisms were proposed. 0.30 0.10 0.10 1.2 × 10–4
Mechanism A: 0.40 0.05 0.20 3.1 × 10–4
H 2 (g) 2ICl(g) 2HCl(g) I 2 (g) Based on given data, the rate equations is:
Mechanism B: (a) Rate = k[CH3COCH3][H+]
(b) Rate = k [CH3COCH3][Br2]
H 2 (g) ICl(g) HI(g);slow
(c) Rate = k [CH3COCH3] [Br2] [H+]2
HI(g) ICl(g) HCl(g) I 2 (g);fast (d) Rate = k [CH3COCH3][Br2] [H+]
EBD_7207
306 CHEMICAL KINETICS
141. Consider a reaction aG + bH Products. When 147. If half-life of a substance is 5 yrs, then the total amount of
concentration of both the reactants G and H is doubled, the substance left after 15 years, when initial amount is 64 grams
rate increases by eight times. However, when concentration is
of G is doubled keeping the concentration of H fixed, the (a) 16 grams (b) 2 grams
rate is doubled. The overall order of the reaction is (c) 32 grams (d) 8 grams.
(a) 0 (b) 1 148. In a 1st order reaction, reactant concentration C varies with
(c) 2 (d) 3 time t as :
142. Diazonium salt decomposes as 1
(a) increases linearly with t
C6 H 5 N 2 Cl C6 H5Cl N 2 C
(b) log C decreases linearly with t
At 0°C, the evolution of N2 becomes two times faster when
the initial concentration of the salt is doubled. Therefore, it 1
(c) C decreases with
is t
(a) a first order reaction 1
(d) log C decreases with
(b) a second order reaction t
(c) independent of the initial concentration of the salt 149. For the first order reaction
(d) a zero order reaction C 2 H 4 O(g) CH 4 (g) CO(g) ,the initial pressure of
143. Consider the following reaction at 25°C:
C2H4O(g) is 80 torr and total pressure at the end of 20
(CH3)3COH(l) + HCl(aq) (CH3)3CCl(l) + H2O(l)
minutes is 120 torr. The time needed for 75% decomposition
The experimentally determined rate law for this reaction of C2H4O would be :
indicates that the reaction is of first order in (CH3)3COH
(a) 20 minutes (b) 40 minutes
and that the reaction is of first order overall. Which of the
(c) 80 minutes (d) 120 minutes
following would produce an increase in the rate of this
150. Which of the following graph(s) is/are correct for a zero
reaction?
order reaction?
(a) Increasing the concentration of (CH3)3COH
(b) Increasing the concentration of HCl
(c) Decreasing the concentration of HCl
Reaction rate

(d) Decreasing the concentration of (CH3)3CCl (i)


144. The following data pertains to reaction between A and B :
S. No. [A] mol L–1 [B] mol L–1 Rate (mol L–1 time–1)
1 1.0 × 10–2 2.0 × 10–2 2.0 × 10–4 Time
2 2.0 × 10 –2 2.0 × 10 –2 4.0 × 10–4
3 2.0 × 10 –2 4.0 × 10 –2 8.0 × 10–4
Concentration
of reactant rate

Which of the following inference(s) can be drawn from the


above data ? (iii)
(i) Rate constant of the reaction is 1.0 × 10–4.
(ii) Rate law of the reaction is : rate = k[A][B]
(iii) Rate of reaction increases four times on doubling the Time
concentration of both the reactants.
Select the correct answer using the codes given below :
Reaction rate

(a) (i), (ii) and (iii) (b) (i) and (ii)


(c) (ii) and (iii) (d) (iii) only (iii)
145. The decomposition of ammonia on tungsten surface at 500 K
follows zero order kinetics. The half-life period of this
reaction is 45 minutes when the initial pressure is 4 bar. The Time
half-life period (minutes) of the reaction when the initial
pressure is 16 bar at the same temperature is
Concentration

(a) 120 (b) 60


of reactant rate

slope = –k
(c) 240 (d) 180
146. A substance 'A' decomposes by a first order reaction starting (iv)
initially with [A] = 2.00 M and after 200 min, [A] becomes
0.15 M. For this reaction t1/2 is Time
(a) 53.72 min (b) 50.49 min (a) (ii) and (iii) (b) (i), (ii) and (iii)
(c) 48.45 min (d) 46.45 min (c) (ii), (iii) and (iv) (d) (i) and (iv)
CHEMICAL KINETICS 307
151. The integrated rate equations can be determined for (a) The temperature of the system
(a) zero order reactions (b) The geometry or orientation of the collision
(b) first order reactions (c) The velocity of the reactants at the point of collision
(c) second order reactions (d) All of the above influence the rate
(d) Both (a) and (b) 158. The activation energy for a hypothetical
152. In a zero-order reaction for every 10° rise of temperature, reaction, A Product, is 12.49 kcal/mole. If temperature is
the rate is doubled. If the temperature is increased from raised from 295 to 305, the rate of reaction increased by
10°C to 100°C, the rate of the reaction will become : (a) 60% (b) 100%
(a) 256 times (b) 512 times (c) 50% (d) 20%
(c) 64 times (d) 128 times 159. A reactant (A) froms two products :
153. For a first order reaction A P, the temperature (T) k1
dependent rate constant (k) was found to follow the A B, Activation Energy Ea
1
k2
1 A C, Activation Energy Ea
equation log k = – (2000) 6.0 . The pre-exponential factor 2
T If Ea = 2 Ea , then k1 and k2 are related as :
2 1
A and the activation energy Ea, respectively, are
(a) 1.0 × 106 s–1 and 9.2 kJ mol–1 (a) k2 k1e Ea1 / RT (b) k2 k1e Ea2 / RT
(b) 6.0 s–1 and 16.6 kJ mol–1 k1 Ak 2 e Ea1 / RT k1 2k 2 e Ea2 / RT
(c) (d)
(c) 1.0 × 106 s–1 and 16.6 kJ mol–1
(d) 1.0 × 106 s–1 and 38.3 kJ mol–1 160. Which of the following graph(s) represents exothermic
reaction?
154. The activation energies of two reactions are E1 and
E2 (E1 > E2). If the temperature of the system is increased (A) Activated complex
from T1 to T2, the rate constant of the reactions changes
from k1 to k1' in the first reaction and k2 to k2' in the second
reaction. Predict which of the following expression is
Energy

correct?
Reactants
k1' k 2' k1' k '2 Products
(a) (b)
k1 k2 k1 k2 Reaction coordinate

k1' k 2' k1' k '2 (B) Activated complex


(c) (d) 1
k1 k2 k1 k2
155. A graph plotted between log k vs 1/T for calculating
Energy

activation energy is shown by


Products
Reactants
Reaction coordinate
(a) log k (b) log k
(C) Activated complex
1/T 1/T
Energy

(c) log k (d) log k Reactants Products

Reaction coordinate
1/T 1/T
(a) (A) only (b) (B) only
156. The rate constant, the activation energy and the arrhenius
(c) (C) only (d) (A) and (B)
parameter of a chemical reaction at 25°C are 3.0 × 10–4s–1,
161. Which of the following statements is not correct for the
104.4 kJ mol–1 and 6.0 × 1014 s–1 respectively. The value of
catalyst?
the rate constant as T is
(a) It catalyses the forward and backward reaction to the
(a) 2.0 × 1018 s–1 (b) 6.0 × 1014 s–1
same extent.
(c) Infinity (d) 3.6 × 1030 s–1
(b) It alters G of the reaction.
157. Collision theory is used to explain how chemical species
(c) It is a substance that does not change the equilibrium
undergo a reaction. Using this theory and the kinetic
constant of a reaction.
molecular model, which of the following does NOT influence
(d) It provides an alternate mechanism by reducing
the rate of a chemcial reaction?
activation energy between reactants and products.
EBD_7207
308 CHEMICAL KINETICS
162. Consider the reaction A f B. The concentration of both the
reactants and the products varies exponentially with time.
Which of the following figures correctly describes the [A]
change in concentration of reactants and products with
time?

Concentration
[B]
(d)
[B]
Concentration

Time
(a)
[A] 163. During decomposition of an activated complex.
Time (i) energy is always released
[B] (ii) energy is always absorbed
(iii) energy does not change
(iv) reactants may be formed
Concentration

(a) (i), (ii) and (iii) (b) (i) and (iv)


(b) (c) (ii) and (iii) (d) (ii), (iii) and (iv)
[A] 164. Which of the following statements is incorrect ?
Time (a) Energy is always released when activated complex
[B] decomposes to form products.
(b) Peak of the energy distribution curve corresponds to
the most probable potential energy.
Concentration

(c) Peak of the energy distribution curve corresponds to


(c) the most probable kinectic energy.
[A] (d) When the temperature is raised maximum of energy
distribution curve moves to higher energy value and
Time broadens out.
CHEMICAL KINETICS 309

FACT / DEFINITION TYPE QUESTIONS 24. (a) For a zero order reaction.
rate =k[A]º i.e. rate = k
1. (b) 2. (b) 3. (a) 4. (a) 5. (c) hence unit of k = M.sec–1
For a first order reaction.
d[C]
6. (d) In the given options will not represent the rate = k [A]
3dt k = M.sec–1/M = sec–1
reaction rate. It should not have –ve sign as it is product. 25. (d) The reaction involving two different reactant can never
1 dC be unimolecular.
since show the rate of formation of product C
3 dt 26. (b) For reaction 3A B C
which will be positive. If it is zero order reaction r = k [A]0, i.e the rate remains
7. (a) If we write rate of reaction in terms of concentration of same at any concentration of 'A'. i.e independent upon
NH3 and H2,then concentration of A.
1 d[ NH 3 ] 1 d[H 2 ] 27. (c) k = (mol lit –1 ) 1–n time –1 . For given reaction
Rate of reaction n = 2. k = mol–1 lit sec–1
2 dt 3 dt 28. (b) The order of a chemical reaction is given by
d[ NH 3 ] 2 d[H 2 ] concentration of reactants appearing in the lowest
So, step.
dt 3 dt
29. (b) Velocity constant ‘k’ is characteristic constant of a
d[C] reaction and depends only on temperature and catalyst.
8. (d) In the given options will not represent the
3.dt 30. (b) Given dx/dt = 2.400 × 10–5 mol litre–1 sec–1
k = 3.10 × 10–5 sec–1
reaction rate. It should not have –ve sign as it is product.
For first order reaction
1 dC 2N 2 O 5 2NO 2 O2
since show the rate of formation of product C
3 dt
dx
which will be positive. k [N 2 O 5 ]
9. (c) Rusting of iron is a slow change. dt
10. (c) The feasibility of a chemical reaction can be predicted or 2.4 × 10–5 = 3.0 × 10–5 [N2O5]
by thermodynamics. Extent to which a reaction will 2.4 10 5
proceed can be determined from chemical equilibrium. or [N2O5] = 5
= 0.8 mol. litre–1
Speed of a reaction i.e. time taken by a reaction to 3.0 10
reach equilibrium, can be predicted by chemical kinetics dx
31. (a) For zero order reaction, k [reactant]0
11. (c) 12. (d) 13. (c) 14. (a) 15. (b) dt
16. (c) 17. (c) 18. (d) Thus the rate of zero order reaction is independent of
19. (d) The rate of a reaction is the speed at which the concentration of reactants.
reactants are converted into products. It depends 32. (c) Rate1 = k [A]n [B]m; Rate2 = k [2A]n [½B]m
upon the concentration of reactants. e.g for the
reaction Rate 2 k [2A]n [½B]m
= [2]n [½]m = 2n.2–m = 2n–m
A + B ¾¾ ® Product ; r µ [A][B] Rate1 k [A]n [B]m
20. (a) k remains constant at constant temperature and CO 33. (a) 2A + B A2B
does not effect the rate of reaction. r1 = k[A]2 [B] ...(i)
21. (d) order of reaction may be zero, whole number or B
fractional. When, [A] = [2A], [B] =
2
22. (b) 2 NO (g) + Cl2(g) 2 NOCl(g)
Rate = k [NO]2 [Cl] 2 B 2 [B]
r2 = k 2A = k 4[A]
The value of rate constant can be increased by 2 2
increasing the temperature.
23. (d) Order of reaction is equal to the number of molecules = k 2[A]2[B] = 2r1 r1 k[A]2[B]
whose concentration is changing with time. It can be
zero or in fractions or an integer. Rate of reaction is increased two times.
EBD_7207
310 CHEMICAL KINETICS
34. (c) Let us consider a reaction, 44. (a) Let the rate law be r = k [A]x[B]y
xX+yY aA+bB 0.10 [0.024]x [0.035] y
rate = [X]x [Y]y Divide (3) by (1)
0.10 [0.012]x [0.035] y
It is given that order of reaction w.r.t. component Y is
zero. 1 = [2]x, x = 0
Hence, rate = [X]x 0.80 [0.024] x [0.070] y
i.e., rate becomes independent of the concentration of Y. Divide (2) by (3)
0.10 [0.024] x [0.035] y
0
35. (a) rate [preactant ] 8 = (2)y , y = 3
i.e., rate = k Hence, rate equation, R = k[A]0[B]3 =k[B]3
So, the order of reaction will be zero. 45. (a) Given r = k [A]3/2 [B]–1/2
3 1 2
36. (b) r k[A]n Order = 3/2 – 1/2 = =1
2 2
if n = 0 46. (b) The order w.r.t. I2 is zero because the rate is not
r k [A]0 dependent on the concentration of I2.
47. (c) As we know that, units of rate constant.
or r = k thus for zero order reactions rate is equal to
= (unit of conc.)1–n (unit of time)–1
t h e
= (mol L–1)1–n (sec)–1
rate constant.
On comparing these units with the given units of rate
37. (c) Since rate of reaction becomes four times on doubling
constant, we get
concentration of reactant, it is second order
(mol L–1)1–n (sec)–1 = L mol–1 sec–1
reaction.
38. (b) Order is the sum of the power of the concentration Þ Ln–1 mol1–n sec–1 = L mol–1 sec–1
terms in rate law expression. On comparing the powers, we get
Hence the order or reaction is = 1 + 2 = 3 n–1=1 Þ n=2
39. (a) Rate = k[A]° So, reaction is of second order.
Unit of k = mol L–1 sec–1 48. (d) In case of (II) and (III), keeping concentration of [A]
constant, when the concentration of [B] is doubled,
1 the rate quadruples. Hence it is second order with
40. (c) N 2O N2 O2
2 respect to B. In case of I & IV Keeping the
concentration of [B] constant. when the concentration
dx
[N 2 O]1 of [A] is increased four times, rate also increases four
dt times. Hence, the order with respect to A is one. hence
i.e. order of reaction = 1
Rate = k [A] [B]2
41. (a) From the first set of data (i) and (ii) it is observed that
3
on keeping concentration of [B] constant and on 49. (a) The order of reaction is and molecularity is 2.
doubling the concentration of [A] rate does not 2
changes hence order of reaction with respect to A is 50. (b) O3
Fast
O2 O; O O 3
Slow
2O 2
zero.
From the second set of data (i) and (iii) it is observed [O 2 ][O]
k (I) Rate = k' [O3][O] put [O] from (I)
that rate becomes 4 times on doubling the concentration [O 3 ]
of [B] keeping [A] constant hence order with respect to
k '[O 3 ]K[O 3 ]
[B] will be 2 r k[O 3 ]2 [O 2 ] 1
rate = k[A]0 [B]2 [O 2 ]
42. (c) Order is the sum of the powers to which the Note intermediates are never represented in rate law
concentration terms are raised in the rate equation. equation.
43. (d) If rate = k[A]x [B]y [C]z 51. (c)
From first two given data 52. (c) Thermal decomposition of HI on gold surface is an
8.08 × 10–3 = k [0.2]x [0.1]y [0.02]z .... (1) example of zero order reaction.
2.01 × 10–3 = k [0.1]x [0.2]y [0.02]z .... (2) 53. (c) For zero order reaction,
Divide (1) by (2) we get, 4 = 2x (1/2)y rate, r = k[R]°
Similarly, from second and third data dR
(9)y (9)z = 3 k
dt
2y + 2z = 1 . R = kt + R0
From first and fourth data 4z = 8 = 23 where R0 is the concentration of reactant at time t = 0.
2z = 3. So z = 3/2, y = – 1, x = 1 Thus [R] increases with time
CHEMICAL KINETICS 311
54. (a) Plots of conc. [A] Vs time, t n
1
61. (c) [A] [A]0
Zero First 2
order order
1 1
log [A]
where [A]0 = initial concentration
[A] Second Third
[ A] [ A]2
order order n
t t t 1 1
1 ;n=3
8 2
1
55. (d) t1/2
a2 t1/ 2
0.693
3 = 100 sec
6.9 10
1
We know that t1/ 2 n 1 T n t1/ 2 = 3 × 100 = 300 sec.
a
i.e. n = 3 62. (b) Given: 75% reaction gets completed in 32 min
Thus reaction is of 3rd order. 2.303 a 2.303 100
56. (a) For a zero order reaction Thus, k = log = log
t (a x ) 32 (100 75)
a 2.303
t1/ 2 = log 4 = 0.0433 min–1
2k 32
57. (a) t1/2 = 4s T = 16s Now we can use this value of k to get the value of time
T 16 required for 50% completion of reaction
n 4 ( T = n × t½)
t1/ 2 4 2.303 a 2.303 100
t= log = log
k (a x ) 0.0433 50
æ 1 ön æ 1 ö4 0.12
[ A] = [ A]o çç ÷÷÷ = 0.12´çç ÷÷÷ = = 0.0075 M
çè 2 ø èç 2 ø 16 2.303
= log 2 = 16 min
0.0433
Where [A] o = initial concentration an d [A]
= concentration left after time t 63. (d) Unit of k for Ist order reaction is (Time)–1
58. (b) A B For a first order reaction 2.303 1 2.303 4
64. (c) t1/ 4 log log
Given a = 0.8 mol, (a – x) = 0.8 – 0.6 = 0.2 k 3/ 4 k 3
2.303 0.8 2.303 2.303
k log or k = 2.303 log 4 (log 4 log 3) (2log 2 log 3)
1 0.2 k k
again a = 0.9, a – x = 0.9 – 0.675 = 0.225
2.303 0.29
2.303 0.9 (2 0.301 0.4771)
k log k k
t 0.225 65. (c) For a first order reaction
2.303 2.303 a
2.303log 4 log 4 k log
t t a x
Hence t = 1 hour when t = t½
59. (b) For a first order reaction, A products
2.303 a
r k log
r k[A] or k t½ a a/2
[A]
2.303 ln 2
2 or t½ log 2
1.5 10 k k
k = 3 × 10–2
0.5 66. (b) Since doubling the concentration of B does not change
half life, the reaction is of 1st order w.r.t. B.
0.693 0.693
Further, t1/ 2 2
23.1 Order of reaction with respect to A = 1 because rate of
k 3 10 reaction doubles when concentration of A is doubled
0.693 keeping concentration of B constant.
60. (c) For first order reaction, k Order of reaction = 1 + 1 = 2 and units of second
t1/ 2
order reaction are L mol–1 sec–1.
where k = rate constant 67. (c) Half life time (t1/2) for nth order reaction is given by,
t1/2 = half life period = 480 sec. t1/2 [a]1– n
0.693 where n is the order of reaction and a is concentration
k = 1.44 × 10–3 sec–1 of reactant.
480
EBD_7207
312 CHEMICAL KINETICS
As decomposition of N2O5 follows 1st order kinetic. 77. (b) As the rate of reaction get doubled for every 10°C rise
So, in temperature. Hence the increase in reaction rate as a
t1/2 [a]1 – 1 t1/2 a0 result of temperature rise from 10°C to 100°C is equal
68. (c) For a first order reaction to = 29 = 512
2.303 a 78. (a) As per Arrhenius equation (k Ae Ea / RT ) , the rate
k log10
t a x constant increases exponentially with temperature.
when t = t½ 79. (b) For a reversible reaction,
2.303 a H = Ea (forward) – Ea (backward)
k log10 H = 150 – 260 = –110 kJ mol–1
t½ a a/2
80. (c) We know that the activation energy of chemical
2.303 ln 2 r e a c t i o n
or t ½ log10 2
k k k2 Ea T2 T1
is given by formula = , where
2.303 a k1 2.303R T1T2
69. (a) t log
k a x k1 is the rate constant at temperature T1 and k2 is the
2.303 2.303 rate constant at temperature T2 and Ea is the activation
or t log a log(a x)
k k energy. Therefore activation energy of chemical
reaction is determined by evaluating rate constant at
n 1 n 1
(t1/ 2 )1 a2 120 4 10 2 two different temperatures.
70. (c) ; ;n 2
(t1/ 2 )2 a1 240 8 10 2 81. (b) In equation k = Ae Ea / RT ; A = Frequency factor
71. (b) k = velocity constant, R = gas constant and Ea = energy
72. (b) In Haber’s process, ammonia is manufactured from N2 of activation
and H2 using iron as catalyst with molybdenum as 82. (d) In Arrhenius equation k = Ae Ea / RT , Ea is the energy
promoter at high temperature and pressure of activation, which is required by the colliding
Fe 2O3 (catalyst) molecules to react resulting in the formation of
N 2 3H 2 2NH3
Mo (catalytic promoter) products.
83. (d)
k Ea 1 1
73. (b) log 2 = 84. (b) When the temperature is increased, energy in form of
k1 2.303R T1 T2 heat is supplied which increases the kinetic energy of
the reacting molecules. This will increase the number
Ea 1 1 of collisions and ultimately the rate of reaction will be
log 2 =
2.303 8.314 293 308 enhanced.
85. (a) Arrhenius equation is given by
Ea 15
0.3 =
2.303 8.314
×
293 308 k = Ae Ea /(2.303 RT )
Taking log on both sides, we get
0.3 2.303 8.314 293 308
Ea = . Ea
15 log k = log A –
2.303 RT
= 34673 J mole–1
= 34.7 J mole–1 1
Arrhenius plot a graph between log k and whose
T
74. (b) H = Ea f Ea b = 0
Ea
75. (d) The activation energy depends upon the nature of slope is .
2.303 R
chemical bonds undergoing rupture during chemical
reaction and is independent of enthalpies of reactants k2 E a T2 T1
and products. 86. (a) 2.303log
k1 R T1T2
E /RT
76. (b) k = Ae– a
lnk = ln A – Ea/RT k2 9.0 103 308 298
log
For ln k vs 1/T k1 2.303 2 308 298
ln A = intercept
– Ea/R = slope = –2 × 104 K k2
1.63; k 2 1.63k1;
Ea = 8.3 × 2 × 104 J mol–1 k1
= 16.6 × 104 J mol–1 or 166 kJ mol–1
CHEMICAL KINETICS 313

k2 k1 108. (b) For a 10 K rise in temperature, collision frequency


Increase in k1 100 increases merely by 1 to 2% but the number of
k1
effective collisions increases by 100 to 200%.
1.63k1 k1
= 100 63.0% 109. (a) 110. (b) 111. (c)
k1

87. (b) H Ea ( f ) Ea (b) STATEMENT TYPE QUESTIONS


Thus energy of activation for reverse reaction depend 112. (b) For the given reaction
upon whether reaction is exothermic or pt(s)
endothermic. 4NH 3 (g) 5O 2 (g) 4NO(g) 6H 2O(g)
If reaction is exothermic, H ve , Ea(b) Ea( f ) NH 3 O2 NO H 2O
1 1 1 1
Rate = =
If reaction is endothermic, H ve Ea(b) Ea ( f ) 4 t 5 t 4 t 6 t

88. (d) 89. (c) 90. (c) 91. (a) 92. (d) 113. (a) For given reaction x and y may or may not be equal to
93. (d) p and q respectively.
114. (a) Average rate depends upon the chan ge in
94. (d) k Ae E a / RT
concentration of reactants or products and the time
Ea taken for that change to occur. Average rate cannot
95. (b) ln k = ln A – , intercept is ln A.
RT be used to predict the rate of a reaction at a particular
96. (c) The rate constant doubles for 10º C rise in temperature. instant as it would be constant for the time interval for
For 20º C rise, the rate constant will be 4 times which it is calculated. So, to express the rate at a
k1 = 4k2 or k2 = 0.25 K1 particular moment of time we determine the
Ea 1 instantaneous rate. It is obtained when we consider
97. (d) k Ae E a / RT log k = log A – . the average rate at the smallest time interval say dt (
2.303R T
i.e. when it approaches zero).
Ea
Equation of straight line slope = . 115. (d) Molecularity is defined as the number of reacting
2.303R
species taking part in an elementary reaction,
98. (c) Applicable to bimolecular reactions. 116. (b) Molecularity cannot be fractional or zero. Correct units
99. (a)
of rate constant for second order reaction are mol–1 Ls–1.
100. (d) All the statements are correct (see text).
117. (d) For zero order reaction, rate of reaction as well as rate
101. (c) Activation energy is lowered in presence of +ve
constant are independent of reactant concentration.
catalyst.
102. (c) [R]0
103. (a) Threshold Energy = Energy of activation + Internal 118. (a) For a first order reaction log varies linearly with
[R]
energy
time as shown below.
Ea 1
104. (b) k Ae E a / RT log k = log A .
2.303R T

1 slop = k/2.303
[R]0
[R]

Plot of log k Vs.


T
log

Ea
Straight line Slope = 0
2.303R Time
105. (d) Enthalpy of reaction ( H) = Ea – Ea 119. (a)
(f) (b)
for an endothermic reaction H = +ve hence for H to 120. (b) According to Arrhenius equation, k = Ae–Ea/RT
be positive when Ea = 0, k = A
Ea < Ea
(b) (f)
106. (a) Presence of catalyst does not affect enthalpy change Also ln k s 1/T is a straight line with slope = –Ea/R.
of reaction H R E f E b = 180 – 200 = – 20 kJ/mol Statements (ii) and (v) are correct.
121. (b)
107. (c) For the exothermic reaction the energy of products is
always less than the reactants. If Ea is the energy of 122. (b) Rate constant gets doubled with every 10°C in
activation for the forward reaction, the energy of temperature for chemical change only not for physical
activation for backward reaction is Ea + H change.
EBD_7207
314 CHEMICAL KINETICS
123. (b) Correct formula for calculation of Ea is 137. (d) A2 B2 2AB ;
k Ea T2 T1
log 2 A2 A A ( Fast );
k1 2.303R T1T2
The word catalyst should not be used when the added A B2 AB B (Slow )
substance reduces the rate of reaction. The substance
is then called inhibiter. Rate law = k[A][B 2 ] put value of [A] from Ist reaction
124. (d) The relevant expressions are as follows.
since A is intermediate k[A 2 ] A
H 1
log Kp = I
R T Rate law equation K k[A 2 ][B2 ]
log [X] = log [X]0 + kt
P/T = constant (V constant) 1 3
Order 1
PV = constant (T constant) 2 2
138. (d) From 1st and 2nd sets of data - no change in rate is
MATCHING TYPE QUESTIONS
observed with the change in concentration of ‘C’. So
125. (a) 126. (b) 127. (a) 128. (c) 129. (b) the order with respect to ‘C’ is zero.
130. (a) 131. (d) From 1st and 4th sets of data
Dividing eq. (4) by eq. (1)
CRITICAL THINKING TYPE QUESTIONS
3 x
1.25 10 0.005
1 d[Br ] 1 d[Br2 ] 3 0.010
132. (c) 5.0 10
5 dt 3 dt
or 0.25 = (0.5)x or (0.5)2 = (0.5)x
d[Br2 ] 3 d[Br ] x=2
dt 5 dt The order with respect to ‘A’ is 2 from the 1st and 3rd
133. (b) The curve Y shows the increase in concentration of sets of data dividing eq. (1) by eq. (3)
products with time.
3 y
134. (b) Rate of disappearance of reactants = Rate of appearance 5.0 10 0.005
of products 2 0.010
1.0 10
1 d(N 2 O5 ) 1 d(NO2 ) d(O 2 ) or (0.5)1 = (0.5)y y= 1
2 dt 4 dt dt The order with respect to ‘B’ is 1
1 1 So the order with respective the reactants A, B and C
k(N 2 O5 ) k (N 2 O5 ) k (N 2 O5 )
2 4 is 2, 1 and 0.
k k 139. (N) None of the given options is correct.
k
2 4 The given reaction is : 2X + Y — Z
k d[X] d[Z]
k = 2k, k –
2 2dt dt
135. (b) Rate law has to be determined experimentally as Cl2 is Rate of formation of Z is half of the rate of
1 disappearance of X.
raised to power in rate law whereas its stichiometric
2 d[X] d[Y]
coefficient in balanced chemical equation is 1. 2dt dt
136. (d) As the slowest step is the rate determining step thus
Rate of disappearrance of X is not equal to rate of
the mechanism B will be more consistent with the given
disappearance of Y.
information also because it involve one molecule of
H2 and one molecule of ICl it can expressed as 140. (a) Rewriting the given data for the reaction
r = k [H2][ICl] H
CH 3COCH 3 ( aq ) Br2 ( aq )
Which shows that the reaction is first order w.r.t. both
H2 & ICl. CH3COCH 2 Br(aq) H ( aq) Br ( aq)
CHEMICAL KINETICS 315

S. Initial concent Initialconcentr Initialconcentr Rate of d [ A]


No. -ration of -ation of Br2 -ation of H disappearance
144. (c) Rate law : k [ A]x [ B ] y
CH3COCH3 in M
dt
in M of Br2 in Ms 1
in M
Doubling [A], rate is doubled. Hence 2x = 2, x =1
d dx
i.e. [Br2 ]or d [ A]
dt dt Similarly y = 1; k [ A][ B ]
dt
1 0.30 0.05 0.05 5.7 10 5

2 0.30 0.10 0.05 5.7 10 5 rate 2.0 10 4


k 1
3 0.30 0.10 0.10 1.2 10 4 [ A][ B ] 1 10 2 2 10 2
4 0.40 0.05 0.20 3.1 10 4
(rate)2 k (2[ A])(2[b])
4
Actually this reaction is autocatalyzed and involves (rate)1 k[ A][ B ]
complex calculation for concentration terms.
145. (d) For a zero order reaction,
We can look at the above results in a simple way to t1/2 a0 (initial concentration or initial pressure)
find the dependence of reaction rate (i.e., rate of (t1/2)1 P1
disappearance of Br2). (t1/2)2 P2
From data (1) and (2) in which concentration of
t1/2 P2 t1/2 2 16
CH3COCH3 and H+ remain unchanged and only the 2 ,
concentration of Br 2 is doubled, there is no change in t1/2 1
P1 45 4
rate of reaction. It means the rate of reaction is
16
independent of concentration of Br2. (t1/2)2 = 45 180 min
Again from (2) and (3) in which (CH3CO CH3) and 4
146. (a) Given initial concentration (a) = 2.00 M; Time taken
(Br2) remain constant but H+ increases from 0.05 M to
(t) = 200 min and final concentration (a – x) = 0.15 M.
0.10 i.e. doubled, the rate of reaction changes from
For a first order reaction rate constant,
5.7×10–5 to 1.2 × 10–4 (or 12 × 10–5), thus it also
becomes almost doubled. It shows that rate of reaction 2.303 a 2.303 2.00
k log log
is directly proportional to [H+]. From (3) and (4), the t a x 200 0.15
rate should have doubled due to increase in conc of 2.303
[H+] from 0.10 M to 0.20 M but the rate has changed (0.301 0.824) 1.29 10 2 min 1 .
200
from 1.2× 10–4 to 3.1×10–4. This is due to change in
Further
concentration of CH3COCH3 from 0.30 M to 0.40 M.
Thus the rate is directly proportional to [CH3 COCH3]. 0.693 0.693
(t1/ 2 ) 53.72 min .
We now get k 1.29 10 2
rate = k [CH3COCH3]1[Br2]0[H+]1 147. (d) t1/2 = 5 years, T = 15 years hence total number of half
= k [CH3COCH3][H+]. 15
life periods 3.
141. (d) Overall order = sum of orders w.r.t each reactant. 5
Let the order be x and y for G and H respectively
64
Amount left 8g
[G]mole [H]mole rate(mole ( 2) 3
Exp.No. 1 1
litre litre litre time 1 )
Slope k / 2.303
1 a b r
2 2a 2b 8r 148. (b)
log C
3 2a b 2r

For (1) and (3), the rate is doubled when conc. of t


G is doubled keeping that of H constant i.e.,
149. (b) Let x torr of C2H4O decompose after 20 min. Then,
rate [G] x=1 80 – x + 2x =120 ; x = 40 torr = 50% of initial pressure.
From (2) and (3), y = 2 Hence t1/2 = 20 min. For 75% reaction, fraction left
Overall order is 3. 2
25 1 1
142. (a) As doubling the initial conc. doubles the rate of =
100 4 2
reaction, order =1
143. (a) No. of half lives = 2. Time needed for 75% reaction.,
2 × 20 = 40 min
EBD_7207
316 CHEMICAL KINETICS
150. (d) 156. (b) T2 = T (say), T1 = 25°C = 298K,
151. (d) The integrated rate equations are different for the Ea = 104.4 kJ mol–1 = 104.4 × 103 J mol–1
reactions of different reaction orders. We shall k1=3 × 10 – 4, k2 = ?,
determine these equations only for zero and first order
chemical reactions. k2 Ea 1 1
log
k1 2.303 R T1 T2
r T2 T1 100 10
100 C
152. (b) 2 10 2 10
29 512 (where 2 is
r
10 C
k2 104.4 103 J mol 1
1
1
log 4 1 1
3 10 2.303 (8.314 J K mol ) 298 T
temperature coefficient of reaction)
Ea 1
153. (d) log k log A …(1) As T , 0
2.303RT T
1 k2 104.4 103 J mol 1
Also given log k 6.0 (2000) …(2) log 4
T 3 10 2.303 8.314 298
On comparing equations, (1) and (2)
k2 k2
log A = 6.0 A = 106 s–1 log 18.297, 1.98 1018
3 10 4 3 10 4
Ea
and 2000 ;
2.303 R k2 (1.98 1018 ) (3 10 4 ) 6 1014 s 1

Ea = 2000 × 2.303 × 8.314 = 38.29 kJ mol–1 157. (d)


154. (b) We know more will be activation energy lesser will be 158. (b) For 10°C rise of temperature the rate is almost doubled.
rate constant. Thus if E 1 > E 2 then k 1 < k2 . As Ea / RT
temperature increases, it will effect both rates in similar 159. (c) k1 A1e 1 .........(i)
way.
Ea / RT
1 k2 A2e 2 ........(ii)
155. (b) A graph plotted between log k vs for calculating
T On dividing eqn (i) from eqn. (ii)
activation energy is shown as
k1 A1
( Ea2 Ea1 ) / RT ........(iii)
k2 A2
log k
Given Ea2 = 2Ea1
1/T On substituting this value in eqn. (iii)
from Arrhenius equation Ea / RT
k1 k2 A e 1
Ea
log k log A 160. (a) 161. (b) 162. (b) 163. (b) 164. (b)
2.303 RT
19
SURFACE CHEMISTRY

FACT / DEFINITION TYPE QUESTIONS (a) HCl (b) O2


(c) CO2 (d) NH3
1. Adsorbed acetic acid on activated charcoal is: 10. Adsorption is accompanied by
(a) adsorber (b) absorber (a) decrease in enthalpy and increase in entropy
(c) adsorbent (d) adsorbate (b) increase in enthalpy and increase in entropy
2. Adsorption is always (c) decrease in enthalpy and decrease in entropy
(a) endothermic (d) increase in enthalpy and decrease in entropy
(b) exothermic 11. Choose the incorrect statement in respect of physisorption?
(c) exothermic in case of physical and endothermic in case (a) It is not specific in nature
of chemical (b) It arises because of van der Waal’s force
(d) Either (a) or (b) (c) It is reversible in nature
3. Which is not correct regarding the physical adsorption of a (d) Enthalpy of adsorption is in the range 80-240 kJ mol–1
gas on surface of solid ? 12. The term ‘sorption’ stands for ___________.
(a) On increasing temperature, adsorption increases (a) absorption
continuously (b) adsorption
(b) Enthalpy and entropy changes are negative (c) both absorption and adsorption
(c) Adsorption is more for some specific substance (d) desorption
(d) Reversible 13. Extent of physisorption of a gas increases with _______.
4. How many layers are adsorbed in chemical adsorption ? (a) increase in temperature.
(a) One (b) Two (b) decrease in temperature.
(c) Many (d) Zero (c) decrease in surface area of adsorbent.
5. Adsorption due to strong chemical forces is called (d) decrease in strength of van der Waal’s forces.
(a) Chemisorption (b) Physisorption 14. Extent of adsorption of adsorbate from solution phase
(c) Reversible adsorption (d) Both (b) and (c) increases with _________.
6. In physical adsorption, gas molecules are bound on the (a) increase in amount of adsorbate in solution.
solid surface by (b) decrease in surface area of adsorbent.
(a) chemical forces (b) electrostatic forces (c) increase in temperature of solution.
(c) gravitational forces (d) van der Waal’s forces (d) decrease in amount of adsorbate in solution.
7. Which of the following statements is not correct ? 15. Which of the following is not a favourable condition for
(a) Physical adsorption is due to van der Waal’s forces physical adsorption ?
(b) Chemical adsorption first decreases with increase in (a) High pressure
temperature. (b) Negative H
(c) Physical adsorption is reversible (c) Higher critical temperature of adsorbate
(d) Adsorption energy for a chemical adsorption is (d) High temperature
generally greater than that of physical adsorption. 16. Physical adsorption of a gaseous species may change to
8. Adsorption of gases on solid surface is exothermic reaction chemical adsorption with ________.
because (a) decrease in temperature
(a) free energy increases (b) enthalpy is positive (b) increase in temperature
(c) entropy increases (d) enthalpy is negative (c) increase in surface area of adsorbent
9. The gas which is least adsorbed on charcoal (under identical (d) decrease in surface area of adsorbent
conditions) is
EBD_7207
318 SURFACE CHEMISTRY
17. In physisorption adsorbent does not show specificity for 26. Which is not correct regarding the adsorption of a gas on
any particular gas because _________. surface of solid?
(a) involved van der Waal’s forces are universal. (a) On increasing temperature, adsorption increases
(b) gases involved behave like ideal gases. continuously
(c) enthalpy of adsorption is low. (b) Enthalpy and entropy changes are –ve
(d) it is a reversible process. (c) Adsorption is more for some specific substance
18. Which of the following is an example of absorption ? (d) This Phenomenon is reversible
(a) Water on silica gel . 27. Which of the following curves is in accordance with
(b) Water on calcium chloride. Freundlich adsorption isotherm ?
(c) Hydrogen on finely divided nickel.
(a) (b)
(d) Oxygen on metal surface.
19. For adsorption of a gas on a solid, the plot of log x/m vs

log x/m
log x/m
log P is linear with slope equal to (n being whole number)
(a) k (b) log k
1 p p
(c) n (d)
n
20. The adsorption of a gas on a solid surface varies with (c) (d)
pressure of the gas in which of the following manner

log x/m
(a) Fast slow independent of the pressure

log x/m
(b) Slow fast independent of the pressure
(c) Independent of the pressure fast slow
(d) Independent of the pressure slow fast log p log p
21. If x is amount of adsorbate and m is amount of adsorbent, 28. Which of the following is related to adsorption?
which of the following relations is not related to adsorption (i) H = – ve (ii) S = – ve
process ? (iii) –T S = – ve (iv) G = – ve
(a) x / m = f (p) at constant T. (a) (i), (ii) and (iv) (b) (ii) and (iii)
(b) x / m = f (T) at constant p. (c) (iii) only (d) (i), (iii) and (iv)
(c) p = f (T) at constant (x / m). 29. The role of a catalyst in a reversible reaction is to
x (a) increase the rate of forward reaction
(d) p T
m (b) decrease the rate of backward reaction
22. In Freundlich adsorption isotherm, the value of 1/n is : (c) alter the equilibrium constant of the reaction
(a) between 0 and 1 in all cases (d) allow the equilibrium to be achieved quickly
(b) between 2 and 4 in all cases 30. Catalytic poisons act by :
(c) 1 in case of physical adsorption (a) making the products chemically inactive.
(d) 1 in case of chemisorption (b) increasing the rate of the backward reaction.
23. Which is adsorbed in maximum amount by activated charcoal (c) chemical combination with any one of the reactants.
? (d) preferential adsorption on the catalyst surface.
(a) N2 (b) CO2 31. A catalyst :
(c) Cl2 (d) O2 (a) lowers the activation energy
24. Freundlich equation for adsorption of gases (in amount of x g) (b) changes the rate constant
on a solid (in amount of m g) at constant temperature can be (c) changes the product
expressed as
(d) itself destroyed in the reaction
x 1 x 1 32. Active charcoal is a good catalyst because it
(a) log log p log K (b) log log K log p
m n m n (a) is made up of carbon atoms.
(b) is very reactive.
x x 1
(c) pn (d) log p log K (c) has more adsorption power.
m m n (d) has inert nature toward reagents.
25. According to Freundlich adsorption isotherm, the amount 33. Which of the following kind of catalysis can be explained
of gas adsorbed at very high pressure by the adsorption theory ?
(a) reaches a constant limiting value (a) Homogeneous catalysis
(b) goes on increasing with pressure (b) Acid - base catalysis
(c) goes on decreasing with pressure (c) Heterogeneous catalysis
(d) increase first and decreases later with pressure (d) Enzyme catalysis
SURFACE CHEMISTRY 319
34. According to the adsorption theory of catalysis, the speed 43. What is the role of molybdenum in Haber’s process for
of the reaction increases because- manufacture of ammonia?
(a) Adsorption lowers the activation energy of the reaction (a) As catalytic poison (b) As a catalytic promoter
(b) The concentration of reactant molecules at the active (c) As a catalyst (d) As a reactant
centres of the catalyst becomes high due to strong 44. Which of the following step(s) is/are not involved in the
adsorption mechanism of adsorption theory of heterogeneous
(c) In the process of adsorption, the activation energy of catalyst?
the molecules becomes large (i) Diffusion of reactants to the surface of the catalyst.
(d) Adsorption produces heat which increases the speed (ii) Sorption of reactant molecules on the surface of the
of the reaction catalyst.
35. Catalyst increases the rate of reaction by (iii) Occurrence of chemical reaction on the catalyst’s
(a) decreasing threshold energy surface through formation of an intermediate.
(b) decreasing activation energy (iv) Desorption of reaction products from the catalyst’s
(c) increasing activation energy surface.
(d) decreasing equilibrium constant (v) Diffusion of reaction products away from the catalyst’s
36. A catalyst can affect reversible reaction by surface.
(a) changing equilibrium constant (a) (i) only (b) (ii) and (iv)
(b) slowing forward reaction (c) (ii) only (d) (i), (ii) and (v)
(c) attaining equilibria in both directions 45. Which of the following equation does not represent
(d) None of these homogeneous catalysis?
37. Which one of the following is an example of homogeneous H 2SO 4 l
catalysis ? (a) CH3 COOCH3 l H2O l
(a) Haber’s process of synthesis of ammonia
CH3COOH aq CH3OH aq
(b) Catalytic conversion of SO2 to SO3 in contact process
(c) Catalytic hydrogenation of oils (b) Pt s
4NH3 g 5O 2 g 4NO g 6H 2O g
(d) Acid hydrolysis of methyl acetate.
NO g
38. Identify the correct statement regarding enzymes (c) 2SO2 g O2 g 2SO 3 g
(a) Enzymes are specific biological catalysts that cannot (d) Both (a) and (b)
be poisoned. 46. Milk is a colloid in which a
(b) Enzymes are normally heterogeneous catalysts that
(a) liquid is dispersed in a liquid
are very specific in their action.
(b) solid is dispersed in a liquid
(c) Enzymes are specific biological catalysts that can
(c) gas is dispersed in a liquid
normally function at very high temperatures (T
1000K). (d) sugar is dispersed in a liquid
(d) Enzymes are specific biological catalysts that possess 47. Butter is a colloid formed when
well-defined active sites. (a) Fat is dispersed in water
39. A biological catalyst is (b) Fat globules are dispersed in water
(a) an enzyme (b) a carbohydrate (c) Water is dispersed in fat
(c) an amino acid (d) a nitrogenous base (d) None of the above
40. The action of enzymes in living system is to : 48. The size of colloidal particles is between
(a) supply energy to tissues (a) 10–7 – 10–9 cm (b) 10–9 – 10–11 cm
–5
(c) 10 – 10 cm –7 (d) 10–2 – 10–3 cm
(b) enhance immunity
(c) circulate oxygen 49. An aerosol is a :
(d) enhance the rate of biochemical reactions. (a) dispersion of a solid or liquid in a gas
41. Hydrolysis of urea is an example of (b) dispersion of a solid in a liquid
(a) homogenous catalysis (b) heterogenous catalysis (c) dispersion of a liquid in a liquid
(c) biochemical catalysis (d) zeolite catalysis (d) solid solution
42. The efficiency of an enzyme in catalysing a reaction is due 50. An example of dispersion of a liquid in a gas is :
to its capacity (a) milk (b) vegetable oil
(a) to form a strong enzyme-substrate complex (c) foam (d) mist
(b) to decrease the bond energies of substrate molecule 51. Alloy is an example of
(c) to change the shape of the substrate molecule (a) gel (b) solidified emulsion
(d) to lower the activation energy of the reaction (c) solid solution (d) sol
EBD_7207
320 SURFACE CHEMISTRY
52. If dispersed phase is a liquid and the dispersion medium is 66. Surface tension of lyophilic sols is
a solid, the colloid is known as (a) lower than that of H2O
(a) a sol (b) a gel (b) more than that of H2O
(c) an emulsion (d) a foam (c) equal to that of H2O
53. Hair cream is an example of (d) either less or more than H2O depending upon the
(a) gel (b) sol nature of disperse phase
(c) aerosol (d) foam 67. Which of the following is a lyophilic colloid ?
54. Which one of the following is correctly matched? (a) Milk (b) Gum
(a) Emulsion-smoke (b) Gel-butter (c) Fog (d) Blood
(c) Aerosol-hair cream (d) Sol-whipped cream 68. Lyophobic colloids are :
55. Cheese is an example of (a) gun proteins (b) protective colloids
(a) solid sol (b) emulsion (c) irreversible colloids (d) reversible colloids
(c) gel (d) foam 69. Which one is an example of multimolecular colloid system
56. Which one of the following in not a colloidal solution? (a) Soap dispersed in water
(a) Smoke (b) Ink (b) Protein dispersed in water
(c) Blood (d) Air (c) Gold dispersed in water
57. Small liquid droplets dispersed in another liquid is called (d) Gum dispersed in water
(a) gel (b) suspension 70. Example of intrinsic colloid is
(c) emulsion (d) true solution (a) glue (b) sulphur
58. When dispersed phase is liquid and dispersion medium is (c) Fe (d) As 2 S 3
gas then the colloidal system is called 71. Associated colloid among the following is
(a) Smoke (b) Clouds (a) enzymes (b) proteins
(c) Jellies (d) Emulsions (c) cellulose (d) sodium stearate
59. Which one is a colloid? 72. The formation of micelles takes place only above
(a) Sodium chloride (b) Urea (a) inversion temperature
(c) Cane sugar (d) Blood (b) Boyle temperature
60. Suspensions are (c) critical temperature
(a) Visible to naked eye (d) Kraft temperature
(b) Not visible by any means 73. A precipitate is changed to colloidal solution by the
(c) Invisible under electron microscope following process :
(d) Invisible through microscope (a) dialysis (b) ultrafiltration
61. Cloud or fog is an example of colloidal system of (c) peptization (d) electrophoresis
(a) Liquid dispersed in gas 74. Which of the following is used for neutralising charge on
(b) Gas dispersed in gas colloidal solution?
(c) Solid dispersed in gas (a) Electrons
(d) Solid dispersed in liquid (b) Electrolytes
62. A colloid always : (c) Positively charged ions
(a) Contains two phases (d) Compounds
(b) Is a true solution 75. Pure water can be obtained from sea water by
(c) Contains three phases (a) Centrifugation (b) Plasmolysis
(d) Contains only water soluble particles (c) Reverse osmosis (d) Sedimentation
63. Which one of the following is correctly matched ? 76. Blood may be purified by
(a) Emulsion - curd (b) Foam - mist (a) Dialysis (b) Electro-osmosis
(c) Aerosol - smoke (d) Solid sol - cake (c) Coagulation (d) Filtration
64. At the critical micelle concentration (CMC) the surfactant 77. During dialysis
molecules (a) only solvent molecules can diffuse
(a) decompose (b) solvent molecules, ions and colloidal particles can
(b) dissociate diffuse
(c) associate (c) all kinds of particles can diffuse through the semi-
(d) become completely soluble permeable membrane
65. How non-polar and polar part in micelle are arranged ? (d) solvent molecules and ions can diffuse
(a) Polar at outer surface and non-polar at inner surface 78. The electrolytic impurities of a sol can most easily be
(b) Polar at inner surface and non-polar at outer surface separated by
(c) Both polar and non-polar at inner surface (a) dialysis (b) electrosmosis
(d) Distributed all over the surface (c) electrophoresis (d) electrodialysis
SURFACE CHEMISTRY 321
79. The formation of colloid from suspension is (a) Tyndall effect (b) Brownian movement
(a) Peptisation (b) Condensation (c) Electrodialysis (d) Measuring particle size
(c) Sedimentation (d) Fragmentation 92. Which of the following is most effective in causing the
80. The separation of colloidal particles from particles of coagulation of ferric hydroxide sol?
molecular dimensions is known as (a) KCl (b) KNO3
(a) sedimentation (b) dispersion (c) K2SO4 (d) K3[Fe(CN)6]
(c) pyrolysis (d) dialysis 93. The ability of an ion to bring about coagulation of a given
81. Which one of the following impurities present in colloidal colloid depends upon
solution cannot be removed by electrodialysis? (a) its size
(a) Sodium chloride (b) Potassium sulphate (b) the magnitude of its charge
(c) Urea (d) Calcium chloride (c) the sign of its charge
82. The migration of dispersion medium under the influence of (d) both magnitude and sign of its charge
an electric potential is called : 94. Which of the following electrolytes is least effective in
(a) Cataphoresis (b) Electroosmosis coagulating ferric hydroxide solution?
(c) Electrophoresis (d) Sedimentation (a) KBr (b) K2SO4
83. The movement of colloidal particles towards their respective (c) K2CrO4 (d) K4 [Fe(CN)6]
electrodes in the presence of an electric field is known as : 95. Which of the following acts as protective colloid?
(a) electrolysis (b) Brownian movement (a) Silica gel (b) Gelatin
(c) electrodialysis (d) electrophoresis (c) Sodium acetate (d) None of these
84. Peptization denotes 96. Tyndall effect shown by colloids is due to :
(a) Digestion of food (a) scattering of light by the particles
(b) Hydrolysis of proteins (b) movement of particles
(c) Breaking and dispersion into the colloidal state (c) reflection of light by the particles
(d) Precipitation of solid from colloidal dispersion (d) coagulation of particles
85. Colloidal gold is prepared by 97. Which of the following is not a property of colloidal
(a) Mechanical dispersion (b) Peptisation solution?
(c) Bredig’s Arc method (d) Hydrolysis (a) Heterogenity (b) Particle size > 100 nm
86. Peptization involves (c) Tyndall effect (d) Brownian movement
(a) precipitation of colloidal particles 98. Which of the following is most powerful to coagulate the
(b) disintegration of colloidal aggregates negative colloid?
(c) evaporation of dispersion medium (a) ZnSO4 (b) Na3PO4
(d) impact of molecules of the dispersion medium on the (c) AlCl3 (d) K4[Fe(CN)6]
colloidal particles 99. The charge on colloidal particles is due to
87. Hardy-Schulze rule explains the effect of electrolytes on the (a) presence of electrolyte
coagulation of colloidal solution. According to this rule, (b) very small size of particles
coagulation power of cations follow the order (c) adsorption of ions from the solution
(a) Ba+2 > Na+ > Al+3 (b) Al+3 > Na+ > Ba+2 (d) None of these
+3
(c) Al > Ba > Na +2 + (d) Ba+2 > Al+3 > Na+ 100. The ion that is more effective for the coagulation of As2S3
88. Tyndall effect is shown by sol is
(a) sol (b) solution (a) Ba2+ (b) Na+
(c) plasma (d) precipitate (c) PO4 3– (d) AI3+
89. The cause of Brownian movement is 101. Which one of the following impurities present in colloidal
(a) heat changes in liquid state solution cannot be removed by electrodialysis?
(b) convectional currents (a) Sodium chloride (b) Potassium sulphate
(c) the impact of molecules of the dispersion medium on (c) Urea (d) Calcium chloride
the colloidal particles. 102. Brownian movement is found in
(d) attractive forces between the colloidal particles and (a) Colloidal solution (b) Suspension
molecules of dispersion medium. (c) Saturated solution (d) Unsaturated solution
90. When a strong beam of light is passed through a colloidal 103. Random motion of colloidal particles is known as
solution, the light will (a) Dialysis (b) Brownian movement
(a) be reflected (b) be scattered (c) Electroosmosis (d) Tyndall effect
(c) be refracted (d) give a rainbow 104. In which of the following Tyndall effect is not observed ?
91. The simplest way to check whether a system is colloidal or (a) Suspensions (b) Emulsions
not is by (c) Sugar solution (d) Gold sol
EBD_7207
322 SURFACE CHEMISTRY
105. Which of the following is not true about the emulsion? (a) Only (iv) is correct
(a) Emulsion can be broken into constituent liquids by (b) (i) and (ii) are correct
heat, freezing (c) (i), (ii) and (iv) are correct
(b) Emulsion of oil in water is generally unstable (d) (i), (ii) and (iii) are correct
(c) Emulsion do not show the Tyndall effect 114. Read the following statements related to physisorption.
(d) They show brownian motion (i) Adsorbent shows preference for gases with high
106. Which of the following process is responsible for the molecular weight.
formation of delta at a place where rivers meet the sea? (ii) Easily liquefiable gases gets readily adsorbed.
(a) Emulsification (b) Colloid formation (iii) Adsorption varies with change in temperature and
(c) Coagulation (d) Peptisation pressure.
(iv) Finely divided and solid metals adsorb gases equally.
107. Which of the following colloid does not contain liquid as a
(v) It is exothermic with low value of enthalpy of
dispersion medium?
adsorption.
(a) Sol (b) Gel
Which of the following is the correct code for the statements
(c) Emulsion (d) Foam above ?
108. Which of the following method is used for coagulation of (a) TFFTF (b) FTTFT
the sol? (c) TFFTT (d) FTTTF
(a) By mixing two oppositely charged sols. 115. Read the following statements related to chemisorption
(b) By electrophoresis. (i) It is highly specific.
(c) By addition of electrolytes. (ii) It increases with increase in temperature and pressure.
(d) All of the above. (iii) It is reversible.
109. Which of the following phenomenon occurs when the (iv) It increases with increase in surface area of adsorbent.
precipitate of Mg(OH)2 attains blue colour in presence of Which of the following is correct code for the statements
magneson reagent? above?
(i) Absorption of solvent (a) TTFT (b) TFFT
(ii) Adsorption of coloured substance (c) FTFT (d) FFTF
(iii) Absorption and adsorption both of solvent 116. Which of the following statement(s) is/are correct about
(iv) Adsorption of solvent solid catalyst?
(a) (i) and (ii) (b) (ii) only (i) Same reactants may give different product by using
(c) (ii) and (iv) (d) (iii) only different catalysts.
110. Which of the following is not the condition for Tyndall (ii) Catalyst is required in large quantities to catalyse
effect? reactions.
(a) The refractive indices of dispersed phase and (iii) Catalyst does not change H of reaction.
dispersion medium should differ greatly in magnitude. (iv) Catalytic activity of a solid catalyst does not depend
upon the strength of chemisorption.
(b) The diameter of the dispersed particles is not much
(a) (i) and (iii) (b) (i) only
smaller than the wavelength of light used.
(c) (ii), (iii) and (iv) (d) (iii) and (iv)
(c) Tyndall effect is observed only when viewed from the
117. Which of the following statement(s) is/are correct?
direction of the passage of light.
(i) Zeolites are good shape selective catalysts because
(d) All of these are required conditions for Tyndall effect. of their honeycomb-like structures.
111. Which of the following is not emulsifying agent for W/O (ii) All zeolites are naturally occurring substance.
emulsion? (iii) An important zeolite catalyst used in the petroleum
(a) Lampblack industry in ZSM-5.
(b) Long chain alcohol (a) (i) only (b) (ii) only
(c) Proteins (c) (i) and (iii) (d) (ii) and (iii)
(d) Heavy metal salts of fatty acids 118. Read the following statements regarding enzyme catalysis
112. Emulsions can be broken into constituent liquid by ______. (i) Enzyme catalysis is highly specific in nature.
(a) heating (b) freezing (ii) Enzyme catalysis to work effectively requires optimum
(c) centrifuging (d) All of these temperature (298-310 K) and optimum pH (3-5)
(iii) Metal ions like Na+, Mn2+, Co2+, Cu2+ etc. increases
STATEMENT TYPE QUESTIONS the activity of enzymes.
(iv) Catalyst used in Ostwald’s process is platinised
113. Read the following statements regarding adsorption and asbestos at 673 K.
choose the correct option. (v) Catalyst used in contact process is platinised asbestos
(i) It is a surface phenomenon. or V2O5 at 673-723 K.
(ii) The material which is adsorbed is termed as adsorbate. Which of the following is the correct coding for the above
(iii) The material on the surface of which the adsorption statements?
takes place is called adsorbent. (a) FTFTF (b) TFTFT
(iv) Adsorption is a bulk phenomenon. (c) TTFFF (d) FTTFT
SURFACE CHEMISTRY 323
119. Which of the following statements are correct? 122. Match the columns
(i) Gelatine sol if evaporated off it can be reobtained Column-I Column-II
simply by mixing gelatine obtained on evaporation (Biochemical reactions) (Enzymes)
with suitable dispersion medium. (A) C6H12O6(aq) (p) Zymase
(ii) Metal sulphide sols need stabilising agents for their 2C2H5OH(aq) + 2CO2(g)
preservation (B) NH2CONH2(aq) + H2O(l) (q) Pepsin
(iii) S8 being a macromolecule forms macromolecular 2NH3(g) + CO2(g)
colloid. (C) Proteins Peptides (r) Urease
(iv) Starch and proteins are natural whereas polythene (D) C12H22O11(aq) + H2O(l) (s) Invertase
and polystyrene are man-made macromolecules. C6H12O6(aq) + C6H12O6(aq)
(v) Micelles are formed above kraft temperature at any (a) A – (p), B – (r), C – (q), D – (s)
concentration (b) A – (p), B – (q), C – (r), D – (s)
(a) (i), (ii) and (iii) (b) (i), (ii) and (iv) (c) A – (r), B – (p), C – (q), D – (s)
(c) (iii), (iv) and (v) (d) (ii), (iv) and (v) (d) A – (p), B – (r), C – (s), D – (q)
120. Read the following statements 123. Match the columns
(i) Tyndall effect is used to distinguish between a colloidal Column-I Column-II
(Catalyst) (Industrial product)
and true solution.
(A) V2O5 (p) High density
(ii) Values of colligative properties are same for true and
poly-ethylene
colloidal solutions.
(B) Ziegler-Natta (q) Polyacrylonitrile
(iii) Random bombardment of the colloidal particles by
(C) Peroxide (r) NH3
the molecules of the dispersion medium does not allow (D) Finely divided Fe (s) H2SO4
colloids to settle thereby providing stability to them. (a) A – (s), B – (p), C – (q), D – (r)
(iv) Most acceptable phenomena to account for the charge (b) A – (s), B – (r), C – (q), D – (p)
of sol particles is electrodispersion. (c) A – (r), B – (p), C – (q), D – (s)
Which of the following is the correct code for statements (d) A – (s), B – (q), C – (p), D – (r)
above? 124. Match the columns
(a) TFTF (b) TTFF Column-I Column-II
(c) FTFT (d) TFFT (A) Oil in water emulsion (p) Clouds
(B) Aerosols containing (q) Vanishing cream
MATCHING TYPE QUESTIONS small droplets of water
121. Match the columns suspended in air
(C) When river water meets (r) Smoke
Column-I Column-II
the sea water
x
(A) kc1 n (p) Adsorption varies (D) Colloidal solution of (s) Formation of delta
m
directly with pressure carbon, arsenic
compounds, dust etc.
x 1 in air
(B) log log k log p (q) Adsorption from
m n (a) A – (q), B – (p), C – (s), D – (r)
1 solution phase (b) A – (p), B – (q), C – (s), D – (r)
0 (c) A – (q), B – (s), C – (p), D – (r)
n
(d) A – (q), B – (p), C – (r), D – (s)
x 1 125. Match the columns
(C) log log k log p (r) Freudlich isotherm
m n Column-I Column-II
1 cannot be explained (A) As2S3 sol (p) Bredig’s Arc method
1 (B) Fe(OH)3 sol (q) Double decomposition
n
x (C) Colloidal sols of metals (r) Peptization
(D) kp1 n (s) Adsorption is like Au, Ag, Pt, etc.
m
independent of pressure (D) Conversion of freshly (s) Hydrolysis
(high pressure) prepared precipitate
into a colloidal sol
(a) A – (q), B – (s), C – (p), D – (r) (a) A – (q), B – (s), C – (r), D – (p)
(b) A – (q), B – (p), C – (s), D – (r) (b) A – (q), B – (p), C – (s), D – (r)
(c) A – (r), B – (p), C – (s), D – (q) (c) A – (s), B – (q), C – (p), D – (r)
(d) A – (r), B – (s), C – (p), D – (q) (d) A – (q), B – (s), C – (p), D – (r)
EBD_7207
324 SURFACE CHEMISTRY
126. Match the columns (a) A – (s), B – (r), C – (p), D – (q)
Column-I Column-II (b) A – (q), B – (r), C – (p), D – (s)
(A) In this process molecules (p) Dialysis (c) A – (s), B – (p), C – (r), D – (q)
and ions diffuse through (d) A – (s), B – (r), C – (q), D – (p)
membrane outside and 130. Match the columns
pure colloidal solution Column-I Column-II
is left behind. (A) Butter (p) dispersion of liquid in liquid
(B) This process is used (q) Ultrafilteration (B) Pumice stone (q) dispersion of solid in liquid
if the dissolved substance (C) Milk (r) dispersion of gas in solid
in the impure colloidal (D) Paints (s) dispersion of liquid in solid
solution is only an (a) A – (r), B – (s), C – (p), D – (q)
electrolyte (b) A – (s), B – (r), C – (p), D – (q)
(C) In this process ordinary (r) Electro-dialysis (c) A – (q), B – (r), C – (p), D – (s)
filter paper is soaked into (d) A – (s), B – (r), C – (q), D – (p)
collodion (4% solution of 131. Match the columns
nitrocellulose in a mixture Column-I Column-II
of alcohol and ether)
(A) Argyrol (p) Kalazar
(a) A – (p), B – (r), C – (q)
(B) Antimony (q) Intramuscular injection
(b) A – (r), B – (p), C – (q)
(C) Colloidal gold (r) Stomach disorders
(c) A – (p), B – (q), C – (r)
(D) Milk of magnesia (s) Eye lotion
(d) A – (q), B – (r), C – (p)
(a) A – (r), B – (p), C – (s), D – (q)
127. Match the columns
(b) A – (r), B – (p), C – (q), D – (s)
Column-I Column-II
(c) A – (s), B – (q), C – (p), D – (s)
(A) Sulphur vapours passed (p) Normal electrolyte
through cold water solution (d) A – (s), B – (p), C – (q), D – (r)
(B) Soap mixed with water (q) Molecular colloids
ASSERTION-REASON TYPE QUESTIONS
above critical micelle
concentration Directions : Each of these questions contains two statements,
(C) White of egg whipped (r) Associated colloid Assertion and Reason. Each of these questions also has four
with water alternative choices, only one of which is the correct answer. You
(D) Soap mixed with water (s) Macro molecular have to select one of the codes (a), (b), (c) and (d) given below.
below critical micelle colloids (a) Assertion is correct, reason is correct; reason is a correct
concentration explanation for assertion.
(a) A – (q), B – (r), C – (s), D – (p) (b) Assertion is correct, reason is correct; reason is not a
(b) A – (r), B – (q), C – (s), D – (p) correct explanation for assertion
(c) A – (p), B – (r), C – (s), D – (q) (c) Assertion is correct, reason is incorrect
(d) A – (q), B – (s), C – (r), D – (p) (d) Assertion is incorrect, reason is correct.
128. Match the columns
x
Column-I Column-II 132. Assertion : The relation k . p1/ n is known as Freundlich
m
(A) Protective colloid (p) FeCl3 + NaOH
(B) Liquid - liquid colloid (q) Lyophilic colloids adsorption isotherm, where x is the mass of gas adsorbed
(C) Positively charged colloid (r) Emulsion by m grams of adsorbate, p is the equilibrium pressure, k
(D) Negatively charged colloid (s) FeCl3 + hot water and n are constants for given system and temperature.
(a) A – (q), B – (r), C – (p), D – (s) 1
Reason : When several substances have same value of ,
(b) A – (p), B – (r), C – (s), D – (q) n
(c) A – (q), B – (r), C – (s), D – (p) the lines by which their adsorption isotherms can be
(d) A – (r), B – (q), C – (s), D – (p) represented will meet at a point.
129. Match the columns 133. Assertion : The enthalpy of physisorption is greater than
Column-I Column-II chemisorption.
(A) Dialysis (p) Cleansing action of soap Reason : Molecules of adsorbate and adsorbent are held
(B) Peptisation (q) Coagulation by van der Waal’s forces in physisorption and by chemical
(C) Emulsification (r) Colloidal sol formatioin bonds in chemisorption.
(D) Electrophoresis (s) Purification
SURFACE CHEMISTRY 325
143. On the basis of data given below predict which of the
x 1/ n
134. Assertion : According to Freundlich: = K . p . following gases shows least adsorption on a definite amount
m
of charcoal?
Reason : The isotherm shows variation of the amount of Gas CO2 SO2 CH4 H2
gas adsorbed by the adsorbent with temperature.
Critical temp./K 304 630 190 33
135. Assertion: Detergents with low CMC are more economical
(a) CO2 (b) SO2
to use.
Reason: Cleansing action of detergents involves the (c) CH4 (d) H2
formation of micelles. These are formed when the 144. Which of the following statements regarding difference
concentration of detergents becomes equal to CMC. between adsorption and absorption is incorrect?
136. Assertion: An ordinary filter paper impregnated with (a) Adsorption is a surface whereas absorption is a bulk
collodion solution stops the flow of colloidal particles. phenomena.
Reason: Pore size of the filter paper becomes more than the (b) Water vapours are absorbed by anhydrous CaCl2 but
size of colloidal particle. adsorbed by silica gel.
137. Assertion : The value of colligative properties are of small (c) Adsorption and absorption take place individually.
order for colloids as compared to true solution. They can not occur simultaneously.
Reason : Number of particles in colloidal solution is (d) All of the above statements are correct.
comparatively smaller than true solutions. 145. Which of the following is not an application of adsorption?
(a) In metallurgy for concentration of sulphide ores.
CRITICAL THINKING TYPE QUESTIONS
(b) In heterogeneous catalysis involving solid catalyst.
138. Which of the following statements is incorrect regarding (c) In homogeneous catalysis.
physisorptions? (d) Separation of inert gas.
(a) More easily liquefiable gases are adsorbed readily. 146. Which of the following statements regarding catalyst is not
(b) Under high pressure it results into multimolecular layer true ?
on adsorbent surface. (a) A catalyst remains unchanged in composition and
(c) Enthalpy of adsorption ( H adsorption) is low and quantity at the end of the reaction
positive. (b) A catalyst can initiate a reaction
(d) It occurs because of van der Waal’s forces. (c) A catalyst does not alter the equilibrium in a reversible
139. Which is correct about physical adsorption? reaction
(a) High temperature and high pressure favour adsorption (d) Catalysts are sometimes very specific in respect of
(b) High temperature and low pressure favour adsorption reaction
(c) Low temperature and high pressure favour adsorption
147. Which of the following statements about a catalyst is true ?
(d) Low temperature and low pressure favour adsorption
(a) A catalyst accelerates the reaction by bringing down
140. Which one of the following is not applicable to the
the free energy of activation
phenomenon of adsorption ?
(b) A catalyst also takes part in the reaction mechanism
(a) H> 0 (b) G < 0
(c) A catalyst makes the reaction more feasible by making
(c) S< 0 (d) H < 0
the Gº more negative
141. Methylene blue, from its aqueous solution, is adsorbed on
activated charcoal at 25°C. For this process, which of the (d) A catalyst makes the equilibrium constant of the
following statement is correct ? reaction more favourable for the forward reaction
(a) The adsorption requires activation at 25°C 148. Which one of the following, statements is incorrect about
(b) The adsorption is accompanied by a decrease in enzyme catalysis?
enthalpy (a) Enzymes are mostly protenious in nature.
(c) The adsorption increases with increase of temperature (b) Enzyme action is specific.
(d) The adsorption is irreversible (c) Enzymes are denaturated by ultraviolet rays and at
142. In the adsorption of a gas on solid, Freundlich isotherm is high temperature.
obeyed. The slope of the plot is zero. Then the extent of (d) Enzymes are least reactive at optimum temperature.
adsorption is 149. Given below, catalyst and corresponding process/reaction
(a) directly proportional to the pressure of the gas are matched. The one with mismatch is
(b) inversely proportional to the pressure of the gas (a) [RhCl(PPh3)2] : Hydrogenation
(c) directly proportional to the square root of the pressure (b) TiCl4 + Al (C2H5)3 : Polymerization
of the gas (c) V2O5 : Haber-Bosch process
(d) independent of the pressure of the gas (d) Nickel : Hydrogenation
EBD_7207
326 SURFACE CHEMISTRY
150. Which one of the following statements is incorrect in the 161. Colloid of which one of the following can be prepared by
case of heterogeneous catalysis ? electrical dispersion method as well as reduction method ?
(a) The catalyst lowers the energy of activation (a) Sulphur (b) Ferric hydroxide
(b) The catalyst actually forms a compound with the (c) Arsenious sulphide (d) Gold
reactant 162. Which of the following ions can cause coagulation of
(c) The surface of the catalyst plays a very important role proteins ?
(d) There is no change in the energy of activation.
(a) Ag+ (b) Na+
151. In petrochemical industry alcohols are directly converted 2+
(c) Mg (d) Ca2+
to gasoline by passing over heated
(a) Platinum (b) ZSM-5 163. Which of the following will be most effective in the
(c) Iron (d) Nickel coagulation of Al(OH)3 sol ?
152. Which of the following feature of catalysts is described in (a) KCN (b) BaCl2
reactions given below? (c) NaCl (d) Mg3(PO4)2
Cu / ZnO Cr2O3 164. Point out the false statement :
(i) CO g 2H2 g CH3OH g
(a) The colloidal solution of a liquid in liquid is called gel
Cu
(ii) CO g H2 g HCHO g (b) Hardy Schulze rule is related with coagulation
(iii) CO g 3H2 g Ni (c) Brownian movement and Tyndall effect are shown by
CH4 g H2O g
colloidal system
(a) Activity (b) Selectivity
(d) Gold number is a measure of the protective power of
(c) Catalytic promoter (d) Catalytic poison
lyophilic colloid
153. The dispersed phase and dispersion medium in soap lather
are respectively 165. The disperse phase in colloidal iron (III) hydroxide and
(a) gas and liquid (b) liquid and gas colloidal gold is positively and negatively charged,
(c) solid and gas (d) solid and liquid respectively. Which of the following statements is NOT
correct?
154. Which of the following is not a colloid ?
(a) Chlorophyll (b) Smoke (a) Coagulation in both sols can be brought about by
(c) Ruby glass (d) Milk electrophoresis
155. Which of the following forms a colloidal solution in water ? (b) Mixing the sols has no effect
(a) NaCl (b) Glucose (c) Sodium sulphate solution causes coagulation in both
(c) Starch (d) Barium nitrate sols
156. Which of the following forms cationic micelles above certain (d) Magnesium chloride solution coagulates, the gold sol
concentration? more readily than the iron (III) hydroxide sol.
(a) Sodium dodecyl sulphate 166. A colloidal solution is subjected to an electric field. The
(b) Sodium acetate particles move towards anode. The coagulation of same sol
(c) Urea is studied using NaCl, BaCl2 and AlCl3 solutions. The order
(d) Cetyl trimethyl ammonium bromide of their coagulation power should be –
157. Which of the following does not contain a hydrophobic (a) NaCl > BaCl2 > AlCl3 (b) BaCl2 > AlCl3 > NaCl
structure ? (c) AlCl3 > BaCl2 > NaCl (d) BaCl2 > NaCl > AlCl3
(a) Linseed oil (b) Lanolin 167. Flocculation value of BaCl2 is much less than that of KCl
(c) Glycogen (d) Rubber for sol A and flocculation value of Na2SO4 is much less
158. Which one of the following is an example for multimolecular than that of NaBr for sol B. The correct statement among
colloid? the following is :
(a) Aqueous starch sol (a) Both the sols A and B are negatively charged.
(b) Aqueous enzyme sol (b) Sol A is positively charged arid Sol B is negatively
(c) Alcoholic polystyrene sol charged.
(d) Aqueous sol of sodium laurylsulphate
(c) Both the sols A and B are positively charged.
159. Bredig arc method cannot be used to prepare colloidal
(d) Sol A is negatively charged and sol B is positively
solution of which of the following
charged.
(a) Pt (b) Fe
(c) Ag (d) Au 168. In Brownian movement or motion, the paths of the particles
are
160. Colloidal solutions are not purified by
(a) Dialysis (b) Electrodialysis (a) Linear (b) Zig-zag
(c) Ultrafiltration (d) Electrophoresis (c) Uncertain (d) Curved
SURFACE CHEMISTRY 327
169. How does a delta form at the meeting place of sea and river (c) Dispersed phase – Gas, Dispersion medium – Gas.
water? (d) Dispersed phase – Liquid, Dispersion medium – Gas.
(a) The electrolyte present in sea water coagulate the clay 173. Which of the following statements is incorrect?
(b) the electrolyte present in sea water has no role (a) Colloidal gold is used for intramuscular injection.
(c) the electrolyte present in river water coagulate the clay (b) Colloidal solution of latex is used in preparation of
(d) Both (a) and (c) are correct rubber.
170. .......... is a silver sol used as an eye lotion. Fill in the blank (c) Photographic films are prepared by coating an
with an appropriate answer.
emulsion of AgBr in gelatin over glass plate.
(a) Amytol (b) Argyrol
(d) Tannin used in leather industry contains positively
(c) Ciprofloxacin (d) Both (a) and (b)
charged colloidal particles.
171. Which of the following will show Tyndall effect?
174. Which of the following is not correctly matched ?
(a) Aqueous solution of soap below critical micelle
(a) Sulphur sol – Oxidation
concentration.
(b) Gold sol – Double decomposition
(b) Aqueous solution of soap above critical micelle
concentration. (c) Fe(OH)3 sol – Hydrolysis
(c) Aqueous solution of sodium chloride. (d) Both (b) and (c)
(d) Aqueous solution of sugar. 175. How many of the following are negatively charged sols?
172. Which of the following combinations of dispersed phase Eosin dye, sol of charcoal, haemoglobin, Al2O3.xH2O,
and dispersion medium will not form a colloid ? As2S3, TiO2.sol, copper sol
(a) Dispersed phase – Solid, Dispersion medium – Solid. (a) 3 (b) 4
(b) Dispersed phase – Solid, Dispersion medium – Gas. (c) 6 (d) All of these
EBD_7207
328 SURFACE CHEMISTRY

20. (a) Adsorption of a gas on solid is represented by


FACT / DEFINITION TYPE QUESTIONS
following equilibria,
1. (d) The substance which is adsorbed is called adsorbate. Gas (Adsorbate) + Solid (Adsorbent)
2. (b) Adsorption is an exothermic process. Gas adsorbed on solid + Heat
3. (a) On increasing temperature physical adsorption of a Initially adsorption increases with increase in pressure
gas on surface of solid decreases. Solid adsorb greater at a particular temperature then got slow. After attaining
amount of gas at lower temperature. equilibrium adsorption become independent of
4. (a) Chemical adsorption involves formation of monolayer. pressure.
5. (a) Chemisorption involves strong chemical forces. 21. (d)
6. (d) In physisorption, gas molecules are held on the solid
22. (a) According to Freundlich adsorption isotherm
surface by weak van der Waal’s forces.
1
7. (b) x
8. (d) Adsorption is accompanied by evolution of heat as kP n
m
the residual forces acting along the surface of
adsorbent decrease i.e., adsorption is accompanied by 1
at low pressure =1
decrease in enthalpy. n
9. (b) The more readily soluble and easily liquefiable gases x
P1
such as NH3, HCl and SO2 are adsorbed more than the m
so called permanent gases like O2. This is due to the 1
van der Waals or intermolecular forces which are at high pressure =0
n
involved in adsorption.
x
10. (c) Adsorption is an exothermic process i.e. DH of P
adsorption is always negative. When a gas is adsorbed, m
the freedom of movement of its molecules becomes i.e., the value of n varies between 0 to 1.
restricted i.e. DS is negative. Hence adsorption 23. (b) The gases having higher values for critical temperature
accompanied by decrease in enthalpy as well as are easily liquified and are adsorbed to the greater
decrease in entropy of the system. extent. CO2 has highest critical temperature of 304K.
11. (d) The heat evolved in physisorption is quite low varying 24. (b) According to Freundlich equation.
generally between 20-40 kJ mol–1. x x
p1/ n or Kp1/ n
12. (c) 13. (b) 14. (a) 15. (d) 16. (b) m m
17. (a) 18. (b) x x 1
19. (d) According to Freundlich adsorption isotherm. or log log Kp1/ n or log log K log p
m m n
At intermediate pressure, extent of adsorption
25. (a) According to Freundlich adsorption isotherm
x x 1
kP1/ n or log log k log P; x
m m n kp1/n (wheren n > 1)
m
At very high pressure x/m = kp0
1 26. (a) On increasing temperature adsorption of a gas on
e = n surface of solid decreases. Solid adsorb greater amount
S lop of substances at lower temperature.
27. (c)
m
x
log

28. (a) For adsorption to occur G = –ve as in adsorption


H = –ve, S = –ve. –T S is positive for adsorption.
29. (d) For a reaction in equilibrium, the increase in rate of
reaction in forward direction by catalyst increases the
Intercept = log k concentration of product(s) and thus the rate of
log P backward reaction also increases with the same
magnitude and allow the equilibrium to be achieved
quickly.
plot of log x vs log P is linear with slope = 1
m n
SURFACE CHEMISTRY 329
30. (d) The catalytic poisons decrease the activity of the 52. (b) Colloid Dispersed phase Dispersion medium
catalyst because they are preferentially adsorbed on Sol Solid Liquid
the surface of catalyst. Gel Liquid Solid
31. (a) A catalyst increases the rate of reaction by decreasing Emulsion Liquid Liquid
the activation energy. Foam Gas Liquid
32. (c) Active charcoal has more adsorption power due to 53. (d) Emulsions are liquid-liquid colloidal systems, Generally
greater surface area. one of the two liquids is water.
33. (c) Adsorption theory is applied to heterogeneous 54. (b) Butter is an example of gel.
catalysis. 55. (c) Cheese is a liquid dispersed in solid phase.
34. (a) According to the adsorption theory of catalysis, the 56. (d) Air is a homogeneous mixture of gases, mainly nitrogen
activity of catalysis is due to the presence of free and oxygen.
valencies on its surface due to which surface of catalyst 57. (c) When the dispersed phase and dispersion medium both
has chemical force of altraction. When a gas comes in are liquid, the colloidal system is called as an emulsion
contact with this surface molecules get attached like milk, vasnishing cream etc.
attached through these valencies. Further the rate of 58. (b) Cloud consists of fine droplets of water suspended in
reaction is always increases by decreases in activation air.
energy. When any of the reactants is strongly 59. (d) Blood is a –vely charged colloidal system. Rest of the
adsorbed on the surface of catalyst, the rate becomes compounds, i.e., NaCl, urea & cane sugar form true
inversely proportional to the concentration of that solution in water.
reaction. The reaction is then said to be inhibited by 60. (a) Suspension particles are visible under a microscope
such reactant. The lowering of activation energy always and sometimes even to a naked eye.
leads to the increase in speed of reaction.
61. (a) Fog is a colloidal system consisting water droplets
35. (b) Catalyst decreases the activation energy of the dispersed in air.
reaction by forming an intermediate product. So no of
62. (a) A collidal solution is biphasic and heterogeneous. It is
molecules having activation energy increases hence
composed of two phases :
rate of reaction increases.
1. Dispersed phase; 2. Dispersion medium.
36. (c) A catalyst can affect reversible reaction by attaining
63. (c) Smoke is solid gas system, solid (D. P) and gas (D.M.)
equilibria in both directions.
Note : D.P. : Dispersed phase
37. (d) In acid hydrolysis of methyl acetate all are present in
D.M. : Dispersion medium
one phase (liquid).
64. (c) The critical micelle concentration is the lowest
38. (d) Enzymes are specific biological catalysts possessing
concentration at which micelle formation appears when
well - defined active sites.
surfactants are present above that CMC, they can act
39. (a) Enzymes are biological catalysts.
as emulsifiers that will solubilise a compound which is
40. (d) Enzymes are biological catalysts and enhance the rate normally insoluble in the solvent being used.
of biochemical reactions.
41. (c) Hydrolysis of urea can be represented as follows
Urease 65. (a) Polarhead
H 2 N C NH 2 H 2O 2NH3 CO 2 Non-polar tail
|| (enzyme)
O (micelle)
Since it involves biological catalyst (enzyme) so it is
an example of biochemical catalysis. 66. (a) Surface tension of lyophilic sols is lower than water
42. (d) Efficiency of catalysing property of a catalyst is (dispersion medium).
inversely proportional to activation energy. 67. (b) Gum is lyophilic colloid.
43. (b) Molybdenum acts as a promoter for iron which is used 68. (c) Lyophobic colloids are irreversible colloids. They are
as a catalyst in Haber’s process. protected by lyophilic colloids.
44. (c) Second step involves adsorption of reactant 69. (c) Example of multimolecular colloid system is a gold
molecules on the surface of the catalyst. dispersed in water.
45. (b) 70. (a) On shaking with the dispersion medium, colloids
46. (a) Milk is a emulsion in which liquid is dispersed in liquid. directly form the colloidal sol. Hence they are called
47. (c) In butter (liquid – solid) water is dispersed in fat. intrinsic colloids. i.e., glue.
48. (c) Size of colloidal particles is 10–5 - 10–7 cm. 71. (d) Sodium stearate is a soap. Soaps and detergents are
49. (a) surface agents which when dissolved in a medium,
50. (d) Mist is a colloid (aerosol) in which liquid is dispersed forms aggregated particles, called associated colloids.
in gas. 72. (d) The formation of micelles takes place only above a
51. (c) Alloy is an example of solid solution. particular temperature called kraft temperature (TK).
EBD_7207
330 SURFACE CHEMISTRY
73. (c) 94. (a) Smaller the charge on anion, lesser will be its
74. (b) Electrolytes are used for neutralising charge on coagulating power.
colloidal particles. KBr have Br– with least charge of – 1 on Br thus
75. (c) The osmotic pressure of sea water is 25 atm at 15°C. KBr is least effective in coagulating Fe(OH)3.
When pressure greater than 26 atm is applied on sea 95. (b) 96. (a)
water separated by a rigid emipermeable membrane, 97. (b) The size of colloidal particles is between 1 nm and
pure water is obtained. 1000 nm i.e., it is not always greater than 100 nm. So (b)
76. (a) Blood is purified by dialysis. is not a property of colloidal solution. All others are
77. (d) The use of membrane for separating colloidal particles the properties of colloidal solution.
is termed as dialysis. Hence it is clear that colloidal 98. (c) According to Hardy-Schulze rule "The amount of
particle cannot pass through animal membrane. Hence electrolyte required to coagulate a fixed amount of a
only solvent molecules and ions (in case of sol depends upon the sign of charge and valency of
electrodialysis) can diffuse. the flocculating ion."
78. (d) Electrolytic (Ionic) impurities can be most easily Thus, the coagulating power vary in the order.
removed on application of electric field. Al3+ > Zn++ > Na+
79. (a) Formation of colloid from suspension is known as 99. (c)
peptization. 100. (d) A negative ion causes the precipitation of positively
80. (d) The separation of colloidal particles of molecular charged sol and vice-versa. Since As2S3 is a negative
dimension is known as dialysis. It is a purification sol so more will be the positive charge on cation more
method of colloid. effective it will be in causing coagulation of As2S3 sol.
81. (c) Electrodialysis involves movement of ions towards Among the given ions, Al3+ has the greatest valency
oppositely charged electrodes. and thus is the most effective coagulating agent.
Urea being a covalent compound does not dissociate 101. (c) Electrodialysis involves movement of ions towards
to give ions and hence it cannot be removed by oppositely charged electrodes.
electrodialysis.However all the other given compounds Urea being a covalent compound does not dissociate
are ionic which can undergo dissociation to give to give ions and hence it cannot be removed by
oppositely charged ions and thus can be separated. electrodialysis.However all the other given compounds
82. (b) The motion of a liquid through a membrane under the are ionic which can undergo dissociation to give
influence of an applied electric field is known as electro- oppositely charged ions and thus can be separated.
osmosis. 102. (a) Brownian movement is exhibited by colloidal system.
83. (d) 103. (b) Brownian movement is random motion.
84. (c) Peptization comes under dispersion methods of 104. (c) Sugar forms homogeneous solution hence no Tyndall
preparation of colloids. effect is exhibited.
85. (c) Colloidal gold is prepared by Bredig's arc method. 105. (c) Emulsion show the tyndal! effect. Refers to ans 280.
86. (b) Peptisation is disintegration of colloidal aggregate. 106. (c)
87. (c) According to this law the coagulating effect of an ion 107. (b) For gel dispersed phase is liquid and dispersion
on dispersed phase of opposite charge increases with medium is solid.
the increase in valency of the ion. The precipitating 108. (d) 109. (c) 110. (c)
power of Al3+ , Ba++, Na+ ions is in order Al3+ > Ba2+ 111. (c) Protein is an emulsifying agent for O/W emulsion.
> Na+. 112. (d)
88. (a) Tyndall effect is shown by sols.
89. (c) It is due to impact of molecules of dispersion medium STATEMENT TYPE QUESTIONS
on the colloidal particles.
113. (d) Adsorption is a surface phenomenon.
90. (b) It is due to Tyndall effect.
114. (b) Statements (i) and (iv) are incorrect. A given surface
91. (a) Tyndall effect is the simplest way to check colloidal of an adsorbent does not show any preference for a
system since path of light beam becomes visible due particular gas as the van der Waal’s forces are
to scattering of light. universal.
92. (d) Fe(OH)3 is positive sol. K3[Fe(CN)6 ] will provide Finely divided metals are better adsorbent as compared
[Fe(CN)6]3– for coagulation having highest magnitude to solid metals because they have large surface area
of –ve charge among given options. and extent of adsorption increases with surface area.
93. (d) According to the Hardy schulze rule the coagulating 115. (a) As chemisorption involves compound formation, it is
effect of an ion on dispersed phase of opposite charge usually irreversible in nature.
increases with the valency of the ion. Therefore more
116. (a)
the charge on oppositely charged ion higher is the
117. (c) Zeolites are found in nature as well as synthesised
coagulation value.
for catalytic selectivity.
SURFACE CHEMISTRY 331
118. (b) Optimum pH for the enzyme catalysis is 5-7. Catalyst 139. (c) Physical adsorption involves weak forces, physical in
used in Ostwald’s process is platinised asbestos at nature with small heat of adsorption. Thus low
573 K. temperature and high pressure favours physical
119. (b) S8 forms a multimolecular colloid. Micelles are formed adsorption.
above kraft temperature and above a particular 140. (a)
concentration called Critical Micelle Concentration 141. (b) The adsorption of methylene blue on activated
(CMC). charcoal is an example of physiosorption which is
120. (a) Values of colligative properties for colloids are of small exothermic, multilayer and does not have energy barrier.
order as compared to values shown by true solution. 142. (d) Freundlich’s isothermal adsorption equation can be
Most acceptable phenomena to account for the charge given as
of sol particles in preferential adsorption.
x
kp1/n
MATCHING TYPE QUESTIONS m

121. (a) 122. (a) x 1 1


log log k log p; slope 0
123. (a) (A) V2O5 is used as a catalyst during the preparation m n n
of H2SO4
x
(B) Ziegler-Natta is used as a catalyst during the Thus, kp 0
m
preparation of HDPE.
(C) Peroxide is used as a catalyst durin g the 143. (d)
preparation of polyacrylonitrile. 144. (c) Both adsorption and absorption can take place
(D) Finely divided Fe is used as a catalyst during the simultaneously. The term sorption is used to describe
preparation of ammonia. both the processes.
124. (a) 125. (d) 126. (a) 127. (a) 128. (c) 145. (c) Homogenous catalysis does not involves adsorption.
129. (a) 130. (b) 146. (b) A catalyst can not initiate a reaction.
131. (d) Argyrol is used as an eye lotion. 147. (a)
Antimony is used in Kalazar. 148. (d) Enzymes are most reactive at optimum temperature.
Collidal gold is used in intramuscular injection. The optimum temperature for enzyme activity lies
Milk of magnesia is used in the stomach disorder. between 40°C to 60°C.
149. (c) V2 O 5 is used as catalyst in contact process of
ASSERTION-REASON TYPE QUESTIONS manufacturing H2SO4.
150. (c) The theory of heterogeneous catalysis is based upon
132. (c) Assertion is true, reason is false. When several lines the phenomenon of adsorption. The activity of catalyst
1 is due to the presence of free valencies on its sufrace
have the same value of , then the lines by which due to which surface of catalyst has force of attraction.
n
151. (b) ZSM-5 is a shape selective catalyst. Zeolites are good
their adsorption isotherms can be represented will be
shape selective catalysts because of the honey comb
parallel and will not meet at a point.
like structure.
133. (d) Assertion is false but Reason is true. The enthalpy
152. (b) Given reactions shows that the selectivity of different
of chemisorption is of the order of 40 - 400 kJmol –1
catalysts for same reactants is different.
while for physical adsorption it is of the order of
20 - 40 kJmol–1. 153. (a) Soap lather is a colloid containing gas as a dispersed
phase and liquid as a dispersion medium.
134. (c) Assertion is true but Reason is false.
154. (a) Chlorophyll. Smoke is an example of solid-gas colloid
Freundlich adsorption isotherm gives an empirical
system Ruby glass is an example of solid-solid colloid
relationship between the quantity of gas adsorbed by
system. Milk is an liquid -liquid colloid system.
unit mass of solid adsorbent and pressure at a particular
temperature. 155. (c) Starch molecules have colloidal dimensions whereas
NaCl, glucose and Ba(NO3)2 are crystalloids and
135. (a) 136. (c)
soluble in water.
137. (a) Colligative properties depend upon number of
156. (d) Cetyl trimethyl ammonium bromide,
particles.
[C16 H33 (CH3 )3 N Br ] is a cationic micelle.
CRITICAL THINKING TYPE QUESTIONS 157. (d) Linseed oil, lanolin and glycogen attract water hence
138. (c) Adsorption is an exothermic process, hence H will contain a hydrophilic structure but rubber does not
always be negative. attract water and thus does not contain a hydrophobic
structure.
EBD_7207
332 SURFACE CHEMISTRY
158. (a) Multimolecular colloids consist of aggregates of atoms 166. (c) As colloidal particles move towards anode so these
or small molecules. Sulphur sol is an example of particles are negatively charged and coagulated by
multimolecular colloids cations of electrolyte.
159. (b) Bredig’s arc method is suitable for the preparation of
According to Hardy Schulze rule,
colloidal solution of metals like gold, silver, platinum
etc. An arc is struck between the metal electrode under Coagulation power charge of ion
the surface of water containing some stabilzing agent Order of coagulation power is Al3+ > Ba2+ > Na+
such as a trace of KOH. However, Fe does not react 167. (b) In first case the given compounds have same anion
with alkalies that is why it is not obtained by Bredig’s- but different cations having different charge hence
arc method. they will precipitate negatively charged sol i.e. ‘A’.
160. (d) Colloidal solutions are not purified by electrophoresis.
In second case the given compounds have similar
Movement of colloidal particles under the influence of
cation but different anion with different charge. Hence
electric field is called electrophoresis. So, it can make
easier. Electrophoresis is the property of colloids not they will precipitate positively charged sol. i.e. ‘B’.
the purification method. 168. (b) Brownian movement is zig zag motion of sol particles.
161. (d) Gold by Bredig’s method (Dispersion method) and by 169. (a) 170. (a) 171. (b)
reduction method 172. (c) A gas mixed with another gas forms a homogeneous
AuCl3 + Tannic acid Gold sol mixture and hence is not a colloidal system.
162. (a) Proteins are coagulated by some heavy metal ions like
173. (d) Tannin used in leather industry contains negatively
Ag+, Hg2+ and Pb2+.
charged colloidal particles.
163. (d) Al(OH)3 is a positive sol so salt having anion with
maximum negative charged (i.e. phosphate ion) will be 174. (b) Gold sol is prepared by reduction as
most effective in coagulation. 2AuCl3 3HCHO 3H 2 O
Reduction
164. (a) Colloid of liquid in liquid is called emulsion. Colloid of
liquid in solid is gel. 2Au sol 3HCOOH 6HCl
165. (b) When oppositely charged sols are mixed their charges 175. (b) Eosin dye, sol of charcoal, As2S3 and copper sol are
are neutralised. Both sols may be partially or example of negatively charged sol.
completely precipitated.
20
GENERAL PRINCIPLES AND PROCESSES
OF ISOLATION OF ELEMENTS
12. Which one of the following is not a sulphide ore?
FACT / DEFINITION TYPE QUESTIONS
(a) Magnetite (b) Iron pyrites
1. Which one of the following is an ore of silver ? (c) Copper glance (d) Sphalerite
(a) Argentite (b) Stibnite 13. The impurities associated with mineral used in
(c) Haematite (d) Bauxite metallurgy are called collectively?
2. Cinnabar is an ore of (a) Slag (b) Flux
(a) Hg (b) Cu (c) Gangue (d) Ore
(c) Pb (d) Zn 14. The most abundant element in the earth’s crust (by weight)
3. An example of an oxide ore is is
(a) Bauxite (b) Malachite (a) Si (b) Al
(c) Zinc blende (d) Feldspar (c) O (d) Fe
15. Malachite is an ore of
4. The natural materials from which an element can be extracted
economically are called (a) iron (b) copper
(c) mercury (d) zinc
(a) ores (b) minerals
16. Cassiterite is an ore of
(c) gangue (d) None of these
(a) Mn (b) Ni
5. The most abundant metal on the surface of the earth is
(c) Sb (d) Sn
(a) Fe (b) Al
17. Galena is an ore of
(c) Ca (d) Na (a) Pb (b) Hg
6. Which of the following is an ore of tin ? (c) Zn (d) None of these
(a) Carborundum (b) Epsomite 18. The metal always found in the free states is
(c) Cassiterite (d) Spodumene (a) Au (b) Ag
7. Which of the following is chalcopyrite? (c) Cu (d) Na
(a) CuFeS2 (b) FeS2 19. Matrix is defined as –
(c) KMgCl3.6H2O (d) Al2O3.2H2O (a) the unwanted foreign material present in the ore
8. Haematite is the ore of (b) the flux added to remove the unwanted impurities from
(a) Pb (b) Cu ore
(c) Fe (d) Au (c) the slag formed as a result of the reaction of flux with
9. Composition of azurite mineral is gangue
(d) the material used in the reduction of metal oxide to
(a) CuCO3CuO (b) Cu(HCO3)2. Cu(OH)2
metal
(c) 2CuCO3.Cu(OH)2 (d) CuCO3. 2Cu(OH)2
20. Which of the following pair is incorrectly matched ?
10. Which one of the following is a mineral of iron ?
(a) Magnetite – Fe3O4 (b) Copper glance – Cu2S
(a) Malachite (b) Cassiterite (c) Calamine – ZnCO3 (d) Zincite – ZnS
(c) Pyrolusite (d) Magnetite 21. Which one of the following ores is best concentrated by
11. All ores are minerals, while all minerals are not ores because froth-flotation method ?
(a) the metal can’t be extracted economically from all the (a) Galena (b) Cassiterite
minerals (c) Magnetite (d) Malachite
(b) minerals are complex compounds 22. Froth floatation process is used for the metallurgy of
(c) the minerals are obtained from mines (a) chloride ores (b) amalgams
(d) all of these are correct (c) oxide ores (d) sulphide ores
EBD_7207
334 GENERAL PRINCIPLES AND PROCESSES OF ISOLATION OF ELEMENTS
23. Cassiterite is concentrated by 35. Main function of roasting is
(a) levigation (a) to remove volatile substances
(b) electromagnetic separation (b) oxidation
(c) floatation (c) reduction
(d) liquefaction (d) slag formation
24. While extracting an element from its ore, the ore is grounded
36. Roasting is generally done in case of the
and leached with dil. potassium cyanide solution to form
(a) oxide ores (b) silicate ores
the soluble product potassium argento cyanide. The element
is (c) sulphide ores (d) carbonate ores
(a) Lead (b) Chromium 37. Heating of pyrites in air for oxidation of sulphur is called
(c) Manganese (d) Silver (a) roasting (b) calcination
25. The method of concentrating the ore which makes use of (c) smelting (d) slagging
the difference in density between ore and impurities is called 38. The role of calcination in metallurgical operations is
(a) levigation (b) leaching (a) to remove moisture
(c) magnetic separation (d) liquifaction (b) to decompose carbonates
26. Leaching is a process of (c) to drive off organic matter
(a) reduction (b) concentration (d) to decompose carbonates and drive off moisture and
(c) refining (d) oxidation organic matter
27. Which one of the following ores is concentrated by chemical
39. General method for the extraction of metal from oxide ore is
leaching method?
(a) carbon reduction
(a) Galena (b) Copper pyrite
(c) Cinnabar (d) Argentite (b) reduction by aluminium
28. Electromagnetic separation is used in the concentration of (c) reduction by hydrogen
(a) copper pyrites (b) bauxite (d) electrolytic reduction
(c) cassiterite (d) cinnabar 40. Function of the flux added during smelting is
29. For which ore of the metal, froth floatation method is used (a) to make ore porous
for concentration? (b) to remove gangue
(a) Horn silver (b) Bauxite (c) to make reduction easier
(c) Cinnabar (d) Heamatite (d) to precipitate slag
30. Which of the following metal is leached by cyanide process ? 41. Process followed before reduction of carbonate ore is
(a) Ag (b) Na (a) calcination (b) roasting
(c) Al (d) Cu
(c) liquation (d) polling
31. Which one of the following ores is not concentrated by
42. Calcination is the process in which :
froth floatation process?
(a) Copper pyrites (b) Pyrargyrite (a) ore is heated above its melting point to expel H2O or
(c) Pyrolusite (d) Zinc blende CO2 or SO2
32. In froth flotation process many chemicals (frother , collector, (b) ore is heated below its melting point to expel volatile
activator, and depressant) are used. Which of the following impurities
is a frother ? (c) ore is heated above its melting point to remove S, As
(a) CuSO 4 (b) NaCN+ alkali and Sb as SO2 ,As2O3 and Sb2O3 respectively
(d) ore is heated below its melting point to expel H2O or
(c) Pine oil (d) Potassium xanthate
CO2
33. Froth flotation process is based on
43. When a metal is to be extracted from its ore and the gangue
(a) wetting properties of ore particle
(b) specific gravity of ore particles associated with the ore is silica, then
(c) magnetic properties of ore particles (a) an acidic flux is needed
(d) electrical properties of ore particles (b) a basic flux is needed
34. In the froth flotation process of concentration of ores, the (c) both acidic and basic fluxes are needed
ore particles float because they: (d) Neither of them is needed
(a) are light 44. Which of the following fluxes is used to remove acidic
(b) are insoluble impurities in metallurgical process?
(c) have the surface which is not wetted easily (a) Silica (b) Lime stone
(d) have a constant electrical charge (c) Sodium chloride (d) Sodium carbonate
GENERAL PRINCIPLES AND PROCESSES OF ISOLATION OF ELEMENTS 335

45. Which of the following reactions is an example for calcination 56. Furnaces are lined with calcium oxide because
process ? (a) it gives off oxygen on heating
(b) it gives strong light on heating
(a) 2Ag 2HCl O 2AgCl H 2 O
(c) it is refractory and basic
(b) 2Zn O 2 2ZnO (d) it is not affected by acids
57. The following reactions take place in the blast furnace in
(c) 2ZnS 3O 2 2ZnO 2SO 2 the preparation of impure iron. Identify the reaction
(d) MgCO3 MgO CO 2 pertaining to the formation of the slag.
(a) Fe2O3(s) + 3 CO(g) 2 Fe (l) + 3 CO2 (g)
46. After partial roasting the sulphide of copper is reduced by
(b) CaCO3 (s) CaO (s) + CO2 (g)
(a) cyanide process (b) electrolysis
(c) CaO (s) + SiO2(s) CaSiO3(s)
(c) reduction with carbon (d) self reduction
(d) 2C(s) + O2 (g) 2 CO(g)
47. Hydro-metallurgical process of extraction of metals is based
on 58. Refractory materials are generally used in furnaces because
(a) complex formation (b) hydrolysis (a) they possess great structural strength
(c) dehydration (d) dehydrogenation (b) they can withstand high temperature
48. (c) they are chemically inert
2CuFeS2 O 2 Cu 2 S 2FeS SO 2
(d) they do not require replacement
Which process of metallurgy of copper is represented by
above equation? 59. Which of the following reactions taking place in the blast
furnace during extraction of iron is endothermic?
(a) Concentration (b) Roasting
(c) Reduction (d) Purification (a) CaCO3 CaO CO2
49. Which of the following is not used as a collector ? (b) 2C O 2 2CO
(a) Pine oil (b) Xanthates (c) C O2 CO 2
(c) Cresols (d) Fatty acids
50. Which of the following reaction represents calcination (d) Fe 2 O 3 3CO 2Fe 3CO 2
process ? 60. Cast iron is
(a) 2PbS 3O 2 2PbO 2SO 2 (a) made by melting pig iron with scrap iron and coke
(b) CaCO3.MgCO3 (s) CaO(s) MgO(s) 2CO2(g) using hot air blast
coke, 1673K
(b) having slightly lower carbon content (about 3%) as
(c) ZnO C Zn CO compared to pig iron
(d) Fe 2 O3 CO 2FeO CO 2 (c) extremely hard and brittle
51. According to Ellingham diagram, the oxidation reaction of (d) All of the above statements are true
carbon to carbon monoxide may be used to reduce which 61. In the extraction of copper from its sulphide ore, the metal is
one of the following oxides at the lowest temperature ? finally obtained by the reduction of cuprous oxide with :
(a) Al2O3 (b) Cu2O (a) Copper (I) sulphide (Cu2S)
(c) MgO (d) ZnO (b) Sulphur dioxide (SO2)
52. Which of the following condition favours the reduction (c) Iron sulphide (FeS)
of a metal oxide to metal? (d) Carbon monoxide (CO)
(a) H = +ve, T S = + ve at low temperature 62. Extraction of zinc from zinc blende is achieved by
(b) H = +ve, T S = – ve at any temperature (a) electrolytic reduction
(c) H = –ve, T S = – ve at high temperature (b) roasting followed by reduction with carbon
(d) H = –ve, T S = + ve at any temperature (c) roasting followed by reduction with another metal
53. Ellingham diagram normally consists of plots of
(d) roasting followed by self-reduction
(a) Sº vs T (b) fGº vs Sº 63. In the extraction of Cu, the metal is formed in the bessemer
(c) Gº vs T (d) Hº vs T
converter due to the reaction :
54. G° vs T plot in the Ellingham’s diagram slopes downward (a) Cu2S + 2Cu2O 6Cu + SO2
for the reaction (b) Cu2S 2Cu + S
1 1 (c) Fe + Cu2O 2Cu + FeO
(a) Mg O2 MgO (b) 2Ag O2 Ag 2O
(d) 2Cu2O 4Cu + O2
2 2
1 1 64. Aluminothermic process is used for the extraction of metals,
(c) C O2 CO (d) CO O2 CO 2 whose oxides are
2 2 (a) fusible
55. In the blast furnace iron oxide is reduced by
(b) not easily reduced by carbon
(a) silica (b) CO
(c) not easily reduced by hydrogen
(c) carbon (d) limestone
(d) strongly basic
EBD_7207
336 GENERAL PRINCIPLES AND PROCESSES OF ISOLATION OF ELEMENTS
65. Electrometallurgical process is used to extract 75. The main reactions occurring in blast furnace during
(a) Fe (b) Pb extraction of iron from haematite are________.
(c) Na (d) Ag (i) Fe2O3 + 3CO — 2Fe + 3CO2
66. The electrolytic method of reduction is employed for the (ii) FeO + SiO2 — FeSiO3
preparation of metals that
(iii) Fe2O3 + 3C — 2Fe + 3CO
(a) are weakly electropositive
(iv) CaO + SiO2 — CaSiO3
(b) are moderately electropositive
(c) are strongly electropositive (a) (i) and (iii) (b) (ii) and (iv)
(d) form oxides (c) (i) and (iv) (d) (i), (ii) and (iii)
67 Aluminium is extracted from alumina (Al2O3 ) by electrolysis 76. The process of zone refining is used in the purification of
of a molten mixture of (a) Si (b) Al
(a) Al2O3 + HF + NaAlF4 (c) Ag (d) Cu
(b) Al2O3 + CaF2 + NaAlF4
77. Van Arkel method of purification of metals involves
(c) Al2O3 + Na3AlF6 + CaF2
converting the metal to a
(d) Al2O3 + KF + Na3AlF6
68. In the extraction of aluminium by Hall-Heroult process, (a) volatile stable compound
purified Al2O3 is mixed with CaF2 to (b) volatile unstable compound
(i) lower the melting point of Al2O3. (c) non volatile stable compound
(ii) increase the conductivity of molten mixture. (d) None of the above
(iii) reduce Al3+ into Al(s). 78. The method not used in metallurgy to refine the impure
(iv) acts as catalyst. metal is
(a) (i) and (ii) (b) (i), (ii) and (iii) (a) Mond’s process
(c) (iii) and (iv) (d) (ii), (iii) and (iv) (b) Van–Arkel process
69. In the extraction of chlorine by electrolysis of brine______. (c) Amalgamation process
(a) oxidation of Cl– ion to chlorine gas occurs.
(d) Liquation
(b) reduction of Cl– ion to chlorine gas occurs.
79. Which of the following pairs of metals is purified by van
(c) For overall reaction G has negative value.
Arkel method ?
(d) a displacement reaction takes place.
70. Brine is electrolysed by using inert electrodes. The reaction (a) Ga and In (b) Zr and Ti
at anode is ________. (c) Ag and Au (d) Ni and Fe
80. The method of zone refining of metals is based on the principle
1
(a) Cl – (aq.) Cl 2 (g) e – ; ECell 1.36V of
2
(a) greater solubility of the impurities in the molten state
(b) 2H 2 O(1) O2 (g) 4 H 4e – ; ECell 1.23V than in the solid
(b) greater mobility of the pure metal than that of the
(c) Na (aq.) e – Na(s); ECell 2.71V impurite
1 (c) higher melting point of the impurities than that of the
(d) H (aq.) e – H 2 (g); ECell 0.00V pure metal
2
71. Blister copper is (d) greater noble character of the solid metal than that of
(a) Impure Cu (b) Cu alloy the impurities
(c) Pure Cu (d) Cu having 1% impurity 81. Method used for obtaining highly pure silicon which is
72. The furnace used to prepare commercial iron is lined with used as a semiconductor material, is
which of the following ? (a) oxidation (b) electrochemical
(a) Haematite (b) Magnetite (c) crystallization (d) zone refining
(c) Ironpyrites (d) Both (a) and (b) 82. What is anode mud?
73. Which form of the iron contains 4% carbon ? (a) Fan of anode
(a) Cast iron (b) Pig iron
(b) Metal of anode
(c) Wrought iron (d) Both (a) and (b)
(c) Impurities collected at anode in electrolysis during
74. Which of the following reaction takes place in blast furnace
during extraction of copper ? purification of metals
(a) 2Cu 2S 3O 2 2Cu 2O 2SO 2 (d) All of these
83. The process of zone refining is used in the purification of
(b) 2FeS 3O 2 2FeO 2SO 2
(a) Si (b) Al
(c) 2Cu 2O Cu 2S 6Cu SO 2
(d) All of these (c) Ag (d) Cu
GENERAL PRINCIPLES AND PROCESSES OF ISOLATION OF ELEMENTS 337
84. Which of the following statements regarding 89. Read the following statements
chromatography is incorrect ? (i) The principle that the impurities are more soluble in
(a) It is based on the principle that different components the melt than in the solid state is used in the
of mixture gets adsorbed differently on an adsorbent manufacture of high purity semiconductors.
(b) Column chromatography involves column of Al2O3 in (ii) Van Arkel method of refining Zr involves heating of
a glass tube as a stationary phase. crude metal with Cl2 to form corresponding halide.
(c) The mobile phase may be a gas, a liquid or a solid. (iii) Mond process for refining of nickel involves formation
(d) Component which is more soluble is stationary phase of metal carbonyls as an intermediate.
takes longer time to travel. Which of the following is the correct code for the statements
85. Which of the following metal is used in the manufacture of above ?
dye-stuffs and paints ? (a) TTT (b) FFT
(a) Copper (b) Zinc (c) TFT (d) FTF
(c) Aluminium (d) Magnesium
MATCHING TYPE QUESTIONS
STATEMENT TYPE QUESTIONS
90. Match the columns
86. Read the following statements Column - I Column - II
(i) Magnetic separation method is employed when one (A) Fe 2 O3 .xH 2O(s) (p) Slag formation
component either ore or gangue is magnetic in nature.
Fe2O3 (s) xH 2O(g)
(ii) Depressant NaCN used in case of ore containing
mixture of ZnS and PbS allows ZnS to come with froth (B) FeO SiO2 FeSiO3 (q) Reduction of
and prevents PbS from coming to the froth. iron oxide
(C) Discharge gas produced (r) Calcination
(iii) For concentration powdered bauxite ore is digested
during this process is utilised
with conc.NaOH at 473–523K and 35–36 bar pressure.
in manufacture of H2SO4.
Which of the following is the correct code for the statements
(D) Fe2 O3 3C 2Fe 3CO (s) Roasting
above ?
(a) A– (r), B – (p), C – (s), D – (q)
(a) TFT (b) TTF
(b) A– (p), B – (r), C – (s), D – (q)
(c) FTF (d) FFT (c) A– (r), B – (s), C – (p), D – (q)
87. Which of the following statements related to Ellingham (d) A– (r), B – (p), C – (q), D – (s)
diagrams are correct ? 91. Match the columns
(i) It provides a sound basis for the choice of reducing Column - I Column - II
agent in the reduction of oxides. (A) According to r G vsT (p) Sulphur oxide
(ii) Each Ellingham plot is represented by a straight line graph, oxide of this metal
untill unless there is some change in phase i.e. can be easily reduced to
solid liquid, liquid gas and gas liquid occurs. corresponding metal by
(iii) Diagrams similar to Ellingham can be constructed for heating with coke
sulphides and halides which clearly indicates why (B) Substance responsible for (q) Copper
reduction of MxS is difficult in comparison to MxO. the blistered appearence
(iv) Ellingham diagrams predicts the tendency of reduction of the copper obtained as
with a reducing agent and kinetics of the reduction result of extraction of
process. copper from cuprous oxide
(C) Metal which during (r) Na3AlF6 or CaF2
(a) (i), (ii) and (iii) (b) (i) and (iii)
purification is distilled
(c) (i), (ii) and (iv) (d) (ii) and (iv) off and collected by
88. Which of the following statement(s) is/are correct ? rapid chilling
(i) Cast iron is used in the manufacture of railway sleepers (D) On addition to Al2O3 its (s) Zinc
(ii) Wrought iron is used in the manufacture of anchors, melting point gets reduced
bolts, chains etc. and conductivity gets
enhanced
(iii) Nickel steel is used in making pendulums.
(a) A – (p), B – (q), C – (s), D – (r)
(a) Only (i) (b) (i) and (ii) (b) A – (q), B – (s), C – (p), D – (r)
(c) (i), (ii) and (iii) (d) Only (iii) (c) A – (q), B – (p), C – (s), D – (r)
(d) A – (q), B – (p), C – (r), D – (s)
EBD_7207
338 GENERAL PRINCIPLES AND PROCESSES OF ISOLATION OF ELEMENTS
92. Match the columns. (a) A – (r), B – (s), C – (t), D – (q), E – (p)
Column-I Column-II (b) A – (s), B – (r), C – (t), D – (q), E – (p)
(A) Blisterred Cu (p) Aluminium (c) A – (s), B – (t), C – (r), D – (q), E – (p)
(B) Blast furnace (q) 2Cu2O + Cu2S 6Cu + SO2 (d) A – (s), B – (r), C – (p), D – (q), E – (t)
(C) Reverberatory (r) Iron 97. Match the columns
furnace Column-I Column-II
(D) Hall-Heroult (s) FeO + SiO2 FeSiO3 (A) This metal is used in (p) Zinc
process extraction of chromium
(t) 2Cu2S + 3O2 2Cu2O + 2SO2 and manganese.
(a) A – (q), B – (r), C – (s), D – (p) (B) Common metal in brass (q) Aluminium
(b) A – (p), B – (q), C – (r), D – (t) and bronze.
(C) Common metal in brass (r) Copper
(c) A – (t), B – (s), C – (r), D – (q)
and german silver.
(d) A – (s), B – (t), C – (r), D – (q)
(D) Substance used in making (s) Stainless steel
93. Match the columns.
cycles, automobiles,
Column-I Column-II
utensils, etc.
(A) Coloured bands (p) Zone refining
(a) A – (q), B – (r), C – (p), D – (s)
(B) Impure metal to volatile (q) Fractional
(b) A – (r), B – (q), C – (p), D – (s)
complex distillation
(c) A – (q), B – (p), C – (r), D – (s)
(C) Purification of Ge and Si (r) Mond Process (d) A – (q), B – (r), C – (s), D – (p)
(D) Purification of mercury (s) Chromatography
(t) Liquation ASSERTION-REASON TYPE QUESTIONS
(a) A – (p), B – (q), C – (s), D – (t)
(b) A – (s), B – (r), C – (p), D – (q) Directions : Each of these questions contain two statements,
(c) A – (r), B – (s), C – (p), D – (q) Assertion and Reason. Each of these questions also has four
(d) A – (t), B – (s), C – (r), D – (q) alternative choices, only one of which is the correct answer. You
have to select one of the codes (a), (b), (c) and (d) given below.
94. Match the columns.
(a) Assertion is correct, reason is correct; reason is a correct
Column-I Column-II
explanation for assertion.
(A) Cyanide process (p) Ultrapure Ge
(b) Assertion is correct, reason is correct; reason is not a
(B) Froth Floatation Process (q) Dressing of ZnS
correct explanation for assertion
(C) Electrolytic reduction (r) Extraction of Al
(c) Assertion is correct, reason is incorrect
(D) Zone refining (s) Extraction of Au
(d) Assertion is incorrect, reason is correct.
(t) Purification of Ni 98. Assertion : Levigation is used for the separation of oxide
(a) A – (s), B – (q), C – (r), D – (p) ores from impurities.
(b) A – (q), B – (r), C – (p), D – (t) Reason : Ore particles are removed by washing in a current
(c) A – (p), B – (q), C – (r), D – (s) of water.
(d) A – (r), B – (s), C – (t), D – (p) 99. Assertion : Zinc can be used while copper cannot be used
95. Match the columns in the recovery of Ag from the complex [Ag(CN)2]–.
Column-I Column-II Reason : Zinc is a powerful reducing agent than copper.
(A) Cyanide process (p) Ultrapure Ge 100. Assertion : Leaching is a process of reduction.
(B) Floatation process (q) Pine oil Reason : Leaching involves treatment of the ore with a
(C) Electrolytic reduction (r) Extraction of Al suitable reagent so as to make it soluble while impurities
(D) Zone refining (s) Extraction of Au remains insoluble.
(a) A – (r), B – (p), C – (s), D – (q) 101. Assertion : Coke and flux are used in smelting.
(b) A – (s), B – (q), C – (r), D – (p) Reason : The phenomenon in which ore is mixed with
(c) A – (r), B – (q), C – (s), D – (p) suitable flux and coke is heated to fusion is known as
(d) A – (s), B – (p), C – (r), D – (q) smelting.
96. Match the columns 102. Assertion : Copper obtained after bessemerization is known
Column-I Column-II as blister copper.
(A) Distillation (p) Zr Reason : Blisters are produced on the surface of the metal
(B) Electrolytic refining (q) Ga due to escaping of dissolved SO2.
(C) Liquation (r) Cu 103. Assertion : Lead, tin and bismuth are purified by liquation
(D) Zone refining (s) Hg method.
(E) Vapour phase refining (t) Sn Reason : Lead, tin and bismuth have low m.p. as compared
to impurities.
GENERAL PRINCIPLES AND PROCESSES OF ISOLATION OF ELEMENTS 339

CRITICAL THINKING TYPE QUESTIONS 114. Consider the following reactions at 1000°C
1
104. Copper can be extracted from
A. Zn(s) O 2 (g) ZnO(s); G 360kJ mol 1
2
(a) Kupfernical (b) Dolomite
1
(c) Malachite (d) Galena B. C(gr) O 2 (g) CO(g); G 460kJ mol –1
105. Which of the following metal is correctly matched with its 2
ore? Choose the correct statement at 1000°C
(a) zinc can be oxidised by carbon monoxide.
Metal Ore
(b) zinc oxide can be reduced by graphite
(a) Zinc Calamine
(c) carbon monoxide can be reduced by zinc.
(b) Silver Ilmenite (d) both statements (a) and (b) are true
(c) Magnesium Cassiterite 115. Which of the following statements, about the advantage of
(d) Tin Azurite roasting of sulphide ore before reduction is not true?
106. Which ore contains both iron and copper?
(a) The G of of the sulphide is greater than those for CS2
(a) Cuprite (b) Chalcocite
and H2S.
(c) Chalcopyrite (d) Malachite
(b) The Gof is negative for roasting of sulphide ore to
107. Sulfide ores are common for the metals
oxide.
(a) Ag, Cu and Pb (b) Ag, Mg and Pb
(c) Roasting of the sulphide to the oxide is thermo-
(c) Ag, Cu and Sn (d) Al, Cu and Pb
dynamically feasible.
108. Which one of the following does not occur as sulphide
(d) Carbon and hydrogen are suitable reducing agents for
ore?
reduction of metal sulphides.
(a) Zn (b) Cr
116. Which of the following statement is not correct about
(c) Ag (d) Fe
Ellingham diagram?
109. Pyrolusite is a/an
(a) G increases with an increase in temperature
(a) oxide ore (b) sulphide ore
(b) It consists of plots of fGº vs T for formation of oxides
(c) carbide ore (d) Not an ore
110. Sulphide ores of metals are usually concentrated by froth (c) a coupling reaction can be well expressed by this
diagram
flotation process. Which one of the following sulphide ores
offer an exception and is concentrated by chemical leaching? (d) It express the kinetics of the reduction process
(a) Galena (b) Copper pyrite 117. A coupled reaction takes place as follow–
(c) Sphalerite (d) Argentite A + B –––– C + D, Gº = + x kj
111. Which of the following statements is correct ? D + E –––– F Gº = – y kj
(a) Gangues are carefully chosen to combine with the slag for the spontaneity of reaction A + B + E ––– C+F,
present in the ore to produce easily fusible flux to carry which of the following is correct?
away the impurities (a) 2x = y (b) x < y
(b) Slags are carefully chosen to combine with the flux (c) x > y (d) x = (y)× T S
present in the ore to produce easily fusible gangue to 118. The value of f Gº for formation of Cr2O3 is – 540 kJmol–1
carry away the impurities and that of Al2O3 is – 827 kJ mol–1 What is the value of
°
(c) Gangues are carefully chosen to combine with the flux rG for the reaction?
present in the ore to produce easily fusible slag to 4 2 2 4
carry away the impurities Al(s) + Cr2 O 3 (s) Al 2 O 3 (s) Cr(s).
3 3 3 3
(d) Fluxes are carefully chosen to combine with the gangue
present in the ore to produce easily fusible slag to (a) – 574 kJ mol–1 (b) –287 kJ mol–1
(c) + 574 kJ mol –1 (d) +287 kJ mol–1
carry away the impurities
112. Carbon and CO gas are used to reduce which of the following 119. Before introducing FeO in blast furnace , it is converted to
pairs of metal oxides for extraction of metals ? Fe2O3 by roasting so that
(a) FeO, SnO (b) SnO, ZnO (a) it may not be removed as slag with silica
(c) BaO, Na2O2 (d) FeO, ZnO (b) it may not evaporate in the furnace
113. In the cyanide extraction process of silver from argentite (c) presence of it may increase the m.pt. of charge
ore, the oxidising and reducing agents used are (d) None of these.
(a) O2 and CO respectively 120. The temperature in °C at which Fe2O3 is finally reduced to
(b) O2 and Zn dust respectively Fe in the blast furnace is
(c) HNO3 and Zn dust respectively (a) 993 (b) 797
(d) HNO3 and CO respectively (c) 897 (d) 1597
EBD_7207
340 GENERAL PRINCIPLES AND PROCESSES OF ISOLATION OF ELEMENTS
121. When copper ore is mixed with silica, in a reverberatory 127. Which of the following reaction(s) occur in temperature
furnace copper matte is produced. The copper matte range 500 – 800 K in blast furnace.
contains________. (i) Fe2O3 CO 2FeO CO2
(a) sulphides of copper (II) and iron (II) (ii) Fe3O4 4CO 3Fe 4CO2
(b) sulphides of copper (II) and iron (III) (iii) FeO CO Fe CO 2
(c) sulphides of copper (I) and iron (II)
(iv) C CO 2 2CO
(d) sulphides of copper (I) and iron (III)
(a) (i) and (ii) (b) (i), (ii) and (iii)
122. In the metallurgy of aluminium _________. (c) (iii) and (iv) (d) (iv) only
(a) Al3+ is oxidised to Al (s). 128. In Hall-Heroult process how much carbon anode is burnt
(b) graphide anode is oxidised to carbon monoxide and away to produce each 1kg of aluminium ?
carbon dioxide. (a) 0.3 kg (b) 0.5 kg
(c) oxidation state of oxygen changes in the reaction at (c) 1 kg (d) 0.1 kg
anode. 129. In electro-refining of metal the impure metal is used to make
(d) oxidation state of oxygen changes in the overall the anode and a strip of pure metal as the cathode, during
reaction involved in the process. the electrolysis of an aqueous solution of a complex metal
123. In the extraction of chlorine from brine_________. salt. This method cannot be used for refining of
(i) G for the overall reaction is negative. (a) Silver (b) Copper
(ii) G for the overall reaction is positive. (c) Aluminium (d) Sodium
(iii) E for overall reaction has negative value. 130. During the process of electrolytic refining of copper, some
(iv) E for overall reaction has positive value. metals present as impurity settle as ‘anode mud’. These are
(a) (i) and (ii) (b) (ii) and (iii) (a) Fe and Ni (b) Ag and Au
(c) (i) and (iv) (d) (iii) and (iv) (c) Pb and Zn (d) Sn and Ag
124. Cu 2S 2Cu 2O 6Cu SO2 131. If the impurities in a metal has a greater affinity for oxygen
In which process of metallurgy of copper, above equation and is more easily oxidised than the metal, then the
is involved? purification of metal may be carried out by
(a) Roasting (b) Self reduction (a) distillation (b) zone refining
(c) Refining (d) Purification (c) electrolytic refining (d) cupellation
125. Which of the following statements regarding metallurgy of 132. Germanium of very high purity is obtained by
iron is incorrect ? (a) liquation (b) vapour phase refining
(a) Reaction Fe3O 4 4CO 3Fe 4CO 2 belongs to (c) distillation (d) zone refining
lower temperature range (500 – 800K) of the blast 133. Which of the following statements regarding electrolytic
furnace. refining of copper is incorrect ?
(b) Reaction FeO CO Fe CO 2 belongs to (a) In this process anode is made up of impure copper and
higher temperature range (900 – 1500K) of the blast pure copper strips are taken as cathode.
furnace. (b) Acidic or basic solution of copper sulphate is used as
(c) The iron obtained from blast furnace is cast iron with electrolyte
3% carbon. (c) Antimony, tellurium, silver and gold are some of the
(d) For reduction of iron oxide to occur G of the couple metals deposits as anode mud during this process
of following reactions should be negative (d) Zinc can be also refined by electrolytic refining method.
1 134. Which of the following is incorrectly matched ?
FeO(s) Fe(s) O 2 (g)
2 Metal Uses
1 (a) Wrought iron Casting stoves, gutter
C(s) O2 (g) CO(g)
2 pipes, toys etc.
126. Extraction of which of the following is based on oxidation ? (b) Copper Coinage alloy
(a) Highly reactive metals (c) Aluminium Extraction of chromium and
(b) Moderately reactive metals
manganese
(c) Non-metals
(d) Nickel steel Measuring tapes
(d) Both (a) and (c)
GENERAL PRINCIPLES AND PROCESSES OF ISOLATION OF ELEMENTS 341

FACT / DEFINITION TYPE QUESTIONS 25. (a)


26. (b) Leaching is a process used for concentration of ore. In
1. (a) Argentite or silver glance (Ag 2S) is an ore of Ag. this process, a powdered ore is treated with a suitable
reagent (such as acids, bases or other chemicals)
2. (a) Cinnabar (HgS) is an ore of Hg.
which can selectively dissolve the ore, but not the
3. (a) Bauxite ore of aluminium is Al 2 O3.2H 2O . impurities.
4. (a) 27. (d) Au and Ag can be extracted from their native ores by
5. (b) Al is most abundant metal on the surface of the earth. leaching (Mac-Arthur Forrest cyanide process).
6. (c) Carborundum - SiC 28. (c)
Epsomite or Epsom salt - MgSO4.7H2O 29. (c) Cinnabar is sulphide ore (HgS). Hence purified by froth
Cassiterite - SnO2 floatation process.
Spodumene - Ore of lithium 30. (a) Ag is leached by cyanide process.
7. (a) Chalcopyrite : CuFeS2 31. (c) Pyrolusite is MnO2. Hence not concentrated by froth
Fool's gold : FeS2 floatation process.
Carnalite : KMgCl3.6H2O 32. (c) Froth reduces the surface tension of water and the
Bauxite : Al2O3.2H2O solution forms froth.
8. (c) Haematite is Fe2O3. Thus it is the ore of iron (Fe). 33. (a) Froth flotation process is based on wetting properties
9. (c) Azurite is a basic carbonate ore of copper. of ore particles.
2CuCO3. Cu(OH)2 34. (c) The surface of particles not wetted hence they float at
10. (d) Fe3O4 – Magnetite the surface
CuCO3·Cu(OH)2 – Malachite 35. (a) To remove moisture and non-metallic impurities like S,
Pyrolusite – MnO2 and Cassiterite – SnO2. P and As are oxidised and are removed as volatile
11. (a) substances.
12. (a) The formula of magnetite is Fe3O4. S8 8O 2 8SO 2 ; P4 5O2 P4 O10
13. (c) Impurities associated with minerals are called gangue
4As 3O2 2As2O3
or matrix.
36. (c) In this process sulphide ores are converted into oxide
14. (c)
ores.
15. (b) Malachite is an ore of copper Cu OH 2 .CuCO3 . 2ZnS + 3O2 2ZnO + 2SO2
16. (d) Cassiterite is an ore of Sn also known as tin stone 37. (a) 38. (d)
SnO2. 39. (a) Carbon reduction, Fe2O3 + 3 C 2Fe + 3CO
17. (a) Galena is an ore of lead. It is PbS. 40. (b) Flux + Gangue Slag
18. (a) Gold being least reactive found native. 41. (a) Calcination is heating ore in absence of air to remove
19. (a) moisture and volatile impurities. Carbonate ores
20. (c) Zincite is ZnO. decomposed to corresponding oxides as a result of
21. (c) Galena is PbS and thus purified by froth floatation calcination.
method. 42. (d) Calcination is a process of heating a substance to a
Froth flotation method is used to concentrate sulphide high temperature but below the melting or fusion point,
ores. This method is based on the fact that the surface causing loss of moisture, reduction or oxidation and
of sulphide ores is preferentially wetted by oils while dissociation into simpler substances.
that of gangue is preferentially wetted by water. 43. (b) Since silica is acidic impurity the flux must be basic.
22. (d) Froth floatation process is used for the concentration CaO SiO2 CaSiO3
of sulphide ores. 44. (b) To remove acidic impurities basic flux is added which
23. (b) Cassiterite contains the magnetic impurities of FeSO 4 is CaCO3 .
and thus concentrated by electromagnetic separation. 45. (d) Decomposition of carbonates and hydrated oxides.
24. (d) Cyanide process is used in the metallurgy of Ag 46. (d) 2CuO CuS 3Cu SO 2 (Self - reduction)
2Ag 2S 8 NaCN O 2 2 H 2 O 47. (a) For example, Ag2S is converted into Na[Ag(CN)2].
4 Na[ Ag (CN ) 2 ] 4 NaOH 2S When Zn is added, Ag is displaced.
48. (b)
2 Na [ Ag ( CN ) 2 ] Zn Na 2 [ Zn (CN ) 4 ] 2 Ag
EBD_7207
342 GENERAL PRINCIPLES AND PROCESSES OF ISOLATION OF ELEMENTS
49. (c) Cresol is used as froth stabiliser. 68. (a) 69. (c) 70. (a)
50. (b) Calcination involves heating when the volatile matter 71. (d) Blister-Copper contains 1 – 2 % impurities. It is
escapes leaving behind the metal oxide. obtained after Bessemerisation of crude copper.
51. (b) In the graph of r G° vs T for formation of oxides, the 72. (a)
Cu2O line is almost at the top. So, it is quite easy to 73. (b) Pig iron contains 4% carbon and many impurities in
reduce oxide ores of copper directly to the metal by smaller amount.
heating with coke both the lines of C, CO and C, CO 2 74. (d) 75. (c) 76. (a)
are at much lower temperature (500 - 600 K). 523K 1700K
77. (a) Ti 2I2 TiI4 Ti 2I 2
Cu 2 O C 2Cu CO Volatile Pure metal
52. (d) Stable compound
53. (c) Ellingham diagram normally consists of plots of fGº 78. (c) Liquation process, Mond’s process and, van Arkel
Vs T for the formation of oxides of elements. process these are the refining processes that are applied
54. (c) depending upon the nature of the metal under treatment
55. (b) Fe2 O3 3CO 2Fe 3CO 2 and nature of the impurities whereas amalgamation
56. (c) process is used for the extraction of noble metals like
gold, silver, etc, from native ores. The metal is
57. (c) In blast furnace at about 1270 K, calcium carbonate is
recovered from the amalgam by subjecting it to
almost completely decomposed to give CaO which acts
distillation, where the mercury distils over leaving
as a flux and combines with SiO2 present as impurity
behind the metal.
(gangue) in the ore to form calcium silicate (fusible
slag) Hg-vapours
CaO(s) (basic flux) + SiO2 (s) (acidic flux) Ore + Hg Amalgam Distilled
CaSiO3 (s) (slag) Metal
58. (b) These are the substances which can withstand very 79. (b) Zr and Ti are purified by van Arkel method.
high temperature without melting or becoming soft. 870K
Zr(s ) 2I2 ( g ) ZrI4 ( g )
59. (a)
2075K
60. (d) Cast iron is different from pig iron and is made by ZrI4 ( g ) Zr(s ) 2I 2 ( g )
Tugsten filament
melting pig iron with scrap iron and coke using hot air
Ti(s) + 2I2(s) 523K TiI4(g)
blast. It has slightly lower carbon content (about 3%)
and is extremely hard and brittle.
1700K
61. (a) Cuprous oxide formed during roasting of cuprous
sulphide is mixed with few amount of cuprous sulphide Ti(s) + 2I2(g)
and heated in a reverberatory furnace to get metallic Pure titanium
copper. 80. (a) Zone refining is based on the difference in solubility
2Cu 2O Cu 2S 6Cu SO2 of impurities in molten and solid state of the metal.
This method is used for obtaining metals of very high
62. (b) Extraction of Zn from ZnS (Zinc blende) is achieved
purity.
by roasting followed by reduction with carbon.
81. (d) Si obtained by reduction of SiCl4 with H2 is further
2ZnS + 3O2 2ZnO + 2SO2 purified by zone refining method to get Si of very high
ZnO + C Zn + CO purity. Silicon is purified by zone-refining process
63. (d) Decomposition of carbonates and hydrated oxides. because the impurities present in it are more soluble in
64. (b) Aluminothermite process involves reduction of oxides the liquid phase than in the solid phase.
which are not satisfactorily reduced by carbon such 82. (c) 83. (a)
as Fe 2 O 3 , Mn 3 O 4 , Cr 2 O 3 , etc. to metals with 84. (c) Mobile phase cannot be solid.
aluminium. 85. (b) Zinc dust is used as a reducing agent in the manufacture
Cr2 O3 2Al Al 2 O 3 2Cr H ve of dye-stuffs, paints etc.
65. (c) Because Na is very reactive and cannot be extracted
by means of the reduction by C, CO etc. So it is extracted STATEMENT TYPE QUESTIONS
by electrolysis. 86. (a) For ore containing mixture of ZnS and PbS, depressant
66. (c) NaCN allows PbS to come with froth and prevents
67. (c) Fused alumina (Al2O3) is a bad conductor of electricity. ZnS from coming to the froth.
Therefore, cryolite (Na3AlF6) and fluorspar (CaF2) are 87. (b) Ellingham diagram represents plot between G and T
added to purified alumina which not only make alumina therefore with increase in temperature phase change
a good conductor of electricity but also reduce the Gas Liquid is not possible. Ellingham diagram does
melting point of the mixture to around 1140 K. not give any information about kinetics of the reduction
reaction.
GENERAL PRINCIPLES AND PROCESSES OF ISOLATION OF ELEMENTS 343

88. (c) 110. (d) Leaching is the selective dissolution of the desired
89. (c) Van Arkel method involves heating crude Zr with mineral leaving behind the impurities in a suitable
iodine to form corresponding iodide. The metal iodide dissolving agent e.g.,
being more covalent volatilises. Argentitie or Silver glance, Ag2S is an ore of silver.
Silver is extracted from argentite by the mac-Arthur
MATCHING TYPE QUESTIONS and Forest process (leaching process).
90. (a) 91. (c) 92. (a) 93. (b) 94. (a) Ag 2S 4NaCN 2Na[Ag CN 2 ] Na 2S
95. (b) Cyanide process is for gold (A - s); floatation process
- pine oil (B - q); Electrolytic reduction - Al (C - r); 4Au 8KCN 2H 2O O 2 4K[Au CN 2 ] 4KOH
Zone refining -Ge (D - p). 111. (d) 112. (d)
96. (b) 97. (a) 113. (b) The reactions involved in cyanide extraction process
are :
ASSERTION-REASON TYPE QUESTIONS
Ag 2 S + 4NaCN 2Na [Ag(CN)2] + Na2S
98. (c) Assertion is true but reason is false. (argentite)
Oxide ores being heavier than the earthy or rocky
4Na2S + 5O 2 + 2H2O 2Na2SO4 + 4NaOH + 2S
gangue particles, settle down while lighter impurities
Oxiding
are washed away. agent
99. (a)
2Na[Ag(CN)2] + Zn Na2 [Zn(CN)4] + 2
100. (d) Assertion is false but reason is true. Leaching is a (reducing
agent)
process of concentration.
101. (b) Both assertion and reason are true but reason is not Ag
the correct explanation of assertion. Non fusible mass 114. (b)
present in ore in mixing with suitable flux are fused 115. (d) The sulphide ore is roasted to oxide before reduction
which are then reduced by coke to give free metal.
because the Gof of most of the sulphides are greater
102. (a) Both assertion and reason are correct and reason is
than those of CS2 and H2S, therefore neither C nor H
the correct explanation of assertion.
can reduce metal sulphide to metal. Further, the
103. (a) standard free energies of formation of oxide are much
less than those of SO2. Hence oxidation of metal
CRITICAL THINKING TYPE QUESTIONS
sulphides to metal oxide is thermodynamically
104. (c) Malachite is CuCO3 . Cu(OH)2 it is ore of copper. favourable.
105. (a) (a) Zinc Calamine is ZnCO3 116. (d) Ellingham diagrams are based on thermodynamic
(b) Silver Ilmenite is FeTiO3 concepts. It does not tell anything about the kinetics
(c) Magnesium Cassiterite is SnO2 of the reduction process.
(d) Tin Azurite is 117. (d) For a spontaneous reaction , Gº must be negative
and it can be possible only in this case when x < y
[2CuCO3.Cu (OH)2]
118. (b) The two equation are:
106. (c) Cuprite : Cu2O; Chalcocite : Cu2S; Chalcopyrite : CuFeS2;
Malachite: Cu(OH)2 .CuCO3. We see that CuFeS2 4 2
Al(s) O 2 (g) Al2 O3 (s), f Gº 827kJ mol 1
contains both Cu and Fe. 3 3
107. (a) Silver, copper and lead are commonly found in earth's … (1)
crust as Ag2S (silver glance), CuFeS2 (copper pyrites) 4 2
and PbS (galena) Cr(s) O 2 (g) Cr2 O3 (s), f Gº 540kJ mol 1
3 3
108. (b) Except chromium all the given metals exists as their
… (2)
sulphides.
Subtracting equation (ii) from equation (i) we have,
Zn exists as zinc blende ZnS.
Silver exists as silver glance Ag2S. 4 2 2 4
Al(s) Cr2O3 (s) Al 2O 3 (s) Cr(s),
Iron exists as iron pyrites FeS2. 3 3 3 3
Mercury exists as mercuric sulphide HgS. 1
rG 287kJ mol
109. (a) MnO2 is pyrolusite (oxide ore).
EBD_7207
344 GENERAL PRINCIPLES AND PROCESSES OF ISOLATION OF ELEMENTS

119. (a) FeO is capable forming slag with SiO2 126. (c) Extraction of non-metals are based on oxidation. For
example extraction of chlorine from brine.
SiO 2 FeO FeSiO 3
2Cl–(aq) + 2H2O(l) 2OH–(aq) + H2(g) + Cl2(g)
120. (a) In blast furnace Fe2O3 is finally reduced to Fe at 993°C 127. (a) (iii) and (iv) reactions occur in the temperature range of
121. (c) 122. (b) 123. (b) 900 – 1500K in blast furnace.
124. (b) This process is also called autoreduction process or 128. (b) For each kg of Al produced, about 0.5 kg of carbon
air reduction process. The sulphide ores of less anode is burnt away.
electropositive metals are heated in air to convert part 129. (d) Na reacts vigorously with water (exothermic process )
of the ore into oxide or sulphate which then react with 130. (b) During the process of electrolytic refining of copper
the remaining sulphide ore to give the metal and Ag and Au are obtained as anode mud.
sulphur dioxide. 131. (d)
2Cu 2S 3O2 2Cu 2O 2SO2 132. (d) Metals of high purity are obtained by zone refining
Cu 2S 2Cu 2O 6Cu SO2 e.g., silicon, germanium, boron, gallium, indium.
133. (b) During electrolytic refining of copper electrolyte used
125. (c) The iron obtained from blast furnace is pig iron with
is acidified solution of copper sulphate.
4% carbon and impurities like S, P, Mn etc., in small
134. (a) Wrought iron is used in making anchors, wires, bolts
amount.
chains and agricultural implements.
21
THE p-BLOCK ELEMENTS
(GROUP 15, 16, 17 AND 18)
FACT / DEFINITION TYPE QUESTIONS 10. Pick out the wrong statement.
(a) Nitrogen has the ability to form p -p bonds with itself.
1. Ionic radii (in Å) of As3+, Sb3+ and Bi3+ follow the order (b) Bismuth forms metallic bonds in elemental state.
(a) As3+ > Sb3+ > Bi3+ (b) Sb3+ > Bi3+ >As3+ (c) Catenation tendency is higher in nitrogen when
3+
(c) Bi > As > Sb3+ 3+ (d) Bi3+ > Sb3+ > As3+ compared with other elements of the same group.
2. Which of the following statements is not correct for (d) Nitrogen has higher first ionisation enthalpy when
nitrogen? compared with other elements of the same group.
(a) Its electronegativity is very high 11. Nitrogen forms N2, but phosphorus is converted into P4
(b) d-orbitals are available for bonding from P, the reason is
(c) It is a typical non-metal (a) Triple bond is present between phosphorus atom
(d) Its molecular size is small (b) p – p bonding is strong
3. Collectively the elements of group 15 are called – (c) p – p bonding is weak
(a) pnicogens (b) pnicopens
(d) Multiple bond is formed easily
(c) nicopen (d) None of these
12. What causes nitrogen to be chemically inert ?
4. Which one of the following elements is most metallic ?
(a) P (b) As (a) Multiple bond formation in the molecule
(c) Sb (d) Bi (b) Absence of bond polarity
5. Which of the following statement is incorrect for group 15 (c) Short internuclear distance
elements ? (d) High bond energy
(a) Order of ionization enthalpies is 13. Among the 15th group elements, as we move from nitrogen
H H H to bismuth, the pentavalency becomes less pronounced
i 1 i 2 i 3
(b) The boiling point and melting point increases from top and trivalency becomes more pronounced due to
to bottom in the group (a) Non metallic character (b) Inert pair effect
(c) Dinitrogen is a gas while all others are solids (c) High electronegativity (d) Large ionization energy
(d) All statements are correct 14. Pentavalence in phosphorus is more stable when compared
6. Which of the follow group 15 element forms metallic bonds to that of nitrogen even though they belong to same group.
in elemental state ? This is due to
(a) As (b) P (a) dissimilar electronic configuration
(c) Sb (d) Bi (b) due to presence of vacant d-orbitals
7. The three important oxidation states of phosphorus are (c) reactivity of phosphorus
(a) –3, +3 and +5 (b) –3, +3 and –5 (d) inert nature of nitrogen
(c) –3, +3 and +2 (d) –3, +3 and +4 15. Which one has the lowest boiling point ?
8. Nitrogen is relatively inactive element because (a) NH3 (b) PH3
(a) its atom has a stable electronic configuration (c) AsH3 (d) SbH3
(b) it has low atomic radius 16. Most acidic oxide among the following is –
(c) its electronegativity is fairly high
(a) N2O5 (b) P2O5
(d) dissociation energy of its molecule is fairly high
(c) N2O4 (d) As 2 O 3
9. Which of the following has the highest p – p bonding
tendency ? 17. Which of the following species has the highest dipole moment?
(a) N (b) P (a) NH3 (b) PH3
(c) As (d) Sb (c) AsH3 (d) SbH3
EBD_7207
346 THE p-BLOCK ELEMENTS (GROUP 15, 16, 17 AND 18)
18. The correct decreasing order of basic strength is: 29. NH3 gas is dried over :
(a) AsH3 SbH3 PH3 NH3 (a) CaO (b) HNO3
(b) SbH 3 AsH 3 PH 3 NH 3 (c) P2O5 (d) CuSO4
30. The shape of ammonia molecule is
(c) NH 3 PH 3 AsH 3 SbH 3
(a) tetrahedral (b) pyramidal
(d) PH 3 AsH 3 SbH 3 NH 3
(c) planar triangle (d) octahedral
19. Which of the following fluorides does not exist? 31. When ammonia is heated with cupric oxide, a molecule of
(a) NF5 (b) PF5 ammonia will
(c) AsF 5 (d) SbF5 (a) gain 3 electrons (b) lose 3 electrons
20. The p-block element of group 15 that forms predominantly (c) gain 2 electrons (d) lose 2 electrons
basic oxide is
32. In which the NH3 is not used ?
(a) N (b) P
(a) Cold storage
(c) As (d) Bi
(b) Anaesthetic
21. With respect to protonic acids, which of the following
statements is correct ? (c) Manufacture of rayon and plastic
(a) PH3 is more basic than NH3 (d) None of these
(b) PH3 is less basic than NH3 33. Liquid ammonia bottles are opened after cooling them in ice
(c) PH3 is equally basic as NH3 for sometime. It is because liquid NH3
(d) PH3 is amphoteric while NH3 is basic (a) Brings tears to the eyes
22. PCl5 is possible but NCl5 does not exist : (b) Has a high vapour pressure
(a) in N, d-sub-shell is absent (c) Is a corrosive liquid
(b) ionization energy of N is very high (d) Is a mild explosive
(c) it does not like Cl 34. Ammonia is generally manufactured for fertilizers by the
(d) None of these reaction
23. Maximum covalency of nitrogen is __________. (a) 2 NH 4 Cl Ca (OH ) 2 CaCl 2 2H 2 O 2 NH 3
(a) 3 (b) 5 (b) By passing an electric discharge in a mixture of N2 and
(c) 4 (d) 6 H2
24. Elements of group-15 form compounds in +5 oxidation state. (c) By passing a mixture of N2 and H2 under high pressure
However, bismuth forms only one well characterised and moderate temperature over a catalyst
compound in +5 oxidation state. The compound is (d) None of these
(a) Bi2O5 (b) BiF5 35. Nitrogen dioxide cannot be obtained by heating :
(c) BiCl5 (d) Bi2S5 (a) KNO3 (b) Pb(NO3)2
25. Pure nitrogen is prepared in the laboratory by heating a mixture (c) Cu(NO3)2 (d) AgNO3
of
36. Which of the following oxides is neutral ?
(a) NH4OH + NaCl (b) NH4NO3 + NaCl
(a) N2O3 (b) N2O4
(c) NH4Cl + NaOH (d) NH4Cl + NaNO2.
(c) N2O5 (d) N2O
26. On heating ammonium dichromate and barium azide
separately we get 37. The bonds present in N2O5 are :
(a) N2 in both cases (a) only ionic (b) covalent and coordinate
(b) N2 with ammonium dichromate and NO with barium (c) only covalent (d) covalent and ionic
azide 38. Which of the following oxides of nitrogen is a coloured
(c) N2O with ammonium dichromate and N2 with barium gas?
azide (a) N2O (b) NO
(d) N2O with ammonium dichromate and NO2 with barium (c) N2O5 (d) NO2
azide 39. Which of the following shows nitrogen with its increasing
27. In Haber’s process for the manufacture of NH 3 : order of oxidation number?
(a) NO < N2O < NO2 < NO3– < NH4+
(a) finely divided nickel is used as a catalyst
(b) NH4+ < N2O < NO2 < NO3– < NO
(b) finely divided iron is used as a catalyst
(c) finely divided molybdenum is used as a catalyst (c) NH4+ < N2O < NO < NO2 < NO3–
(d) no catalyst is necessary (d) NH4+ < NO < N2O < NO2 < NO3–
28. Ammonia on reaction with hypochlorite anion can form : 40. In which one of the following oxides of nitrogen, one
(a) NO (b) N2H4 nitrogen atom is not directly linked to oxygen?
(c) NH4Cl (d) Both (b) and (c) (a) NO (b) N2O4
(c) N2O (d) N2O3
p-BLOCK ELEMENTS (GROUP 15, 16, 17 AND 18) 347
41. Which of the following oxides of nitrogen reacts with FeSO4 53. The nitrogen oxides that contain(s) N–N bond(s) is /are
to form a dark brown compound (i) N2O (ii) N2O3
(a) N2O (b) NO (iii) N2O4 (iv) N2O5
(c) NO2 (d) N2O3 (a) (i) , (ii) (b) (ii) , (iii) , (iv)
42. Which oxide of nitrogen is obtained on heating ammonium (c) (iii) , (iv) (d) (i), (ii) and (iii)
nitrate at 250ºC ? 54. Zinc on reaction with dilute HNO3 gives x and zinc on
(a) Nitric oxide (b) Nitrous oxide reaction with concentrated HNO3 gives y. Identify x and y.
(c) Nitrogen dioxide (d) Dinitrogen tetraoxide (a) x = NO2 , y = N2O (b) x = N2O , y = NO
43. Which of the following can be used as an anaesthesia ? (c) x = NO , y = NO2 (d) x = N2O , y = NO2
(a) N2O (b) NO 55. Which of the following is incorrect for white and red
(c) NCl3 (d) NO2 phosphorus ?
44. A deep brown gas is formed by mixing two colourless gases (a) They are both soluble in CS2
which are (b) They can be oxidised by heating in air
(a) NO2 and O2 (b) N2O and NO (c) They consist of the same kind of atoms
(d) They can be converted into one another
(c) NO and O2 (d) NH3 and HCl
56. Which of the following phosphorus is most reactive ?
45. Which of the following elements does not form stable
diatomic molecules ? (a) Red phosphorus (b) White phosphorus
(c) Scarlet phosphorus (d) Violet phosphorus
(a) Iodine (b) Phosphorus
57. White phosphorus is
(c) Nitrogen (d) Oxygen
(a) a monoatomic gas (b) P4, a tetrahedral solid
46. The catalyst used in the manufacture of HNO 3 by (c) P8, a crown (d) a linear diatomic molecule
Ostwald’s process is : 58. Which property of white phosphorus is common to
(a) platinum gauze (b) vanadium pentoxide red phosphorous ?
(c) finely divided nickel (d) platinum black . (a) It burns when heated in air.
47. Concentrated nitric acid, upon long standing, turns yellow (b) It reacts with hot caustic soda solution to give
brown due to the formation of phosphine.
(a) NO (b) NO2 (c) It shows chemiluminescence.
(c) N2O (d) N2O4 (d) It is soluble in carbon disulphide.
48. Which of the following trihalide is unstable? 59. Which of the following statements regarding allotropic
(a) NF3` (b) AsCl3 forms of phosphorus is incorrect?
(a) White phosphorus is more reactive than red and black
(c) SbBr3 (d) NCl3
due to high angular strain.
49. What will be the A and B in the following equations.
(b) Red phosphorus on heating catches fire and give
8NH3 3Cl2 6NH 4Cl N 2 dense red fumes of P4O10.
A (c) Red phosphorus is polymeric in nature consisting of
NH3 3Cl 2 NCl3 3HCl chains of P4 tetrahedral.
B (d) Black phosphorus has two forms -black and -black
(a) A = Excess, B = Excess phosphorus
(b) A = Limited, B = Excess 60. Which of the following is incorrect?
(c) A = Excess, B = Limited (a) M.p of monoclinic sulphur > m.p. of rhombic sulphur.
(d) A = Limited, B = Limited (b) Specific gravity of rhombic sulphur > specific gravity
of monoclinic sulphur.
50. Which of the following is the strongest reducing agent ?
(c) Monoclinic sulphur is stable below 369 K.
(a) NH3 (b) PH3
(d) Both rhombic sulphur and monoclinic sulphur have
(c) BiH3 (d) SbH3 S8 molecules.
51. Which of the following element will form acidic oxides of 61. One mole of calcium phosphide on reaction with excess
type E2O3? water gives
(a) As (b) Sb (a) one mole of phosphine
(c) Bi (d) P (b) two moles of phosphoric acid
52. Which one of the following is not an use of ammonia ? (c) two moles of phosphine
(a) To produce various nitrogenous fertilizers. (d) one mole of phosphorus pentoxide
(b) In manufacture of nitric acid 62. PH3, the hydride of phosphorus is
(c) As a refrigerate (a) metallic (b) ionic
(d) In the pickling of stainless steel (c) non-metallic (d) covalent
EBD_7207
348 THE p-BLOCK ELEMENTS (GROUP 15, 16, 17 AND 18)
63. Phosphine is not obtained by which of the following reaction (a) zero (b) two
(a) White P is heated with NaOH (c) three (d) four
(b) Red P is heated with NaOH 75. Oxidation states of P in H4 P2O5 , H4 P2O6 , and H4 P2O7 ,
(c) Ca3P2 reacts with water
are respectively:
(d) Phosphorus trioxide is boiled with water
(a) + 3, + 5, + 4 (b) + 5, + 3, + 4
64. Phosphine is not evolved when
(c) + 5, + 4, + 3 (d) + 3, + 4, + 5
(a) white phosphorus is boiled with a strong solution of
76. How many bridging oxygen atoms are present in P4O10?
Ba(OH)2
(a) 5 (b) 6
(b) phosphorus acid is heated
(c) 4 (d) 2
(c) calcium hypophosphite is heated
77. Which of the following statements is not valid for oxoacids
(d) metaphosphoric acid is heated.
of phosphorus?
65. Pure phosphine is not combustible while impure phosphine
(a) Orthophosphoric acid is used in the manufacture of
is combustible, this combustibility is due to presence of
triple superphosphate.
(a) P2 H 4 (b) N2 (b) Hypophosphorous acid is a diprotic acid.
(c) PH 5 (d) P2 O5 (c) All oxoacids contain tetrahedral four coordinated
phosphorus.
66. When orthophosphoric acid is heated to 600°C, the product
(d) All oxoacids contain atleast one P = O and one P —
formed is
OH group.
(a) PH3 (b) P2O5
78. What is hybridization of P in PCl5 ?
(c) H3PO3 (d) HPO3
(a) sp3 (b) sp3 d 2
67. P2O5 is heated with water to give 3
(c) sp d (d) sp2
(a) hypophosphorous acid(b) phosphorous acid
79. Which of the following is a cyclic phosphate ?
(c) hypophosphoric acid (d) orthophosphoric acid
(a) H3 P3 O10 (b) H6 P4 O13
68. Basicity of orthophosphoric acid is
(c) H5 P5 O15 (d) H7 P5 O16
(a) 2 (b) 3
80. P—O—P bond is present in
(c) 4 (d) 5
(a) H4P2O6 (b) H4P2O5
69. PCl3 reacts with water to form
(c) Both (a) and (b) (d) Neither (a) nor (b)
(a) PH3 (b) H3PO4 and HCl
81. Orthophosphoric acid is
(c) POCl3 (d) H3PO4
(a) monobasic (b) dibasic
70. H3PO2 is the molecular formula of an acid of phosphorus.
(c) tribasic (d) tetrabasic
Its name and basicity respectively are
82. The oxyacid of phosphorous in which phosphorous has
(a) phosphorus acid and two
the lowest oxidation state is
(b) hypophosphorous acid and two
(a) hypophosphorous acid
(c) hypophosphorous acid and one
(b) orthophosphoric acid
(d) hypophosphoric acid and two
(c) pyrophosphoric acid
71. The structural formula of hypophosphorous acid is
(d) metaphosphoric acid
O O 83. The number of P—O—P bonds in cyclic metaphosphoric
P P acid is
(a) H (b) H (a) zero (b) two
OH OH
H OH (c) three (d) four
84. Among the oxyacids of phosphorus, the dibasic acid is
O O (a) H4P2O7 (b) H3PO2
P P (c) HPO3 (d) H3PO3
(c) HO (d) H
OH OOH 85. The basicity of pyrophosphorus acid is
OH OH (a) 2 (b) 4
72. Number of sigma bonds in P4O10 is (c) 1 (d) 5
(a) 6 (b) 7 86. The oxidation state of phosphorus in
(c) 17 (d) 16. cyclotrimetaphosphoric acid is
73. The number of hydrogen atom(s) attached to phosphorus (a) +3 (b) +5
atom in hypophosphorous acid is (c) –3 (d) +2
(a) three (b) one 87. Which acid has P – P linkage ?
(c) two (d) zero (a) Hypophosphoric acid (b) Pyrophosphoric acid
74. The number of P – O – P bonds in cyclic metaphosphoric (c) Metaphosphoric acid (d) Orthophosphoric acid
acid is
p-BLOCK ELEMENTS (GROUP 15, 16, 17 AND 18) 349
88. In a cyclotrimetaphosphoric acid molecule, how many single 100. Which of the following is not correctly matched ?
and double bonds are present? (a) SF4 – gas (b) SeF4 – liquid
(a) 3 double bonds; 9 single bonds (c) TeF4 – solid (d) SF6 – solid
(b) 6 double bonds; 6 single bonds 101. The compound which gives off oxygen on moderate heating
(c) 3 double bonds; 12 single bonds is :
(d) Zero double bonds; 12 single bonds (a) cupric oxide (b) mercuric oxide
89. Strong reducing behaviour of H3PO2 is due to (c) zinc oxide (d) aluminium oxide
(a) Low oxidation state of phosphorus 102. Oxygen molecule is
(b) Presence of two –OH groups and one P–H bond (a) diamagnetic with no-unpaired electron(s)
(c) Presence of one –OH group and two P–H bonds (b) diamagnetic with two unpaired electrons
(d) High electron gain enthalpy of phosphorus (c) paramagnetic with two unpaired electrons
90. In solid state PCl5 is a ________. (d) paramagnetic with no unpaired electron(s)
(a) covalent solid 103. The number of electrons that are paired in oxygen molecule
are
(b) octahedral structure
(a) 16 (b) 12
(c) ionic solid with [PCl6 ]+ octahedral and [PCl 4] –
(c) 14 (d) 7
tetrahedra
104. On heating KClO3 we get
(d) ionic solid with [PCl4 ]+ tetrahedral and [PCl 6 ]–
(a) KClO2 + O2 (b) KCl + O2
octahedra
(c) KCl + O3 (d) KCl + O2 + O3
91. Electron affinity of sulphur is
105. Which of the following is not oxidized by O3 ?
(a) more than O and Se
(a) KI (b) FeSO4
(b) more than O but less than Se
(c) KMnO4 (d) K2MnO4
(c) less than O but more than Se
106. About 20 km above the earth, there is an ozone layer. Which
(d) equal to O and Se one of the following statements about ozone and ozone
92. All the elements of oxygen family are layer is true?
(a) non metals (b) metalloids (a) Ozone has a triatomic linear molecule
(c) radioactive (d) polymorphic (b) It is harmful as it stops useful radiation
93. Which shows maximum catenation property ? (c) It is beneficial to us as it stops U.V radiation
(a) S (b) Se (d) Conversion of O3 to O2 is an endothermic reaction
(c) Te (d) O 107. Oxygen gas can be prepared from solid KMnO4 by :
94. Oxygen and sulphur both are the members of the same group
in periodic table but H2O is liquid while H2S is gas because (a) treating the solid with H 2 gas
(a) molecular weight of water is more (b) strongly heating the solid
(b) electronegativity of sulphur is more (c) dissolving the solid in dil. H 2SO 4
(c) H2S is weak acid
(d) dissolving solid in dil. HCl
(d) water molecules are having weak hydrogen bonds
108. Which of the following statements is correct :
between them
(a) Ozone is a resonance hybrid of oxygen
95. Which of the following hydrides has the lowest boiling
(b) Ozone is an isomer of oxygen
point?
(c) Ozone has no relationship with oxygen
(a) H2O (b) H2S
(d) Ozone is an allotropic modification of oxygen
(c) H2Se (d) H2Te 109. Which of the following on thermal decomposition gives
96. Which of the following hydrides is most acidic ? oxygen gas ?
(a) H2Te (b) H2Se (a) Ag2 O (b) Pb3O4
(c) H2O (d) H2S (c) PbO2 (d) All of these
97. Which of the following hydrides shows the highest boiling 110. Which of the following is an acidic oxide?
point ? (a) Mn 2O7 (b) Na2O
(a) H2O (b) H2S (a) N2O (b) BaO
(c) H2Se (d) H2Te 111. Atomicity of sulphur in rhombic sulphur is
98. Which is the best oxidising agent among the following ? (a) 1 (b) 2
(a) S (b) O (c) 8 (d) 6
(c) Se (d) Te 112. Which of the following form of the sulphur shows
99. Which of the following oxide is amphoteric ? paramagnetic behaviour ?
(a) SnO2 (b) CaO (a) S8 (b) S6
(c) SiO2 (d) CO2 (c) S2 (d) All of these
EBD_7207
350 THE p-BLOCK ELEMENTS (GROUP 15, 16, 17 AND 18)
113. What is X in the following reaction ? 126. Hot conc. H2SO4 acts as moderately strong oxidising agent.
X It oxidises both metals and nonmetals. Which of the
2SO2(g) + O2(g) 2SO3(g) following element is oxidised by conc. H2SO4 into two
(a) V2O5 (b) CuO gaseous products?
(c) CuCl2 (d) MnO2 (a) Cu (b) S
114. Which of the following oxo acid of sulphur has O–O bond ? (c) C (d) Zn
(a) H2S2O7 (b) H2S2O8 127. Caro’s acid is
(c) H2S2O6 (d) H2S2O5 (a) H2SO3 (b) H3S2O5
115. Carbohydrates on reaction with conc. H2SO4 becomes (c) H2SO5 (d) H2S2O8
charred due to
128. Sulphuric acid reacts with PCl5 to give
(a) hydrolysis (b) dehydration
(a) thionyl chloride (b) sulphur monochloride
(c) hydration (d) oxidation
(c) sulphuryl chloride (d) sulphur tetrachloride
116. Which of the following is the key step in the manufacture of
sulphuric acid ? 129. Which one of the following reacts with conc. H2SO4?
(a) Burning of sulphur or sulphide ores in air to generate (a) Au (b) Ag
SO2 (c) Pt (d) Pb
(b) Conversion of SO2 to SO3 by the reaction with oxygen 130. The number of dative bonds in sulphuric acid molecule is
in presence of catalyst. (a) 0 (b) 1
(c) Absorption of SO3 in H2SO4 to give oleum. (c) 2 (d) 4
(d) Both (b) and (c) 131. What is the number of sigma ( ) and pi ( ) bonds present
117. Hybridization of S in SO3 is in sulphuric acid molecule ?
(a) sp 2 (b) sp 3 (a) 6 , 2 (b) 6 , 0
(c) sp d2 (d) sp 3 d 2 (c) 2 , 4 (d) 2 , 2
118. By which of the following SO2 is formed ? 132. Which characteristic is not correct about H2SO4 ?
(a) Reaction of dil. H2SO4 with O2 (a) Reducing agent (b) Oxidising agent
(b) Hydrolysis of dil. H2SO4 (c) Sulphonating agent (d) Highly viscous
(c) Reaction of conc. H2SO4 with Cu 133. Among F, Cl, Br and I the lowest ionization potential will be
(d) None of these of
119. Number of bonds in SO2 are (a) fluorine (b) chlorine
(a) two and two (b) two and one (c) bromine (d) iodine
(c) two and three (d) None of these 134. The electronegativity follows the order
120. Bleaching action of SO2 is due to its (a) F > O > Cl > Br (b) F > Cl > Br > O
(a) oxidising property (b) acidic property (c) O > F > Cl > Br (d) Cl > F > O > Br
(c) reducing property (d) basic property
135. The bond energies of F2 , Cl2, Br2 and I2 are 155, 244, 193
121. The acid which has a peroxy linkage is and 151 kJ mol–1 respectively. The weakest bond will be in
(a) Sulphurous acid (b) Pyrosulphuric acid
(a) Br2 (b) Cl2
(c) Dithionic acid (d) Caro’s acid
(c) F2 (d) I2
122. S – S bond is not present in
136. The outer electronic structure of 3s2 3p5 is possessed by
(a) S 2 O 24 (b) S2 O 52 (a) O (b) Cl
(c) S2 O 32 (d) S2 O 27 (c) Br (d) Ar
137. Electron gain enthalpy with negative sign of fluorine is less
123. Oleum is
than that of chlorine due to :
(a) castor Oil (b) oil of vitriol
(a) High ionization enthalpy of fluorine
(c) fuming H 2SO 4 (d) None of them
(b) Smaller size of chlorine atom
124. On addition of conc. H2SO4 to a chloride salt, colourless
fumes are evolved but in case of iodide salt, violet fumes (c) Smaller size of fluorine atom
come out. This is because (d) Bigger size of 2p orbital of fluorine
(a) H2SO4reduces HI to I2 138. Which one of the following order is correct for the bond
(b) HI is of violet colour energies of halogen molecules ?
(c) HI gets oxidised to I2 (a) I2 > Cl2 > Br2 (b) Br2 > Cl2 > I2
(d) HI changes to HIO3 (c) I2 > Br2 > Cl2 (d) Cl2 > Br2 > I2
125. Which of the following are peroxoacids of sulphur? 139. The correct order of reactivity of halogens with alkalies is
(a) H2SO5 and H2S2O8 (b) H2SO5 and H2S2O7 (a) F > Cl > Br > I (b) F < Cl > Br < I
(c) H2S2O7 and H2S2O8 (d) H2S2O6 and H2S2O7 (c) F < Cl < Br < I (d) F < Cl < Br > I
p-BLOCK ELEMENTS (GROUP 15, 16, 17 AND 18) 351
140. The correct order of increasing oxidising power is 154. Cl2 reacts with hot and conc. NaOH to give –
(a) F2 > Br2 > Cl2 > I2 (b) F2 < Cl2 < Br2 < I2 (a) NaClO (b) NaClO3
(c) Cl2> Br2 > F2 > I2 (d) I2 < Br 2 < Cl2 < F2 (c) NaClO2 (d) NaClO4
141. Fluorine is a stronger oxidising agent than chlorine in 155. When chlorine reacts with cold and dilute solution of sodium
aqueous solution. This is attributed to many factors except hydroxide, the products obtained are
(a) heat of dissociation (b) ionisation potential (a) Cl– + ClO– (b) Cl– + ClO 2
(c) heat of hydration (d) electron affinity
142. Fluorine exhibits an oxidation state of only –1 because (c) Cl– + ClO3 (d) Cl– + ClO 4
(a) it can readily accept an electron 156. Chlorine is liberated when we heat
(b) it is very strongly electronegative (a) KMnO4 + NaCl (b) K2Cr2O7 + MnO2
(c) it is a non-metal (c) Pb(NO3)2 + MnO2 (d) K2Cr2O7 + HCl
(d) it belongs to halogen family 157. Which of the following is used in the preparation of chlorine?
143. Which of the following halogen does not exhibit positive (a) Only MnO2
oxidation state in its compounds? (b) Only KMnO4
(a) Cl (b) Br (c) Both MnO2 and KMnO4
(c) I (d) F (d) Either MnO2 or KMnO4
144. The halogen that is most easily reduced is 158. The reaction of KMnO4 and HCl results in
(a) F2 (b) Cl2 (a) oxidation of Mn in KMnO4 and production of Cl2
(c) Br2 (d) I2 (b) reduction of Mn in KMnO4 and production of H2
145. Which one of the following elements shows more than one (c) oxidation of Mn in KMnO4 and production of H2
oxidation states ? (d) reduction of Mn in KMnO4 and production of Cl2
(a) Sodium (b) Fluorine 159. Bleaching powder on standing forms mixture of :
(c) Chlorine (d) Potassium (a) CaO Cl 2 (b) CaO CaCl 2
146. Which of the following halogens exhibit only one oxidation (c) HOCl Cl 2 (d) CaCl 2 Ca (ClO3 ) 2
state in its compounds ?
160. Hydrochloric acid at 25ºC is
(a) Bromine (b) Chlorine
(a) ionic and liquid (b) covalent and liquid
(c) Fluorine (d) Iodine
(c) ionic and gas (d) None of these
147. Which of the following is the best description for the
161. Gaseous HCl is a poor conductor of electricity while its
behaviour of bromine in the reaction given below ?
aqueous solution is a good conductor this is because
H 2 O Br2 HOBr HBr
(a) H 2 O is a good conductor of electricity
(a) Proton acceptor only
(b) Both oxidized and reduced (b) a gas cannot conduct electricity but a liquid can
(c) HCl gas does not obey Ohm’s law, whereas the solution
(c) Oxidized only
does
(d) Reduced only
(d) HCl ionises in aqueous solution
148. Among the following which is the strongest oxidising agent?
162. Which one is most stable to heat –
(a) Br2 (b) I2 (a) HClO (b) HClO2
(c) Cl2 (d) F2 (c) HClO3 (d) HClO4
149. The correct order of heat of formation of halogen acids is 163. Interhalogen compounds are more reactive than the
(a) HI > HBr > HCl > HF (b) HF > HCl > HBr > HI individual halogen because
(c) HCl > HF > HBr > HI (d) HCl > HBr > HF > HI (a) two halogens are present in place of one
150. Which is the weakest out of HF, HCl, HBr and HI? (b) they are more ionic
(a) HF (b) HCl (c) their bond energy is less than the bond energy of the
(c) HBr (d) HI halogen molecule
151. Which of the following is most volatile ? (d) they carry more energy
(a) HI (b) HBr 164. Which of the following is not the characteristic of
(c) HCl (d) HF interhalogen compounds ?
152. At room temperature, HCl is a gas while HF is a low boiling (a) They are more reactive than halogens
liquid. This is because (b) They are quite unstable but none of them is explosive
(a) H- F bond is covalent (b) H- F bond is ionic (c) They are covalent in nature
(c) HF has metallic bond (d) HF has hydrogen bond (d) They have low boiling points and are highly volatile.
153. The bleaching action of chlorine is due to 165. The hybridization in ICl7 is
(a) reduction (b) hydrogenation (a) sp3 d 3 (b) d2 sp 3
3
(c) sp d (d) sp3
(c) chlorination (d) oxidation
EBD_7207
352 THE p-BLOCK ELEMENTS (GROUP 15, 16, 17 AND 18)
166. In which of the following reactions chlorine is both reduced 180. The noble gas which was discovered first in the sun and
and oxidized? then on the earth
(a) 2KMnO4 + 16HCl 2KCl + 2MnCl2 + 8H2O + 5Cl2 (a) argon (b) xenon
(b) 6NaOH + 3Cl2 5NaCl + NaClO3 + 3H2O (c) neon (d) helium
(c) NH3 + 3Cl2 NCl3 + 3HCl 181. A radioactive element X decays to give two inert gases X is
(d) I2 + 6H2O + 5Cl2 2HIO3 + 10HCl (a) 238 (b) 226
92 U 88 Ra
167. Which of the following is observed when Cl2 reacts with (c) Both (a) and (b) (d) Neither (a) nor (b)
hot and concentrated NaOH? 182. Which of the following noble gases has the highest positive
(a) NaCl , NaOCl (b) NaCl , NaClO2 electron gain enthalpy value?
(c) NaCl , NaClO3 (d) NaOCl , NaClO3 (a) Helium (b) Krypton
168. Which one of the following noble gases is not found in the (c) Argon (d) Neon
atmosphere
183. Which inert gas show abnormal behaviour on liquefaction
(a) Rn (b) Kr
(a) Xe (b) He
(c) Ne (d) Ar
(c) Ar (d) Kr
169. The last member of the family of inert gases is
184. The ease of liquefaction of noble gases increases in the
(a) argon (b) radon
order
(c) xenon (d) neon
(a) He < Ne < Ar < Kr < Xe
170. Which of the following is the correct sequence of the noble
(b) Xe < Kr < Ne < Ar < He
gases in their group in the periodic table?
(c) Kr < Xe < He < Ne < Ar
(a) Ar, He, Kr, Ne, Rn, Xe (b) He, Ar, Ne, Kr, Xe, Rn
(c) He, Ne, Kr, Ar, Xe, Rn (d) He, Ne, Ar, Kr, Xe, Rn (d) Ar < Kr < Xe < Ne < He
171. Which of the following noble gases do not have an octet 185. The correct order of solubility in water for He, Ne, Ar, Kr,
of electrons in its outermost shell? Xe is
(a) Neon (b) Radon (a) He > Ne > Ar > Kr > Xe
(c) Argon (d) Helium (b) Ne > Ar > Kr > He > Xe
172. Number of unpaired electrons in inert gas is (c) Xe > Kr > Ar > Ne > He
(a) zero (b) 8 (d) Ar > Ne > He > Kr > Xe
(c) 4 (d) 18 186. Which one of the following elements is most reactive ?
173. In the following four elements, the ionisation potential of (a) He (b) Ne
which one is the highest ? (c) Ar (d) Xe
(a) Oxygen (b) Argon 187. Noble gases are group of elements which exhibit very
(c) Barium (d) Cesium (a) high chemical activity
174. Gradual addition of electronic shells in the noble gases (b) low chemical activity
causes a decrease in their (c) minimum electronegativity
(a) ionisation energy (b) atomic radius (d) much paramagnetic properties
(c) boiling point (d) density 188. In XeF2, XeF4, XeF6 the number of lone pairs on Xe are
175. Which of the following noble gas is least polarisable? respectively
(a) He (b) Xe (a) 2, 3, 1 (b) 1, 2, 3
(c) Ar (d) Ne (c) 4, 1, 2 (d) 3, 2, 1.
176. In which of the following groups, when He is placed, its all 189. Total number of lone pair of electrons in XeOF4 is
the properties are satisfied (a) 0 (b) 1
(a) with alkali metals (b) with halogens (c) 2 (d) 3
(c) with inert gases (d) None of these 190. Noble gases do not react with other elements because
177. The most abundant inert gas in the atmosphere is (a) they have completely filled valence shell (ns2np6)
(a) He (b) Ne (b) the sizes of their atoms are very small
(c) Ar (d) Kr (c) they are not found in abundance
178. The lowest boiling point of helium is due to its (d) they are monoatomic
(a) inertness 191. Which one of the following reactions of xenon compounds
(b) gaseous nature is not feasible?
(c) high polarisability (a) 3XeF4 + 6H 2 O 2Xe + XeO 3 +12HF +1.5O 2
(d) weak van der Waal’s forces between atoms (b) 2XeF2 + 2H 2 O 2Xe + 4HF + O 2
179. Which of the noble gas has highest polarisability?
(a) He (b) Ar (c) XeF6 + RbF Rb[XeF7 ]
(c) Kr (d) Xe (d) XeO3 + 6HF XeF6 + 3H 2 O
p-BLOCK ELEMENTS (GROUP 15, 16, 17 AND 18) 353
192. Which of the following has maximum number of lone pairs 207. Which statement about noble gases is not correct?
associated with Xe ? (a) Xe forms XeF6
(a) XeF4 (b) XeF6 (b) Ar is used in electric bulbs
(c) XeF2 (d) XeO3 (c) Kr is obtained during radioactive disintegration
193. The shape of XeO2F2 molecule is (d) He has the lowest b.pt among all the noble gases
(a) trigonal bipyramidal (b) square planar 208. The geometry of XeF6 is
(c) tetrahedral (d) see-saw (a) planar hexagon (b) regular octahedron
194. XeF4 on partial hydrolysis produces (c) distorted octahedron (d) square bipyramid
(a) XeF4 (b) XeOF2 209. Trigonal bipyramidal geometry is shown by :
(c) XeOF4 (d) XeO3 (a) XeO3F2 (b) XeO3F2
(c) FXeOSO2F (d) [XeF8]2–
195. Which element out of He, Ar, Kr and Xe forms least number
210. Which has trigonal bipyramidal shape ?
of compounds ?
(a) XeOF4 (b) XeO3
(a) He (b) Ar
(c) XeO3F2 (d) XeOF2
(c) Kr (d) Xe
211. Argon is used
196. The element which has not yet been reacted with F2 is
(a) in filling airships
(a) Ar (b) Xe (b) to obtain low temperature
(c) Kr (d) Rn (c) in high temperature welding
197. XeF6 on complete hydrolysis gives (d) in radiotherapy for treatment of cancer
(a) Xe (b) XeO2 212. Noble gases are used in discharge tubes to gives different
(c) XeO3 (d) XeO4 colours. Reddish orange glow is due to
198. XeF4 involves which hybridization (a) Ar (b) Ne
(a) sp (b) sp 2 (c) Xe (d) Kr
(c) sp d2 (d) sp 3 d 2 213. Which one of the following statements regarding helium is
199. Shape of XeOF4 is incorrect ?
(a) octahedral (b) square pyramidal (a) It is used to produce an d sustain powerful
(c) pyramidal (d) T-shaped superconducting magnets.
200. The hybridization of Xe in XeF2 is (b) It is used as a cryogenic agent for carrying out
(a) sp 3 (b) sp 2 experiments at low temperatures.
(c) sp d3 (d) sp 2 d (c) It is used to fill gas balloons instead of hydrogen
201. Which is a planar molecule ? because it is lighter and non-inflammable.
(a) XeO4 (b) XeF4 (d) It is used in gas-cooled nuclear reactors.
(c) XeOF4 (d) XeO2F2 214. The coloured discharge tubes for advertisement mainly
202. Which of the following has sp3 hybridization ? contain
(a) XeO3 (b) BCl3 (a) xenon (b) helium
(c) neon (d) argon
(c) XeF4 (d) BBr3
215. Sea divers go deep in the sea water with a mixture of which
203. The number of lone pair of electrons present on Xe in XeF 2
of the following gases
is
(a) O2 and He (b) O2 and Ar
(a) 3 (b) 4
(c) O2 and CO2 (d) CO2 and Ar
(c) 2 (d) 1 216. Which of the following is the life saving mixture for an
204. Hybridization and structure of XeF4 is asthma patient ?
(a) sp3d, trigonal bipyramidal (a) Mixture of helium and oxygen
(b) sp3, tetrahedral (b) Mixture of neon and oxygen
(c) sp3d2, square planar (c) Mixture of xenon and nitrogen
(d) sp3d2, hexagonal (d) Mixture of argon and oxygen
205. Number of lon e pairs of electrons on Xe atoms 217. Which of the following statements are true?
XeF2, XeF4 and XeF6 molecules are respectively (i) Only type of interactions between particles of noble
(a) 3, 2 and 1 (b) 4, 3 and 2 gases are due to weak dispersion forces.
(c) 2, 3 and 1 (d) 3, 2 and 0 (ii) Ionisation enthalpy of molecular oxygen is very close
206. Which one of the following is correct pair with respect to to that of xenon.
molecular formula of xenon compound and hybridization (iii) Hydrolysis of XeF6 is redox reaction.
state of Xenon in it? (iv) Xenon fluorides are not reactive.
(a) XeF4, sp3 (b) XeF2, sp (a) (i) and (iii) (b) (i) and (ii)
(c) XeF2, sp3d (d) XeF4, sp2 (c) (ii) and (iii) (d) (iii) and (iv)
EBD_7207
354 THE p-BLOCK ELEMENTS (GROUP 15, 16, 17 AND 18)
218. Which of the following element has the property of diffusing (iii) All hydrides of oxygen family possess reducing
through most commonly used laboratory materials such as property which increases from H2S to H2Te.
rubber, glass or plastics. (iv) Among hexahalides of group 16 hexafluorides are the
(a) Xe (b) Rn onlys table halides.
(c) He (d) Ar (v) Dimeric monohalides of group 16 un dergo
219. Which of the following is used to produce and sustain disproportionation.
powerful superconducting magnets to form an essential (a) TFFTT (b) FTTFF
part of NMR spectrometer ? (c) FTFTF (d) TFTFT
(a) Ar (b) Ne 224. The correct statement(s) about O3 is(are)
(c) Rn (d) He (i) O—O bond lengths are equal
(ii) Thermal decomposition of O3 is endothermic
STATEMENT TYPE QUESTIONS (iii) O3 is diamagnetic in nature
220. Which of the following statements are correct? (iv) O3 has a bent structure
(i) Arsenic and antimony are metalloids. (a) (i) and (iii) (b) (ii) and (iii)
(ii) Phosphorus, arsenic and antimony are found mainly (c) (i), (ii) and (iv) (d) (i) and (iv)
as sulphide minerals. 225. Consider the following statements
(iii) Covalent redii increases equally from N to Bi.
(iv) Elements of group 15 have extra stability and higher (i) Reaction 2Fe3+ + SO2 + 2H2O 2Fe2+ + SO24 + 4H+
ionisation energy due to exactly half filled ns2np3 shows reducing character of sulphur dioxide
electronic configuration. (ii) H2S2O8 contains four S = O, two S – OH and one O–
(v) In group 15 elements only nitrogen is gas whereas all O bond
others are solids. (iii) NH3 gas can be dried effectively by using conc.
(a) (i), (iv) and (v) (b) (ii), (iii) and (iv) H2SO4.
(c) (i), (ii) and (iii) (d) (ii), (iii) and (v) (iv) One of the major use of H2SO4 is in the manufacture
221. Read the following statements regarding chemical reactivity of fertilizers.
of group 15 elements. Which of the following is the correct code for the statements
(i) Only compound of Bi with +5 oxidation state is BiF5. above?
(ii) Intermediate oxidation states for both nitrogen and (a) TTFF (b) TTFT
phosphorus disproportionate in both acid and alkali. (c) FTFT (d) TFFT
(iii) Nitrogen due to absence of d-orbitals in its valence
226. Which of the following statements regarding properties of
shell cannot form d -p bond as the heavier elements
halogens are correct?
thus R3P = O exists but R3N = O does not exists.
(i) Due to small size electron gain enthalpy of fluorine is
(iv) BiH3 is the strongest reducing agent amongst the
less than that of chlorine.
hydrides of nitrogen family.
(v) P2O3 is more acidic than P2O5. (ii) Iodine has same physical state but different colour as
Which of the following is the correct code for the statements compare to other members of the group.
above? (iii) Fluorine shows no positive oxidation state.
(a) FTFFT (b) FFTTF (iv) In X2(g) + H2O(l) HX(aq) + HOX(aq)
(c) TFTTF (d) TFTFT (where X2 = Cl or Br)
222. Which of the following statements are correct? (a) (i), (ii) and (iv) (b) (i), (iii) and (iv)
(i) All the three N—O bond lengths in HNO3 are equal. (c) (ii), (iii) and (iv) (d) (iii) and (iv)
(ii) All P—Cl bond lengths in PCl5 molecule in gaseous 227. Consider the following statements regarding interhalogen
state are equal. compounds
(iii) P4 molecule in white phohsphorus have angular strain (i) For all types of interhalogen compounds
therefore white phosphorus is very reactive.
(iv) PCl5 is ionic in solid state in which cation is tetrahedral XX1 , XX13 , XX15 and XX17 X is the halogen of
and anion is octahedral. lesser electronegativity in comparison to X1.
(a) (i) and (iv) (b) (iii) and (iv) (ii) At 298 K all interhalogen compounds are either
(c) (ii) and (iii) (d) (ii) only
volatile solids or liquids.
223. Which of the following is the correct code for statements
(iii) ClF undergoes hydrolysis as below,
below ?
(i) Due to small size oxygen has less negative electron ClF + H2O HF + HOCl
gain enthalpy than sulphur. (iv) Fluorine containing interhalogen compounds are very
(ii) Oxygen shows only –2 oxidation state whereas S, Se useful as fluorinating agents.
and Te shows +4 O.S in their compounds with oxygen (a) TTFF (b) TFTT
and +6 with fluorine. (c) FTFT (d) TFFT
p-BLOCK ELEMENTS (GROUP 15, 16, 17 AND 18) 355
228. Which of the following statements are correct? 232. Match the columns.
(i) Among halogens, radius ratio between iodine and Column-I Column-II
fluorine is maximum. (Oxyacid) (Materials for
(ii) Leaving F—F bond, all halogens have weaker X—X preparation)
bond than X—X bond in interhalogens. (A) H3PO2 (p) Red P + alkali
(iii) Among interhalogen compounds maximum number of (B) H3PO3 (q) P4O10 + H2O
atoms are present in iodine fluoride.
(C) H3PO4 (r) P2O3 + H2O
(iv) Interhalogen compounds are more reactive than
(D) H4P2O6 (s) White P + alkali
halogen compounds.
(a) (A) – (s), (B) – (r), (C) – (q), (D) – (p)
(a) (i) and (ii) (b) (i), (ii) and (iii)
(c) (ii) and (iii) (d) (i), (iii) and (iv) (b) (A) – (p), (B) – (r), (C) – (q), (D) – (s)
229. Which of the following statements are correct? (c) (A) – (s), (B) – (r), (C) – (p), (D) – (q)
(i) Natural abundance of noble gases is ~ 1% by volume (d) (A) – (q), (B) – (r), (C) – (p), (D) – (s)
of which Ar is the major constituent. 233. Match the columns
(ii) Noble gases have high positive values of electron Column - I Column - II
gain enthalpy. (A) POCl3 (p) Contains four P – OH
(iii) Preparation of XeF2 requires F2 in excess amount. two P = O and one
(iv) Complete hydrolysis of all three XeF2, XeF4 and XeF6 P–O–P
gives Xe as one of product.
(B) H4P2O5 (q) Yellowish white
(a) (i) and (iii) (b) (ii) and (iv)
chloride of phosphorus
(c) (i) and (ii) (d) (ii) and (iii)
reacts with moist air
MATCHING TYPE QUESTIONS (C) H4P2O6 (r) Contains four P – OH,
two P = O and one P – P
230. Match the columns bond
Column-I Column-II
(D) H4P2O7 (s) Colourless oily chloride
(A) 2Pb(NO3)2 673K (p) High pressure favours of phosphorus reacts
4NO2 + 2PbO + O2 the formation of with orthophosphoric
product acid
(B) N2(g) + O2(g) (q) Product formed is acidic (a) A – (q), B – (s), C – (p), D – (r)
2NO(g) brown gas (b) A – (s), B – (q), C – (r), D – (p)
(c) A – (q), B – (s), C – (r), D – (p)
(C) NH4NO3 (r) This reaction occurs at
(d) A – (q), B – (r), C – (s), D – (p)
N2O + 2H2O a high temperature 234. Match the columns
about 2000 K Column - I Column - II
(D) N2(g) + 3H2(g) (s) Product formed is a (A) Metal that shows no (p) Platinum
2NH3(g) neutral colourless gas reaction with dioxygen
(a) A – (r, s), B – (q), C – (s), D – (p) (B) Metal forms strong (q) Nitrogen
(b) A – (q), B – (r,s), C – (s), D – (p) acidic oxide with oxygen
(c) A – (q), B – (s), C – (r, s), D – (p) (C) A non-metal discharge (r) Manganese
(d) A – (q), B – (r, s), C – (p), D – (s)
of whose oxide might
231. Match the columns
be slowly depleting the
Column - I Column - II
concentration of the
(A) Used in manufacture (p) Ammonia
of calcium cyanamide ozone layer
(B) Used in manufacture (q) Nitric acid (D) Metal which forms (s) Aluminium
of nitric acid amphoteric oxide
(C) Used in pickling of (r) Dinitrogen (a) A – (p), B – (r), C – (q), D – (s)
stainless steel (b) A – (r), B – (p), C – (q), D – (s)
(a) A – (r), B – (p), C – (q) (b) A – (p), B – (r), C – (q) (c) A – (p), B – (q), C – (r), D – (s)
(c) A – (r), B – (q), C – (p) (d) A – (q), B – (p), C – (r) (d) A – (p), B – (r), C – (s), D – (q)
EBD_7207
356 THE p-BLOCK ELEMENTS (GROUP 15, 16, 17 AND 18)
235. Match the columns. (a) A – (p), B – (r, s), C – (p, q), D – (p, s)
Column-I Column-II (b) A – (r, s), B – (p), C – (r, s), D – (p, s)
(A) Pb3O4 (p) Neutral oxide (c) A – (p), B – (p, q), C – (r, s), D – (p, s)
(B) N2O (q) Acidic oxide (d) A – (p), B – (r, s), C – (p, s), D – (p, q)
(C) Mn 2O7 (r) Basic oxide 240. Match the columns.
(D) Bi2O3 (s) Mixed oxide Column-I Column-II
(a) A – (p), B – (q), C – (r), D – (s) (A) Partial hydrolysis of the (p) He
(b) A – (s), B – (p), C – (q), D – (r) compound does not change
(c) A – (r), B – (q), C – (s), D – (p) oxidation state of central atom
(d) A – (s), B – (r), C – (p), D – (q) (B) It is used in modern diving (q) XeF6
236. Match the columns. apparatus
Column-I Column-II (C) It is used to provide inert (r) XeF4
(A) SF4 (p) Tetrahedral atmosphere for filling electrical
(B) BrF3 (q) Pyramidal bulbs
(C) BrO3– (r) Sea-saw shaped (D) Its central atom is in sp3d2 (s) Ar
(D) NH4+ (s) Bent T-shaped hybridisation
(a) A – (r), B – (q), C – (p), D – (s) (a) A – (p), B – (s), C – (p), D – (r)
(b) A – (r), B – (s), C – (q), D – (p) (b) A – (p), B – (q), C – (r), D – (s)
(c) A – (p), B – (q), C – (r), D – (s) (c) A – (q), B – (p), C – (s), D – (r)
(d) A – (p), B – (s), C – (r), D – (q) (d) A – (p), B – (r), C – (q), D – (s)
237. Match the columns
241. Match the columns.
Column - I Column - II
Column-I Column-II
(A) HClO2 (p) Contains all different
(A) XeF6 (p) sp3d3– distorted octahedral
bonds
(B) XeO3 (q) sp3d2 – square planar
(B) HClO3 (q) Contains maximum
(C) XeOF4 (r) sp3 – pyramidal
Cl = O bond
(D) XeF4 (s) sp3d2 – square pyramidal
(C) HClO (r) Contains Cl with lowest
O.S. (a) A – (p), B – (r), C – (s), D – (q)
(D) HClO4 (s) Contains three types of (b) A – (p), B – (q), C – (s), D – (r)
bonds (c) A – (s), B – (r), C – (p), D – (q)
(a) A – (s), B – (p, s), C – (p, r), D – (q, s) (d) A – (s), B – (p), C – (q), D – (s)
(b) A – (p, s), B – (s), C – (p, r), D – (q, s)
ASSERTION-REASON TYPE QUESTIONS
(c) A – (s), B – (p, r), C – (p, s), D – (q, s)
(d) A – (p, s), B – (s), C – (q, s), D – (p, r) Directions : Each of these questions contain two statements,
238. Match the columns. Assertion and Reason. Each of these questions also has four
Column - I Column - II alternative choices, only one of which is the correct answer. You
(Oxides of halogens) (Uses) have to select one of the codes (a), (b), (c) and (d) given below.
(A) O2F2 (p) in water treatment (a) Assertion is correct, reason is correct; reason is a correct
(B) ClO2 (q) in estimation of CO explanation for assertion.
(C) I2O5 (r) for removing plutonium (b) Assertion is correct, reason is correct; reason is not a
from spent nuclear fuel. correct explanation for assertion
(a) A – (q) , B – (p), C – (r) (c) Assertion is correct, reason is incorrect
(b) A – (r), B – (p), C – (q) (d) Assertion is incorrect, reason is correct.
(c) A – (p), B – (r), C – (q) 242. Assertion : Dinitrogen is inert at room temperature.
(d) A – (r), B – (q), C – (p) Reason : Dinitrogen directly combines with lithium to form
239. Match the columns ionic nitrides.
Column - I Column - II 243. Assertion : N2 is less reactive than P4.
(A) XeF4 (p) Contains similar types Reason : Nitrogen has more electron gain enthalpy than
of bonds phosphorus.
(B) XeOF4 (q) Contains maximum lone 244. Assertion : When a metal is treated with conc. HNO3 it
pair generally yields a nitrate, NO2 and H2O.
(C) XeF2 (r) Square pyramidal Reason : Conc. HNO3 reacts with metal and first produces
geometry a metal nitrate and nascent hydrogen. The nascent hydrogen
(D) XeO3 (s) Contains one lone pair then further reduces HNO3 to NO2.
p-BLOCK ELEMENTS (GROUP 15, 16, 17 AND 18) 357
245. Assertion : White phosphorus is more reactive than red 256. What is the change observed when AgCl reacts with NH3?
phosphorus. (a) White ppt is formed
Reason : Red phosphorus consists of P4 tetrahedral units (b) Solution become colourless
linked to one another to form linear chains. (c) Yellow ppt is formed
246. Assertion : Bond angle of H2S is smaller than H2O. (d) No change is observed
Reason : Electronegativity of the central atom increases, 257. In which of the following equations the product formed
bond angle decreases. has similar oxidation state for nitrogen?
247. Assertion : Both rhombic and monoclinic sulphur exist as
(i) NH4NO3 N2O + 2H2O
S8 but oxygen exists as O2.
Reason : Oxfygen forms p – p multiple bond due to small (ii) 2Pb(NO3)2 673K 4NO2 + 2PbO + O2
size and small bond length but p – p bonding is not
possible in sulphur. (iii) 4HNO3 + P4O10 4HPO3 + 2N2O5
248. Assertion : SF6 cannot be hydrolysed but SF4 can be.
Reason : Six F atoms in SF6 prevent the attack of H2O on Cool
(iv) 2NO2 N2O4
sulphur atom of SF6. Heat
(a) (i) and (iii) (b) (ii) and (iv)
CRITICAL THINKING TYPE QUESTIONS
(c) (i) and (v) (d) (iii) and (iv)
249. In nitrogen family, the H-M-H bond angle in the hydrides 258. What is Z in following reaction
gradually becomes closer to 90º on going from N to Sb. This CuSO4 + Z Cu3P2 +H2SO4
shows that gradually HgCl2 + Z Hg3P2 +HCl
(a) The basic strength of the hydrides increases (a) White phosphorus (b) Red phosphorus
(b) Almost pure p-orbitals are used for M-H bonding (c) Phosphine (d) Orthophosphoric acid
(c) The bond energies of M-H bonds increase 259. Electronegativity of oxygen is more than sulphur yet H2S is
(d) The bond pairs of electrons become nearer to the acidic while water is neutral. This is because
central atom (a) water is highly associated compound
250. Bond dissociation enthalpy of E—H (E = element) bonds is (b) molecular mass of H2S is more than H2O
given below. Which of the compounds will act as strongest (c) H2S is gas while H2O is a liquid
reducing agent? (d) H–S bond is weaker than H–O bond
Compound NH3 PH3 AsH3 SbH3 260. It is possible to obtain oxygen from air by fractional
(E—H)/kJ mol–1 389 322 297 255 distillation because
diss
(a) NH3 (b) PH3 (a) oxygen is in a different group of the periodic table
(c) AsH3 (d) SbH3 from nitrogen
251. The deep blue colour produced on adding excess of (b) oxygen is more reactive than nitrogen
ammonia to copper sulphate is due to presence of (c) oxygen has higher b.p. than nitrogen
(d) oxygen has a lower density than nitrogen.
(a) Cu 2 (b) Cu ( NH 3 ) 24 261. Which of the following structures is the most preferred and
(c) Cu ( NH 3 ) 26 (d) Cu( NH3 ) 22 hence of lowest energy for SO3 ?

252. Blue solid which is obtained on reacting equimolar amounts O


of two gases at is? S
S
(a) N2O (b) N2O3 (a) (b) O O
O O
(c) N2O4 (d) N2O5
253. Concentrated nitric acid, upon long standing, turns yellow
brown due to the formation of O O
(a) NO (b) NO2 S S
(c) N2O (d) N2O4 (c) (d)
254. In the reaction O O O O O
4HNO 3 P4 O10 4HPO 3 X , the product X is 262. Which one of the following arrangements does not give
(a) N2O5 (b) N2O3 the correct picture of the trends indicated against it ?
(c) NO2 (d) H2O (i) F2 > Cl2 > Br2 > I2 : Oxidizing power
255. Ammonia on catalytic oxidation gives an oxide from which (ii) F2 > Cl2 > Br2 > I2 : Electron gain enthalpy
nitric acid is obtained. The oxide is : (iii) F2 > Cl2 > Br2 > I2 : Bond dissociation energy
(iv) F2 > Cl2 > Br2 > I2 : Electronegativity.
(a) N 2 O3 (b) NO
(a) (ii) and (iv) (b) (i) and (iii)
(c) NO 2 (d) N 2 O5 (c) (ii) and (iii) (d) (ii), (iii) and (iv)
EBD_7207
358 THE p-BLOCK ELEMENTS (GROUP 15, 16, 17 AND 18)
263. The correct order of the thermal stability of hydrogen halides 268. Which pair gives Cl 2 at room temperature :
(H–X) is
(a) HI > HCl < HF > HBr (b) HCl< HF > HBr < HI (a) NaCl + Conc. H 2SO 4 (b) Conc. HCl + KMnO4
(c) HF > HCl > HBr > HI (d) HI < HBr > HCl < HF
264. In the case of alkali metals, the covalent character decreases (c) NaCl + Conc. HNO 3 (d) NaCl + MnO2
in the order: 269. The elements which occupy the peaks of ionisation energy
(a) MF > MCl > MBr > MI (b) MF > MCl > MI > MBr curve are
(c) MI > MBr > MCl > MF (d) MCl > MI > MBr > MF
(a) Na, K, Rb, Cs (b) Na, Mg, Cl, I
265. Which of the following order is/are incorrect regarding the
property indicated against it? (c) Cl, Br, I, F (d) He, Ne, Ar, Kr
(i) HF > HI > HBr > HCl : Thermal stability 270. End-product of the hydrolysis of XeF6 is
(ii) Cl2O7 > Cl2O6 > ClO2 > Cl2O : Acidic character (a) XeF4O (b) XeF2O2
(iii) SbCl3 > SbCl5 : Covalent character (c) XeO3 (d) XeO3 –
(iv) MCl > MBr : Ionic character + –
271. The formation of O2 [PtF6] is the basis for the formation of
(a) (iii) only (b) (ii) only
xenon fluorides. This is because
(c) (i) and (iii) (d) (ii) and (iv)
266. What is X and Y in the given reactions ? (a) O2 and Xe have comparable sizes
2X2 (g) + 2H2O (l) 4H+ (aq) + 4X– (aq) + O2 (g) (b) both O2 and Xe are gases
Y2 (g) + H2O (l) HY(aq) + HOY(aq) (c) O2 and Xe have comparable ionisation energies
(a) X = Cl , Y = F (b) X = Cl , Y = Br (d) Both (a) and (c)
(c) X = F , Y = Cl (d) X = I , Y = F
272. What are the products formed in the reaction of xenon
267. Which of the following is correct about the reaction?
hexafluoride with silicon dioxide ?
heat
3NaClO NaClO3 2NaCl (a) XeSiO4 + HF (b) XeF2 + SiF4
(a) It is disproportionation reaction (c) XeOF4 + SiF4 (d) XeO3 + SiF2
(b) Oxidation number of Cl decreases as well as increases 273. XeO4 molecule is tetrahedral having :
in this reaction
(a) Two p – d bonds (b) One p – d bonds
(c) This reaction is used for the manufacture of halates
(d) All of these (c) Four p – d bonds (d) Three p – d bonds
p-BLOCK ELEMENTS (GROUP 15, 16, 17 AND 18) 359

FACT / DEFINITION TYPE QUESTIONS 20. (d) Bi forms basic oxides whereas N and P form acidic and
As and Sb form amphoteric oxides.
1. (d) Ionic radii increases down the group 21. (b) The basic character decreases from NH3 to BiH3. The
2. (b) In case of nitrogen, d-orbitals are not available. basic nature is due to the presence of lone pair of
3. (a) Collectively these elements are called pnicogens and electrons on the central atom. NH3 is the strongest
their compound pniconides. electron pair donor due to its small size as the electron
4. (d) Metallic character increases down the group, Bi is density of the electron pair is concentrated over a small
most metallic region. As the size increases the electron density gets
5. (b) The melting point in group 15 increases upto arsenic diffused over a large region and hence the ability to
and then decreases upto bismuth. donate the electron pair (basic nature) decreases.
6. (d) Bismuth forms metallic bonds in elemental state. 22. (a) NCl5 in not possible because N does not contain
7. (a) –3, +3, +5 d-orbitals.
8. (d) N2 molecule contains triple bond between N atoms
Only nitrogen has a tendency to form p – p multiple
having very high dissociation energy (946 kJ mol–1)
due to which it is relatively inactive. bonds. Other forms d – p multiple bonds easily..
9. (a) Nitrogen due to small size is able to show p -p lateral 23. (c) 24. (b)
overlap forming N N, rest elements due to bigger size
Heat
are not able to show p -p lateral overlap. 25. (d) NH 4 Cl NaNO 2 NH 4 NO 2
NaCl
10. (c) Catenation tendency is higher in phosphorus when
Heat
compared with other elements of same group. N2 2H 2O.
11. (c) Nitrogen form N2 (i.e. N N) but phosphorus form 26. (a)
P4, because in P2, p — p bonding is present which
is a weaker bonding. 27. (b) In Haber’s process for manufacture of NH3, finely
12. (d) The cause of inert nature of N2 is the presence of divided iron is used as catalyst and molybdenum is
.. .. used as catalytic promoter
triple bond N N Fe Mo
13. (b) N2 ( g ) 3H2 ( g ) 800K, High P
2NH 3 ( g )
14. (b) Phosphorous can achieve coordination number 5 due
28. (d) N2H4 and NH4Cl are obtained by reaction of ammonia
to vacant d atomic orbitals in valence shell which is
with hypochlorite anion.
not possible in nitrogen
15. (b) The order of boiling points of the group 15 hydrides 3NH 3 NaOCl N 2 H 4 NH 4 Cl NaOH
is : BiH3 > SbH3 > NH3 > AsH3 > PH3 29. (a) HNO3 and CuSO4 are not drying agents, while P2O5
16. (a) Oxide in which central atom has higher charge and reacts with NH3. The moisture present in NH3 is
more electronegativity is more acidic, i.e. removed by passing it through a tower packed with
N2O5 > N2O4 > P2O5 > As2O3. quicklime (CaO).
17. (a) Order of dipole moment 30. (b) Ammonia has pyramidal shape with sp3 hybridisation.
NH3 > PH3 > AsH3 > SbH3
31. (b) 3CuO 2 NH 3 3Cu 3H 2 O N 2 ,
(Based upon electronegativity)
18. (c) As the size of central atom increases the lone pair of O.S. of N in NH3 is –3 and in N2 is zero. Hence loss of
electrons occupies a larger volume. In other words 3 electrons
electron density on the central atom decreases and 32. (b) NH3 is not used as anaesthetic
consequently its tendency to donate a pair of electrons 33. (b) Liquid ammonia has high vapour pressure which is
decreases along with basic character from NH3 to
lowered down by cooling, otherwise the liquid will
BiH3.
bump.
19. (a) NF 5 does not exist because N does not form
pentahalides due to the absence of d-orbital in its 34. (c) By Haber’s process
valence shell. While P, As and Sb form pentahalides of 35. (a) Only nitrates of heavy metals and lithium decompose
the general formula MX5 (where, M = P, As and Sb) on heating to produce NO2.
due to the presence of vacant d-orbitals in their 36. (d) N2O3, N2O4 and N2O5 are acidic oxides. Only N2O is
respective valence shell. neutral oxide.
EBD_7207
360 THE p-BLOCK ELEMENTS (GROUP 15, 16, 17 AND 18)
56. (b) White phosphorous is most reactive
O O
57. (b) White phosphorous is P4 and tetrahedral
37. (b) N–O–N 58. (a) Except (a) all other properties are shown by white
phosphorous.
O O 59. (b) White phosphorus on heating readily catches fire in
The structure clearly shows the presence of covalent air to give dense white fumes of P4O10.
and co-ordinate bonds. P4 + 5O2 P4O10.
38. (d) NO2 is reddish brown coloured gas. Rest of the oxides
are colourless. 60. (c) Monoclinic sulphur is stable above 369 K.
39. (c) Compound O.S. of N 61. (c) Ca3P2 + 6H2O 3Ca(OH)2 + 2PH3 ; i.e 2 moles of
N2O +1 phosphine are produced from one mole of calcium
phosphide.
NO +2
NO2 +4 62. (d) PH3 is covalent hydride
NO3– +5 63. (b) Red P does not react with NaOH to give PH3.
NH4+ –3 64. (d) PH3 is not obtained when metaphosphoric acid is
Therefore increasing order of oxidation state of N is: heated.
65. (a) The combustibility of PH3 is due to presence of P2H4.
NH 4 < N2O < NO < NO2 < NO3 . The pure PH3 is not combustible.
40. (c) In N2O (nitrous oxide) two N atoms are covalently 600 C
bonded through triple bond 66. (d) 2H 3 PO 4 2 HPO 3
2H 2O
[N N O]
41. (b) FeSO 4 NO FeSO 4 .NO 67. (d) P2 O 5 3H 2 O 2 H 3 PO 4
42. (b) 68. (b) Orthophosphoric acid, H 3PO 4 contains three P – OH
43. (a) N2O is used as anaesthetic bonds and is therefore, tribasic.
44. (c) 2NO O 2 2 NO 2 brown O
||
45. (b) Phosphorus from stable P4 molecule.
P
| OH

|
Pt. gauge
46. (a) 4NH 3 5O 2 4NO 6H 2 O OH
47. (b) The slow decomposition of HNO3 is represented by orthophosphoric acid
the eqn. 69. (b) PCl3 + H2O POCl3 + 2HCl
4HNO3 4NO2 + 2H2O + O2 POCl3 + 3H2O H3PO4 + 3HCl
(yellow-brown)
70. (c) H 3PO 2 is named as hypophosphorous acid. It is
48. (d) For nitrogen, only NF3 is known to be stable.
monobasic as it contains only one P – OH bond, its
49. (a) 8NH3 3Cl2 6NH 4Cl N 2
basicity is one.
(excess)
NH3 3Cl 2 NCl3 3HCl O
||
excess
P
H | OH
|

50. (c) BiH3 is the strongest reducing agent while NH3 is the
weakest reducing agent. H
51. (d) The oxides of the type E 2 O 3 of nitrogen and 71. (a) We know that empirical formula of hypophosphorus
phosphorus are purely acidic. acid is H3PO2. In this only one ionisable hydrogen
52. (d) NH3 is not used in the pickling of stainless steel. atom is present i.e. it is monobasic. Therefore option
O (a) is correct structural formula of it.
O
=

53. (d) N O N–N = O


=

(i) (ii) O ||
O O P
O O
=

N–N N–O–N O
O
=

O
=

O O O 72. (d)
(iii) (iv) O
O P P O
||

54. (d) 4 Zn + 10 HNO3 (dil.) 4 Zn(NO3)2 + 5H2O + N2O


||

Zn + 4 HNO3 (conc.) Zn(NO3)2 + 2H2O + 2NO2 O


O P
55. (a) Both white and red phosphorus are not soluble in CS2
|| O
only white phosphorus is soluble in CS2.
O
p-BLOCK ELEMENTS (GROUP 15, 16, 17 AND 18) 361
73. (c) Structure of hypophosphorous acid O
O
H
| P
H O P O (b) (c) HO P O
| HO H
HPO3
H H
H3PO2
Two H-atoms are attached to P atom.
74. (c) In cyclic metaphosporic acid number of P–O–P bonds O O
is three.
O OH (d) P (e) P
P HO OH H OH
O O
O OH OH
O
P P H 3PO 4 H3PO3
O OH
HO The H–atom of the –OH group is ionisable whereas
75. (d) H–atom which is directly linked to P–atom is non-
76. (b) O ionisable. Thus H3PO3 is dibasic acid.
85. (a) Pyrophosphorous acid (H4P2O5) is a dibasic acid as it
bridging P bridging contains two P—OH bonds.
O
O O
O
O
H—P—O—P—H
P O
OH OH
O P
O 86. (b) Formula of cyclotrimetaphosphoric acid is (HPO3)3
O P O Oxidation state of ‘P’ is 3(+ 1 + x + 3 (– 2)) = 0
bridging
O bridging x+ –6+1 =0 x= +5
oxygen
87. (a) O O

O HO — P — P — OH
HO OH
77. (b) H P H Hypophosphorous acid (H3PO2) is a Hypophosphoric acid
O
OH OH
H

monobasic acid. i.e., it has only one ionisable hydrogen O P O P O


atom or one OH is present. OH OH
1 Pyrophosphoric acid
78. (c) Hybridisation in PCl5 = 5 5 0 0 5 sp3d
2
HO O OH
79. (c) H5P5O15 (HPO3)5. It is metaphosphoric acid which is P P
a cyclic phosphate. O O O
80. (b) H4P2O5 is pyrophosphorous acid it contains P–O–P Metaphosphoric acid
bond
O
81. (c) H3PO4 is tribasic
82. (a) Hypophosphorous acid is H3PO2 in which O.S. of P HO P OH
is +1
83. (c) OH
84. (d) Structures of given oxyacids are following Orthophosphoric acid
88. (a) 89. (c) 90. (d)
O O
91. (a) Electron affinity increases from left to right in period
and decreases from top to bottom in a group but
P P
electron affinity of O is less than S due to small size.
(a) HO O OH
OH 92. (d) All exhibit polymorphism
OH
H4P2O7
93. (a)
EBD_7207
362 THE p-BLOCK ELEMENTS (GROUP 15, 16, 17 AND 18)
94. (d) H2O is liquid but H2S is a gas. This can be attributed 120. (c) SO 2 2H 2 O H 2SO 4 2H . Bleaching action is
to the presence of intermolecular hydrogen bonding
due to reduction.
in case of H2O.
95. (b) 121. (d) Caro’s acid is H 2SO 5 which contains one S – O
96. (a) – O – H peroxy linkage. It is also known as
97. (a) H2O (due to intermolecular H - bonding) permonosulphuric acids.
98. (b) Oxygen being more electronegative
99. (a) SnO2 is an amphoteric oxide because it reacts with O
||
acids as well as with bases to form corresponding salts. H – O – O – S – OH
SnO2 + 2H2SO4(conc) Sn(SO4)2 + 2H2O ||
SnO2 + 2NaOH Na2SnO3 + H2O O
100. (d) All hexafluorides of group 16 elements are gaseous in Caro's acid
nature. 122. (d)
101. (b) Oxygen can be prepared by heating oxides of Hg, Pb,
Ag, Mn and Ba. 123. (c) Oleum is H 2S2 O 7 ( H 2SO4 SO3 ) which is obtained
by dissolving SO3 in H2SO4 and is called fuming
2HgO 2Hg + O2 sulphuric acid.
102. (c) It is paramagnetic with two unpaired electrons 124. (c) 125. (a) 126. (c)
103. (c) Total number of electrons in O2 is 16. It has 2 unpaired 127. (c) It is H2SO5.
electrons, the rest 14 are paired.
128. (c) HO.SO 2OH 2PCl5 ClSO2Cl 2POCl3 2HCl
104. (b) 2KClO 3 2 KCl 3O 2 Sulphuryl chloride
105. (c) In KMnO4 manganese is already present in its highest 129. (b) 2Ag 2H 2SO 4 2H 2 O SO 2 Ag 2SO 4 .
possible oxidation state i.e. +7.So no further oxidation
Au, Pt does not react. Pb forms insoluble PbSO4
is possible.
130. (c)
106. (c) Ozone layer is beneficial to us, because it stops
harmful ultraviolet radiations from reaching the earth. O
||
107. (b) 2KMnO 4 K 2 MnO4 4MnO 2 O 2 131. (a) H O S O H; 6 &2
||
108. (d) Ozone is an allotrope of oxygen. O
109. (d) 2Ag2O (s) 4Ag (s) + O2 (g)
2Pb3O4 (s) 6PbO (s) + O2 (g) 132. (a) In H 2SO 4 , the S atom is present in its highest
2PbO2 (s) 2PbO (s) + O2 (g) oxidation state of +6. Hence H 2SO 4 can act an oxidant
110. (a) Mn2O7 is an acidic oxide. BaO and Na2O are basic only by gain of electrons
oxides while N2O is a neutral oxides. 133. (d) Ionisation potential decreases down the group.
111. (c) It is 8 134. (a)
112. (c) S2 is paramagnetic. It contains two unpaired electrons 135. (d) The lesser the bond energy, the weaker is the bond
in the antibonding * orbital 136. (b) 3s2 3p5 is electronic configuration of Cl
113. (a) 2SO2(g) + O2(g) V2O5
2SO3(g) 137. (c) The electron gain enthalpy order for halogens is
Cl > F > Br > I
O O Due to small size of fluorine the extra electron to be
=

114. (b) O = S–O–O–S=O added feels more electron-electron repulsion.


OH OH Therefore fluorine has less value for electron affinity
than chlorine.
Peroxodisulphuric acid
138. (d)
(H2S2O8)
139. (a) Reactivity follows the order F > Cl > Br > I
115. (b) Conc. H2SO4 is a strong dehydrating agent due to
140. (d)
which carbohydrates becomes charred on reaction with
141. (b) Except ionisation potential other factors are true to
conc. H2SO4 acid.
116. (b) The key step in the manufacture of H2SO4 is catalystic explain the oxidising (strong) behaviour of F2 .
oxidation of SO2 with O2 to give SO3 in presence of 142. (b)
V2O5 . 143. (d) Fluorine exhibit -ve oxidation state
117. (a) In SO3, sp2 hybridisation 144. (a) Since F2 is most oxidising, it is easily reduced
118. (c) Cu 2H 2SO 4 (conc) CuSO 4 SO 2 2H 2O 145. (c) Chlorine shows O.S. from –1,+1 to +7, whereas others
show O.S. as Na +1 ; K +1 ; F –1
119. (b) 2 , one see structure
p-BLOCK ELEMENTS (GROUP 15, 16, 17 AND 18) 363
146. (c) Fluorine always exhibit –1 oxidation state. 0 1 +5
0 1 1
166. (b) 6NaOH + 3Cl2 5NaCl + NaCl O3 + 3H 2O
147. (b) H 2O Br2 HOBr HBr 167. (c) 6NaOH + 3Cl2 5NaCl + NaClO3 + 3H2O
Thus here oxidation number of Br increases from 0 to (hot and conc.)
+1 and also decreases from 0 to –1. Thus it is oxidised 168. (a) Rn because it is radioactive element obtained by the
as well as reduced. disintegration of radium
148. (d) Since all the halogens have a strong tendency to 226
88 Ra Rn 222 2 He4
86
accept electrons. Therefore halogens act as strong
oxidising agents and their oxidising power decreases 169. (b) Radon is the last member of family
from fluorine to iodine. 170. (d)
149. (b) On moving from top to bottom of halogen group the 171. (d) Electronic configuration of He is 1s2
bond dissociation energy of hydrogen h alides 172. (a) Inert gases do not contain unpaired electrons
decreases and so the heat of formation of halogen 173. (b) Ionization potential of inert gases is highest in periodic
acids also decreases. table due to stable electronic configuration.
150. (a) HF, due to intermolecular H-bonding is weakest among 174. (a) Ionisation energy decreases as we move away from
HX acids nucleus due to less electrostatic attraction between
151. (c) Volatile character HCl > HBr >HI > HF electrons and nucleus
152. (d) Due to hydrogen bonding HF is a liquid 175. (a) The smaller the size the least is the polarisability
153. (d) Bleaching action of chlorine is due to oxidation in 176. (c) The differentiating electron enter in s subshell in case
presence of moisture. of He, hence it is s- block element. Its electronic
Cl 2 H 2O HCl HClO configuration l s2 makes it inert in nature hence it is
HClO HCl O placed with inert gases.
Colouring matter + | O | Colourless matter 177. (c) Ar is the most abundant in atmosphere
154. (b) Cl2 + 2NaOH NaCl + NaClO + H2O 178. (d) Due to weak van der Waal’s forces, He has lowest
(cold & dil) boiling point
3Cl2 + 6NaOH 5NaCl + NaClO3 + 3H2O 179. (d) The larger the size the more is the polarisiability
(hot & conc.)
180. (d) He was observed in the spectrum of the sun
155. (a) 2NaOH Cl2 NaCl NaOCl H 2 O
226
181. (b) 88 Ra 86 Rn 222 2 He 4 . Both are inert gases
hence Cl and OCl
182. (d) Electron gain enthalpy for noble gases is positive and
156. (d) K 2 Cr2O 7 conc.HCl Cl2 it becomes less positive with increase in size of atom.
157. (d) MnO2 or KMnO4 with conc HCl give Cl2. Value of electron gain enthalpy
He – 48 kJ mol–1, Ne – 116 kJ mol–1
7 2
158. (d) 2 KMnO 4 16HCl 2 MnCl 2 2 KCl 8H 2 O 5Cl 2 Ar, Kr – 96 kJ mol–1, Xe – 77 kJ mol–1
O.S of Mn changes from +7 to +2 hence reduction Hence, Ne has highest positive electron gain enthalpy.
occurs and Cl2 is formed. 183. (b)
184. (a) As size increases, van der Waal's forces of attraction
159. (d) 6CaOCl 2 Ca (ClO3 ) 2 5CaCl 2
between n oble gas atoms also increases.
It is autooxidation. Consequently, ease of their liquefaction increases.
160. (d) HCl acid at 25º C is a gas and polar in nature 185. (c) Solubility increases from He to Rn
161. (d) In gaseous state the HCl is covalent in nature while in
186. (d) Xe forms maximum compounds hence it is most
aqueous solution it ionises to give H+ and C l ions reactive
162. (d) As the oxidation state of the central halogen atom 187. (b) Noble gases exhibit low chemical activity
increases, the halogen-oxygen bond becomes more 188. (d)
and more covalent. As a result the thermal stability of 189. (b) In XeOF4, Xenon is sp3d 2 hybridised and has one
the oxoacid increases. Thus, HClO4 is most stable to
heat, whereas HClO is least stable to heat. lone pair of electrons.
163. (c) The bond energy of interhalogen compounds is less 190. (a)
than the bond energy of halogens. 191. (d) The products of the concerned reaction react each
164. (d) Interhalogen compounds are not highly volatile other forming back the reactants.
1 XeF6 3H 2 O XeO3 6HF .
165. (a) ICl7. The hybridisation is 7 7 0 0 7 (sp3d3 )
2
EBD_7207
364 THE p-BLOCK ELEMENTS (GROUP 15, 16, 17 AND 18)
198. (d) Hybridisation in
F 1
XeF4 (8 4 0 0) 6 sp3d 2
2
199. (b) XeOF4 square pyramidal
200. (c) Hybridisation of XeF2 is sp3d
192. (c) XeF2 : Xe 201. (b) XeF4 is planar
202. (a) In XeO3 the hybridisation is sp3
F
F 203. (a) XeF2 has Xe structure hence number of lone
3lp
F
pair of electrons 3
F F 204. (c) Hybridisation of XeF4 is sp3d2 and structure is square
planar
XeF4 : 205. (a) XeF2 XeF4 XeF6
Xe
Valence electrons of Xe 8 8 8
F F Electrons involved 2 4 6
in bond formation
2lp Lone pairs left 3 2 1
206. (c) Hybridisation in each case is XeF4sp3d2, XeF2sp3d,
207. (c) He is obtained during radioactive decay
F
208. (c) The geometry of XeF6 is distorted octahedral in which
F all the six positions are occupied by fluorine atoms
and the lone pair of electrons of Xe atom is present at
XeF6 : F Xe F the corner of one of the triangular faces.
F
F F
F F
Xe

F F
F
209. (b) The hybridization of XeO3F2 is sp3d and its structure
XeO3 : Xe is trigonal bipyramidal in which oxygen atoms are
situated on the plane and the fluoride atoms are on the
top and bottom.
O O
O F
O
1 lp

Hence XeF2 has maximum no. of lone pairs of electrons.


O Xe
193. (d) XeO2F2 has trigonal bipyramidal geometry, but due to
presence of lone pair of electrons on equitorial position,
its actual shape is see-saw. O
F
F 210. (b) The shape of XeO3 is Trigonal Pyramidal.
O
Xe
Xe (Trigonal Pyramidal Structure)
O O O
F O

194. (b) XeF4 H 2O 2HF XeOF2 211. (c) Argon is used in high temperature welding and other
operations which require a non-oxidising atmosphere
195. (a) No compound of He as yet been reported and the absence of nitrogen.
196. (a) No compound of Ar as yet been reported with F2 212. (b) Neon gives a distinct reddish glow when used in either
197. (c) XeF6 3H 2 O 6HF XeO 3 low-voltage neon glow lamps or in high voltage
discharge tube.
p-BLOCK ELEMENTS (GROUP 15, 16, 17 AND 18) 365
213. (c) Helium is twice as heavy as hydrogen it is inflammable MATCHING TYPE QUESTIONS
but not lighter than hydrogen. Helium has the lowest
melting and boiling point of any element which makes 230. (b) 231. (a) 232. (a) 233. (c) 234. (a)
liquid helium an ideal coolant for many extremely low 235. (b) 236. (b) 237. (b) 238. (b) 239. (a)
temperature application such as super conducting 240. (c) 241. (a)
magnet and cryogenic research where temperature
close to absolute zero are needed. He is used in gas ASSERTION-REASON TYPE QUESTIONS
cooled atomic reactors as a heat transfer agent.
242. (c) At higher temperatures, dinitrogen combines with
214. (c) Coloured discharge tubes mainly contain Neon metals to form ionic nitrides.
215. (a) Breathing mixture is (O2 + He) 243. (c)
216. (a) Mixture of (He + O2) is used for asthma patient 244. (a) Both assertion and reason are true and reason is the
217. ( b) 218. (c) 219. (d) correct explanation of assertion.

STATEMENT TYPE QUESTIONS M HNO3 MNO3 H


(metal) (conc.) (metal nitrate) (nascent hydrogen)

220. (a) Phosphorus occurs in minerals of the apatite family,


2HNO3 2H 2NO2 2H 2 O
Ca9(PO4)6, CaX2 (X = F, Cl or OH) which are main (nascent hydrogen)
components of phosphate rocks whereas arsenic and 245. (b) White phosphorus exists as P4 tetrahedral molecule
antimony are found as sulphide minerals. The increase having P-P-P bond angle 60º. Hence the molecule is
in covalent radii from N to P is greater in comparison under strain and more reactive. On the other hand red
to increase from As to Bi. phosphorus exists as P4 tetrahedra which are joined
221. (c) For nitrogen oxidation states from +1 to +4 together through covalent bonds giving polymeric
disproportionate in acidic solution only. Oxidation structure.
state of phosphorous in P2O5 is + 5 whereas in P2O5 246. (c) Bond angle of H2 S (92°) < H2O (104°31). As the
is +3 thus P2O5 is more acidic than P2O3. electronegativity of the central atom decreases, bond
222. (b) angle decreases. In the present case, S is less
electronegative than oxygen. Thus bond pairs in H2S
223. (a) Oxygen shows oxidation state of +2 in OF2. H2O which
are more away from the central atom than in H2O and
is a hydride of oxygen element of group 16 is neutral
thus repulsive forces between bond pairs are smaller
in nature.
producing smaller bond angle.
224. (c) 247. (a) 248. (a)
O O
CRITICAL THINKING TYPE QUESTIONS
O O– –O O
249. (b) With the decrease in the electronegativity of central
Ozone is diamagnetic in nature (due to presence of
paired electron) and both the O – O bond length are atom the bond angle decreases
equal. It has a bent structure. 250. (d)
251. (b) CuSO 4 4NH 3 [Cu ( NH 3 ) 4 ] SO 4
225. (b) NH3 being basic reacts with acidic H2SO4 thus H2SO4
cannot be used for drying NH3. Blue complex due to Cu(NH 3 ) 2
226. (b) Physical state of iodine is different from other halogens 4
as iodine is solid, bromine is a liquid whereas fluorine 252. (b) 2NO + N2O4 250K 2N2O3
and chlorine are gases. 253. (b) The slow decomposition of HNO3 is represented by
227. (b) At 298K, ClF exits as a gas. the eqn.
228. (d) 4HNO3 4NO2 + 2H2O + O2
229. (c) For statement (iii) preparation of XeF2 requires Xe in (yellow-brown)
excess amount 254. (a)
255. (c) [Fe(H2O)5NO]2+ ion is formed
673K,1 bar 256. (b) Ag+ (aq) + Cl– (aq) AgCl(s)
Xe g F2 g XeF2 s
excess Colourless White ppt
AgCl (s) + 2NH3 (aq) [Ag(NH3 )2] Cl (aq)
For statements (iv)
White ppt Colourless
2XeF2(s) + 2H2O(l) 2Xe(g) + 4HF(aq) + O2(g)
257. (b) NO2 and N2O4 has + 4 oxidation state for nitrogen.
6XeF4 + 12H2O 4Xe + XeO3 + 24HF + 3O2 258. (c) 3CuSO4 + 2PH3 Cu3P2 + 3H2SO4
XeF6 + 3H2O XeO3 + 6HF 3HgCl2 + 2PH3 Hg3P2 + 6HCl
EBD_7207
366 THE p-BLOCK ELEMENTS (GROUP 15, 16, 17 AND 18)
259. (d) SH–bond is weaker than, O–H bond. Hence H2S will 266. (c) 2F2 (g) + 2H2O (l) 4H+ (aq) + 4F– (aq) + O2 (g)
furnish more H+ ions Cl2 (g) + 2H2O (l) HCl (aq) + HOCl
260. (c) Air is liquified by making use of the joule-Thompson 1 5 1
effect (cooling by expansion of the gas) Water vapour 267. (d) NaClO NaClO3 2NaCl All statements are
and CO2 are removed by solidification. The remaining correct as evident from the reaction
major constituents of liquid air i.e., liquid oxygen and
liquid nitrogen are separated by means of fractional 268. (b) 2KMnO 4 16HCl 2KCl 2MnCl 2 8H 2 O 5Cl 2
distillation (b.p. of O2 = –183°C : b. P. of N2 = – 195.8°C) 269. (d)
261. (d) Formal charges help in the selection of the lowest 270. (c) XeF6 + 3H2O XeO3 + 6HF
energy structure from a number of possible Lewis
Complete hydrolysis of XeF6 gives XeO3 (an
structures for a given compound. The lowest energy
explosive).
structure means the structure with the smallest formal
271. (d) (i) The first ionization energy of xenon (1, 170 kJ mol–
charge on each atom of the compound. A Lewis dot 1) is quite close to that of dioxygen (1,180 kJ mol–1).
structure is preferable when all formal charges are zero.
262. (c) From the given options we find option (a) is correct. The (ii) The molecular diameters of xenon and dioxygen are
oxidising power of halogens follow the order almost identical.
F2 > Cl2 > Br2 > I2. Option (b) is incorrect because it in Based on the above similarities Barlett (who prepared
not the correct order of electron gain enthalpy of O2+[PtF6]– compound) suggested that since oxygen
halogens. combines with PtF6, so xenon should also form similar
The correct order is Cl2 > F2 > Br 2 > I2. The low value compound with PtF6.
of F2 than Cl2 is due to its small size. 272. (c) 2XeF6 SiO 2 SiF4 2XeOF4
Option (c) is incorrect. The correct order of bond
273. (c) Xenon undergo sp3 hybridization.
dissociation energies of halogens is
Cl2 > Br2 > F2 > I2. (ground
Option (d) is correct. It is the correct order of state)
5s 5p 5d
electronegativity values of halogens. Thus option (b)
and (c) are incorrect. (third excited
263. (c) The H–X bond strength decreases from HF to HI. i.e. state)
HF > HCl > HBr > HI. Thus HF is most stable while HI 5s 5p 5d
is least stable. The decreasing stability of the hydrogen In the fourth excited state xenon atom, has 8 unpaired
halide is also reflected in the values of dissociation electrons
energy of the H–X bond
H F H Cl H Br H I
135kcal mol 103kcal mol 87 kcal mol 71kcal mol 5s 5p 5d
264. (c) MI > MBr > MCl > MF. As the size of the anion 3
One s and three p orbital undergo sp hybridization.
decreases covalent character also decreases. Four sp3 hybrid orbitals form four bonds with oxygen
265. (a) Metal halides with higher oxidation state are more atoms. They are sp3 – p. Four p – d bonds are also
covalent than the one in lower oxidation state. formed with oxygen atoms by the unpaired electrons.
22
THE d-AND f-BLOCK ELEMENTS

FACT / DEFINITION TYPE QUESTIONS 9. An atom has electronic configuration


1s 2 2s 2 2 p 6 3s 2 3 p6 3d 3 4s 2 in which group would it be
1. The transition elements have a general electronic
configuration placed?
(a) Fifth (b) Fifteenth
(a) ns 2, np 6, nd 1 10
(c) Second (d) Third
(b) ( n 1) d 1 10
, ns 0 2
, np 0 6
10. In 3d-series atomic number (Z) varies from
(c) (n 1) d1 10, ns1 2 (a) Z 21 30 (b) Z 22 30
(d) n d1 10, ns1 2 (c) Z 20 30 (d) Z 31 40
2. Correct electronic configuration of Cr (Z = 24) is 11. The valence shell of transition elements consists of
(a) 1s2 2s2 2p6 3s2 3p6 3d 7 4s1 (a) nd orbitals (b) (n-1) d orbitals
(b) 1s2 2s2 2p6 3s2 3p6 3d 5 4s1 (c) ns np nd orbitals (d) (n-1) d ns orbitals
(c) 1s2 2s2 2p6 3s2 3p6 3d 7 4s2 12. Number of unpaired electrons in Ni2+(Z=28) is
(d) 1s2 2s2 2p6 3s2 3p6 3d 6 4s2
(a) 4 (b) 2
3. Which of the following configuration is correct for iron ?
(c) 6 (d) 8
(a) 1s2 ,2 s2 2 p6 ,3s2 3 p6 3d 4 13. Which of the following element is not a member of transition
2 2 6 2
(b) 1s , 2s 2 p ,3s 3 p 3d 4s
6 6 2 elements ?
(a) Zn (b) Pt
(c) 1s2 ,2s2 2 p6 ,3s2 3 p6 3d 2 (c) Ce (d) Mo
2 2 6 2 6 2 2
(d) 1s , 2s 2 p ,3s 3 p 3d 4s 14. The number of unpaired electrons in gaseous species of
4. Which one of the following ions has electronic Mn3+, Cr 3+ and V3+ respectively are.
configuration [Ar] 3d 6 ? (a) 4, 3 and 2 (b) 3, 3 and 2
(a) Ni3+ (b) Mn 3+ (c) 4, 3 and 2 (d) 3, 3 and 3
(c) Fe3+ (d) Co3+ 15. The first element in the 3d-transition series is
(At. Nos. Mn = 25, Fe = 26, Co = 27, Ni = 28) (a) Sc (b) Ti
5. Which of the following element does not belong to first (c) V (d) Ca
transition series? 16. Which of the following has more unpaired d-electrons?
(a) Fe (b) V
(c) Ag (d) Cu
(a) Zn (b) Fe 2
6. n 1 d 10 ns 2 is the general electronic configuration of (c) Ni (d) Cu
(a) Fe, Co, Ni (b) Cu, Ag, Au 17. The number of unpaired electrons in a nickel atom in ground
(c) Zn, Cd, Hg (d) Se, Y, La state are At. No. of Ni 28
7. The last electron in d-block elements goes to
(a) (n-1) d (b) nd (a) 2 (b) 5
(c) np (d) (n-1) s (c) 3 (d) 7
8. The elements which exhibit both vertical and horizontal 18. Which one of the following is an example of non-typical
similarites are transition elements ?
(a) inert gas elements (b) representative elements (a) Li, K, Na (b) Be, Al, Pb
(c) rare elements (d) transition elements (c) Zn, Cd, Hg (d) Ba, Ga, Sr.
EBD_7207
368 THE d-AND f-BLOCK ELEMENTS
19. Which of the following has the maximum number of unpaired 32. Iron exhibits +2 and + 3 oxidation states. Which of the
electrons? following statements about iron is incorrect ?
(a) Ti2+ (b) Fe2+ (a) Ferrous oxide is more basic in nature than the ferric
(c) Cr+ (d) Cu+ oxide.
20. The outer electronic configuration of Ag is 4d 10 5s1, it (b) Ferrous compounds are relatively more ionic than the
belongs to corresponding ferric compounds.
(a) 5th period, group 4 (b) 4th period, group 5 (c) Ferrous compounds are less volatile than the
(c) 5 period, group 11 (d) 6th period, group 9
th
corresponding ferric compounds.
21. Manganese belongs to (d) Ferrous compounds are more easily hydrolysed than
(a) 1st transition series (b) 2nd transition series the corresponding ferric compounds.
rd
(c) 3 transition series (d) 4th transition series 33. Four successive members of the first row transition
22. The no. of unpaired electrons in Mn7+ ions elements are listed below with their atomic numbers.
(At. no. of Mn = 25) is Which one of them is expected to have the highest third
(a) 0 (b) 1 ionization enthalpy?
(c) 2 (d) 3 (a) Vanadium (Z = 23) (b) Chromium (Z = 24)
23. Which one of the following species is paramagnetic? (c) Manganese (Z = 25) (d) Iron (Z = 26)
(a) N2 (b) Co 34. Of the following outer electronic configurations of atoms,
(c) Cu+ (d) Zn the highest oxidation state is achieved by which one of
24. Which of the following species is/are paramagnetic? them ?
Fe2+, Zn0, Hg2+, Ti4+ (a) (n – 1)d 3 ns2 (b) (n – 1)d 5 ns1
(a) Fe2+ only (b) Zn 0 and Ti4+ 8
(c) (n – 1)d ns 2 (d) (n – 1)d 5 ns2
2+
(c) Fe and Hg 2+ (d) Zn 0 and Hg2+ 35. For d block elements the first ionization potential is of the
25. In first transition series, the melting point of Mn is low order
because (a) Zn > Fe > Cu > Cr (b) Sc = Ti < V = Cr
(a) due to d10 configuration, metallic bonds are strong (c) Zn < Cu < Ni < Co (d) V > Cr > Mn > Fe
(b) due to d7 configuration, metallic bonds are weak 36. Which of the following does not represent the correct order
(c) due to d5 configuration, metallic bonds are weak of the properties indicated ?
(d) None of these (a) Ni2+ > Cr2+ > Fe2+ > Mn2+ (size)
26. The transition metals have a less tendency to form (b) Sc > Ti > Cr > Mn (size)
ions due to
(c) Mn2+ > Ni2+ < Co2+ <Fe2+ (unpaired electron)
(a) high ionisation energy
(d) Fe2+ > Co2+ > Ni2+ > Cu2+ (unpaired electron)
(b) low heat of hydration of ions
37. Zinc and mercury do not show variable valency like d-block
(c) high heat of sublimation
elements because
(d) All of these
27. The common oxidation states of Ti are (a) they are soft
(a) + 2 and + 3 (b) + 3 and + 4 (b) their d-shells are complete
(c) – 3 and – 4 (d) + 2, + 3 and + 4 (c) they have only two electrons in the outermost subshell
28. Maximum oxidation state is shown by (d) their d-shells are incomplete
(a) Os (b) Mn 38. Which of the following transition element shows the
(c) Co (d) Cr highest oxidation state ?
29. Which one of the elements with the following outer orbital (a) Mn (b) Fe
configurations may exhibit the largest number of oxidation (c) V (d) Cr
states? 39. Which of the following elements does not show vari able
(a) 3d 54s1 (b) 3d 54s2 oxidation states?
(c) 3d 4s 2 2 (d) 3d 34s2 (a) Copper (b) Iron
30. Which of the following pairs has the same size? (c) Zinc (d) Titanium
40. Which one of the following transition elements does not
(a) Fe 2+ , Ni 2+ (b) Zr 4+ , Ti 4+
exhibit variable oxidation state?
(c) Zr 4+ , Hf 4+ (d) Zn 2+ , Hf 4+ (a) Ni (b) Cu
31. For the four successive transition elements (Cr, Mn, Fe and (c) Fe (d) Sc
Co), the stability of +2 oxidation state will be there in which 41. Electronic configuration of a transition element X in +3
of the following order? oxidation state is [Ar]3d5. What is its atomic number ?
(a) Mn > Fe > Cr > Co (b) Fe > Mn > Co > Cr (a) 25 (b) 26
(c) Co > Mn > Fe > Cr (d) Cr > Mn > Co > Fe (c) 27 (d) 24
THE d-AND f-BLOCK ELEMENTS 369
42. Metallic radii of some transition elements are given below. 51. What is wrong about transition metals?
Which of these elements will have highest density ? (a) Diamagnetic
Element Fe Co Ni Cu (b) Paramagnetic
Metallic radii/pm 126 125 125 128 (c) Form complexes
(a) Fe (b) Ni (d) Shows variable oxidation state
(c) Co (d) Cu 52. Which of the following ions has the maximum magnetic
43. Transition metals mostly are moment?
(a) diamagnetic (a) Mn +2 (b) Fe+2
(b) paramagnetic (c) Ti 3+ (d) Cr+2.
(c) neither diamagnetic nor paramagnetic 53. Four successive members of the first row transition elements
(d) both diamagnetic and paramagnetic are listed below with atomic numbers. Which one of them is
44. Transition metals usually exhibit highest oxidation states in
their expected to have the highest E value ?
M3 / M 2
(a) chlorides (b) fluorides (a) Cr(Z = 24) (b) Mn(Z = 25)
(c) bromides (d) iodides (c) Fe(Z = 26) (d) Co(Z = 27)
45. Which of the following statements is incorrect? 54. Which one of the following ions exhibit highest magnetic
(a) Zn,Cd and Hg due to presence of completely filled moment?
d-orbitals [(n–1)d10ns2] are not studied along with
(a) Cu 2 (b) Ti3
other transition metals.
(b) Zn, Cd and Hg have low m.p and are comparitively (c) Ni 2 (d) Mn 2
softer than other transition metals.
(c) Metallic bond made by elements with d 5 configuration 55. A compound of a metal ion M x Z 24 has a spin
is stronger as compared to metalic bond made by only magnetic moment of 15 Bohr Magnetons. The
elements with d 3 configuration. number of unpaired electrons in the compound are
(d) Metals of 5d series forms strong metallic bonds as (a) 2 (b) 4
compared with metals of 3d series.
(c) 5 (d) 3
46. Which of the following is incorrect?
56. Titanium shows magnetic moment of 1.73 B.M. in its
(a) Mn shows oxidation state of +7 in MnF7 compound. What is the oxidation number of Ti in the
(b) Fe and Co shows +3 oxidation state in FeX3 and CoF3. compound?
(c) V shows oxidation state of + 5 in VF5. (a) +1 (b) +4
(d) Cu does not shows +2 oxidation state with I–. (c) +3 (d) +2
47. Which of the following is not correct about transition 57. Which of the following ions having following electronic
metals? structure would have maximum magnetic moment?
(a) Their melting and boiling points are high
(a) 1s 2 2 s 2 2 p 6 3s 2 3 p 6 3d 3
(b) Their compounds are generally coloured
(c) They can form ionic or covalent compounds (b) 1s 2 2 s 2 2 p 6 3s 2 3 p 6 3d 5
(d) They do not exhibit variable valency (c) 1s 2 2s 2 2 p 6 3s 2 3 p 6 3d 7
48. Transition elements
(a) have low melting point (d) 1s 2 2 s 2 2 p 6 3s 2 3 p 6 3d 9
(b) exhibit variable oxidation states 58. If n is the number of unpaired electrons, the magnetic
(c) do not form coloured ions moment (in BM) of transition metal/ion is given by
(d) show inert pair effect (a) n n 2 (b) 2n n 1
49. Which one of the following ions is the most stable in
aqueous solution? n n 2 2n n 1
(c) (d)
(a) V3+ (b) Ti3+
(c) Mn 3+ (d) Cr3+ 59. Which one of the following ions has the maximum
(At.No. Ti = 22, V = 23, Cr = 24, Mn = 25) magnetic moment?
50. Which one of the following does not correctly represent (a) Sc3+ (b) Ti3+
the correct order of the property indicated against it? (c) Cr3+ (d) Fe3+
(a) Ti < V < Cr < Mn : increasing number of oxidation 60. The magnetic nature of elements depend on the presence
states of unpaired electrons. Identify the configuration of
(b) Ti3+ < V3+ < Cr3+ < Mn3+ : increasing magnetic moment transition element, which shows highest magnetic moment.
(c) Ti < V < Cr < Mn : increasing melting points (a) 3d 7 (b) 3d 5
(c) 3d 8 (d) 3d 2
(d) Ti < V < Mn < Cr : increasing 2nd ionization enthalpy
EBD_7207
370 THE d-AND f-BLOCK ELEMENTS
61. Transition elements show magnetic moment due to spin 73. Bronze is an alloy of
and orbital motion of electrons. Which of the following (a) Pb + Sn + Zn (b) Cu + Sn
metallic ions have almost same spin only magnetic moment ? (c) Pb + Zn (d) Cu + Zn
(i) Co2+ (ii) Cr2+ 74. An alloy of transition metal containing a non transition
(iii) Mn 2+ (iv) Cr3+ metal as a constituent is
(a) (i) and (iii) (b) (i) and (iv) (a) invar (b) bronze
(c) (ii) and (iii) (d) (ii) and (iv) (c) chrome steel (d) stainless steel
62. The aqueous solution containing which one of the
75. Choose the correct increasing order of the oxidation state
following ions will be colourless? (Atomic number:
of the central metal atom in the following oxoanions.
Sc = 21, Fe = 26, Ti = 22, Mn = 25)
(a) Sc3+ (b) Fe2+ VO2 , VO2 , TiO 2 , CrO 24
(c) Ti 3+ (d) Mn 2+
63. Transition elements form coloured ions due to (a) VO2 VO2 TiO 2 CrO 24
(a) d-d transition (b) fully filled d-orbitals
(c) smaller atomic radii (d) availability of s-electrons (b) VO2 TiO 2 VO 2 CrO 42
64. The catalytic activity of transition metals and their
compounds is mainly due to
(c) CrO 24 TiO 2 VO2 VO2
(a) their magnetic behaviour
(d) TiO 2 VO 2 VO 2 CrO 42
(b) their unfilled d-orbitals
(c) their ability to adopt variable oxidation state 76. Which of the following ion(s) is/are oxidising in nature?
(d) their chemical reactivity
65. Which of the following is colourless in water? (i) V2+ EM 2 / M 1.18

(a) Ti3 (b) V3 (ii) Mn3 E 1.57


M3 / M 2
(c) Cu 3 (d) Sc3
66. Which group contains coloured ions out of (iii) Cr 2 E 0.91
M2 / M
2 4
(i) Cu (ii) Ti (a) (i) and (iii) (b) only (ii)
2
(iii) Co (iv) Fe 2 (c) (ii) and (iii) (d) only (iii)
(a) (i), (ii), (iii), (iv) (b) (i), (iii), (iv) 77. Which of the following transition metal ion is colourless in
(c) (ii), (iii) (d) (i), (ii) aqueous solution?
67. Which of the following statements about the interstitial (a) Ti4+ (b) Zn 2+
compounds is incorrect ? (c) V 4+ (d) Both (a) and (b)
(a) They are chemically reactive. 78. Transition metals show catalytic activity
(b) They are much harder then the pure metal. (a) Due to their ability to form complexes.
(c) They have higher melting points than the pure metal. (b) Due to their ability to show multiple oxidation state.
(d) They retain metallic conductivity. (c) Due to availabiltiy of d orbitals for bond formation.
68. Formation of interstitial compound makes the transition (d) Both (a) and (b).
metal
79. Which of the following transition metal on catalysis the
(a) more soft (b) more ductile
reaction between iodide and persulphate ion?
(c) more metallic (d) more hard
(a) Fe2+ (b) Fe3+
69. If a non metal is added to the interstital sites of a metal, then 2+
(c) Ni (d) Both (a) and (c)
the metal becomes
(a) softer (b) less tensile 80. Which of the following reactions are disproportionation
(c) less malleable (d) more ductile reactions ?
70. Gun metal is an alloy of (i) Cu Cu 2 Cu
(a) Cu and Al (b) Cu and Sn
(c) Cu, Zn and Sn (d) Cu, Zn and Ni (ii) 3MnO 4 4H 2MnO4 MnO 2 2H 2 O
71. Brass is an alloy of
(a) Zn and Sn (b) Zn and Cu (iii) 2KMnO 4 K 2 MnO4 MnO 2 O 2
(c) Cu, Zn and Sn (d) Cu and Sn
(iv) 2MnO 4 3Mn 2 2H 2O 5MnO 2 4H
72. Which one of the following is coinage metal ?
(a) Zn (b) Cu (a) (i) and (ii) (b) (i), (ii) and (iii)
(c) Sn (d) Pb. (c) (ii), (iii) and (iv) (d) (i) and (iv)
THE d-AND f-BLOCK ELEMENTS 371
81. In the form of dichromate, Cr (VI) is a strong oxidising agent 91. In neutral or faintly alkaline medium, thiosulphate is
in acidic medium but Mo (VI) in MoO3 and W (VI) in WO3 quantitatively oxidized by KMnO4 to
are not because ____________ . (a) SO32– (b) SO42–
(i) Cr (VI) is more stable than Mo(VI) and W (VI). (c) SO2 (d) SO52–
(ii) Mo (VI) and W(VI) are more stable than Cr(VI). 92. KMnO4 can be prepared from K2MnO4 as per the reaction:
(iii) Higher oxidation states of heavier members of group-
6 of transition series are more stable. 3MnO 24 2H 2 O 2MnO24 MnO2 + 4OH
(iv) Lower oxidation states of heavier members of group-6 The reaction can go to completion by removing OH– ions
of transition series are more stable. by adding.
(a) (i) and (ii) (b) (ii) and (iii) (a) KOH (b) CO2
(c) (i) and (iv) (d) (ii) and (iv) (c) SO2 (d) HCl
82. K2Cr2O7 on heating with aqueous NaOH gives 93. In the laboratory, manganese (II) salt is oxidised to
permanganate ion in aqueous solution by
(a) CrO 24 (b) Cr(OH)3
(a) hydrogen peroxide (b) conc. nitric acid
(c) Cr2O 72 (d) Cr(OH)2 (c) peroxy disulphate (d) dichromate
94. The starting material for the manufacture of KMnO4 is
83. CrO3 dissolves in aqueous NaOH to give
(a) pyrolusite (b) manganite
(a) Cr2O72– (b) CrO42–
(c) magnatite (d) haematite
(c) Cr(OH)3 (d) Cr(OH)2 95. An explosion take place when conc. H2SO4 is added to
84. The oxidation state of chromium in the final product formed KMnO4. Which of the following is formed?
by the reaction between KI and acidified potassium
dichromate solution is (a) Mn 2O7 (b) MnO 2
(a) + 3 (b) + 2 (c) MnSO4 (d) M 2O3
(c) + 6 (d) + 4 96. If KMnO4 is reduced by oxalic acid in an acidic medium
85. The bonds present in the structure of dichromate ion are then oxidation number of Mn changes from
(a) four equivalent Cr – O bonds only (a) 4 to 2 (b) 6 to 4
(b) six equivalent Cr – O bonds and one O – O bond (c) +7 to +2 (d) 7 to 4
(c) six equivalent Cr – O bonds and one Cr – Cr bond 97. KMnO4 acts as an oxidising agent in alkaline medium. When
(d) six equivalent Cr – O bonds and one Cr – O – Cr bond alkaline KMnO4 is treated with KI, iodide ion is oxidised to
86. Potassium dichromate when heated with concentrated ________ .
sulphuric acid and a soluble chloride, gives brown-red
(a) I2 (b) IO
vapours of
(a) CrO3 (b) CrCl3 (c) IO3 (d) IO4
(c) CrO2Cl2 (d) Cr2O3
98. On the basis of data given below,
87. The acidic, basic or amphoteric nature of Mn2O7, V2O5 and
CrO are respectively E 0.37 , E 1.57
Sc3 /Sc2 Mn 3 /Mn 2
(a) acidic, acidic and basic
(b) basic, amphoteric and acidic E 0.90 , E 0.34
Cr 2 /Cr Cu 2 /Cu
(c) acidic, amphoteric and basic
Which of the following statements is incorrect?
(d) acidic, basic and amphoteric
(a) Sc 3+ has good stability due of [Ar]3d 0 4s 0
88. Which of the following oxides of Cr is amphoteric configuration.
(a) CrO2 (b) Cr2O3 (b) Mn3+ is more stable than Mn 2+.
(c) CrO5 (d) CrO3 (c) Cr2+ is reducing in nature.
89. Which of the following is amphoteric oxide ? (d) Copper does not give H2 on reaction with dil. H2SO4.
Mn 2 O7 , CrO3 , Cr2O3 ,CrO, V2 O5 , V2 O4 99. Which of the following is most acidic?
(a) Mn 2O7 (b) V2O5
(a) V2 O5 , Cr2 O3 (b) Mn 2 O7 , CrO3
(c) Fe2O3 (d) Cr2O3
(c) CrO, V2 O5 (d) V2O5 , V2 O 4 100. Which of the following is the use of potassium
permanganate?
90. When acidified K 2Cr2 O7 solution is added to Sn2+ salts
(a) Bleaching of wool, cotton and silk fibers.
then Sn2+ changes to (b) decolourisation of oils.
(a) Sn (b) Sn3+ (c) In analytical chemistry.
(c) Sn4+ (d) Sn+ (d) All of these.
EBD_7207
372 THE d-AND f-BLOCK ELEMENTS
101. Which of the following is not correctly matched? 110. Lanthanum is grouped with f-block elements because
Compound of Use (a) it has partially filled f-orbitals
transition metal (b) it is just before Ce in the periodic table
(a) TiO Pigment industry (c) it has both partially filled f and d-orbitals
(d) properties of lanthanum are very similar to the elements
(b) MnO2 Dry battery cell
of f-block
(c) V2O5 Manufacture of H2SO4
111. A reduction in atomic size with increase in atomic number is
(d) PdCl2 Manufacture of polyethylene a characteristic of elements of
102. A series1 metal ion , M(II) aqueous solution react with the (a) high atomic masses (b) d-block
KI to form iodine and a precipitate is formed, this M(II) can (c) f-block (d) radioactive series
be: 112. Which of the following oxidation states is the most common
(a) Zn2+ (b) Mn 2+ among the lanthanoids?
(c) Cu2+ (d) Ni2+ (a) 3 (b) 4
103. Total number of inner transition elements in the periodic (c) 2 (d) 5
table is 113. Identify the incorrect statement among the following:
(a) 10 (b) 14 (a) 4f and 5f orbitals are equally shielded.
(c) 28 (d) 30 (b) d-Block elements show irregular and erratic chemical
104. Which of the following ions will exhibit colour in aqueous properties among themselves.
solutions? (c) La and Lu have partially filled d-orbitals and no other
partially filled orbitals.
(a) La 3+ (Z = 57) (b) Ti3+ (Z = 22) (d) The chemistry of various lanthanoids is very similar.
114. In context of the lanthanoids, which of the following
(c) Lu 3+ (Z = 71) (d) Sc3+ (Z = 21) statements is not correct?
105. The lanthanoide contraction is responsible for the fact that (a) There is a gradual decrease in the radii of the members
(a) Zr and Y have about the same radius with increasing atomic number in the series.
(b) Zr and Nb have similar oxidation state (b) All the members exhibit +3 oxidation state.
(c) Zr and Hf have about the same radius (c) Because of similar properties the separation of
(d) Zr and Zn have the same oxidation state lanthanoids is not easy.
(d) Availability of 4f electrons results in the formation of
(Atomic numbers : Zr = 40, Y = 39, Nb = 41, Hf = 72, Zn = 30)
compounds in +4 state for all the members of the series.
106. Which one of the following elements shows maximum
115. The outer electronic configuration of Gd (Atomic No. : 64) is
number of different oxidation states in its compounds?
(a) 4f 3 5d5 6s2 (b) 4f 8 5d 0 6s2
(a) Eu (b) La 4
(c) 4f 5d 6s 4 2 (d) 4f 7 5d 1 6s2
(c) Gd (d) Am 116. The correct order of ionic radii of Y3+, La3+, Eu3+ and Lu3+
107. Lanthanoids are is
(a) 14 elements in the sixth period (atomic no. = 90 to 103)
that are filling 4f sublevel
(a) La 3 Eu 3 Lu 3 Y3
(b) 14 elements in the seventh period (atomic no. = 90 to (b) Y3 La 3 Eu 3 Lu 3
103) that are filling 5f sublevel
(c) Y3 Lu 3 Eu 3 La 3
(c) 14 elements in the sixth period (atomic no. = 58 to 71)
that are filling 4f sublevel (d) Lu 3 Eu 3 La 3 Y3
(d) 14 elements in the seventh period (atomic no. = 58 to (Atomic nos. Y =39, La = 57, Eu = 63, Lu = 71)
71) that are filling 4f sublevel 117. Which of the following lanthanoid ions is diamagnetic ?
108. Which of the following factors may be regarded as the main (At nos. Ce = 58, Sm = 62, Eu = 63, Yb = 70)
cause of lanthanoide contraction? (a) Sm2+ (b) Eu2+
(a) Greater shielding of 5d electrons by 4f electrons (c) Yb 2+ (d) Ce2+
(b) Poorer shielding of 5d electrons by 4f electrons 118. Lanthanide contraction can be observed in
(c) Effective shielding of one of 4f electrons by another in (a) At (b) Gd
the subshell (c) Ac (d) Lw
(d) Poor shielding of one of 4f electron by another in the 119. The approximate percentage of iron in mischmetal is
subshell (a) 10 (b) 20
(c) 50 (d) 5
109. Lanthanoid which has the smallest size in +3 state is
120. The most common lanthanide is
(a) Tb (b) Er
(a) lanthanum (b) cerium
(c) Ce (d) Lu (c) samarium (d) plutonium
THE d-AND f-BLOCK ELEMENTS 373
121. Non-lanthanide atom is 133. The increasing order of the shielding of electrons by the
(a) La (b) Lu orbitals ns,np,nd,nf is
(c) Pr (d) Pm (a) ns,np,nd,nf (b) np,ns,nd,nf
122. In which of the following lanthanides oxidation state +2 is (c) nd,nf,np,ns (d) nf,nd.np,ns
most stable? 134. Which of the following in its oxidation state shows the
(a) Ce (b) Eu paramagnetism ?
(c) Tb (d) Dy (a) Tb(IV) (b) Lu(III)
123. Actinoides
(c) Ce(IV) (d) La(III)
(a) are all synthetic elements
(b) include element 104 STATEMENT TYPE QUESTIONS
(c) have any short lived isotopes
135. Mark the correct statement(s).
(d) have variable valency
124. Which of the following exhibit only + 3 oxidation state ? (i) Manganese exhibits +7 oxidation state
(a) U (b) Th (ii) Zinc forms coloured ions
(c) Ac (d) Pa (iii) [CoF6]3– is diamagnetic
125. Larger number of oxidation states are exhibited by the (iv) Sc forms +4 oxidation state
actinoids than those by the lanthanoids, the main reason (v) Zn exhibits only +2 oxidation state
being (a) (i) and (ii) (b) (i) and (v)
(a) 4f orbitals more diffused than the 5f orbitals (c) (ii) and (iv) (d) (iii) and (iv)
(b) lesser energy difference between 5f and 6d than 136. Which of the following statements are correct ?
between 4f and 5d orbitals
(i) The maximum oxidation state of Mn with the oxygen is
(c) more energy difference between 5f and 6d than between
+VII while with fluorine is +IV.
4f and 5d orbitals
(ii) Fluorine is more oxidizing in nature than oxygen.
(d) more reactive nature of the actionids than the
lanthanoids (iii) Fluorine exhibit an oxidation state of –1.
126. The maximum oxidation state exhibited by actinide ions is (iv) Seven fluorine cannot be accommodated around Mn.
(a) +5 (b) +4 (a) (i), (ii) and (iii)
(c) +7 (d) +8 (b) (ii), (iii) and (iv)
127. There are 14 elements in actinoid series. Which of the (c) (i) and (iv)
following elements does not belong to this series ? (d) (i), (ii), (iii) and (iv)
(a) U (b) Np 137. Which of the following statements are correct ?
(c) Tm (d) Fm (i) Chromium has the highest melting point among the
128. Which of the following actinoids show oxiation states upto series 1 metals.
+7 ?
(ii) Number of unpaired electrons is greater in Cr than other
(i) Am (ii) Pu
elements of series 1.
(iii) U (iv) Np
(iii) In any row the melting point of transition metal
(a) (i) and (ii) (b) (ii) and (iv)
increases as the atomic number increases.
(c) (iii) and (iv) (d) (i) and (iii)
(a) (i) and (iii) (b) (i) and (ii)
129. Which of the following lanthanoid element is steel hard in
nature? (c) (ii) and (iii) (d) (i), (ii) and (iii)
(a) Eu (b) Pm 138. Read the following statements?
(c) Sm (d) Ce (i) Aqueous solutions formed by all ions of Ti are
130. What is the percentage of lanthanoid metal in mischmetall? colourless.
(a) 90% (b) 20% (ii) Aqueous solution of ferrous ions is green in colour.
(c) 5% (d) 95% (iii) Small size and presence of vacant d-orbitals make
131. Which of the following is the use of mischmetall ? transition metal ions suitable for formation of complex
(a) In bullets compounds.
(b) In lighter flint (iv) Catalytic action of transition metals involves the
(c) As catalyst in petroleum cracking increase of reactant concentration at catalyst surface
(d) Both (a) and (b) and weakening of the bonds in the reacting molecules.
132. Which of the following actinoid element has 5f 7 6d1 7s2 Which of the following is the correct code for above
configuration? statements?
(a) Bk (b) Cm (a) FTTT (b) TFFT
(c) Pa (d) No (c) TFTT (d) FFTT
EBD_7207
374 THE d-AND f-BLOCK ELEMENTS
139. Which of the following statements are correct? Which of the following is the correct code for the statements
(i) Interstitial compounds contain non-metal atoms above?
trapped inside the metal crystal whereas alloys are (a) FTTT (b) TFTT
homogeneous blend of metals. (c) TFFT (d) FFTT
(ii) Steel and bronze are alloys of transition and non- 144. Which of the following statement(s) regarding Hf and Zr
transition metals. is/are correct ?
(iii) Some boride containing interstitial compounds are very (i) Hf has greater density than Zr.
hard comparable to that of diamond. (ii) Lanthanoid contraction is responsible for such radii.
(iv) Interstitial compounds are chemically more reactive (a) Both (i) and (ii) are correct.
than parent metal. (b) Both (i) and (ii) are incorrect
(a) (i) and (iii) (b) (ii) and (iv) (c) Statement (i) is correct only
(c) (ii) and (iii) (d) (i), (ii) and (iii) (d) Statement (ii) is correct only.
140. Which of the following statements are correct?
(i) As a result of lanthanoid contraction members of 4d MATCHING TYPE QUESTIONS
and 5d series exhibit similar radii.
145. Match the columns
(ii) IE2 is high for Cr and Cu whereas IE3 is very high for
Zn. Column-I Column -II
(iii) Heavier members of d-block elements like p-block (A) Metal of the 3d-series which (p) Manganese
elements favours lower oxidation states. does not form MO type oxide.
(iv) In any transition series maximum number of oxidation (B) Metal of the 3d-series which (q) Vanadium
states is shown by middle elements or elements near forms most covalent oxide.
middle elements. (C) Metal of the 3d-series which (r) Scandium
(a) (i) and (ii) (b) (i), (ii) and (iv) forms the amphoteric oxide.
(c) (i), (ii) and (iii) (d) (ii) and (iv) (a) A – (p), B – (r), C – (q)
141. Consider the following statements (b) A – (r), B – (p), C – (q)
(i) La(OH)3 is the least basic among hydroxides of (c) A – (r), B – (q), C – (p)
lanthanides. (d) A – (q), B – (p), C – (r)
(ii) Zr4+ and Hf4+ posses almost the same ionic radii. 146. Match the columns
(iii) Ce4+ can as an oxidizing agent. Column-I Column-II
Which of the above is/are true ? (Ion) (M calculated)
(a) (i) and (iii) (b) (ii) and (iii) (A) Ti2+ (p) 2.84
(c) (ii) only (d) (i) and (ii) (B) Zn2+ (q) 5.92
142. Read the following statements. (C) Mn 2+ (r) 0
(i) Chemistry of actinoids is complex in comparsion to (D) Sc3+ (s) 4.90
chemistry of lanthanoids. (a) A – (s), B – (p), C – (q), D – (r).
(ii) Ce4+ is very good reducing agent. (b) A – (r), B – (p), C – (q), D – (s).
(iii) Eu2+ is a strong reducing agent. (c) A – (p), B – (r), C – (q), D – (s).
(iv) Out of all lanthanides Ce,Pr,Nd,Dy and Ho shows +4 (d) A – (p), B – (s), C – (q), D – (r).
oxidation state. 147. Match the columns
Which of the following is the correct code for the statements Column-I Column-II
above? (A) Compound formed when (p) acidified
(a) TTFF (b) TFTF yellow CrO24 is acidified. MnO4
(c) FTFT (d) FTTF
143. Read the following statements? (B) reagent oxidises Fe2+ to Fe3+ (q) Cr2 O72
(i) Only Pu show maximum oxidation state of +7 in
(C) Compound produced when (r) K2MnO4
actinoids.
MnO2 is fused with KNO3
(ii) M4+ ion of Th is the only diamagnetic M4+ ion of
(D) Compound having dark (s) KMnO4
actinoid series.
purple crystals isostructural
(iii) Electrons present in the 5f orbitals of actinides can
with KClO4
participate in bonding to a firm greater extent as
compared to electrons present in 4f orbitals of (a) A – (q),B – (p), C – (r), D – (s)
lanthanides. (a) A – (p),B – (q), C – (r), D – (s)
(iv) Magnetic properties of actinoids are more complex (a) A – (q),B – (r), C – (p), D – (s)
than lanthanoids (a) A – (q),B – (p), C – (s), D – (r)
THE d-AND f-BLOCK ELEMENTS 375
148. Match the columns 154. The electronic configuration of Cu(II) is 3d9 whereas that
Column-I Column-II of Cu(I) is 3d10. Which of the following is correct ?
(A) Lanthanide hard as steel. (p) Lu (a) Cu (II) is more stable
(B) Lanthanide with maximum (q) Tb (b) Cu (II) is less stable
paramagnetic character in (c) Cu (I) and (II) are equally stable
Ln4+ state. (d) Stability of Cu (I) and Cu (II) depends on nature of
(C) Lanthanide with maximum (r) Sm copper salts
value of E° for reaction 155. Highest oxidation state of manganese in fluoride is +4
(MnF4) but highest oxidation state in oxides is +7 (Mn2O7)
Ln3+(aq)+3e– Ln(s). because _______.
(D) Lanthanide whose Ln 3+ ion is (s) Eu (a) fluorine is more electronegative than oxygen.
diamagnetic in nature (b) fluorine does not possess d-orbitals.
(a) A – (r), B – (s), C – (p), D – (q) (c) fluorine stabilises lower oxidation state.
(b) A – (r), B – (q), C – (s), D – (p) (d) in covalent compounds fluorine can form single bond
(c) A – (s), B – (r), C – (q), D – (p) only while oxygen forms double bond.
(d) A – (r), B – (s), C – (q), D – (p) 156. Four successive members of the first series of the transition
metals are listed below. For which one of them the standard
ASSERTION-REASON TYPE QUESTIONS º
potential E M 2 /M
value has a positive sign?
Directions : Each of these questions contain two statements,
Assertion and Reason. Each of these questions also has four (a) Co (Z = 27) (b) Ni (Z = 28)
alternative choices, only one of which is the correct answer. You (c) Cu (Z = 29) (d) Fe (Z = 26)
have to select one of the codes (a), (b), (c) and (d) given below. 157. The standard redox potentials for the reactions
Mn2+ + 2e– Mn and Mn3+ + e– Mn2+ are –1.18 V and
(a) Assertion is correct, reason is correct; reason is a correct
1.51 V respectively. What is the redox potential for the
explanation for assertion.
reaction Mn3+ + 3e– Mn?
(b) Assertion is correct, reason is correct; reason is not a
(a) 0.33 V (b) 1.69 V
correct explanation for assertion
(c) – 0.28 V (d) – 0.85 V
(c) Assertion is correct, reason is incorrect
158. Which one of the following transition metal ions shows
(d) Assertion is incorrect, reason is correct. magnetic moment of 5.92 BM?
149. Assertion : Cuprous ion (Cu+) has unpaired electrons while (a) Mn 2+ (b) Ti3+
cupric ion (Cu++) does not. (c) Cr 3+ (d) Cu2+
Reason : Cuprous ion (Cu+) is colourless whereas cupric 159. In the following salts the lowest value of magnetic moment
ion (Cu++) is blue in the aqueous solution is observed in
150. Assertion : Transition metals show variable valency. (a) MnSO4. 4H2O (b) CuSO4.5H2O
Reason : Transition metals have a large energy difference (c) FeSO4.6H2O (d) ZnSO4.7H2O
between the ns2 and (n – 1)d electrons. 160. In which of the following pairs both the ions are coloured in
151. Assertion : Transition metals are good catalysts. aqueous solutions ?
Reason : V2O5 or Pt is used in the preparation of H2SO4 by (a) Sc3+, Ti3+ (b) Sc3+, Co2+
contact process. 2+
(c) Ni , Cu + (d) Ni2+, Ti3+
152. Assertion : Magnetic moment values of actinides are lesser (At. no. : Sc = 21, Ti = 22, Ni = 28, Cu = 29, Co = 27)
than the theoretically predicted values. 161. For the ions Zn2+, Ni2+ and Cr3+ which among the following
Reason : Actinide elements are strongly paramagnetic. statements is correct?
(atomic number of Zn = 30, Ni = 28 and Cr = 24)
CRITICAL THINKING TYPE QUESTIONS (a) All these are colourless
(b) All these are coloured
153. Among the following series of transition metal ions, the
(c) Only Ni2+ is coloured and Zn2+ and Cr3+ are colourless
one where all metal ions have 3d2 electronic configuration
(d) Only Zn2+ is colourless and Ni2+ and Cr3+ are coloured
is (At. nos. Ti = 22; V = 23; Cr = 24; Mn = 25)
162. Cuprous ion is colourless while cupric ion is coloured
(a) 3 2 3 4 because
Ti , V , Cr , Mn
(a) both have half filled p-and d-orbitals
(b) Ti , V 4 , Cr 6 , Mn 7 (b) cuprous ion has incomplete d-orbital and cupric ion
has a complete d-orbital
(c) Ti 4 , V 3 , Cr 2 , Mn 3 (c) both have unpaired electrons in the d-orbitals
(d) cuprous ion has complete d-orbital and cupric ion has
(d) Ti 2 , V 3 , Cr 4 , Mn 5
an imcomplete d-orbital.
EBD_7207
376 THE d-AND f-BLOCK ELEMENTS
163. The colour of the following ions V2+, V3+, V4+, Fe2+, Fe3+ 172. Which of the following oxidising reaction of KMnO4 occurs
are respectively in acidic medium?
(a) green, violet, blue, green, yellow (i) Fe2+ (green) is converted to Fe3+ (yellow).
(b) yellow, green, violet, green, blue (ii) Iodide is converted to iodate.
(c) violet, green, yellow, green, blue (iii) Thiosulphate oxidised to sulphate.
(d) yellow, green, blue, green, violet (iv) Nitrite is oxidised to nitrate.
164. Which of the following arrangements does not represent (a) (i) and (iii) (b) (i) and (iv)
the correct order of the property stated against it ? (c) (iv) only (d) (ii) and (iv)
(a) V2+ < Cr2+ < Mn2+ < Fe2+ : Paramagnetic behaviour 173. Arrange the following increasing order of acidic character?
(b) Ni2+ < Co2+ < Fe2+ < Mn2+ : Ionic size Mn2O7(A), Mn2O3(B), MnO(C)?
(c) Co3+ < Fe3+ < Cr 3+ < Sc3+ : Stability in aqueous (a) C, A, B (b) A, C, B
solution (c) B, A, C (d) C, B, A
(d) Sc < Ti < Cr < Mn : Number of oxidation states 174. Solution of oxalate is colourless. It is made acidic by adding
165. Acidified K2Cr2O7 solution turns green when Na2SO3 is excess of H+, then titrated with KMnO4. Now at a moment
added to it. This is due to the formation of : if someone has added large amount of KMnO4, in it then
(a) Cr2(SO4)3 (b) CrO42– no. of possible products are
(c) Cr2(SO3)3 (d) CrSO4 (a) CO2, Mn2+, H2O (b) CO2, MnO2, H2O
166. Which of the statements is not true? (c) MnO2, H2O, CO2 (d) CO2, MnO2, H2O, Mn2+
(a) On passing H2S through acidified K2Cr2O7 solution, 175. Knowing that the chemistry of lanthanoids(Ln) is dominated
a milky colour is observed. by its + 3 oxidation state, which of the following statements
(b) Na2Cr2O7 is preferred over K2Cr2O7 in volumetric is incorrect?
analysis. (a) The ionic size of Ln (III) decrease in general with
(c) K2Cr2O7 solution in acidic medium is orange. increasing atomic number
(d) K2Cr2O7 solution becomes yellow on increasing the (b) Ln (III) compounds are generally colourless.
pH beyond 7. (c) Ln (III) hydroxide are mainly basic in character.
167. Which one of the following is an amphoteric oxide ? (d) Because of the large size of the Ln (III) ions the bonding
(i) Mn 2O7 (ii) CrO in its compounds is predominantly ionic in character.
(iii) V2O4 (iv) Cr2O3 176. The +3 ion of which one of the following has half filled 4f
(a) (i) and (ii) (b) (ii), (iii) and (iv) subshell?
(c) (iii) and (iv) (d) (ii) and (iv) (a) La (b) Lu
168. Among the oxides, Mn 2O7 (I), V2O3 (II), V2O5 (III), CrO (c) Gd (d) Ac
(IV) and Cr 2O3 (V) the basic oxides are 177. Although + 3 is the characteristic oxidation state for
(a) I and II (b) II and III lanthanoids but cerium also shows + 4 oxidation state
(c) III and IV (d) II and IV because _________ .
169. When a small amount of KMnO4 is added to concentrated (i) it has variable ionisation enthalpy
H2SO4, a green oily compound is obtained which is highly (ii) it has a tendency to attain noble gas configuration
explosive in nature. Compound may be (iii) it has a tendency to attain f 0 configuration
(a) MnSO4 (b) Mn 2O7 (iv) it resembles Pb4+
(c) MnO2 (d) Mn 2O3 (a) (ii) and (iii) (b) (i) and (iv)
170. Identify the product and its colour when MnO2 is fused (c) (ii) and (iv) (d) (i), (ii) and (iii)
with solid KOH in the presence of O2. 178. Dichromate [Cr(VI) ] is a strong oxidizing agent whereas
(a) KMnO4, purple (b) K2MnO4, dark green Mo(VI) and W(VI) are found to be not. This is due to
(c) MnO, colourless (d) Mn2O3, brown (a) Lanthanoid contraction
171. When KMnO4 solution is added to oxalic acid solution, the (b) Down the group metallic character increases
decolourisation is slow in the beginning but becomes (c) Down the group metallic character decreases
instantaneous after some time because (d) Both (a) and (b)
(a) CO2 is formed as the product. 179. Which of the following conversions can be carried out by
(b) reaction is exothermic. both acidified K2Cr2O4 and acidified KMnO4?
(c) MnO4 catalyses the reaction.
(i) Fe2+ Fe3+ + e– (ii) I–
(iii) I– I2 (iv) H2S S
(d) Mn 2 acts as autocatalyst.
(a) (i) and (iii) (b) (ii) and (iv)
(c) (i), (iii) and (iv) (d) (i), (ii) and (iii)
THE d-AND f-BLOCK ELEMENTS 377

FACT / DEFINITION TYPE QUESTIONS 16. (b) Zn [Ar]3d10 4s1,Fe 2 [Ar]3d 6 4s0 , Ni [Ar]3d 8 4s1,
1. (c) General electronic configuration of transition elements
Cu [Ar]3d10 4s 0 ;
1 10 1 2
is ( n 1) d ns
Fe 2 contain maximum number of unpaired electrons.
2. (b) Cr (24) = 1s , 2 s2 2 p 6 , 3s 2 3 p 6 , 3d 5 , 4 s1 ,
2

3. (b) Configuration of Fe ( Z = 26) 17. (a) Ni 28 Ni Ar 3d8 4s 2 contain 2 unpaired electrons.


1s2, 2s2, 2p6, 3s2, 3p6, 3d 6, 4s2 18. (c) Zn, Cd, Hg do not show properties of transition
4. (d) Ni 3+
: [Ar] 3d 7 elements hence they are known as non typical
transition elements.
Mn3+ : [Ar] 3d 4 19. (c) The outer electronic configuration of the given ions is
as
Fe3+ : [Ar] 3d 5
Co3+ : [Ar] 3d 6 d s
5. (c) Ag belongs to second transition series. Ti2+
6. (c) 7. (a) d s
8. (d) Transition elements due to similar (almost) sizes exhibit Fe2+
both vertical and horizontal similarities. d s
9. (a) Group number is given by ns n 1 d electrons. Cr+
2 3 5 d s
Cu+
10. (a) 3d series starts from Sc Z 21 and ends with
Zn(Z –30).
20. (c) 21. (a)
11. (d) Since transition metals can lose electrons from
22. (a) Mn7+ = 25 – 7 = 18e– = [Ar]
n 1 d ns orbitals hence they are valence orbitals. 0 unpaired electrons.
12. (b) Atomic no. of Ni = 28
23. (b) Co [Ar]3d 7 4s 2
Ni (Ground state) = 1s2, 2s2, 2p6, 3s2, 3p6, 3d 8, 4s2,

2 2 6 2 6 8 0
Ni 2 = 1s , 2s , 2p , 3s , 3p , 3d , 4s
Since it contains three unpaired electrons. Hence it is
3d
paramagnetic.
24. (a) The outermost electronic configuration of Fe is
It has 2 unpaired electrons Fe = [Ar] 3d6 4s2
13. (c) Cerium (Ce) belongs to lanthanide series and is member Fe2+ = [Ar] 3d6 4s0
of inner-transition metals.
14. (c) Mn3+ = [Ar]3d4
= [Ar] Since Fe2+ has 4 unpaired electrons it is paramagnetic
in nature.
Number of unpaired electrons = 4 Zn = [Ar] 3d10 4s2 —— no unpaired e–
Cr3+ = [Ar]3d3 Hg2+ = [Ar] 4f 14 5d10 —— no unpaired e–
= [Ar] Ti4+ = [Ar] 3d0 4s0 —— no unpaired e–
No. of unpaired electrons = 3 25. (c) Due to d 5 configuration, Mn has exactly half filled
V3+ = [Ar]3d2 d-orbitals. As a result the electronic configuration is
stable means 3d electrons are more tightly held by the
= [Ar] nucleus and this reduces the delocalization of electrons
No. of unpaired electrons = 2 resulting in weaker metallic bonding.
15. (a) 26. (d) All statements are correct.
EBD_7207
378 THE d-AND f-BLOCK ELEMENTS
27. (d) The minimum oxidation state in transition metal is equal 37. (b) 30Zn and 80Hg have their d orbitals completely filled
to the number of electrons in 4s shell and the maximum so they do not show any variable valency.
oxidation state is equal to the sum of the 4s and 3d 38. (a) Highest O.S. by Mn (+7)
electrons. 39. (c) Zinc does not show variable oxidation state due to
Ti = [Ar] 3d24s2 completely filled d-orbitals.
Hence minimum oxidation state is +2 and maximum 40. (d) Sc does not show variable valency.
oxidation state is +4. Thus the common oxidation states 41. (b) 42. (d)
of Ti are +2, +3 and +4 43. (b) Transition metals are generally paramagnetic since they
28. (a) Os shows maximum oxidation state of +8. contain unpaired electrons.
44. (b) Since reduction potential of fluorine is highest
29. (b) Mn - 3d 5 4s2
transition metals exhibit highest oxidation state with
The no. of various oxidation states possible are + 2, fluorine.
+ 3, + 4, + 5, + 6 and + 7. 45. (a) Zn, Cd and Hg due to presence of completely filled
30. (c) Due to lanthanide contraction, the size of Zr and Hf d-orbitals in ground state as well as in their common
(atom and ions) become nearly similar. oxidation states are not regarded as a transition metals
31. (a) but they are studied along with the transition metals.
32. (d) Fe3+ is easily hydrolysed than Fe2+ due to more 46. (a) The +7 oxidation state of Mn is not represented in
positive charge. simple halides but MnO3F is known
33. (c) Electronic configuration 47. (d) Transition metals exhibit variable valency
3d 4s 48. (b) In transition metals d electrons also take part in
V2+ –3d 3 4s0 bonding, so they show variable oxidation states.
49. (d) For chromium ion + 3 oxidation state is most stable.
Cr – 3d 4 4s0 50. (c) The melting points of the transition element first rise
Mn – 3d 5 4s0 to a maximum and then fall as the atomic number
increases manganese have abnormally low melting
Fe – 3d 6 4s0 point.
For third ionization enthalpy Mn has stable 51. (a) They may or may not be diamagnetic
configuration due to half filled d-orbital. 52. (a) Mn++ –5 unpaired electrons
34. (d) (n – 1)d 5ns2 attains the maximum O.S. of + 7. Fe++ – 4 unpaired electrons
35. (a) The ionisation energies increase with increase in Ti++ – 2 unpaired electrons
atomic number. However, the trend is some irregular Cr++ – 4 unpaired electrons
among d-block elements. On the basis of electronic Hence maximum no. of unpaired electron is present in
configuration, the Mn++.
Zn : 1s 2 2s 2 p 6 3s 2 p6 d 10 4s 2 Magnetic moment µ number of unpaired electrons
53. (d) E 3 2 = – 0.41 V
E 3 2 = + 0.77 V
Cr / Cr Fe / Fe
Fe : 1s 2 2s 2 p 6 3s 2 p 6 d 6 4s 2
E E 3
Mn3 / Mn 2 = + 1.57 V,, Co / Co 2 = + 1.97 V
Cu : 1s 2 2s 2 p6 3s 2 p 6 d 10 4s1 2+
54. (d) Since Mn contains maximum number of unparied
2 2 6
Cr : 1s 2s p 3s p d 4s 2 6 5 1 electrons hence it has maximum magnetic moment
IE1 follows the order : Zn > Fe > Cu > Cr 55. (d) Magetic moment n n 2 where n = number of
36. (a) In a period on moving from left to right, ionic radii
decreases. unpaired electrons 15 nn 2 n=3
(a) So order of cationic radii is
Cr2+ > Mn2+ > Fe2+ > Ni2+ and 56. (c) Magnetic moment n n 2 BM
(b) Sc > Ti > Cr > Mn (correct order of atomic radii)
1.73 n n 2 n 1 , it has one unpaired electron
(c) For unpaired electrons
Mn 2 (Five) Ni 2 (Two) hence electronic configuration is Ar 3d 1 and

Co 2 (Three) Fe2 (Four) electronic configuration for Z = 22 is Ar 3d 2 4s2 .


(d) For unpaired electrons > Hence charge on Ti is +3
2
Fe (Four) Co (Three) >2 57. (b) The more the number of unpaired electrons, the more
is magnetic moment.Therefore the answer is (b).
Ni2 (Two) Cu 2 (One) 58. (a)
THE d-AND f-BLOCK ELEMENTS 379
59. (d) Fe3+(d5) has 5 unpaired electrons therefore magnetic
79. (b) 2Fe3+ + 2I– 2Fe2+ + I2
moment n(n 2) 5(5 2) 5.91 which is
maximum among given options. As Sc3+, Ti3+, Cr 3+, 2Fe 2 S2 O82 2Fe3 2SO 24
V3+ contains 0, 1, 3, and 2 number of unpaired electrons 80. (a)
respectively. 81. (b)
60. (b) 61. (b)
82. (a) Cr2 O 72– 2OH – 2CrO 42 H 2O
62. (a) Sc3+ 3d 04s 0
Hence CrO 24 ion is obtained.
Fe2+ 3d 6 4s 0
83. (b) CrO3 2NaOH Na 2CrO 4 H2O
Ti3+ 3d 14s 0
2 + 3+
84. (a) Cr2O 7 + 6I + 14H 3I 2 + 7H 2 O + 2Cr
Mn2+ 3d 5 4s 0 oxidation state of Cr is +3.
In Sc3+ there is/are no unpaired electrons. So the 2–
aqueous solution of Sc3+ will be colourless. 85. (d) O O
63. (a) Transition elements form coloured ions due to d-d
transitions. In the presence of ligands, there is splitting Cr Cr
of energy levels of d-orbitals. They no longer remain O O O
O O
degenerated. So, electronic transition may occur Dichromate ion
between two d-orbitals. The required amount of energy There are six equivalent Cr — O bonds and one
to do this is obtained by absorption of light of a Cr — O — Cr bond.
particular wavelength in the region of visible light. 86. (c) Solid potassium dichromate when heated with
64. (c) The transition metals and their compounds are used concentrated sulphuric acid and a soluble chloride
as catalysts. Because of the variable oxidation states gives orange red vapours of a volatile oily liquid
they may form intermediate compound with one of the CrO2Cl2
readtants. These intermediate provides a new path with K2Cr2O7 + 4NaCl + 6H2SO4
lowe activation energy. V2O5 + SO2 V2O4 + SO3 2KHSO4 + 4NaHSO4 + 2CrO2Cl2
2V2O4+ O2 2V2O5 chromyl chloride
65. (d) Since Sc3+ does not contain any unpaired electron it 87. (c) Mn2O7 is acidic, V2O5 is amphoteric acid and CrO is
is colourless in water. basic.
88. (a) CrO2 is amphoteric in nature
66. (b) Cu 2 Ar 3d 9 , Ti 4 Ar 3d 0 , Co 2 Ar 3d 7 , Fe 2 Ar 3d 6
89. (a) 90. (c)
1, 3, 4 are coloured ions hence the answer is b. 91. (b) In neutral or faintly alkaline medium thiosulphate is
67. (a) In interstitial compounds small atoms like H, B and C quantitatively oxidized by KMnO4 to SO42–
enter into the void sites between the packed atoms of 8KMnO4 + 3Na2S2O3 + H2O
crystalline metal. They retain metallic conductivity and 3K2SO4 + 8MnO2 + 3Na2SO4 + 2KOH
are chemically inert. 92. (b) HCl and SO2 are reducing agents and can reduce
68. (d) A covalent bond is formed between small interstial MnO4–. CO2 which is neither oxidising and nor
non-metal and transition metal which make it hard reducing will provide only acidic medium. It can shift
69. (c) If non metal is added to the interstital site the metal reaction in forward direction and reaction can go to
becomes less malleable due to formation of covalent completion.
bond between metal and non metal 93. (c) In laboratory, manganese (II) ion salt is oxidised to
70. (c) Gun metal is an alloy of Cu, Zn and Sn. It contains 88% permagnate ion in aqueous solution by
Cu, 10% Sn and 2% Zn. peroxodisulphate.
71. (b) Brass is an alloy of Cu and Zn 2Mn 2 S2O82 8H 2O 2MnO4 10SO42 16H
72. (b) Cu, Ag and Au are called coinage metals. peroxodisulphate ion
73. (b) Bronze is an alloy of Cu and Sn.
74. (b) Bronze - 10% Sn, 90% Cu 94. (a) Pyrolusite (It is MnO 2 )
(Sn is a non transition element) 95. (a) 2KMnO4 H 2SO4 Conc
4 4 5 6 K 2SO 4 Mn O H 2O
2 7
75. (b) VO2 Ti O2 VO2 Cr O42 Explosive
76. (b)
96. (c) In acid medium MnO4 8H 5e Mn 2 4H 2O
77. (d) Ti4+ (3d0) and Zn2+ (3d10) are colourless.
78. (d) (O.S. of Mn changes form +7 to +2)
EBD_7207
380 THE d-AND f-BLOCK ELEMENTS
97. (c) [Xe]4f7 6s2 – 7 unpaired e–
98. (b) Mn2+ (d5)is more stable than Mn 3+ (d4), thus Yb2+(Z = 70)
[Xe]4f14 6s2 – 0 unpaired e–
E ve
Mn 3 /Mn 2 Ce2+(Z = 58)
99. (a) As the oxidation state of metal associated with oxygen [Xe]4f 5d 6s – 2 unpaired e–
1 1 2

increases, the acidic character of oxide increases. Only Yb2+ is diamagnetic.


100. (d) 118. (b) Amongst the given elements, only Gd is a lanthanide .
101. (d) PdCl2 is used as a catalyst in Wacker’s process. 119. (d) Mischmetal is an alloy which contains rare earth
102. (c) Only Cu in its +2 oxidation state is able to oxidizes the elements (94-95%), iron (5%) and traces of sulphur,
I– to I2 carbon, silicon, calcium and aluminium. It is used in
103. (c) The number is 28(14 lanthanide +14 Actinides) gas lighters, tracer bullets and shells.
120. (b) Cerium is the most common lanthanide
104. (b) La 3+ : 54 e– = [Xe] 121. (a) La ( lanthanum ) is non lanthanide atom
Ti3+ : 19 e– = [Ar] 3 d 1 (Coloured) 122. (b) Eu 2 has electronic configuration Xe 4f 7 hence
Lu 3+ : 68 e– = [Xe] 4 f 14 stable due to half filled atomic orbitals.
123. (d) Actinides have variable valency due to very small
Sc3+ : 18 e– = [Ar] difference in energies of 5f, 6d and 7s orbitals. Actinides
105. (c) A regular decrease in the size of the atoms and ions in are the elements from atomic number 89 to 103.
lanthanoid series from La3+ to Lu3+ is called lanthanide 124. (c) Ac (89) = [Rn] [6d1] [7s2]
contraction. The similarity in size of the atoms of Zr 125. (b) The main reason for exhibiting larger number of
and Hf is due to the lanthanide contraction. oxidation states by actinoids as compared to
106. (d) We know that lanthanides La, Gd shows +3, oxidation lanthanoids is lesser energy difference between 5 f and
state, while Eu shows oxidation state of +2 and + 3. Am 6d orbitals as compared to that between 4f and 5d
shows +3, +4, +5 and +6 oxidation states. Therefore orbitals.
Americium (Am) has maximum number of oxidation In case of actinoids we can remove electrons from 5f
states. as well as from d and due to this actinoids exhibit larger
107. (c) Lanthanides are 4 f-series elements starting from cerium number of oxidation state than lanthanoids.
(Z= 58) to lutetium (Z = 71). These are placed in the 126. (c) Actinoids exhibit variable oxidation states, which vary
sixth period and in third group. from +3 to +7.
108. (b) In lanthanides, there is poorer shielding of 5d electrons 127. (c) 128. (b ) 129. (c)
by 4 f electrons resulting in greater attraction of the 130. (d) Mischmetall consists of a lanthanoid metal
nucleus over 5 d electrons and contraction of the atomic (~95%) and iron (~ 5%) and traces of S,C,Ca and Al.
radii. 131. (d)
109. (d) On going from left to right in lanthanoid series ionic, 132. (b) Curium (Cm) has configuration 5f 7 6d1 7s2.
size decreases i.e. 133. (d)
Ce+3 > Tb+3 > Er+3 > Lu+3. 134. (a) Tb4+ = 4f 7 — 3 unpaired e–
110. (d) 3+
Lu = 4f 14 — 0 unpaired e–
111. (c) Lanthanide contraction results into decrease in atomic 4+
Ce = 4f 0 — 0 unpaired e–
and ionic radii. 3+ 0
La = 4f — 0 unpaired e–
112. (a)
113. (a) 4f orbital is nearer to nucleus as compared to 5 f orbital STATEMENT TYPE QUESTIONS
therefore, shielding of 4 f is more than 5 f.
114. (d) 135. (b) (i) Outer electronic configuration of Mn is 3d54s2 and
115. (d) The configuration of Gd is [xe] 4f 7 5d1 6s2. hence exhibits +7 oxidation state.
116. (c) In lanthanide series there is a regular decrease in the (ii) Zinc does not form coloured ions as it has
atomic as well as ionic radii of trivalent ions (M3+) as completely filled 3d104s7 configuration.
the atomic number increases. Although the atomic radii (iii) In [CoF6]3–, Co3+ is a d7 system. Fluoride is a weak
do show some irregularities but ionic radii decreases field ligand and hence does not cause pairing of
from La(103 pm) to Lu (86pm). Y3+ belong to second electrons.
transition series there fore have greater ionic radii then Co3+ ; Paramagnetic
other ions of third transition series.
(iv)Sc can form a maximum of +3 oxidation state as it
117. (c) Sm2+(Z = 62)
has an outer electronic configuration of 3d14s2.
[Xe]4f6 6s2 – 6 unpaired e–
(v) Zn exhibits only +2 oxidation state as this O.S. is
Eu2+(Z = 63)
the most stable one.
THE d-AND f-BLOCK ELEMENTS 381

136. (d) Thus options (a) and (c) are discarded; now let us
137. (b) In any row the melting points of transition metals rise observe the second point of difference.
to a maximum at d5 except for anomalous values of 4
:1s 2 2s 2 p6 3s 2 p 6 d1
23 V
Mn and Tc and falls regularly as the atomic number
Thus option (b) is discarded
increases.
3
138. (a) Aqueous solution formed by Ti3+ ions has purple 23 V :1s 2 2 s 2 p 6 3s 2 p 6 d 2
colour. 4
24 Cr :1s 2 2s 2 p6 3s 2 p 6 d 2
139. (a) Steel is an alloy of Fe and C (non-metal). Interstitial
5
compounds are chemically inert. 25 Mn :1s 2 2 s 2 p 6 3s 2 p 6 d 2
140. (b) Heavier members of d-block elements unlike p-block 154. (a) 155. (d)
elements shows higher oxidation states. For example
156. (c) Eo = 0.34 V
W(VI) is more stable than Cr(VI). Cu 2 / Cu
141. (b) As a result of lanthanide contraction Zr 4+ and Hf4+ other has – ve E oR.P.
possess almost the same ionic radii. Ce4+ is an
oxidising agent. Ce4+ gains electron to acquire more Eo
Co / Co = – 0.28 V
stable Ce3+state. La(OH)3 is the most basic among
lanthanide hydroxides. Eo = – 0.25V
Ni / Ni
142. (b) Ce4+ is a strong oxidant reverting to the common +3
state. Eo = – 0.44V
Fe / Fe
Ho does not show oxidation state of +4. Lanthanoids
157. (c) E nE
showing +4 oxidation state are Ce, Pr, Nd, Dy and Tb.
2
143. (a) Both Np and Pu shows oxidation state of +7. Mn 2e Mn 1.18 2.36 V
144. (a) Atomic mass of Hf is greater than that of Zr, Hf is a Mn 3 e Mn 2 1.51 1.51 V
series 3 metal, so for almost similar radius Hf has greater Mn 3 3e Mn 0.28 0.85 V
density, Lanthanoid contraction is responsible for
158. (a) Given magnetic moment of transition metal
almost similar radii.
= n n 2 5.92
MATCHING TYPE QUESTIONS i.e., n = 5
145. (b) 146. (c) 147. (a) 148. (d) Number of unpaired electrons in Mn2+ = 5
Number of unpaired electrons in Ti3+ = 1
ASSERTION-REASON TYPE QUESTIONS Number of unpaired electrons in Cr3+ = 3
Number of unpaired electrons in Cu2+ = 1
149. (d) Number of unpaired electrons in Co2+ = 3
150. (c) The assertion is correct but the reason is false. Actually Thus Mn2+ have magnetic moment = 5.92 BM
transition metal show variable valency due to very 159. (a) Mn++ = 3d5 i.e. no. of unpaired e– = 5
small difference between the ns2 and (n – 1)d electrons. Cu++ = 3d9 i.e. no. of unpaired e– = 1
151. (b) Due to larger surface area and variable valencies to Fe++ = 3d6 i.e. no. of unpaired e– = 4
form intermediate absorbed complex easily, transition Zn++ = 3d10 i.e. no. of unpaired e– = 0
metals are used as catalysts. Ni++ = 3d8 i.e. no. of unpaired e– = 3
152. (b) The magnetic moments are lesser than the fact that 5f Higher the number of unpaired electrons higher will be
electrons of actinides are less effectively shielded the magnetic moment. Hence Mn ++ having maximum
which results in quenching of orbital contribution. unpaired electrons will have the maximum magnetic
moment.
CRITICAL THINKING TYPE QUESTIONS 160. (d) Sc3+ : 1s2, 2s2p6, 3s2p6d0, 4s0; no unpaired electron.
Cu+ : 1s2, 2s2p6, 3s2p6d 10, 4s0; no unpaired electron.
153. (d) The electronic configuration of different species given
Ni2+: 1s2, 2s2p6, 3s2p6d 8, 4s0;
in the question are unpaired electrons are present.
(a) 22 Ti3 :1s2 2s2 p6 3s2 p6 d1 Ti3+ : 1s2, 2s2p6, 3s2p6d 1, 4s 0;
unpaired electron is present
(b) 22 Ti :1s 2 2s 2 p6 3s 2 . p6 d 2 4s1
Co2+ : 1s2, 2s2p6, 3s2p6d 7, 4s0;
(c) 22 Ti4 :1s2 2s2 p6 3s 2 p6 unpaired electrons are present
2 So from the given options the only correct combination
(d) 22 Ti :1s 2 2 s 2 p 6 3s 2 p 6 d 2
is Ni2+ and Ti3+.
EBD_7207
382 THE d-AND f-BLOCK ELEMENTS
161. (d) The ions with unpaired electrons are colourled and 165. (a) The green colour appears due to the formation of
those with paired electrons are colourless. Cr+++ion
Zn 2 = 1s 2, 2s2p6, 3s2 p6 d10 Cr2 O72– 3SO32– 8H 3SO 2–
4 2Cr 3 4H 2 O
(No. of e s 28)
166. (b) Na2Cr2O7 is hygroscopic.
Cr 3 = 1s 2, 2s2p6, 3s2 p6 d3 167. (c) Mn2O7 acidic
(No. of e s 21) CrO basic
V2O4 amphoteric
Ni 2 = 1s2, 2s2p6, 3s2 p6 d8 Cr2O3 amphoteric
(No. of e s 26)
168. (d) Oxide Mn2O7 : Oxidation state of metal + 7
Thus Zn 2 , Cr 3 and Ni 2 have zero, 3 and 2 Oxide V2O3 : Oxidation state of metal + 3
unpaired electrons respectively. Oxide V2O5 : Oxidation state of metal + 5
10 Oxide CrO : Oxidation state of metal + 2
162. (d) In Cu Ar 3d there is no unpaired electron,
Oxide Cr 2O3 : Oxidation state of metal + 5
Cu 2 Ar 3d 9 contains one unpaired electron hence 169. (b) KMnO4 reacts with H2SO4 to form Mn2O7 which is
coloured. highly explosive substance.
163. (d) V2+ – violet, V3+ – green V4+ – blue 2KMnO4 + H2SO4 K2SO4 + Mn2O7 + H2O
Fe2+ – green Fe3+ – yellow 170. (b) 2MnO2 + 4KOH + O2 2K 2 MnO4 2H 2O
164. (a) dark green

(a) V = 3d 3 4s 2 ; V2+ = 3d 3 = 3 unpaired electrons 171. (d)


Cr = 3d 5 4s 1 ; Cr 2+ = 3d 4 = 4 unpaired electrons
172. (b) 5Fe2+ + MnO4 +8H+ Mn2+ +4H2O + 5Fe3+
Mn = 3d 5 4s2 ; Mn2+ = 3d 5 = 5 unpaired electrons
Fe = 3d 6 4s 2 ; Fe2+ = 3d 6 = 4 unpaired electrons 5NO 2 2MnO 4 6H 2Mn 2 5NO 3 3H 2O
Hence the correct order of paramagnetic behaviour
173. (d) As the oxidation state increases the acidity increases.
V2+ < Cr 2+ = Fe2+ < Mn2+
174. (d) If KMnO4 was added slowly than option a was correct,
(b) For the same oxidation state, the ionic radii generally
but at a moment due to addition of large amount of
decreases as the atomic number increases in a
KMnO4, reduction of whole KMnO4 added does not
particular transition series. hence the order is
take place, it also react with Mn2+ which had formed
Mn++ > Fe++ > Co++ > Ni++
in the solution to give MnO2.
(c) In solution, the stability of the compound depends
2MnO4 + 3Mn2+ + 2H2O 5MnO2 + 4H+
upon electrode potentials, SEP of the transitions metal
ions are given as 175. (b) Most of the Ln 3+ compounds except La3+ and Lu3+
Co3+ / Co = + 1.97, Fe3+ / Fe = + 0.77 ; are coloured due to the presence of f-electrons.
Cr3+ / Cr2+ = – 0.41, Sc 3+ is highly stable as it does not 176. (c) 177. (a)
show + 2 O. S. 178. (b) Down the group metallic character increases hence
tendency to loose electron increases.
(d) Sc – (+ 2), (+ 3)
Ti – (+ 2), (+ 3), (+ 4) 179. (c) I– is converted to IO3 by neutral or faintly alkaline
Cr – (+ 1), (+ 2), (+ 3), (+ 4), (+ 5), (+ 6) MnO 4 as shown below..
Mn – (+ 2), (+ 3), (+ 4), (+ 5), (+ 6), (+ 7) 2MnO 4 H2O I 2MnO 2 2OH IO 3
i.e. Sc < Ti < Cr = Mn
23
COORDINATION COMPOUNDS

FACT / DEFINITION TYPE QUESTIONS (a) CrCl3. 6H2O


(b) [ Cr (H2O)3 Cl3]. (H2O)3
1. According to the postulates of Werner for coordination
compounds (c) [ CrCl2 (H2O)4 ] Cl . 2H2O
(a) primary valency is ionizable (d) [ CrCl (H2O)5 ] Cl2 . H2O
(b) secondary valency is ionizable 7. K 4 [Fe(CN) 6 ] is a :
(c) primary and secondary valencies are non-ionizable
(a) double salt (b) complex compound
(d) only primary valency is non-ionizable.
(c) acid (d) base
2. Which of the following postulates of Werner’s theory is
incorrect? 8. The number of ions formed on dissolving one molecule of
(a) Primary valencies are satisfied by negative ions. FeSO4(NH4)2SO4.6H2O in water is:
(b) Secondary valencies are satisfied by neutral molecules (a) 4 (b) 5
or negative ions. (c) 3 (d) 6
(c) Secondary valence is equal to the coordination 9. The solution of K4[Fe(CN)6] in water will
number and it depends upon the nature of ligand (a) give a test K+ (b) give a test Fe2+
(c) give a test of CN – (d) give a test of [Fe(CN)6]4–
attached to metal.
(d) The ions/ groups bound by the secondary linkages 10. In the coordination compound, K4[Ni(CN)4], the oxidation
to the metal have charecteristic spatial arrangements. state of nickel is
3. CrCl3 has primary valence of (a) 0 (b) +1
(a) 3 (b) 4 (c) +2 (d) –1
(c) 2 (d) 1 11. The coordination number of a central metal atom in a
4. One mole of the complex compound Co(NH3)5Cl3, gives 3 complex is determined by
moles of ions on dissolution in water. One mole of the same (a) the number of ligands around a metal ion bonded by
complex reacts with two moles of AgNO3 solution to yield sigma and pi-bonds both
two moles of AgCl (s). The structure of the complex is (b) the number of ligands around a metal ion bonded by
(a) [Co(NH3)3Cl3]. 2 NH3 pi-bonds
(b) [Co(NH3)4Cl2] Cl . NH3 (c) the number of ligands around a metal ion bonded by
(c) [Co(NH3)4Cl] Cl2. NH3 sigma bonds
(d) [Co(NH3)5Cl] Cl2 (d) the number of only anionic ligands bonded to the metal
5. When AgNO3 is added to a solution of Co(NH3)5Cl3, the ion.
precipitate of AgCl shows two ionisable chloride ions. This
means : 12. The oxidation state of Cr in [Cr ( NH 3 ) 4 Cl 2 ] is
(a) Two chlorine atoms satisfy primary valency and one (a) 0 (b) + 1
secondary valency (c) + 2 (d) + 3
(b) One chlorine atom satisfies primary as well as 13. In Ni(CO)4– , oxidation number of Ni is :
secondary valency (a) 4 (b) – 4
(c) Three chlorine atoms satisfy primary valency (c) 0 (d) + 2
(d) Three chlorine atoms satisfy secondary valency
6. Which one is the most likely structure of CrCl3. 6H2O if 1/ 14. [EDTA]4 is a :
3 of total chlorine of the compound is precipitated by adding (a) monodentate ligand (b) bidentate ligand
AgNO3 (c) quadridentate ligand (d) hexadentate ligand
EBD_7207
384 COORDINATION COMPOUNDS
15. The compound having the lowest oxidation state of iron is: 26. NH2-NH2 serves as
(a) K 4 Fe(CN ) 6 (b) K 2 FeO 4 (a) Monodentate ligand (b) Chelating ligand
(c) Bridging ligand (d) Both (a) and (c)
(c) Fe 2O 3 (d) Fe(CO)5 27. Which one of the following is NOT a ligand ?
16. The coordination number and the oxidation state of the (a) PH3 (b) NO+
element ‘E’ in the complex (c) Na + (d) F–
[E (en)2 (C2O4)]NO2 (where (en) is ethylene diamine) are, 28. Glycinato ligand is:
respectively,
(a) 6 and 2 (b) 4 and 2 NH2
(c) 4 and 3 (d) 6 and 3 (a) CH 2

17. Some salts although containing two different metallic COO
elements give test for only one of them in solution. Such (b) bidentate ligant
salts are (c) two donor sites N and O–
(a) complex (b) double salts (d) All of the above
(c) normal salts (d) None of these 29. Which one does not belong to ligand?
18. Coordination number of Ni in [Ni(C2O4)3]4– is (a) PH3 (b) NO+
(a) 3 (b) 6 (c) BF3 (d) Cl–
(c) 4 (d) 5 30. Which ligand is expected to be bidentate?
19. According to Lewis, the ligands are
(a) acidic in nature (a) C2 O42 (b) CH3C N
(b) basic in nature (c) Br – (d) CH3NH2
(c) some are acidic and others are basic 31. Which one of the following ligands forms a chelate
(d) neither acidic nor basic (a) Acetate (b) Oxalate
20. Ligand in a complex salt are (c) Ammonia (d) Cyanide
(a) anions linked by coordinate bonds to a central metal 32. Choose the correct statement.
atom or ion (a) Coordination number has nothing to do with the
(b) cations linked by coordinate bonds to a central metal number of groups or molecules attached to the central
or ion atom
(c) molecules linked by coordinate bonds to a central (b) Coordination number is the number of coordinating
metal or ion sites of all the ligands connected to the central atom
(d) ions or molecules linked by coordinate bonds to a or the number of coordinate bonds formed by the metal
central atom or ion atom with ligands
21. The ligand N(CH2CH2NH2)3 is (c) Werner’s coordination theory postulates only one type
(a) tridentate (b) pentadentate of valency
(c) tetradentate (d) bidentate (d) All the above are correct
22. An example of ambidentate ligand is 33. O2 is a
(a) Ammine (b) Aquo (a) Monodentate ligand (b) Bidenate ligand
(c) Chloro (d) Thiocyanato (c) Tridentate ligand (d) Hexadenate ligand
23. Which of the following does not form a chelate ? 34. The stabilisation of cooordination compounds due to
(a) EDTA (b) Oxalate chelation is called the chelate effect. Which of the following
(c) Pyridine (d) Ethylenediamine is the most stable complex species ?
24. A bidenate ligand always
(a) has bonds formed to two metals ions (a) [Fe(CO)5 ] (b) [Fe(CN)6 ]3
(b) has a charge of +2 or – 2
(c) forms complex ions with a charge of +2 or –2
(c) [Fe(C2 O 4 )3 ]3 (d) [Fe(H 2 O)6 ]3
(d) has two donor atoms forming simultaneously two 35. A chelating agent has two or more than two donor atoms to
sigma ( ) bonds. bind to a single metal ion. Which of the following is not a
25. An ambident ligand is one which chelating agent ?
(a) is linked to the metal atom through two donor atoms (a) thiosulphato (b) oxalato
(b) has two donor atoms, but only one of them has the (c) glycinato (d) ethane - 1, 2-diamine
capacity to form a coordinate bond [or a sigma ( ) 36. Which of the following species is not expected to be a
bond] ligand?
(c) has two donor atoms, but either of two can form a
(a) NO (b) NH 4
coordinate bond
(d) forms chelate rings. (c) NH 2 CH 2 CH 2 NH 2 (d) Both (a) and (b)
COORDINATION COMPOUNDS 385
37. Which of the following complexes are homoleptic ? 47. As per IUPAC nomenclature, the name of the complex
[Co(H2O)4(NH3)2]Cl3 is :
(i) [Co(NH3 )6 ]3 (ii) [Co(NH3 )4Cl] (a) Tetraaquadiaminecobalt (III) chloride
(iii) [Ni(CN) 4 ]2 (iv) [Ni(NH 3 ) 4Cl 2 ] (b) Tetraaquadiamminecobalt (III) chloride
(c) Diaminetetraaquacoblat (II) chloride
(a) (i) and (ii) (b) (ii) and (iii) (d) Diamminetetraaquacobalt (III) chloride
(c) (iii) and (iv) (d) (i) and (iii) 48. The IUPAC name of the complex [Co(NH3)4(H2O)Cl]Cl2 is
38. Which of the following complexes are heteroleptic ? (a) aquatetramminechloridocobalt (III) chloride
(b) chloridoaquatetramminechloridocobalt (III) chloride
(i) [Cr(NH3 )6 ]3 (ii) [Fe(NH3 )4 Cl 2 ]
(c) chloridoaquatetramminechloridocobalt (III) chloride
(iii) [Mn(CN)6 ]4 (iv) [Co(NH3 )4Cl2 ] (d) tetrammineaquachloridocobalt (III) chloride
49. As per IUPAC nomenclature, the name of the complex
(a) (i), (iv) (b) (ii) and (iv) [Co(H2O)4(NH3)2]Cl3 is :
(c) (i) and (ii) (d) (i) and (iv) (a) Tetraaquadiaminecobalt (III) chloride
39. Central atoms/ions in coordination compounds are. (b) Tetraaquadiamminecobalt (III) chloride
(a) Lewis acid (b) Lewis bases (c) Diaminetetraaquacobalt (II) chloride
(c) Neutral molecules (d) All of these (d) Diamminetetraaquacobalt (III) chloride
40. What is the denticity of the ligand ethylenediaminetetra 50. Chemical formula for iron (III) hexacyanoferrate (II) is
actetate ion? (a) Fe[Fe(CN)6] (b) Fe3[Fe(CN)6]
(a) 4 (b) 2 (c) Fe3[Fe(CN)6]4 (d) Fe4[Fe(CN)6]3
(c) 6 (d) 1 51. The IUPAC name of K3[Ir(C2O4)3] is
(a) potassium trioxalatoiridium (III)
41. K 3[Al(C 2O 4 )3 ] is called
(b) potassium trioxalatoiridate (III)
(a) Potassium aluminooxalate (c) potassium tris (oxalato) iridium (III)
(b) Potassium trioxalatoaluminate (III) (d) potassium tris (oxalato) iridate (III)
(c) Potassium aluminium (III) oxalate
52. IUPAC name of Na 3[Co(ONO) 6 ] is
(d) Potassium trioxalatoaluminate (VI)
(a) Sodium cobaltinitrite
42. The hypothetical complex chlorodiaquatriamminecobalt (III)
(b) Sodium hexanitrito cobaltate (III)
chloride can be represented as
(c) Sodium hexanitrocobalt (III)
(a) [CoCl(NH3)3(H2O)2]Cl2
(d) Sodium hexanitritocobaltate (II)
(b) [Co(NH3)3(H2O)Cl3]
53. Which of the following is the correct name of compound.
(c) [Co(NH2)3(H2O)2 Cl] [Co(NH3)4(H2O)Cl]Cl2
(d) [Co(NH3)3(H2O)3]Cl3 (a) Tetraamineaqua chloride cobalt (III) chloride
43. The IUPAC name of the coordination compound (b) Tetraamineaqua chlorido cobalt (II) chloride
K 3[Fe(CN ) 6 ] is (c) Tetraamineaqua chloride cobalt (II) chloride
(a) Tripotassium hexacyanoiron (II) (d) Tetraamineaqua chlorido cobalt (III) chloride
(b) Potassium hexacyanoiron (II) 54. The number of geometrical isomers for [Pt(NH3)2Cl2] is
(c) Potassium hexacyanoferrate (III) (a) 2 (b) 1
(c) 3 (d) 4
(d) Potassium hexacyanoferrate (II)
55. Which of the following will give maximum number of
44. The IUPAC name for the complex [Co(ONO)(NH3)5]Cl2 is
isomers?
(a) pentaamminenitrito-N-cobalt(II) chloride (a) [Ni(C2O4) (en)2]2– (b) [Ni(en) (NH3)4]2+
(b) pentaamminenitrito-N-cobalt(III) chloride (c) [Cr(SCN)2 (NH3)4] + (d) [Co(NH3)4 Cl2]
(c) nitrito-N-pentaamminecobalt(III) chloride 56. Which one of the following octahedral complexes will not
(d) nitrito-N-pentaamminecobalt(II) chloride show geometric isomerism? (A and B are monodentate
45. The IUPAC name of K2[PtCl6] is ligands)
(a) hexachloroplatinate potassium (a) [MA5B] (b) [MA2B4]
(b) potassium hexachloroplatinate (IV) (c) [MA3B3] (d) [MA4B2]
(c) potassium hexachloroplatinate 57. Which of the following coordination compounds would
(d) potassium hexachloroplatinum (IV) exhibit optical isomerism?
46. The IUPAC name of [Ni(NH3)4] [NiCl4] is (a) pentamminenitrocobalt(III) iodide
(a) Tetrachloronickel (II) - tetraamminenickel (II) (b) diamminedichloroplatinum(II)
(b) Tetraamminenickel (II) - tetrachloronickel (II) (c) trans-dicyanobis (ethylenediamine) chromium (III)
(c) Tetraamminenickel (II) - tetrachloronickelate (II) chloride
(d) Tetrachloronickel (II) - tetrachloronickelate (0) (d) tris-(ethylendiamine) cobalt (III) bromide
EBD_7207
386 COORDINATION COMPOUNDS
58. The type of isomerism present in Pentaminenitrochromium 70. Which one of the following has an optical isomer?
(III) chloride is (a) [Zn(en) (NH3)2]2+ (b) [Co(en)3]3+
(a) optical (b) linkage (c) [Co(H2O)4(en)]3+ (d) [Zn(en)2]2+
(c) ionisation (d) polymerisation. (en = ethylenediamine)
59. Which of the following compounds shows optical 71. Which one of the following complex ions has geometrical
isomerism? isomers ?
(a) [Ni(NH3)5Br]+ (b) [Co(NH3)2(en)2]3+
(a) [ Co (CN ) 6 ]3 (b) [ Cr ( C 2 O 4 ) 3 ]3 3+
(c) [Cr(NH3)4(en)2] (d) [Co(en)3]3+
(d) [Cu ( NH 3 ) 4 ]2 (en = ethylenediamine)
(c) [ ZnCl4 ]2
72. The ionisation isomer of [Cr(H2O)4Cl(NO2)]Cl is
60. Which of the following ions can exhibit optical isomerism– (a) [Cr(H2O)4(O2N)]Cl2
(a) [Co(NH3)4Cl2]+ (b) [Co(NH3)2Cl4]– (b) [Cr(H2O)4Cl2](NO2)
(c) Cis-[Co(en)2Cl2] + (d) trans-[Co(en)2Cl2]+ (c) [Cr(H2O)4Cl(ONO)]Cl
61. Which would exhibit co-ordination isomerism (d) [Cr(H2O)4Cl2(NO2)].H2O
(a) [Cr(NH3)6][Co(CN)6] (b) [Co(en)2Cl2] 73. Isomerism exhibited by [Cr(NH3)2(H2O)2Cl2]+ are –
(c) [Cr(NH3)6]Cl3 (d) [Cr(en)2Cl2]+ (a) ionization, optical (b) hydrate, optical
62. [Co(NH3)5NO2]Cl2 and [Co(NH3)5(ONO)]Cl2 are related to (c) geometrical, optical (d) coordinate, geometrical
each other as 74. Type of isomerism which exists between [Pd(C6H5)2(SCN)2]
(a) geometrical isomers (b) optical isomers and [Pd(C6H5)2(NCS)2] is :
(c) linkage isomers (d) coordination isomers (a) Linkage isomerism (b) Coordination isomerism
63. Coordination isomerism is caused by the interchange of (c) Ionisation isomerism (d) Solvate isomerism
ligands between the 75. Which of the following complex will show geometrical as
(a) cis and trans structure well as optical isomerism (en=ethylenediammine)
(b) complex cation and complex anion (a) Pt(NH3)2Cl2 (b) [Pt(NH3)Cl4]
(c) inner sphere and outer sphere (c) [Pt(en)3]4+ (d) [Pt(en)2Cl2]
(d) low oxidation and higher oxidation states 76. The number of geometrical isomers from [Co(NH3)3(NO2)3]
64. Change in composition of co-ordination sphere yields which is
type of isomers (a) 2 (b) 3
(a) optical (b) geometrical (c) 4 (d) 0
(c) ionisation (d) None of these 77. The number of isomers exhibited by [Cr(NH3)3Cl3] is
65. Which of the following does not show optical isomerism? (a) 2 (b) 3
(c) 4 (d) 5
(a) [Co(NH3)3Cl3]0 (b) [Co (en) Cl2 (NH3)2]+
78. For the square planar complex [M (a) (b) (c) (d)] (where M
(c) [Co (en)3]3+ (d) [Co (en)2Cl2]+
= central metal and a, b, c and d are monodentate ligands),
(en = ethylenediamine) the number of possible geometrical isomers are
66. The complexes [Co(NH3)6] [Cr(CN)6] and [Cr(NH3)6 ] (a) 1 (b) 2
[Co(CN)6] are the examples of which type of isomerism? (c) 3 (d) 4
(a) Linkage isomerism 79. Which of the following will exhibit optical isomerism ?
(b) Ionization isomerism (a) [Cr(en) (H2O)4]3+
(c) Coordination isomerism (b) [Cr(en)3]3+
(d) Geometrical isomerism (c) trans-[Cr(en)(Cl2)(NH3)2]+
67. The complex, [Pt(py)(NH3)BrCl] will have how many (d) [Cr(NH3)6]3+
geometrical isomers ? 80. Which of the following will give maximum number of
(a) 3 (b) 4 isomers?
(c) 0 (d) 2 (a) [Co(NH3)4Cl2] (b) [Ni(en)(NH3)4]2+
68. Which of the following has a square planar geometry? (c) [Ni(C2O4)(en)2] (d) [Cr(SCN)2(NH3)4]2+
(a) [PtCl4]2– (b) [CoCl4]2– 81. The compounds [PtCl2(NH3)4]Br2 and [PtBr2(NH3)4]Cl2
(c) [FeCl4] 2– (d) [NiCl4]2– constitutes a pair of
(At. nos.: Fe = 26, Co = 27, Ni = 28, Pt = 78) (a) coordination isomers (b) linkage isomers
69. Which of the following pairs represent linkage isomers? (c) ionization isomers (d) optical isomers
(a) [Pd(PPh3)2(NCS)2] and [Pd ( PPh3)2 (SCN)2] 82. Which one of the following will not show geometrical
(b) [Co(NH3)5NO3] SO4 and [Co(NH3)5SO4]NO3 isomerism ?
(c) [PtCl2(NH3)4] Br2 and [Pt Br 2 (NH3)4] Cl2 (a) [Cr(NH3)4Cl2]Cl (b) [Co(en)2Cl2]Cl
(d) [Cu(NH3)4][Pt Cl4] and [Pt(NH3)4][CuCl4] (c) [Co(NH3)5NO2]Cl2 (d) [Pt(NH3)2Cl2 ]
COORDINATION COMPOUNDS 387
83. A similarity between optical and geometrical isomerism is (c) Ni(CO)4and [NiCl4]2–are diamagnetic and [Ni(CN)4]2–
that is paramagnetic
(a) each gives equal number of isomers for a given (d) [NiCl4]2– and [Ni(CN)4]2– are diamagnetic and Ni(CO)4
compound is paramagnetic
(b) if in a compound one is present then so is the other 92. Which of the following species represent the example of
(c) both are included in stereoisomerism dsp2 - hybridisation ?
(d) they have no similarity (a) [Fe(CN)6]3– (b) [Ni(CN)4]2–
84. The type of isomerism present in n itropenta- (c) [Ag(CN)2] – (d) [Co(CN)6]3–
amminechromium (III) chloride is 93. Which one of the following will show paramagnetism
(a) optical (b) linkage corresponding to 2 unpaired electrons?
(c) ionization (d) polymerization (Atomic numbers : Ni = 28, Fe = 26)
85. What kind of isomerism exists between [Cr(H 2 O)6 ]Cl3 (a) [FeF6]3– (b) [NiCl4]2–
(c) [Fe(CN)6] 3– (d) [Ni(CN)4]2–
(violet) and [Cr(H 2O)5Cl]Cl 2 H 2O (greyish-green) ? 94. Atomic number of Cr and Fe are respectively 25 and 26,
(a) linkage isomerism (b) solvate isomerism which of the following is paramagnetic?
(c) ionisation isomerism (d) coordination isomerism (a) [Cr(CO)6] (b) [Fe(CO)5]
86. Which of the following type of isomerism is shown by (c) [Fe(CN)6]–4 (d) [Cr(NH3)6]+3
given complex compound? 95. CN– is a strong field ligand. This is due to the fact that
(a) it carries negative charge
NH3
H3N NO2 (b) it is a pseudohalide
(c) it can accept electrons from metal species
Co
(d) it forms high spin complexes with metal species
O2N NO2 96. Which one of the following cyano complexes would exhibit
NH3
the lowest value of paramagnetic behaviour ?
(a) Facial (b) Meridional
(a) [Co(CN) 6 ]3 (b) [Fe(CN) 6 ]3
(c) Cis (d) Both b and c
87. Due to the presence of ambidentate ligands coordination (c) [Mn (CN) 6 ]3 (d) [Cr (CN) 6 ]3
compounds show isomerism. Palladium complexes of the
(At. Nos : Cr = 24, Mn = 25, Fe = 26, Co = 27)
type [Pd(C6H5)2(SCN)2] and [Pd(C6H5)2(NCS)2] are
97. [Sc(H2O)6]3+ ion is :
(a) linkage isomers (b) coordination isomers (a) colourless and diamagnetic
(c) ionisation isomers (d) geometrical isomers (b) coloured and octahedral
88. The geometry of Ni(CO)4 and Ni(PPh 3)2Cl2 are (c) colourless and paramagnetic
(a) both square planar (d) coloured and paramagnetic
(b) tetrahedral and square planar 98. Which has maximum paramagnetic nature ?
(c) both tetrahedral 2
(d) None of these (a) [Mn (H 2 O) 6 ]2 (b) [Cu( NH 3 ) 4 ]
89. The number of unpaired electrons in the complex
(c) [Fe(CN) 6 ]4 (d) [Cu(H 2 O) 4 ]2
[Cr(NH3)6]Br3 is (Atomic number Cr = 24)
(a) 4 (b) 1 99. The compound which is not coloured is
(c) 2 (d) 3 (a) K 4 [Fe(CN)6 ] (b) K3[Fe(CN)6 ]
90. Which of the following facts about the complex
(c) Na 2 CdCl 4 (d) Na 2 CuCl 4
[Cr(NH3)6]Cl3 is wrong?
(a) The complex involves d 2 sp3 hybridisation and is 100. Which of the following complexes exhibits the highest
octahedral in shape. paramagnetic behaviour ?
(b) The complex is paramagnetic. (a) [V(gly)2(OH)2(NH3)2]+ (b) [Fe(en)(bpy)(NH3)2]2+
(c) The complex is an outer orbital complex (c) [Co(ox)2(OH)2]2– (d) [Ti(NH3)6]3+
(d) The complex gives white precipitate with silver nitrate where gly = glycine, en = ethylenediamine and bpy =
solution. bipyridyl moities)
(At.nosTi = 22, V = 23, Fe = 26, Co = 27)
91. Which of the following statements is correct ?
101. Which of the following complex ion is not expected to absorb
(Atomic number of Ni = 28)
visible light ?
(a) Ni(CO)4 is diamagnetic and [NiCl4]2– and [Ni(CN)4]2–
2 3
are paramagnetic (a) Ni(CN)4 (b) Cr(NH3 )6
(b) Ni(CO)4and [Ni(CN)4]2– are diamagnetic and [NiCl4]2– 2 2
(c) Fe(H 2 O)6 (d) Ni(H 2O)6
is paramagnetic
EBD_7207
388 COORDINATION COMPOUNDS
102. Of the following complex ions, which is diamagnetic in nature? 111. Which of the following statements is incorrect?
(a) [NiCl4]2– (b) [Ni(CN)4]2– (a) [MnCl6]3– is more paramagnetic than [Mn(CN)6]3–
(c) [CuCl4]2– (d) [CoF6]3– (b) Both [Co(C2O4)3]3– and [CoF6]3– are paramagnetie.
103. Which one of the following is an outer orbital complex and (c) [Fe(CN)6]3– forms inner orbital complex whereas
exhibits paramagnetic behaviour ? [FeF6]3– forms outer orbital complex.
(a) [Ni(NH3)6]2+ (b) [Zn(NH3)6)]2+ (d) Both (a) and (b).
(c) [Cr(NH3)6] 3+ (d) [Co(NH3)6]3+ 112. Which of the following is not correctly matched?
104. Atomic number of Mn, Fe, Co and Ni are 25, 26, 27 and 28 Coordination polyhedron Shape
respectively. Which of the following outer orbital octahedral (A) [Co(NH3)6]3+ Octahedral
complexes have same number of unpaired electrons ? (B) [Ni(CO)4] Square planar
(C) [PtCl4]2– Tetrahedral
(i) [MnCl6 ]3 (ii) [FeF6 ]3
(a) C (b) B and C
(iii) [CoF6 ]3 (iv) [Ni(NH 3 )6 ]2 (c) A and C (d) B
113. Which of the following are inner orbital complex (i.e.,
(a) (ii) and (iii) (b) (i) and (iv)
involving d2sp3 hybridisation) and is paramagnetic in
(c) (i) and (ii) (d) (i) and (iii)
nature?
105. Which of the following options are correct for [Fe(CN)6 ]3 (a) [Mn(CN)6]3– , [Fe(CN)6]3–, [Co(C2O4)3]3–
complex ? (b) [MnCl6]3–, [FeF6]3–, [CoF6]3–
(i) Possess d2sp3 hybridisation (c) [Mn(CN)6]3–, [Fe(CN)6]3–
(ii) Possess sp3 d2 hybridisation (d) [MnCl6]3– , [Fe(CN)6]3–, [Co(C2O4)3]3–
(iii) It is paramagnetic 114. Which of the following is the limitation of valence bond
(iv) It is diamagnetic theory?
(a) (i) and (iii) (b) (ii) and (iii) (a) It does not distinguish between weak and strong
(c) (i) and (iv) (d) (ii) and (iv) ligands.
106. Which of these statements about [Co(CN)6]3– is true ? (b) It does not give quantitative interpretation of magnetic
(a) [Co(CN)6]3– has four unpaired electrons and will be in data.
a low-spin configuration. (c) It does not explain the colour exhibited by
(b) [Co(CN)6]3– has four unpaired electrons and will be in coordination compounds
a high spin configuration. (d) All of these
(c) [Co(CN)6]3– has no unpaired electrons and will be in a 115. Which complex of Co2+ will have the weakest crystal field
high-spin configurtion. splitting –
(d) [Co(CN)6]3– has no unpaired electrons and will be in a (a) [CoCl6]4– (b) [Co(CN)6]4–
low-spin configuration. (c) [Co(NH3)6]2+ (d) [Co(en)3]2+
107. The complex ion which has highest magnetic moment among 116. The crystal field stabilization energy (CFSE) is the highest
the following is for
(a) [CoF6]3– (b) [Co(NH3)6]3+ (a) [CoF4]2– (b) [Co(NCS)4]2–
2+ (c) [Co(NH3)6] 3+ (d) [CoCl4]2–
(c) [Ni(NH3)4] (d) [Ni(CN)4]2–
108. Which of the following complex ions is diamagnetic? 117. In which of the following complexes of the Co (at. no. 27),
(a) [FeF6]3– (b) [CoF6]3– will the magnitude of o be the highest?
(c) [Co(C2O4)3] 3– (d) [Fe(CN)6]3– (a) [Co(CN)6]3– (b) [Co(C2O4)3]3–
(c) [Co(H2O)6] 3+ (d) [Co(NH3)6]3+
109. Which of the following has square planar structure?
(a) [Ni(CO)4] (b) [NiCl4]2– 118. Among the ligands NH3, en, CN– and CO the correct order
of their increasing field strength, is :
2
(c) [Ni(CN)4]2– (d) Ni H 2O (a) NH3 en CN CO
6
110. Which of the following correctly explains the fact that
[Co(NH3)6]3+ is an inner orbital complex whereas [CoF6]3– (b) CN NH3 CO en
is an outer orbital complex?
(c) en CN NH3 CO
(a) NH3 being a strong ligand results into pairing of 3d
orbital electrons in Co3+. (d) CO NH3 en CN
(b) F– being a strong ligand results into pairing of 3d 119. Among the following complexes the one which shows zero
orbital electrons in Co3+. crystal field stabilization energy (CFSE):
(c) F– being a weak ligand cannot cause the pairing of (a) [Mn(H2O)6]3+ (b) [Fe(H2O)6]3+
electrons present in 3d orbital of Co3+. 2+
(c) [Co(H2O)6] (d) [Co(H2O)6]3+
(d) Both (a) and (c).
COORDINATION COMPOUNDS 389
120. The crystal field splitting energy for octahedral ( 0) and (iii) It does not explain how colour of coordination
tetrahedral ( t) complexes is related as compounds depends on ligand attached to central
metal atom/ion.
1 4
(a) t – 0 (b) t – 0 (a) (i) and (ii) (b) (ii) and (iii)
2 9
(c) (ii) only (d) (i), (ii) and (iii)
3 2 128. If magnetic moment of [MnBr4]2– is 5.9 BM. Predict the
(c) t – 0 (d) t – 0
5 5 number of electrons?
121. Which one of the following is the correct order of field (a) 2 (b) 3
strength of ligands in spectrochemical series? (c) 6 (d) 5
(a) I – < Cl– < F – < H2O < CN– 129. Arrange the following complexes in increasing order toward
(b) F – < H2O < I – < CN – < Cl– the wavelength of light they absorb? Where M is metal ion.
(c) CN – < I – < F – < Cl– < H2O [M(NH3)]3+ = a, [M(CN)6]3– = b, [M(C2O4)3]3– = c,
[MF6]3– = d,
(d) H2O < F – < CN – < Cl– < I–
(a) d, c, a, b (b) d, a, c, b
122. The colour of the coordination compounds depends on the
crystal field splitting. What will be the correct order of (c) b, a, c, d (d) a, b, c, d
absorption of wavelength of light in the visible region, for 130. Which of the following does not have a metal- carbon bond?
the complexes, (a) Al(OC 2 H 5 ) 3 (b) C 2 H 5MgBr
3 3 3 (c) K[Pt (C 2 H 4 )Cl 3 ] (d) Ni(CO) 4
[Co(NH3 )6 ] ,[Co(CN)6 ] ,[Co(H 2 O)6 ]
131. In Fe(CO)5, the Fe – C bond possesses
(a) [Co(NH)6 ]3 [Co(NH3 )6 ]3 [Co(H 2O)6 ]3 (a) ionic character (b) -character only
(c) -character (d) both and characters
(b) [Co(NH3 )6 ]3 [Co(H 2O)6 ]3 [Co(CN)6 ]3
132. The charge on the central metal ion in the complex [Ni(CO)4]
(c) [Co(H 2 O)6 ] 3
[Co(NH3 ) 6 ] 3
[Co(CN)6 ] 3 is
(a) + 2 (b) + 4
(d) [Co(CN)6 ]3 [Co(NH3 )6 ]3 [Co(H 2 O)6 ]3 (c) 0 (d) + 3
123. Which of the following statements related to crystal field 133. The unpaired electrons in Ni(CO)4 are
splitting in octahedral coordination entities is incorrect? (a) zero (b) one
(a) The dx2 –y2 and dz2 orbitals has more energy as (c) three (d) four
compared to dxy, dyz and dxz orbitals. 134. The correct structure of Fe(CO)5 is (Z=26 for Fe)
(b) Crystal field spitting energy ( o) depends directly on (a) octahedral (b) tetrahedral
the charge of the metal ion and on the field produced (c) square pyramidal (d) trigonal pyramidal
by the ligand. 135. For the reaction of the type M 4L ML 4
(c) In the presence of Br– as a ligand the distribution of
(a) larger the stability constant, lower the proportion of
electrons for d4 configuration will be t 32 g , eg1, ML4 that exists in solution
(d) In the presence of CN– as a ligand o < P. (b) larger the stability constant, higher the proportion of
124. Which of the following is incorrect regarding ML4 that exists in solution
spectrochemical series? (c) smaller the stability constant, higher the proportion of
ML4 that exists in solution
(a) NH3 > H2O (b) F C 2O 24 (d) None of the above
(c) NCS– > SCN– (d) en > edta4– 136. Coordination compounds have great importance in
125. For which of the following ligands magnitude of the crystal biological systems. In this context which of the following
field splitting ( o)will be greater than pairing energy (P)? statements is incorrect ?
(a) Cl– (b) SCN– (a) Cyanocobalamin is B12 and contains cobalt
(c) CO (d) S2– (b) Haemoglobin is the red pigment of blood and contains
126. Violet colour of [Ti(H2O)6]Cl3 on heating changes to___. iron
(a) Green (b) Colourless (c) Chlorophylls are green pigments in plants and contain
(c) White (d) Red calcium
127. Which of the following is the limitation of crystal field (d) Carboxypeptidase - A is an exzyme and contains zinc.
theory? 137. Which one of the following coordination compounds is
used to inhibit the growth of tumours?
(i) Ligands are assumed as point charges.
(a) Trans-platin (b) EDTA complex of calcium
(ii) It does not accounts for the covalent character of
bonding between the ligand and the central atom. (c) [(Ph3P)3RhCl] (d) Cis-platin
EBD_7207
390 COORDINATION COMPOUNDS
138. For [Co2(CO)8], what is the total number of metal – carbon (i) During nomenclature names of neutral ligands are kept
bonds and number of metal–metal bonds. same except for H2O, NH3 and CO.
(a) 10 ,1 (b) 8, 2 (ii) If the complex is anion, the name of the metal ends
(c) 8, 1 (d) 10, 0 with the suffix–ate.
139. Consider the following reactions.
(iii) While writing formula of coordination compounds
[XY] polydentate ligands are listed alphabeticaly.
X Y XY K1=
[X][Y] (iv) The cation is named first in both positively and
[XY2 ] negatively charged coordination entities.
XY Y XY2 K2 = (a) (i), (ii) and (iii) (b) (ii), (iii) and (iv)
[XY][Y]
(c) (i), (iii) and (iv) (d) (i), (ii), (iii) and (iv)
XY3 144. Which of the following statements are correct?
XY2 Y XY3 K3 = XY Y
2 (i) Square planar complexes of MABXL type show three
On the basis of reactions above which of the following is isomers-two cis and one trans.
incorrect? (ii) Complexes of Ma3B3 type show three isomers-two
(a) Overall stability constant = K1K2K3 cis and one trans.
[XY3 ] (iii) Optical isomerism is common in octahedral complexes
(b) K1K2K3 = involving bidentate ligands.
[X][Y]3
(iv) [Co(NH3)4Cl (NO2)]Cl show linkage isomerism.
1 (v) Hydrate isomerism is another name of solvate
(c) Dissociation constant = .
Formation constant isomerism.
(d) All of the above are correct. (a) (i), (ii) and (iii) (b) (i), (iii) and (iv)
140. Calculate the value of log K3 when log values of K2, K1, K4 (c) (ii), (iii) and (v) (d) (iii), (iv) and (v)
and 4 respectively are 4.0, 3.20, 4.0 and 11.9 ? 145. Which of the following statements are correct?
(a) 2.0 (b) 2.7 (i) When light of wavelength 600nm is absorbed by
(c) 3.0 (d) 2.5
complex [Ti(H2O)6]3+ its configuration changes from
STATEMENT TYPE QUESTIONS t12g e0g t 02g e1g and it appears violet in colour..
141. Identify the correct statements for the behaviour of
(ii) Anhydrous CuSO4 is white but CuSO4 .5H2O is blue
ethane- 1, 2-diamine as a ligand.
in colour as presence of H2O as a ligand causes crystal
(i) It is a neutral ligand.
field spitting.
(ii) It is a didentate ligand.
(iii) It is a chelating ligand. (iii) Ruby is aluminum oxide containing 0.5 – 1% Cr 3+
(iv) It is a unidentate ligand. ions with d3 configuration.
(a) (i), (ii) and (iii) (b) (ii) and (iii) (iv) Crystal field theory predict correctly that anionic
(c) (iii) and (iv) (d) (i), (iii) and (iv) ligands should exert the greater splitting effect.
142. Read the following statements (a) (i), (ii) and (iii) (b) (ii) and (iii)
(i) Macromolecules cannot behave as a ligand. (c) (i), and (ii) (d) (ii), (iii) and (iv)
(ii) [EDTA]4– can bind through two oxygen and four 146. Which of the following statement(s) is/are incorrect?
nitrogen atom.
(i) In metal carbonyls M–C bond is formed by the
(iii) Chelate complexes are more stable than similar
donation of lone pair of electrons on the carbonyl
complexes containing unidentate ligands.
carbon into a vacant orbital of metal.
(iv) Coordination number of the central atom/ion is
determined only by the number of sigma bonds formed (ii) M—C bond is formed by the donation of a pair of
by the ligand with central atom/ion electrons from a filled d orbital of metal into the vacant
Which of the following is the correct code for statements antibonding * orbital of CO.
above? (iii) Bonding in metal carbonyls is called synergic
(a) FFTT (b) FTFT bonding.
(c) TFTF (d) FFFT (a) (i) and (ii) (b) (iii) only
143. Which of the following statements regarding formulas and (c) (ii) only (d) None of these
naming of coordination compounds are correct?
COORDINATION COMPOUNDS 391

MATCHING TYPE QUESTIONS 151. Match the columns.


Column-I Column-I
147. Match the columns.
(A) Estimation of (p) [Ag(CN)2]–
Column-I Column-II
water hardness.
(Ligand) (Type of ligand)
(B) Extraction of silver. (q) [Ni(CO)4]
(A) Triphenylphosphine (p) Unidenate
(C) Hydrogenation of (r) Na2EDTA
(B) BF3 (q) Didentate
alkenes.
(C) Ethylenediamine (r) Not a ligand
(D) Photography (s) [(Ph3P)3RhCl]
(D) Ethylenediaminetetracetateion (s) Hexadenate
(E) Purification of (t) [Ag(S2O3)2]3–
(a) A – (p), B – (r), C – (q), D – (s)
(b) A – (p), B – (q), C – (r), D – (s) Nickel.
(c) A – (p), B – (r), C – (q), D – (s) (a) A – (r) , B – (p), C – (s), D – (t), E– (q)
(d) A – (p), B – (q), C – (s), D – (p) (b) A – (p) , B – (r), C – (s), D – (t), E– (q)
148. Match the complex species given in Column-I with the (c) A – (r) , B – (s), C – (p), D – (t), E– (q)
isomerism exhibited in Column-II and assign the correct (d) A – (r) , B – (p), C – (s), D – (q), E– (t)
code: 152. Match the columns.
Column-I Column-II Column-I Column-II
(Complex species) (Isomerism) (Coordination compound) (Central metal atom)
(A) [Co[NH3)4Cl2]+ (p) optical (A) Chlorophyll (p) Rhodium
(B) cis-[Co(en)2Cl2]+ (q) ionisation (B) Blood pigment (q) Cobalt
(C) [Co(NH3)5(NO2)]Cl2 (r) coordination
(C) Wilkinson catalyst (r) Calcium
(D) [Co(NH3)6][Cr(CN)6] (s) geometrical
(D) Vitamin B12 (s) Iron
(a) A – (s), B – (p), C – (q), D – (r)
(t) Magnesium
(b) A – (p), B – (r), C – (q), D – (s)
(a) A – (t), B – (s), C – (p), D – (q)
(c) A – (q), B – (s), C – (p), D – (r)
(b) A – (s), B – (q), C – (p), D – (r)
(d) A – (p), B – (r), C – (s), D – (q)
(c) A – (p), B – (q), C – (r), D – (s)
149. Match the columns.
(d) A – (r), B – (t), C – (p), D – (q)
Column-I Column-II
(A) [Ni(CN)4]2– (p) Ti4+
ASSERTION-REASON TYPE QUESTIONS
(B) Chlorophyll (q) sp3; paramagnetic
(C) Ziegler - Natta (r) Non-planar Directions : Each of these questions contain two statements,
catalyst Assertion and Reason. Each of these questions also has four
(D) [NiCl4]2– (s) Mg2+ alternative choices, only one of which is the correct answer. You
(E) Deoxyhaemoglobin (t) Planar have to select one of the codes (a), (b), (c) and (d) given below.
(u) dsp2; diamagnetic (a) Assertion is correct, reason is correct; reason is a correct
explanation for assertion.
(a) A – (u), B – (s), C – (p), D – (q), E – (r)
(b) A – (q), B – (s), C – (p), D – (u), E – (r) (b) Assertion is correct, reason is correct; reason is not a
correct explanation for assertion
(c) A – (q), B – (s), C – (p), D – (u), E – (t)
(d) A – (u), B – (s), C – (p), D – (q), E – (t) (c) Assertion is correct, reason is incorrect
150. Match the columns. (d) Assertion is incorrect, reason is correct.
Column-I Column-II 153. Assertion : NF3 is a weaker ligand than N(CH3)3.
(Complexes) (Absorbed Light) Reason : NF3 ionizes to give F– ions in aqueous solution.
(A) [Ni(H2O)4(en)]2+(aq) (p) Yellow Orange 154. Assertion : [Fe(CN)6]3– is weakly paramagnetic while
(B) [Ni(H2O)4(en)2]2+(aq) (q) Blue–Green [Fe(CN)6]4– is diamagnetic.
(C) [Ni(en)3]2+(aq) (r) Red Reason : [Fe(CN)6 ]3– has +3 oxidation state while
(a) A – (r), B – (q), C – (p) [Fe(CN)6]4– has +2 oxidation state.
(b) A – (p), B – (r), C – (q) 155. Assertion : [Ti(H2O)6]3+ is coloured while [Sc(H2O)6]3+ is
(c) A – (q), B – (r), C – (p) colourless.
(d) A – (r), B – (p), C – (q) Reason : d-d transition is not possible in [Sc(H2O)6]3+.
EBD_7207
392 COORDINATION COMPOUNDS
163. The total number of possible isomers for the complex
CRITICAL THINKING TYPE QUESTIONS
compound [CuII (NH3)4] [PtII Cl4]
156. A co-ordination complex compound of cobalt has the (a) 3 (b) 6
molecular formula containing five ammonia molecules, one (c) 5 (d) 4
nitro group and two chlorine atoms for one cobalt atom. 164. Which of the following will give a pair of enantiomorphs?
One mole of this compound produces three mole ions in an (a) [Cr(NH3)6][Co(CN)6] (b) [Co(en)2Cl2]Cl
aqueous solution. On reacting this solution with excess of (c) [Pt(NH3)4] [PtCl6] (d) [Co(NH3)4Cl2]NO2
AgNO3 solution, we get two moles of AgCl precipitate. The (en =NH2CH2CH2NH2)
ionic formula for this complex would be
165. The existence of two different coloured complexes with the
(a) [Co(NH3)4 (NO2) Cl] [(NH3) Cl]
(b) [Co (NH3)5 Cl] [Cl (NO2)] composition of [Co(NH3 )4 Cl2 ]+ is due to :
(c) [Co (NH3)5 (NO2)] Cl2 (a) linkage isomerism (b) geometrical isomerism
(d) [Co (NH3)5] [(NO2)2Cl2] (c) coordination isomerism(d) ionization isomerism
157. What is the secondary valence of following compounds 166. Which of the following has an optical isomer
PtCl2.2NH3, CoCl3.4NH3 and NiCl2.6H2O (a) [Co(en) (NH3)2]2+ (b) [Co(H2O)4(en)]3+
(c) [Co(en)2 (NH3)2] 3+ (d) [Co(NH3)3Cl] +
if moles of AgCl precipitated per mole of the given
compounds with excess AgNO3 respectively are: 0, 1 and 2 167. Among the following complexes, optical activity is possible
(a) 6, 4, 4 (b) 4, 6, 6 in
(c) 4, 4, 6 (d) 2, 4, 6 (a) [ Co ( NH 3 ) 6 ]3
158. C63H88CoN14O14P is the formulae of the Cyanocobalamine, (b) [ Co ( H 2 O) 2 ( NH 3 ) 2 Cl 2 ]
(vitamin B12) it contain CN– and CN– is very poisonous,
than why this compound does not prove to be fatal for (c) [ Cr ( H 2 O) 2 Cl 2 ]
us? (it inhibit the electron transport chain ? (d) [ Co ( CN ) 5 NC ]
(a) CN– forms covalent bond 168. [Co(NH3)4 (NO2)2] Cl exhibits
(b) CN– is coordinating to the cobalt as the ligand (a) linkage isomerism, ionization isomerism and
(c) CN– hydrolysis immediately geometrical isomerism
(d) All of these (b) ionization isomerism, geometrical isomerism and optical
159. Suppose someone made aqueous solution of NiCl2 and isomerism
recrystallized its aqueous solution in excess of water and (c) linkage isomerism, geometrical isomerism and optical
if two moles of precipitate AgCl was formed on treatment isomerism
with AgNO3, what is the most probable structure of the (d) linkage isomerism, ionization isomerism and optical
compound ? isomerism
(a) [Ni(Cl)2(H2O)4] (b) [Ni (H2O)6]Cl2 169. Which of the following compounds exhibits linkage
(c) [Ni(H2O)5Cl] (d) [Ni (H2O)4Cl2].2H2O isomerism ?
(a) [Co(en)2]Cl3 (b) [Co(NH3)6][Cr(en)3]
160. Total number of electron count in Ni(CO)4 and Fe(CO)5
respectively are. (c) [Co(en)2NO2Cl]Br (d) [Co(NH3)5Cl]Br2
170. Among the following coordination compounds/ions
(a) 36, 36 (b) 34, 36
(c) 36, 34 (d) 34, 34 3
(i) Fe CN 6 (ii) Pt NH3 2 Cl 2
161. The formula for the complex, dichlorobis (urea) copper (II)
is 3
(iii) Co NH 3 6 (iv) Cr H 2 O 6 Cl3
(a) [Cu{O = C (NH2)2}] Cl2
(b) [Cu{O = C (NH2)2}Cl]Cl Which species exhibit geometrical isomerism?
(c) [CuCl2 {O = C(NH2)2}2] (a) (ii) only (b) (i) and (ii)
(d) [CuCl2] [{O = C (NH2)2}]2 (c) (ii) and (iv) (d) (i) and (iii)
162. According to IUPAC nomenclature sodium nitroprusside is 171. Identify the optically active compounds from the following :
named as
(i) [Co(en)3 ]3 (ii) trans [Co(en)2 Cl 2 ]
(a) Sodium pentacyanonitrosylferrate (III)
(b) Sodium nitroferrocyanide (iii) cis [Co(en)2 Cl2 ] (iv) [Cr(NH3 )5 Cl]
(c) Sodium nitroferricyanide (a) (i) and (iii) (b) (ii) and (iii)
(d) Sodium pentacyanonitrosylferrate (II) (c) (iii) and (iv) (d) (i), (iii) and (iv)
COORDINATION COMPOUNDS 393
172. For which value of the x, and y, the following square (a) sp3, sp3 (b) sp3, dsp2
planar compound shows geometrical isomers [Pt (Cl)x 2
(c) dsp , sp 3 (d) dsp2, sp2
(Br)y]2– 180. Which one of the following is an inner orbital complex
(a) 1, 3 (b) 3, 1 as well as diamagnetic in behaviour? (Atomic number:
(c) 2, 2 (d) 1, 1 Zn = 30, Cr = 24, Co = 27, Ni = 28)
173. How many geometrical isomers are possible for following (a) [Zn(NH3 )6]2+ (b) [Cr(NH3)6]3+
square planar compound [M (Cl) (Br) (I) (F)] (where M is (c) [Co(NH3)6] 3+ (d) [Ni(NH3)6]2+
a metal ion)
181. The value of the ‘spin only’ magnetic moment for one of the
(a) 2 (b) 3 following configurations is 2.84 BM. The correct one is
(c) 9 (d) 8
174. The terahedral complex [M(A)(B)(X)(Y)], where A,B,X and (a) d 5 (in strong ligand field)
Y are different ligands and M is a metal ion is
(b) d 3 (in weak as well as in strong fields)
(a) optically inactive
(b) rotate plane polarized light (c) d 4 (in weak ligand fields)
(c) incomplete information
(d) d 4 (in strong ligand fields)
(d) can’t be said
175. The complex given is 182. The d electron configurations of Cr2+, Mn2+, Fe2+ and Ni2+
are 3d 4, 3d 5, 3d 6 and 3d 8 respectively. Which one of the
en 3+ following aqua complexes will exhibit the minimum
paramagnetic behaviour?
(a) [Fe(H2O)6]2+ (b) [Ni(H2O)6]2+
(c) [Cr(H2O)6] 2+ (d) [Mn(H2O)6]2+
Co
(At. No. Cr = 24, Mn = 25, Fe = 26, Ni = 28)
en 183. According to valence bond theory which of the following
en statement is correct about the complexes Ni(CO)4 and
(i) non-superimposable on its mirror images [Ni(CN)4]2– if both are diamagnetic in nature
(ii) optically inactive (a) both are tetrahedral
(iii) rotate plane polarised light (b) both are square planar
(iv) planar (c) one is square planar and other is tetrahedral
(a) (i) and (ii) (b) (i) and (iv) (d) one is tetrahedral and other is square planar
(c) (i), (ii) and (iii) (d) (ii) only 184. Correct statements about the following complexes
176. The correct statement with respect to the complexes Ni(CO)4 [MnCl6]3– and [Mn(CN)6]3– respectively are.
and [Ni(CN)4]2– is (a) Magnetic moment is 4.8 and 2.8
(a) nickel is in the same oxidation state in both (b) inner sphere and outer sphere complexes.
(b) both have tetrahedral geometry (c) sp3d2 and d2sp3 complexes.
(c) both have square planar geometry (d) Both (a) and (c).
(d) have tetrahedral and square planar geometry 185. In which of the following coordination entities the magnitude
respectively 0 (CFSE in octahedral field) will be maximum?
177. Which one of the following complexes is an outer orbital (a) [Co(H2O)6]3+ (b) [Co(NH3)6]3+
complex ? (c) [Co(CN)6] 3– (d) [Co (C2O4)3]3–
(a) [Co(NH3)6]3+ (b) [Mn(CN)6]4– (At. No. Co = 27)
(c) [Fe(CN)6] 4– (d) [Ni(NH3)6]2+ 186. Which of the following complex ions is expected to absorb
(Atomic nos. : Mn = 25; Fe = 26; Co = 27, Ni = 28) visible light?
178. The d-electron configurations of Cr2+, Mn2+, Fe2+ and Co2+ (a) [Ti (en)2(NH3)2]4 + (b) [Cr (NH3)6]3 +
are d 4 , d 5, d 6 and d 7, respectively. Which one of the (c) [Zn (NH3)6] 2 + (d) [Sc (H2O)3 (NH3)3]3+
following will exhibit minimum paramagnetic behaviour?
(At. no. Zn = 30, Sc = 21, Ti = 22, Cr = 24)
(a) [Mn(H2O)6]2+ (b) [Fe(H2O)6]2+
(c) [Co(H2O)6] 2+ (d) [Cr(H2O)6]2+ 187. Crystal field stabilization energy for high spin d 4 octahedral
(At, nos. Cr = 24, Mn = 25, Fe = 26, Co = 27) complex is:
179. Both [Ni(CO)4 ] and [Ni(CN)4]2– are diamagnetic. The (a) – 1.8 (b) – 1.6 0 + P
0
hybridisations of nickel in these complexes, respectively,
are (c) – 1.2 0 (d) – 0.6 0
EBD_7207
394 COORDINATION COMPOUNDS
188. Low spin complex of d 6 -cation in an octahedral field will
have the following energy : (a) Cu 2 4NH3 [Cu(NH3 ) 4 ]2 , log K = 11.6

12 12 (b) Cu 2 4NH [Cu(NH) 4 ]2 , log K = 27.3


(a) 0 P (b) 0 3P
5 5
2 2
(c) Cu 2 2en [Cu(en)2 ]2 , log K = 15.4
(c) 0 2P (d) 0 P
5 5
(d) Cu 2 4H 2O [Cu(H 2O)4 ]2 , Log K = 8.9
( 0= Crystal Field Splitting Energy in an octahedral field,
P = Electron pairing energy) 191. Atomic number of Mn, Fe and Co are 25, 26 and 27
189. Which of the following carbonyls will have the strongest respectively. Which of the following inner orbital octahedral
C – O bond ? complex ions are diamagnetic ?
(a) [Mn (CO)6]+ (b) [Cr (CO)6] (i) [Co(NH3 )6 ]3 (ii) [Mn(CN)6 ]3
(c) [V (CO)6]– (d) [Fe (CO)5]
(iii) [Fe(CN)6 ]4 (iv) [Fe(CN)6 ]3
190. Which of the following complexes formed by Cu 2 ions is
most stable ? (a) (i) and (ii) (b) (i) and (iii)
(c) (iii) and (iv) (d) (ii) and (iv)
COORDINATION COMPOUNDS 395

FACT / DEFINITION TYPE QUESTIONS 14. (d) [EDTA]4– is a hexadentate ligand, because it has six
donor atoms and donate 6 pairs of electrons to central
1. (a) The primary valencies are ionisable and represented metal atom in the complex.
by dotted line. 15. (d) In Fe(CO)5, Fe is in minimum oxidation state (zero).
2. (c) 16. (d) In the given complex we have two bidentate ligands
3. (a) CrCl3 has primary valence of 3. (i.e en and C2O4), so coordination number of E is 6
4. (d) Co (NH3)5 Cl3 [Co(NH3)5Cl]+2 + 2Cl- (2 × 2 + 1 × 2 = 6)
Structure is [Co (NH3)5 Cl] Cl2. Let the oxidation state of E in complex be x, then
Now [Co(NH3 )5 Cl]Cl2 2AgNO3 [x + (–2) = 1] or x – 2 = 1
or x = + 3, so its oxidation state is + 3
[Co(NH3 )5 Cl](NO3 )2 2AgCl Thus option (d) is correct.
5. (a) Since the precipitate of AgCl shows two ionisable 17. (a) Salt may be complex salt. Metal atom present in the
chloride ion the complex must have the structure. coordination sphere appears in the form of complex
ion and not as simple cation
[Co( NH 3 ) 5 Cl]Cl 2 2AgNO 3
18. (b) Coordinate number is = 2 (number of bindentate ligands
[Co( NH 3 )5 Cl]( NO 3 ) 2 2AgCl C.N. of Ni = (2 × 3) = 6
Hence two chlorine atoms satisfy the primary valency 19. (b) In the complex formation the ligands whether negative,
and one, secondary valency neutral or positive always donate electrons to the
6. (c) The ions present in the ionisation sphere are precipited central metal atom hence they act as Lewis bases.
20. (c) K[Co(CN)4] let the O. N. of Co be x then
Hence [CrCl 2 (H 2O) 4 ]Cl.2H 2 O contains 1/3 Cl in
1 × (+1) + 1(+x) + 4(–1) = 0 x = + 3
ionisation sphere to be precipited by AgNO3 as AgCl 21. (c) Number of donor atoms (N) in N(CH2CH2NH2)3 are
7. (b) Complex compounds do not dissociate into constituent four.
ions. So, N(CH2CH2NH2)3 is a tetradentate ligand.
K4[Fe(CN)6] 4K+ + [Fe(CN6)]4– 22. (d) Ambidentate ligands are those unidentate ligands
It is a complex because no CN– is formed on which contain more than one coordinating atoms.
dissociation. Thiocyanate is an example of such a ligand.
8. (b) It is a double salt: M SCN M NCS
Thiocyanato Isothiocyanato
FeSO 4 . NH 4 2 SO 4 .6H 2 O
23. (c) 24. (d)
Fe2 2SO24 2NH 4 25. (c) Ambident ligand has two donor atoms, either of two
9. (a) can form a coordinate bond.
10. (a) Let the o.s. of Ni in K4[Ni(CN)4] be = x then 26. (c)
4 (+ 1) + x + (–1) × 4 = 0 4+x–4=0 27. (c) Na+ is not a ligand.
28. (d) These are facts about glycinato ligand.
x=0
29. (c) BF3 has incomplete octet and is Lewis acid; it cannot
11. (c) The coordination number of central metal atom in a
donate electron pair.
complex is equal to number of monovalent ligands,
twice the number of bidentate ligands and so on, COO
30. (a) It has two donor atoms, i.e., |
around the metal ion bonded by coordinate bonds. COO
Hence coordination number = no. of bonds formed 31. (b) 32. (b)
by metals with ligands 33. (b) O2 is a bidentate ligand.
12. (d) Oxidation state of Cr in [Cr ( NH 3 ) 4 Cl 2 ] . 34. (c) 35. (a) 36. (b) 37. (c) 38. (b)
39. (a)
Let it be x, 1 × x + 4 × 0 + 2 × (–1) = 1 Therefore x =3.
40. (c) EDTA4– can bind through two nitrogen and four
13. (c) The oxidation state of metal in metal carbonyls is always
oxygen atoms to a central metal ion.
zero.
EBD_7207
396 COORDINATION COMPOUNDS

– 57. (d) The optical isomers are pair of molecules which are
CH2COO non superimposable mirror images of each other.
H2C N –
CH2COO en en

CH2COO
H2C N –
CH2COO
en Co Co en
41. (b) IUPAC name is Potassium trioxalatoaluminate (III).
42. (a) Chlorodiaquatriamminecobalt (III) chloride is
[CoCl( NH 3 )3 (H 2O) 2 ]Cl 2 . en en

43. (c) K 3[Fe(CN ) 6 ] is Potassium hexacyanoferrate (III). [Co(en)3]3+ [Co(en)3]3+


Mirror
(dextro) (laevo)
44. (b) [Co(ONO)(NH3)5]Cl2
pentaamminenitrito-N-cobalt (III) chloride The two optically active isomers are collectivity called
45. (b) K2[Pt Cl6] enantiomers.
Potassium hexachloroplatinate (IV) 58. (b) The chemical formula of Pentaminenitrochromium (III)
Oxidation state of Pt is + 4 in the complex and anion is chloride is
present in form of complex. Cr NH3 5 NO2 Cl2
46. (c) The correct IUPAC name of the given compound is
tetramminenickel (II) - tetrachloronickelate (II) thus (c) It can exist in following two structures
is the correct answer. Cr NH 3 5 NO 2 Cl2 and
47. (d) [Co(H2O)4(NH3)2]Cl3
= Diamminetetraaquacobalt (III) chloride. Cr NH3 5ONO Cl 2
48. (d)
Therefore the type of isomerism found in this
49. (d) [Co(H2O)4(NH3)2]Cl3
compound is linkage isomerism as nitro group is linked
= Diamminetetraaquacobalt (III) chloride.
through N as –NO2 or through O as – ONO.
50. (d) Chemical formula of iron (III) hexacynaoferrate (II) is
59. (b)
Fe4[Fe(CN)6]3
51. (b) IUPAC name of K3[Ir(C2O4)3] is 60. (c) cis-[Co(en)2Cl2]+: cis-[M(aa)2b2] can show optical
potassium trioxalato iridate (III) isomerism.
61. (a)
52. (b) Na 3[Co(ONO) 6 ] IUPAC name is
sodium hexanitritocobaltate (III) 62. (c) NH3 NH3
53. (d) According to IUPAC nomenclature anionic ligands H3N ONO H3N NO2
will end with – ido so that chloro would become Co Co
H3N NH3 H3N NH3
chlorid.
NH3 NH3
54. (a) Pt(NH3)2Cl2 is a disubstituted complex and shows only
cis-& trans-isomers Here more than one atom function as donor, as oxygen
Cl NH3 NH3 Cl in first one and nitrogen in second, so they show
linkage isomerism.
Pt Pt 63. (b) Co-ordination isomerism is caused by the interchange
Cl of ligands between cation and anion complexes.
Cl NH3 NH3
cis trans 64. (c) Change in composition of co-ordination sphere yield
55. (c) [Cr(SCN ) 2 ( NH 3 ) 4 ] shows linkage, geometrical and ionisation isomers.
[Cr(H2O)6]Cl3 and [CrCl3(H2O)3].3H2O
optical isomerism. Hence produces maximum no. of
65. (a) The octahedral coordination compounds of the type
isomers.
MA3B3 exhibit fac-mer isomerism.
56. (a) MA3B3 – 2 geometrical isomers
66. (c) Coordination isomerism occurs when cationic and
MA2B4 – 2 geometrical isomers
anionic complexes of different metal ions are present
MA4B2 – 2 geometrical isomers
in a salt. The two isomers differ in the distribution of
The complexes of general formula MA6 and MA5B ligands in cation and anion e.g.,
having octahedral geometry do not show geometrical [Co (NH3)6] [Cr (CN)6] is an isomer of [Co (CN)6]
isomerism.
[Cr (NH3)6]
COORDINATION COMPOUNDS 397

67. (a) Complexes of the type MABCD may exist in three


3+
isomeric forms. NH 3
A B A C

en Co en
M M

D C D B NH 3
(I) (II)
trans-
A C A C 72. (b) Ionisation isomer of [Cr(H2 O) 4 Cl(NO 2 )]Cl is
[Cr(H2O)4Cl2]NO2.
73. (c) Ma 2 b2 c 2 can show both optical & geometrical
M M isomerism.
74. (a) The compound shows linkage isomerism because the
ligand in the compound is an ambidenate ligand that
D B B D can bond at more than one atomic site.
(III)
i.e., NCS and SCN
Similarly, [Pt (py) (NH3) BrCl] may exist in three isomeric 75. (d) [Pt(en)2Cl2] is a complex of the type M(AA)2 B2 which
form in which is octahedral Such compounds exhibit optical and
M = Pt, A = Py, B = NH3, C = Br, D = Cl. geometrical isomerism both
68. (a) Complexes with dsp2 hybridisation are square planar. 76. (a) [Co(NH3)3(NO2)3] is of the type MA3B3. They give
So, [PtCl4]2– is square planar in shape. two geometrical Isomers
69. (a) The SCN– ion can coordinate through S or N atom 77. (d) [Cr(NH3)3Cl3] is of the type MA3B3 and exists in two
giving rise to linkage isomerism isomeric forms.
M SCN thiocyanato 78. (d) Square planar complex of the formula Mabcd give three
geometrical isomers
M NCS isothiocyanato.
79. (b)
70. (b) Option (b) shows optical isomerism [Co(en)3]3+
80. (d) The complex ion [Cr(SCN)2(NH3)4]2+ can exhibit
3+ 3+ geometrical and linkage isomerism
en en
81. (c) [PtCl2(NH3)4]Br2 and [PtBr2(NH3)4]Cl2 are ionisation
isomers
en Co Co en 82. (c) Octahedral complex of the type MA5B do not show
en
geometrical isomerism
en
83. (c) Similarity between optical and geometrical isomerism
d–form Mirror –form is that both are included in stereo isomerism
Complexes of Zn++ cannot show optical isomerism as 84. (b) The compound is [Cr(NH3)5NO2]Cl2 and can exhibit
they are tetrahedral complexes with plane of symmetry. O
3 linkage isomerism due to NO2 group (– N or
Co(H2 O)4 (en) have two planes of symmetry
O
hence it is also optically inactive.
O – N= O)
[Zn(en)2]2+ cannot show optical isomerism
85. (b)
en 3+ 86. (b) Given compound shows meridional isomerism.
71. (b)
NH3 87. (a)
88. (c) Ni(CO)4 Ni(PPh3)2 Cl2
Co O.S. Ni0 Ni2+
E.C. [Ar]3d84s2 [Ar]3d84s0
NH3
en Pairing of e– No pairing of e–
cis Hybridization sp3 (tetrahedral) sp3 (tetrahedral)
EBD_7207
398 COORDINATION COMPOUNDS
89. (d) In [Cr (NH3)6]Br3, Cr is in +3 oxidation state 98. (a) Paramagnetic species has unpaired electron. More the
3d 4s 4p no. of unpaired electrons, more will be paramagnetic
Cr character.
24 Complex O. S. of metal
d2sp3 hybridized
2
3d 4p Mn H 2 O Mn 2+
4s 6
+3
Cr
21 2
Cu NH3 4
Cu2+
Its ion is octahedral in nature. Due to the presence of
three unpaired electrons it is paramagnetic. 4
90. (c) [Cr(NH3)6]Cl3 is an inner orbital complex, because in Fe CN 6
Fe2+
this complex inner d-orbital is used for hybridisation
2
(d2 sp3 ) Cu H 2 O Cu2+
4
91. (b)
Electronic configuration of the ion present in complex
No. of
Atom/Ion Configuration
unpaired
Magnetic 3d 4s 4p
Complex nature 2+
electrons Mn
3d 4s 4p
2+ 8
Ni (d ) 2 No. of unpaired electrons = 5
2–
[NiCl4]
2 Cu2+
sp3
[Ni(CN)4]
2–
0 No. of unpaired electron = 1
Rearrangement 2 Fe2+
dsp
8 2 2
Ni (d s )
[Ni(CO)4]
However, CN– is a strong ligand, so pairing of electrons
0
will occur in the complex having CN– ions.
Rearrangement
sp
3 Fe2+ in presence of CN–

92. (b)
93. (b) As in [NiCl4]–2 chloride ion being a weak ligand is not No. of unpaired electron = 0
able to pair the electrons in d orbital. Thus [Mn (H2O)6]2+ having maximum no. of unpaired
94. (d) Cr 3+ has 4so 3d3 electronic configuration with 3 electrons has maximum paramagnetic nature.
unpaired electrons, hence paramagnetic. In other cases 99. (c) In Na2[CdCl4], Cd has oxidation state +2.
pairing of d-electrons take place in presence of strong So, its electronic configuration is 4d105s0
field ligands such as CO or CN–. or all the 4d orbitals are fully filled.
In Cr(CO)6 molecule 12 electrons are contributed by Hence, there will be no d-d transition. So, it is
CO group and it contain no odd electron colourless.
95. (b) CN– is a strong field ligand as it is a pseudohalide ion. 100. (c)
These ions are strong coordinating ligands and hence 2
have the tendency to form -bond (from the pseudo 101. (a) Ni(CN)4 : Number of unpaired electrons = 0
halide to the metal) and -bond (from the metal to 3
pseudo halide) Cr(NH3 ) 6 : Number of unpaired electrons = 3

: 2
96. (a) Co3+ Fe(H 2 O) 6 : Number of unpaired electrons = 4
[Co(CN)6]3– : 2
Ni(H 2O)6 : Number of unpaired electrons = 2
CN–
is a strong field ligand and it causes pairing of
102. (b) Ni++ = 3d 8 4s0
electrons as a result number of unpaired electrons in
Co3+ becomes zero and hence it has lowest value of
paramagnetic behaviour.
97. (a) Sc = [Ar] 3d 1, 4s2 Since, the coordination number of Ni in this complex is
Oxidation state of Sc in [Sc(H2O)6]3+ is Sc3+ 4, the configuration of Ni++ at first sight shows that
Sc3+ = [Ar] 3d 0, 4s0. the complex is paramagnetic with two unpaired
It does not have unpaired electron electron. However, experiments show that the complex
Sc3+ is diamagnetic and colourless.
COORDINATION COMPOUNDS 399

is diamagnetic. This is possible when the 3d electrons 109. (c) Electronic configuration of Ni2+ is [Ar] 3d8 4s0 4p0.
rearrange against the Hund’s rule as shown below. CN– is strong ligand and will do pairing of electrons
This is in accordance with the fact that the ligand so will have one d orbital left empty. C.N. is 4 so dsp 2
involved here is strong i.e., CN– ion. hybridisation will take place which is square planar,
Ni++ (after rearrangement) Ni(CO)4 and [Ni(CN)4]2– are tetrahedral.
4s 4p 110. (d)
111. (b) [Co(C2O4 )3]3– is dimagnetic as oxalate is a strong
ligand causing pairing of 3d electrons in Co3+thereby
leading to d2sp3 hybridisation.
Hence, now dsp2 hybridization involving one 3d, one
4s and two 4p orbitals, takes place leading to four dsp2 3d 4s 4p
hybrid orbitals, each of which accepts four electron Orbitals of
3+
pairsfrom CN– ion forming [Ni (CN)4]2– ion. Co ion
[Ni (CN)4]2– 2 3 3d 4s 4p
d sp hybridised
3+
×× ×× ×× ×× ×× oribitals of Co
2 3
2 d sp hybrid
four dsp hybrid bonds
Thus, the complex is diamagnetic as it has no unpaired 112. (b) [Ni(CO4)] tetrahedral
electron.
[PtCl4]2– square planar
103. (a) [Ni(NH3)6]2+
113. (c) [Mn(CN)6 ]3– and [Fe(CN) 6 ] 3– are inner orbital
Ni2+ = 3d 8, according to CFT = t 62g eg2 therefore, complexes and paramagnetic while [Co(C2O4)3]3– is
hybridisation is sp3d 2 and complex is paramagnetic. diamagnetic in nature.
104. (d) 105. (a) 114. (d)
106. (d) In [Co(CN)6]3– O.N. of Co is +3 115. (a) Cl– is a weak field ligand.
Co+3 = 3d6 4s0 116. (c) Higher the oxidation state of the metal, greater the
CN– is a strong field ligand crystal field splitting energy. In options (a), (b) and
Pairing of electrons occurs so in this complex no (d), Co is present in + 2 oxidation state and in (c) it is
unpaired electron is present and it is low spin present in + 3 oxidation state and hence has a higher
complex. value of CFSE.
107. (a) 117. (a) In octahedral complex the magnitude of o will be
Complex Configuration No. of unpaired highest in a complex having strongest ligand. Out of
electrons the given ligands CN– is strongest. So, o will be
[CoF6] 3– 4 highest for [Co(CN)6]3–. Thus option (a) is correct.
[Co(NH3)6]3+ 0 118. (a) Ligands can be arranged in a series in the orders of
increasing field strength as given below :
[Ni(NH3)4]2+ 0
Weak field ligands :
3d 4s 4p
[Ni(CN)4]2– : : : : : 0
I Br S2 SCN Cl N3 , F
2
Rearrangement dsp < Urea, OH– < oxalate
Magnetic moment = n n 2 , Strong field ligands

where n = no. of unpaired electrons O H 2O NCS EDTA Py, NH3 <


[CoF6]3– has highest magnetic moment (4.8) due to
en = SO3– < bipy, Phen NO2 CH3
the presence of 4 unpaired electrons.
108. (c) [Co(C2O4)3]3– has Co3+ (d6 system) C6 H 5CN CO
due to presence of stronger C2O42– chelating ligand
pairing of electrons occurs in this case. Such a series is termed as spectrochemical series. It is
an experimentally determined series based on the
Co3+ :
absorption of light by complexes with different ligands.
; Diamagnetic. 119. (b) Due to d5 configuration CFSE is zero.
EBD_7207
400 COORDINATION COMPOUNDS
120. (b) The crystal field splitting in tetrahedral complexes is 137. (d)
lower than that in octahedral complexes, and 138. (a) Structure of [Co2(CO)8]
4
t 0. O
9
121. (a) 122. (c)
CO C CO
123. (d) CN– is a strong field ligand and form low spin
complexes thus o > P. CO Co Co CO
124. (b) According to spectrochemical series C2 O24 F . CO C CO
125. (c) CO is a strong field ligand and for strong field ligands
O
o > P.
126. (b) In the absence of ligand , crystal field splitting does Total M – C bonds = 10, Total M – M bonds = 1
not occur and hence the substance is colourless.
127. (a) 1 [X][Y]3
128. (d) Magnetic moment can be calculated by using the 139. (a) Overall stability constant = K =
1K 2 K 3 [XY3 ]
relation n(n 2)B.M. Where n = number of
140. (b) = K1K2K3K4
electrons and B.M. is Bohr magneton.
log = log (K1 K2 K3 K4)
5.9 = n(n 2)
n=5 log = logK1 + logK2 + logK3 + logK4
129. (c) Energy for excitation ( ) = hc /wavelength. logK3 = 11.9– (3.20 + 2.0 + 4.0)
Therefore lower the wavelength of light higher will be logK3 = 2.7
the energy for excitation, ( ) i.e crystal field splitting
energy. STATEMENT TYPE QUESTIONS
correct order will be b < a < c < d
141. (a)
130. (a) Triethoxyaluminium has no Al – C linkage
142. (a) Macromolecules like proteins can acts as ligand.
O CH 2 CH 3 [EDTA]4– can bind through two nitrogen and four
Al O CH 2 CH 3 oxygen atoms.
O CH 2 CH 3 143. (d)
144. (b) Complexes of Ma3B3 type shows fac– meridional
131. (d) Due to some backbonding by sidewise overlapping isomerism.
between d-orbitals of metal and p-orbital of carbon, Solvate isomerism is refered to as a hydrate isomerism
the Fe–C bond in Fe(CO)5 has both and character. when solvent is water.
132. (c) In case of [Ni(CO)]4, the ligand CO, is neutral thus the
charge on Ni is zero. 145. (b) [Ti(H 2O)6 ]3 gives violet colour if light
133. (a) Ni(CO) 4 . The O. S. of Ni is Zero. Electronic corresponding to the energy of blue-green region of
configuration is [Ar] 3d8 4s2 4p0 . In presence of wavelength 498 nm is absorbed by it.
strong ligand CO the paring of electrons take place Irrespective of prediction of crystal field theory on
and electronic configuration will be [Ar] 3d10 4s0 4p0.
the basis of experimental observation shows that
Hence unpaired electrons is zero .
anionic ligands are found at the low end of the
134. (d) Fe(CO)5 (Z=26) O. S. of Fe is zero . Electronic
spectrochemical series.
configuaration is [Ar]] 3d6, 4s24p0. After pairing of
electrons of d and s orbitals, we have one d atomic 146. (d)
orbital empty. C. N. is 5 so hybridisation is dsp3 which
MATCHING TYPE QUESTIONS
is trigonal bipyramidal.
147. (c)
135. (b) For the reaction of the type M 4L ML4 , larger
148 (d) A – (p), B – (r), C – (s), D – (q)
the stability constant, the higher the proportion of 149. (a) [NiCl4]2– is sp3 hybridised and paramagnetic in nature
ML4 that exists in solution. [Ni(CN)4 ]2– is square planar and diamagnetic.
136. (c) The chlorophyll molecule plays an important role in Chlorophyll contains Mg2+ Ziegler – Natta catalyst
photosynthesis, contain porphyrin ring and the metal
contains Ti4+ Deoxyhaemoglobin is nonplanar and
Mg not Ca.
oxyhaemoglobin planar.
COORDINATION COMPOUNDS 401

150. (d) Crystal field splitting energy increases with increase 161. (c) [CuCl2{(O = C(NH2)2}2]
in ligand field strength i.e., with increase in no. of ‘en’ 162. (a) IUPAC name of sodium nitroprusside Na2[Fe(CN)5NO]
groups and wavelength of absorbed light decrease is sodium pentacyanonitrosylferrate (III) because in it
with increase in ligand field strength NO is neutral ligand. Hence
[Ni(H2O)4(en)]2+(aq) will absorb light of higher 2×O.N. of Na + O.N. of Fe + 5×O.N. of CN
wavelength i.e., Red. 1×O.N. of NO = 0
[Ni(en)3]2+ will absorb light of lower wavelength i.e., 2×(+1) + O.N. of Fe + 5 ×(–1) +1×0 = 0
blue-green and [Ni(H2O)4(en)2]2+ will absorb yellow O.N. of Fe = 5 – 2 = +3, Hence ferrate (III)
orange light. 163. (d) The total number of isomers for the complex compound
151. (a) 152. (a)
[Cu II ( NH 3 ) 4 ][Pt II Cl 4 ] is four..
ASSERTION-REASON TYPE QUESTIONS These four isomers are
153. (c) It is correct statement that NF3 is a weaker ligand than [Cu ( NH 3 )3 Cl] [Pt ( NH 3 )Cl 3 ],
N(CH3 ) 3 , the reason is that fluorine is highly
[Cu ( NH 3 )Cl 3 ] [Pt ( NH 3 ) 3 Cl] ,
electronegative therefore, it with draw electrons from
nitrogen atom. Hence, the lone pair of nitrogen atom [CuCl 4 ][Pt ( NH 3 ) 4 ]
cannot be ligated. While N(CH3)3 is a strong ligand
because CH3 is electron releasing group. and Cu(NH3 )4 PtCl4 .
154. (b) Both Assertion and Reason are true but Reason is not The isomer [Cu (NH3)2 Cl2][Pt (NH3)2 Cl2] does not
the correct explanation of statement-1. [Fe(CN)6]3– is exist due to both parts being neutral.
weakly paramagnetic as it has unpaired electrons while 164. (b) Non superimposable mirror images are called optical
[Fe(CN)6]2– has no unpaired electron. isomers and may be described as “chiral’. They are
It is diamagnetic. also called enantiomers and rotate plane polarised light
155. (a) Both Assertion and Reason are true and Reason is the in opposite directions.
correct explanation of statement-1. [Sc(H2O6]3+ has
Cl Cl
no unpaired electron in its d subshell and thus d–d
Cl Cl
transition is not possible whereas [Ti(H2O)6]3+ has
one unpaired electron in its d subshell which gives
en Co Co en
rise to d–d transition to impart colour.

CRITICAL THINKING TYPE QUESTIONS


en en
156. (c) As it forms two moles of silver chloride thus it has two
moles of ionisable Cl. 165. (b)

[Co(NH 3 )5 NO 2 ]Cl 2 [Co(NH 3 )5 NO 2 ] 2Cl


2Cl 2AgNO3 2AgCl 2NO 3
157. (b) Pt Cl2.2NH3= [Pt(NH3)2Cl2]
CoCl3.4NH3= [Co(NH3)4Cl2] Cl
NiCl2.6H2O = [Ni(H2O)6]Cl2.
158. (b) CN– is coordinated to cobalt as the ligand and trans (green)
coordinated compounds have different properties
than the individual species. Cl +
HN
3 Cl
159. (b) Since complex compound gives 2 moles of AgCl on
treatment with AgNO3.
most probable structure is (b). Co
160. (a) Total number of electron count in Ni(CO)4
= Atomic number – oxidation state + 2 × no. of ligands HN
3 NH3
NH3
= 28 – 0 + 2 × 4 = 36
Similarly for Fe(CO)5, cis (violet)
= 26 – 0 + 10 = 36
EBD_7207
402 COORDINATION COMPOUNDS

NH3 NH3 F Cl
166. (c) NH3
NH3

en en M
Co Co

Br I
en en 174. (b) Non –superimposable mirror images are optically
active, hence rotate plane polarized light.
3
Enantiomers of cis- Co(en) 2 (NH 3 ) 2
a a
167. (b) It is optically active.
168. (a) The given compound may have linkage isomerism due
M M
to presence of NO2 group which may be in the form d
b d b
–NO2 or –ONO. c c
It may have ionisation isomerism due to presence of whatever angle
175. (c) Complex is not superimposable onofitsmolecule
mirror image
two ionisable group –NO2 & –Cl. It may have
hence optically active i.e., rotate plane polarized light.
geometrical isomerism in the form of cis-trans form as
follows : 180°
[Co(NH3)4Cl(NO2)]NO2 & [Co(NH3) (NO2)2]Cl 3+ 3+
––– Ionisation isomers. en en
[Co(NH3)5(NO2)2]Cl & [Co(NH3)5(ONO)2]Cl
––– Linkage isomers en Co Co en
NO2 NO2
en en
H3N H3N
180°
NH3 NO2 cannot be
Co Co superimposed en 3+

NH3 NH3 NH3 NH3 non-superimposable en Co


en
NO2 NH3
Trans-form Cis-form
Geometrical isomers 176. (d) Nickel in Ni(CO)4 is sp3 hybridised therefore geometry
169. (c) The complex compound [Co(en)2NO2Cl] Br can have of Ni(CO)4 is tetrahedral whereas in [Ni(CN)4]2– nickel
NO2 group differently linked to central metal atom – is dsp2 hybridised therefore geometry of [Ni(CN)4]2–
O is square planar.
N or O – N = O 177. (d) Hybridisation
O
170. (a) Geometrical isomerism is possible only in square planar [Fe(CN)6 ]4 ,[Mn(CN)6 ]4 ,
complexes of the type MA2B2 and MA2BC and for d 2 sp3 d 2 sp 3

octahedral complexes of the type MA4B2 and MA4BC.


[Co(NH 3 ] 3 ,[Ni(NH 3 ) 6 ] 2
Hence only (ii) will show geometrical isomerism.
d 2 sp 3 sp 3d 2
171. (a)
172. (b) Geometrical isomers of following type of square Hence [ Ni( NH 3 ) 6 ]2 is outer orbital complex.
planar complexes is possible. Ma2b2 type, Ma2bc
type and Mabcd type. 178. (c) Cr2+ d4 4
173. (b) Three isomer are possible.
Mn2+ d5 5
F Cl F I

Fe2+ d6 4
M M
Co2+ d7 3
I Br Br Cl
Minimum paramagnetic behaviour = [Co (H2O)6]2+
COORDINATION COMPOUNDS 403

179. (b) In carbonyls O.S. of metal is zero


In [Ni(CO)4], the oxidation state of nickel is zero. Its = 3(5) = 15 = 3.87 B.M.
configuration in Ni(CO)4 is d 4– in weak ligand field

3d 4s 4p
[Ni(CO)4] ; t2g eg

sp3 hybridisation = 4(4+2) = 24 = 4.89


d 4– in strong ligand field
In [Ni(CN)4]2– the oxidation state of Ni is 2+ and its
configuration is

3d 4s 4p t2g eg
2
[Ni(CN)4]
= 2(4) = 8 = 2.82.
182. (b) Lesser is the number of unpaired electrons smaller will
dsp2 hybridisation
be the paramagnetic behaviour. As Cr ++, Mn++, Fe++
Thus the hybridisations of nickel in these compounds and Ni++ contains.
are sp3 and dsp2 respectively.
Cr++ (3d 4) =
Hence (b) is the correct answer.
180. (c) [Co(NH3 )6 ]3+ , Co3+ (27 –3 = 24) = 4 unpaired e–.

Mn++ (3d 5) =
2
d sp3 (inner octahedral = 5 unpaired e–.
complex & diamagnetic)
Fe++ (3d 6) =
[Cr(NH3 )6]3+ , Cr 3+ (24 –3 = 21)
= 4 unpaired e–.

2 Ni++ (3d 8) =
d sp3 (inner octahedral
complex & paramagnetic) = 2 unpaired e–.
[Ni(NH3 )6]2+ , Ni2+ (28 – 2 = 26) As Ni++ has minimum no. of unpaired e– thus this is
least paramagnetic.
183. (d) In case of diamagnetic complexes the electrons are
2 paired. In case of Ni (CO)4 3d orbital is fully filled give
sp3d (outer octahedral
complex & paramagnetic) rise to sp3 hybridisation while in case of [Ni(CN)4]2–
nickel is in +2 oxidation state, so one d-orbital is vacant
[Zn(NH3)6]2+ , Zn 2+ (30 – 2 = 28) give rise to dsp2 hybridization which is square planar
in nature.
184. (d) Number of unpaired electrons in [MnCl6]3– and
sp3d 2 (outer octahedral [Mn(CN)6]3– respectively are 4 and 2
complex &diamagnetic) Magnetic moment will respectively be 4.8 and 2.8
181. (d) d 5 –––– strong ligand field [MnCl6]3– is sp3d2 hybridised and [Mn(CN)6]3– will
be d2sp3 hybridised.
185. (c) In octahedral field the crystal field splitting of d- orbitals
t2g eg of a metal ion depends upon the field produced by the
ligands. In general ligands can be arranged in a series
= n(n 2) = 3 = 1.73BM in the order of increasing fields and splittings which
d 3–– in weak as well as in strong field they produce around a central metal ion. A portion of
the series is given below.
cyanide > ethylene - diamine > ammonia > pyridine >
t2g eg thiocyanate > water > oxalate > hydroxide > fluoride >
chloride > bromide > iodide.
EBD_7207
404 COORDINATION COMPOUNDS
Out of the given ligands water, ammonia, cyanide and Where, x electrons in t2g orbital
oxalate, we can find from the above series of ligands that y electrons in eg orbital
the maximum splitting will occur in case of cyanide (CN–)
i.e. the magnitude of 0 will be maximum in case of CFSE = [0.6 × 1] + [–0.4 × 3] = – 0.6 0
[Co(CN)6]3+. 6
188. (b) d – t2g 2, 2, 2 0,0 (in low spin)
eg
186. (b) Since Cr3+ in the complex has unpaired electrons in
the d orbital, hence it will absorb visible light and will
C.F.S.E = – 0.4 × 6 0 + 3P
be coloured
Ti = [Ar]3d 2 4 s2 ; Ti4 + = 3d 0 12
Cr = [Ar] 3d 5 4s1; Cr3+ = 3d 3 = 0 + 3P
5
Zn= [Ar] 3d 10 4s2; Zn2+= 3d 10
189. (a) As positive charge on the central metal atom increases,
Sc = [Ar] 3d 1 4s2; Sc3+ = 3d 0
the less readily the metal can donate electron density
187. (d) d 4 in high spin octahedral complex
into the * orbitals of CO ligand (donation of electron
eg — density into * orbitals of CO result in weakening of
C – O bond). Hence, the C – O bond would be
t 2g strongest in [Mn(CO)6]+.
CFSE = (–0.4x + 0.6y) 0 190. (b) 191. (b)
24
HALOALKANES AND
HALOARENES
FACT / DEFINITION TYPE QUESTIONS 9. Which of the following halide is 2° ?
(a) Isopropyl chloride (b) Isobutyl chloride
1. Which of the following is a primary halide? (c) n-propyl chloride (d) n-butyl chloride
(a) Isopropyl iodide (b) Secondary butyl iodide 10. Benzene hexachloride is
(c) Tertiary butyl bromide (d) Neohexyl chloride (a) 1, 2, 3, 4, 5, 6 - hexachlorocyclohexane
(b) 1, 1, 1, 6, 6, 6 - hexachlorocyclohexane
2. When two halogen atoms are attached to same carbon atom
(c) 1, 6 - phenyl - 1, 6 - chlorohexane
then it is : (d) 1, 1 - phenyl - 6, 6 -chlorohexane
(a) vic-dihalide (b) gem-dihalide 11. Phosgene is a common name for
(c) , -halide (d) , -halide (a) phosphoryl chloride
3. Gem-dibromide is (b) thionyl chloride
(a) CH3CH(Br)CH2(Br) (b) CH3CBr2CH3 (c) carbon dioxide and phosphine
(d) carbonyl chloride
(c) CH2(Br)CH2CH2 (d) CH2BrCH2Br
12. C – X bond is strongest in
4. How many structural isomers are possible for a compound
(a) CH 3Cl (b) CH 3Br
with molecular formula C3H7Cl ?
(a) 2 (b) 5 (c) CH 3F (d) CH 3I
(c) 7 (d) 9 13. Which of the following will have the maximum dipole
5. The compound which contains all the four 1°, 2°, 3° and 4° moment?
carbon atoms is (a) CH 3F (b) CH 3Cl
(a) 2, 3-dimethyl pentane
(b) 3-chloro-2, 3-dimethylpentane (c) CH 3Br (d) CH 3I
(c) 2, 3, 4-trimethylpentane 14. The decreasing order of boiling points of alkyl halides is
(d) 3, 3-dimethylpentane (a) RF > RCl > RBr > RI (b) RBr > RCl > RI > RF
6. IUPAC name of (CH3)3CCl (c) RI > RBr > RCl > RF (d) RCl > RF > RI > RBr
(a) 3-Chlorobutane 15. C 3 H 8 Cl 2 Light C 3 H 7 Cl HCl is an example of
(b) 2-Chloro-2-methylpropane (a) substitution (b) elimination
(c) t-butyl chloride (c) addition (d) rearrangement reaction
(d) n-butyl chloride
16. The reaction conditions leading to the best yields of C2H5Cl
7. IUPAC name of
are :
CH3CH 2C(Br) CH — Cl is
UV light
(a) 2-bromo-1-chloro butene (a) C2H6 (excess) + Cl2
(b) 1-chloro-2-bromo butene dark
(c) 3-chloro-2-bromo butene (b) C2H6 + Cl2 room temperatur e
(d) None of the above
UV light
8. The IUPAC name of CH 2 CH CH 2Cl is (c) C2H6 + Cl2 (excess)
(a) Allyl chloride (b) 1-chloro-3-propene UV light
(c) Vinyl chloride (d) 3-chloro-1-propene (d) C2H6 + Cl2
EBD_7207
406 HALOALKANES AND HALOARENES
17. Halogenation of alkanes is CS2
27. Silver acetate Br2 . The main product of this
(a) a reductive process (b) an oxidative process
reaction is
(c) an isothermal process (d) an endothermal process
(a) CH3 Br (b) CH 3COI
18. Ethylene dichloride can be prepared by adding HCl to
(c) CH 3COOH (d) None of these
(a) Ethane (b) Ethylene
28. Which of the following reactions is an example of
(c) Acetylene (d) Ethylene glycol nucleophilic substitution reaction?
19. In which of the following conversions, phosphorus (a) 2 RX + 2 Na R – R + 2 NaX
pentachloride is used as the reagent? (b) RX + H2 RH + HX
(a) H 2 C CH 2 CH 3CH 2 Cl (c) RX + Mg RMgX
(d) RX + KOH ROH + KX
(b) CH 3CH 2 OH CH 3 CH 2 Cl
29. Which one is most reactive towards SN 1 reaction ?
(c) H 3 C O CH 3 CH 3Cl (a) C6 H5 CH(C6 H 5 )Br
(d) CH CH CH2 = CHCl (b) C6 H5 CH(CH3 )Br
20. The best method for the conversion of an alcohol into an (c) C6 H 5 C(CH 3 )(C6 H 5 )Br
alkyl chloride is by treating the alcohol with
(a) PCl5 (d) C6 H 5 CH 2 Br
(b) dry HCl in the presence of anhydrous ZnCl2 30. A Grignard reagent may be made by reacting magnesium
(c) SOCl2 in presence of pyridine with
(d) None of these (a) Methyl amine (b) Diethyl ether
21. Which of the following is liquid at room temperature (b.p. is (c) Ethyl iodide (d) Ethyl alcohol
shown against it) ? 31. Which one of the following halogen compounds is difficult
to be hydrolysed by SN1 mechanism?
(a) CH3I 42ºC
(a) Tertiary butyl chloride (b) Isopropyl chloride
(b) CH3Br 3ºC
(c) Benzyl chloride (d) Chlorobenzene
(c) C2H5Cl 12ºC
32. The order of reactivity of the given haloalkanes towards
(d) CH3F –78ºC
nucleophile is :
22. The catalyst used in the preparation of an alkyl chloride by
(a) RI > RBr > KCl (b) RCl > RBr > RI
the action of dry HCl on an alcohol is
(c) RBr > RCl > RI (d) RBr > RI > RCl
(a) anhydrous AlCl3 (b) FeCl3
33. Most reactive halide towards SN1 reaction is
(c) anhydrous ZnCl2 (d) Cu
(a) n-Butyl chloride (b) sec-Butyl chloride
23. Chlorobenzene is prepared commercially by
(c) tert-Butyl chloride (d) Allyl chloride
(a) Raschig process 34. In SN1 reaction, the recemization takes place. It is due to
(b) Wurtz Fittig reaction (a) inversion of configuration
(c) Friedel-Craft’s reaction (b) retention of configuration
(d) Grignard reaction (c) conversion of configuration
24. In the preparation of chlorobenzene from aniline, the most (d) Both (a) and (b)
suitable reagent is 35. The order of reactivities of the following alkyl halides for a
(a) Chlorine in the presence of ultraviolet light SN2 reaction is
(b) Chlorine in the presence of AlCl3 (a) RF > RCl > RBr > RI (b) RF > RBr > RCl > RI
(c) Nitrous acid followed by heating with Cu2Cl2 (c) RCl > RBr > RF > RI (d) RI > RBr > RCl > RF
(d) HCl and Cu2Cl2 36. Which of the following is an example of SN2 reaction?
25. Which of the following possesses highest melting point? (a) CH 3 Br OH CH 3OH Br
(a) Chlorobenzene (b) m-dichlorobenzene
(b) CH 3 C H CH 3 OH CH 3 C H CH 3
(c) o-dichlorobenzene (d) p-dichlorobenzene | |
26. Conant Finkelstein reaction for the preparation of alkyl Br OH
iodide is based upon the fact that (c) H 2O
CH 3CH 2 OH CH 2 CH 2
(a) Sodium iodide is soluble in methanol, while sodium (d) (CH3 )3 C Br OH (CH3 )3COH Br
chloride is insoluble in methanol
37. SN2 mechanism proceeds through intervention of
(b) Sodium iodide is soluble in methanol, while NaCl and
(a) carbonium ion (b) transition state
NaBr are insoluble in methanol
(c) free radical (d) carbanion
(c) Sodium iodide is insoluble in methanol, while NaCl
38. Which among MeX, RCH2X, R2CHX and R3CX is most
and NaBr are soluble
reactive towards SN2 reaction?
(d) The three halogens differ considerably in their
(a) MeX (b) RCH2X
electronegativity
(c) R2CHX (d) R3CX
HALOALKANES AND HALOARENES 407
39. Isopropyl chloride undergoes hydrolysis by 47. Which of the following will have a mesoisomer also?
(a) SN1 mechanism (a) 2, 3- Dichloropentane
(b) SN2 mechanism (b) 2, 3-Dichlorobutane
(c) SN1 and SN2 mechanisms (c) 2-Chlorobutane
(d) Neither SN1 nor SN2 mechanism (d) 2-Hydroxypropanoic acid
40. Tertiary alkyl halides are practically inert to substitution by 48. Which of the following compounds is optically active ?
SN2 mechanism because of (a) CH3CHClCOOH (b) CH3CH2CH2CH3
(a) steric hindrance (b) inductive effect (c) (CH3)2CHOH (d) (CH3)3CCl
(c) instability (d) insolubility 49. Racemic compound has
41. Which of the following is the correct order of decreasing (a) equimolar mixture of enantiomers
SN2 reactivity? (b) 1 : 1 mixture of enantiomer and diastereomer
(a) R2CHX > R3CX > RCH2X (c) 1 : 1 mixture of diastereomers
(b) RCHX > R3CX > R2CHX (d) 1 : 2 mixture of enantiomers
(c) RCH2X > R2CHX > R3CX 50. An organic molecule necessarily shows optical activity if it
(d) R3CX > R2CHX > RCH2X. (a) contains asymmetric carbon atoms
(X is a halogen) (b) is non-polar
42. The reaction is described as (c) is non-superimposable on its mirror image
CH3 (CH2 )5 OH

(CH2)5CH3 (d) is superimposable on its mirror image
C – Br HO – C 51. Optically active isomers but not mirror images are called
H H (a) enantiomers (b) mesomers
CH3 CH3 (c) tautomers (d) diastereomers
(a) SE2 (b) SN1 52. Which of the following alkyl halides is used as a methylating
(c) SN2 (d) SN0 agent?
43. Which of the following is an optically active compound ? (a) C 2 H 5 Br (b) C6 H 5Cl
(a) 1-Butanol (b) 1-Propanol
(c) 2-Chlorobutane (d) 4-Hydroxyheptane (c) CH3I (d) C 2 H 5Cl
44. An important chemical method to resolve a racemic mixture 53. Mg reacts with RBr best in
makes use of the formation of (a) C2H5OC2H5 (b) C6H5OCH3
(a) a meso compound (b) enantiomers (c) C6H5N(CH3)2 (d) Equally in all the three
(c) diasteromers (d) racemates 54. 2-Bromopentane is heated with potassium ethoxide in
45. The process of separation of a racemic modification into ethanol. The major product obtained is
d and -enantiomers is called (a) 2-ethoxypentane (b) pentene-1
(a) Resolution (b) Dehydration (c) trans-2-pentene (d) cis-pentene-2
(c) Revolution (d) Dehydrohalogenation 55. An alkyl halide reacts with metallic sodium in dry ether. The
46. Which of the following pairs of compounds are enantiomers? reaction is known as :
(a) Frankland’s reaction (b) Sandmeyer’s reaction
CH3 CH3
(c) Wurtz reaction (d) Kolbe’s reaction
(a) HO H HO H
and 56. When 2-bromobutane reacts with alcoholic KOH, the
H OH HO H
CH3 CH3
reaction is called
(a) halogenation (b) chlorination
CH3 CH3 (c) hydrogenation (d) dehydrohalogenation
(b) H OH HO H 57. An alkyl halide by formation of its Grignard reagent and
and
HO H H HO
heating with water yields propane. What is the original alkyl
CH3 CH3
halide ?
CH3 CH3 (a) Methyl iodide (b) Ethyl iodide
(c) H OH HO H
and (c) Ethyl bromide (d) Propyl bromide
HO H HO H
CH3 CH3 58. An organic compound A (C4H9 Cl) on reaction with
Na/diethyl ether gives a hydrocarbon which on
CH3 CH3 monochlorination gives only one chloro derivative, then A
H OH H OH is
(d) and
HO H H OH (a) tert-butyl chloride (b) sec-butyl chloride
CH3 CH3 (c) isobutyl chloride (d) n-butyl chloride
EBD_7207
408 HALOALKANES AND HALOARENES
59. Elimination of bromine from 2-bromobutane results in the AlCl 3
(c) C6H5Cl + CH3CH2Cl C6H5C2H5 + Cl2
formation of –
(a) predominantly 2-butyne AlCl3
(d) C6H5CH3+CH3CH2Cl C6H5C2H5 + CH3Cl
(b) predominantly 1-butene
68. On sulphonation of C6 H 5Cl
(c) predominantly 2-butene
(d) equimolar mixture of 1 and 2-butene (a) benzene sulphonic acid is formed
60. Isobutyl magnesium bromide with dry ether and ethyl (b) metachlorobenzene sulphonic acid is formed
alcohol gives : (c) orthochlorobenzene sulphonic acid is formed
(a) CH3 CHCH 2 OH and CH3CH 2 MgBr (d) ortho and para chlorobenzene sulphonic acids are
| formed
CH3
69. C – Cl bond of chlorobenzene in comparison to C – Cl bond
(b) CH3 CHCH3 and MgBr(OC2 H5 ) in methyl chloride is
| (a) Longer and weaker (b) Shorter and weaker
CH3 (c) Shorter and stronger (d) Longer and stronger
(c) CH3 CHCH CH 2 and Mg(OH)Br 70. Which of the following is not used in Friedel-Crafts reaction?
| (a) N–Phenyl acetanilide (b) Bromobenzene
CH3
(c) Benzene (d) Chlorobenzene
(d) CH3 CHCH3 and CH3CH 2OMgBr 71. Which one of the following is most reactive towards
| nucleophilic substitution reaction?
CH3
(a) CH 2 CH Cl (b) C6 H 5Cl
61. Reactivity order of halides for dehydrohalogenation is
(a) R – F > R – Cl > R – Br > R –I (c) CH 3CH CH Cl (d) ClCH 2 CH CH 2
(b) R –I > R – Br > R – Cl > R – F 72. Which one is most reactive towards SN1reaction ?
(c) R –I > R – Cl > R – Br > R – F (a) C6 H5CH(C6 H5 )Br
(d) R – F > R –I > R – Br > R – Cl
(b) C6 H5CH(CH3 )Br
62. Arrange the following alcohols in increasing order of their
reactivity towards the reaction with HCl. (c) C6 H5C(CH3 )(C6 H5 )Br
(CH3)2CH-OH (1), (CH3)3C-OH (2), (C6H5)3C-OH (3) (d) C6 H5CH 2 Br
(a) 1 < 2 < 3 (b) 2 < 1 < 3 73. Chlorobenzene can be prepared by reacting aniline with :
(c) 3 < 1 < 2 (d) 2 < 3 < 1 (a) hydrochloric acid
63. Which of following can be used as solvent for grignard (b) cuprous chloride
reagent ? (c) chlorine in presence of anhydrous aluminium chloride
(a) H2O (b) C2H5OH (d) nitrous acid followed by heating with cuprous chloride
(c) CH3OH (d) C2H5OC2H5 74. Aryl halides can not be prepared by the reaction of aryl
64. Benzene reacts with CH3Cl in the presence of anhydrous alcohols with PCl3, PCl5 or SOCl2 because
AlCl3 to form : (a) phenols are highly stable compounds.
(a) chlorobenzene (b) benzylchloride (b) carbon-oxygen bond in phenols has a partial double
(c) xylene (d) toluene bond character.
65. Chlorobenzene reacts with Mg in dry ether to give a (c) carbon-oxygen bond is highly polar
compound (A) which further reacts with ethanol to yield (d) all of these
(a) Phenol (b) Benzene 75. Haloarenes are ortho and para directing due to
(c) Ethylbenzene (d) Phenyl ether (a) Resonance in aryl halide
66. Benzene reacts with n-propyl chloride in the presence of (b) – I effect of halogen atom
anhydrous AlCl3 to give (c) + I effect of halogen atom
(a) 3 – Propyl – 1 – chlorobenzene (d) Both (a) and (b)
(b) n-Propylbenzene 76. Chloropicrin is obtained by the reaction of
(c) No reaction (a) steam on carbon tetrachloride
(d) Isopropylbenzene (b) nitric acid on chlorobenzene
67. Which of the following is the example of Friedal Craft (c) chlorine on picric acid
reaction ? (d) nitric acid on chloroform
AlCl3 77. Which of these can be used as moth repellant
(a) C6 H6 CH3CH 2 Cl C6 H5 C 2H5 HCl
(a) Benzene hexachloride (b) Benzal chloride
(b) C6H5OH + CH3CH2Cl AlCl3
C6H5C2H5 + HOCl (c) Hexachloroethane (d) Tetrachloroethane
HALOALKANES AND HALOARENES 409
78. CFxCly [where x + y = 4]. These compounds are not used 86. Which of the following is responsible for depletion of the
because ozone layer in the upper strata of the atmosphere?
(a) these are fluorocarbons (a) Polyhalogens (b) Ferrocene
(b) these are difficult to synthesise (c) Fullerenes (d) Freons
(c) they deplete ozone layer 87. Haloforms are trihalogen derivatives of
(d) None of the these (a) Ethane (b) Methane
79. Freon (dichlorodifluoro methane) is used (c) Propane (d) Benzene
(a) as local anaesthetic 88. Which of the following compounds is used as a refrigerant ?
(b) for dissolving impurities in metallurgical process (a) Acetone (b) CCl4
(c) in refrigerator (c) CF4 (d) CCl2F2
(d) in printing industry 89. Chloroform is used as :
80. Use of chlorofluorocarbons is not encouraged because (a) Fire extinguisher (b) Industrial solvent
(a) They are harmful to the eyes of people that use it (c) Refrigerant (d) Insecticide
(b) They damage the refrigerators and air conditioners 90. AgNO3 does not give precipitate with CHCl3 because
(c) They eat away the ozone in the atmosphere (a) CHCl3 does not ionise in water
(d) They destroy the oxygen layer (b) CHCl3 does not react with AgNO3
81. Which of the following is used in fire extinguishers (c) CHCl3 is chemically inert
(a) CH4 (b) CHCl3 (d) None of these
(c) CH2Cl2 (d) CCl4 91. When chloroform is exposed to light and air, it forms
82. Solvent which is used in the synthesis of chlorofluorocarbons (a) chlorine gas (b) methyl chloride
(a) iodoform (b) chloroform (c) phosgene gas (d) carbon tetrachloride
(c) carbon tetrachloride (d) methylene chloride
83. Uses of dichloromethane is 92. Cl
(a) paint remover C–CCl3
(b) solvent in drugs manufacturing
(c) metal cleansing and finishing solvent Cl H
(d) All of the above
84. Trichloroacetaldehyde, CCl3CHO reacts with chlorobenzene The above structural formula refers to
in presence of sulphuric acid and produces: (a) BHC (b) DNA
(c) DDT (d) RNA
Cl 93. If chloroform is left open in air in the presence of sunlight, it
gives
(a) carbon tetrachloride (b) carbonyl chloride
(c) mustard gas (d) lewisite
(a) Cl C Cl 94. Full name of DDT is
(a) 1, 1, 1-trichloro-2, 2-bis(p-chlorophenyl) ethane
H
(b) 1, 1-dichloro-2, 2-diphenyl trimethylethane
OH (c) 1, 1-dichloro-2, 2-diphenyl trichloroethane
(b) Cl (d) None of these
C Cl
95. Freon(s) is/are :
Cl (a) CClF3 (b) CFCl3
(c) CCl2F2 (d) All of these
(c) Cl CH Cl 96. Freon-12 is commonly used as
CCl3 (a) insecticide (b) refrigerant
(c) a solvent (d) a fire extinguisher
Cl 97. Freon used as refrigerant is
(a) CF2 == CF2 (b) CH2F2
(d) Cl C Cl
(c) CCl2F2 (d) CF4
CH2Cl 98. Methylene chloride can be used as
(a) paint remover
85. Chloroform on treatment with conc. HNO3 gives
(b) propellant in aerosols
(a) Chloropicrin (b) Nitromethane (c) solvent in manufacturing of drugs
(c) Picric acid (d) Acetylene (d) All of these
EBD_7207
410 HALOALKANES AND HALOARENES
99. Which of the following are the harmful effects of methylene (ii) Among isomeric dihalobenzenes the para-isomers
chloride? have higher melting point than their ortho and meta-
(a) Impaired hearing and vision isomers.
(b) Dizziness, nausea and tingling (iii) The isomeric dihalobenzene have large difference in
(c) Skin burning their boiling and melting points
(d) All of these (iv) The isomeric dihalobenzene have nearly same boiling
point.
STATEMENT TYPE QUESTIONS (a) (i), (ii) and (iii) are correct
100. Read the following statements and choose the correct (b) (i) and (iii) are correct
option. (c) (ii) and (iv) are correct
(i) The general formula of alkyl halides is CnH2n+1 X (d) (i), (ii) and (iv) are correct
(ii) The general formula of aryl halides is Cn Hn–1 X 104. Read the following statements and choose the correct code
(iii) In alkyl halides halogen atom(s) is attached to sp2 (i) SN2 reactions follows a second order kinetics whereas
hybridised carbon atom SN1 reactions follows the first order kinetics
(iv) In aryl halides halogen atom(s) is attached to sp2 (ii) SN1 reactions follows the second order kinetics
hybridised carbon atom. whereas SN2 follows the first order kinetics
(a) (i), (ii) and (iii) are correct (iii) SN2 reactions take place in a single step whereas
(b) (i), (ii) and (iv) are correct SN1 reactions take place in a two steps
(c) (ii), (iii) and (iv) are correct (iv) Tertiary alkyl halides are least reactive towards SN2
(d) (i), (ii), (iii) and (iv) are correct reactions but we show high reactivity towards SN1
101. Following statements are given regarding the preparation reaction.
of aryl halides from toluene. Read the following statements (a) (ii) and (iv) are correct
and choose the correct option. (b) (i) and (iii) are correct
(i) Aryl chlorides and bromides can be easily prepared (c) (i), (ii) and are correct
by this method. (d) (ii), (iii) and (iv) are correct
(ii) The ortho and para isomers formed in the reaction can 105. Read the following statements and choose the correct
not be separated easily due to small difference in their option.
melting point. (i) SN1 reactions are carried out through formation of
(iii) Reactions with iodine are reversible in nature and carbocation as an intermediate.
require the presence of an oxidising agent. (ii) SN1 reactions are two step reactions in which step 1 is
(iv) Fluoro compounds are not prepared by this method fast and irreversible.
due to low reactivity of fluorine. (iii) Step 1 involves breaking of C–Br bond which obtain
(a) (i) and (iii) are correct energy through solvation of halide.
(b) (ii) and (iv) are correct (iv) SN1 reactions are two step reactions in which step 2 is
(c) (i), (ii), and (iii) are correct slow and reversible.
(d) All statements are correct (v) Allylic and benzylic halides show high reactivity
102. Read the following statements and choose the correct toward SN1 reactions.
option. (a) (i), (iii) and (v) are correct
(i) For the same alkyl group, the boiling points of alkyl (b) (ii), (iii) and (v) are correct
halides decreases in the order. (c) (i), (iii) and (iv) are correct
RI > RBr > RCl > RF (d) (i), (ii) and (iv) are correct
(ii) With the increases in size and mass of halogen atom, 106. Read the following statements and choose the correct code
the magnitude of van der Waal’s forces increases. (i) SN2 reaction proceed with complete stereochemical
(a) Both statements (i) and (ii) are correct inversion.
(b) Statement (i) is correct and (ii) is incorrect (ii) SN1 reaction proceed with recimisation.
(c) Statement (ii) is correct and (i) is incorrect (iii) A dextrorotatory compound rotate the plane polarised
(d) Both statement (i) and (ii) are incorrect light to the left.
103. Read the following statements and choose the correct (iv) A laevorotatory compound rotate the plane polarised
answer light to the right
(i) The boiling points of isomeric haloalkanes decrease (a) TFTT (b) TTFF
with increase in branching. (c) FFTT (d) TFTF
HALOALKANES AND HALOARENES 411
110. Match the columns
C2H5 C2H5 C2H5 Column - I Column - II
Y H Y H
107. H Cl2 / UV light
Y X CH3 Y CH3 (A) C2H6 (p) Finkelstein reaction
CH3
C2H5Cl
B A
(B) C6H5NH2 (q) Free radical substitution
Y
NaNO 2 HCl/Cu 2 Cl 2

A+B 273 278K

C6H5Cl
For the reaction scheme given above some statements are
given. Read the statements and choose the correct option. (C) CH3Cl + NaI (r) Swarts reaction
(i) If (A) is the only compound obtained, the process is CH3I + NaCl
called retention of configuration. (D) CH3 – Br + AgF (s) Sandmeyer’s reaction
(ii) If (B) is the only compound obtained, the process is CH3F + AgBr
called inversion of configuration. (a) A – (q), B – (s), C – (p), D – (r)
(iii) If a 50 : 50 mixture of the above two is obtained then (b) A – (q), B – (r), C – (p), D – (s)
the process is called racemisation (c) A – (r), B – (p), C – (s), D – (q)
(iv) The product A + B is optically active (d) A – (s), B – (r), C – (p), D – (q)
(a) TTTF (b) TFTF 111. Match the columns
Column - I Column - II
(c) TTFF (d) TFFT
(A) Chloroform (p) Antiseptic
108. Which of the statement(s) is/are true, regarding following (B) Iodoform (q) Insecticide
reaction? (C) Trichloromethane (r) Anesthetic
(D) DDT (s) Propellant
R R
Nu – –
(a) A – (s), B – (p), C – (r), D – (q)
R' CBr R' CNu + Br (b) A – (r), B – (p), C – (s), D – (q)
R'' R'' (c) A – (q), B – (s), C – (p), D – (q)
(i) The reaction involves the formation of transition state. (d) A – (r), B – (s), C – (p), D – (q)
112. Match the columns.
(ii) Higher the nucleophilic character of the nucleophile,
Column - I Column - II
faster will be the reaction.
(Haloalkane/arene) (Applications)
(iii) The product is always optically inactive. (A) Iodoform (p) CF4
(a) (ii) only (b) (ii) and (iii) (B) BHC (q) Antiseptic
(c) (i), (ii) and (iii) (d) Neither (i), (ii) nor (iii) (C) Freon - 14 (r) Moth repellant
(D) Halothanes (s) Inhalative anesthetic
MATCHING TYPE QUESTIONS (E) p-dichlorobenzene (t) Termite pesticide
(a) A – (q), B – (s), C – (t), D – (r), E – (p)
109. Match the columns (b) A – (q), B – (t), C – (p), D – (s), E – (r)
Column - I Column - II (c) A – (r), B – (s), C – (q), D – (p), E – (t)
(A) CH2 = CH – CH2Cl (p) Gem-dichloride (d) A – (p), B – (r), C – (t), D – (q), E – (s)
(B) CH2 = CHX (q) Vinylic halide 113. Match the columns
Column-I Column-II
(C) CH3CHCl2 (r) Dichloride
(A) Chloramphenicol (p) Goiter
(D) CH2Cl CH2Cl (s) Allylic halide (B) Thyroxine (q) Surgery
(a) A – (r), B – (q), C – (p), D – (s) (C) Chloroquine (r) Typhoid
(b) A – (q), B – (p), C – (s), D – (r) (D) Halothane (s) Malaria
(a) A – (q), B – (p), C – (s), D – (r)
(c) A – (s), B – (q), C – (p), D – (r)
(b) A – (r), B – (p), C – (s), D – (q)
(d) A – (r), B – (p), C – (s), D – (q) (c) A – (s), B – (r), C – (q), D – (p)
(d) A – (p), B – (s), C – (q), D – (r)
EBD_7207
412 HALOALKANES AND HALOARENES

ASSERTION-REASON TYPE QUESTIONS 122. The catalyst used in the preparation of an alkyl chloride by
the action of dry HCl on an alcohol is
Directions : Each of these questions contain two statements, (a) Anhydrous AlCl3 (b) FeCl3
Assertion and Reason. Each of these questions also has four (c) Anhydrous ZnCl2 (d) Cu
alternative choices, only one of which is the correct answer. You
have to select one of the codes (a), (b), (c) and (d) given below. 123. CH 3 CH 2 C H CH 3 obtained by chlorination of
|
(a) Assertion is correct, reason is correct; reason is a correct Cl
explanation for assertion. n-butane, will be
(b) Assertion is correct, reason is correct; reason is not a (a) l-form (b) d-form
correct explanation for assertion
(c) Meso form (d) Racemic mixture
(c) Assertion is correct, reason is incorrect
124. The number of possible enantiomeric pairs that can be
(d) Assertion is incorrect, reason is correct.
produced during monochlorination of 2-methylbutane is
114. Assertion : SN2 reaction of an optically active aryl halide
(a) 2 (b) 3
with an aqueous solution of KOH always gives an alcohol
with opposite sign of rotation. (c) 4 (d) 1
Reason : SN2 reactions always proceed with inversion of 125. The number of structural and configurational isomers of a
configuration. bromo compound, C5H9Br, formed by the addition of HBr
115. Assertion : Alkylbenzene is not prepared by Friedel-Crafts to 2-pentyne respectively are
alkylation of benzene. (a) 1 and 2 (b) 2 and 4
Reason : Alkyl halides are less reactive than acyl halides. (c) 4 and 2 (d) 2 and 1
116. Assertion : Exposure of ultraviolet rays to human causes 126. Which of the following reagent produces pure alkyl halides
the skin cancer, disorder and disrupt the immune system. when heated with alcohols?
Reason : Carbon tetrachloride is released into air it rises to (a) PCl5 (b) PCl3
atmosphere and deplets the ozone layer. (c) SOCl2 (d) dry HCl
117. Assertion : CHCl3 is stored in dark bottles. 127. If C5H12 undergoes reaction with chlorine in the presence
Reason : CHCl3 is oxidised in dark. of sunlight, only one product is formed, than reactant is
118. Assertion : CCl4 is not a fire extinguisher. (a) 3, 3-dimethylpropane (b) 2, 3-dimethylpropane
Reason : CCl4 is insoluble in water.
(c) 1, 3-dimethylpropane (d) 2, 2,-dimethylpropane
CRITICAL THINKING TYPE QUESTIONS 128. Hydrocarbon (CH3)3CH undergoes reaction with Br2 and
Cl2 in the presence of sunlight, if the reaction with Cl is
119. The IUPAC name of the compound shown below is highly reactive and that with Br is highly selective so
Cl no.of possible products respectively is (are)
(a) 2, 2 (b) 2, 1
(c) 1, 2 (d) 1, 1
129. Possible major product formed in the reaction of
neopentylalcohol with HCl is
Br
(a) 2 -chloro-2-methylbutane
(a) 2-bromo-6-chlorocyclohex-1-ene (b) 2, 2 -dimethyl 1-chloropropane
(b) 6-bromo-2-chlorocyclohexene (c) 2 -chloro -3-methylbutane
(c) 3-bromo-1-chlorocyclohexene (d) 3, chloro -3-methylbutane
(d) 1-bromo-3-chlorocyclohexene
130. Fluorobenzene (C6H5F) can be synthesized in the laboratory
120. A compound is formed by substitution of two chlorine for
(a) by direct fluorination of benzene with F2 gas
two hydrogens in propane. The number of possible isomeric
compounds is (b) by reacting bromobenzene with NaF solution
(a) 4 (b) 3 (c) by heating phenol with HF and KF
(c) 5 (d) 2 (d) from aniline by diazotisation followed by heating the
121. Which one of the following is not an allylic halide? diazonium salt with HBF4
(a) 4-Bromopent-2-ene 131. Which chloride is least reactive with the hydrolysis point
(b) 3-Bromo-2-methylbut-1-ene of view?
(c) 1-Bromobut-2-ene (a) CH3Cl (b) CH3CH2Cl
(d) 4-Bromobut-1-ene (c) (CH3)3CCl (d) CH2 = CH – Cl
HALOALKANES AND HALOARENES 413
132. In a SN2 substitution reaction of the type (a) Primary < secondary < Tertiary
R Br Cl DMF
R Cl Br (b) Primary > Secondary > Tertiary
which one of the following has the highest relative rate ? (c) Primary > Secondary < Tertiary
(a) CH3 – CH2 – CH2Br (b) (d) Primary < Secondary > Tertiary
CH3 CH CH 2 Br
| 139. The correct order of reactivity of the halides, ethyl
CH3 chloride (I) iso-propyl chloride (II) and benzyl chloride
(III) in SN1 reaction is
CH (a) I > II > III (b) III > II > I
| 3
(c) II > I > III (d) I > III > II
(c) CH3 C CH 2 Br (d) CH3CH2Br
| 140. Which of the following represents correct set of ambident
CH3 nucleophiles?
(a) CN– and NH3 (b) CN–, NO2
133. Which will undergo SN2 reaction fastest among the following
halogen compounds? (c) OH–, RO– (d) CN–, OH–
(a) CH3CH2F (b) CH3CH2Cl 141. Which of the following statements is correct?
(c) CH3CH2Br (d) CH3CH2I (a) S N 2 reactions of optically active halides are
134. Consider the following bromides : accompanied by inversion of configuration.
(b) S N 1 reactions of optically active halides are
Me Me
Me Br Me accompanied by racemisation.
Br Br (c) Carbocation formed in SN1 reaction is sp2 hybridized.
(A) (B)
(d) All of the above.
The correct order of SN1 reactivity is 142. The replacement of chlorine of chlorobenzene to give phenol
(a) B > C > A (b) B > A > C requires drastic conditions, but the chlorine of 2,
(c) C > B > A (d) A > B > C 4-dinitrochlorobenzene is readily replaced since,
135. Which one is most reactive towards SN 1 reaction ? (a) nitro groups make the aromatic ring electron rich at
(a) C 6 H 5 CH(C 6 H 5 )Br ortho/para positions
(b) nitro groups withdraw electrons from the meta position
(b) C6 H5CH(CH3 )Br
of the aromatic ring
(c) C6 H5C(CH3 )(C6 H5 )Br
(c) nitro groups donate electrons at meta position
(d) C6 H5CH 2 Br
(d) nitro groups withdraw electrons from ortho/para
136. Consider the reactions : positions of the aromatic ring
(i) C 2 H5OH
(CH 3 )2 CH CH 2Br 143. A set of compounds in which the reactivity of halogen atom
(CH 3 )2 CH CH 2OC 2H 5 HBr in the ascending order is
C 2H 5O (a) chlorobenzene, vinyl chloride, chloroethane
(ii) (CH 3 ) 2 CH CH 2 Br
(b) chloroethane, chlorobenzene, vinyl chloride
(CH 3 )2 CH CH 2OC 2 H 5 Br (c) vinyl chloride, chlorobenzene, chloroethane
The mechanisms of reactions (i) and (ii) are respectively : (d) vinyl chloride, chloroethane, chlorobenzene
(a) SN1 and SN2 (b) SN1 and SN1 144. Aryl halides are extremely less reactive towards nucleophilic
(c) SN2 and SN2 (d) SN2 and SN1 substitution than alkylhalides. Which of the following
137. The organic chloro compound, which shows complete accounts for this ?
stereochemical inversion during a SN2 reaction, is (i) Due to resonance in aryl halides.
(a) (C2H5)2CHCl (b) (CH3)3CCl
(ii) In alkyl halides carbon atom in C–X bond is sp2
(c) (CH3)2 CHCl (d) CH3Cl hybridised whereas in aryl halides carbon atom in C–X
138. Under certain conditions an alkyl halide reacts with base to bond is sp3 hybridized.
give an alkene and HCl [Elimination Reaction] for example
(iii) Due to stability of phenyl cation.
R – CH2 – CH2 – Cl R – CH = CH2 + HCl
(iv) Due to possible repulsion there are less chances of
The extent of these reactions depends on the structure of
nucleophile to approach electron rich arenes.
alkyl halides (e.g. primary, secondary or tertiary). The relative
extent to which such reactions take place is in the order (a) (i), (ii) and (iv) (b) (i), (ii) and (iii)
(of haloalkanes) : (c) (i) and (iv) (d) (ii), (iii) and (iv)
EBD_7207
414 HALOALKANES AND HALOARENES

145. The organic compound used as feedstock in the synthesis 149. Exposure of CCl4 causes
of chlorofluorocarbons is (a) Liver cancer in human
(a) CH2Cl2 (b) CHCl3 (b) Damage to nerve cells
(c) CH3Cl (d) CCl4 (c) Coma, unconsciousness
146. CCl4 is well known fire extinguisher. However after using it (d) All of these
to extinguish fire, the room should be well ventilated. This
150. Chloroform cannot be prepared from which of the following?
is because
(a) CH3OH (b) C2H5OH
(a) It is flammable at higher temperatures
(c) CH3CHO (d) (CH3)2CO
(b) It is toxic
151. Which one of the following has antiseptic property?
(c) It produces phosgene by reaction with water vapour
at higher temperatures (a) Dichloromethane (b) Trifluoromethane

(d) It is corrosive (c) Triiodomethane (d) Tetrachloromethane

147. In which part of the atmosphere, does the freon remain 152. Chronic chloroform exposure may cause damage to liver
unchanged ? and kidney, due to the formation of

(a) Stratosphere (b) Troposphere (a) phosgene (b) methylene chloride

(c) Mesosphere (d) Thermosphere (c) methyl chloride (d) carbon tetrachloride

148. Natural ozone layer is unbalanced due to 153. The spatial arrangement of four groups around a central
carbon atom is tetrahedral and if all the substituents
(a) cloudiness of poisonous gases
attached to that a carbon are different. Such a carbon is
(b) presence of rain in the atmosphere called ______.
(c) initiation of radical chain by freon (a) asymmetric carbon (b) stereocentre
(d) All of the above (c) chiral (d) All of these
HALOALKANES AND HALOARENES 415

14. (c) For the same alkyl group, the boiling points of alkyl
FACT / DEFINITION TYPE QUESTIONS
halides decrease in the order :
1. (d) Neohexyl chloride is a primary halide as in it Cl-atom is RI RBr RCl RF
attached to a primary carbon. This is because with the increase in size and mass of
halogen atom, the magnitude of van der Waal's forces
CH3
increases.
CH3 C CH2 CH2Cl 15. (a)
16. (a) C2H6 UV light
Cl2 C2 H5Cl+HCl
CH3
17. (b)
2. (b) CHCl 2 CH 2 Cl 18. (d) Ethylene dichloride can be prepared by adding HCl to
| | ethylene glycol (CH2OH. CH2OH).
CH3 CH 2 Cl
19. (b) When ethyl alcohol is treated with PCl 5 , then ethyl
(gem-dihalide) (vic-dihalide)
3. (b) Gem-dihalides are those in which two halogen atoms chloride is formed.
are attached on the same carbon atom. CH 3 CH 2 OH + PCl 5
4. (a)
CH 3 CH 2 Cl + HCl + POCl 3
1 1 20. (c) The best method for the conversion of an alcohol into
CH3 CH3
1 | | 2 1 an alkyl chloride is reaction of the alcohol with thionyl
5. (b) CH3 CH C CH 2 CH 3 chloride (SOCl2) in the presence of pyridine.
3
4
|
Cl R – OH + SOCl2 Pyridine RCl + SO2 + HCl
3-chloro-2-3-dimethylpentane SO2 and HCl being gases escape leaving behind pure
1 alkyl halide.
CH 3
| 21. (a) Boiling point of CH3I is 42°C which indicates that it is
6. (b) 2
H 3C C Cl liquid at room temperature. CH3I is larger molecule so
| it has stronger vander Waal’s force of attraction than
3 CH
3
others.
IUPAC name : 2-Chloro-2-methylpropane. 22. (c) In preparation of an alkyl chloride by the action of dry
Br HCl, the catalyst generally used is anhydrous ZnCl2.
| 23. (a)
7. (a) CH3 CH 2 C CH Cl
2 24. (c)
4 3
2 - bromo - 1 - chloro but - 1 - ene + –
NH2 N NCl Cl
8. (d) CH 2 CH CH 2Cl
(3 chloro 1 propene)
HNO2 Cu2Cl2
2
9. (a) Isopropyl chloride CH3 CH CH3 chlorine atom is HCl Sandmeyer’s reaction
(Diazotization)
|
Cl 25. (d) Para-di-chlorobenzene has most symmetrical structure
attached to 2° carbon atom. than others. It is found as crystalline lattice form,
10. (a) 11. (d) therefore, it has highest melting point (52°C) due to
12. (c) Because of the small size of F, the C–F bond is strongest symmetrical structure.
in CH3F. Cl
13. (b) CH3Cl has higher dipole moment than CH3F due to
much longer C–Cl bond length than the C–F bond.
The much longer bond length of the C–C bond
outweighs the effect produced by lower
electronegativity of Cl than that of F. Cl
EBD_7207
416 HALOALKANES AND HALOARENES
Para-chlorobenzene 39. (c) Isopropyl chloride, being 2° alkyl halides, can undergo
SN1 as well as SN2 mechanism.
acetone
26. (b) R X NaI R I NaX 40. (b) Due to steric hindrance tertiary alkyl halide do not
Soluble in react by S N2 mechanism they react by S N 1
(CH 3OH, Me 2CO)
mechanism. SN2 mechanisam is followed in case of
primary and secondary alkyl halides of
R X NaI R I NaX
CH3 – X CH3 – CH2X (CH3)2 – CH.X (CH3)3– C–X
Insoluble in
(CH 3OH, Me 2CO) 41. (c) In SN2 mechanism transition state is pentavalent. Thus
bulky alkyl group will be sterically hindered and smaller
(where X = Cl or Br) alkyl group will favour the SN2 mechanism. So the
27. (a) CH3COOAg Br2
CS2
CH3Br AgBr CO 2 decreasing order of reactivity of alkyl halides is
RCH2X > R2CHX > R3CX
28. (d) In nucleophilic substitution, a nucleophile provides
42. (c) Inversion in configuration occurs in SN2 reactions.
an electron pair to the substrate and the leaving group
departs with an electron pair. H
|
43. (c) H3C CH 2 C* CH3
|
These are usually written as S N (S stands for Cl
substitution and N for nucleophilic) and are common
in aliphatic compounds especially in alkyl halides and The compound containing a chiral carbon atom i.e., (a
acyl halides. carbon atom which is attached to four different atoms
29. (c) SN1 reactions involve the formation of carbocations, is known as a chiral carbon atom) is optically active.
hence higher the stability of carbocation, more will be A s
reactivity of the parent alkyl halide. Thus tertiary 2-chlorobutane contains a chiral C* atom hence it is
carbocation formed from (c) is stabilized by two phenyl
optically active.
groups and one methyl group, hence most stable.
44. (c) Diastereomers since they have different melting points,
Dry
30. (c) CH3 CH 2 I Mg CH 3 CH 2 MgI boiling points, solubilities etc.
Ether
Ethyl magnesium
iodide 45. (a)
46. (b) Compound which are mirror image of each other and
31. (d) Chlorobenzene does not undergo hydrolysis by SN1
are non superimposable are termed as enantiomers.
mechanism because in this halogen is present on sp2
hybridised carbon atom, such halogens are relatively CH3 CH3
inert. H OH HO H
32. (a) For a given alkyl group, the order of reactivity is HO and H OH
H
R – I > R – Br > R – Cl > R – F CH3 CH3
increasing bond energy
These are enantiomers
decreasing halogen reactivity. 47. (b) The compound has two similar assymmetric
This order depends on the carbon-halogen bond C-atoms. It has plane of symmetry and exist in meso
energy; the carbon-fluorine bond energy is maximum form.
and thus fluorides are least reactive while carbon-
iodine bond energy is minimum hence iodides are most
reactive. Plane of symmetry
33. (c) More stable the carbocation, more reactive will be the
parent alkyl halide towards SN1 reaction.
3° > Benzyl > Allyl > 2° > 1° > methyl Meso - 2, 3 dichlorobutane
34. (d) SN1 reaction involves carbocation which are planar 48. (a) Compounds having chiral carbon atom are optically
(sp2 hybridised) and thus can be attacked on either active.
face of the carbon.
H
35. (d) Weaker the C–X bond, greater is the reactivity. |
36. (a) Only 1° alkyl halides (i.e. CH3Br) undergo SN2 reaction. CH3 C* COOH (* is chiral carbon atom)
37. (b) |
Cl
38. (a) 1° Alkyl halides (having least steric hindrance at
-carbon atom) are most reactive towards SN2 reaction. 49. (a) A mixture of equal amounts of the two enantiomers is
called a racemic mixture.
HALOALKANES AND HALOARENES 417
50. (c) 51. (d) Br
52. (c) C2H5Br and C 2 H5Cl are ethylating agents, while |
Alc. KOH
C6H5Cl is inert. 59. (c) CH3 CH CH 2 CH3
53. (a) Although all the three compounds can be used for CH3 CH CH CH3 HBr
preparing Grignard reagents, diethyl ether is
The formation of 2-butene is in accordance to
considered as the best because it provides electron
Saytzeff’s rule. The more substituted alkene is formed
pairs to Mg of the reagent fully for coordination, in
in major quantity.
case of C6H5OCH3 and C6H5N(CH3)2 electron pair
on O and N are partialy delocalised over the benzene C2 H5OH
60. (b) (CH3)2CHCH2MgBr (CH3)2CHCH3
and hence are less available for coordination with Mg.
OC2H5
R O(C2H5)2
Mg + Mg
X O(C2H5)2 Br
61. (b) The order of atomic size of halogens decrease in the
54. (c) Potassium ethoxide is a strong base, and 2- bromopentane order I > Br > Cl > F. On moving down a group atomic
is a 2º bromide, so elimination raction predominates size increases. Further the bond length of C-X bond
OC 2 H 5 decreases in the order
CH 3 CH (Br )CH 2 CH 2 CH 3
C – I > C – Br > C – Cl > C – F
CH 3 CH CHCH 2 CH 3 CH 2 CHCH 2 CH 2 CH 3 and hence the bond dissociation energy decreases in
Pentene - 2(major) trans Pentene 1(min or ) cis
the order
Since trans- alkene is more stable than cis. Thus R – F > R – Cl > R – Br > R – I
trans-pentene -2 is the main product. hence R – I being a weakest bond break most easily.
55. (c) Wurtz reaction : It involves the interaction of two hence R – I is most reactive.
molecules of an alkyl halide (preferably bromide or 62. (a) Alkylhalide formation in the reaction of alcohol with
iodide) with metalic sodium in presence of dry ether to HCl undergoes SN1 reaction in which formation of
form symmetrical alkanes containing double the number the carbocation as intermediate occurs. Stability of
of carbon atoms present in the alkyl halide. For example, carbocation is greatest for (C6H5)3 C+ due to
Dry ether resonance effect, and stability of tertiary carbocation
R X 2Na X R R R 2NaX
Alkyl halide Alkane is greater than the secondary carbocation hence the
option (a) shows the correct order.
Br 63. (d) Except (d) all contain abstractable proton
|
56. (d) alc KOH CH3
CH 3 CH CH 2 CH 3

CH 2 CH CH 2 CH 3 Anhyd.
64. (d) + CH3Cl + HCl
In this reaction both hydrogen and halogen atom has AlCl3
been removed so it is known as dehydro halogenation Toluene
reaction. Mg
65. (b) C6 H 5 Cl C6 H 5 MgBr
57. (d) CH3CH 2 CH 2 Br Mg CH3CH2CH2MgBr CH 3CH 2OH
(Pr opyl Bromide) C6 H 6 CH 3CH 2OMgBr
CH3CH 2CH 2 MgBr + H2O CH3CH2CH3 + CH 3
Anhyd.
|
OH 66. (d) C6 H 6 CH3CH 2 CH 2Cl C6 H 5 CH CH 3
AlCl3 Isopropyl benzene
Mg 67. (a) Friedel-Craft’s reaction is mainly applied on benzene.
Br 3 AlCl
C6H6 + CH3CH2Cl C6H5C2H5 + HCl
CH3 CH3 68. (d) On sulphonation of chlorobenzene, ortho and para
58. (a) CH3 – C – Cl + 2Na + Cl – C – CH3 chlorobenzene is formed because – Cl group is para
CH3 CH3 and ortho directing.
t-Butyl chloride
Cl Cl Cl
Wurtz
Rxn

SO3 H
CH3 CH3 CH3 CH3
CH3 – C – C –CH2Cl Mono
CH3– C – C – CH3
H 2S 2O 7 +
Clorination
Fuming H 2SO4
CH3CH3 CH 3
CH3

SO3 H
EBD_7207
418 HALOALKANES AND HALOARENES
69. (c) Due to resonance in chlorobenzene.
70. (a) N-Phenylacetanilide, C6H5N(C6H5)COCH3, Cl
Cl
precipitates out to a complex with anhydrous AlCl3. 84. (c) Cl 3C–C=O + conc. H2SO4
Cl3C–CH
71. (d) More the stability of the carbocation, higher will be H Cl
Cl Cl
the reactivity of the parent chloride. Allyl chloride >
Vinyl chloride > Chlorobenzene DDT
72. (c) SN1 reactions involve the formation of carbocations,
hence higher the stability of carbocation, more will be 85. (a) Cl3C H HO NO 2 Cl3C NO 2
reactivity of the parent alkyl halide. Thus tertiary Chloropicrin (used
as an insecticide)
carbocation formed from (c) is stabilized by two phenyl
groups and one methyl group, hence most stable. 86. (d) Chlorofluorocarbons, e.g. CF 2 Cl 2 , CHF 2 Cl 2 ,
HCF2CHCl2. These are non-inflammable colourless
HONO CuCl
73. (d) C6H5NH2 C6H5N2Cl C6H5Cl and stable upto 550ºC. These are emitted as propellants
HCl
74. (b) This method is not applicable for the preparation of in aerosol spray, cans refrigerators, fire fighting reagents
aryl halides because the C–O bond in phenol has a etc. They are chemically inert and hence do not react
partial double bond character and is difficult to break with any substance with which they come in contact
being stronger than a single bond. and therefore float through the atmosphere and as a
result enter the stratosphere. There they absorb UV-rays
75. (d) Due to resonance, the electron density increases more
at ortho- and para-positions than at meta-positions. and due to this they produce free atomic chlorine which
Further, the halogen atom because of its – I effect has results decomposition of ozone which cause depletion
some tendency to withdraw electrons from the benzene of ozone layer.
ring. As a result, the ring gets somewhat deactivated 87. (b) Haloform compounds with the formula CHX3, where
as compared to benzene and hence the electrophilic X is a halogen atom.
substitution reactions in haloarenes occur slowly and Haloforms are trihalogen derivatives of methane.
require more drastic conditions as compared to those Example : Chloroform CHCl3.
in benzene. 88. (d) Under ordinary conditions freon is a gas. Its boiling
76. (d) Chloropicrin is nitrochloroform. It is obtained by the point is –29.8°C. It can easily be liquified. It is chemically
nitration of chloroform with HNO3. intert. It is used in air-conditioning and in domestic
HNO 3
refrigeratiors for cooling purposes (as refrigerant)
HCCl 3 O 2 NCCl 3 89. (b) Chloroform (CHCl3) is used as industrial solvent.
Chloroform Chloropicr in
90. (a) Chloroform is an organic compound which does not
Chloropicrin is a liquids, poisonous and used as an ionise in water. Since it can not provide Cl–, therefore,
insecticide and a war gas it is not precipitated with AgNO3.
77. (c) 78. (c) 91. (c) When chloroform is exposed to light it is oxidised to a
79. (c) Freon (CCl2F2) is an odourless, non-corrosive, non poisonous gas known as phosgene.
toxic gas which is stable even at high temperatures
2 CHCl 3 2O 2 2 COCl 2 Cl 2
and pressures. It has low b.p. low specific heat and
can be easily liquified by applying pressure at room 92. (c) 93. (b)
temperature. It is therefore, widely used as refrigerant
(cooling agent) in refrigerators and air conditioners. Cl
80. (c) Chlorofluorocarbon is used in air-conditioners and in
94. (a) CCl3CH
domestic refrigerators for cooling purposes. Its main
drawback is this, it is responsible for ozone depletion. Cl
81. (d) Its vapours are non-inflammable (i.e. do not catch fire). (DDT)
Hence used as fire extinguishers under the name
pyrene. 95. (d) Freons are chlorofluorocarbons.
82. (c) Tetrachloromethane (carbon tetrachloride) is also used CClF3, CFCl3 and CCl2F2, all are freons.
as feedstock in the synthesis of chlorofluorocarbon 96. (b)
and other pharmaceutical manufacturing and general 97. (c) Freons are chlorofluorocarbon.
solvents etc. 98. (d) 99. (d)
83. (d) Dichloromethane is widely used as solvent as a paint
remover, as a propellant in aerosols and as a process STATEMENT TYPE QUESTIONS
solvent in the manufacture of drugs. It is also used as 100. (b) In alkyl halides halogen atom(s) is attached to sp3
a metal cleaning and finishing solvent. hybridised carbon atom.
HALOALKANES AND HALOARENES 419
101. (a) Aryl chlorides and bromides can be easily prepared 108. (d) tert-Alkyl halides undergo SN1 reactions, hence they
by electrophilic substitution of arenes with chlorine involve the formation of quite stable carbocations, and
and bromine respectively in the presence of Lewis not the transition state. In S N1 reactions, the
acid catalysis like iron or iron (III) chloride. nucleophile is not involved in rate determining (first)
The ortho and para isomers can be easily separated step, hence its stronger or weaker nature does not
due to large difference in their melting points. influence the reaction rate. In SN1, the product has
Reactions with iodine are reversible in nature and more percentage of the inverted configuration than
require the presence of an oxidising agent (HNO3, the retained configuration, i.e. only partial racemization
HIO4) to oxidise the HI formed during iodination. takes place, hence the product will be having some
Fluoro compounds are not prepared by this method optical activity.
due to high reactivity of fluorine.
102. (a) MATCHING TYPE QUESTIONS
103. (d) The boiling points of isomeric haloalkanes decrease
with increase in branching. For example, 2-bromo-2- 109. (c) In allylic halides hydrogen atom is bonded to sp3
methylpropane has the lowest boiling point among hybridized carbon atom. Whereas in vinylic halide,
the three isomers. hydrogen atom is bonded to sp2 hybridized carbon
CH3 atom.
CH3CHCl2 CH 2 CH 2
| | |
CH3CH 2CH 2CH 2Br CH3CH 2CH CH3 H3C C CH 3 Ethylidene chloride Cl Cl
| | (gem-dihalide) Ethylene dichloride (vic-dihalide)
Br Br
b.p. / K 375 364 346 110. (a) Alkyl iodides are often prepared by the reaction of
Boiling points of isomeric dihalobenzenes are very alkyl chlorides/bromides with NaI in dry acetone. This
nearly the same. However, the para-isomers are high reaction is known as Finkelstein reaction.
melting as compared to their ortho and meta-isomers. R X NaI R I NaX
It is due to symmetry of para-isomers that fits in crystal X = Cl, Br
lattice better as compared to ortho- and meta-isomers. NaCl or NaBr thus formed is precipitated in dry
Cl Cl Cl acetone.
It facilitates the forword reaction according to le
Cl
chatelier’s principle. The synthesis of alkyl fluorides
is best accomplished by heating an alkyl chloride/
Cl bromide in the presence of a metallic fluoride such as
Cl AgF, Hg2F2, CoF2 or SbF3. The reaction is termed as
Swarts reaction.
b.p/K 453 446 448
H 3C Br AgF H 3C F AgBr
m.p.K 256 249 323
111. (b) 112. (b)
104. (b) SN2 reaction follow a 2nd order kinetic ie the rate
depends upon the concentration of both the reactants, 113. (b) Chloramphenicol, produced by soil microorganism is
where in SN1 reactions rate depends only upon the very effective for the treatment of typhoid fever. Our
concentration of only one reactants. body produces iodine containing hormone thyroxine,
The order of reactivity order of alkyl halides for SN2 the deficiency of which causes a disease called goiter.
reaction 3° > 2° > 1° and for SN1 reactions 3° < 2° < 1° Synthetic halogen compounds, viz chloroquine is used
105. (a) In SN1 reactions step 1 is slow and reversible and the for the treatment of malaria; halothane is used as an
slowest step is the rate determining step anaesthetic during surgery. Certain fully fluorinated
106. (b) If the compound rotates the plane polarised light to compounds are being considered as potential blood
the right, i.e., clockwise direction, it is called substitutes in surgery.
dextrorotatory (Greek for right rotating) or the d-form
and is indicated by placing a positive(+) sign before ASSERTION-REASON TYPE QUESTIONS
the degree of rotation. If the light is rotated towards
114. (d) Assertion is false, because aryl halides do not undergo
left (anticlockwise direction), the compound is said to
nucleophilic substitution under ordinary conditions.
be laevorotatory or the l-form and a negative (–) sign
This is due to resonance, because of which the carbon–
is placed before the degree of rotation.
chlorine bond acquires partial double bond character,
107. (a) If a 50 : 50 mixture of the (A) and (B) is obtained then
hence it becomes shorter and stronger and thus cannot
the process is called racemisation and the product is
optically inactive, as one isomer will rotate light in the be replaced by nucleophiles. However Reason is true.
direction opposite to another. 115. (c) Alkyl halides give polyalkylation products.
EBD_7207
420 HALOALKANES AND HALOARENES
116. (b) Carbon tetrachloride rises to atmosphere and deplete 124. (d) First draw possible different structures obtained on
the ozone layer. This depletion of ozone layer increases monochlorination of 2-methylbutane,
exposure of UV rays to human being which lead to CH3CH(CH3)CH2CH3.
increase of skin cancer, eye diseases and disorder with CH3
CH3
discruption of the immune system. | |
117. (c) CHCl3 is stored in dark bottles to prevent oxidation of (i) ClCH 2 * C HCH 2 CH3 (ii) CH 3 C CH 2 CH 3
CHCl3 in presence of sunlight. |
Optically active Cl
118. (d) CCl4 is used as a fire extinguisher. The dense, non
combustible vapours cover the burning substance and Optically inactive
prevents the availability of oxygen around burning CH 3 Cl CH 3
material. | |* |
(iii) CH 3 CH CH CH 3 (iv) CH 3 CH CH 2 CH2 Cl
CRITICAL THINKING TYPE QUESTIONS Optically active Optically inactive
Thus structures (i) and (iii) are optically active, each
Cl
has one chiral carbon; so each structure will give one
1 enantiomeric pair; thus total enantiomeric pairs will
2 be two.
119. (c)
125. (b) Addition of HBr of 2-pentyne gives two structural
3 Br isomers (I) and (II)
HBr
120. (c) The compound is C3H 6Cl 2 and the number of CH3 C C CH 2 CH 3
possible isomeric compunds is 5 CH3C(Br) CHCH 2 CH 3 CH3CH C(Br)CH 2CH3
asymmetric carbon atom (I) (II)
H H H H H H Each one of these will exist as a pair of geometrical
| | | | | | isomers. Thus, there are two structural and four
H C C C H H C C* C H
| | | | | | configurational isomers.
Cl H Cl Cl Cl H 126. (c) Thionyl chloride is preferred because the other two
(d.l pair)
products formed in the reaction are escapable gases.
H H H H Cl H Hence the reaction gives pure alkyl halides
| | | | | | ROH + SOCl2 R–Cl + SO2 + HCl
Cl C C C H H C C C H
| | | | | | 127. (d) This compound have only one type of hydrogen
Cl H H H Cl H available.
121. (d) 4-Bromobut-l-ene is not an allylic halide 128. (b) Chlorine atom is highly reactive so it will react with
all type of hydrogen available while the Br atom is
BrH 2C —CH 2 —CH CH 2
4 Bromobut 1 ene highly selective so it will react with that hydrogen
122. (c) Primary and secondary alkyl chlorides are prepared which give the highly stabilize tertiary alkyl radical
from the respective alcohols by using HCl gas and so only one product is formed.
anhydrous ZnCl2 (Groove’s process). CH3 CH3
129. (a) CH3 – C – CH2OH HCl CH3 – C – CH2 – OH2+
– –

– –

123. (d) Chlorination of n-butane taken place via free radical


formation i.e., Cl2 h • • CH3 CH3
Cl Cl
Neopentyl alcohol
Cl 2 / h CH3
CH3 — CH2 — CH2 — CH3 –H2O
– –

CH3 – C – CH2
CH3 CH3 CH3
H — C — Cl + Cl — C — H 1, 2 shift

C2H5 C2H5 +
d l CH3 – C – CH2.CH3

Racemic mixture CH3


50% d form + 50% l form
Cl• may attack on either side and give a racemic mixture CH3 – C = CH – CH3 Cl
– –

of 2 chloro butane which contain 50% d form and 50% CH3 CH3 – C – CH2 – CH3
l-form. 2-methyl-2-butene CH3
2, chloro-2-methyl butane
HALOALKANES AND HALOARENES 421
+ Since S N 1 reactions involve the formation of
NH2 N2 Cl–
carbocation as intermediate in the rate determining

Na NO 2 HCl HBF4 step, more is the stability of carbocation higher will
130. (d) be the reactivity of alkyl halides towards SN1 route.
0 5º diazotisation
Now we know that stability of carbocations follows
N2+BF4– F the order : 3° > 2° > 1°, so SN1 reactivity should also
follow the same order.
+ BF3+ N2 3° > 2° > 1° > Methyl (SN1 reactivity)
135. (c) SN1 reactions involve the formation of carbocations,
Benzene diazonium (Balz-Schiemann order of stability of carbocation is 3° > 2° > 1° hence
tetrafluoroborate reaction)
higher the stability of carbocation, more will be the
131. (d) CH 2 = CH - Cl reactivity of the parent alkyl halide. Moreover the
(Vinyl Chloride) tertiary carbocation formed from (c) is stabilized by
The halogen atom in vinyl chloride is not reactive as in two phenyl groups.
other alkyl halides. The non-reactivity of chlorine atom 136. (a) A strong nucleophile favours the SN2 reaction and a
is due to resonance stabilisation. The .p. on Cl-atom weak nucleophile favours the SN1 reaction.
can participate in delocalisation (Resonance) to give First reaction is SN1 reaction because C2H5OH is used
two canonical structure. as solvent which is a weak nucleophile.
Second reaction is SN2 reaction because C2H5O– is
Q +
CH 2 = CH - Cl ¾¾
® CH 2 - CH = C l strong nucleophile.
137. (d) SN2 reaction is favoured by small groups on the carbon
132. (d) The rate of SN2 substitution reaction is maximum in atom attached to halogen.
case of CH3CH2Br because SN2 mechanism is followed So, the order of reactivity is
in case of primary and secondary halides i.e., SN2 CH 3 Cl (CH 3 ) 2 CHCl (CH 3 )3CCl
reaction is favoured by small groups on the carbon
atom attached to halogens so (C 2 H 5 )2CHCl
CH3 CH2 Br > CH3 CH2 CH2 Br > SN2 reaction is shown to maximum extent by primary
halides. The only primary halides given is CH3Cl so
CH3 the correct answer is (d).
|
CH3– CH – Br > CH3 – C – Br 138. (a) Primary halide < Secondary halide < Tertiary halide.
| | 139. (b) Since SN1 reaction involve the formation of carbocation
CH3 CH3 as intermediate in the rate determining step. More stable
133. (d) Smaller the R group reactivity will be higher towards the carbocation, more is the reactivity of the halide
SN2 reaction. For alkyl halides containing similar alkyl toward SN1 route. As we know that the stability of the
group better will be the leaving group, more facile is carbocations decreases in the order :
the nucleophilic substitution reaction. Benzyl 2º > 1°.
Amongst the halide ions, the order in which the leaving Hence the correct order of stability is
groups depart follows the sequence : III > II > I
I– > Br – > Cl– > F– 140. (b) Cyanides and nitrites possess two nucleophilic
centres and are called ambident nucleophiles. Actually
It is because of this reason that the order of reactivity
cyanide group is a nucleophile in two different ways
of haloalkanes follows the sequence :
iodoalkanes > bromoalkanes > chloroalkanes > [ C N : C N ] . Similarly nitrite ion also
fluoroalkanes represents an ambident nucleophile with two different
134. (a)
+ –
points of linkage [ O N O]. The linkage through
Me Br ionisation +Br
Me oxygen results in alkyl nitrites while through nitrogen
(A) atom, it leads to nitroalkanes.
141. (d) In case of optically active alkyl halides, the product
Me Me formed as a result of SN2 mechanism has the inverted
ionisation Me
+ + Br – configuration as compared to the reactant. This is
Br
because the nucleophile attaches itself on the side
(B)
opposite to the one where the halogen atom is present.
Me In case of optically active alkyl halides, SN1 reactions
ionisation Me – are accompanied by racemisation. The carbocation
Me + Br
Me +
Br formed in the slow step being sp2 hybridised is planar
(C) (achiral).
EBD_7207
422 HALOALKANES AND HALOARENES
142. (d) —NO2 group is electron attractive group, so it is able SbCl5
145. (d) 3CCl4 2 SbF5 2SbCl3 3CCl 2 F2
to deactivate the benzene ring.
freon 12
Cl C,FeCl3
CCl 4 2H 2HCl CCl 2 F2
NO2 146. (c) Carbon tetrachloride vapours react with steam above
500° C to from phosgene, a poisonous gas.
500 C
NO2 CCl 4 H 2O COCl2 2HCl
phosgene
hence withdrawl of electrons from ortho and para 147. (a) In stratosphere, freon is able to initiate radical chain
position cause easy removal of –Cl atom due to reaction that can upset the natural ozone balance.
development of +ve charge on o- and p positions. 148. (c) In stratosphere freon is able, to initiate radical chain
143. (a) On the same basis as above reactions that can upset the natural ozone balance.
CH3CH2Cl > CH2 = CHCl > C6H5Cl 149. (d) Exposure of carbon tetrachloride causes liver cancer
in humans. The most common effects are dizziness,
144. (c) The carbon-halogen bonds of aryl halides are both
light headedness, nausea and vomiting which cause
shorter and stronger (due to possibility of resonance)
permanent damage to nerve cells. In severe cases,
than the carbon-halogen bonds of R–X and in this these effects can lead rapidly to stupor, coma,
respect as well as in their chemical behaviour, they unconsciousness or death.
resemble vinyl halides (CH2= CHX) more than alkyl oxidation
halides. 150. (a) CH 3CH 2OH + Cl2 CH 3CHO + 2HCl
Ethanol Ethanal

X CH 3CHO + 3Cl2 CCl 3 CHO + 3HCl


Ethanal Chloral
2CCl3CHO + Ca(OH)2
2CHCl 3 + (HCOO) 2 Ca
CH2= CH–X Chloroform Cal. formate

Aryl halides Vinyl halides CH 3COCH3 + 3Cl2 CCl 3COCH 3 + 3HCl


Acetone Trichloroacetone
Halogen attached to C is sp2 hybridised C, 2CCl3CO.CH3 + Ca(OH)2
C–X bond is shorter and stronger because of 2CHCl3 + (CH3COO)2Ca
partial double bond character due to
delocalisation of electrons on halogens 151. (c) Triiodomethane (CHI3) when comes in contact with
organic matter decomposes easily to free iodine which
CH2– X has antiseptic property.
152. (a) Cl Cl Cl Cl
C
H +[O]
Light and air
Cl C
Cl OH
CH = CH–X R–CH 2–X CH2 = CH–CH2–X Chloroform

Vinyl halides Alkyl halides Allyl halides Benzyl halides Cl


HCl+ C=O
Cl Phosgene
hybridised C, C–X
tronger because of Phosgene is an extremely poisonous gas.
153. (d)
25
ALCOHOLS, PHENOLS AND
ETHERS
FACT / DEFINITION TYPE QUESTIONS CH 3
|
1. The characteristic grouping of secondary alcohols is 9. The IUPAC name of CH 3 CH CH 2 C CH 3 is
(a) CH 2OH (b) CHOH | |
OH OH
| OH
(c) C OH (d) C (a) 1, 1-dimethyl-1, 3-butanediol
| OH (b) 2-methyl-2, 4-pentanediol
2. The compound HOCH 2 CH 2 OH is (c) 4-methyl-2, 4-pentanediol
(a) ethane glycol (b) ethylene glycol (d) 1, 3, 3-trimethyl-1, 3-propanediol
(c) ethylidene alcohol (d) dimethyl alcohol 10. Number of metamers represented by molecular formula
3. The structural formula of cyclohexanol is C4H10O is
(a) 4 (b) 3
CH 2 CH 2 (c) 2 (d) 1
11. Which of the following compounds is aromatic alcohol?
H2 C CHOH H2 C CHOH
(a) (b) OH CH2OH CH2OH OH
H2 C CH 2 H2 C CH 2
CH 2 CH 2
CH3 CH3
(A) (B) (C) (D)
CH 2 OH
(a) A, B, C, D (b) A, D
(c) (d) (c) B, C (d) A
12. How many alcohol(s) with molecular formula C4H10O are
4. Which of the following is dihydric alcohol ? chiral in nature?
(a) Glycerol (b) Ethylene glycol (a) 1 (b) 2
(c) Catechol (d) Resorcinol (c) 3 (d) 4
5. An example of a compound with functional group – O – is : 13. Give IUPAC name of the compound given below
(a) acetic acid (b) methyl alcohol
CH3 –CH – CH 2 – CH 2 –CH – CH 3
(c) diethyl ether (d) acetone | |
6. Butane-2-ol is Cl OH
(a) primary alcohol (b) secondary alcohol (a) 2-Chloro-5-hydroxyhexane
(c) tertiary alcohol (d) aldehyde (b) 2-Hydroxy-5-chlorohexane
7. Cresol has (c) 5-Chlorohexane-2-ol
(a) Alcoholic – OH (b) Phenolic – OH (d) 2-Chlorohexan-5-ol
(c) – COOH (d) – CHO
14. IUPAC name of m-cresol is ___________
8. How many isomers of C5H11OH will be primary alcohols ?
(a) 2-methylphenol (b) 3-chlorophenol
(a) 5 (b) 4
(c) 2 (d) 3 (c) 3-methoxyphenol (d) benzene-1, 3-diol
EBD_7207
424 ALCOHOLS, PHENOLS AND ETHERS

15. IUPAC name of the compound CH3 – CH – OCH3 is 23. Ethyl alcohol is industrially prepared from ethylene by
| (a) Permanganate oxidation
CH3 (b) Catalytic reduction
_______________ . (c) Absorbing in H2SO4 followed by hydrolysis
(a) 1-methoxy-1-methylethane (d) All the three
(b) 2-methoxy-2-methylethane 24. Sodium salt of benzene sulphonic acid on fusion with caustic
(c) 2-methoxypropane soda gives
(d) isopropylmethyl ether (a) Benzene (b) Phenol
16. Which of the following are benzylic alcohols? (c) Thiophenol (d) Benzoic acid
(i) C6 H5 – CH 2 – CH 2OH 25. Acid catalyzed hydration of alkenes except ethene leads to
the formation of
(ii) C6 H5 – CH 2 OH (a) primary alcohol
(b) secondary or tertiary alcohol
(iii) C 6 H5 – C H – OH
| (c) mixture of primary and secondary alcohols
CH3 (d) mixture of secondary and tertiary alcohols
26. Ethyl alcohol can be prepared from Grignard reagent by the
(iv) C6 H5 – CH 2 – C H – OH reaction of :
|
CH3 (a) HCHO (b) R2CO
(a) (i) and (ii) (b) (ii) and (iii) (c) RCN (d) RCOCl
(c) (i), (ii) and (iv) (d) (i) and (iv) 27. Isopropyl alcohol is obtained by reacting which of the
17. In which of the following structures hydroxyl group is following alkenes with concentrated H2SO4 followed by
attached to sp2 carbon atom? boiling with H2O?
(a) Ethylene (b) Propylene
OH (c) 2-Methylpropene (d) Isoprene
CH2OH CH
OH 28. Alkenes convert into alcohols by
(a) (b) (a) hydrolysis by dil. H 2SO 4
(b) hydration of alkene by alkaline KMnO4
OH CH(CH3)OH (c) hydrolysis by water vapours and conc. HNO3
CH3 (d) hydration of alkene by aqueous KOH
(c) (d) 29. Which of the following reacts with NaOH to give an alcohol?
(a) Propene (b) Butene
18. Which of the following is an example of unsymmetrical (c) Ethanal (d) Methanal
ether? 30. By which of the following methods alcohol can be prepared
(a) C2H5OC2H5 (b) C6H5OC6H5 in excellent yield?
(c) C6H5OC2H5 (d) CH3OCH3 (a) From alkenes
19. Which of the following will not form phenol or phenoxide ? (b) By hydroboration-oxidation
(a) C6H5N2Cl (b) C6H5SO3Na (c) From carbonyl compounds
(c) C6H5Cl (d) C6H5CO2H (d) From Grignard reagent
20. Benzyl alcohol is obtained from benzaldehyde by 31. Which of the following are used to convert RCHO into
(a) Fittig’s reaction (b) Cannizzaro’s reaction RCH2OH?
(c) Kolbe’s reaction (d) Wurtz’s reaction (i) H2/Pd
21. In the reduction (ii) LiAlH4
(iii) NaBH4
R CHO H2 RCH 2 OH
(iv) Reaction with RMgX followed by hydrolysis
the catalyst used is : (a) (i) and (ii) (b) (i), (ii) and (iii)
(a) Ni (b) Pd (c) (ii), (iii) and (iv) (d) (i) and (iii)
(c) Pt (d) Any of these 32. Commercially carboxylic acids are reduced to alcohols by
22. Ethylene reacts with Baeyer’s reagent to give converting them to the ______.
(a) ethane (b) ethyl alcohol (a) esters (b) aldehydes
(c) ethylene glycol (d) None of these (c) ketones (d) amines
ALCOHOLS, PHENOLS AND ETHERS 425
33. The hydrocarbon which produce phenol and acetone as a 42. Which of the following is correct ?
by product in the large quantity is (a) On reduction of any aldehyde, secondary alcohol is
formed
CH2CH3
CH3 (b) Reaction of vegetable oil with H2SO4 gives glycerine
(c) Sucrose on reaction with NaCl gives invert sugar
(a) (b) (d) Alcoholic iodine gives iodoform with NaOH
43. Which of the following is not true in case of reaction with
heated copper at 300°C?
CH3 CH3 (a) Phenol Benzyl alcohol
(b) Secondary alcohol Ketone
(c) H3C C CH3 (d) CH3 CH
(c) Primary alcohol Aldehyde
(d) Tertiary alcohol Olefin
44. Phenol is more acidic than alcohol because
(a) phenol is more stable than water
HNO2 (b) phenol is aromatic and alcohol is aliphatic
34. In the reaction, RNH 2 ROH H 2 O C ;
273 278K (c) phenoxide ion is resonance stabilised
C is (where R = C6H5) (d) None of these
(a) NH3 (b) N2 45. Acidity of phenol is due to
(c) O2 (b) CO2 (a) hydrogen bonding
35. The correct order of boiling points for primary (1º), (b) phenolic group
secondary (2º) and tertiary alcohol (3º) is (c) benzene ring
(a) 1º > 2º > 3º (b) 3º > 2º > 1º (d) resonance stabilisation of its anion
(c) 2º > 1º > 3º (d) 2º > 3º > 1º 46. Which one of the following compounds has the most acidic
36. Alcohols of low molecular weight are nature?
(a) soluble in water
CH2OH OH
(b) soluble in water on heating (a) (b)
(c) insoluble in water
(d) insoluble in all solvents
OH
37. Which of the following has lowest boiling point ?
(a) p-Nitrophenol (b) m-Nitrophenol OH CH
(c) o-Nitrophenol (d) Phenol (c) (d)
38. Which statement is not correct about alcohol?
(a) Molecular weight of alcohol is higher than water 47. The ionization constant of phenol is higher than that of
(b) Alcohol of less no. of carbon atoms is less soluble in ethanol because :
water than alcohol of more no. of carbon atoms (a) phenoxide ion is bulkier than ethoxide
(c) Alcohol evaporates quickly (b) phenoxide ion is stronger base than ethoxide
(d) All of the above (c) phenoxide ion is stabilized through delocalization
39. Which one of the following alcohols is least soluble in (d) phenoxide ion is less stable than ethoxide
water? 48. Which one of the following on oxidation gives a ketone ?
(a) CH 3 OH (b) C 3 H 7 OH (a) Primary alcohol (b) Secondary alcohol
(c) Tertiary alcohol (d) All of these
(c) C 4 H 9 OH (d) C10 H 21OH 49. Primary and secondary alcohols on action of reduced copper
40. Methanol and ethanol are miscible in water due to give
(a) covalent character (a) Aldehydes and ketones respectively
(b) hydrogen bonding character (b) Ketones and aldehydes respectively
(c) oxygen bonding character (c) Only aldehydes
(d) None of these (d) Only ketones
41. If ethanol dissolves in water, then which of the following 50. When ethyl alcohol reacts with acetic acid, the products
would be observed formed are
(a) absorption of heat and contraction in volume (a) Sodium ethoxide + hydrogen
(b) emission of heat and contraction in volume (b) Ethyl acetate + water
(c) absorption of heat and increase in volume (c) Ethyl acetate + soap
(d) emission of heat and increase in volume (d) Ethyl alcohol + water
EBD_7207
426 ALCOHOLS, PHENOLS AND ETHERS
51. Which of the following compounds is oxidised to prepare 64. Ethyl alcohol exhibits acidic character on reacting it with
methyl ethyl ketone? (a) acetic acid (b) sodium metal
(a) 2-Propanol (b) l-Butanol (c) hydrogen chloride (d) acidic K2Cr2O7
(c) 2-Butanol (d) t-Butyl alcohol 65. For the reaction,
52. HBr reacts fastest with ZnCl 2
(a) 2-Mehtylpropan-1-ol C 2 H 5 OH HX C 2 H 5X H 2O
(b) 2-Methylpropene-2-ol the order of reactivity is
(c) propan-2-ol (a) HBr > HI > HCl (b) HI > HCl > HBr
(d) propan-1-ol (c) HI > HBr > HCl (d) HCl > HBr > HI
53. n-Propyl alcohol and isopropyl alcohol can be chemically 66. In the following reaction,
distinguished by which reagent? Conc . H 2SO 4
(a) PCl5 C 2 H 5 OH Z identify Z:
443 K
(b) Reduction
(a) CH2 = CH2 (b) CH3CH2OCH2CH3
(c) Oxidation with potassium dichromate
(c) CH3CH2–HSO4 (d) (CH3CH2)2SO4
(d) Ozonolysis
67. Chemical name of salol is
54. Lucas reagent is
(a) acetylsalicyclic acid (b) sodium salicylate
(a) Conc. HCl and anhydrous ZnCl2
(c) phenyl salicylate (d) methyl salicylate
(b) Conc. HNO3 and hydrous ZnCl2
68. Aspirin is an acetylation product of
(c) Conc. HCl and hydrous ZnCl2
(a) p-Dihydroxybenzene (b) o-Hydroxybenzoic acid
(d) Conc. HNO3 and anhydrous ZnCl2
(c) o-Dihydroxybenzene (d) m-Hydroxybenzoic acid
55. The compound which reacts fastest with Lucas reagent at
room temperature is 69. Phenol, when it first reacts with concentrated sulphuric acid
and then with concentrated nitric acid, gives
(a) Butan-1-ol (b) Butan-2-ol
(a) 2, 4, 6-trinitrobenzene (b) o-nitrophenol
(c) 2-Methyl propan-1-ol (d) 2-Methylpropan-2-ol
(c) p-nitrophenol (d) nitrobenzene
56. When phenol is treated with excess bromine water, it gives:
70. 3 moles of ethanol react with one mole of phosphorus
(a) m-Bromophenol (b) o- and p-Bromophenol
tribromide to form 3 moles of bromoethane and one mole of
(c) 2, 4-Dibromophenol (d) 2, 4, 6-Tribromophenol
X. Which of the following is X?
57. When phenol is heated with CHCl3 and alcoholic KOH when
(a) H3PO4 (b) H3PO2
salicyladehyde is produced. This reaction is known as
(c) HPO3 (d) H3PO3
(a) Rosenmund’s reaction (b) Reimer-Tiemann reaction
71. Methanol and ethanol can be distinguished by the following:
(c) Friedel-Crafts reaction (d) Sommelet reaction
(a) By reaction with metallic sodium
58. On distilling phenol with Zn dust, one gets :
(b) By reaction with caustic soda
(a) Toluene (b) Benzaldehyde + ZnO
(c) By heating with iodine and washing soda
(c) ZnO + benzene (d) Benzoic acid
(d) By heating with zinc and inorganic mineral acid
59. Phenols do not react with one of the following :
72. Monochlorination of toluene in sunlight followed by
(a) Alkali metals (b) Sodium hydroxide
hydrolysis with aq. NaOH yields.
(c) Potassium hydroxide (d) Sodium bi-carbonate
(a) o-Cresol (b) m-Cresol
60. In the reaction
(c) 2, 4-Dihydroxytoluene (d) Benzyl alcohol
NaOH CO2 HCl 73. What is the correct order of reactivity of alcohols in the
Phenol (A) 140
(B), here B is
following reaction?
(a) benzaldehyde (b) chlorobenzene
ZnCl 2
(c) benzoic acid (d) salicylic acid R – OH HCl R – Cl H 2O
61. Dehydration of 2-butanol yields (a) 1° > 2° > 3° (b) 1° < 2° > 3°
(a) 1-butene (b) 2-butene (c) 3° > 2° > 1° (d) 3° > 1° > 2°
(c) 2-butyne (d) Both (a) and (b) 74. CH3CH2OH can be converted into CH3CHO by ________
62. Lucas test is done to differentiate between (a) catalytic hydrogenation
(a) alcohol and ketone (b) treatment with LiAlH4
(b) alcohol and aromatic ketones (c) treatment with pyridinium chlorochromate
(c) 1°, 2° and 3° alcohols (d) treatment with KMnO4
(d) None of these 75. Which of the following compounds will react with sodium
63. To distinguish between salicylic acid and phenol, one can hydroxide solution in water?
use (a) C6H5OH (b) C6H5CH2OH
(a) NaHCO3 solution (b) 5% NaOH solution (c) (CH3)3COH (d) C2H5OH
(c) neutral FeCl3 (d) bromine water
ALCOHOLS, PHENOLS AND ETHERS 427
76. Phenol is less acidic than_______________ . 82. Which enzyme converts glucose and fructose both into
(a) ethanol (b) o–nitrophenol ethanol ?
(c) o-methylphenol (d) o-methoxyphenol (a) Diastase (b) Invertase
77. Which of the following is most acidic? (c) Zymase (d) Maltase
(a) Benzyl alcohol (b) Cyclohexanol 83. An industrial method of preparation of methanol is :
(c) Phenol (d) m-Chlorophenol (a) catalytic reduction of carbon monoxide in presence of
78. Mark the correct increasing order of reactivity of the ZnO–Cr2O3
following compounds with HBr/HCl (b) by reacting methane with steam at 900ºC with a nickel
catalyst
CH2OH CH2OH CH2OH (c) by reducing formaldehyde with lithium aluminium
hydride
(d) by reacting formaldehyde with aqueous sodium
hydroxide solution
84. Ethyl alcohol is industrially prepared from ethylene by
NO2 Cl (a) Permanganate oxidation
(A) (B) (C) (b) Catalytic reduction
(c) Absorbing in H2SO4 followed by hydrolysis
(a) A < B < C (b) B < A < C (d) All the three
(c) B < C < A (d) C < B < A 85. ‘Drinking alcohol’ is very harmful and it ruins the health.
79. Arrange the following in increasing order of their acidity? ‘Drinking alcohol’ stands for
o–cresol(a), salicyclic acid(b), phenol(c) (a) drinking methyl alcohol
(a) c < a < b (b) b < c < a (b) drinking ethyl alcohol
(c) a < b < a (d) a < c < b (c) drinking propyl alcohol
80. In the reaction (d) drinking isopropyl alcohol
OH ONa 86. The fermentation reactions are carried out in temperature
range of
+ NaOH + H2O (a) 20-30°C (b) 30-40°C
(c) 40-50°C (d) 50-60°C
87. Ethanol is prepared industrially by
Phenol behaves as (a) hydration of ethylene (b) fermentation of sugar
(a) Bronsted base (b) Bronsted acid (c) Both the above (d) None of these
(c) Lewis acid (d) Lewis base 88. The fermentation of starch to give alcohol occurs mainly
81. In the given reaction with the help of
OH (a) O2 (b) air
(c) CO2 (d) enzymes
Na 2 Cr2O 7 A 89. In the commercial manufacture of ethyl alcohol from starchy
H 2SO 4
substances by fermentation method. Which enzymes
slipwise complete the fermentation reaction
(a) Diastase, maltase and zymase
A is (b) Maltase, zymase and invertase
(c) Diastase, zymase and lactase
OH O
(d) Diastase, invertase and zymase
90. Methyl alcohol is toxic. The reason assigned is
(a) (b) (a) it stops respiratory track
(b) it reacts with nitrogen and forms CN– in the lungs
OH O (c) it increses CO2 content in the blood
(d) it is a reduction product of formaldehyde
O CHO 91. In order to make alcohol undrinkable pyridine and methanol
are added to it. The resulting alcohol is called
(a) Power alcohol (b) Proof spirit
(c) (d) (c) Denatured spirit (d) Poison alcohol
92. Wine (alcoholic beverages) contains
CH2 (a) CH3OH (b) Glycerol
(c) C2H5OH (d) 2-propanol
EBD_7207
428 ALCOHOLS, PHENOLS AND ETHERS
93. Tonics in general contain (a) H-bonding in ethanol
(a) Ether (b) Methanol (b) H-bonding in dimethyl ether
(c) Ethanol (d) Rectified spirit (c) CH3 group in ethanol
94. Widespread deaths due to liquor poisoning occurs due to (d) CH3 group in dimethyl ether
(a) presence of carbonic acid in liquor 103. Ether which is liquid at room temperature is
(b) presence of ethyl alcohol in liquor (a) C2H5OCH3 (b) CH3OCH3
(c) presence of methyl alcohol in liquor (c) C2H5OC2H5 (d) None of these
(d) presence of lead compounds in liquor 104. Ether can be used
95. Select the incorrect statement about the fermentation.
(a) as a general anaesthetic
(a) When grapes are crushed, sugar and the enzyme come
in contact and fermentation starts (b) as a refrigerant
(b) Fermentation takes place in anaerobic conditions (c) in perfumery
(c) Carbon monoxide is released during fermentation (d) all of the above
(d) If air gets into fermentation mixture, the oxygen of air 105. Which of the following compound is soluble in ether?
oxidises ethanol to ethanoic acid which in turn (a) Oils and fats (b) Water
destroys the taste of alcoholic drinks (c) NaCl (d) PCl5
96. Denaturation of alcohol is the 106. An ether is more volatile than an alcohol having the same
(a) mixing of CuSO4 (a foul smelling solid) and pyridine molecular formula. This is due to
(to give the colour) to make the commercial alcohol (a) dipolar character of ethers
unfit for drinking (b) alcohols having resonance structures
(b) mixing of CuSO4 (to give the colour) and pyridine (a (c) inter-molecular hydrogen bonding in ethers
foul smelling solid) to make the commercial alcohol (d) inter-molecular hydrogen bonding in alcohols
unfit for drinking 107. Which of the following has strongest hydrogen bonding?
(c) mixing of Cu(OAc)2 and ammonia to make the
(a) Ethyl amine (b) Ethanal
commercial alcohol unfit for drinking
(c) Ethyl alcohol (d) Diethyl ether
(d) mixing of Cu(OAc) 2 and pyridine to make the
108. Oxygen atom in ether is
commercial alcohol unfit for drinking
97. Which one is formed when sodium phenoxide is heated (a) very active (b) replaceable
with ethyl iodide ? (c) comparatively inert (d) active
(a) Phenetole (b) Ethyl phenyl alcohol 109. The ether that undergoes electrophilic substitution
(c) Phenol (d) None of these reactions is
98. Williamson’s synthesis is used to prepare (a) CH3OC2H5 (b) C6H5OCH3
(a) acetone (b) diethyl ether (c) CH3OCH3 (d) C2H5OC2H5
(c) P.V.C. (d) bakelite 110. Diethyl ether on heating with conc. HI gives two moles of
99. The reaction of sodium ethoxide with ethyl iodide to form (a) ethanol (b) iodoform
diethyl ether is termed (c) ethyl iodide (d) methyl iodide
(a) electrophilic substitution 111. Methylphenyl ether can be obtained by reacting
(b) nucleophilic substitution (a) phenolate ions and methyl iodide
(c) electrophilic addition
(b) methoxide ions and bromobenzene
(d) radical substitution
100. Which of the following cannot be made by using (c) methanol and phenol
Williamson’s synthesis? (d) bromo benzene and methyl bromide
(a) Methoxybenzene 112. Diethyl ether can be decomposed by heating with
(b) Benzyl p-nitrophenyl ether (a) HI (b) NaOH
(c) Methyl tertiary butyl ether (c) Water (d) KMnO4
(d) Di-tert-butyl ether 113. The major organic product in the reaction,
101. The reaction given below is known as CH3 — O — CH(CH3)2 + HI Product is
C 2 H 5 ONa IC 2 H 5 C 2 H 5 OC 2 H 5 NaI (a) ICH2OCH(CH3)2 (b) CH 3O C( CH 3 ) 2
(a) Kolbe’s synthesis |
I
(b) Wurtz synthesis
(c) CH3I + (CH3)2CHOH (d) CH3OH + (CH3)2CHI
(c) Williamson’s synthesis
114. An aromatic ether is not cleaved by HI even at 525 K. The
(d) Grignard’s synthesis
compound is
102. Ethanol and dimethyl ether form a pair of functional isomers.
The boiling point of ethanol is higher than that of dimethyl (a) C6H5OCH3 (b) C6H5OC6H5
ether, due to the presence of (c) C6H5OC3H7 (d) Tetrahydrofuran
ALCOHOLS, PHENOLS AND ETHERS 429
115. When 2-methoxypropane is heated with HI, in the mole (iii) Only primary alcohols on dehydration give ethers.
ratio 1 : 1, the major products formed are (iv) Secondary and tertiary alcohols on dehydration give
(a) methanol and 2-iodopropane ethers having 2° and 3° carbon attached with O atom.
(b) methyl iodide and 2-propanol (a) TTFF (b) TFTF
(c) methyl iodide and 2-iodopropane (c) TTTF (d) FTTF
(d) methanol and 2-propanol 123. Which of the following statements are correct ?
116. Formation of diethyl ether from ethanol is based on a (i) In phenols, the —OH group is attached to sp 2
(a) dehydration reaction hybridised carbon of an aromatic ring
(b) dehydrogenation reaction (ii) The carbon – oxygen bond length (136 pm) in phenol
(c) hydrogenation reaction is slightly more than that in methanol
(d) heterolytic fission reaction (iii) Partial double bond character is due to the
117. The cleavage of an aryl-alkyl ether with cold HI gives : conjugation of unshared electron pair of oxygen with
(a) alkyl iodide and water the aromatic ring.
(b) aryl iodide and water (iv) sp2 hybridised state of carbon to which oxygen is
(c) alkyl iodide, aryl iodide and water attached.
(d) phenol and alkyl iodide (a) (i), (ii) and (v) (b) (i), (ii) and (iii)
118. Which of the following compounds is resistant to (c) (i), (iii) and (iv) (d) (i) and (iv)
nucleophilic attack by hydroxyl ions? 124. Which of the following statements are correct ?
(a) Methyl acetate (b) Acetonitrile (i) Ethanol mixed with methanol is called denatured
(c) Acetamide (d) Diethyl ether alcohol.
(ii) Excess of methanol in body may cause blindness.
STATEMENT TYPE QUESTIONS (iii) In the body methanol is oxidised to methanoic acid.
119. When an alcohol is prepared by reaction of ethylmagnesium- (iv) A methanol poisoned patient is treated by giving
bromide with 2–pentanone, product formed does not rotate intravenous injections of ethanoic acid.
plane polarised light. For this reaction which of the following (a) (i), (ii) and (iii) (b) (ii), (iii) and (iv)
statement(s) is/are correct ? (c) (i) and (v) (d) (i), (iii) and (iv)
(i). Product formed is achiral.
(ii) Racemic mixture is formed. MATCHING TYPE QUESTIONS
(a) Both statements (i) and (ii) are correct. 125. Match the columns
(b) Statement (i) is correct only. Column-I Column-II
(c) Statement (ii) is correct only. OH
(d) Both statements (i) and (ii) are incorrect.
120. Which of the following statements are correct ?
(A) (p) Quinol
(i) Alcohols react as nucleophiles in the reactions
involving cleavage of O–H bond.
(ii) Alcohols react as electrophiles in the reactions OH
involving cleavage of O–H bond. OH
(iii) Alcohols react as nucleophile in the reaction involving (B) (q) Phenol
cleavage of C–O bond.
(iv) Alcohols react as electrophiles in the reactions OH
involving C–O bond.
(a) (i) only (b) (i) and (iv)
(c) (ii) and (iii) (d) (ii) only (C) (r) Catechol
121. Which of the following are correct statement(s) ?
OH
(i) Polar nature of O–H bond is responsible for acidic OH
character of alcohols.
(ii) Acidic strength of alcohols follow the order 1° > 2° > 3°.
(iii) Alcohols are stronger acids than water.
(iv) Alcohols also react as Bronsted base. (D) (s) Resorcinol
(a) (i), (ii) and (iii) (b) (i), (ii) and (iv)
(c) (ii), (iii) and (iv) (d) (i), (iii) and (iv) OH
(a) A – (q), B – (p), C – (s), D – (r)
122. Read the following statements and choose the correct
(b) A – (r), B – (p), C – (s), D – (q)
option.
(c) A – (s), B – (q), C – (p), D – (r)
(i) Ethanol on dehydration at 443 K gives ethene
(d) A – (q), B – (r), C – (s), D – (p)
(ii) Ethanol on dehydration at 413 K gives diethyl ether
EBD_7207
430 ALCOHOLS, PHENOLS AND ETHERS
126. Match the columns 132. Assertion : In case of phenol, bromination takes place
Column-I Column-II even in absence of Lewis acid whereas bromination of
(A) Methanol (p) Conversion of phenol benzene takes place in presence of Lewis acid like FeBr 3.
to o-hydroxysalicylic Reason : – OH group attached to benzene ring is highly
acid deactivating.
(B) Kolbe’s reaction (q) Wood spirit 133. Assertion : ter - Butyl methyl ether is not prepared by the
(C) Williamson’s synthesis (r) Heated copper at 573 K reaction of ter-butyl bromide with sodium methoxide.
(D) Conversion of 2° alcohol (s) Reaction of alkyl halide Reason : Sodium methoxide is a strong nucleophile.
to ketone with sodium alkoxide 134. Assertion : Ethers behave as bases in the presence of
(a) A – (s), B – (r), C – (q), D – (p) mineral acids.
(b) A – (q), B – (s), C – (p), D – (r) Reason : Due to the presence of lone pairs of electrons on
(c) A – (q), B – (p), C – (s), D – (r) oxygen.
(d) A – (r), B – (q), C – (p), D – (s) 135. Assertion : With HI, anisole gives iodobenzene and methyl
127. Match the columns alcohol.
Column-I Column-II Reason : Iodide ion combines with smaller group to avoid
(A) Antifreeze used in (p) Methanol steric hindrance.
car engine 136. Assertion : With HI at 373 K, ter-butyl methyl ether gives
(B) Solvent used in (q) Phenol ter-butyl iodide and methanol.
perfumes Reason : The reaction occurs by SN2 mechanism.
(C) Starting material for (r) Ethleneglycol 137. Assertion : Ethyl phenyl ether on reaction with HBr form
picric acid phenol and ethyl bromide.
(D) Wood spirit (s) Ethanol Reason : Cleavage of C–O bond takes place on ethyl-
(a) A – (s), B – (q), C – (p), D – (r) oxygen bond due to the more stable phenyl-oxygen bond.
(b) A – (r), B – (s), C – (q), D – (p)
(c) A – (s), B – (q), C – (r), D – (p) CRITICAL THINKING TYPE QUESTIONS
(d) A – (p), B – (r), C – (q), D – (s) 138. Vinyl carbinol is
ASSERTION-REASON TYPE QUESTIONS (a) HO CH 2 CH CH 2

Directions : Each of these questions contain two statements, (b) CH 3C(OH ) CH 2


Assertion and Reason. Each of these questions also has four
(c) CH 3 CH CH OH
alternative choices, only one of which is the correct answer. You
have to select one of the codes (a), (b), (c) and (d) given below. (d) CH 3 C(CH 2 OH) CH 2
(a) Assertion is correct, reason is correct; reason is a correct
139. Propene, CH 3CH CH 2 can be converted into 1-propanol
explanation for assertion.
(b) Assertion is correct, reason is correct; reason is not a by oxidation. Indicate which set of reagents amongst the
correct explanation for assertion. following is ideal to effect the above conversion ?
(c) Assertion is correct, reason is incorrect. (a) KMnO4 (alkaline)
(d) Assertion is incorrect, reason is correct. (b) Osmium tetraoxide (OsO4/CH2Cl2)
128. Assertion : The bond angle in alcohols is slightly less (c) B6H6 and alk. H2O2
than the tetrahedral angle. (d) O3/Zn
Reason : In alcohols, the oxygen of –OH group is attached 140. The product of the following reaction is
to sp3 hybridized carbon atom. (i ) BH 3 / THF
129. Assertion : In Lucas test, 3º alcohols react immediately. (ii ) H 2 O 2 , OH
Reason : An equimolar mixture of anhyd. ZnCl2 and conc.
(a) 1-Pentanol (b) 2-Pentanol
HCl is called Lucas reagent.
130. Assertion : Reimer-Tiemann reaction of phenol with CCl4 (c) Pentane (d) 1,2-Pentanediol
in NaOH at 340 K gives salicyclic acid as the major product. 141. C6 H5 CH CHCHO X C6 H5 CH CHCH 2 OH .
Reason : The reaction occurs through intermediate In the above sequence X can be
formation of dichlorocarbene.
131. Assertion : Phenol is more reactive than benzene towards (a) H 2 / Ni (b) NaBH 4
electrophilic substitution reaction.
(c) K 2 Cr2 O7 / H (d) Both (a) and (b)
Reason : In the case of phenol, the intermediate carbocation
is more resonance stabilized.
ALCOHOLS, PHENOLS AND ETHERS 431
142. Which of the following reactions will yield phenol? 147. Mechanism of acid catalysed hydration reaction involves
Cl (i) Protonation of alkene to form carbocation by
electrophilic attack of H3O+
(i) fusion with NaOH at 300 atm (ii) Nucleophilic attack of water on carbocation.
(i) (ii)H 2O/H (iii) Deprotonation to form alcohol.
(a) (i) and (ii) (b) (i) and (iii)
NH2
(c) (i), (ii) and (iii) (d) (ii) and (iii)
(i)NaNO2 /HCl 148. Phenol is less acidic than
(ii) (ii)H 2O(Warming) (a) acetic acid (b) p-methoxyphenol
(c) acetylene (d) ethanol
(i)Oleum 149. The correct order of acid strength of the following
(iii) (ii)NaOH,(Heating)
compounds :
(iii)H
(A) Phenol (B) p–Cresol
Cl (C) m–Nitrophenol (D) p–Nitrophenol
(i)NaOH(aq.).298K/1atm (a) D > C > A > B (b) B > D > A > C
(iv) (ii)HCl (c) A > B > D > C (d) C > B > A > D
150. Arrange the following compounds in order of decreasing
(a) (i), (ii) and (iii) (b) (i) and (iii) acidity :
(c) (i), (iii) and (iv) (d) (ii), (iii) and (iv)
143. Hydration of styrene is carried out in presence of acid as OH OH OH OH
catalyst. The major product is.
(a) 1–hydroxy–2–phenylethane. ; ; ; ;
(b) 1–hydroxy–1–phenylethane.
(c) 2–hydroxy–1–phenylethane.
(d) 2–hydroxy–2–phenylethane. Cl CH3 NO2 OCH3
(I) (II) (III) (IV)
144. Which of the following reagents can be used for preparation
of cumene ? (a) II > IV > I > III (b) I > II > III > IV
(i) C6H6, Cl2, hv; Mg.THF; acetone. (c) III > I > II > IV (d) IV > III > I > II
(ii) C6H6, CH3CH2CH2Cl, AlCl3. 151. ClCH2CH2OH is stronger acid than CH3CH2OH because
(iii) C6H6, CH3CHClCH3, AlCl3. of:
(iv) C6H6, CH3CH2Cl, AlCl3;
(a) – I effect of Cl increases negative charge on O atom of
(a) (i) and (ii) (b) (ii) and (iii)
alcohol
(c) (i), (ii) and (iii) (d) (ii) and (iv)
145. The hydroboration of an alkene is carried out, then on (b) – I effect of Cl disperses negative charge on O atom to
oxidation with hydrogen peroxide, the alcohol so obtained produce more stable cation
is achiral. Possible structure of alkene is (are) : (c) – I effect of Cl disperses negative charge on O atom to
(i) 2, 3– dimethylbut–2–ene. produce more stable anion
(ii) 3, 4–dimethylbut –3–ene. (d) None of these
(iii) 2–methyl–but–2–ene. 152. Which one of the following compounds will be most readily
(iv) 2–methylpropene. attacked by an electrophile ?
(a) (i) and (iv) (b) (ii) and (iii) (a) Chlorobenzene (b) Benzene
(c) (iii) and (iv) (d) (i) and (iii)
(c) Phenol (d) Toluene
146. Which of the following shows structure of allylic alcohol?
153. Consider the following reaction:
(i) CH 2 CH CH 2OH
Zn dust CH3Cl
(ii) CH 2 CH OH Phenol X Y
Anhydrous AlCl3

(iii) CH 2 CH CH CH3 OH Alkaline KMnO4


Z
(iv) CH 2 CH C CH3 2OH The product Z is
(a) (i), (iii) and (iv) (b) (i), (ii) and (iv) (a) benzaldehyde (b) benzoic acid
(c) (ii), (iii) and (iv) (d) (i), (ii), (iii) and (iv) (c) benzene (d) toluene
EBD_7207
432 ALCOHOLS, PHENOLS AND ETHERS
154. When phenol is reacted with CHCl3 and NaOH followed by 162. Which one of the following will show the highest pH value ?
acidification, salicylaldehyde is obtained. Which of the (a) m–nitrophenol. (b) p–nitrophenol.
following species are involved in the above mentioned (c) o–nitrophenol. (d) Both (b) and (c).
reaction as intermediate? 163. Which of the following is most reactive towards aqueous
O OH HBr ?
(a) 1-Phenyl-1-propanol
H CHCl2
+ (b) 1-Phenyl-2-propanol
(a) CCl2 (b) (c) 3-Phenyl-1-propanol
(d) All are equally reactive
164. The major product of the reaction between tert-butyl chloride
and sodium ethoxide is
(c) (d) (a) 2-methylprop-1-ene (b) 1-butene
(c) 2-butene (d) ethene
155. The reagent used for dehydration of an alcohol is 165. In Williamson synthesis if tertiary alkyl halide is used than
(a) phosphorus pentachloride (a) ether is obtained in good yield
(b) calcium chloride (b) ether is obtained in poor yield
(c) aluminium oxide (c) alkene is the only reaction product
(d) sodium chloride (d) a mixture of alkene as a major product and ether as a
156. The alcohol which does not give a stable compound on minor product forms.
dehydration is 166. In the reaction:
(a) ethyl alcohol (b) methyl alcohol
(c) n-Propyl alcohol (d) n-Butyl alcohol CH3
|
157. A compound of the formula C4H10O reacts with sodium CH3 CH CH 2 O CH 2 CH3 HI
Heated
and undergoes oxidation to give a carbonyl compound
which does not reduce Tollen’s reagent, the original Which of the following compounds will be formed?
compound is
(a) CH3 CH CH3 CH 3CH 2 OH
(a) Diethyl ether (b) n-Butyl alcohol
|
(c) Isobutyl alcohol (d) sec-Butyl alcohol CH3
158. Which of the following fact(s) explain as to why
p-nitrophenol is more acidic than phenol? (b) CH3 CH CH 2 OH CH3CH3
I. –I Effect of nitro group. |
II. Greater resonance effect of p-nitrophenoxy group CH3
III. Steric effect of bulky nitro group
CH3
(a) I and II (b) I and III |
(c) II and III (d) II alone (c) CH3 CH CH 2 OH CH3 CH 2 I
159. In the following sequence of reactions,
P I2 Mg HCHO CH3
CH 3CH 2 OH A B |
ether
H O
2 (d) CH3 CH CH 2 I CH3CH 2OH
C D
the compound D is
HBr
(a) propanal (b) butanal 167. In the reaction OCH3 the products are
(c) n-butyl alcohol (d) n-propyl alcohol.
160. Which of the following species can act as the strongest
base? (a) OBr and CH4
(a) OH (b) OR
(c) OC6H5 (d) (b) Br and CH3Br
O

NO2
(c) Br and CH3OH
161. Which of the following reagents can be used to oxidise
primary alcohols to aldehydes?
(i) CrO3 in anhydrous medium.
(d) OH and CH3Br
(ii) KMnO4 in acidic medium.
(iii) Pyridinium chlorochromate. 168. An aromatic ether is not cleaved by HI even at 525 K. The
(iv) Heat in the presence of Cu at 573K. compound is
(a) (i) and (iii) (b) (ii), (iii) and (iv) (a) C6H5OCH3 (b) C6H5OC6H5
(c) (i), (iii) and (iv) (d) (i), (iii) and (iv) (c) C6H5OC3H7 (d) Tetrahydrofuran
ALCOHOLS, PHENOLS AND ETHERS 433

FACT / DEFINITION TYPE QUESTIONS 19. (d) Benzoic acid (C 6 H 5COOH) will not form phenol or
phenoxide.
1. (b) 2. (b)
20. (b) By heating benzaldehyde with conc. NaOH or KOH
CH 2 (Cannizzaro reaction).
H2 C CH OH
OH C6 H5 CHO NaOH
3. (a) or Benzaldehyde

H2 C CH 2 C6 H 5 CH 2 OH C6 H 5 COONa
Benzyl alcohol Sod. benzoate
CH 2
21. (d) Any one of Ni, Pt or Pd can be used in the reduction of
4. (b) Glycols are dihydric alcohols (having two hydroxyl aldehydes.
groups). Ethylene glycol is the first member of this
KMnO4
series. 22. (c) CH 2 CH 2 CH 2 CH 2
OH | |
CH 2OH
| Baeyer’s OH OH
CH 2 OH reagent Glycol
(Ethylene glycol)
This reaction is known as Baeyer’s test for unsaturation.
5. (c) Ethers contain the functional group – O – 23. (c) Ethylene is passed into concentrated sulphuric acid at
6. (b) CH3CH2CH(OH)CH3 – a secondary alcohol 75–80°C under pressure when a mixture of ethyl
7. (b) Cresol has phenolic group – OH hydrogen sulphate and diethyl sulphate is formed.
8. (b) Four primary alcohols of C5H11OH are possible. These H 2C CH 2 H 2SO 4
100 C
C2 H5 HSO 4
are: Ethylene
(i) CH 3 CH 2 CH 2 CH 2 CH 2 OH C 2 H 5 OH H 2SO 4
H 2O
(ii) CH 3 CH 2 CH CH 2 OH
| SO 3 Na O Na
CH 3
24. (b) NaOH + Na2SO3.H2O
(iii) CH 3 CH CH 2CH 2 OH 300°
|
CH 3 HCl

CH 3 OH
|
(iv) CH 3 C CH 2 OH + NaCl
|
CH 3 25. (b)
CH3 H H
| | CH3MgBr
|
9. (b)
5 4 3 2 1 26. (a) H C O H C OMgBr
CH3 CH CH 2 C CH3
| | |
OH OH CH 3
2-methyl- 2, 4-pentanediol. H
10. (b) C4H10O : (i) C2H5OC2H5 (ii) CH3OC3H7 H3O |
(iii) CH3OCH(CH3)2 H C OH
|
11. (c) 12. (a) 13. (c) 14. (a) 15. (c) CH 3
16. (b)
27. (b) Since the compound is formed by hydration of an
17. (c) In this structure –OH group is directly attached to
alkene, to get the structure of alkene remove a molecule
double bonded carbon atom i.e. sp2 hybridized carbon
of water from the alcohol.
atom.
H 2O
18. (c) If two groups attached to the oxygen atom are CH3 C H CH3 CH 2 CHCH 3
|
different then ethers are known as unsymmetrical or OH
Propylene
mixed ethers. Isopropyl alcohol
EBD_7207
434 ALCOHOLS, PHENOLS AND ETHERS

alk. KMnO 4 CH2–CH2 – – –


28. (b) CH 2 CH 2 H 2 O [O] 40. (b) Hydrogen bonding : O–H O–H O –H
OH OH | | |
Glycol
R H R
Alcohol Water Alcohol

Conc. H 2SO 4 41. (b) When ethanol dissolves in water then there is emission
CH 2 CH 2 H 2 O CH 3 CH 2 OH of heat and contraction in volume.
Ethane Ethanol
42. (d) C 2 H 5 OH 4 I 2 6 NaOH
29. (d) The aldehydes which do not have -hydrogen atom
react with NaOH when half of molecules are reduced CHI 3 HCOONa 5 NaI 5H 2O
Iodoform
to alcohol and other half of molecules are oxidised to
43. (a) When primary (1°) alcohols are treated with copper at
acid (Cannizzaro reaction).
300°C, then aldeh ydes are obtained by
NaOH dehydrogenation of alcohols. Similarly secondary (2°)
2 HCHO CH3OH + HCOONa
Methanal Methyl Sod. formate alcohols form ketone and alkene is obtained by
alcohol dehydration of tertiary (3°) - alcohols. But phenol does
30. (b) 31. (b) not respond to this test.
32. (a) Commercially, acids are reduced to alcohols by
converting them to the esters, followed by their R CH 2 CH2 OH
Cu
R CH 2 CHO + H 2
reduction using hydrogen in the presence of catalyst 1° alcohol 300 C Aldehyde
(catalytic hydrogenation).
H
R'OH H2
RCOOH RCOOR RCH 2OH R OH Cu
H Catalyst R C OH R – C – R + H2
300 C
33. (d) O
R
34. (b) A diazonium salt is formed by treating an aromatic
2° alcohol Ketone
primary amine with nitrous acid (NaNO2 + HCl) at
273-278 K. Diazonium salts are hydrolysed to phenols CH 3
by warming with water or by treating with dilute acids. Cu
R C OH R C CH 2 + H 2 O
300 C |
NH2 N2 Cl OH CH3 CH 3
NaNO2 H2O 3° alcohol Alkene
+ N2 + HCl
+HCl Warm Cu
C 6 H 5 – OH No reaction
Aniline Benzene diazonium 300 C
chloride
O– O O
35. (a) Among isomeric alcohols surface area decreases from –
1º to 2º to 3º alcohols and hence the boiling point.
..

44. (c) etc.


36. (a) The lower alcohols are readily soluble in water and the
. .–
solubility decreases with the increase in molecular Resonance stabilisation of phenoxide ion
weight. The solubility of alcohols in water can be (conjugate base of phenol)
explained due to the formation of hydrogen bond
Conjugate base of ethyl alcohol, i.e., C2H5O– does
between the highly polarised —OH groups present
not show resonance.
both in alcohol and water.
.. .. –
37. (c) o-Nitrophenol has intramolecular H-bonding. OH O
..

..
..

38. (b) The solubility of alcohols depend on number of


C-atoms of alcohols. The solubility of alcohols in water
45. (d) + H+
is decreased by increasing number of C-atoms of
alcohol. As resulting molecular weight increases, the Phenol Phenoxide ion
(resonance not (resonance stabilised)
polar nature of O – H bond decreases and hence significant)
strength of hydrogen bond decreases.
46. (b) Phenol is most acidic because its conjugate base is
39. (d) Solubility of alcohol in water decreases with increase stabilised due to resonance, while the rest three
in molecular mass due to increase in water repelling compounds are alcohols, hence, their corrosponding
alkyl part in alcohol. conjugate bases do not exhibit resonance.
ALCOHOLS, PHENOLS AND ETHERS 435

[O]
47. (c) C6 H 5 OH H 2 O C6 H5O H 3O CH 3CH 2 CH 2 OH
Phenoxide ion n propyl alcohal

The phenoxide ion is stable due to resonance. [O]


CH 3 CH 2 CHO CH 3 CH 2 COOH

O O O H3C [O] H3C
– CH – OH C=O
•• isopropyl alcohal Ketone

54. (a) Lucas reagent is conc. HCl + anhyd. ZnCl2.


••
– 55. (d) The rates of reaction with lucas reagent follows the
order.
3° alcohol > 2° alcohol > 1° alcohol
OH O– O since carbocations are formed as intermediate, more
– –
•• stable the carbocation, higher will be the reactivity of
– the parent compound (alcohol). 2-Methylpropan-2-ol
– generates a 3º carbocation, so it will react fastest; other
three generates either 1º or 2º carbocations.
The negative charge is delocalized in the benzene ring CH3
which is a stabilizing factor in the phenoxide ion and |
H
because of this reason ionization constant of phenol CH3 C OH
|
is higher whereas no resonance is possible in alkoxide CH3
ions (RO–)derived from alcohol. The negative charge 2-Methyl-2-propanol
is localized on oxygen atom in case of alcohols.
48. (b) Secondary alcohols on oxidation give ketones. CH3 CH3
| |
Note : – Primary alcohols from aldehydes. Br
CH3 C CH3 C Br
| |
R R CH3 CH3
[O]
CHOH C=O
R R 56. (d) OH OH
Isopropyl | Br Br
Ketone
alcohol 3Br2 (aq.)

49. (a) Alcohols are oxidized by removal of H2 in presence


of a heated metal catalyst (Cu) Br
2, 4,6-Tribromophenol
Cu
CH3CH 2OH CH3CHO H 2 Note : The –OH group in phenol, being activating
300
1 alcohol Aldehyde
group, facilitates substitution in the o- and p-positions.
H3 C OH OH
CH OH
Cu CHCl2
300 CH3CCH3 H 2
57. (b) + CHCl3
H3 C ||
2° Alcohol O
Ketone
OH OH
50. (b) C 2 H 5 OH CH 3 COOH CH 3 COO.C 2 H 5 H 2O CH(OH)2 CHO
51. (c) Secondary alcohols oxidise to produce kenone. 2KOH
–H2O
( O)
CH3CHOHCH2CH3 CH3COCH2CH3
Reimer-Tiemann reaction.
2-Butanol Ethyl methyl ketone 58. (c) When phenol reacts with Zinc dust, then benzene and
52. (b) Greater the stability of the intermediate carbocation, zinc oxide are formed
more reactive is the alcohol. Since 2-methylpropan-2-ol
OH
generates 3° carbocation, therefore, it reacts fastest
with HBr.
53. (c) Primary alcohol on oxidation give aldehyde which on + Zn + ZnO
further oxidation give carboxylic acid whereas
secondary alcohols give ketone.
Phenol Benzene
EBD_7207
436 ALCOHOLS, PHENOLS AND ETHERS
59. (d) Phenol does not react with NaHCO3.
70. (d) 3C 2 H 5OH PBr3 3C 2 H 5Br H 3PO 3
OH ONa [X ]
71. (c) Methanol and ethanol can be distinguished by heating
NaOH with iodine and washing soda
60. (d)
I2
Phenol C 2 H 5 OH CHI 3 HCOONa
Na 2 CO 3
Iodoform
(yellow ppt)
OH
COOH NaI 2NaOH H 2O
CO 2 HCl
I2
140 C CH3OH No reaction
Na 2CO3
Salicylic acid
This is also called iodoform test.
72. (d) 73. (c) 74. (c) 75. (a) 76. (b)
H 2O
61. (d) CH3CH 2 CHOH 77. (d) 78. (c)
|
CH3 OH OH OH
CH3 COOH
CH3 CH = CHCH3 + CH3CH2 CH = CH2
79. (d)
(Major product) (Minor product)
62. (c) Lucas test is used to distinguish between 1°, 2° and 3° o-cresol Salicylic acid Phenol
alcohols. Lucas reagent is anhy. ZnCl2 and HCl. Electron releasing groups (–CH3, –OCH3, –NCH3 etc)
3° alcohol + Lucas reagent turbidity immediately intensify the negative charge of phenoxide ion, i.e.,
2° alcohol + Lucas reagent turbidity after destablises it hence decrease ionization of parent
5 minutes phenol. Therefore decreases acidity while electron
1° alcohol + Lucas reagent No turbidity at donating groups (–NO2, –COOH, –CHO etc.) increases
room temperature. acidity.
63. (a) OH OH 80. (b) Alcohols and phenols are acidic in nature. In fact they
are Bronsted acids i.e., they can donate a proton to a
COOH stronger base (B:)
Salicyclic acid Phenol
(evolves CO 2 with NaHCO3) (No reaction with NaHCO3 )
– –
B+H–O–R B–H+ O–R
64. (b) Other options are acids, only Na metal is a base. Base Acid Conjugate Conjugate
65. (c) Reactivity increases as the nucleophilicity of the halide acid base
ion increases, i.e. I– > Br– > Cl–. 81. (b) Oxidation of phenol with chromic acid produces a
66. (a) In presence of acid, alcohols always form carbocations conjugated diketone known as benzoquinone. In the
as intermediates. presence of air, phenols are slowly oxidised to dark
67. (c) 68. (b) coloured mixtures containing quinones.
69. (b) Phenol on reaction with conc. H2SO4 gives a mixture 82. (c) Glucose and fructose obtained by hydrolysis of
of o- and p- products (i.e., –SO3H group, occupies sucrose, are converted into alcohol by enzyme zymase.
o-, p- position). At room temperature o-product is more
zymase
stable, which on treatment with conc. HNO3 will yield C 6 H12 O 6 2C 2 H 5 OH 2CO 2
o-nitrophenol. Cr2O3 – ZnO
OH OH OH 83. (a) CO + H2 + H2 CH3OH
300ºC
SO3H water gas
Methanol
Conc.H2SO4
+ 84. (c) Ethylene is passed into concentrated sulphuric acid at
o-product 75–80°C under pressure when a mixture of ethyl
SO3H hydrogen sulphate and diethyl sulphate is formed.
p-product 100 C
At room temperature o- product is more stable H 2C CH 2 H 2SO 4 C2 H5 HSO4
Ethylene
OH OH
SO3H NO2 C2 H5 OH H 2SO 4
Conc. HNO 3 H 2O
85. (b) 86. (a)
o- nitrophenol
ALCOHOLS, PHENOLS AND ETHERS 437
87. (c) Hydration of alkenes 101. (c) Preparation of ethers by reacting sodium ethoxide with
– alkyl halide is called Williamson synthesis.
CH 2 CH 2 H HSO 4 CH 3 – CH 2 HSO 4 102. (a) Due to H-bonding, the boiling point of ethanol is much
H 2O higher than that of the isomeric diethyl ether.
CH3 CH 2 HSO 4 CH 3 – CH 2 OH H 2SO 4
Boil 103. (c) CH3OCH3 and C2H5OCH3 are gases while C2H5OC2H5
Fermentation of sugar : (b. p. 308 K) is low boiling liquid.
Invertase 104. (d) Ether is used (i) as a general anaesthetic, (ii) as
C12 H 22O11 H 2O C2H12O6 C6H12O6 refrigerant since it produces cooling on evaporation,
Glucose Fructose
(iii) as solvent for oils, fats, resins etc. (iv) for providing
C6H12O 6
Zymase
2C2H 5OH 2CO 2 inert medium in Wurtz reaction, (v) for preparing
Glucose or Fructose Grignard reagent, (vi) in perfumery.
105. (a) Like dissolves like. Oils and fats, being covalent,
88. (d) Enzymes
Starch Alcohol dissolve in ether, a non-polar solvent.
Diastase 106. (d) Due to inter-molecular hydrogen bonding in alcohols
89. (a) 2(C6H10O5 )n nH 2O boiling point of alcohols is much higher than ether.
(from germinated barley)
Starch
107. (c) 108. (c)
n(C12H 22O11 ) 109. (b) Only alkyl aryl ethers e.g., C6 H5OCH3 undergoes
Maltose electrophilic substitution reactions.
Maltase
C12 H 22 O11 H 2 O (from yeast)
2C6 H12 O6 110. (c) C2H5OC2H5 + 2 HI 2 C2H5I + H2O
Glucose
111. (a) C6 H5O CH3 I C6 H5 OCH3 I
Zymase
C6 H12 O6 2C 2 H 5 OH 2CO 2
(from yeast) 112. (a) Ethers are readily cleaved by HI as follows :
90. (b)
H I
91. (c) Denaturing can also be done by adding 0.5% pyridine, R O R R O R ROH + RI.
petroleum naptha, CuSO4 etc. |
92. (c) H
93. (c) Tonics contain ethyl alcohol. 113. (c) In case of unsymmetrical ethers, the site of cleavage
94. (c) Due to presence of methyl alcohol in liquor. depends on the nature of alkyl group e.g.,
95. (c) The quantity of sugar increases and yeast grows on CH 3O CH(CH 3 ) 2 HI
373K
CH 3I (CH 3 )2CHOH
the outer skin as grapes ripen. When grapes are Methyl Isopropyl
iodide alcohol
crushed, sugar and the enzyme come in contact and
fermentation starts. Fermentation takes place in The alkyl halide is always formed from the smaller alkyl
anaerobic contidions i.e., in absence of air CO2 is group.
released during fermentation. If air gets into 114. (b) Due to greater electronegativity of sp2-hybridized
fermentation mixture the oxygen of air oxidises ethanol carbon atoms of the benzene ring, diaryl ethers are not
to ethanoic acid which in turn destroys the taste of attacked by nucleophiles like I–.
alcoholic drinks. CH3 CH3
96. (b) The commercial alcohol is made unfit for drinking by 115. (b)
CHOCH3 + HI CH3I + CHOH
mixing in it some copper sulphate (to give it colour) CH3 CH3
2-propanol
and pyridine (a foul smelling liquid). It is known as
denaturation of alcohol. 116. (a) Dehydration of alcohols gives ethers
117. (d) R – O – Ar + HI Ar – OH + RI
97. (a) C 6 H 5 ONa C 2 H 5 I C6 H 5OC 2 H 5 NaI Aryl-alkyl ether Phenol Alkyl iodide
Phenetole
Due to steric hinderance, smaller alkyl group is always
98. (b) C2 H 5 Br C 2 H 5ONa C2 H5 O C2H 5 attached to iodine.
NaBr
Sod. ethoxide diethyl ether
118. (d) Diethyl ether, being a Lewis base, is not attacked by
99. (b) Reaction of sodium ethoxide with ethyl iodide to nucleophiles, while all others contain electrophilic carbon,
produce diethyl ether is known as Williamson hence attacked by nucleophiles like OH– ions.
synthesis.
It is a nucleophilic substitution reaction and proceeds O
|
via SN2 mechanism. CH3 C OCH3 CH3 C N
100. (d) The two components should be (CH3 )3 CONa +
(CH3)3CBr. However, tert-alkyl halides tend to undergo
elimination reaction rather than substitution leading O
|| ..
to the formation of an alkene, Me2C = CH2 CH 3 C NH 2 C2H5 O
. . C 2 H5
EBD_7207
438 ALCOHOLS, PHENOLS AND ETHERS

STATEMENT TYPE QUESTIONS ASSERTION-REASON TYPE QUESTIONS


119. (c) Product formed is 2–methyl–pentan–2–ol hence :O :
carbon is attached to four different group therefore 128. (a) The bond angle in alcohols is slightly less
C H
the molecule is chiral but because the carbonyl group than the tetrahedral angle (109°-28 ). It is due to the
is planar so attack of methyl group can take place either repulsion between the unshared electron pairs of
ways above and below the plane of the of molecule oxygen.
hence equal number of enantiomers are formed and 129. (b) The correct explanation is : In Lucas test, tertiary
hence the racemic mixture is formed. alcohols react immediately because of the formation
120. (b) Alcohols are versatile compounds. They react both of the more stable tertiary carbocations.
as nucleophiles and electrophiles. The bond between 130. (c) The correct reason is : Nucleophilic attack of phenolate
O—H is broken when alcohols react as nucleophiles. ion through the ortho-carbon atom occurs on CCl4 (a
neutral electrophile) to form an intermediate which on
Alcohols as nucleophiles
hydrolysis gives salicylic acid (ArSE reaction).
131. (a) R is the correct explanation of A. Due to +M effect of
H ..
(i) R–O–H+ +C – R –+O
.. – C – R–O– C– +H +
.. H , its intermediate carbocation is more stable than
O
the one in benzene.
(ii) The bond between C — O is broken when they 132. (c) The usual halogenation of benzene takes place in the
react as electrophiles. Protonated alcohols react presence of a Lewis acid, such as FeBr 3, which
in this manner. polarises the halogen molecule. In case of phenol, the
Protonated alcohols as electrophiles polarisation of bromine molecule takes place even in
the absence of Lewis acid. It is due to the highly
R CH 2 OH H R CH2 OH 2 activating effect of –OH group attached to the benzene
ring.
133. (b)
Br– + CH2 – OH2+ Br – CH2 + H2O 134. (a) R is the correct explanation of A.
135. (d) 136. (c)
R R
137. (c) Alkyl aryl ethers are cleaved at the alkyl-oxygen bond
121. (b) Alcohols are, weaker acids than water. This can be due to the more stable aryl-oxygen bond. The reaction
illustrated by the reaction of water with an alkoxide. yields phenol and alkyl halide
– – O–R OH
R – O: + H – O – H R – O – H + :OH
: :

: :

: :

Base Acid Conjugate Conjugate


acid base +H–X +R–X
This reaction shows that water is a better proton donor Ethers with two different alkyl groups are also cleaved
(i.e., stronger acid) than alcohol. Also in the above in the same manner.
reaction, we note that an alkoxide ion is a better proton R O R ' HX R X R ' OH
acceptor than hydroxide ion, which suggests that
alkoxide are stronger bases (sodiumethoxide is a CRITICAL THINKING TYPE QUESTIONS
stronger base than sodium hydroxide).
138. (a) Methyl alcohol (CH3OH) is also known as carbinol.
122. (c) The dehydration of secondary and tertiary alcohols Hence vinyl carbinol is CH2 = CH – CH2OH.
to give corresponding ethers is unsuccessful as 139. (c) KMnO4 (alkaline) and OsO4 / CH2Cl2 are used for
elimination competes over substitution and as a hydroxylation of double bond while O3 /Zn is used for
consequence, alkenes are easily formed. ozonolysis. Therefore, the right option is (c), i.e.,
123. (c) The C — O bond length (136 pm) in phenol is slightly 3CH 3CH CH 2 BH3 in THF (CH 3CH 2 CH 2 )3 B
less than that in methanol (142 pm). 3H 2O 2
3CH 3CH 2CH 2OH + H3BO3
124. (a) A methanol poisoned patient is treated by giving NaOH
intravenous injection of ethanol. 1-propanol
140. (a) Hydroboration-oxidation leads to anti-Markownikoff’s
MATCHING TYPE QUESTIONS hydration, thus
125. (d) 126. (c) 127. (b)
ALCOHOLS, PHENOLS AND ETHERS 439

141. (b) NaBH4 and LiAlH4 attacks only carbonyl group and O
reduce it into alcohol group.
O

CH3– C–O > >
NaBH 4
C6 H5 CH CHCHO Most stable because
cinnamic aldehyde resonating structures
are equivalent
O–
C6 H 5 CH CH.CH 2 OH
cinnamic alcohol
CH C
> Least stable
142. (a) (resonance not
possible)
143. (b) Carbocation is formed as intermediate which is most
OCH3
stabilized when protonation occurs on terminal carbon. –OCH3 group intensifies
144. (b) Reaction of 1–chloropropane leads to the formation of –ve charge due to + M effect
the primary carbocation which rearranges to more OH OH OH OH
stable secondary carbocation, hence (ii) and (iii) give
similar products.
149. (a)
145. (a) In case of (ii) and (iii), the alcohol so obtained contain
carbon which is attached to four different groups i.e., NO2
chiral carbon while in case of (i) and (iv) achiral alcohol CH3 NO 2
is obtained. +I –I – M, – I
146. (a) CH 2 CH OH represents vinylic alcohol. In vinylic (A) (B) (C) (D)
alcohols –OH group is attached to sp2 hybridized Electron withdrawing substituents increases the
acidity of phenols; while electron releasing substituents
carbon whereas in allylic alcohols – OH group is
decreases acidity. Further the particular effect (acidity
attached to sp3 hybridized carbon. increasing or decreasing) is more when a substituent
147. (c) The mechanism of the reaction involves the following is present in o-ortho position to phenolic group. Thus
three steps: the correct order will be D > C > A > B.
Step 1: Protonation of alkene to form carbocation by 150. (c) Electron withdrawing substituents like –NO2 , Cl
electrophilic attack of H3O+ increase the acidity of phenol while electron releasing
substituents like – CH3, – OCH3 decreases acidity.
H2O + H+ H3O+ hence the correct order of acidity will be
H H OH OH OH OH
+ +
C=C +H—O
.. —H —C—C + H2O
> > >
H
Step 2: Nucleophilic attack of water on carbocation. NO2 Cl CH3 OCH3
H H H III I II IV
+ + (–M, –I) (–I > +M) (+I, + HC) (+ M)
—C—C + H2O —C—C—O —H
151. (c) ClCH 2 CH 2 OH is stronger acid than CH3CH 2 OH due
H H to – I effect of Cl.
Step 3: Deprotonation to form an alcohol. Cl CH 2CH 2OH Cl CH 2CH 2O H
Stronger acid ve charge on O
dispersed hence
.. conjugate base, stable
H H H OH
..

+ .. + CH3CH 2 OH CH3 CH 2O H
— C — C — O — H + H2O — C — C — +H3 O
Weaker acid ve charge intensified,
hence conjugate
H H base unstable

148. (a) More the stability of the conjugate base, higher is the 152. (c) Due to strong electron-donating effect of the OH
acidic character of the parent acid. Stability order of group, the electron density in phenol is much higher
than that in toluene, benzene and chlorobenzene and
the four conjugate bases is arranged below. hence phenol is readily attacked by the electrophile.
EBD_7207
440 ALCOHOLS, PHENOLS AND ETHERS
OH 163. (a) Here also, carbocation is formed as an intermediate,
hence the species capable of forming most stable
Zn CH3Cl carbocation will be most reactive.
153. (b) anhy + + +
AlCl3 CHCH2CH
CHCH CH3 2CHCH
CHCH2CHCH
3 3 CH2CH2CH2
Phenol X
CH3 COOH

alc. > >


KMnO 4
Y Z Benzyl
Benzylcarbocation
carbocation 2º carbocation 1º carbocation
154. (d) Riemer-Tiemann reaction involves electrophilic 164. (a) In Williamson’s synthesis the reaction of alkyl halides
substitution on the highly reactive phenoxide ring. with sodium alkoxides give ethers. However, if the alkyl
HCCl3 + OH– H2O + – :CCl3 halide is 3°, it undergoes elimination to give an alkene.
Cl– + : CCl 2 CH3
Note the C has only a
| +–
sextet of electrons H3C – C – Cl + NaOC2H5
|
sod. cthoxide
CH3
tert-butyl chloride
CH2
– – ||
O O CH3 – C – CH3 + C2H5OH + NaBr
CHCl2 – CHO 2-Methyl-1-propene
OH
165. (c) If a tertiary alkyl halide is used, an alkene is the only
reaction product and no ether is formed. For example,
A benzal chloride the reaction of CH3 ONa with (CH3)3C–Br gives
Al 2 O 3 Al 2 O3 exclusively 2-methylpropene.
155. (c) CH 2 CH 2 CH 3 CH 2 OH
650 K 525 K CH3
CH3CH2 OCH2CH3 | + –

: :
156. (b) Dehydration of CH3OH gives carbene (methylene), an CH3 C Br + NaO – CH3
unstable intermediate. |
CH3
H 2SO 4
CH 3OH [: CH 2 ] H 2 O
Carbene CH3 C CH 2 NaBr CH 3OH
157. (d) Since the compound (C4H10O) react with sodium, it |
must be alcohol (option b, c, or d). As it is oxidised to CH3
carbonyl compound which does not reduce Tollen’s 2-Methylpropene
reagent, the carbonyl compound should be a ketone It is because alkoxides are not only nucleophiles but
and thus C4H10O should be a secondary alcohol, i.e. strong bases as well. They react with alkyl halides
sec-butyl alcohol; other two given alcohols are 1º. leading to elimination reactions.
158. (a) 166. (c) In the cleavage of mixed ethers having two different
P I2 Mg alkyl groups, the alcohol and alkyl iodide that form
159. (d) CH3CH 2 OH CH3CH 2 I depend on the nature of alkyl group. When primary or
Ether
A
secondary alkyl groups are present, it is the lower alkyl
CH 2 CH 3 group that forms alkyl iodide therefore
HCHO
|
CH 3CH 2 MgI H C OMgI CH3 CH CH 2 O CH 2 CH 3 HI
|
(B) | CH3
H CH3
(C) |
CH 2 CH3 CH3 CH CH 2 OH CH 3CH 2 I
H2 O
|
H C OH H+ + Br –
167. (d) OCH3 O–CH3
|
H H
(D)
n propyl alcohol OH + CH3Br
160. (b) 161. (c)
162. (a) In case of m–nitrophenol operational effect of nitrogroup 168. (b) Due to greater electronegativity of sp2-hybridized
is electron withdrawing inductive effect while in case of carbon atoms of the benzene ring, diaryl ethers are not
b and c, both –R and –I effect are operational. attacked by nucleophiles like I–.
26
ALDEHYDES, KETONES AND
CARBOXYLIC ACIDS

FACT / DEFINITION TYPE QUESTIONS 8. Which of the following is correct for carbonyl compounds?
R R + –
– +
1. Choose the correct IUPAC name for (a) C=O (b) C=O
CH3 C H CHO R R
| + –
(c) (d) + –
CH2 CH 3 R–C=O R–C=O
(a) Butan - 2- aldehyde
(b) 2- methylbutanal R R
(c) 3- methylisobutyraldehyde 9. Which of the following contain an aldehyde?
(d) 2- ethylpropanal (a) Vanilla beans (b) Meadow sweet
2. The IUPAC name of the compound having the molecular (c) Cinnamon (d) All of these
formula Cl3C –CH2CHO is 10. Which of the following have pleasant smell?
(a) 3, 3, 3- trichloropropanal (a) Methanal (b) Propanal
(b) 1, 1, 1- trichloropropanal (c) Ethanal (d) Hexanal
(c) 2, 2, 2- trichloropropanal 11. Which one of the following can be oxidised to the
(d) Chloral corresponding carbonyl compound?
3. The IUPAC name of CH3COCH(CH3)2 is (a) 2-hydroxy-propane
(a) 2-methyl-3-butanone (b) 4-methylisopropyl ketone (b) Ortho-nitrophenol
(c) 3-methyl-2-butanone (d) Isopropylmethyl ketone (c) Phenol
4. IUPAC name of following will be (d) 2-methyl-2 hydroxy-propane
CHO 12. Which one of the following on oxidation gives a ketone ?
(a) Primary alcohol (b) Secondary alcohol
CH3 (c) Tertiary alcohol (d) All of these
13. What is formed when a primary alcohol undergoes catalytic
dehydrogenation ?
(a) Aldehyde (b) Ketone
(c) Alkene (d) Acid
OH 14. Primary and secondary alcohols on action of reduced copper
(a) 4-formyl 3-methyl 1-hydroxy benzene give
(b) 4-formyl 3-methyl phenol (a) Aldehydes and ketones respectively
(c) 4-hydroxy 2-methyl benzaldehyde (b) Ketones and aldehydes respectively
(d) 4-hydroxy 2-methyl carbaldehyde (c) Only aldehydes
5. IUPAC name of ethyl isopropyl ketone is (d) Only ketones
(a) 4-methyl pent-3-one (b) 2-methyl pent-3-one 15. Which alkene on ozonolysis gives CH3 CH2CHO and
(c) 4-methyl pent-2-one (d) 2-methyl pent-2-one CH3CCH3
6. In > C = O group sigma bond is formed by ||
(a) sp2-p-overlapping (b) sp3-p-overlapping
O
CH3
(c) sp-p-overlapping (d) s-p-overlapping (a) CH3CH2CH = C (b) CH3CH2CH = CHCH2CH3
7. The -bond in carbonyl group is formed by CH3
(a) s-s-overlapping (b) p-p-overlapping (c) CH3CH2CH = CH CH3 (d) CH 3 C CHCH 3
(c) s-p-overlapping (d) p-d-overlapping |
CH3
EBD_7207
442 ALDEHYDES, KETONES AND CARBOXYLIC ACIDS
16. The catalyst used in Rosenmund's reduction is 24. Find out B in the given reactions
(a) HgSO4 (b) Pd/BaSO4
(c) anhydrous AlCl3 (d) anhydrous ZnCl2 AlCl3 CrO3 in (CH3CO)2O
+ CH3 – X A B
+
SnCl 2 / HCl H3O
17. C 6 H 5C N + [H] C 6 H 5CHO + NH 3 .
(a) acetophenone
The above reaction is
(b) benzaldehyde
(a) Mendius reaction (b) Sandorn's reaction
(c) cyclohexyl carbaldehyde
(c) Rosenmund's reaction (d) Stephen’s reaction (d) benzoic acid
18. Benzaldehyde can be prepared by oxidation of toluene by 25. The reaction
(a) Acidic KMnO4 (b) K2Cr2O7 / H+
CHO
(c) CrO2Cl2 (d) All of these AlCl3
19. The oxidation of toluene to benzaldehyde by chromyl + CO + HCl
chloride is called
(a) Rosenmund reaction (b) Wurtz reaction (a) Rosenmund’s reaction (b) Stephen’s reaction
(c) Etard reaction (d) Fittig reaction (c) Cannizzaro’s reaction (d) Gatterman-Koch reaction
20. An aldehyde group can be present 26. Which aldehyde cannot be obtained by Rosenmund’s
(a) in between carbon chain reaction?
(b) at any position in carbon atom (a) CH3CHO (b) HCHO
(c) only at the end of carbon chain (c) CH3CH2CHO (d) All of these
(d) at the second carbon atom of the carbon chain 27. The conversion PhCN PhCOCH 3 , can be achieved
21. Benzaldehyde is obtained from Rosenmund’s reduction of most conveniently by reaction with
(a) CH3MgBr followed by hydrolysis
O O
(b) I2 – NaOH, CH3I
(c) Dil. H2SO4 followed by reaction with CH2N2
(a) CH3 (b) X (d) LiAlH4 followed by reaction with CH3I
28. Which of the following is used to prepare ketone from acyl
chloride ?
(a) R-MgX (b) R2Cd
O (c) CO + HCl (d) CrO3
Cl 29. Which of the following forces explain the boiling point of
aldehydes and ketones?
(c) (d) OH (a) Hydrogen bonding (b) van der Waal’s forces
Cl
(c) Dipole-dipole attraction(d) None of these
30. Which is highly soluble in water?
22. Which of the following is not used in the preparation of (a) Methanal (b) Propanal
ketone? (c) Propanone (d) Butanone
(a) Oxidation of secondary alcohols 31. Propanal and propanone, both have same molecular
(b) Dehydrogenation of 2° alcohol formula(C3H6O), what do you expect about their boiling
(c) Pyrolysis of calcium acetate points?
(d) Acid hydrolysis of alkyl cyanide (a) Both have same boiling point
23. Product of the following reaction is (b) Boiling point of propanal is higher than the boiling
CN point of propanone.
(c) Boiling point of propanal is lower than the boiling point
SnCl 2 + HCl
+ 2(H) ? of propanone
+
H3O (d) Nothing can be predicted
32. Less reactivity of ketone is due to
COOH CHO (a) + I inductive effect decrease positive charge on
carbonyl carbon atom
(a) (b) (b) steric effect of two bulky alkyl groups
(c) sp2 hybridised carbon atom of carbonyl carbon atom
(d) Both (a) and (b)
O 33. Acetaldehyde reacts with
NH2
(a) Electrophiles only
C–CH3 (b) Nucleophiles only
(c) (d) (c) Free radicals only
(d) Both electrophiles and nucleophiles
ALDEHYDES, KETONES AND CARBOXYLIC ACIDS 443
34. Carbonyl compounds undergo nucleophilic addition (a) Cu (b) CuO
because of (c) Cu2O (d) Cu(OH)2
(a) electronegativity difference of carbon and oxygen 45. Aldol condensation would not occur in :
atoms (a) CH3COCH3 (b) CH3CH2CHO
(b) electromeric effect (c) HCHO (d) CH3CHO
(c) more stable anion with negative charge on oxygen 46. Cannizzaro reaction occurs with
atom and less stable carbonium ion
(a) CH 3 CH 2 OH (b) C 6 H 5 CHO
(d) None of the above
35. Which of the following statement is false ? (c) CH 3 CHO (d) CH 3 CO CH 3
(a) Cannizzaro reaction is given by aldehydes in presence 47. Which of the following compound will show positive silver
of alkali mirror test ?
(b) Aldol condensation is given by aldehydes in presence
(a) HCOOH (b) CH 3 (CHOH ) 3 CHO
of alkali
(c) Aldol condensation is given by aldehydes and ketones (c) CH 3CO(CHOH )CH 3 (d) Both (a) and (b)
in presence of acids 48. Aldehydes and ketones are distinguished by which of the
(d) None of the above following test ?
36. If formaldehyde and KOH are heated, then we get (a) Lucas test
(a) methane (b) methyl alcohol (b) Tollen’s test
(c) ethyl formate (d) acetylene (c) KMnO4 solution (Baeyer’s test)
37. The reagent which can be used to distinguish acetophenone (d) None of these
from benzophenone is 49. Aldehydes and ketones are generally reduced by :
(a) 2,4- dinitrophenylhydrazine
(a) Clemmensen reduction (b) H2S
(b) aqueous solution of NaHSO3
(c) H2/Ni (d) None of these
(c) benedict reagent
50. In which reaction, > C = O can be reduced to > CH2?
(d) I2and Na2CO3
(a) Wolf-Kishner reaction (b) Reimer-Tiemann reaction
38. Benzaldehyde reacts with ethanoic KCN to give
(c) Wurtz reaction (d) None of these
(a) C6H5CHOHCN (b) C6H5CHOHCOC6H5
51. A compound does not react with 2, 4-dinitrophenylhydrazine,
(c) C6H5CHOHCOOH (d) C6H5CHOHCHOHC6H5
the compound is :
39. Acetone reacts with iodine (I2) to form iodoform in the
(a) Acetone (b) Acetaldehdye
presence of
(c) CH3OH (d) CH3CH2COCH3
(a) CaCO3 (b) NaOH
52. Which gives lactic acid on hydrolysis after reacting with
(c) KOH (d) MgCO3
HCN ?
40. (CH3)3C–CHO does not undergo aldol condensation due
(a) HCHO (b) CH3CHO
to
(a) three electron donating methyl groups (c) C6H5CHO (d) CH3COCH3
(b) cleavage taking place between —C— CHO bond 53. The most appropriate reagent to distinguish between
(c) absence of alpha hydrogen atom in the molecule acetaldehyde and formaldehyde is :
(d) bulky (CH3)3C—group (a) Fehling’s solution
41. Acetaldehyde reacts with semicarbazide and forms (b) Tollen’s reagent
semicarbazone. Its structure is (c) Schiff’s reagent
(a) CH3CH = NNHCON = CHCH3 (d) Iodine in presence of base
(b) CH3CH = NNHCONH2 54. Aldehydes can be oxidised by :
(a) Tollen’s reagent (b) Fehling solution
(c) CH 3 CH N — N — CONH 2
| (c) Benedict solution (d) All the above
OH 55. 2-pentanone and 3-pentanone can be distinguished by :
(d) CH3CH = N—CONHNH2 (a) Cannizaro's reaction (b) Aldol condensation
42. Iodoform test is not given by (c) Iodoform reaction (d) Clemmensen's reduction
(a) 2-Pentanone (b) Ethanol 56. Cross aldol condensation occurs between
(c) Ethanal (d) 3-Pentanone (a) two same aldehydes
43. Phenylmethyl ketone can be converted into ethylbenzene (b) two same ketones
in one step by which of the following reagents? (c) two different aldehydes and ketones
(a) LiAlH4 (b) Zn-Hg/HCl (d) None of these
(c) NaBH4 (d) CH3MgI 57. Ketone upon treatment with Grignard Reagent gives
44. When acetaldehyde is heated with Fehling’s solution it gives (a) primary alcohol (b) secondary alcohol
a precipitate of (c) tertiary alcohol (d) aldehyde
EBD_7207
444 ALDEHYDES, KETONES AND CARBOXYLIC ACIDS
58. When acetaldehyde reacts with alcohol then produce 67. Schiff’s reagent gives pink colour with
(a) Acetal (b) Ketal (a) acetaldehyde (b) acetone
(c) Acetone (d) None (c) acetic acid (d) methyl acetate
59. The product formed in Aldol condensation is 68. Benzophenone can be converted into benzene by using
(a) a beta-hydroxy aldehyde or a beta-hydroxy ketone (a) fused alkali
(b) an alpha-hydroxy aldehyde or ketone (b) anhydrous AlCl3
(c) an alpha, beta unsaturated ester (c) sodium amalgam in water
(d) a beta-hydroxy acid (d) acidified dichromate
60. Clemmensen reduction of a ketone is carried out in the 69. In the reaction of NaHSO3 with carbonyl compounds to
presence of which of the following ? form bisulphite product, the nucleophile is
(a) Glycol with KOH (b) Zn-Hg with HCl (a) HSO3– (b) SO3Na
(c) LiAlH4 (d) H2 and Pt as catalyst (c) SO3– – (d) None of the above
61. Which of the following products is formed when 70. Wolf-Kishner reduction is
benzaldehyde is treated with CH3MgBr and the addition (a) reduction of carbonyl compound into alcohol
product so obtained is subjected to acid hydrolysis ? (b) reduction of carbonyl compound into alkene
(a) A secondary alcohol (b) A primary alcohol (c) reduction of carboxyl compound into alkane
(c) Phenol (d) tert-Butyl alcohol (d) reduction of nitro compound into aniline
62. During reduction of aldehydes with hydrazine and 71. Tollen’s reagent is
potassium hydroxide, the first step is the formation of (a) ammonical CuSO4
(a) R — CH — — N — NH 2 (b) R — C N (b) ammonical AgNO3
(c) R — C— NH (d) R — CH — — NH (c) alkaline solution containing complex of copper nitrate
2
|| (d) none of these
O R OR
63. A and B in the following reactions are
72. Compound of general formula C are called
OH
HCN B
R–C–R' A R– C H OR
KCN
R' CH2NH2 (a) diester (b ) acid anhydride
O
(c) hemiacetal (d) acetal
CN 73. Imine derivatives of aldehyde and ketone is called as
(a) A = RR'C , B = LiAlH4 (a) Schiff’s reagent (b) Fehling’s reagent
OH
(c) Schiff’s base (d) Schiff’s acid
OH 74. Which reaction is used for detecting the presence of
(b) A = RR'C , B = NH3 carbonyl group?
COOH (a) Reaction with hydrazine
CN (b) Reaction with phenyl hydrazine
(c) A = RR'C , B H 3O (c) Reaction with hydroxylamine
OH (d) All of the above
(d) A = RR'CH2CN, B = NaOH 75. The difference between aldol condensation and
64. The product obtained by the reaction of an aldehyde and Cannizzaro’s reaction is that:
hydroxylamine is (a) the former takes place in the presence of -H-atom.
(a) hydrazone (b) aldoxime (b) the former takes place in the absence of -H-atom.
(c) primary amine (d) alcohol (c) the former takes place in the presence of -H-atom.
65. Which one gives positive iodoform test ? (d) none of the above
(a) (CH 3 ) 2 CHCH 2 OH X
76. C6H5CH=CHCHO C6H5CH=CHCH2OH
(b) C 6 H 5 OH In the above sequence X can be :
H (a) H2/Ni (b) NaBH4
(c) K2Cr2O7/H + (d) Both (a) and (b)
|
(c) CH 3 CH 2 C CH 2 CH 3 77. Which of the following pairs of compounds will undergo
| aldol and Cannizzaro reaction respectively ?
OH
(i) acetone; benzaldehyde
(d) CH 3CH 2 OH (ii) acetaldehyde; butan–2–one
66. The compound that neither forms semicarbazone nor oxime (iii) propanone; formaldehyde.
is (iv) cyclopentanone, benzaldehyde
(a) HCHO (b) CH3COCH2Cl (a) (i), (ii) and (iii) (b) (ii) and (iii)
(c) CH3CHO (d) CH3CONHCH3 (c) (ii), (iii) and (iv) (d) (iii) and (iv)
ALDEHYDES, KETONES AND CARBOXYLIC ACIDS 445
78. Two compounds benzyl alcohol and benzoic acid are formed 88. In the given reaction
from this compound, when this compound is heated in the CH3 CHO
presence of conc.NaOH, this compound is. Cl2/h H2O
(a) Benzaldehyde (b) Benzylalcohol A
373K
(c) Acetophenone (d) Benzophenone
79. The reagent which does not react with both, acetone and A is
benzaldehyde.
CH2Cl CHCl2
(a) Sodium hydrogensulphite
(b) Phenyl hydrazine (a) (b)
(c) Fehling’s solution
(d) Grignard reagent CCl3 CH3
80. Which of the following compounds will give butanone on (c) (d)
oxidation with alkaline KMnO4 solution? Cl
(a) Butan-1-ol (b) Butan-2-ol
89. R HCl gas R O CH2
(c) Both of these (d) None of these
C = O +A C + H2O
81. Which of the following compounds is most reactive towards R dil HCl R O CH2
nucleophilic addition reactions?
A is
O O
|| || (a) CH3OH (b) CH3COOH
(a) CH3 – C– H (b) CH3 – C– CH3 CH 2COOH CH 2OH
O O (c) (d)
(c) C–H (d) C–CH3 CH 2COOH CH2OH
90. In the reaction
82. Which of the following does not represent the natural source
CHO
of the corresponding acids ?
(a) Formic acid : Red ant HNO3 /H 2SO4
(b) Acetic acid : Vinegar A
273 283K
(c) Butyric acid : Rancid butter
(d) Isobutyric acid : Automobile exhausts Benzaldehyde
83. Vinegar is a solution of acetic acid which is : A is
(a) 15 – 20% (b) 20 –25%
CHO
(c) 6 – 8% (d) 2 – 4%
CHO
84. Methyl cyanide can be converted into acetic acid by one of
the following reactions. NO2
(a) (b)
(a) Reduction (b) Hydrolysis
(c) Electrolysis (d) Decarboxylation NO2
85. Toluene can be oxidised to benzoic acid by
(a) KMnO4 (alk.) (b) K2Cr2O7 (alk.) CHO
(c) Both (a) and (b) (d) Neither (a) nor (b)
86. Which of the following does the best represent the structure
(c) (d) Both (a) and (b)
of the carboxylate ion ?
– – NO2
O O 91. Which of the following can not be oxidised to give carboxylic
(a) R–C (b) R – C acid?
O+ O– CH2CH2CH3
CH3
+
(a) (b)
O
(c) R–C (d) None of these
CH3
O+ CH3
HC CH3 C CH3
87. Select the acid(s) which cannot be prepared by Grignard CH3
reagent.
(c) (d)
(a) Acetic acid (b) Succinic acid
(c) Formic acid (d) All of the above
EBD_7207
446 ALDEHYDES, KETONES AND CARBOXYLIC ACIDS
92. Lower carboxylic acids are soluble in water due to 100. Which of the following acids has the smallest dissociation
(a) low molecular weight (b) hydrogen bonding constant ?
(c) dissociation into ions (d) easy hydrolysis (a) CH3CHFCOOH (b) FCH2CH2COOH
93. Dimerisation of carboxylic acids is due to (c) BrCH2CH2COOH (d) CH3CHBrCOOH
(a) ionic bond 101. Which one of the following esters is obtained by the
(b) covalent bond esterification of propan-2-ol with ethanoic acid ?
(c) coordinate bond (a) (CH3)2CHCOOCH3 (b) CH3COOCH2CH3
(d) intermolecular hydrogen bond (c) CH3COOCH(CH3)2 (d) (CH3)2CHCOOCH2CH3
94. Boiling points of carboxylic acids are 102. The major product of nitration of benzoic acid is
(a) lower than corresponding alcohols (a) 3- Nitrobenzoic acid (b) 4- Nitrobenzoic acid
(b) higher than corresponding alcohols (c) 2- Nitrobenzoic acid (d) 2, 4- dinitrobenzoic acid
(c) equal to that of corresponding alcohols
103. Among the following acids which has the lowest pK a
(d) None of the above
95. MgBr value?
(a) CH 3 CH 2 COOH (b) (CH3 ) 2 CH COOH
(c) HCOOH (d) CH 3COOH
( i ) CO 2
P 104. The correct order of increasing acidic strength is
( ii ) H 3O __________
(a) Phenol < Ethanol < Chloroacetic acid < Acetic acid
In the above reaction product 'P' is
(b) Ethanol < Phenol < Chloroacetic acid < Acetic acid
CHO COOH
(c) Ethanol < Phenol < Acetic acid < Chloroacetic acid
(d) Chloroacetic acid < Acetic acid < Phenol < Ethanol
(a) (b) 105. Which reagent can convert acetic acid into ethanol ?
(a) Na + alcohol (b) LiAIH4 + ether
(c) H2 + Pt (d) Sn + HCl
OH 106. Which is false in case of carboxylic acids?
O (a) They are polar molecules
||
(c) (d) C 6 H 5 C C6H5 (b) They form H-bonds
(c) They are stronger than mineral acids
(d) They have higher b.p. than corresponding alcohols
96. In the anion HCOO– the two carbon-oxygen bonds are found 107. The elimination of CO2 from a carboxylic acid is known as
to be of equal length. What is the reason for it? (a) hydration (b) dehydration
(a) Electronic orbitals of carbon atom are hybridised (c) decarboxylation (d) carboxylation
(b) The C=O bond is weaker than the C–C bond 108. The reaction of carboxylic acid gives effervescences of
(c) The anion HCOO– has two reasonating structures CO2 with NaHCO3. The CO2 comes from.
(d) The anion is obtained by removal of a proton from the (a) R – COOH (b) NaHCO3
acid molecule (c) Both (a) and (b) (d) None of these
97. Carboxylic acids are more acidic than phenol and alcohol 109. Acetic anhydride is obtained by the reaction of
because of (a) sodium and acetic acid
(a) intermolecular hydrogen bonding
(b) ammonia and acetic acid
(b) formation of dimers
(c) ethanol and acetic acid
(c) highly acidic hydrogen
(d) P2O5 and acetic acid
(d) resonance stabilization of their conjugate base
110. Benzoic acid may be converted to ethyl benzoate by reaction
98. Which of the following has the maximum acidic strength ?
with
(a) o- nitrobenzoic acid (b) m-nitrobenzoic acid
(a) sodium ethoxide (b) ethyl chloride
(c) p-nitrobenzoic acid (d) p-nitrophenol
(c) dry HCl—C2H5OH (d) ethanol
99. Which of the following is the weakest acid ?
111. Propionic acid with Br2 /P yields a dibromo product. Its
OH
structure would be:
Br
(a) (b) CH3COOH |
(a) H– C – CH COOH (b) CH2Br – CH2 – COBr
2
COOH |
Br
Br
|
(c) HCOOH (d) (c) CH3– C – COOH (d) CH2 Br – CHBr – COOH
|
Br
ALDEHYDES, KETONES AND CARBOXYLIC ACIDS 447
112. The product obtained when acetic acid is treated with 119. The strongest acid among the following is –
phosphorus trichloride is (a) Salicylic acid (b) m-hydroxybenzoic acid
(a) CH3COOPCl3 (b) ClCH2COCl (c) p-hydroxybenzoic acid (d) Benzoic acid
(c) CH3COCl (d) ClCH2COOH 120. Among the following, the most acidic is :
113. The reaction (a) CH3COOH (b) ClCH2COOH
RCH 2 CH 2 COOH
Red P
R CH 2 CH COOH
(c) Cl2CHCOOH (d) Cl2CHCH2COOH
Br2 | 121. Which of the following is the correct decreasing order of
Br
acidic strength of
is called as
(i) Methanoic acid (ii) Ethanoic acid
(a) Reimer- Tiemann reaction
(iii) Propanoic acid (iv) Butanoic acid
(b) Hell-Volhard Zelinsky reaction
(a) (i) > (ii) > (iii) > (iv) (b) (ii) > (iii) > (iv) > (i)
(c) Cannizzaro reaction
(c) (i) > (iv) > (iii) > (ii) (d) (iv) > (i) > (iii) > (ii)
(d) Sandmeyer reaction
122. Among the following the strongest acid is
114. Benzoic acid reacts with conc. HNO3 and H2SO4 to give :
(a) CH 3COOH (b) CH 2 ClCH 2COOH
(a) 3-Nitrobenzoic acid
(b) 4-Benzene sulphonic acid (c) CH 2 ClCOOH (d) CH3CH 2COOH
(c) 4-Nitrobenzoic acid 123. Arrange the following carboxylic acid in their decreasing
(d) 2-Nitrobenzoic acid acidity.
115. In the following reaction
1. COOH Oxalic acid
Br2 / P excess NH 3
RCH 2 COOH X Y
COOH
The major compounds X and Y are
2. HOOC – CH2 – COOH Malonic acid
(a) RCHBrCONH2 ; RCH(NH2)COOH
(b) RCHBrCOOH ; RCH(NH2)COOH 3. CH2– COOH Succinic acid
(c) RCH2COBr ; RCH2COONH4
(d) RCHBrCOOH ; RCH2CONH2 CH2– COOH
116. The yield of ester in esterification can be increased by (a) 3 > 2 > 1 (b) 1 > 2 > 3
CH3CH2OH + CH3COOH CH3COOCH2CH3 + H2O (c) 2 > 3 > 1 (d) 2 > 1 > 3
(a) removing water
(b) taking ethanol in excess STATEMENT TYPE QUESTIONS
(c) taking acetic acid in excess 124. Read the following statements and choose the correct option
(d) all the above factors (i) The carbonyl carbon atom is sp2 -hybridised
117. A carboxylic acid can best be converted into acid chloride (ii) The carbonyl carbon is an electrophilic (Lewis acid)
by using centre
(a) PCl5 (b) SOCl2 (iii) The carbonyl oxygen is a nucleophilic (Lewis base)
(c) HCl (d) ClCOCOCl centre
118. Arrange the following four acids in their decreasing order (iv) Carbonyl compounds are non- polar in nature.
of acidity (a) (i), (ii) and (iv) are correct
COOH COOH (b) (i), (ii) and (iii) are correct
CH3 (c) (ii), (iii) and (iv) are correct
(d) (ii) and (iv) are correct
125. Which of the following statement(s) is/are true regarding
preparation of aldehydes and ketones?
I II
(i) Both can be prepared by the oxidation of the concerned
COOH COOH alcohol with copper at about 250ºC.
CMe3 H3C CH3 (ii) Both can be prepared by the oxidation of the concerned
alcohol by Oppenauer oxidation.
(iii) Both can be prepared by the oxidation of respective
alcohol with acidic dichromate.
III IV
(a) (i) only (b) (ii) and (iii)
(a) I > II > III > IV (b) IV > III > II > I (c) (i) and (iii) (d) All the three
(c) II > IV > III > I (d) III > IV > II > I
EBD_7207
448 ALDEHYDES, KETONES AND CARBOXYLIC ACIDS
126. Which of the following statements are false? (i) It is carried out in presence of a strong acid which acts
(i) No aldehyde can be prepared by the oxidation of as a catalyst.
primary alcohol with acidic KMnO4. (ii) The strong acid makes the carbonyl carbon more
(ii) Aldehydes having a boiling point less than 100°C can electrophilic, and hence causes the alcohol, a strong
be prepared by the oxidation of primary alcohol with nucleophile to attack on the carbonyl carbon.
acidic dichromate. (iii) The strong acid makes the carbonyl group more
(iii) Secondary alcohols on oxidation with PCC in electrophilic which is thus attacked easily by an
dichloromethane give carboxylic acids having lesser alcohol, a weak nucleophile.
number of carbon atoms (iv) Esterification can be done even in absence of a strong
(iv) Tertiary alcohols can't be oxidised at all acid.
(a) (ii) and (iii) (b) (ii), (iii) and (iv) (a) (i) and (ii) (b) (i) and (iii)
(c) (i), (iii) and (iv) (d) (i), (ii) and (iii) (c) (i) only (d) (iv) only
127. Read the following statements and choose the correct option
(i) The boiling points of aldehydes and ketones are lower MATCHING TYPE QUESTIONS
than those of alcohols of similar molecular masses
131. Match the columns
(ii) Alcohols show intermolecular hydrogen bonding
Column-I Column-II
whereas aldehydes and ketones do not show
(Common names) (IUPAC names)
intermolecular hydrogen bonding.
(A) Cinnamaldehyde (p) Pentanal
(iii) The lower members of aldehydes and ketones are
(B) Acetophenone (q) Prop-2-enal
miscible with water in all proportions, because they
form hydrogen bond with water. (C) Valeraldehyde (r) 4-Methylpent-3-en-2-one
(iv) The solubility of aldehydes and ketones increases (D) Acrolein (s) 3-Phenylprop-2-enal
rapidly on increasing the length of alkyl chain (E) Mesityl oxide (t) 1-Phenylethanone
(a) TTFF (b) TFFT (a) A – (s), B – (t), C – (p), D – (q), E – (r)
(c) FTTT (d) TTTF (b) A – (p), B – (q), C – (s), D – (t), E – (r)
128. Aldehydes are generally more reactive than ketones in (c) A – (t), B – (s), C – (p), D – (r), E – (q)
nucleophilic addition reactions. Which of the following (d) A – (q), B – (t), C – (r), D – (s), E – (p)
statements accounts for this ? 132. Match the columns
(i) Sterically, the presence of two relatively large Column-I Column-II
substituents in ketones hinders the approach of (A) R CO CH 3 Zn Hg / HCl (p) Friedel-Craft’s
nucleophile to carbonyl carbon reaction
(ii) Aldehydes show resonance whereas ketones do not R CH 2 CH 3
(iii) Electronically, the presence of two alkyl groups reduce (B) 2C6 H5 CHO NaOH
(q) Kolbe’s reaction
the electrophilicity of the carbonyl carbon more
C6 H5 COONa C6 H5 CH 2 OH
effectively.
(iv) Electronically carbonyl carbon atom in ketones is more (C) C6 H 6 CH 3 COCl
Anhyd.
(r) Clemmensen’s
electrophilic than in aldehydes AlCl3
reaction
(a) (i) and (iii) (b) (i) and (iv) C6 H5 COCH3
(c) (ii) and (iii) (d) (ii) and (iv)
(D) C 6 H 5 OH CO 2 NaOH (s) Cannizzaro’s
OH
129. 2C6 H5CHO C6 H5CH 2OH C6 H5COO HOC6 H 4COONa reaction
H 2O
Which of the following statements are correct regarding (a) A – (p), B – (q), C – (r), D – (s)
the above reduction of benzaldehyde to benzyl alcohol? (b) A – (q), B – (p), C – (r), D – (s)
(i) One hydrogen is coming from H2O as H+ and another (c) A – (r), B – (s), C – (p), D – (q)
from C6H5CHO as H– (d) A – (s), B – (r), C – (p), D – (q)
(ii) One hydrogen is coming from H2O as H– and another 133. Match the columns
from C6H5CHO as H+ Column-I Column-II
(iii) One hydrogen from H2O and another from C6H5CHO, (A) Etard reaction (p) Alcoholic KOH
both in the form of H– (B) Hydroxylation (q) Anhydrous AlCl3
(iv) The reduction is an example of disproportionation (C) Dehydrohalogenation (r) Chromyl chloride
reaction (D) Friedel-Crafts reaction (s) Dilute alkaline KMnO4
(a) (i), (ii) and (iii) (b) (i) and (iv) (a) A – (p), B – (q), C – (r), D – (q)
(c) (ii), (iii) and (iv) (d) (iii) and (iv) (b) A – (s), B – (r), C – (p), D – (q)
130. Which of the following statement(s) is/are true regarding (c) A – (r), B – (s), C – (p), D – (q)
esterification of a carboxylic acid with an alcohol ? (d) A – (q), B – (p), C – (s), D – (r)
ALDEHYDES, KETONES AND CARBOXYLIC ACIDS 449
134. Match the columns 137. Match the columns
Column-I Column-II Column-I Column-II
(Reactions) (Reagents) COOH
(A) Benzophenone (p) LiAlH4 (A) CH2 (p) Glutaric acid
Diphenylmethane COOH
(B) Benzaldehyde (q) DIBAL–H
CH2 COOH
1-Phenylethanol
(B) (q) Adipic acid
(C) Cyclohexanone (r) Zn(Hg)/Conc HCl CH2 COOH
Cyclohexanol
(D) Phenyl benzoate (s) CH3MgBr (C) CH2 COOH (r) Succinic acid
Benzaldehyde
CH2
(a) A – (p), B – (s), C – (r), D – (q)
(b) A – (q), B – (s), C – (p), D – (r) CH2 COOH
(c) A – (s), B – (r), C – (q), D – (p)
(d) A – (r), B – (s), C – (p), D – (q) (D) CH2 COOH (s) Malonic acid
135. Match the columns
Column-I Column-II CH2

R CH2
(A) C = NH (p) Oxime
R
CH2 COOH
R
(B) C = NOH (q) Semicarbazone (a) A – (q), B – (p), C – (s), D – (r)
R (b) A – (r), B – (p), C – (s), D – (q)
R (c) A – (s), B – (r), C – (p), D – (q)
(C) C = N – NH2 (r) Imine (d) A – (r), B – (q), C – (s), D – (p)
R
ASSERTION-REASON TYPE QUESTIONS
O
R Directions : Each of these questions contain two statements,
(D) C = N – NH – C – NH 2 (s) Hydrazone Assertion and Reason. Each of these questions also has four
R
alternative choices, only one of which is the correct answer. You
(a) A – (q), B – (s), C – (p), D – (r) have to select one of the codes (a), (b), (c) and (d) given below.
(b) A – (r), B – (p), C– (s), D– (q) (a) Assertion is correct, reason is correct; reason is a correct
(c) A – (r), B – (s), C – (p), D– (q) explanation for assertion.
(d) A – (s), B – (r), C – (q), D– (p) (b) Assertion is correct, reason is correct; reason is not a
136. Match the acids given in Column-I with their correct IUPAC correct explanation for assertion
names given in Column-II. (c) Assertion is correct, reason is incorrect
Column-I Column-II (d) Assertion is incorrect, reason is correct.
(Acids) (IUPAC names) 138. Assertion : The boiling points of aldehydes and ketones
are higher than hydrocarbons and ethers of comparable
(A) Phthalic acid (p) Hexane-1, 6-dioic acid
molecular masses.
(B) Oxalic acid (q) Benzene-1, 2-dicarboxylic acid Reason : There is a weak molecular association in aldehydes
(C) Succinic acid (r) Pentane-1, 5-dioic acid and ketones arising out of the dipole-dipole interactions.
(D) Adipic acid (s) Butane-1, 4-dioic acid 139. Assertion : Formaldehyde is a planar molecule.
(E) Glutaric acid (t) Ethane-1, 2-dioic acid Reason : It contains sp2 hybridised carbon atom.
(a) A – (t), B – (q), C – (r), D – (p), E – (s) 140. Assertion : Compounds containing –CHO group are easily
(b) A – (p), B – (s), C – (t), D – (q), E – (r) oxidised to corresponding carboxylic acids.
(c) A – (q), B – (t), C – (s), D – (p), E – (r) Reason : Carboxylic acids can be reduced to alcohols by
(d) A – (r), B – (t), C – (p), D – (s), E – (q) treatment with LiAlH4.
EBD_7207
450 ALDEHYDES, KETONES AND CARBOXYLIC ACIDS
141. Assertion : The molecular mass of acetic acid in benzene is O
120 instead of 60.
Reason : The carboxylic acids exist as cyclic dimers in which
the two molecules of the acid are held together by two (a) (b) OH
strong hydrogen bonds.
O
CRITICAL THINKING TYPE QUESTIONS
142. IUPAC name of the following compound is
(c) (d)
O
146. Which of the following reagent reacts in different ways
with CH3CHO, HCHO and C6H5CHO ?
H (a) Fehling solution (b) C6H5NHNH2
(c) Ammonia (d) HCl
147. A new carbon - carbon bond is formed in
(a) 2-(2-propenyl) butanal
(i) Aldol condensation (ii) Kolbe’s reaction
(b) 2-(1-propenyl) butanal
(iii) Reimer-Tiemann reaction
(c) 4-formyl 4-ethyl but-2-ene
(iv) Wurtz Fittig reaction
(d) 2-ethyl pent-3-en-l-al
(a) (i) and (iii) (b) (ii) and (iii)
143. Observe the following structures and pick up the correct
(c) (i), (ii) and (iiv) (d) All the four
statement.
148. Which of the following is an example of nucleophilic
+
C=O C = OH addition ?
I II O NNH 2
|| NH 2 NH 2 ,H ||
(a) Carbonyl carbon of I is more electrophilic than that (a) C 6 H 5 C CH 3 C 6 H 5 C CH 3
of II
(b) Carbonyl carbon of I is less electrophilic than that O OH
|| |
of II (b) C6 H 5 CCH 3
LiAlH 4
C6 H5CHCH 3
(c) Carbonyl carbon of both structures have equal
electrophilic character (c) Both (a) and (b)
(d) It depends upon the complete structure of the (d) None of the two
compound 149. Acetal formation is a reversible reaction
144. The boiling points of aldehydes and ketones lie in between R H
+
R OH R'OH, H +
alkanes and alcohols of comparable masses because C = O + R'OH C
(a) alkanes are polar H H OR'
(b) aldehydes and ketones are non-polar R OR'
C+ H2O
(c) alkanes are non-polar and aldehydes and ketones H OR'
Under what conditions, the reaction can be forced to proceed
contain polar C = O group and lower alcohols only in right (forward) direction ?
(a) Using excess of alcohol
have H-bonding. (b) Using high temperature
(d) alkanes are held together by weak van der Waal’s (c) Using dilute acid and excess of alcohol
forces (being non-polar), aldehydes and ketones (d) Using dry acid and excess of alcohol
150. In the crossed Cannizzaro reaction involving HCHO as one
of the components
contain polar C = O group and held together by (a) HCHO is always oxidised because of electronic effect
(b) HCHO is always oxidised because of steric effect
strong dipole-dipole attraction and lower alcohols (c) both of the above statements are true
have H-bonding, which is stronger than dipole-dipole (d) none of the above statement is true
attraction. 151. Which of the following acts as a nucleophile in the aldol
145. Product of the following reaction is condensation of ethanal?
CN MgX (i) OH– (ii) H2O
dry ether (iii) –CH2CHO
+
H3O
+ (a) Only (i) (b) (i) and (ii)
(c) (i) and (iii) (d) All the three
ALDEHYDES, KETONES AND CARBOXYLIC ACIDS 451
152. Which of the following acts as a nucleophile in the 161. Cannizaro’s reaction is not given by _____________
Cannizzaro reaction involving benzaldehyde ? CHO

(i) OH– (ii) C6H4CHO (a) (b) CHO
CH3
(iii) C6H5CH(OH)O– (iv) H2O
(c) HCHO (d) CH3CHO
(a) (i) and (iv) (b) (i) and (ii)
162. Benzophenone can be obtained by_____________.
(c) (i) and (iii) (d) Only (i)
(i) Benzoyl chloride + Benzene + AlCl3
153. Which of the following undergoes haloform reaction ? (ii) Benzoyl chloride + Diphenyl cadmium
(i) CH3CH2COCH2Cl (ii) C6H5COCH3 (iii) Benzoyl chloride + Phenyl magnesium chloride
(iii) C6H5COCHCl2 (iv) CH3CH2COCCl3 (iv) Benzene + Carbon monoxide + ZnCl2
(a) Only (ii) (b) (ii) and (iv) (a) (i), (ii) and (iii) (b) (ii) and (iii)
(c) (i), (ii) and (iv) (d) All the four (c) (iii) and (iv) (d) (i), (ii) and (iv)
154. When ethanal reacts with propanal in the presence of a 163. Which of the following conversions can be carried out by
base, the number of products formed is Clemmensen Reduction ?
(a) 2 (b) 3 (i) Benzaldehyde into benzyl alcohol
(c) 4 (d) 5 (ii) Cyclohexanone into cyclohexane
155. Aldehydes and ketones will not form crystalline derivatives (iii) Benzoyl chloride into benzaldehyde
with (iv) Benzophenone into diphenyl methane
(a) sodium bisulphite (a) (ii) and (iv) (b) (i) and (iv)
(b) phenylhydrazine (c) (i) and (iii) (d) (iii) and (iv)
(c) semicarbazide hydrochloride 164. Benzaldehyde is less reactive than propanal because
(d) dihydrogen sodium phosphate. (i) the carbon atom of the carbonyl group of benzaldehyde
is less electrophilic as in propanal.
156. Which of the following compound will undergo self aldol
(ii) the carbon atom of the carbonyl group of benzaldehyde
condensation in the presence of cold dilute alkali ?
is more electrophilic as in propanal.
(a) CH2 CH CHO (b) CH C CHO (iii) carbonyl group in benzaldehyde is more polar due to
(c) C 6 H 5 CHO (d) CH 3 CH 2 CHO . resonance
157. Which of the following is an example of aldol condensation? (iv) carbonyl group in benzaldehyde is less polar due to
resonance
CH 3
| (a) (i) and (iii) (b) (i) and (iv)
dil NaOH
(a) 2CH3COCH3 CH 3 C(OH)CH 2 COCH 3 (c) (i) only (d) (iv) only
165. Addition of hydrogen cyanide to aldehydes and ketones
dil NaOH
(b) 2HCHO CH3OH occurs in presence of a base.The role of base is to
dil NaOH
(i) catalyse the reaction
(c) C6H5CHO + HCHO C6H5CH2OH (ii) generate CN– ion
(d) None of the above (iii) slow down the reaction
158. Identify X, (iv) to stabilize the cyanohydrins
H3C (a) (i) and (iii) (b) (i) and (ii)
CH3MgI H 2O (c) (i) and (iv) (d) (ii) and (iv)
C=O Intermediate X
H3C dry ether 166. Addition of alcohols to aldehydes and ketones takes place
in presence of dry HCl gas because it
(a) CH3OH (b) Ethyl alcohol (i) Protonates the oxygen of the carbonyl compounds
(c) Methyl cyanide (d) tert-Butyl alcohol (ii) Increases the electrophilicity of the carbonyl carbon
159. An organic compound of formula, C3H6O forms phenyl (iii) Removes the excess moisture from the reaction
hydrazone, but gives negative Tollen’s test. The compound is (iv) Helps the reaction to move in the forward direction
(a) CH3CH2COCH3 (b) CH3CH2CHO (a) (i), (ii) and (iv) (b) (i), (ii), (iii) and (iv)
(c) CH3COCH3 (d) Both (a) and (c) (c) (ii) ,(iii), and (iv) (d) (i), (iii) and (iv)
167. When benzaldehyde and acetaldehyde undergoes reaction
R NH Hydrolysis
160. C = O HCN (A) 3
(B) (C) with the 2, 4–DNP ?
R (a) Benzaldehyde reacts slowly than acetaldehyde
Compound (C) in above reaction is (b) Acetaldehyde reacts slowly than benzaldehyde
(a) -hydroxy acid (b) -amino acid (c) Both reacts equally
(c) -amino alkanol (d) -amino -hydroxy acid (d) Both do not react with 2, 4-DNP
EBD_7207
452 ALDEHYDES, KETONES AND CARBOXYLIC ACIDS
168. Suppose the reaction of compound containing ketone as Identify the product(s) formed in the given reaction.
functional group is carried in basic medium of NaOH. Which I II
of the following will one use to protect the unwanted (a) 2 molecules of benzoic acid 2 molecules of ethanoic acid
reaction due presence of carbonyl moiety.
(b) 2 molecules of benzoic acid 1 molecules of benzoic acid
(a) NaHSO3
and 1 molecule of ethanoic acid
(b) HCN
(c) ethylene glycol and HCl (c) 1 molecule of ethanoic acid 1 molecule of benzoic acid
(d) None of these (d) 1 molecule of benzoic acid 1 molecule of butanoic acid
169. A compound C5H10O forms orange–red precipitate upon 175. Ethanoic acid can’t be obtained by which of the following
reaction with 2,4–DNP, but does not give positive Tollen’s reaction ?
test and iodoform test. Possible compound is (i) KCN
(a) 2, 2–dimethylpropanal (b) 3–methylbutan–2–one (i) C2 H5Cl
(ii) H3O
(c) Pentan–3–one (d) None of the above
170. Nitration of the compound is carried out, this compound (i) AgCN
(ii) CH3Cl
gives red–orange ppt. with 2,4–DNP, this compound (ii) H3O
undergoes Cannizzaro reaction but not aldol, than possible
product due to nitration is KMnO 4 / OH
(iii) CH 3CH CH 2
(a) 3–nitroacetophenone heat
(b) (2–nitro)–2–phenylethanal
(i) Mg
(c) (2–nitro)–1–phenylpropan–2–one (iv) CH3Br
(ii) CO2
(d) 3–nitrobezaldehyde (iii) H3O
171. Structure of the compound whose IUPAC name is 3-ethyl-
2-hydroxy-4-methylhex-3-en-5-ynoic acid is : (a) (iii) and (iv) (b) (i) and (ii)
(c) (ii) and (iii) (d) (i) and (iv)
OH
OH 176. Primary alcohols can be readily oxidised to carboxylic acids
COOH by.
(a) COOH (b)
(i) KMnO4 in neutral medium.
(ii) KMnO4 in acidic or alkaline medium.
OH (iii) K2Cr2O7 in alkaline medium.
(iv) K2Cr2O7 in acidic medium.
(c)
COOH
(d) COOH (a) (i), (ii) and (iv) (b) (i), (ii) and (iii)
(c) (ii) and (iii) (d) (i) and (iii)
OH
177. Which of the following is correct order of acidity?
172. The end product B in the sequence of reactions,
(a) HCOOH > CH3COOH > ClCH2COOH > C2H5 COOH
CN NaOH
R X A B is (b) ClCH2COOH > HCOOH > CH3COOH > C2H5COOH
(a) an alkane (c) CH3COOH > HCOOH > ClCH2COOH > C2H5COOH
(b) a carboxylic acid (d) C2H5COOH > CH3COOH > HCOOH > ClCH2COOH
(c) sodium salt of carboxylic acid 178. An organic compound A upon reacting with NH3 gives B.
(d) a ketone On heating B gives C. C in presence of KOH reacts with Br2
173. Which is the most suitable reagent for the following to given CH3CH2NH2. A is :
conversion? (a) CH3COOH (b) CH3CH2CH2COOH
O
|| (c) CH3 CH COOH (d) CH3CH2COOH
CH3 – CH CH – CH 2 – C– CH 3 |
CH3
O
||
CH3 – CH CH – CH 2 – C– OH 179. The correct order of increasing acid strength of the
compounds
(a) Tollen’s reagent (b) Benzoyl peroxide
(A) CH3CO2H (B) MeOCH2CO2H
(c) I2 and NaOH solution (d) Sn and NaOH solution
174. In the given reaction, Me
(C) CF3CO2H (D) CO2H is
H 2O Me
(C 6 H 5 CO) 2 O I
H 2O (a) D < A < B < C (b) A < D < B < C
C 6 H 5 COOCOCH 3 II
(c) B < D < A < C (d) D < A < C < B
ALDEHYDES, KETONES AND CARBOXYLIC ACIDS 453
180. Through which of the following reactions number of carbon 184. Kolbe’s electrolytic method can be applied on
atoms can be increased in the chain?
(i) CH 2 COONa (ii) CHCOONa
(i) Grignard reaction (ii) Cannizzaro’s reaction | ||
(iii) Aldol condensation (iv) HVZ reaction CH 2 COONa CHCOONa
Choose the correct option.
(a) Only (iii) and (i) (b) Only (iii) and (ii) (iii) C6 H5COOK (iv) CH3COOK
(c) Only (iii) and (iv) (d) (i), (ii), (iii) and (iv) (a) (i), (ii) and (iv) (b) (i), (ii) and (iii)
181. In a set of the given reactions, acetic acid yielded a (c) (ii), (iii) and (iv) (d) (iii) and (iv)
product C. 185. Which of the following represents the correct order of the
C6 H6 acidity in the given compounds?
CH3COOH PCl5 A B
Anh.AlCl3 (a) FCH 2 COOH > CH 3 COOH > Br CH 2 COOH >
C2H5MgBr ClCH2COOH
C
Ether (b) BrCH2 COOH > ClCH2 COOH > FCH2 COOH >
Product C would be CH3COOH
C2 H 5 (c) FCH2 COOH > ClCH2 COOH > BrCH2 COOH >
| CH3COOH
(a) CH 3 C (OH)C 6 H 5 (b) CH 3 CH (OH) C 2 H 5
(d) CH3 COOH > BrCH2 COOH > ClCH2 COOH >
(c) CH 3 COC 6 H 5 (d) CH 3 CH (OH ) C 6 H 5 FCH2COOH
186. The correct order for the acidic character of the following
K 2Cr2O 7+H2SO4
182. Z. Here Z is carboxylic acids is

COOH COOH COOH

(a) HOOC OH

(b) (CH3)3CCOOH I II OH
(c) Both (a) and (b)
III

(d) HOOC COOH COOH


COOH
HO OH
183. RCOOH can be reduced to RCH2OH by OCH3
(i) NaBH4 (ii) LiAlH4
(iii) Na/C2H5OH (iv) H2/Catalyst IV
V
(a) (ii) and (iv) (b) (i) and (iii)
(a) IV > I > II > III > V (b) V > II > III > I > IV
(c) (i), (ii) and (iv) (d) (i), (iii) and (iv)
(c) V > II > IV > III > I (d) V > II > IV > I > III
EBD_7207
454 ALDEHYDES, KETONES AND CARBOXYLIC ACIDS

FACT / DEFINITION TYPE QUESTIONS H CH3 H O CH3


O3
15. (a) CH3– CH2– C C CH3– CH 2– C– C
2 1 CH3
1. (b) CH3 – CH – CHO CH3 O O
O O
3 4
CH2 – CH3 CH3– C – CH 3+ CH3 – CH2 – CHO (–H2O)

2. (a) 16. (b) Catalyst used in Rosenmund reduction is Pd/BaSO4.


O CH3 Rosenmund reduction is used for reduction of acid
1 2 || 3| 4 chloride.
3. (c) CH 3 C C H CH 3 ; 3- methyl-2-butanone
O O
4. (c) 5. (b) 6. (a) 7. (b) Pd / BaSO4
R C Cl R C H
8. (b) O is more electronegative than C.
17. (d) Phenyl cyanide is reduced into benzaldehyde in the
9. (d) Vanillin -vanilla beans
presence of SnCl2 / HCl reagent. This reaction is
Salicylaldehyde - meadow sweet
Cinnamaldehyde -from cinnamon. known as Stephen’s reaction.
10. (d) The lower aldehydes have sharp pungent odours. As C 6 H 5C N 2[H]
SnCl 2
the size of the molecule increases, the odour becomes HCl
less pungent and more fragrant.
H 2O / H
11. (a) Carbonyl compounds (aldehydes and ketones) are C 6 H 5CH NH C 6 H 5CHO NH 3
obtained by the oxidation of 1° and 2° alcohols 18. (c)
respectively. Among the given options, only (a) is 2°
alcohol hence it can be oxidized to ketone. COOH CH3 CHO

acidic K 2 Cr2 O7 CrO 2 Cl2


OH O
or KMnO 4
| ||
oxidation
H3CCHCH3 H3CCCH 3
(Etard reaction)
2 hydroxypropane Acetone
Acidic KMnO4 and K2Cr 2O7 oxidise toluene to
12. (b) Secondary alcohols on oxidation give ketones. benzoic acid but CrO2Cl2 oxidises it to benzaldehyde.
Note : Primary alcohols form aldehydes. 19. (c) 20. (c) 21. (b) 22. (d) 23. (b)
24. (a) 25. (d)
R R
[O] 26. (b) Formyl chloride is unstable at room temperature.
CHOH C=O 27. (a) Alkanenitriles (other than methanenitrile) and
R R benzonitrile give ketones with Grignard reagents.
Isopropyl Ketone
alcohol 28. (b)
13. (a) 1° Alcohols on catalytic dehydrogenation give + –
29. (c) C=O C – O ; the polarity exists in
aldehydes.
Cu
carbonyl group due to resonance.
RCH2OH RCHO + H2 30. (a) Solubility decreases with increase in mol. wt.
300°C
1° alcohol Aldehyde 31. (c) Propanone has symmetrical structure.
14. (a) Alcohols are oxidized by removal of H2 in presence 32. (d)
of a heated metal catalyst (Cu) 33. (b) Acetaldehyde reacts only with nucleophiles. Since the
mobile p electrons of carbon–oxygen double bond are
Cu
CH3CH 2OH CH3CHO H 2 strongly pulled towards oxygen, carbonyl carbon is
300
1 alcohol Aldehyde electron-deficient and carbonyl oxygen is electron-rich.
The electron deficient (acidic) carbonyl carbon is most
H3 C susceptible to attack by electron rich nucleophilic
Cu
CH OH CH3CCH3 H2 reagents, that is, by base. Hence the typical reaction
300
H3 C || of aldehydes and ketones is nucleophilic addition.
2° Alcohol O 34. (c)
Ketone
ALDEHYDES, KETONES AND CARBOXYLIC ACIDS 455
35. (d) Cannizzaro reaction is given by aldehydes having no 44. (c) CH3 CHO + 2Cu2+ + OH– CH3COOH + Cu2O
-hydrogen atom in the presence of conc. alkali, aldol Fehling solution (red)
condensation is given by aldehydes and ketones 45. (c) Aldol condensation is given by carbonyl compounds
having at least one -atom in presence of alkali or in which have -hydrogen atoms.
presence of acids HCHO does not have any -hydrogen atom, so it
36. (b) Aldehydes containing no -hydrogen atom on does not give aldol condensation.
warming with 50% NaOH or KOH undergo 46. (b) Cannizzaro reaction is given by aldehydes and ketones
disproportionation i.e. self oxidation - reduction known which do not have -hydrogen atom. Benzaldehyde
as Cannizzaro’s reaction. (C6H5CHO) does not have -H atom and hence gives
2 HCHO
50% NaOH
HCOONa CH 3OH
Cannizzaro reaction.
47. (d) Compounds having – CHO group reduce Tollen’s
37. (d) I2 and Na2CO3 react with acetophenone (C6H5COCH3)
reagent to silver mirror. It is called silver mirror test.
to give yellow ppt. of CHI 3 but benzophenone
(C6H5COC6H5) does not and hence can be used to O O
|| ||
distinguish between them. H C OH CH3 (CHOH) 3 C H
38. (b) When benzaldehyde is refluxed with aqueous alcoholic
(a) (b)
potassium cyanide, two molecules of benzaldehyde
condense together to form benzoin O
H O Both (a) and (b) have – C – H group so both of them
KCN (alc)
—C +C— give positive silver mirror test.
O H 48. (b) Tollen’s reagent is ammonical AgNO3. Aldehydes form
H O silver mirror with it and ketones do not show any
— C– C — change. So Tollen’s reagent is used to distinguish
OH between aldehydes and ketones.
Benzoin 49. (a) Aldehydes and ketones are reduced to alkanes by
39. (b) CH 3 COCH 3 3I 2 4 NaOH Clemmensen reduction.
Acetone
Zn–Hg/HCl H
CHI 3 3NaI CH 3COONa 3H 2 O C=O C + H2O
Iodoform H
Thus acetone reacts with iodine to form iodoform in 50. (a) Wolf- Kishner reduction
the presence of NaOH. (i) NH2 – NH2 H
40. (c) Aldol condensation is given by the compounds which C=O C
(ii) KOH H
contain hydrogen atom. As the given compound
51. (c) Only aldehydes and ketones react with
does not contain hydrogen atom. Hence it does not
2, 4-dinitrophenylhydrazine.
undergo aldol condensation.
O OH
O || |
( H 2O ) HCN
41. (b) CH3–C=O + H2N–NH–C–NH2 52. (b) CH3 C H CH3 C H
acetaldehyde |
H CN
O OH
CH3–CH=N–NH–C–NH2 Hydrolysis
|
acetaldehyde semicarbazone CH3 C H
|
42. (d) Iodoform test is exhibited by ethyl alcohol, COOH
acetaldehyde, acetone, methyl ketone and those Lactic acid
alcohols which possess CH3 CH(OH)-group. As 53. (d) HCHO does not undergo iodoform test, while
3-pentanone does not contain CH3CO-group therefore
acetaldehyde undergoes iodoform test ( I 2 in presence
it does not give iodoform test.
of base) to form yellow precipitate of iodoform.
O
54. (d) Aldehydes can be oxidised by all the three given
C–CH3 CH2–CH3 reagents.
55. (c) Iodoform test is given by compounds which have
43. (b) Zn-Hg/HCl CH3CO group.
O
Phenyl methyl Ethyl ||
ketone benzene CH 3 CH 2 CH 2 C CH 3
This reaction is known as Clemmensen's reduction. 2-pentanone
EBD_7207
456 ALDEHYDES, KETONES AND CARBOXYLIC ACIDS

O 59. (a) Aldehydes and ketones having at least one -hydrogen


|| atom in presence of dilute alkali give -hydroxy
CH 3 CH 2 C CH 2 CH 3 aldehyde or -hydroxy ketone
3-pentanone
O
2-pentanone has CH3CO group, so it gives iodoform ||
test, while 3-pentanone does not have CH3CO group, CH3 C HCH 2 CHO
|
so it does not give iodoform test. H
56. (c) In cross aldol condensation aromatic aldehydes or Acetaldehyde
ketones (with or without -hydrogen) react with
aldehydes, ketones or esters having -hydrogen atoms OH
in the presence of dilute alkali to form a -unsaturated |
dil.NaOH
CH3 C CH 2 CHO
carbonyl compound. |
Example, H
Aldol
OH
(i) C6 H5CHO CH3CHO CH3 CH CH.CHO
Benzaldehyde Acetaldehyde H 2O Crotonaldehyde
C6 H5CHOHCH 2 CHO 60. (b) Clemmensen reduction is
– H2O Zn–Hg/HCl
C=O CH 2
C6 H 5 CH CHCHO 61. (a) Aldehydes, other than formaldehyde, when treated
Cinnamaldehyde with RMgX give 2º alcohols.

CH3 O
O
| 62. (a) R C H + NH2 NH2 R C N NH2
(ii) C6 H 5 C O H 3C C C 6 H 5 H
Acetophenone Aldehyde Hydrazine Aldehyde hydrazone

CH3 O O OH
| || |
Aluminium HCN
C6 H 5 C CH C C6 H5 63. (a) R– C –R R-– C — CN
t-butoxide KCN |
Dyprone
R (A)
R R OMgX
57. (c) C = O + R'MgX C OH
R R R' |
Reduction by
HOH
R – C – CH 2 NH 2
LiAlH 4 (B) |
R R
R C – OH + Mg(OH)X 64. (b)
R'
3° alcohol 65. (d) Iodoform test is given by compounds having CH3CO–
group or secondary alcohols having CH3– as one of
58. (a) When acetaldehyde is treated with alcohol in the alkyl groups, i.e., CH3CHOHR or CH3CH2OH because
presence of dry HCl , then acetal is formed it is readily oxidised by halogen (present in reagent) to
H
CH 3 CH 3 OR |
dry HCl CH3CHO which has CH C O group.
C= O + ROH C 3
66. (d)
H H OH 67. (a) Aldehydes (e.g. CH3CHO) restore the pink colour of
Hemiacetal Schiff’s reagent.
68. (a) fusion
CH 3 OR C 6 H 5 COC 6 H 5 KOH
ROH
H2O + C C6 H 6 C 6 H 5COO K
69. (c) The nucleophile is SO3 HSO3– , SO3Na
– – not
H OR
Acetal 70. (c) Wolf-Kishner reduction is reduction of carboxyl
compound into alkane.
Hence, option (a) is correct. 71. (b) Ammonical AgNO3 is Tollen’s reagent.
ALDEHYDES, KETONES AND CARBOXYLIC ACIDS 457
72. (d) 73. (c) O
74. (d) These reactions lead to replacement of oxygen atom C OMgBr
MgBr
of carbonyl group to form hydrazones and oximes.
+
75. (a) Cannizzaro’s reaction is shown by aldehydes lacking (i) CO2 H3O
-H-atom. Aldol condensation reactions are shown
by aldehydes having -H-atoms.
O
76. (b) NaBH4 selectively reduces the aldehyde group to
alcohol without affecting double bond in a organic C O H
compound. So, X is NaBH4.
NaBH + Mg(OH)Br
4
C6 H5 CH = CHCHO C6 H5 CH = CHCH2OH
77. (a) All ketones in (i), (ii) and (iii) contain abstractable 'P'
Benzoic acid
alpha–proton while all aldehydes do not contain
alpha–hydrogen. 96. (c)
97. (d) In carboxylates (conjugate base of carboxylic acids),
78. (a) Benzaldehyde undergoes Cannizzaro reaction, which
resonance is more significant because the two
forms benzoic acid and benzylalcohol as the product.
resonating structures are similar, while in phenoxide,
79. (c) 80. (b) 81. (a) the resonating structures are not equivalent, alkoxide
82. (d) Automobile exhausts are artificial source of isobutyric ions do not show resonance.
acid. 98. (a) 99. (a)
83. (c) Vinegar is 6 - 8% solution of acetic acid. 100. (c) Bromine is less electronegative than F, further in
84. (b) The overall reaction involved is BrCH2CH2COOH, Br is more away from the –COOH
H 2O H 2O group than in CH3CHBrCOOH.
CH 3 CN CH 3 CONH 2
O CH3
HCl || |
CH3 COO NH 4 CH3COOH + NH 4 Cl 101. (c) CH 3 C OH HO– CHCH 3
Ethanoic acid Propan-2-ol
On reduction cyanides yield 1 amines. They do not
undergo decarboxylation or electrolysis.
O CH3
R || |
COOH CH3 C O– CHCH3
85. (a) KMnO 4 / OH – 102. (a)
K 2Cr2O7 or dil. HNO3 103. (c) pKa = –logKa; HCOOH is the strongest acid and hence
R = Any alkyl group it has the highest Ka or lowest pKa value.
Benzoic acid
104. (c)
86. (b) Both C–O bonds are identical and each O possesses 105. (b) LiAlH4 in presence of ether can be used to convert
partial negative charge. acetic acid into ethanol.
87. (c) Formic acid cannot be prepared by Grignard reagent.
ether
88. (b) CH3COOH + 3LiAlH4
acetic acid
CH3 CHCl2 CHO [CH3CH2O]4AlLi + 2LiAlO2 + 4H2
H 2O
Cl 2 / h
373 K H
[CH3CH2O]4AlLi CH3CH2OH
Toluene Benzal chloride Benzaldehyde ethanol
89. (d) 106. (c) Carboxylic acids are weak acids.
90. (c) Carbonyl group acts as a deactivating and 107. (c) Removal of CO2 from carboxylic acid is called
metadirecting group. decarboxylation.
91. (d) Primary and secondary alkyl groups oxidised to give 108. (b) It is a test for –COOH gp.;
carboxylic acid while tertiary alkyl group remain R–COOH+NaHCO3 RCOONa + H2O + CO2 .
unaffected. 109. (d)
92. (b) 93. (d) COOH COOC2H5
94. (b) Due to H-bonding.
HCl
95. (b) Grignard reagent forms addition product with bubbled 110. (c) + C2H5OH + H2O
Ethanol
dry
carbondioxide which on hydrolysis with HCl yields
benzoic acid. This process is known as esterification.
EBD_7207
458 ALDEHYDES, KETONES AND CARBOXYLIC ACIDS
111. (c) This reaction is an example of Hell - Volhard Zelinsky 122. (c) Chlorine is electron withdrawing group. Further
reaction. In this reaction acids containing – H on inductive effect is stronger at position than
treatment with X2 /P give di-halo substituted acid. -position. i.e.,
Br2 /P
CH 3 – CH 2 COOH CH 3 CBr2 COOH
CH 2ClCOOH CH 2 ClCH 2 COOH
112. (c) 3CH 3COOH PCl3 CH 3 COCl + H3PO3 123. (b)
Acetyl Phosphorous
Chloride acid STATEMENT TYPE QUESTIONS
113. (b) 124. (b) Carbonyl compounds have substantial dipole moments
114. (a) –COOH group when attached to benzene ring and are polar in nature. The high polarity of the
deactivates the ring and substitution occurs at carbonyl group is due to resonance.
m-position. (HNO3 + H2SO4) is a source of + NO2 125. (a) Primary alcohols on oxidation give carboxylic acids as
(electrophile) which attacks at m-position. the final product, of course through aldehydes.
COOH COOH Oppenauer oxidation involves oxidation of 2º alcohols
to ketones, and not for the oxidation of 1º alcohols.
+ HNO3 H2SO4 126. (c) If the aldehyde has a boiling point less than 100°C, it
can be prepared by the oxidation of 1° alcohols with
NO2
3- nitrobenzoic acid
regular oxidising agents like acidic permanganate or
dichromate. Since the aldehyde has a lower boiling
115. (b) Br2 / P point than the alcohol, it is distilled off as soon as it is
RCH 2 COOH R CH COOH
HVZ reaction | formed ; so further oxidation to a carboxylic acid is
Br minimized.
'X' 127. (d) The solubility of aldehydes and ketones decreases
rapidly on increasing the length of alkyl chain.
NH3
R CH COOH 128. (a) Aldehydes are generally more reactive than ketones
(Excess) | in nucleophilic addition reactions due to steric and
NH 2 electronic reasons. Sterically, the presence of two
'Y' relatively large substituents in ketones hinders the
116. (d) The yield of product in a reversible reaction can be approach of nucleophile to carbonyl carbon than in
increased by (i) removing one of the products, (ii) aldehydes having only one such substituent.
taking either of the reactant in excess. Electronically, aldehydes are more reactive than
117. (d) Use of SOCl2 and ClCOCOCl forms gaseous by- ketones because two alkyl groups reduce the
products which can be easily removed, giving better electrophilicity of the carbonyl carbon more effectively
yield of RCOCl. Further, oxalyl chaloride is particularly than in former.
easy to use becasue any excess of it can be easily 129. (b) The hydrogen atom that is added to the carbonyl
evaporated due to its low b.p. (62ºC) carbon of the aldehyde in the reduction is derived
directly from the other aldehyde molecule as a hydride
O O O
|| || || ion. The second hydrogen that is added to the
R C OH Cl C C Cl negatively charged oxygen is coming from the solvent
O (consult mechanism of Cannizzaro reaction). Oxidation
|| of one molecule of the compound at the expense of
R C Cl HCl CO CO 2 other molecule of the same compound is known as
118. (b) disproportionation.
119. (a) Salicylic acid, because it stabilizes the corresponding 130. (b) First two steps of the esterification make the question
salicylate ion by intramolecular H-bonding. clear
120. (c) Cl 2CHCOOH is most acidic because it has two O OH
+

chlorine at -position. H
+
R–O–H
121. (a) An electron releasing substituent (+I) intensify the CH3 — C — OH CH3 — C — OH
negative charge on the anion resulting in the decrease Protonated acid
(carbonyl C is more
of stability and thus decreases the acidity of the acid. electrophilic than
Hence acid character decreases as the + I-effect of the that of parent acid)
alkyl group increases as OH
CH3– < CH3CH2– < CH3CH2CH2– < CH3CH2CH2CH2–
R–O–H
Hence the order becomes : (i) > (ii) > (iii) > (iv) CH — C — OH CH3 — C — OH
Protonated acid +
HO R
ALDEHYDES, KETONES AND CARBOXYLIC ACIDS 459
+ +
MATCHING TYPE QUESTIONS 148. (c) (a) C = O + H2NNH2 H
C – OH
H
C = NNH2
131. (a) 132. (c) 133. (b) 134. (d) 135. (b) NHNH2
136. (c) 137. (c) (b) In the reduction of carbonyl group with LiAlH4 or
NaBH4, a hydride ion is transferred from the metal
ASSERTION-REASON TYPE QUESTIONS to the carbonyl carbon (nucleophilic addition)
138. (a) 139. (a) 140. (b) H
141. (a) The molecular mass of acetic acid in benzene is 120 – –
C = O + H– AlH3 C – OAlH3
instead of 60 because the carboxylic acids exists as
cyclic dimers in which two molecules of the acid are 149. (d) Being reversible reaction, the backward reaction i.e.
held together by two strong hydrogen bond. acetal -hemiacetal step can be restricted by minimizing
water content, i.e. by using dry HCl. The step hemiacetal
CRITICAL THINKING TYPE QUESTIONS - aldehyde can be restricted by using excess of alcohol.
150. (c) First step in Cannizzaro reaction is the nucleophilic
142. (d) addition of OH– on the carbonyl carbon.
143. (b) In structure II, presence of positive charge on oxygen
causes the displacement of electrons toward oxygen, H –
H
making carbon more electron deficient than that in OH –
R–C=O (fast) R–C–O
unprotonated carbonyl group.
144. (c) It is the reason for the given fact. OH
145. (d) Higher the electron deficiency on cabonyl carbon, more
146. (c) With ammonia, HCHO forms hexamethylenetetramine, easier will be the attack of the nucleophile
CH3 CHO gives acetaldehydeammonia addition (OH–) on its carbon. Futher, the attack of OH– on the
product, while C6H5CHO gives hydrobenzamide. carbonyl carbon is more easy in case of HCHO because
147. (d) Aldol condensation : its carbon is least hindered having two hydrogens
(steric effect). Thus the intermediate I is formed very
OH New C–C bond
easily which donates hydride ion to another aldehyde

OH
2CH3CHO H3C–C–CH2CHO and thus itself oxidised.
H H
|
Kolbe reaction : Ist
H C O OH step
Easier because of electronic
ONa
ONa OH New C–C bond and steric effects
COOH H H H
CO
CO2,140ºC
,140ºC – –
R–C=O
Pressure
Pressure H–C–O 2nd Step
H–C=O +R–C –O
(slow)
OH OH H
OH I
151. (c) OH– and –CH2CHO act as nucleophile in the first two
steps.
CHCl3
Reimer-Tiemann reaction : O
3KOH
|
OH CH 3CHO
CH3 CHO C H 2 CHO CH 3 C H
|
New C–C bond CH 2CHO
OH
CHO OH
CHCl |
+ 3KCl + 2H2O H 2O
3KOH
CH3 CH
(as an acid) |
CH2CHO
Wurtz Fittig reaction :

Na
H3CCl + 2Na + Cl
152. (c)

H3C + 2NaCl H
C6 H5CHO –
C6H5 – C – O + C6H5 – C = O
New C–C bond
H OH
EBD_7207
460 ALDEHYDES, KETONES AND CARBOXYLIC ACIDS
153. (d) If we observe the haloform reaction carefully, we see 157. (a) Aldol condensation involves an aldehyde or ketone
that –COCH3 group is first halogenated to the trihalo having an –hydrogen atom. This type of
–COCX3 through monohalogeno and dihalogeno condensation occurs in presence of dilute base (i.e.,
compound. It is the –COCX 3 part which then dil NaOH).
undergoes nucleophilic addition. The product easily Only CH3COCH3 will give aldol condensation (Both
loses –CX3 since it is a very good leaving group. HCHO and C6H5CHO lack -hydrogen).

O O 158. (d) H3 C

X2, OH C = O + CH 3 MgI
CH3 – C – CH3 CH3 – C – CH2Cl
H 3C
I II

H3 C
O O H OH H2 O
C O MgI
CH3 – C – CHCl2 CH3 – C – CCl3 H3 C
III IV CH3
– CH3
O O
OH
– |
CH3 – C – CCl3 CH3 – C – CCl3 H3C C OH + Mg(OH)I
|
OH CH3

Tert butyl
O alcohol

CH3 – C + CCl3 159. (d) Ketones do not respond to Tollen’s test. Aldehydes
respond to Tollen’s test.
OH
160. (b) R R OH
Thus all compounds (I to IV) are ultimately converted NH3
to CHCl3 (chloroform).
C = O HCN C
R R CN
154. (c) 2CH3CHO 1st Product,
2 CH 3CH 2CHO 2 nd Product R NH2 R NH2
HOH
C C
CH3CH 2CHO CH3CHO 3rd Product; R CN R COOH

CH3 C H 2 CHO CH 3CHO 4 th Product 161. (d) 162. (a) 163. (a)
164. (b) The car bon atom of the carbonyl group of
155. (d) Dihydrogen sodium phosphate (NaH2PO4) does not
benzaldehyde is less electrophilic than carbon atom of
have a lone pair of electrons on the P atom. As such it the carbonyl group present in propanal. The polarity
can not act as a nucleophile and hence does not react of the carbonyl group is reduced in benzaldehyde due
with aldehydes and ketones. to resonance as shown below and hence it is less
156. (d) Aldehydes which contain a -hydrogen on a reactive than propanal.
saturated carbon, i.e., CH 3CH 2CHO undergo aldol 165. (b) Aldehydes and ketones react with hydrogen cyanide
condensation. (HCN) to yield cyanohydrins. This reaction occurs
very slowly with pure HCN. Therefore, it is catalysed
H CH3 by a base and the generated cyanide ion (CN– ) being
| |
a stronger nucleophile readily adds to carbonyl
CH3CH 2 – C O H C CHO
propanol | compounds to yield corresponding cyanohydrins
H 166. (b) Dry hydrogen chloride protonates the oxygen of the
carbonyl compounds and therefore, increases the
H CH3 electrophilicity of the carbonyl carbon facilitating the
| |
OH nucleophilic attack by the alcohol molecule. Dry HCl
CH3CH 2 C CHCHO
| gas also absorbs the water produced in these reactions
OH thereby shifting the equilibrium in the forward
3-hydroxy-2-methylpentanal
direction.
ALDEHYDES, KETONES AND CARBOXYLIC ACIDS 461

167. (a) Because of resonance in benzaldehyde which is not


O
possible in case of acetaldehyde the positive charge
3 ® CH CH COO- NH+
CH3CH 2 - C- OH ¾¾¾
NH
on the carbonyl carbon decreases and hence there is 3 2 4
decrease in reactivity.
(A) (B)
168. (c) Acetal formed upon reaction of ethylene glycol and
D
HCl, which is unaffected by base hence unwanted ¾¾® CH 3CH 2 CONH 2
reaction does not occur due to presence of carbonyl (C)
group.
169. (c) 2, 2-dimethyl propanal gives Tollen’s test and KOH Br2
3-methylbutan-2-one gives iodoform test. CH 3CH 2 NH 2
170. (d)
179. (a) The correct order of increasing acid strength
OH CF3COOH > MeOCH2COOH > CH3COOH
3 1 > (Me)2CH.COOH
H3C 2 COOH
171. (a) Electron withdrawing groups increase the acid strength
6 4
and electron donating groups decrease the acid
5 CH 3
strength.
IUPAC name of the structure is 3-ethyl-2-hydroxy 180. (a) Grignard reagents and nitriles are useful for converting
-4-methylhex-3-en-5-ynoic acid. alkyl halide into corresponding carboxylic acids having
one carbon atom more than that present in alkyl halides.
172. (c) CN NaOH
R X R CN RCOONa
COCH3
173. (c) C6H 6
181. (a) CH COOH +PCl CH3COCl
3 5 AnhydAlCl 3
H 2O [A]
(C 6 H5CO)2 O 2C 6 H5 – COOH
174. (b) Friedle Craft
benzoic anhydride benzoic acid reaction

H2O OH OMgBr
C6 H5COOCOCH3 C6 H5 COOH CH 3COOH H
+
MgBrC2H5
bezoyl ethanoic benzoic acid ethanoic acid C2H5 – C – CH3 ether C2H5 – C – CH3
anhydride
hydrolysis
175. (b) 176. (a)
177. (b) Recall that presence of electron-withdrawing group (C)
increases, while presence of electron-releasing group
decreases the acidity of carboxylic acids. 182. (c) An alkyl group attached to benzene ring can be
oxidised only when it contains at least one -hydrogen
O O atom. Thus here –CH3 group is oxidised and Me3C–
|| || group not. However, Me 3 C– group may cause
ClCH 2COOH H C OH CH 3
C OH
(electron-withdrawing gp.)
oxidation of the benzene ring to –COOH.
(Electron-releasing character
increasing from Left to Right) 183. (a) 184. (a)
185. (c) Electron withdrawing substituent (like halogen, —NO2,
O C6H5 etc.) would disperse the negative charge and
|| hence stabilise the carboxylate ion and thus increase
C2H5 C OH acidity of the parent acid. On the other hand, electron-
releasing substituents would intensify the negative
charge, destabilise the carboxylate ion and thus
NH3 Br2
178. (d) A B C CH3CH 2 NH 2 decrease acidity of the parent acid.
(I) II KOH,(III)
Electronegativity decreases in order
Reaction (III) is a Hofmann bromamide reaction. Now F > Cl > Br
formation of CH3CH2NH2 is possible only from a and hence –I effect also decreases in the same order,
compound CH3CH2CONH2(C) which can be obtained therefore the correct option is
from the compound CH3CH2COO– NH+4 (B). [FCH2COOH > ClCH2COOH > BrCH2COOH >
Thus (A) should be CH3CH2COOH CH3COOH]
EBD_7207
462 ALDEHYDES, KETONES AND CARBOXYLIC ACIDS

186. (d) V is most stable because its anion is stabilized to a COOH COOH
greater extent through H – bonding with H atom of OH OH OH
HO
present on both ortho-positions ; followed by II in > >
which one OH group is present. Compound IV comes
next to II because here –OCH3 group is present in ortho V II

position which although is not capable of forming


COOH COOH COOH
H–bonding yet more acidic than p-HOC6H4COOH (III)
due to ortho effect. Compound III is less acidic than OCH3
benzoic acid because of electron-releasing group in > >
the para position. Thus IV I
OH
III
27
AMINES

7. The correct IUPAC name for CH2 = CHCH2NHCH3 is


FACT / DEFINITION TYPE QUESTIONS
(a) Allylmethylamine
1. A secondary amine is (b) 2-amino-4-pentene
(a) a compound with two carbon atoms and an –NH2 (c) 4-aminopent-1-ene
group. (d) N-methylprop-2-en-1-amine
(b) a compound containing two –NH2 groups. 8. Amines play an important role in the survival of life.
(c) a compound in which hydrogens of NH3 have been Naturally they are found in
replaced by two alkyl groups. (a) proteins (b) vitamins
(d) a compound with an –NH2 group on carbon atom in (c) alkaloids (d) All of these
number two position. 9. Intermediates formed during reaction of RCONH2 with Br 2
2. The general formula of quaternary ammonium compound and KOH are
is (a) RCONHBr and RNCO (b) RNHCOBr and RNCO
(a) R–NH2 (b) R3N (c) RNHBr and RCONHBr (d) RCONBr2
+ – 10. Which of the following reactions will not give a primary
(c) R4N X (d) NH4X
amine?
3. The total number of electrons around the nitrogen atom in
(a) Br2 / KOH
amines are CH 3 CONH 2
(a) 8 (b) 7 (b) CH 3CN
LiAlH 4
(c) 4 (d) 3 LiAlH 4
(c) CH 3 NC
4. The IUPAC name of the compound having formula,
(d) LiAlH 4
O C CH CH2 is CH 3CONH 2
11. Propionamide on Hofmann degradation gives –
OH NH2 OH (a) methyl amine (b) ethyl amine
(a) 3-amino-hydroxy propine acid (c) propyl amine (d) ethyl cyanide
12. Secondary amines could be prepared by
(b) 2-amino-propan-3-oic acid
(a) reduction of nitriles
(c) amino hydroxy propanoic acid
(b) Hofmann bromamide reaction
(d) 2-amino-3-hydroxy propanoic acid
(c) reduction of amides
5. The number of primary amines of formula C4H11N is : (d) reduction of isonitriles
(a) 1 (b) 3 13. Gabriel’s phthalimide synthesis is used for the preparation
(c) 4 (d) 2 of
6. What is the IUPAC name of the following compound ? (a) Primary aromatic amines
(b) Secondary amines
NH 2
(c) Primary aliphatic amines
(d) Tertiary amines
14. Ethyl amine can be obtained by the
(a) 2-methyl-4-hexanamine (a) Action of NH3 on ethyl iodide.
(b) 5-methyl-3-hexanamine (b) Action of NH3 on ethyl alcohol.
(c) 2-methyl-4-amino hexane (c) Both (a) and (b)
(d) 5-methyl-3-amino hexane (d) Neither (a) nor (b)
EBD_7207
464 AMINES
15. Treatment of ammonia with excess of ethyl iodide will yield 24. Which of the following will give primary amine only ?
(a) diethylamine (i) ammonia + propylchloride
(b) ethylamine (ii) potassium pthalimide + ethylchloride
(c) triethylamine (iii) potassium pthalimide + chlorobenzene
(d) tetraethylammonium iodide (a) (i) and (ii) (b) (i) and (iii)
16. For alkylation of ammonia which of the following is not (c) (ii) and (iii) (d) (i), (ii) and (iii)
used ? 25. Amines have
(a) CH3–X (b) CH3–CH2–X (a) Garlic odour (b) Fishy odour
(c) (CH3)2CH–X (d) (CH3)3C–X (c) Jasmine odour (d) Bitter almonds odour
17. Which of the following amines can be prepared by Gabriel 26. Aniline is less soluble in water than ethyl amine due to
method ? (a) resonance stablization of benzene ring
(i) CH3CH2NH2 (ii) (CH3)2CHNH2 (b) resonance stabilization of anilium ion
(iii) (CH3)3CNH2 (iv) C6H5NH2 (c) more hydrophobic nature of C6H5 group than C2H5
(a) (i) and (iii) (b) (ii) and (iv) group
(c) (i), (ii) and (iii) (d) (i) and (ii) (d) more hydrophobic nature of C6H5 group than C2H5
18. Amongst the given set of reactants, the most appropriate group
for preparing 2° amine is ___________. 27. Which of the following should be most volatile?
(a) 2°R–Br + NH3 (I) CH3CH2CH2NH2 (II) (CH3)3N
(b) 2°R–Br + NaCN followed by H2/Pt CH3CH2
(c) 1°R–NH2 + RCHO followed by H2/Pt (III) NH (IV) CH3CH2CH3
(d) 1°R–Br (2 mol) + Potassium phthalimide followed by CH3
H3O+/heat (a) II (b) IV
19. The best reagent for converting 2 – phenylpropanamide (c) I (d) III
into 2-phenylpropanamine is ___________. 28. Amines behave as
(a) excess H2 (a) lewis acids (b) lewis bases
(b) Br2in aqueous NaOH (c) aprotic acids (d) amphoteric compounds
(c) iodine in the presence of red phosphorus 29. The basic character of amines is due to
(d) LiAlH4in ether (a) presence of nitrogen atom
20. Which of the following methods of preparation of amines (b) lone pair of electrons on nitrogen atom
will give same number of carbon atoms in the chain of (c) tetrahedral structure
amines as in the reactant? (d) high electronegativity of nitrogen
(a) Reaction of nitrite with LiAlH4. 30. Aliphatic amines are.....basic than NH3 but aromatic amines
(b) Reaction of amide with LiAlH4 followed by treatment are......basic than NH3.
with water. (a) more, less (b) less, more
(c) Heating alkylhalide with potassium salt of phthalimide (c) both (a) and (b) (d) None of these
followed by hydrolysis. 31. Substitution of one alkyl group by replacing hydrogen of
(d) Treatment of amide with bromine in aquesous solution primary amines
of sodium hydroxide. (a) increases the base strength
21. The reduction of nitro compounds is most preferred in the (b) decreases the base strength
presence of (c) remains the same
(a) Pd/H2 in ethanol (b) Sn + HCl (d) None of the above
(c) finely divided Ni (d) iron scrap and HCl. 32. Which of the following is not characteristic of amines?
22. An alkyl or benzyl halide on reaction with an ethanolic (a) They smell like ammonia
solution of ammonia undergoes (b) They are inflammable in air
(a) electrophilic substitution reaction (c) They show the property of hydrogen bonding
(b) nucleophilic substitution reaction. (d) They are amphoteric in nature
(c) free radical mechanism. 33. The correct order of basicity in amines
(d) nucleophilic addition reaction. (i) C2H5NH2 (ii) CH3NH2
23. In the ammonolysis of alkyl halides the halogen atom is (iii) (CH3)2NH (iv) (CH3)3N
replaced by an amino(–NH2) group which of the following (a) (i) < (iv) < (ii) < (iii) (b) (iv) < (ii) < (iii) < (i)
represent the correct order of reactivity of halides with (c) (i) < (ii) < (iii) < (iv) (d) (ii) < (iii) < (iv) < (i)
amines. 34. The conjugate base of (CH3)2NH+2 is
(a) RBr > RI > RCl (b) RI > RCl > RBr (a) (CH3)2NH (b) (CH3)2N+
(c) RI > RBr > RCl (d) RCl > RBr > RI (c) (CH3)3N + (d) (CH3)2N–
AMINES 465

35. High basicity of Me2 NH relative to Me3N is attributed to: 42. Which statement is not true among the following?
(a) effect of solvent (b) inductive effect of Me (a) Amines are bases
(c) shape of Me2NH (d) shape of Me3N (b) They turn red litmus blue
36. The correct order of basicity of the following compounds (c) Trimethyl amine is less basic than dimethyl amine
NH 2 NH 2 NH 2 (d) Amines yield alcohols on aqueous hydrolysis.
43. Aniline is used
(a) in crimping of wool (b) in dyeing industry
A B C (c) in making of glue (d) in fast drying vanish
44. Which of the following statements about primary amines is
NO 2 ‘False’ ?
(a) B > A > C (b) A > B > C (a) Alkyl amines are stronger bases than aryl amines
(c) C > A > B (d) C > B > A (b) Alkyl amines react with nitrous acid to produce
37. Which of the following statement is correct ? alcohols
(a) Ammonia is more basic than methylamine. (c) Aryl amines react with nitrous acid to produce phenols
(b) Methylamine is more basic than ammonia. (d) Alkyl amines are stronger bases than ammonia
(c) Dimethylamine is less basic than methylamine.
45. Mark the correct statement
(d) Dimethylamine is less basic than trimethylamine.
(a) Methylamine is slightly acidic
38. Which of the following compounds is most basic?
(b) Methylamine is less basic than ammonia
(a) O2N NH2 (c) Methylamine is a stronger base than ammonia
(d) Methylamine forms salts with alkalies.
46. For carbylamine reaction, we need hot alcoholic KOH and
(b) CH2NH2 (a) any primary amine and chloroform
(b) chloroform and silver powder
(c) a primary amine and an alkyl halide
(c) N – COCH3
(d) a monoalkylamine and trichloromethane.
H 47. The compound obtained by heating a mixture of a primary
amine and chloroform with ethanolic potassium hydroxide
(d) NH2 (KOH) is
(a) an alkyl cyanide (b) a nitro compound
39. Which of the following compounds is the weakest Bro nsted
base? (c) an alkyl isocyanide (d) an amide
NH2 NH 48. R NH 2 CH3COCl A
2
(excess)
(a) (b) The product (A) will be –
(a) RNHCOCH3 (b) RN(COCH3)2
OH OH (c) RN(COCH3 )3 Cl (d) R – CONH2
(c) (d) 49. Carbylamine reaction is used for the detection of
(a) aliphatic 2° amines (b) aliphatic 1° amines
40. The correct decreasing order of basic strength of the (c) aromatic 1° amines (d) Both (b) and (c)
following species is __________ H2O, NH3, OH–, NH2– 50. In the reaction,
(a) NH2– > OH– > NH3 > H2O RNH 2
HNO2
ROH H 2O C ; C is
(b) OH– > NH2– > H2O > NH3 (a) NH3 (b) N2
(c) NH3 > H2O > NH2– > OH–
(c) O2 (d) CO2
(d) H2O > NH3 > OH–> NH2–
51. An organic amino compound reacts with aqueous nitrous
41. Which of the following factors affect the basic strength of
amine? acid at low temperature to produce an oily nitrosoamine.
(i) Inductive effect The compound is
(ii) Steric hinderance (a) CH3NH2 (b) CH3CH2NH2
(iii) Solvation effect (c) CH3CH2NHCH2CH3 (d) (CH3CH2)3N
(iv) Solubility in organic solvents. 52. Ethylamine reacts with HNO2 giving :
(a) (i) and (iv) (b) (i), (ii) and (iii) (a) C2H5OH (b) C2H5NO2
(c) (ii) and (iii) (d) (ii) and (iv) (c) NH3 (d) C2H6
EBD_7207
466 AMINES
53. Primary amines can be distinguished from secondary and (b) benzene sulphonic acid
tertiary amines by reacting with (c) ethyl oxalate
(a) Chloroform and alcoholic KOH (d) acetyl chloride
(b) Methyl iodide 66. The reaction,
(c) Chloroform alone C6H5NH2+ ClCOC6H5 C6H5NHCOC6H5 is called:
(d) Zinc dust (a) Friedel-crafts reaction
54. Which of the following is not correct ? (b) Claisen condensation
(a) Ethyl amine and aniline both have – NH2 group (c) Benzoylation or Schotten Baumann reaction
(b) Ethyl amine and aniline dissolve in HCl (d) None of these
(c) Ethyl amine and aniline both react with CHCl3 and 67. Which of the following statements is not correct regarding
KOH to form unpleasant smelling compound aniline?
(d) Ethyl amine and aniline both react with HNO2 in cold (a) It is less basic than ethylamine
to give hydroxy compounds (b) It can be steam-distilled
55. Hinsberg reagent is (c) It reacts with sodium to give hydrogen
(a) C6H5SO3H (b) C6H5NO (d) It is soluble in water
(c) C6H5SO2Cl (d) C6H5N2Cl 68. Benzylamine may be alkylated as shown in the following
56. Reaction of aniline with benzaldehyde is equation:
(a) substitution (b) addition C6 H5 CH 2 NH 2 R – X C 6 H5CH 2 NHR
(c) condensation (d) polymerization
Which of the following alkylhalides is best suited for this
57. The amine that does not react with acetyl chloride is
reaction through SN1 mechanism?
(a) CH3NH2 (b) (CH3)2NH (a) CH3Br (b) C6H5Br
(c) (CH3)3N (d) None of these (c) C6H5CH2Br (d) C2H5Br
58. Which of the following compounds cannot be identified
69. The product of the following reaction is _________.
by carbylamine test?
NHCOCH3
(a) CH3CH2NH2 (b) CHCl3
(c) C6H5NH2 (d) C6H5–NH–C6H5
59. (CH3)2CHNH2 is reacted with excess acetic anhydride, the + Br2/CH3COOH
compound formed is
NHCOCH3 NHCOCH3
(a) (CH3)2CHNCOCH3 (b) (CH3)2CN(COCH3)2
(c) (CH3)2CHOH (d) (CH3)2CN(COOCH3)2 Br
(i) (ii)
60. In order to distinguish between C2H5–NH2 and C6H5–NH2,
which of the following reagent is useful
(a) Hinsberg’s reagent (b) HNO2 Br
(c) CHCl3 + KOH (d) NaOH NHCOCH3 NHCOCH3
61. All three amines 1°, 2°, 3° react with Br Br
1. H2O 2. R–X (iii) (iv)
3. HCl 4. (CH3CO)2O Br
(a) 1, 2 (b) 4 only Br
(c) 1, 2, 4 (d) 1, 2, 3 (a) (i) and (iii) (b) (i) and (ii)
62. –NH2 group in aniline is (c) (iii) and (iv) (d) (i), (ii) and (iii)
(a) only o-directing (b) only p-directing 70. Aniline and other arylamines are usually colourless but
(c) only m-directing (d) o-and p-directing get coloured on storage due to___________.
63. Strong activating effect of –NH2 group is reduced by using (a) hydrolysis (b) dehydration
(a) CH3COCl (b) CH3Cl (c) reduction (d) atmospheric oxidation
(c) CH3ONa (d) CH3–CHO 71. The acylation reaction of amines is carried out in presence
64. When bromination of aniline is carried out by protecting of pyridine because
–NH2. The product is (i) pyridine is stronger base than amine.
(a) o-bromoaniline (ii) pyridine is weaker base than amine.
(b) 2, 4, 6 tribromoaniline (iii) pyridine removes HCl formed and shifts the equilibrium
(c) p-bromoaniline to the right hand side.
(d) mixture of o-and p-bromoaniline (iv) pyridine removes HCl formed and shifts the equilibrium
65. Hinsberg’s method to separate amines is based on the to the left hand side.
use of (a) (i) and (iii) (b) (ii) and (iv)
(a) benzene sulphonyl chloride (c) (ii) and (iii) (d) (i) and (iv)
AMINES 467
72. N– ethyl benzene sulphonyl amide is strongly acidic and 81. In the chemical reactions,
soluble in alkali due to presence of
NH2
(a) strong electron donating sulphonyl group.
(b) strong electron withdrawing sulphonyl group. NaNO2 HBF 4
HCl, 278 K
A B
(c) weak electron donating sulphonyl group.
(d) weak electron withdrawing sulphonyl group. the compounds ‘A’ and ‘B’ respectively are
73. Arrange the following in increasing order of their basic (a) nitrobenzene and fluorobenzene
strength? (b) phenol and benzene
p–nitroaniline(1); m–nitroaniline (2); 2,6–trimethylaniline(3); (c) benzene diazonium chloride and fluorobenzene
3–methylanline(4). (d) nitrobenzene and chlorobenzene
(a) 1, 3, 2, 4 (b) 2, 3, 4, 1 82. In the chemical reactions :
(c) 3, 1, 2, 4 (d) 1, 2, 4, 3
NH 2
NH2
NaNO CuCN
NaNO 2 /HCl CuBr 2 A
74. (P) (Q) HCl, 278K
B,
0 C HBr

The compound Q is – the compounds A and B respectively are :


(a) bromobenzene (b) chlorobenzene (a) benzene diazonium chloride and benzonitrile
(c) benzyl bromide (d) benzyl chloride (b) nitrobenzene and chlorobenzene
75. Diazonium salt is obtained when aniline reacts with : (c) phenol and bromobenzene
(a) cold NaOH (b) NaNO2 and HCl (0–5°C) (d) fluorobenzene and phenol
(c) SnCl2 at 10°C (d) N2O at (0 – 5°C) 83. Replacement of N 2 Cl from benzene diazonium chloride
76. Azo dye is prepared by the coupling of phenol and
by iodine can be done by using
(a) diazonium chloride (b) o-nitroaniline
(a) HI (b) NaOI
(c) benzoic acid (d) chlorobenzene (c) PI3 (d) KI
77. In the reaction sequence 84. Product of the following reaction is
NH2 C6H5–N2Cl+C6H5–NH2
NaNO 2 ,HCl CuCN LiAlH 4 NH2
A B C,
0 C
N=N
the product ‘C’ is:
(a)
(a) benzonitrile (b) benzaldehyde
(c) benzoic acid (d) benzylamine
– NH2
N2+Cl Cl

Conc. HCl + N2 N=N


78. Cu2Cl2
(b)
Chlorobenzene

Above reaction is known as:


(a) Strecker's reaction (b) Sandmeyer's reaction N=N NH2
(c) Wohl-Ziegler reaction (d) Stephen's reaction
(c)
79. Which of the following reagents will convert
p-methylbenzenediazonium chloride into p-cresol?
(a) Cu powder (b) H2O NH2
(c) H3PO2 (d) C6H5OH
80. When phenol and benzene diazonium chloride are coupled, N=N
the main product is :
(a) aniline (b) p-hydroxyazobenzene (d)
NH2
(c) azobenzene (d) chlorobenzene
EBD_7207
468 AMINES
85. Which of the following are intermediates in Sandmeyer (iii) The angle C–N–C or C–N–H is slightly more than
reaction ? 109.5°.
(i) C6 H 5 N NCl (ii) C6 H5 N N (a) (i), (ii) and (iii) (b) (i) and (ii)
(c) (i) and (iii) (d) (ii) and (iii)
(iii) C 6 H 5 (iv) C6H5Cl 94. Which of the following statements are correct ?
(a) (ii) and (iii) (b) (i) and (iv) (i) Lower aliphatic amines are soluble in water.
(c) (ii) and (iv) (d) (i) and (ii) (ii) Solubility increases with decrease in molar mass of
86. In the diazotization of arylamines with sodium nitrite and amines.
hydrochloric acid, an excess of hydrochloric acid is used (iii) Higher amines are insoluble in water.
primarily to (iv) Amines are soluble in organic solvents.
(a) Supress the concentration of free aniline available for (a) (i), (ii) and (iii) (b) (i), (iii) and (iv)
coupling (c) (ii), (iii) and (iv) (d) (i) and (iv)
(b) Supress hydrolysis of phenol 95 Which of the following statements are correct ?
(c) Ensure a stoichiometric amount of nitrous acid (i) Primary amines show more intermolecular association
(d) Neutralise the base liberated than secondary amines.
87. Which of the following reagent can be used to convert (ii) Tertiary amines do not show intermolecular
benzenediazonium chloride into benzene? association.
(a) CH3OH (b) H3PO2 (iii) Boiling points of isomeric alkenes follow the order
(c) Br2–H2O (d) LiAlH4 3° > 2° > 1°
88. When benzenediazonium chloride in hydrochloric acid (a) (i) and (iii) (b) (i) and (ii)
reacts with cuprous chloride, then chlorobenzene is formed. (c) (i), (ii) and (iii) (d) (ii) and (iii)
The reaction is called 96. Which of the following is/are correct regarding nitration of
(a) Gattermann reaction (b) Perkin reaction aniline with conc. HNO3 and conc. H2SO4 ?
(c) Etard reaction (d) Sandmeyer reaction
NH2 NH2 NH2
Cu/HCl
89. The reaction ArN 2Cl – ArCl N 2 CuCl is NO2
conc. HNO3
named as _________ . (i) +
conc. H 2SO4
(a) Sandmeyer reaction (b) Gatterman reaction
(c) Claisen reaction (d) Carbylamine reaction NO2
90. Which of the following compound will not undergo azo
coupling reaction with benzene diazonium chloride. + +
(a) Aniline (b) Phenol NH3 NH3
(c) Anisole (d) Nitrobenzene
91. Which of the following cannot be prepared by Sandmeyer’s (ii)
reaction? NO2
(i) Chlorobenzene (ii) Bromobenzene
(iii) The substitution can be explained on the basis of
(iii) Iodobenzene (iv) Fluorobenzene
inductive effect (– I)
(a) (i) and (ii) (b) (ii) and (iii)
(iv) The substitution can be influenced by +M and +E
(c) (iii) and (iv) (d) (i) and (iv) effects.
92. The reagents that can be used to convert benzenediazonium (a) (i), (ii) and (iii) (b) (ii) and (iii)
chloride to benzene are _________. (c) (ii) and (iv) (d) (i) and (iii)
(i) SnCl2/HCl (ii) CH3CH2OH 97. Which of the following statements are correct ?
(iii) H3PO2 (iv) LiAlH4 (i) In Sandmeyer reaction nucleophiles like Cl–, Br– and
(a) (i) and (ii) (b) (ii) and (iii) CN– are indroduced in benzene ring in the presence of
(c) (iii) and (iv) (d) (i) and (iii) Cu+ ion
(ii) In Gattermann reaction nucleophiles are introduced in
STATEMENT TYPE QUESTIONS benzene ring in the presence of copper powder and
HCl.
93. Read the following statements and choose the correct (iii) The yield in Gattermann reaction is found to be better
option. than Sandmayer reaction.
(i) Nitrogen atom in amines is sp3-hybridised. (a) (i) and (ii) (b) (i), (ii) and (iii)
(ii) The geometry of amines is pyramidal. (c) (ii) and (iii) (d) (i) and (iii)
AMINES 469

MATCHING TYPE QUESTIONS (a) A – (q), B – (p), C – (s), D – (r)


(b) A – (s), B – (p), C – (q), D – (r)
98. Match the columns
(c) A – (q), B – (s), C – (p), D – (r)
Column-I Column-II
(d) A – (q), B – (s), C – (r), D – (p)
(A) Gabriel phthalimide (p) C6H5CH2NH2
reaction ASSERTION-REASON TYPE QUESTIONS
(B) Reduction with (q) C6H5NH2
LiAlH4 Directions : Each of these questions contain two statements,
(C) Reaction with (r) C6H5CN Assertion and Reason. Each of these questions also has four
alc. KOH + CHCl3 alternative choices, only one of which is the correct answer. You
have to select one of the codes (a), (b), (c) and (d) given below.
(D) 1° Amide with (s) CH3CH2NH2
Br2 + KOH (a) Assertion is correct, reason is correct; reason is a correct
explanation for assertion.
(a) A – (p, s); B – (p, s); C – (p, q, s); D – (p, q, s)
(b) A – (s); B – (p); C – (q); D – (p, q) (b) Assertion is correct, reason is correct; reason is not a
correct explanation for assertion
(c) A – (p, s); B – (r); C – (q); D – (s)
(d) A – (p, q); B – (p); C – (p, q); D – (s) (c) Assertion is correct, reason is incorrect
99. Match the columns (d) Assertion is incorrect, reason is correct.
Column-I Column-II 102. Assertion : Aromatic 1°amines can be prepared by Gabriel
(A) Ammonolysis (p) Amine with lesser number phthalimide synthesis.
of carbon atoms Reason : Aryl halides undergo nucleophilic substitution
(B) Gabriel phthalimide (q) Detection test for primary with anion formed by phthalimide.
synthesis amines. 103. Assertion : Only a small amount of HCl is required in the
(C) Hoffmann bromamide (r) Reaction of Phthalimide reduction of nitro compounds with iron scrap and HCl in
the presence of steam.
reaction with KOH and R–X
(D) Carbylamine reaction (s) Reaction of alkylhalides Reason : FeCl2 formed gets hydrolysed to release HCl
during the reaction.
with NH3
(a) A – (s), B – (r), C – (p), D – (q) 104. Assertion : Amines are basic in nature.
(b) A – (r), B – (p), C – (r), D – (s) Reason : Amines have lone pair of electrons on nitrogen
(c) A – (q), B – (r), C – (s), D – (p) atom.
(d) A – (s), B – (p), C – (q), D – (r) 105. Assertion : Acetanilide is less basic than aniline.
100. Match the columns Reason : Acetylation of aniline results in decrease of
Column-I Column-II electron density on nitrogen.
(A) Benzene sulphonyl (p) Zwitter ion 106. Assertion : Nitration of aniline can be conveniently done
chloride by protecting the amino group by acetylation.
(B) Sulphanilic acid (q) Hinsberg reagent Reason : Acetylation increases the electron-density in the
(C) Alkyl diazonium salts (r) Dyes benzene ring.
(D) Aryl diazonium salts (s) Conversion to alcohols 107. Assertoin : Aniline does not undergo Friedel-Crafts
(a) A – (s), B – (q), C – (r), D – (p) reaction.
(b) A – (q), B – (p), C – (s), D – (r) Reason : –NH2 group of aniline reacts with AlCl3 (Lewis
acid) to give acid-base reaction.
(c) A – (r), B – (s), C – (p), D – (q)
(d) A – (s), B – (p), C – (r), D – (q) 108. Assertion : Acylation of amines gives a monosubstituted
product whereas alkylation of amines gives
101. Match the columns
polysubstituted product.
Column-I Column-II
Reason : Acyl group sterically hinders the approach of
(A) ArN 2 Cl ArOH (p) HBF4 / NaNO 2 further acyl groups
(B) ArN 2 Cl ArNO 2 (q) H2O 109. Assertion : Nitrating mixture used for carrying out nitration
of benzene consists of conc. HNO3 + conc. H2SO4.
(C) ArN2 Cl ArH (r) HBF4
Reason : In presence of H2SO4, HNO3 acts as a base and
(D) ArN 2 Cl ArF (s) CH3CH2OH produces NO2+ ions.
EBD_7207
470 AMINES
118. What is the decreasing order of basicity of primary,
CRITICAL THINKING TYPE QUESTIONS
secondary and tertiary ethylamines and NH3 ?
110. The IUPAC name of diethyl isopropyl amine is (a) NH3 C2H5 NH 2 (C 2 H5 )2 NH (C2H5 )3N
(a) N, N-diethylpropan-2-amine
(b) N, N-diethylpropan-1-amine (b) (C2 H 5 )3 N (C2 H 5 )2 NH C2 H5 NH 2 NH 3
(c) N, N-diethylisopropylamine (c) (C2 H 5 )2 NH C2 H 5 NH 2 (C 2 H 5 )3 N NH 3
(d) N, N-diethylaminopropane (d) (C2H5)2 NH > C2H5NH2 > (C2H5)3 NH > NH3
111. IUPAC name of the following compound is 119. The correct order of the increasing basicity of methyl amine,
ammonia and aniline is
NH2 (a) methyl amine < aniline < ammonia
Cl (b) aniline < ammonia < methyl amine
(a) 2-chloro pentanamine (c) aniline < methyl amine < ammonia
(b) 4-chloro pentan-1-amine (d) ammonia < aniline < methyl amine
(c) 4-chloro pent-2-en-1-amine 120. Arrange the following amines in the decreasing order of
(d) 2-chloro pent-3-en-5-amine their basicity
112. Which of the following is the correct IUPAC name of the
compound ? NH2

N (CH 3 ) 2 (1) (2)


NH2

CH2 – CH2 – NH2


Cl
Cl (3)

(a) 1, 2-dichloro-4-(N, N-dimethyl) aniline


(b) Dimethyl – (3, 4-dichlorophenyl) amine (a) 1 > 3 > 2 (b) 3 > 2 > 1
(c) 3, 4-dichloro - N, N-dimethyl aniline (c) 1 > 2 > 3 (d) 2 > 1 > 3
(d) N, N-dimethylamino - 3, 4-dichlorobenzene
113. Acetamide is treated with the following reagents separately. 121. (CH3CO) 2 O,Pyridine
NH2
Which one of these would yield methylamine?
(a) NaOH – Br2 (b) Sodalime (I)
(c) Hot conc. H 2SO4 (d) PCl5
(i) LiAlH 4
114. Amine that cannot be prepared by Gabriel phthalimide (II) III
(ii) H2O
synthesis is
(a) aniline (b) benzylamine The basicity order of I, II and III is –
(c) methylamine (d) iso-butylamine (a) III > I > II (b) I > II > III
115. A primary amine is formed by an amide on treatment with (c) III > II > I (d) II > III > I
bromine and alkali. The primary amine has
122. Which of the statement is true regarding the basicity of the
(a) 1 carbon atom less than amide
following two primary amines ?
(b) 1 carbon atom more than amide
(c) 1 hydrogen atom less than amide CH2NH2 CH2NH2
(d) 1 hydrogen atom more than amide
116. High basicity of Me 2 NH relative to Me3N is attributed to:
(a) effect of solvent (b) inductive effect of Me I II
(c) shape of Me2NH (d) shape of Me3N
(a) Both are equally basic because both are 1º amines
117. Which one of the following is the strongest base in aqueous
(b) I > II because it is an aromatic amine
solution ?
(a) Methylamine (b) Trimethylamine (c) II > I because it is an aliphatic amine
(c) Aniline (d) Dimethylamine (d) I < II because of difference in the nature of -carbon
AMINES 471
123. The correct order of decreasing basic character is
NO2 NO2
C 6 H 5 NH 2 C 6 H 5CH 2 NH 2
, ,
I II
C6 H5 2 NH (c) (d)
C 6 H11 NH 2
, IV
III NO2
(a) II > I > III > IV (b) IV > II > I > III NO2
(c) IV > III > II > I (d) IV > II > III > I
124. Aniline when treated with conc. HNO3 gives NH2
(a) p-Phenylenediamine (b) m-Nitroaniline
(c) p-Benzoquinone (d) Nitrobenzene 128. CH3COCl
A
Br2 H 2O
B
H3O
C
125. The correct increasing order of basic strength for the
following compounds is __________.
C (major product) is –
NH2 NH2 NH2
NH2 NH2
Br
(a) (b)
NO2 CH3 Br
(I) (II) (III)
NH2
(a) II < III < I (b) III < I < II
(c) III < II < I (d) II < I < III (c) (d) None of these
126. Which of the following compounds is most basic?

Br
(a) O2N NH2
129. Towards electrophilic substitution, the most reactive will
be
(b) CH2NH2 (a) Nitrobenzene
(b) Aniline
(c) Aniline hydrochloride
(c) N – COCH3 (d) N-Acetylaniline
H 130. The most reactive amine towards dilute hydrochloric acid
is _________ .
(d) NH2 H3C
(a) CH3–NH2 (b) NH
H3C
NH2
NH2
127. (CH 3CO) 2 O
(X )
HNO 3
(Y )
H
(Z ) H3C
H 2SO 4 H 2O (c) N–CH3 (d)
H3C
Product Z of the reaction
131. Nitration of nitrobenzence is carried out than obtained
NH2 NH2 product is reduced with Fe/HCl, product so formed on
reaction with HNO2 and than with H2O, forms
(a) 1,3–dihydroxybenzene
(a) (b)
(b) 3–nitrophenol
NO2
(c) 2– nitrophenol
NO2
(d) 1,2–dihydroxybenzene
EBD_7207
472 AMINES

132. A compound of molecular formulae C3H6N shows following The structure of C would be :
characteristics
(i) Get dissolved in acidic medium. (a) – N = N – CH2– N –
(ii) Does not react with benzoylchloride CH3
(iii) Does not give carbylamine test
(iv) Does not evolute nitrogen gas on reacting with HNO2 CH3 CH3
than structure of the compound is
–N=N–
(a) trimethylamine (b) isopropylamine (b)
(c) propylamine (d) None of these
133. In a reaction of aniline a coloured product C was obtained. CH3
– NH – NH – –N
CH 3 (c) CH3
NH2 –N
NaNO2 CH3
B C CH3
HCl Cold
(d) –N=N– –N
A CH3
AMINES 473

FACT / DEFINITION TYPE QUESTIONS 14. (c) C2 H5I NH3 C2 H5 NH 2 HI


Ethyl
1. (c) RNH 2 R 2 NH R3N iodide
Primary amine Secondary amine Tertiary amine C 2 H 5 OH NH 3 C 2 H 5 NH 2 H2O
2. (c) 3. (a) 4. (d) 15. (d) 16. (d)
5. (c) 1° amines have –NH2 group in their structure. 4 primary 17. (d) For the preparation of Me3CNH2, the required alkyl
amines are possible by C4H11N. halide is Me3CX which will react with potassium
CH 3 CH 2 CH 2 CH 2 NH 2 phthalimide, a strong base, to form alkene rather than
(i) substituted product. For preparing C6H5NH2, C6H5Cl
CH 3 CH 2 CH CH 3 will be the starting halide in which Cl is non-reactive.
| 18. (c) 19. (d) 20. (c)
NH 2
21. (d) Reduction with iron scrap and hydrochloric acid is
(ii) preferred because FeCl2 formed gets hydrolysed to
CH3 CH3 release hydrochloric acid during the reaction. Thus,
| |
CH3 C CH3 only a small amount of hydrochloric acid is required
CH3 C CH 2 NH 2
| | to initiate the reaction.
H NH 2 22. (b) 23. (c) 24. (a)
(iii) (iv) 25. (b) Amines possess fishy smell.
6. (b) The compound contains longest chain of 6C atoms 26. (c) 27. (b) 28. (b)
and amino group. Hence it is an alkanamine. 29. (b) Basic nature of amines arises due to presence of lone
7. (d) 8. (d) 9. (a) pair of e–1s on the N-atom, which can be shared with
10. (c) CH3NC (methyl isocyanide) on reduction with LiAlH4 an electron deficient species.
gives secondary amine 30. (a)
O 31. (a) Secondary amine is more basic than primary amine.
||
11. (b) CH 3 CH 2 C NH 2 Br2 KOH 32. (d) Amines are basic in nature
CH3 CH 2 NH 2 33. (b) (CH3)3N < CH3NH2 < (CH3)2 NH < C2H5NH2
12. (d) R N C
4[H]
RNH CH3 34. (a)
alkyl isocyanide secondary amine 35. (a) Secondary amines are more basic than tertiary amines
O O due to stabilisation of 2° amine by hydrogen bonding
with solvent molecule.
KOH –+
13. (c) NK 36. (c) Aliphatic amines are more basic than aromatic amines.
NH
Resonance decreases the basic character due to
delocalisation of shared pair of electrons on nitrogen
O O within benzene nucleus
Phthalimide N-Potassium phthalimide 37. (b) Basic character of amines is
(a nucleophile)
2° > 1 ° > 3° > NH3
O

RX / DMF RNH 2 38. (b) CH2–NH2 compound is most basic due


NR
SN 2 rxn
to localized lone pair of electron on nitrogen atom while
O other compounds have delocalized lone pair of electron.
N-alkyl phthalimide 39 (c) 40. (a) 41. (b)
O 42. (d) Amines give alcohols only on reaction with HNO2
and not on hydrolysis.
OH
+ H2N R 43. (b) Aniline gives dyes on coupling reaction with phenols
OH Primary and p-amines.
amine 44. (c) Aryl amines do not produce phenol on treatment with
O
Phthalic acid
nitrous acid
EBD_7207
474 AMINES
45. (c) Methyl amine is a stronger base than ammonia due to 54. (d) Nitrous acid reacts differently with aliphatic and
+I effect. The alkyl groups which are electron releasing aromatic amines in cold.
groups increase the electron density around the
C2 H5 NH 2 HONO C2 H5OH N 2
nitrogen thereby increasing the availability of the lone 1 Amine Alcohol
pair of electrons to proton or lewis acid and making
the amine more basic
NH3 CH3NH2 NH 2 N N
Kb = 1.8 × 10–5 44 × 10–5
+ Cold
46. (a) Any primary amine means both aliphatic as well as HONO Cl
aromatic but monoalkylamines means only 1° aliphatic
amines. Therefore, option (a) is correct while (d) is (NaNO 2 HCl) Benzene diazonium
wrong. chloride

RNH2+ CHCl3+ 3KOH RNC 3KCl 3H 2 O 55. (c)


alkyl
isocyanide NH2 OHC
47. (c) We know that
CH 3CH 2 NH 2 CHCl 3 3KOH 56. (c) + N = CH
CH 3CH 2 NC 3KCl 3H 2O
In this reaction, bad smelling compound ethyl 57. (c) The compounds containing active H-atoms (H atoms
isocyanide (CH 3CH 2 NC) is produced. This equation attached to N, O or S) react with CH3COCl to form
acetyl derivatives.
is known as carbylamine reaction.
48. (a) Acylation occurs in one step only because lone pair 58. (d) 59. (b) 60. (b) 61. (d) 62. (d)
of nitrogen is delocalized with acyl group. 63. (a) 64. (d)
65. (a) Hinsberg’s method is based on the use of benzene
O O–
.. sulphonyl chloride.
R – NH – C – CH 3 R – NH – C – CH 3 66. (c) The inclusion of C6H5CO gp.in a molecule is called

benzoylation
49. (d) R – NH2 + CHCl3 alc. KOH
R N C 67. (d) Aniline is insoluble in water, because its –NH2 group
can’t form H- bond with water due to bulky phenyl
group.
NH2 + CHCl3
68. (c) 69. (b) 70. (d)
71. (a) Pyridine is a stronger base than the amine, pyridine
+ – removes HCl formed in acylation reaction of amines
alc. KOH N C and shifts the equilibrium to the right hand side.
72. (b) The hydrogen attached to nitrogen in sulphonamide
HNO2 is strongly acidic due to the presence of strong
50. (b) RNH2 ROH + H2O + N2
electron withdrawing sulphonyl group. Hence, it is
(A) (B) (C)
soluble in alkali.
51. (c) Since the organic amino compound on reaction with
nitrous acid at low temperature produces an oily 73. (d) In case of substituted aniline, electron releasing groups
nitrosoamine so the organic amino compound is a like –OCH3, –CH3 increase basic strength whereas
secondary aliphatic amines. electron withdrawing groups like –NO2 , –SO3H,
–COOH, –X decrease it.
273 278K
52. (a) C 2 H 5 NH 2 + HNO 2
Ethyl amine Nitrous acid .. –
NH2 N NCl
C2 H5OH + N2 + H2O
Ethanol 74. (a) NaNO2 /HCl
Note : This reaction is used as a test for aliphatic amines 0C
Diazotisation
since no other class of amines liberates N2 gas on
treatment with HNO2. Br
53. (a) 1° amines (aliphatic and aromatic) react with
CHCl3/KOH to yield isocyanide (foul smelling) This is CuBr/HBr
known as carbylamine test which is not given by 2° Sandmeyer reaction
and 3° amines.
AMINES 475
81. (c) Primary aromatic amines react with nitrous acid to
NH2 N2 Cl yield arene diazonium salts.
cold
NaNO2 ,HCl
ArNH2 + NaNO2 + 2HX
75. (b) ¾¾¾¾¾® 1° Aromatic amine
(0-5°C) Ar—N = N+X– + NaX + 2H2O
aniline diazonium Arene diazonium salt
chloride The diazonium group can be replaced by fluorine by
treating the diazonium salt with fluoroboric acid
76. (a) Azo dye is prepared by diazo coupling reaction of
(HBF4). The precipitated diazonium fluoroborate is
phenol with diazonium salt.
isolated, dried and heated until decomposition occurs
– to yield the aryl fluoride. This reaction is known as
N+2Cl OH Balz-Schiemann reaction.
HBF4 heat
Ar—N2+X– Ar—N2+BF4–
+ N=N
Ar—F + BF3 + N2
Benzene + –
p–hydroxyazobenzene NH 2 N 2 Cl
diazonium
chloride

(– N = N –) group is called azo – group. 82. (a) NaNO2 CuCN


HCl, 278K
+
NH2 N NCl Diazotization Benzene diazonium
chloride
77. (d) NaNO2 ,HCl CuCN (A)
C N
0°C
Aniline (diazotisation) (A)
benzene
diazonium
chloride Benzonitrile
(B)
Sandmeyer reaction
83. (d) 84. (c) 85. (a)
86. (a) Excess of HCl is used to convert free aniline to aniline
hydrochloride otherwise free aniline would undergo
coupling reaction with benzenediazonium chloride.
87. (b) 88. (d) 89. (b) 90. (d) 91. (b)
92. (b)

78. (b) The given reaction is known as Sandmeyer’s reaction. STATEMENT TYPE QUESTIONS

N2+Cl OH 93. (b) The fourth orbital of nitrogen in all amines contains an
unshared pair of electrons. Due to the presence of
H 2O
unshared pair of electrons, the angle C–N–E, (where E
79. (b) + N2+HCl is C or H) is less than 109.5°.
94. (b) Lower aliphatic amines are soluble in water solubility
decreases with increase in molar mass of amines.
CH3 CH3
p- cresol
Higher amines are essentially insoluble in water.
95. (b) Primary and secondary amines are engaged in
intermolecular association due to hydrogen bonding
+ – between nitrogen of one and hydrogen of another
80. (b) N NCl + OH
molecule. This intermolecular association is more in
Benzene diazonium chloride Phenol primary amines than in secondary amines as there are
two hydrogen atoms available for hydrogen bond
formation in it. Tertiary amines do not have
N= N OH intermolecular association due to the absence of
hydrogen atom available for h ydrogen bond
p-Hydroxyazobenzene formation. Therefore, the order of boiling points of
(orange dye)
isomeric amines is as follows :
Primary > Secondary > Tertiary
EBD_7207
476 AMINES
96. (b) 115. (a) The reaction is Hoffmann bromamide reaction
97. (a) The yield in Sandmayer reaction is found to be better O
than Gattermann reaction. ||
R C NH 2 Br2 4NaOH
MATCHING TYPE QUESTIONS
R NH 2 2NaBr Na 2CO3 2H 2O
98. (a) 99. (a) 100. (b) 101. (c)
O
R – NH2 contains one carbon less than ||
ASSERTION-REASON TYPE QUESTIONS R C NH 2
102. (a) 103. (d) 116. (a) Secondary amines are more basic than tertiary amines
104. (a) Amines are basic due to the presence of a lone pair of due to stabilisation of 2° amine by hydrogen bonding
electrons on nitrogen atom. The lone pair can be easily with solvent molecule.
donated. 117. (d) Aromatic amines are less basic than aliphatic amines.
Among aliphatic amines the order of basicity is
105. (d)
2° > 1° > 3°. The electron density is decreased in 3°
106. (c) Acetylation decreases the electron-density in the amine due to crowding of alkyl group over N atom
benzene ring thereby preventing oxidation. which makes the approach and bonding by a proton
107. (a) 108. (c) relatively difficult. Therefore the basicity decreases.
Further Phenyl group show – I effect, thus decreases
109. (a) HNO3 2H 2SO4 2HSO4 NO2 H3O the electron density on nitrogen atom and hence the
basicity.
CRITICAL THINKING TYPE QUESTIONS dimethylamine (2° aliphatic amine) is strongest base
among given choices.
CH3 The correct order of basic strength is
110. (a) C2H5
Dimethylamine > Methyl amine > Trimethyl amine >
HC – N
Aniline.
CH3 C2H5
118. (d) All aliphatic amines are stronger bases than NH3 and
among different ethylamines order of basictity is
N, N- diethyl propan - 2-amine
2° > 1° > 3°. Thus, the correct order is (d) i.e.,
111. (c) (C2H5)2 NH > C2H5NH2 > (C2H5)3N > NH3
112. (c) The compound is derivative of aniline. The positions This anomolous behaviour of tertiary ethyl amine is
of groups are shown by numbering the nuclear C- due to steric factors i.e., crowding of alkyl groups cover
atoms. nitrogen atom from all sides and thus makes the
approach and bonding by a lewis acid relatively
NaOH
113. (a) CH3CONH2 CH3 NH 2 difficult which results the maximum steric strain in
Br2
tertiary amines. The electrons are there but the path is
(Hofmann bromamide reaction) blocked resulting the reduction in its basicity.
114. (a) Aniline cannot be prepared by this method because 119. (b) In aniline the lone pair on N is involved in delocalization
aryl halides do not undergo nucleophilic substitution with benzene ring and is not available for protonation.
with potassium phthalimide under ordinary conditions Methyl amine is a stronger base than ammonia because
to give N-phenyl phthalimide (i.e., cleavage of C–X +I effect of methyl group increases electron density
bond in haloarenes is quite difficult). on N making it more basic than NH3.

CO alc CO NH 2 NH 2
KOH
NH N+K–
CO CO
CO
RX
N–R Cannot be protonated. least basic
CO
H+/H2O CH3 – NH 2
I Effect increases
COOH basicity.
+ RNH2
COOH 120. (a)
AMINES 477
125. (d)
121. (a) (CH3CO) 2 O,Pyridine
NH2
126. (b) CH2–NH2 compound is most basic due
(I)
(i) LiAlH 4 to localized lone pair of electron on nitrogen atom while
NH – C – CH3 other compounds have delocalized lone pair of electron.
(ii) H 2 O
(II) O NH2 NH – CO – CH3

NH – CH2 – CH3 (CH3CO)2 O


127. (b)
(III)

Product (II) is NH – C – CH3 and (III) is


NH – CO – CH3
O
(II)
HNO3

NH – CH2 – CH3 H2SO4

(III) NO2
III > I > II, As + I effect increases the basic strength +
and – R, – I effect shown by – COCH3 reduces the H3O
basic strength.
122. (d) Here again the two amines differ in the nature of
-carbon atom NH2
CH2NH2 CH2NH2
3 2
sp C > sp C

II I
123. (b) Greater the delocalisation of electron pair on N, lesser
is its availability for protonation leading to lesser basic NO2
NH2
character.
.. H
: NH2 CH2NH2 :NH2 :N CH3COCl
128. (c)

> > >

IV II I III O O
Delocalisation not Delocalisation not Delocalisation possible, more
possible, cyclohexyl gp. possible, phenyl gp. is
in III than in I. Further C6H 5– gp. is NH – C – CH3
is electron-releasing electron-withdrawing NH – C – CH3
electron-withdrawing
124. (b) Although – NH2 group is o,p - directing but in
presence of conc. HNO3 it undergoes protonation to Br2 H2O

form – N H3 which, being electron - deficient, becomes


m- directing. Br
+
: NH2 NH3
(A) (B)
conc. HNO3 conc. HNO3
NH2
– NH2 group is – NH+ group is
3
o, p - directing m - directing H 3O
+
N H3

Br
(C)
NO2
Protonated
m - nitroaniline
EBD_7207
478 AMINES
.. 131. (b)
NO2 NH2
129. (b) NO2 NO2 NO2

HNO3 Fe/HCl
H2SO4
NO2 NH2
nitrobenzene Aniline

O HNO2
+ – ..
NH3Cl H–N–C–CH3
NO2 NO2

–N2

H2 O + –
Aniline hydrochloride N- Acetylaniline OH N2Cl
Nitrobenzene and aniline hydrochloride have electron- 132. (a) It is a tertiary amine hence shows above observations.
133. (d) The reaction can be completed as follows:
withdrawing (–NO2 and – N H3 ) groups, hence these
NH2
will undergo electrophilic substitution with difficulty. NaNO2 /HCl CH3
Aniline and N– acetylaniline (acetanilide) have N2Cl + N
(diazotisation) CH3
electron– releasing groups, however –NHCOCH3 is ‘A’ ‘B’ N, N-dimethylaniline
less electron- releasing than – NH 2 due to (Aniline) Benzene
delocalisation of lone pair of electron on N toward diazonium chloride
cold
carbonyl group. Hence aniline (having – NH2) will
undergo electrophilic substitution most easily. CH3
N =N N
130. (b) CH3
‘C’
p-dimethylaminoazobenzene
28
BIOMOLECULES

FACT / DEFINITION TYPE QUESTIONS 11. Reducing sugars reduce.


(a) only Fehling’s solution
1. Biomolecules are (b) only Tollen’s solution.
(a) aldehydes and ketones (c) both (a) & (b)
(b) acids and esters (d) neither (a) nor (b)
(c) carbohydrates, proteins and fats 12. Which among the following is the simplest sugar?
(d) alcohols and phenols (a) Glucose (b) Starch
2. Which of the following is a disaccharide ? (c) Cellulose (d) None of these
(a) Lactose (b) Starch 13. Glucose can’t be classified as
(c) Cellulose (d) Fructose
(a) hexose (b) carbohydrate
3. The sugar that is characteristic of milk is
(c) aldose (d) oligosaccharide
(a) maltose (b) ribose
14. Which of the following properties of glucose cannot be
(c) lactose (d) galactose explained by its open chain structure?
4. Which one is a disaccharide ?
(i) Glucose does not form hydrogen sulphite with NaHSO3
(a) Glucose (b) Fructose
(ii) On oxidation with HNO3 glucose gives saccharic acid.
(c) Xylose (d) Sucrose
(iii) Glucose is found to exist in two different crystalline
5. Which of the following monosaccharide is pentose ? forms which are named as and .
(a) Glucose (b) Fructose (a) (ii) only (b) (i) and (iii)
(c) Arabinose (d) Galactose
(c) (ii) and (iii) (d) (i) and (ii)
6. The commonest disaccharide has the molecular formula
15. Which of the following gives positive Fehling solution test?
(a) C10 H18 O9 (b) C10 H20 O10
(a) Protein (b) Sucrose
(c) C18 H22 O11 (d) C12 H22 O11
(c) Glucose (d) Fats
7. Monosaccharides usually contains ... carbon atoms.
16. Which of the following statements is incorrect regarding
(a) C3 to C10 (b) C1 to C6
glucose?
(c) C4 to C10 (d) C5 to C8
(a) It is an aldohexose.
8. Which one of th e following compounds is found
(b) It is also known as dextrose
abudnantly in nature?
(c) It is monomer of cellulose.
(a) Fructose (b) Starch
(d) It is the least abundant organic compound on earth.
(c) Glucose (d) Cellulose
17. Glucose gives silver mirror test with Tollen’s reagent. It
9. A carbohydrate that cannot be hydrolysed into simpler
units is called shows the presence of
(a) polysaccharides (b) trisaccharides (a) acidic group (b) alcoholic group
(c) disachharides (d) monosaccharides (c) ketonic group (d) aldehyde group
10. Which of the following statements is incorrect ? 18. The symbols D and L represents
(a) Maltose gives two molecules of glucose only. (a) the optical activity of compounds.
(b) Cellulose and sucrose are polysaccharide. (b) the relative configuration of a particular stereoisomer.
(c) Polysaccharides are not sweet in taste. (c) the dextrorotatory nature of molecule.
(d) Polysaccharides are also known as non-sugars. (d) the levorotatory nature of molecule
EBD_7207
480 BIOMOLECULES
19. Glucose is found to exist in two different and crystalline 31. A solution of D-glucose in water rotates the plane polarised
forms. These forms can be obtained by. light
(i) The form of glucose is obtained by crystallisation (a) to the right (b) to the left
from concentrated solution of glucose at 303 K. (c) to either side (d) None of these
(ii) The form of glucose is obtained by crystallisation 32. The number of chiral carbon atoms present in cyclic
from concentrated solution of glucose at 303 K. structure -D(+) glucose
(iii) The form is obtained by crystallisation from hot and (a) 3 (b) 4
saturated aqueous solution at 371 K. (c) 5 (d) 6
(iv) The form is obtained by crystallisation from hot and 33. The -D glucose and -D glucose differ from each other
saturated aqueous solution at 371 K. due to difference in carbon atom with respect to its
(a) (i) and (iii) (b) (ii) and (iv) (a) conformation (b) configuration
(c) (ii) and (iii) (d) (i) only (c) number of OH groups (d) size of hemiacetal ring
20. The function of glucose is to 34. The two forms of D glucopyranose obtained from the
(a) provides energy (b) promote growth solution of D glucose are called
(c) prevent diseases (d) perform all above (a) isomers (b) anomers
(c) epimers (d) enantiomers
21. Which one of the following compounds is different from
35. Which of the following carbohydrates are branched polymer
the rest?
of glucose?
(a) Sucrose (b) Maltose
(i) Amylose (ii) Amylopectin
(c) Lactose (d) Glucose
(iii) Cellulose (iv) Glycogen
22. The two functional groups present in a typical carbohydrate (a) (i) and (ii) (b) (ii) and (iv)
are: (c) (iii) and (iv) (d) (i), (ii) and (iii)
(a) – CHO and – COOH (b) > C = O and – OH 36. The number of chiral carbon atoms present in cyclic
(c) – OH and – CHO (d) – OH and – COOH structure -D(+) glucose
23. When glucose reacts with bromine water, the main product (a) 3 (b) 4
is (c) 6 (d) 5
(a) gluconic acid (b) glyceraldehyde 37. Which of the following reagent cannot distinguish between
(c) saccharic acid (d) acetic acid glucose and fructose?
24. Glucose does not react with (a) Fehling’s solution (b) Tollen’s reagent
(a) Br2/H2O (b) H2NOH (c) Benedict’s solution (d) All of these
(c) HI (d) NaHSO3 38. Maltose and glucose are
25. Glucose reacts with acetic anhydride to form (a) oxidising sugar
(a) monoacetate (b) tetra-acetate (b) reducing sugar
(c) penta-acetate (d) hexa-acetate (c) first is oxidising and second is reducing sugar
26. Reduction of glucose by HI suggest that (d) both are non-reducing sugar
(a) presence of OH groups 39. Choose the correct relationship for glucose and fructose
(b) presence of –CHO group (a) these are functional isomers
(c) cyclic structure of glucose (b) these are chain isomers
(d) six carbon atoms are arranged in straight chain (c) these are position isomers
27. The reaction of glucose with red P + HI is called (d) All of these
(a) Sandmeyer’s reaction (b) Reformatsky reaction 40. The pair of compounds in which both the compounds give
positive test with Tollen’s reagent is
(c) Gattermann’s reaction (d) Reduction
(a) Glucose and Sucrose
28. Which of the following reactions of glucose can be explained
(b) Fructose and Sucrose
only by its cyclic structure?
(c) Acetophenone and Hexanal
(a) Glucose forms pentaacetate
(d) Glucose and Fructose
(b) Glucose reacts with hydroxylamine to form an oxime 41. The letter D and L in carbohydrates represent
(c) Pentaacetate of glucose does not react with (a) its optical rotation (b) its mutarotation
hydroxylamine (c) its direct synthesis (d) its configuration
(d) Glucose is oxidised by nitric acid to gluconic acid 42. Which of the following statement is correct about fructose?
29. Which is the least stable form of glucose ? (a) It is dextrorotatory compound
(a) -D-Glucose (b) -D-Glucose (b) It exists in the two cyclic forms which is obtained by
(c) Open chain structure (d) All are equally stable the addition of OH at C-5 to the >C=O group
30. Isomerization of glucose produces (c) It exists as six membered ring
(a) galactose (b) fructose (d) It is named as furanose as it contain one oxygen and
(c) mannose (d) allose six carbon atom
BIOMOLECULES 481
43. Fructose is (i) Ribose (ii) Glucose
(a) a hemiacetal (b) an acetal (iii) Fructose (iv) Galactose
(c) a hemiketal (d) a ketal (a) (i) and (ii) (b) (i) and (iii)
44. The sugar present in fruits is (c) (iii) and (iv) (d) (ii) and (iii)
(a) fructose (b) glucose 56. Invert sugar is
(c) sucrose (d) galactose (a) chemically inactive form of sugar
45. Three cyclic structures of monosaccharides are given below (b) equimolecular mixture of glucose and fructose
which of these are anomers (c) mixture of glucose and sucrose
(d) a variety of cane sugar
HO H 57. Which one of the followin g does not exhibit the
H OH HO H
HO H phenomenon of mutarotation ?
H OH H OH
HO H
O (a) (+) – Sucrose (b) (+) – Lactose
HO H O HO H O
H OH
(c) (+) – Maltose (d) (–) – Fructose
H OH H OH 58. Glycogen is a branched chain polymer of -D-glucose units
HO H
H H in which chain is formed by C1–C4 glycosidic linkage
H
CH2OH CH2OH CH2OH whereas branching occurs by the formation of C1-C6
(I) (II) (III) glycosidic linkage. Structure of glycogen is similar to
______.
(a) I and II (b) II and III (a) Amylose (b) Amylopectin
(c) I and III (d) III is anomer of I and II (c) Cellulose (d) Glucose
46. The sugar present in honey is 59. Which of the following correctly represents the cyclic
(a) sucrose (b) glucose structure of –D–(–)– fructo furanose.
(c) fructose (d) maltose
47. Which of the following is the sweetest sugar? 12
(a) Sucrose (b) Glucose (a) HOH2C–C–OH
(c) Fructose (d) Maltose 3
HO——H O
48. Cellulose is a polymer of H 4
——OH
(a) Glucose (b) Fructose
5
(c) Ribose (d) Sucrose H 6
49. Sucrose on hydrolysis gives CH2OH
(a) fructose+ribose (b) glucose + fructose
(c) glucose+glucose (d) fructose + fructose
2 1
50. The presence or absence of hydroxyl group on which carbon (b) HO–C–CH2OH
atom of sugar differentiates RNA and DNA? 3
(a) 1st (b) 2nd HO——H O
rd 4
(c) 3 (d) 4 th H——OH
51. Carbohydrates are stored in the body as 5
H
(a) sugars (b) starch 6
CH2OH
(c) glucose (d) glycogen
52. A carbohydrate insoluble in water is
(a) glucose (b) fructose 6 O 1
(c) cellulose (d) sucrose (c) HOH2C CH2OH
53. Which of the following carbohydrate does not correspond 5 2
to the general formula Cx(H2O)y ? H OH
H OH
(a) Glucose (b) 2-Deoxyribose 4 3
(c) Fructose (d) Arabinose
OH H
54. Lactose is made of
(a) -D-glucose only
6 O
(b) -D-glucose and -D-glucose
(d) HOH2C OH
(c) -D-galactose and -D-glucose
5 2
(d) -D-galactose and -D-glucose H CH2OH
55. Which of the following monosaccharides are present as H OH 1
five membered cyclic structure (furanose structure)? 4 3
OH H
EBD_7207
482 BIOMOLECULES
60. Sucrose which is dextrorotatory in nature after hydrolysis 72. Which of the following is not an optically active amino
gives glucose and fructose, among which acid?
(i) Glucose is laevorotatory and fructose is dextrorotatory. (a) Valine (b) Glycine
(ii) Glucose is dextrorotatory and fructose is laevorotatory (c) Leucine (d) Arginine
(iii) The mixture is laevorotatory. 73. In aqueous solution, an amino acid exists as
(iv) Both are dextrorotatory. (a) cation (b) anion
(a) (i) and (iii) (b) (ii) and (iii) (c) dianion (d) zwitter ion
(b) (iii) and (iv) (d) (iii) only 74. Which one of the following statements is correct?
61. Chemically amylose is a _________ with 200–1000 (a) All amino acids except lysine are optically active
-D-(+)-glucose units held by ______ glycosidic linkage (b) All amino acids are optically active
(a) long unbranched chain, C1– C6. (c) All amino acids except glycine are optically active
(b) branched chain, C1 – C4. (d) All amino acids except glutamic acids are optically
(c) long unbranched chain, C1– C4. active
(d) branched chain, C1– C6. 75. Amino acids generally exist in the form of Zwitter ions. This
62. Amylopectin is a ________ polymer of -D-glucose units means they contain
in which chain is formed by _______ glycosidic linkage (a) basic—NH2 group and acidic —COOH group
whereas branching occurs by ________ glycosidic linkage.
(b) the basic— NH3 group and acidic —COO– group
(a) branched chain, C1– C6, C1– C4.
(b) branched chain, C1– C4, C1– C6. (c) basic—NH2 and acidic —H+ group
(c) unbranched chain, C1– C4, C1– C6. (d) basic–COO– group and acidic — N H 3 group
(d) unbranched chain, C1– C6, C1– C4. 76. Which of the following molecules is capable of forming
63. Which of the following is incorrect about cellulose? Zwitter ion?
(a) It is a major constituent of cell wall of plant cells. (a) NH2CH2COOH (b) CH3CH2NH2
(b) It is a branched chain disaccharide (c) CH3CH2COOH (d) All of these
(c) It is composed of only -D-glucose units. 77. The structural feature which distinguishes proline from
(d) The glycosidic linkage between two units is found natural -amino acids?
between C1 of one unit and C4 of next unit. (a) Proline is optically inactive
64. Which of the following is also known as animal starch? (b) Proline contains aromatic group
(a) Glycine (b) Glycogen (c) Proline is a dicarboxylic acid
(c) Amylose (d) Cellulose (d) Proline is a secondary amine
65. Select the uses of carbohydrates. 78. The linkage present in proteins and peptides is
(a) Honey is used as instant source of energy by vaids in
ayurvedic system of medicine O O
|| ||
(b) These are used as storage molecules (a) C NH (b) C O
(c) They are used in furniture, cotton fibre, lacquers
(d) All of the above O O
|| ||
66. The number of essential amino acids in man is (c) (d) – NH –
C O C
(a) 8 (b) 10
(c) 18 (d) 20 79. Which one of the following structures represents the
67. An acidic amino acid among the following is peptide chain?
(a) glycine (b) valine H O
(c) proline (d) leucine | | ||
(a) N C N C NH C NH
68. Amino acids are the building blocks of || | |
(a) fats (b) proteins O H
(c) vitamins (d) carbohydrates H H
69. Which one of the amino acids can be synthesised in the | | | | | | | |
body? (b) N C C C C N C C C
(a) Alanine (b) Lysine || | | | | | |
(c) Valine (d) Histidine O
70. One of essential -amino acid is H H H O
(a) lysin (b) serine | | | | | | ||
(c) glycine (d) proline (c) N C C N C C N C C
| || | || |
71. Two functional group that are present in all amino acids are O O
the H O H
(a) hydroxy, amine (b) hydroxy, amide | | | || | | | | |
(c) carboxyl, amino (d) carboxyl, amide (d) N C C C N C C N C C C
| | | | | || | |
H O
BIOMOLECULES 483
80. Simplest proteins has one peptide linkage. It is 93. Primary structure of a protein is
(a) tripeptide (b) dipeptide (a) sequence in which -amino acid are linked to one
(c) tetrapeptide (d) oligopeptide another
81. A nanopeptide contain how many peptide bond (b) sequence in which amino acids of one polypeptide
(a) 7 (b) 9 chain are joined to other chain
(c) 8 (d) 10 (c) the folding patterns of polypeptide chains
82. Proteins are polypeptide of (d) the pattern in which the polypeptide chain are arranged
(a) -amino acid (b) -hydroxy acid 94. The protein that transport oxygen in the blood stream is
(c) D- -amino acid (d) L- -amino acid (a) haemoglobin (b) insulin
83. Globular proteins are present in (c) collagen (d) albumin
(a) blood (b) eggs 95. The helical structure of protein is stabilized by
(c) milk (d) all of these (a) dipeptide bonds (b) hydrogen bonds
84. In fibrous proteins, polypeptide chains are held together (c) ether bonds (d) peptide bonds
by 96. Which of the following statements is incorrect?
(a) van der waals forces (a) In -helix structure a polypeptide chain forms all
(b) electrostatic forces of attraction possible hydrogen bonds by twisting into a right
(c) hydrogen bonds handed screw.
(d) covalent bonds (b) In -structure of proteins all peptide chains are
85. Which of the following is not a function of proteins? stretched out to nearly maximum extension.
(a) Formation of hair, wool, skin and nails (c) During denaturation 1° and 2° structures are destroyed
(b) As a biological catalysts in the form of enzymes. but 3° structure remains intact.
(c) As food in the form of meat, eggs (d) All the above statements are incorrect.
(d) As energy provider for metabolism 97. Which of the following indicates the order in which amino
86. Which of the following is not a fibrous protein? acids are linked together in a protein ?
(a) Keratin (b) Myosin (a) Primary structure (b) Secondary structure
(c) Insulin (d) Both (a) and (b) (c) Tertiary structure (d) Quaternary structure
87. A polypeptide with more than hundred amino acid residues,
98. Which of the following statement is not true about
having molecular mass higher than 10,000 u is called_____.
secondary structure of protein ?
(a) nucleic acid (b) hormone
(a) The alpha helix, beta pleated sheet and beta turns are
(c) protein (d) enzyme examples of secondary structure of protein.
88. An insulin is a ______ which contains _____ amino acids.
(b) The ability of peptide bonds to form intramolecular
(a) protein, 74 (b) protein, 51 hydrogen bonds is important to secondary structure.
(c) hormone, 51 (d) hormone, 74
(c) The steric influence of amino acid residues is important
89. Which of the following is an example of globular proteins? to secondary structure.
(a) Glycine (b) Albumin
( d) The hydrophilic/ hydrophobic character of amino acid
(c) Alanine (d) Both (a) and (b) residues is important to secondary structure.
90. Which of the following is not a characteristics of fibrous
99. Which of the following terms indicates to the arrangement
proteins?
of different protein subunits in a multiprotein complex ?
(a) In the fibrous proteins polypeptide chains are held
(a) primary structure (b) secondary structure
together by hydrogen and disulphide bonds.
(c) tertiary structure (d) quaternary structure
(b) These have fibre like structure.
100. Secondary structure of protein is mainly governed by
(c) These are generally soluble in water.
(a) hydrogen bonds (b) covalent bonds
(d) These have elongated shape.
91. Which of the following statements is true about a peptide (c) ionic bonds (d) disulphide bonds
bond (RCONHR)? 101. The secondary structure of a protein refers to
(a) It is non planar. (a) fixed configuration of the polypeptide backbone
(b) It is capable of forming a hydrogen bond. (b) helical backbone
(c) The cis configuration is favoured over the trans (c) hydrophobic interactions
configuration. (d) sequence of amino acids
(d) Single bond rotation is permitted between nitrogen 102. Tertiary structure of protein arises due to
and the carbonyl group. (a) folding of polypeptide chain
92. Proteins are condensation polymers of (b) folding, coiling and bonding of polypeptide chain
(a) -amino acids (b) -amino acids (c) linear sequence of amino acid in polypeptide chain
(c) -hydroxy acids (d) -hydroxy acids (d) denatured proteins
EBD_7207
484 BIOMOLECULES
103. Denaturation of proteins leads to loss of its biological 114. Insulin production and its action in human body are
activity by responsible for the level of diabetes. This compound belongs
(a) Formation of amino acids to which of the following categories?
(b) Loss of primary structure (a) An enzyme (b) A hormone
(c) Loss of both primary and secondary structures (c) A co-enzyme (d) An antibiotic
(d) Loss of both secondary and tertiary structures 115. Enzymes are essential as biocatalysts. They function in
104. Coagulation of protein is known as (a) aqueous medium, temp = 30–35ºC; pH=7
(a) dehydration (b) decay (b) organic medium
(c) deamination (d) denaturing (c) aqueous medium under extreme pH conditions
105. Which of the following terms refers to the overall three (d) None of these
dimensional shape of a protein. 116. Which of the following statements is incorrect?
(a) Primary structure (b) Secondary structure (a) Enzymes are organic catalysts
(c) Tertiary structure (d) Quaternary structure
(b) Enzymes have a very large turnover number
106. Which of the following indicates to ‘regions of ordered
(c) Enzymes action is specific
structure within a protein’.
(d) Enzymes always require a coenzyme in their catalytic
(a) Primary structure (b) Secondary structure
action.
(c) Tertiary structure (d) Quaternary structure
107. The strongest form of intermolecular bonding that could 117. Among the following vitamins the one whose deficiency
be formed involving the residue of the amino acid serine is. causes rickets (bone deficiency) is :
(a) ionic bond (a) Vitamin A (b) Vitamin B
(b) hydrogen bond (c) Vitamin D (d) Vitamin C
(c) van der Waals interactions 118. The vitamin that is not soluble in water is
(d) None of the above (a) Vitamin B1 (b) Vitamin B2
108. Which of the following protein destroy the antigen when it (c) Vitamin B6 (d) Vitamin D
enters in body cell? 119. Deficiency of vitamin B1 causes the disease
(a) Antibodies (b) Insulin (a) Convulsions (b) Beri-Beri
(c) Chromoprotein (d) Phosphoprotein (c) Cheilosis (d) Sterility
109. Which of the following is incorrect regarding enzymes? 120. Anaemia is caused by the deficiency of vitamin
(a) Most of them are globular proteins. (a) B6 (b) B1
(b) They are very specific for a particular reaction but not (c) B2 (d) B12
for a particular substrate. 121. Vegetable oils like wheat gram oil, sunflower oil etc. are the
(c) They are generally named after the compound or class good source of
of compounds upon which they work. (a) vitamin K (b) vitamin E
(d) All the above statements are incorrect. (c) vitamin D (d) vitamin A
110. Enzymes take part in a reaction and 122. Which is a fat soluble vitamin?
(a) decrease the rate of a chemical reaction (a) Vitamin A (b) Vitamin B6
(b) increase the rate of a chemical reaction (c) Vitamin C (d) Vitamin B2
(c) both (a) and (b) 123. Vitamin B2, a water soluble vitamin is also known as
(a) ascorbic acid (b) riboflavin
(d) None of these
(c) thiamine (d) pyridoxine
111. Enzymes are made up of
124. Which of the following statements about vitamin B12 is
(a) Edible proteins
(b) Proteins with specific structure incorrect ?
(c) Nitrogen containing carbohydrates (a) It has a cobalt atom
(d) Carbohydrates (b) It also occurs in plants
112. The enzyme which hydrolyses triglycerides to fatty acids
(c) It is also present in rain water
and glycerol is called
(a) Maltase (b) Lipase (d) It is needed for human body in very small amounts
(c) Zymase (d) Pepsin 125. The couplings between base units of DNA is through :
113. Which one of the following, statements is incorrect about (a) Hydrogen bonding (b) Electrostatic bonding
enzyme catalysis? (c) Covalent bonding (d) van der Waals forces
(a) Enzymes are mostly proteinous in nature.
126. Which of the following is correct about H-bonding in
(b) Enzyme action is specific.
nucleotide?
(c) Enzymes are denaturated by ultraviolet rays and at
(a) A --- A and T --- T (b) G --- T and A --- C
high temperature.
(c) A --- G and T --- C (d) A --- T and G --- C
(d) Enzymes are least reactive at optimum temperature.
BIOMOLECULES 485
127. In DNA, the complimentary bases are: 140. Which of the following structures represents thymine ?
(a) Adenine and thymine; guanine and cytosine O NH2
(b) Adenine and thymine ; guanine and uracil
(c) Adenine and guanine; thymine and cytosine HN N
(d) Uracil and adenine; cytosine and guanine (a) (b)
O N O N
128. The segment of DNA which acts as the instrumental manual H H
for the synthesis of the protein is:
(a) ribose (b) gene NH2 OH
(c) nucleoside (d) nucleotide N N CH3
(c) (d)
129. In DNA the linkages between different nitrogenous bases
are : HO N HO N
(a) peptide linkage (b) phosphate linkage 141.When adenine is attached to ribose sugar, it is called
(c) H-bonding (d) glycosidic linkage adenosine. To make a nucleotide from it, it would require
130. DNA multiplication is called as (a) oxygenation (b) addition of a base
(a) translation (b) transduction (c) addition of phosphate (d) hydrogenation
(c) transcription (d) replication 142.Which of the following is not present in a nucleotide?
131. Chromosomes are made from (a) Guanine (b) Cytosine
(a) proteins (c) Adenine (d) Tyrosine
(b) nucleic acids 143.The function of DNA in an organism is
(c) proteins and nucleic acids (a) to assist in the synthesis of RNA molecule
(d) carbohydrates and nucleic acids (b) to store information of heredity characteristics
132. The double helical structure of DNA was proposed by (c) to assist in the synthesis of proteins and polypeptides
(a) Watson and Crick (b) Meichers (d) All of these
(c) Emil Fischer (d) Khorana 144. Which of the following statements regarding DNA
133. - Helix is found in fingerprinting is incorrect?
(a) DNA (b) RNA (a) It is used in forensic laboratories for identification of
(c) lipid (d) carbohydrates criminals.
134. Which of the following compounds is responsible for the (b) It cannot be altered by surgery.
transmission of heredity characters? (c) It is different for every cell and cannot be altered by
(a) RNA (b) DNA any known treatment.
(c) Glucose (d) Haemoglobin (d) It is used to determine paternity of an individual.
135. The latest discovery in cytology is that of
(a) respiration (b) genetic code STATEMENT TYPE QUESTIONS
(c) enzyme (d) None of these
145. Read the following statements and choose the correct
136. Energy is stored in our body in the form of answer?
(a) ATP (b) ADP (i) All monosaccharides are reducing sugars.
(c) fats (d) carbohydrates (ii) All monosaccharides are not reducing sugars.
137. The chemical change in DNA molecule that could lead to (iii) In disaccharides if aldehydic or ketonic groups are
synthesis of protein with an altered amino acid sequence bonded, these are non– reducing sugars.
is called (iv) In disaccharides if aldehydic or ketonic groups are
(a) replication (b) lipid formation free, these are reducing sugars.
(c) cellular membrane (d) mutation (a) (i), (iii) and (iv) (b) (ii), (iii) and (iv)
138. DNA has deoxyribose, a base and the third component which (c) (i) and (iv) (d) (ii) and (iv)
is 146. Which of the following statement(s) is/are correct?
(a) phosphoric acid (b) ribose (i) Glucose is reducing sugar
(c) adenine (d) thymine (ii) Sucrose is reducing sugar
139. The process by which synthesis of protein takes place based (iii) Maltose is non reducing sugar
on the genetic information present in m-RNA is called (iv) Lactose is reducing sugar
(a) Translation (b) Transcription (a) (i) and (ii) only (b) (i) and (iii) only
(c) Replication (d) Messenger hypothesis (c) (i) and (iv) only (d) All of these
EBD_7207
486 BIOMOLECULES
147. Which of the following statements regarding carbohydrates (iv) Fructose contains a ketonic functional group at C–2
are correct? carbon atom.
(i) Lactose is the carbohydrate found in milk. Which of the following is the correct code for the statements
(ii) More than 25 monosaccharides occur naturally. above?
(iii) Sucrose on hydrolysis gives one molecule each of (a) FTTT (b) FFTT
glucose and fructose. (c) TFFT (d) FTFT
(iv) Maltose is a non-reducing sugar whereas sucrose is a 151. Read the following statements and choose the correct–
reducing disaccharide sugar. option?
(a) (i), (ii) and (iii) (b) (i) and (iii) (i) Starch is a polymer of – glucose.
(c) (ii), (iii) and (iv) (d) (iii) and (iv) (ii) Starch consists of amylose and amylopectin.
148. Read the following statements. (iii) Amylose is insoluble in water.
(i) Pyran is a cyclic organic compound with one oxygen (iv) Amylopectin is soluble in water.
atom and five carbon atom. (a) (i) (iii) and (iv) (b) (i), (ii) and (iii)
(c) (i) and (ii) (d) (iii) and (iv)
(ii) The cyclic structure of glucose is correctly represented
152. Which among the following statements are true for glycine?
by Haworth strucure.
(i) It exists in crystalline form
(iii) Five membered cyclic structure of glucose is called
(ii) It is optically active
pyranose structure. (iii) It is soluble in water
Which of the following statement(s) is/are true? (iv) It can form Zwitter ions
(a) (i) and (iii) (b) (i) and (ii) (a) (i), (ii) and (iii) (b) (i), (ii) and (iv)
(c) Only (iii) (d) (i), (ii) and (iii) (c) (i), (iii) and (iv) (d) (ii), (iii) and (iv)
149. Consider the following statements. 153. Which of the following statements are correct?
(i) Linkage between two monosaccharide units through (i) Proteins on hydrolysis gives only -amino acids.
oxygen atom is called glycosidic linkage. (ii) Gln stands for glutamic acid.
(ii) Sucrose on hydrolysis gives an equimolar mixture of (iii) Amino acids with equal number of amino and carboxyl
fructose and glucose which is dextrorotatory. groups are neutral.
(iii) Lactose consists of linkage between C1 of galactose (iv) All naturally occuring -amino acids are optically
and C4 of glucose. active.
(iv) Out of two components of starch the component (a) (i) and (iii) (b) (i), (ii) and (iv)
present in greater proportion is insoluble in water. (c) (iii) and (iv) (d) (ii), (iii) and (iv)
(v) Glycogen is also known as animal starch because it is 154. Which of the statements about "Denaturation" given below
structurally similar to amylose a component of starch. are correct ?
Which of the following is the correct code for statements (i) Denaturation of proteins causes loss of secondary and
above? tertiary structures of the protein.
(a) FFFTT (b) FTTTF (ii) Denaturation leads to the conversion of double strand
(c) TFTFT (d) TFTTF of DNA into single strand
150. Read the following statements. (iii) Denaturation affects primary strucrture which gets
distorted
(i) Haworth structure of –D– glucose will be .
(a) (ii) and (iii) (b) (i) and (iii)
6 (c) (i) and (ii) (d) (i), (ii) and (iii)
CH2OH
O 155. Of the following statements about enzymes which ones
5 are true?
H H OH (i) Enzymes lack in nucleophilic groups
4 1 (ii) Enzymes are highly specific both in binding chiral
HO OH H H substrates and in catalysing their reactions
3 2 (iii) Enzymes catalyse chemical reactions by lowering
OH the energy of activation
H
(iv) Pepsin is a proteolytic enzyme
(ii) Fructose belongs to D–series and is a laevorotatory (a) (i) and (iv) (b) (i) and (iii)
compound. (c) (ii), (iii) and (iv) (d) (i)
(iii) Haworth structure for –D–fructose will be. 156. Which of the following statements are correct ?
6 O (i) Vitamins A, D, E and K are insoluble in water.
HOH2C (ii) Vitamins A, D, E and K are stored in liver and adipose
OH
5 2 tissues.
H CH2OH (iii) Vitamin B and vitamin C are water soluble.
H OH 1 (iv) Water soluble vitamins should not be supplied
4 3 regularly in diet.
OH H (a) (i), (ii) and (iv) (b) (i), (ii) and (iii)
(c) (i) and (iv) (d) (ii) and (iv)
BIOMOLECULES 487
157. Which of the following statement(s) is/are correct? (a) A – (p), B – (r), C – (s), D – (q)
(i) Information regarding the sequence of nucleotides in (b) A – (r), B – (s), C – (p), D – (q)
the chain of a nucleic acid is called its primary structure.
(c) A – (r), B – (p), C – (s), D – (q)
(ii) In secondary structure of DNA adenine forms
hydrogen bonds with guanine whereas cytosine forms (d) A – (r), B – (p), C – (q), D – (s)
hydrogen bonds with thymine. 160. Match the columns
(iii) RNA molecules are of three types m-RNA, r-RNA and
Column - I Column - II
t-RNA and they all perform different functions.
(a) (ii) only (b) (i) and (iii) (Enzymes) (Reactions)
(c) (ii) and (iii) (d) (iii) only (A) Invertase (p) Decomposition of urea into
158. Consider the following statements. NH3 and CO2
(i) Nucleic acids are long chain polymers of nucleotides.
(B) Maltase (q) Conversion of glucose into
(ii) Sugar moiety in DNA molecules is -D-ribose whereas
in RNA molecules it is -D-2-deoxyribose. ethyl alcohol
(iii) RNA contains four bases viz. adenine (A), guanine (C) Pepsin (r) Hydrolysis of maltose into
(G), cytosine (C) and uracil (U)
glucose
(iv) Nucleotide is a nucleoside linked to phosphoric acid
at 4 – position of sugar moiety. (D) Urease (s) Hydrolysis of cane sugar
Which of the following is the correct code for the statements (E) Zymase (t) Hydrolysis of proteins into
above? peptides
(a) TFFT (b) TFTF
(a) A – (s), B – (r), C – (t), D – (p), E – (q)
(c) FFTT (d) FTFF
(b) A – (r), B – (q), C – (s), D – (p), E – (t)
MATCHING TYPE QUESTIONS (c) A – (q), B – (p), C – (r), D – (s), E – (t)
159. Match the columns. (d) A – (s), B – (p), C – (t), D – (q), E – (r)
Column - I Column - II 161. Match the columns
Reaction of glucose Characteristic of
glucose molecule Column - I Column - II
CHO (A) Vitamin B6 (p) Fat soluble
HI, (B) Vitamin K (q) Xerophthalmia
(A) (CHOH)4 CH3 (CH2) 4 CH3 (p) Presence of
(C) Vitamin D (r) Convulsions
CH2OH
(D) Vitamin A (s) Delayed blood clotting
C = O group
(a) A – (p,q), B – (p,s), C – (p), D – (r)
CN
H–C (b) A – (r), B – (p,s), C – (p), D – (p, q)
(B) CHO OH (q) Presence of
(i) NH2OH (CHOH)4
(c) A – (p,s), B – (r), C – (p), D – (p,q)
(CHOH)4 aldehydic group
(ii) HCN
CH2OH
(d) A – (r), B – (p,s), C – (p,q), D – (p)
CH2OH 162. Match the columns
Column - I Column - II
CHO O
(C) CHO (r) All six carbon (A) Vitamin A (p) Scurvy
(CH–O–C–CH3)4
(CH3COO)2O (B) Vitamin B12 (q) Hemorrhagic condition
(CHOH)4 atoms are linked
CH2–O–C–CH3 in a straight (C) Vitamin C (r) Sterility
CH2OH chain.
O
(D) Vitamin E (s) Xerophthalmia
(E) Vitamin K (t) Pernicious anaemia
(a) A – (t), B – (s), C – (p), D – (r), E – (r)
(D) CHO COOH (s) Presence of five
Br2water
(b) A – (s), B – (t), C – (p), D – (q), E – (r)
(CHOH)4 (CHOH)4 —OH groups
(c) A – (s), B – (t), C – (p), D – (r), E – (q)
CH2OH (d) A – (t), B – (s), C – (p), D – (r), E – (q)
CH2OH
EBD_7207
488 BIOMOLECULES
171. Select the false statement about the cyclic glucose.
ASSERTION-REASON TYPE QUESTIONS
(a) If the OH group is added to CHO group it will form
Directions : Each of these questions contain two statements, cyclic hemiacetal structure
Assertion and Reason. Each of these questions also has four (b) Glucose form six-membered ring in which –OH is at
alternative choices, only one of which is the correct answer. You C–5 position
have to select one of the codes (a), (b), (c) and (d) given below. (c) Melting point of -glucose is 423 K and of -glucose
(a) Assertion is correct, reason is correct; reason is a correct is 419 K
explanation for assertion. O
(b) Assertion is correct, reason is correct; reason is not a 1 1 1
correct explanation for assertion H C OH H C HO C H
H 2 OH H 2 OH H 2 OH
(c) Assertion is correct, reason is incorrect HO 3 H O HO 3 H HO 3 H O
4 4
(d) Assertion is incorrect, reason is correct. H OH H OH H 4 OH
(d) H 5 H 5
OH H 56
163. Assertion : D(+)– Glucose is dextrorotatory in nature. 6 6
Reason : ‘D’ represents its dextrorotatory nature. CH2OH CH2OH CH2OH
-D(+)-glucose -D(+)-glucose
164. Assertion : Sucrose is called an invert sugar.
Reason : On hydrolysis, sucrose bring the change in the 172.When -D-glucose and -D-glucose are dissolved in water
sign of rotation from dextro (+) to laevo(–). in two separate beakers I and II respectively and allowed to
165. Assertion : -glycosidic linkage is present in maltose, stand, then –
(a) specific rotation in beaker I will decrease while in II will
CH2OH CH2OH increase upto a constant value
H O H OH (b) the specific rotation of equilibrium mixture in two
H H beakers will be different
OH H O OH H
HO H OH (c) the equilibrium mixture in both beakers will be
H OH H OH leavorotatory
(d) the equilibrium mixture in both beakers will contain
Reason : Maltose is composed of two glucose units in which only cyclic form of glucose
C–1 of one glucose unit is linked to C–4 of another glucose 173. In disaccharides, if the reducing groups of monosaccharides
unit. i.e., aldehydic or ketonic groups are bonded, these are non-
166. Assertion : At isoelectric point, the amino group does not reducing sugars. Which of the following disaccharide is a
migrate under the influence of electric field. non-reducing sugar?
Reason : At isoelectric point, amino acid exists as a CH2OH CH2OH
zwitterion. H O H H OH
H H
167. Assertion : Vitamin D cannot be stored in our body O
(a) OH H OH H
Reason : Vitamin D is fat soluble vitamin and is excreted HO OH
from the body in urine. H OH H OH
CH2OH
CRITICAL THINKING TYPE QUESTIONS O H HOH2C O
H H
H
168. Which one of the following is the reagent used to identify O
glucose? (b) HO OH H H HO CH2OH
(a) Neutral ferric chloride H OH OH H
(b) Chloroform and alcoholic KOH
(c) Ammoniacal silver nitrate CH2OH CH2OH
HO O HO O OH
(d) Sodium ethoxide H H
169. Glucose molecule reacts with 'X' number of molecules of OH H O OH H
(c)
H H H
phenylhydrazine to yield osazone. The value of 'X' is
H OH H OH
(a) four (b) one
(c) two (d) three CH2OH CH2OH
H O H O OH
170. In the acetylation of glucose, which group is involved in H H
the reaction OH H O OH H
(d)
(a) CHO group (b) >C = O group
HO H H
(c) alcoholic OH group (d) all of these H OH H OH
BIOMOLECULES 489
174. Which of the following pairs represents anomers? f
CH2OH a
CHO CHO e O H HOH2C O
H H
H OH HO H d
H a b e
OH H O
HO H HO H HO c b
H HO CH 2OH
f
c d
H OH H OH H OH OH H
(a)
H OH H OH (a) ‘a’ carbon of glucose and ‘a’ carbon of fructose.
CH2OH CH2OH (b) ‘a’ carbon of glucose and ‘e’ carbon of fructose.
(c) ‘a’ carbon of glucose and ‘b’ carbon of fructose.
CHO CHO (d) ‘f’ carbon of glucose and ‘f’ carbon of fructose.
H OH HO H 177. Cyclic structure of fructose resembles with
HO H H OH (a) pyran (b) furan
(c) pyridine (d) oxiran
(b) H OH HO H 178. Sucrose in water is dextro-rotatory, [ ]D= + 66.4º. When
H OH HO H boiled with dilute HCl, the solution becomes leavo-rotatory,
CH2OH CH2OH [ ]D= –20º. In this process the sucrose molecule breaks
into
(a) L-glucose + D-fructose
H OH HO H
(b) L-glucose + L-fructose
H OH H OH (c) D-glucose + D-fructose
HO H O HO H O (d) D-glucose + L-fructose
179. Which one of the following statements is not true regarding
(c) H OH H OH (+) Lactose ?
H H (a) On hydrolysis (+) Lactose gives equal amount of D(+)
CH2OH CH2OH glucose and D(+) galactose.
(b) (+) Lactose is a -glycoside formed by the union of a
H OH HO H molecule of D(+) glucose and a molecule of D(+)
galactose.
H OH HO OH (c) (+) Lactose is a reducing sugar and does not exhibit
HO H O H OH O mutarotation.
(d) (+) Lactose, C12H22O11 contains 8-OH groups.
(d) H OH HO H
180. Which one of the following sets of monosaccharides forms
H H
sucrose?
CH2OH CH2OH (a) –D-Galactopyranose and –D-Glucopyranose
175. Optical rotation of some compound along with their (b) –D-Glucopyranose and –D-Fructofuranose
structures are given below which of them have D (c) –D-Glucopyranose and –D- Fructofuranose
configuration. (d) –D-Glucopyranose and –D-Fructopyranose
181. Which of the following statements is correct?
CHO CH2OH (a) Only the compounds following general formula
H OH Cx(H2O)y are carbohydrates.
CHO C=O (b) Acetic acid (CH3COOH) having general formula
HO H HO H C2(H2O)2 falls in this category.
H OH
H OH H OH (c) Rhamnose having formula C6H12O5 is a carbohydrate.
Though this is not according to general formula of
CH2OH H OH H OH
carbohydrates.
CH2OH CH2OH
(+) rotation (+) rotation (–) rotation (d) Chemically the carbohydrates may be defined as
(I) (II) (III) optically inactive polyhydroxy aldehydes or ketones.
182. The strongest form of intermolecular bonding that could
(a) I, II, III (b) II, III be formed involving the residue of the amino acid valine is
(c) I, II (d) III (a) ionic bond
176. Structure of a disaccharide formed by glucose and fructose (b) hydrogen bond
is given below. Identify anomeric carbon atoms in (c) van der Waals interactions
monosaccharide units. (d) none of the above
EBD_7207
490 BIOMOLECULES

183. Which functional group participates in disulphide bond 186. The function of enzymes in the living system is to
formation in proteins? (a) transport oxygen
(a) Thioester (b) Thioether (b) provide energy
(c) provide immunity
(c) Thiol (d) Thiolactone
(d) catalyse biochemical reactions
184. Glycosidic linkage is actually an 187. Vitamin C must be supplied regularly in diet because
(a) Carbonyl bond (b) Ether bond (a) it is water soluble hence excreted in urine and can’t be
(c) Ester bond (d) Amide bond stored in the body
(b) it is fat soluble hence stored in the body and cannot
O be used on regular basis
||
185. For C N H (peptide bond ) (c) it is required in a large amount by the body hence
supplied regularly
Which statement is incorrect about peptide bond? (d) it is water soluble hence used by the body on daily
(a) C — N bond length in proteins is longer than usual basis and is to be supplied regularly.
bond length of the C — N bond 188. In both DNA and RNA, heterocylic base and phosphate
ester linkages are at –
(b) Spectroscopic analysis shows planar structure of the
(a) C5' and C1' respectively of the sugar molecule
— C — NH — group
||
O (b) C1' and C5' respectively of the sugar molecule
(c) C — N bond length in proteins is smaller than usual (c) C '2 and C5' respectively of the sugar molecule
bond length of the C—N bond
(d) None of the above (d) C5' and C '2 respectively of the sugar molecule
BIOMOLECULES 491

FACT / DEFINITION TYPE QUESTIONS 16. (d) It is the most abundant organic compound on earth.
17. (d) Tollen’s reagent is reduced by glucose due to aldehydic
1. (c) Carbohydrates, proteins and fats are biomolecules. group and gives grey colour as silver metal.
2. (a) Lactose is a disaccharide. 18. (b) The letter ‘D’ or ‘L’ before the name of any compound
3. (c) It is found in the milk of all animals and imparts indicate the relative configuration of a particular
sweetness to milk (hence named milk sugar). stereoisomer.
4. (d) The disaccharides are sugars which on hydrolysis give 19. (a) 20. (a)
two moles of the same or different monosaccharides. 21. (d) Glucose is a monosaccharide, others are disaccharides.
Sucrose, maltose and lactose (C12H22O11 ) are the Sucrose is a combination of glucose and fructose.
common examples. Maltose is a combination of two glucose units. Lactose
5. (c) Aldo-(keto) pentoses having 5 carbon (or milk sugar) is a combination of glucose and
Aldo-(keto) hexoses having 6 carbon galactose (a hexose sugar).
is an example of Pentose Sugar, arabinose 22. (c) Glucose is considered as a typical carbohydrate which
(aldopentose) glucose, galactose and fructose are contains –CHO and –OH group.
important examples of hexose sugar. 23. (a) Glucose contains an aldehyde group. It is oxidised
6. (d) The most common disaccharide, Lactose has the into acidic group by bromine water and gluconic acid
molecular formula C12H22O11. is formed
7. (a) 8. (d) (O)
CH 2 OH (CHOH) 4 CHO
9. (d) Monosaccharides cannot be hydrolysed to simpler
molecules. CH 2OH (CHOH ) 4 COOH
10. (b) Sucrose is an oligosaccharide and cellulose is a
polysaccharide. Br2 H 2O 2HBr O ob
11. (c) All those carbohydrates which reduce Fehling’s 24. (d) Weak reagent like NaHSO3 is unable to open the chain
solution and Tollens’ reagent are referred to as reducing and can’t react with glucose. This explains the inability
sugars. of glucose to form aldehyde bisulphite compound.
12. (a) Glucose (C 6 H12O 6 ) is the simplest molecule which 25. (c) Glucose has 5 hydroxyl groups, hence it reacts with
is monosaccharide while others are polysaccharides acetic anhydride to form a penta-acetate
which on hydrolysis give monosaccharides. 26. (d)
Option (a) is correct. 27. (d) Red P + HI is reducing agent.
13. (d) Glucose is aldohexose. Glucose is a monosaccharide, 28. (c) Pentaacetate of glucose does not react with
i.e. it can not be hydrolysed further to simple sugars. hydroxylamine
Oligosaccharides on hydrolysis give 2-10 molecules 29. (c) Open chain structure is unstable and converted to
of monosaccharides. cyclic.
14. (b) To explain the properties which can not be explained 30. (b)
by open chain structure of glucose it was proposed 31. (a) Natural glucose is dextrorotatory and thus, glucose is
that one of the –OH groups may add to the –CHO also known as dextrose.
group and form a cyclic hemiacetal structure as shown
32. (c)
below. 1
H OH
O
2
1
H—C—OH 1 1
HO —C—H H OH
H —C
2 2 2 3 O
H OH H OH H OH HO H
3 3
HO H HO 3 H HO H
H
4
OH
H 4 OH H 4 OH H 4 OH 5
H
H 5 H 5 H 5
6 OH CH2OH
6CH2OH CH OH
6 2 6CH2OH -D-(+) - Glucose
– D – (+) – Glucose – D – (+) – Glucose
15. (c) Glucose contain aldehyde group. Hence it give positive (Fischer formula)

Fehling solution test.


EBD_7207
492 BIOMOLECULES
33. (b) -D glucose and –D glucose are the isomers which H H
H
differ in the orientation (configuration) of H and OH ROH
43. (c) – C = O + ROH – C – OH – C – OR + H2O
groups around C1 atom.
OR OR
1 1 An aldehyde A hemiacetal An acetal
H —C —OH HO —C —H
2
H —C —OH H —C2—OH – C = O + ROH – C – OH
ROH
– C – OR + H2O
O O
HO —C3—H HO —C3—H OR OR
A hemiketal A ketal
H —C4—OH H —C4—OH
A ketone

5 5
In cyclic structure of fructose, ketonic group has
H —C H —C reacted with an alcoholic group, it is said to be an
6 6
CH2OH CH2OH example of an intramolecular cyclic hemiketal.
– D - glucose – D - glucose 44. (a) Sweet taste of fruits is due to fructose.
45. (a)
34. (b) T h e t wo i somer i c for m s ( – a n d –) of
D-glucopyranose differ in configuration only at 46. (c) Honey is collected from flowers by honey bee which
C–1; hence these are called anomers. contains fructose.
47. (c) Fructose is the sweetest sugar.
35. (b)
48. (a) We know that cellulose (C 6 H12 O 6 ) n is the chief
36. (d) constituent of cell walls of plants. It is the most
1
H OH abundant organic substance found in nature. It is a
2 polymer of glucose with 3500 repeat units in a chain.
H OH 49. (b) Sucrose is a disaccharide which on hydrolysis gives
3 O one molecule of glucose (monosaccharide) and
HO H
fructose (monosaccharide).
4
H OH
H
5 C12 H 22 O11 H 2 O C6 H12 O 6 C6 H12 O 6
H sucrose glucose fructose
CH2OH 50. (b) RNA has D (–) – Ribose and the DNA has 2–Deoxy
-D-(+) - Glucose D (–) – ribose as the carbohydrate unit.
O 5 O
(Fischer formula) HOCH2 OH HOCH2 OH
H H 4 H H 1
37. (b) Glucose and fructose both are reduced by Fehling’s
solution, Tollen’s reagent and Bendict’s solution. H H H H
3 2
Therefore, these three reagents can not be used to OH OH OH H
distinguish between glucose and fructose. ribose 2-deoxy ribose
38. (b) Maltose and glucose are reducing sugars. From the structures it is clear that 2nd carbon in DNA
do not have OH group.
39. (a) Glucose contains -CHO group and fructose contains
> C = O group,. Hence these are functional isomers. 51. (d) Carbohydrates are stored in the body as glycogen.
40. (d) Glucose being an aldose responds to Tollen’s test 52. (c)
while fructose, although a ketose, undergoes 53. (b) CH2OHCH2CHOHCHOHCH2OH does not correspond
rearrangement in presence of basic medium (provided to Cx(H2O)y.
by Tollen’s reagent) to form glucose, which then 54. (d) Lactose (milk sugar) is a disaccharide, it is made of
responds to Tollen’s test. -D-galactose and -D-glucose
41. (d) CH2OH
CH2OH H O H
42. (b) Fructose has the molecular formula C6 H12 O6. It HO O H
H OH H
belongs to D-series and is laevorotatory compound. It OH H OH
H H
also exists in two cyclic forms which are obtained by H OH
H OH
the addition of –OH at C-5 to the >C=O group. The
ring thus formed is a five membered ring and is named 55. (b) 56. (b)
as furanose with analogy to the compund Furan. Furan 57. (a) Sucrose does not have free — CHO or CO group, hence
is a five membered cyclic compound with one oxygen it does not undergo mutarotation.
and four carbon atoms. 58. (b) 59. (d)
BIOMOLECULES 493
60. (b) Sucrose is dextrorotatory but after hydrolysis gives 73. (d) In neutral solution, amino acids exists as dipolar ion (also
dextrorotatory glucose and laevorotatory fructose. known as zwitter ions or inner salts) where the proton of
Since the laevorotation of fructose (–92.4°) is more –COOH group is transferred to the – NH2 group to form
than dextrorotation of glucose (+52.5°), the mixture is inner salt, known as dipolar ion.
laevororatory. R R
61. (c) Chemically amylose is a long unbranched chain with .. | H 2O .. |
H2N–CH–COOH H2N–CHOO–+H+
200-1000 -D-(+ -glucose units held by C1-C4 -Amino acid
glycosidic linkage. R
+ |
62. (b) It is a branched chain polymer of -D-glucose units in H3N–CH–COO–
which chain is formed by C1-C4 glycosidic linkage Zwitter ion
whereas branching occurs by C1-C6 glycosidic linkage. 74. (c) With the exception of glycine all the 19 other common
63. (b) Cellulose is a straight chain polysaccharide. amino acids have a uniquely different functional group
64. (b) The carbohydrates are stored in animal body as on the central tetrahedral alpha carbon.
glycogen. It is also known as animal starch because
H
its structure is similar to amylopectin and is rather more |
highly branched. H — C — COOH
|
65. (d) Carbohydrates are essential for life in both plants and
NH 2
animals. Honey has been used for a long time as an
instant source of energy by ‘Vaids’ in ayurvedic system glycine
of medicine. Carbohydrates are used as storage 75. (d) Zwitter ion contains both +ve and –ve charge. Proton
molecules as starch in plants and glycogen in animals. of –COOH group is transferred to the –NH2 group.—
Cell wall of bacteria and plants is made up of cellulose. NH3+ group is acidic since it can donate a proton and
We build furniture etc., from cellulose in the form of —COO– group is basic since it can accept a proton.
wood and cloth ourselves with cellulose in the form of 76. (a) Amino Acids are amphoteric in nature. So for it a special
cotton fibre. They provide raw materials for many term is coined called Zwitter ion.
important industries like textiles, paper lacquers and They have following structure in solution
breweries. R
66. (b) There are 20 amino acids in man out of which 10 amino + –
acids are essential amino acids. These essential amino H3N–C–COO
acids are supplied to our bodies by food which we H
take because they cannot be synthesised in the body. [Zwitter Ion]
These are (1) valine (2) leucine (3) Isoleucine (4) Phenyl 77. (d) Proline is a secondary amine
alanine (5) Threonine (6) Methionine (7) Lysine (8) 78. (a) Proteins and peptides are linked by peptide linkages
Tryptho phone (9) Arginine (10) Histidine.
67. (N) All the given options are example of neutral amino O
||
acids. ( C NH) .
68. (b) - Amino acid is the building block unit of protein
79. (c) The bond formed between two amino acids by the
which is formed by polymerisation of amino acid
elimination of a water molecule is called a peptide
through peptide linkage.
linkage or bond. The peptide bond is simply another
NH C name for amide bond.
|| C OH H — N— — C– N— H 2 O
O | | || |
O H O H
Carboxyl group Amine group of Peptide bond
69. (a) Except alanine, all amino acids are essential amino acids of one amino acid other amino acid
which cannot be synthesised in the body and must be
The product formed by linking amino acid molecules
obtained through diet.
through peptide linkages. —CO—NH—, is called a
70. (a)
peptide.
71. (c) Amino acids are the compounds having one or more 80. (b) 81. (c)
amino groups and one or more carboxyl groups in the 82. (d) Proteins are highly complex, natural compounds,
same molecule. composed of a large number of different -amino-acids
72. (b) Except glycine, all other naturally occurring joined together with peptide linkage, i.e., they are
- amino acids are optically active, since the -carbon naturally occuring polypeptides.
atom is asymmetric. 83. (d) All these are the examples of globular proteins. These
are soluble in water.
EBD_7207
494 BIOMOLECULES
84. (c) Polypeptide chains in fibrous proteins are held together 102. (b) In this structure of protein atoms are highly coiled
by disulphide and hydrogen bonds. and form a spherical form.
85. (d) Proteins are building blocks of the body but they do 103. (d)
not provide energy for metabolism. 104. (d) When a protein, in its native form, is subjected to a
86. (c) Insulin is an example of globular protein. physical change like change in temperature, or a
87. (c) 88. (b) 89. (b) chemical change like change in pH, the native
90. (c) These are generally insoluble in water. conformation of the molecule is disrupted and proteins
91. (b) The NH of the amide can act as a hydrogen bond donor so formed are called denaturated proteins.
and the carbonyl group can act as a hydrogen bond The denaturation may be reversible or irreversible. The
acceptor. Statements (a), (c) and (d) are false. The coagulation of egg on boiling is an example of
peptide bond has double bond character due to the irreversible protein denaturation.
interaction of the nitrogen lone pair with the carbonyl However, it has been shown now that in some cases,
group. This prevents bond rotation and makes the bond the process is actually reversible. The reverse process
planar. The trans isomer is favoured over the cis isomer. is called renaturation.
92. (a) 105. (c) Tertiary structure indicates the overall structure of the
93. (a) The sequence in which the -amino acids are linked to protein.
one another in a protein molecule is called its primary 106. (b)
structure. 107. (b) Serine contains a hydroxyl functional group on its side
94. (a) chain and so the strongest possible interaction will be
95. (b) The -helix structure is formed when the chain of hydrogen bonding where the hydroxyl group could
-amino acids coils as a right handed screw (called act as a hydrogen bond donor or hydrogen bond
-helix) because of the formation of hydrogen bonds acceptor.
between amide groups of the same peptide chain, i.e., 108. (a) When antigens enter in to the body cells and destroy
NH group in one unit is linked to carbonyl oxygen of them, then antibodies being proteins are synthesised
the third unit by hydrogen bonding. This hydrogen in the body and combine with antigens and destroy
bonding between different units is responsible for these antigens by forming inactive complexes.
holding helix in a position. Therefore antibodies protein destroy antigens.
96. (c) During denaturation 2° and 3° structures are destroyed 109. (b) Enzymes are highly specific for a particular reaction
but 1° structure remains intact. and also for a particular substrate.
97. (a) Primary structure refers to the order of the amino acids
in a protein. 110. (b) Enzymes being biocatalyst can increase the rate of a
98. (d) The hydrophilic/ hydrophobic character of amino acid reaction upto 10 million times. Even very small amount
residues is important to tertiary structure of protein can accelerate a reaction.
rather than to secondary structure. In secondary 111. (b) Enzymes are made up of protein with specific structure.
structure, it is the steric size of the residues that is 112. (b) Triglycerides are lipids, hence these are hydrolysed
important and residues are positioned to minimise by lipases to glycerol and fatty acids.
interactions between each other and the peptide chain. 113. (d) Enzymes are most reactive at optimum temperature.
99. (d) Quaternary structure refers to the overall structure of The optimum temperature for enzyme activity lies
a multiprotein complex where as primary, secondary between 40°C to 60°C.
and tertiary structure refer to the different structural 114. (b) Insulin is a biochemically active peptide harmone
levels of a single protein. secreted by pancreas.
100. (a) The arrangement of polypeptide chains formed as a 115. (a)
result of hydrogen bonding is called secondary 116. (d) Enzymes may or may not require a coenzyme for their
structure of proteins. catalytic action.
-helix is formed by intramolecular H-bonding. 117. (c) Deficiency of vitamin D causes rickets.
-pleated sheet is formed by intermolecular H-bonding. 118. (d) Vitamin D is a fat soluble vitamin.
101. (b) The secondary structure of a protein refers to the
119. (b) Beri-Beri.
shape in which a long peptide chain can exist. There
120. (d) Vitamin Disease caused by deficiency
are two different conformations of the peptide linkage
B6 Dermatitis
present in protein, these are -helix and -conformation.
The -helix always has a right handed arrangement. In B1 Beri-beri
-conformation all peptide chains are streched out to B2 Photophobia, glossitis
nearly maximum extension and then laid side by side B12 Pernicious anaemia
and held together by intermolecular hydrogen bonds. 121. (b) Vitamin E is mainly present in vegetable oils like wheat
The structure resembles the pleated folds of drapery gram oil, sunflower oil, etc.
and therefore is known as -pleated sheet. 122. (a) Vitamin A or retinol.
BIOMOLECULES 495
123. (b) 140. (d) The correct structure of thymine is
124. (c) It is found in liver, egg, milk, meat, and fish. Minute OH
amounts are probably present in all animal cells.
N CH3
Peculiarly, unlike other vitamins, B12 is not found in
significant amounts in green plants.
HO N
125. (a) DNA consists of two polynucleotide chains, each Thymine (T)
chain forms a right handed helical spiral with ten bases
in one turn of the spiral. The two chains coil to double 141. (c)
helix and run in opposite direction held together by 142. (d) Tyrosine is an -amino acid, and not a purine
143. (d)
hydrogen bonding.
144. (c) DNA fingerprinting is same for every cell and cannot
126. (d) O Deoxyribose-Adenine ... Thymine -Deoxyribose O be altered by any known treatment.
OH P Deoxyribose-Guanine ... Cytosine -Deoxyribose
P OH
O O STATEMENT TYPE QUESTIONS
P P
OH OH
O Deoxyribose-Guanine ... Cytosine -Deoxyribose O 145. (a)
OH P Deoxyribose-Adenine ... Thymine -Deoxyribose
P OH 146. (c) Sucrose is non-reducing in nature. It does not contain
a free aldehydic or ketonic group. Maltose is a
The hydrogen bonds are formed between the base reducing sugar.
(shown by dotted lines). Because of size and 147. (b) Naturally occurring monosaccharides are 20 only.
geometrics of the bases, the only possible pairing in Sucrose is a non–reducing sugar whereas maltose is a
DNA and between G(Guanine) and C(Cytosine) reducing sugar.
through three H-bonds and between A (Adenosine) 148. (b) The six membered cyclic structure of glucose is called
and T (Thymine) through two H-bonds. pyranose structure ( or ), in analogy with pyran.
127. (a) In DNA the complimentary base are Adenine and Pyran is a cyclic organic compound with one oxygen
thymine. atom and five carbon atoms in the ring. The cyclic
Guanine and cytosine structure of glucose is correctly represented by
The genetic information for cell is contained in the Haworth structure.
sequence of bases A, T, G and C in DNA molecule. 149. (d) Sucrose is a dextrorotatory but on hydrolysis gives
128. (b) The DNA sequence that codes for a specific protein is dextrorotatory glucose and laevorotatory fructose.
called a Gene and thus every protein in a cell has a Since the laevorotation of fructose (–92.4°) is more
corrosponding gene. than dextrorotation of glucose (+52.5°), thus the
129. (c) The base pairs of the two strands of DNA are linked resulting mixture is laevorotatory. Glycogen is
together through H-bonds. structurally similar to amylopectin not amylose.
130. (d) DNA has the property of self - replication . It is therefore 150. (a) For statement (i) correct Haworth structure for –D
a reproducing molecule. This unique property of DNA glucose will be.
is at the root of all reproduction. Through its 6
replication, DNA acts as the key to heredity. In the CH2OH
replication of DNA, the two strands of a double helix 5 O
unwind and separate as a template for the formation of H H H
a new complementary strand. 4 1
131. (c) Each chromosome is made up of DNA tightly coiled OH OH OH
H
many times around proteins called histones that 3 2
supports its structure.
H OH
132. (a) 151. (c) Amylose is water soluble component which constitutes
133. (a) DNA has double stranded -helical structure. about 15-20% of strach. Amylopectin is insoluble in
134. (b) DNA is responsible for transmission of heredity and constitutes about 80-85% of starch.
character. 152. (c) Glycine is optically inactive.
135. (b) 153. (a) Gln stands for glutamine. Except glycine, all other
136. (a) Energy is stored in our body in the form of A.T.P. naturally occurring –amino acids are optically active.
137. (d) 154. (c) When the proteins are subjected to the action of heat,
138. (a) Phosphoric acid is the third component in DNA. mineral acids or alkali, the water soluble form of globular
139. (a) Synthesis of polypeptide is known as translation. For protein changes to water insoluble fibrous protein. This
this process three type of RNA are essential. is called denaturation of proteins. During denaturation
secondary and tertiary structures of protein destroyed
but primary structures remains intact.
EBD_7207
496 BIOMOLECULES

155. (c) group, the third molecule of phenylhydrazine condenses


156. (b) Water soluble vitamins must be supplied regularly in to glucosazone. Therefore the value of X is 3
diet because they are readily excreted in urine and
cannot be stored (except vitamin B12) in our body.
157. (b) In secondary structure of DNA adenine forms CHO + H2NNHC6H5 CH = NNHC6H5
hydrogen bonds with thymine whereas cytosine forms
CHOH CHOH
hydrogen bonds with guanine. warm
158. (b) The sugar moiety in DNA molecule is -D-2- (CHOH)3 CH(OH)3
deoxyribose whereas in RNA molecule, it is -D-ribose.
CH2OH CH2OH
Nucleotide is a nucleoside linked to phosphoric acid
Glucose Glucose Phenylhydrozone
at 51– position of sugar moiety.
H2NNHC6 H5
MATCHING TYPE QUESTIONS H2NNHC6 H5
CH = NNHC6H5 CH = NNHC6H5
– H2 O
159. (c) 160. (a) 161. (b)
162. (c) Vitamin A - Xerophthalmia C = NNHC6H5 C=O
Vitamin B12 - Pernicious anaemia
CH(OH)3 (CHOH)3
Vitamin C - Scurvy
Vitamin E - Sterility CH2OH CH2OH
Vitamin K - Haemorrhage Glucosazone keto compound of glucose
phenyle hydra zone
ASSERTION-REASON TYPE QUESTIONS
163. (c)
164. (a) The hydrolysis of sucrose brings about a change in 170. (c) OHC.(CHOH)4.CH2OH + 5Ac2O
Au.ZnCl2
the sign of rotation from dextro (+) to laevo (–) and the
Glucose Acetic anhydride
product is named as invert sugar.
165. (d) Maltose is compound of two -D - glucose units in OHC. (CHOAc) 4 . CH2 OAc
Glucose penta-acetate
which C1 of one glucose is linked to C4 of another
or penta-acetyl Glucose
glucose unit.
171. (c) Melting point of -glucose 419 K and -glucose is
6
CH2OH 6
CH2OH 323 K.
H 5 OH H 5 OH 172. (a) -D-glucose or -D-glucose when dissolved in water
H H
4 1 O 4 1 and allowed to stand, following equilibrium is
OH H OH H
HO 3 2 3 2 OH stablished, which is called mutarotation.
H OH H OH -D-glucose Open chain form -D-glucose
I II
-D-glucose -D-glucose (+111°) (+19°)
Specific rotation of -form falls until a constant value
166. (a) R is the correct explanation of A. of +52.5° is reached. On the other hand, specific
167. (d) Vitamin D is a fat soluble vitamin and can be stored in rotation of form increases. Specific rotation of
the body since it is not excreted out of the body. equilibrium mixture is 52.5°.
CRITICAL THINKING TYPE QUESTIONS 173. (b) The two monosoccharides are held together by a
glycosidic linkage between C1 of -glucose and C2 of
168. (c) Glucose contains aldehyde group due to which it gives -fructose. Since the reducing groups and glucose and
positive test with ammoniacal silver nitrate. fructose are involved in glycosidic bond formed.
169. (d) We know that glucose reacts with one molecule of Sucrose is non-reducing sugar.
phenyl hydrazine to give phenyl hydrazone. When
174. (c) Cyclic hemiacetal forms of monosaccharide which differ
warmed with excess of phenylhydrazine, the
only in the configuration of the hydroxyl group at C1
secondary alcoholic group adjacent to the aldehyde
are anomers.
group is oxidised by another molecule of
phenylhydrazine to a ketonic group. With this ketonic 175. (a) 176. (c)
BIOMOLECULES 497

Sucrose is a disaccharide of –D-Glucopyranose and


6 O 1 –D-Fructofuranose.
177. (b) HOH2C CH2OH 181. (c) Most of the carbohydrates have a general formula,
5 2 Cx(H2O)y, and were considered as hydrates of carbon.
H OH
4 3 All the compounds which fit into this formula may not
H OH
be classified as carbohydrates. Acetic acid (CH3COOH)
OH H fits into this general formula, C2(H2O)2 but is not a
-D-(–)-Fructofuranose carbohydrate. Similarly, rhamnose, C6H12O5 is a
carbohydrate but does not fit in this definition. A large
number of their reactions have shown that they contain
6 O specific functional groups. Chemically, the
HOH2C OH carbohydrates may be defined as optically active
5
H OH
2 polyhydroxy aldehydes or ketones or the compounds
4 3 which produce such units on hydrolysis.
H CH2OH
1 182. (c) Valine has no functional groups on its side chain. There
OH H is only an alkyl group and so only van der Waals
-D-(–)-Fructofuranose interactions are possible.

178. (c) The hydrolysis of sucrose by boiling with mineral acid 183. (c) 2R S H R S S R
Thiol Disulphide
or by enzyme invertase or sucrase produces a mixture
Example :
of equal molecules of D(+) glucose and
D(–) Fructose.
2HO 2C CHC H 2SH
|
HCl NH2
C12 H 22O11 H 2O C 6 H12O 6 C 6 H12O 6
sucrose D glu cos e D Fructose Cystine
[ D] 66.5º [ D ] 52.5º [ D ] 92 º
Invert sugar ,[ D ] 20º HO 2CCHC H 2S SCH 2 CHC O 2 H
| |
NH 2 NH2
CH2OH H OH Cystine
O
HO OH H
179. (c) H H 184. (b) Glycosidic linkage is actually an ether bond as the
O
OH H H
H H H O OH linkage forming the rings in an oligosaccharide or
H OH CH2OH polysaccharide is not just one bond, but the two bonds
(Lactose) sharing an oxygen atom e.g. sucrose
All reducing sugar shows mutarotation.

HO
180. (b) CH2 OH
O CH2 O
OH + OH
OH HO
OH H2C
OH OH OH
-D-Glucopyranose -D-Fructofuranose

HO
CH2 OH

O CH2 O
OH HO + H2O
O
OH CH2
OH OH
OH
Fructose
EBD_7207
498 BIOMOLECULES
185. (a) Due to resonance C — N bond in protein acquires 187. (a) Vitamin C is water soluble. Therefore, it is readily
double bond character and is smaller than usual excreted in urine and cannot be stored in our body and
C — N bond. is supplied regularly in diet.

O O 188. (b) In DNA and RNA heterocyclic base and phosphate
C NH C NH ester are at C1' and C5' respectively of the sugar
molecule.
186. (d) The function of enzymes in the living system is to
catalyse biochemical reactions which occur in living
HO N
systems. e.g. invertase, pepsin, amylase. | 5
N
HO – P – O – C H 2 4 O N
Invertase || N
Sucrose glucose + fructose O
C
C
H H
(polymer) (monomer) H | |
C3––– C2 H
amylase
Starch glucose | |
OH OH
(polymer) (monomer)
29
POLYMERS

FACT / DEFINITION TYPE QUESTIONS 12. A polymer is formed when simple chemical units
(a) combine to form long chains
1. Which is not true about polymers? (b) combine to form helical chains
(a) Polymers do not carry any charge (c) break up
(b) Polymers have high viscosity (d) become round
(c) Polymers scatter light 13. Polymer formation from monomers starts by
(d) Polymers have low molecular weight (a) condensation reaction between monomers
2. Which of the following belongs to the class of natural (b) coordinate reaction between monomers
polymers? (c) conversion of monomer to monomer ions by protons
(d) hydrolysis of monomers.
(a) Proteins (b) Cellulose
14. On the basis of mode of formation, polymers can be
(c) Rubber (d) All of these
classified?
3. Which of the following natural products is not a polymer?
(a) as addition polymers only
(a) DNA (b) Cellulose (b) as condensation polymers only
(c) ATP (d) Urease (c) as copolymers
4. Among the following a natural polymer is (d) both as addition and condensation polymers
(a) cellulose (b) PVC 15. In addition polymer monomer used is
(c) teflon (d) polyethylene (a) unsaturated compounds
5. Which of the following is not a biopolymer ? (b) saturated compounds
(a) Proteins (b) Rubber (c) bifunctional saturated compounds
(c) Cellulose (d) RNA (d) trifunctional saturated compounds
6. Rayon is : 16. Nylon 66 belongs to the class of
(a) synthetic plastic (b) natural rubber (a) Addition polymer
(c) natural silk (d) artificial silk (b) Condensation polymer
7. Protein is a polymer of: (c) Addition homopolymer
(d) Condensation heteropolymer
(a) glucose (b) terephthalic acid
17. A polymer made from a polymerization reaction that
(c) amino acids (d) None of these
produces small molecules (such as water) as well as the
8. Natural silk is a
polymer is classified as a/an ..... polymer.
(a) polyester (b) polyamide (a) addition (b) natural
(c) polyacid (d) polysaccharide (c) condensation (d) elimination
9. Polymers are: 18. In elastomer, intermolecular forces are
(a) micromolecules (b) macromolecules (a) strong (b) weak
(c) sub-micromolecules (d) None of the above (c) nil (d) None of these
10. Which of the following is/are a semisynthetic polymers? 19. A thermoplastic among the following is
(a) Cellulose acetate (b) Polyvinyl chloride (a) bakelite (b) polystyrene
(c) Cellulose nitrate (d) Both (a) and (c) (c) terylene (d) urea-formaldehyde resin
11. Which of the following is not linear polymer ? 20. Which is an example of thermosetting polymer?
(a) Bakelite (b) Polyester (a) Polythene (b) PVC
(c) Cellulose (d) High density polyethene (c) Neoprene (d) Bakelite
EBD_7207
500 POLYMERS
21. Which of the following is thermoplastic ? 32. Fibres that have good resistance to stains, chemicals,
(a) Bakelite (b) Polyethylene insects and fungi is
(c) Terylene (d) All of these (a) Acrylic (b) Terylene
22. Thermosets are: (c) Nylon (d) All of these
(a) cross-linked polymers 33. Which of the following statements is not true about low
(b) don’t melt or soften on heating density polythene?
(a) Tough
(c) cross-linking is usually developed at the time of
(b) Hard
moulding where they harden reversibly
(c) Poor conductor of electricity
(d) all of the above
(d) Highly branched structure
23. Which is/are true for elastomers? 34. Low density polythene is prepared by
(a) These are synthetic polymers possessing elasticity (a) Free radical polymerisation
(b) These possess very weak intramolecular forces or (b) Cationic polymerisation
attractions between polymer chains (c) Anionic polymerisation
(c) Vulcanised rubber is an example of elastomer (d) Ziegler-Natta polymerisation
(d) All of the above 35. The monomer of teflon is
24. Among the following polymers the strongest molecular (a) CHF == CH2 (b) CF2 == CF2
forces are present in (c) CHC1 == CHCl (d) CHF == CHC1
(a) elastomers (b) fibres 36. The monomer(s) used in the preparation of Orlon, a
(c) thermoplastics (d) thermosetting polymers substitute for wool is/are
25. Three dimensional molecular structure with cross links are (a) caprolactam
formed in the case of a (b) tetrafluoroethene
(a) thermoplastic (b) thermosetting plastic (c) styrene and 1, 3-butadiene
(d) acrylonitrile
(c) Both (a) and (b) (d) None of the above
37. Orlon is a polymer of
26. Which of the following polymer is an example of fibre ?
(a) styrene (b) tetrafluoroethylene
(a) Silk (b) Dacron (c) vinyl chloride (d) acrylonitrile
(c) Nylon-66 (d) All of these 38. Which of the following polymer is used for manufacturing
27. Which of the following statements is not correct for fibres? of buckets, dustbins, pipes etc ?
(a) Fibres possess high tensile strength and high (a) Low density polythene
modulus. (b) High density polythene
(b) Fibres impart crystalline nature. (c) Teflon
(c) Characteristic features of fibres are due to strong (d) Polyacrylonitrile
intermolecular forces like hydrogen bonding. 39. Which of the following catalyst is used in preparation of
(d) All are correct. high density polythene ?
28. Which of the following is/are examples of fibres? (a) Peroxide catalyst
(a) Polyesters (b) Polyamide (b) Ziegler - Natta catalyst
(c) Polythene (d) Both (a) and (b) (c) Wilkinson’s catalyst
29. Which of the following can be repeatedly soften on (d) Pd - catalyst
40. Which of the following statements is false?
heating?
(a) Artificial silk is derived from cellulose.
(i) Polystyrene (ii) Melamine
(b) Nylon-66 is an example of elastomer.
(iii) Polyesters (iv) Polyethylene
(c) The repeat unit in natural rubber is isoprene.
(v) Neoprene (d) Both starch and cellulose are polymers of glucose.
(a) (i) and (iii) (b) (i) and (iv) 41. Melamine plastic crockery is a copolymer of:
(c) (iii), (iv) and (v) (d) (ii) and (iv) (a) HCHO and melamine (b) HCHO and ethylene
30. Which of the following does not undergo addition (c) melamine and ethylene (d) None of these
polymerization? 42. Caprolactam polymerises to give
(a) Vinylchloride (a) terylene (b) teflon
(b) Butadiene (c) glyptal (d) nylon-6
(c) Styrene 43. Nylons, polysters and cotton, all posses strength due to:
(d) All of the above undergoes addition polymerizations (a) intermolecule H-bonding
31. Which of the following is a cross linked polymer? (b) van der Waals’ attraction
(a) PVC (b) Bakelite (c) dipole-dipole interaction
(c) Polyethylene (d) Rubber (d) None of the above
POLYMERS 501
44. Nylon 66 is a polyamide obtained by the reaction of 55. Which of the following polymer is a polyamide ?
(a) COOH(CH2)4 COOH + NH2C6H4NH2 (a) Terylene (b) Nylon
(b) COOH(CH2)4 COOH + NH2 (CH2)6 NH2
(b) Rubber (d) Vulcanised rubber
(c) COOH (CH2)6 COOH + NH2 (CH2)4 NH2
(d) COOHC6H4 COOH–(p) + NH2 (CH2)6 NH2 56. Which of the following compound is used for preparation
45. Interparticle forces present in nylon-66 are of melamine formaldehyde polymer ?
(a) van der waal’s NH2
(b) hydrogen bonding
(c) dipole-dipole interactions (a)
(d) None of these
46. The plastic household crockery is prepared by using
(a) melamine and tetrafluoroethane N
N N
(b) malonic acid and hexamethyleneamine
(c) melamine and vinyl acetate
(d) melamine and formaldehyde (b)
H2N NH2
47. Which of the following is currently used as a tyre cord ? NH2
(a) Terylene (b) Polyethylene
(c) Polypropylene (d) Nylon - 6
48. Of the following which one is classified as polyester H2N NH2
N
polymer ?
(a) Terylene (b) Bakelite
N N
(c) Melamine (d) Nylon-66 (c)
49. Which one of the following is not a condensation polymer ?
(a) Melamine (b) Glyptal NH2
(c) Dacron (d) Neoprene
50. Bakelite is obtained from phenol by reacting with NH2
(a) (CH2OH)2 (b) CH3CHO
(c) CH3 COCH3 (d) HCHO
51. The polymer containing strong intermolecular forces e.g. (d)
N
hydrogen bonding, is H2N
(a) teflon (b) nylon 6, 6
(c) polystyrene (d) natural rubber 57. Acetic acid is added in the preparation of Nylon-6 due to
52. Nylon threads are made of (a) initiate polymerisation
(a) polyester polymer (b) polyamide polymer (b) avoid polymerisation at first step
(c) polyethylene polymer (d) polyvinyl polymer (c) avoid oxidation
53. Which compound/set of compounds is used in the (d) removal of water
manufacture of nylon 6? 58. The bakelite is made from phenol and formaldehyde. The
initial reaction between the two compounds is an example
(a) CH = CH2 of:
(a) aromatic electrophilic substitution
(b) HOOC(CH2)4COOH + NH2(CH2)6 NH2 (b) aromatic nucleophilic substitution
(c) free radical reaction
CH3 (d) aldol reaction
| 59. Melamine plastic crockery is a codensation polymer of
(c) CH 2 CH C CH 2
(a) HCHO and melamine
O (b) HCHO and ethylene
(c) melamine and ethylene
(d) None of these
(d) 60. Which of the following polymer is used for making
phonograph records ?
54. The repeating unit present in Nylon 6 is (a) Bakelite (b) Dacron
(a) — [NH(CH2)6NHCO(CH2)4CO] — (c) Teflon (d) PVC
(b) — [CO(CH2)5NH] — 61. Novolac is
(c) — [CO (CH2)6NH] — (a) cross-linked polymer (b) linear polymer
(d) — [CO (CH2)4NH] — (c) addition polymer (d) synthetic rubber
EBD_7207
502 POLYMERS
62. Dacron is a – 67. Which of the following is an use of butadiene – styrene
(a) crease resistant copolymer?
(b) polyamide (a) Manufacture of autotyres
(c) addition polymer (b) Footwear components
(d) polymer of ethylene glycol and phthalic acid (c) Cable insulation
(d) All of these
63. The monomeric units of terylene are glycol and which of
the following 68. A homopolymer is obtained by polymerisation of:
(a) one type of monomer units
OH (b) two types of monomer units
OH (c) either of the above
(a) (b) (d) None of the above
OH OH 69. A copolymer of isobutylene and isoprene is called:
(a) align butyl rubber (b) buna-S
OH (c) buna-N (d) thiokol
(c) COOH COOH (d) OH 70. Which one is a homopolymer?
OH (a) Bakelite (b) Nylon
(c) Terylene (d) Neoprene
64. Which of the following is novolac ?
71. Polymerisation in which two or more chemically different
CH3 CH3 monomers take part is called:
(a) addition polymerisation
–O O–
(a) (b) copolymerisation
(c) chain polymerisation
(d) homo polymerisation
72. Natural rubber is a polymer of
OH (a) butadiene (b) isoprene
–H2C CH3 (c) 2-methylbutadiene (d) hexa-1, 3-diene
73. Which one of the following statement is not true?
(b) (a) In vulcanization the formation of sulphur bridges
between different chains make rubber harder and
stronger.
(b) Natural rubber has the trans -configuration at every
OH double bond
CH2 – (c) Buna-S is a copolymer of butadiene and styrene
(c) (d) Natural rubber is a 1, 4 - polymer of isoprene
74. Natural rubber is polymer of
(a) CH2 = CH – Cl

CH2 – CH3
|
(b) cis CH 2 C CH CH 2
OH OH
CH2 CH3
CH2 – |
(d) (c) trans CH 2 C CH CH 2

CH3 CH3
65. Soft drinks and baby feeding bottles are generally made up | |
of (d) cis CH 2 C C CH 2
(a) Polystyrene (b) Polyurethane
75. Which of the following is not the property of natural rubber
(c) Polyurea (d) Polyamide
66. Which is not an example of copolymer ? (a) Low tensile strength
(b) High water absorption capacity
(a) SAN (b) ABS
(c) Soft and sticky
(c) Saran (d) PVC
(d) High elasticity
POLYMERS 503
76. Natural rubber is 86. Which of the following polymer is formed on reaction of
(a) all trans polyisoprene (b) all cis-polysioprene 1,3–butadiene and acrylonitrile?
(c) chloroprene (d) Buna-N (a) Buna–S (b) Buna – N
77. The process involving heating of rubber with sulphur is (c) Neoprene (d) Dacron
called 87. Which of the following monomers form biodegradable
(a) Galvanisation (b) Vulcanization polymers?
(c) Bessemerisaion (d) Sulphonation (a) 3-hydroxybutanoic acid + 3-hydroxypentanoic acid
78. Isoprene is a valuable substance for making (b) Glycine + amino caproic acid
(a) propene (b) liquid fuel (c) Ethylene glycol + phthalic acid
(c) synthetic rubber (d) petrol (d) both (a) and (b)
79. Synthetic polymer which resembles natural rubber is : 88. The polymer which undergoes environment degradation
by microoganism is known as
(a) neoprene (b) chloroprene
(a) chain-growth polymer
(c) glyptal (d) nylon
(b) chain step polymer
80. Synthetic rubber is:
(c) biodegradable polymer
(a) polyester (b) polyamide (d) non-biodegradable polymer
(c) polysaccharide (d) poly (halodiene) 89. Generally, molecular mass of a polymer is over
81. Which of the following are example of synthetic rubber? (a) 100 (b) 500
(i) Polychloroprene (ii) polyacrylonitrile (c) 1,000 (d) 10,000
(iii) Buna-N (iv) cis-polyisoprene 90. For natural polymers PDI is generally
(a) (i) and (iii) (b) (i) and (ii) (a) 0 (b) 1
(c) (iii) and (iv) (d) (ii) and (iii) (c) 100 (d) 1000
82. Buna-N synthetic rubber is a copolymer of :
(a) H2C = CH – CH = CH2 and H5C6 – CH = CH2 STATEMENT TYPE QUESTIONS
(b) H2C = CH – CN and H2C = CH – CH = CH2
91. Which of the following statement(s) is/are correct?
(c) H2C = CH – CN and H 2 C = CH – C = CH 2 (i) Macromolecules have high molecular mass of order
|
CH3 103 – 107u.
(ii) Monomeric units are joined together by ionic or
Cl covalent bond.
|
(d) H 2 C = CH – C = CH 2 and H 2C = CH – CH = CH 2 (a) Only (i) (b) Only (ii)
83. Which of the following structures represents neoprene (c) Both (i) and (ii) (d) Neither (i) nor (ii)
polymer? 92. Consider the following statements.
(i) Polystyrene is a homopolymer whereas Buna–N is a
(a) –(CH 2 – C = CH – CH 2 –) n copolymer.
| (ii) Condensation polymers can be obtained by
Cl condensation between two similar bi–functional
monomeric units.
CN
(iii) Elastomers are the polymers in which the polymeric
|
(b) –(CH 2 – CH –) n
chains are held together by the weakest intermolecular
forces.
(iv) Buna–S and Buna–N consist of close packing of chains
Cl which impart them crystalline nature.
| Which of the following is the correct code for the statements
(c) –( CH 2 – CH –)n above?
(a) TTFF (b) TFTF
(d) –( CH – CH 2 –)n (c) FTFT (d) TFFT
| 93. Two condensation polymers are made
C6 H5 (1) ethylene diamine + ethane–1, 2– dicarboxylic acid
(2) trimethylenediamine + ethane–1, 2– dicarboxylic acid
84. Which of the following is not a copolymer?
if both polymers of same molecular weight are obtained
(a) Buna–S (b) Baketite
then which of the following statements is/are correct ?
(c) Neoprene (d) Dacron
(i) Polymer (1) is found to melt at lower temperature.
85. In the manufacture of tyre rubber, the percentage of sulphur (ii) Polymer (2) is found to melt at lower temperature.
used as a crosslinking agent is (iii) H-bonding is major factor.
(a) 2% (b) 5% (a) (i), (ii) and (iii) (b) Only (ii)
(c) 10% (d) 0.5% (c) (i) and (iii) (d) (ii) and (iii)
EBD_7207
504 POLYMERS
94. Which of the following statements are correct? 98. Match the columns.
(i) A polyamide nylon 6,6 prepared by the condensation Column-I Column-II
polymerisation of hexamethylene diamine with adipic (A) Highly branched (p) Teflon
acid is used in the manufacture of tyre cords. chemically inert polymer
(ii) Terylene is crease resistant and is blended with cotton used in the insulation of
and wool fibres for various applications.
electric wires.
(iii) Condensation reaction of phenol and formaldehyde
(B) Linear polymer (q) Polyacrylonitrile
to form novolac can be catalysed either by acid or
base. prepared in presence of
(iv) Melamine formaldehyde polymer is mainly used in the Al(C2H5)3 and TiCl4.
manufacture of electrical switches (C) Corrosion resistant (r) HDPE
(a) (i), (ii) and (iii) (b) (ii), (iii) and (iv) polymer used in
(c) (iii) and (iv) (d) (ii) and (iii) manufacture of non–stick
95. Read the following statements. surface coated utensils.
(i) Rubber latex is a colloidal dispersion of rubber in water. (D) Addition polymer (s) LDPE
(ii) Natural rubber is a cis –1, 4–polyisoprene having used as a substitute for
elastic properties due to coiled structure and weak wool.
van der Waal’s forces. (a) A–(s), B–(r), C–(q), D–(p)
(iii) Vulcanisation of natural rubber with sulphur and an (b) A–(s), B–(p), C–(r), D–(q)
appropriate additive is carried out above 415K. (c) A–(s), B–(r), C–(p), D–(q)
(iv) In the manufacture of tyre rubber, 5% of sulphur is (d) A–(r), B–(s), C–(p), D–(q)
used as a cross-linking agent.
99. Match Column-I (Monomer) with Colum-II (Polymer) and
(v) Synthetic rubbers are homopolymers.
select the correct answer using the codes given below the
Which of the following is the correct code for the lists:
statements above?
Column-I Column-II
(a) FFTFT (b) TTFTF
(A) Hexamethylenediamine (p) Bakelite
(c) TTFFF (d) FFTTF
(B) Phenol (q) Dacron
96. Which of the following statements are correct?
(C) Phthalic acid (r) Glyptal
(i) Buna–N being resistant to the action of petrol,
lubricating oil and organic solvents is used in making (D) Terephthalic acid (s) Melamine
oil seals. (t) Nylon
(ii) Biodegradable polymers are manufactured because (a) A – (t), B – (p), C – (q), D – (r)
of low chemical resistance, strength and durability of (b) A – (t), B – (p), C – (r), D – (q)
conventional polymers. (c) A – (s), B – (r), C – (p), D – (q)
(iii) PHBV is a copolymer used in the manufacture of (d) A – (s), B – (r), C – (q), D – (p)
orthopaedic devices. 100. Match the columns
(iv) Nylon 2–nylon 6 is a biodegradable polymer. Column-I Column-II
(a) (i), (ii) and (iii) (b) (ii), (iii) and (iv) (A) Polyester of glycol and (p) Novalac
(c) (i), (iii) and (iv) (d) (i) and (iv) phtalic acid
(B) Copolymer of 1, 3-butadiene (q) Glyptal
MATCHING TYPE QUESTIONS
and styrene
97. Match the columns. (C) Phenol and formaldehyde (r) Buna-S
Column-I Column-II resin
(A) Linear polymer (p) Melamine (E) Polyester of glycol and (s) Buna-N
(B) Semisynthetic polymer (q) Polyvinyl chloride terephthalic acid
(C) Branched chain polymer (r) LDPE (F) Copolymer of 1, 3-butadiene (t) Dacron
(D) Network polymer (s) Cellulose nitrate and acrylonitrile
(a) A– (s), B – (q), C – (r), D – (p) (a) A – (q), B – (s), C – (p), D – (t), E – (r)
(b) A– (q), B – (s), C – (r), D – (p) (b) A – (q), B – (r), C – (p), D – (t), E – (s)
(c) A– (q), B – (r), C – (s), D – (p) (c) A – (r), B – (p), C – (t), D – (s), E – (q)
(d) A– (q), B – (s), C – (p), D – (r) (d) A – (p), B – (s), C – (q), D – (t), E – (r)
POLYMERS 505
101. Match the polymers given in Column-I with their chemical 104. Assertion : Olefinic monomers undergo addition
names given in Column-II polymerisation.
Column-I Column-II
Reason : Polymerisation of vinylchloride is initiated by
(A) Nylon 6 (p) Polyvinyl chloride
peroxides/ persulphates.
(B) PVC (q) Polyacrylonitrile
(C) Acralin (r) Polycaprolactum 105. Assertion : Teflon has high thermal stability and chemical
(D) Natural rubber (s) Low density polythene inertness.
(E) LDP (t) cis-polyisoprene Reason : Teflon is a thermoplastic.
(a) A – (r), B – (p), C – (q), D – (t), E – (s) 106. Assertion : Bakelite is a thermosetting polymer.
(b) A – (s), B – (q), C – (t), D – (r), E – (p) Reason : Bakelite can be melted again and again without
(c) A – (t), B – (s), C – (p), D – (q), E – (r) any change.
(d) A – (s), B – (t), C – (r), D – (q), E – (p)
107. Assertion : In vulcanisation of rubber, sulphur cross links
102. Match the columns
are introduced.
Column-I (Polymers) Column-II (Uses)
(A) Nylon 6,6 (p) Fabrics and ropes Reason : Vulcanisation is a free radical initiated chain
(B) Nylon 6 (q) Electrical switches reaction.
(C) Dacron (r) Bristles for brushes 108. Assertion : The time of vulcanisation and temperature is
(D) Bakelite (s) As glass reinforcing increased by adding accelerators.
materials in safety Reason : By vulcanising, a material of high tensile strength
helmets.
can be obtained.
(a) A – (q), B – (s), C – (r), D – (p)
109. Assertion : Most of the Synthetic polymers are not
(b) A – (r), B – (q), C – (s), D – (p)
biodegradable.
(c) A – (r), B – (p), C – (s), D – (q)
(d) A – (r), B – (p), C – (q), D – (s) Reason : Polymerisation process induces toxic character in
103. Match the columns organic molecules.
Column-I Column-II
(A) Polymer of styrene (p) used in making handles CRITICAL THINKING TYPE QUESTIONS
of utensils and computer
110. The polymer containing strong intermolecular forces e.g.
discs
hydrogen bonding, is
(B) Polymer of ethylene (q) used as an insulator
glycol and phtalic acid (a) teflon (b) nylon 6, 6
(C) Polymer of phenol (r) used in making paints (c) polystyrene (d) natural rubber
and formaldehyde and lacquers.
111. Among cellulose, poly (vinyl chloride), nylon and natural
(D) Polymer of vinyl (s) used in manufacture of
rubber, the polymer in which the intermolecular force of
chloride rain coats and flooring.
attraction is weakest is
(a) A – (q), B – (r), C– (p), D – (s)
(b) A – (r), B – (q), C– (p), D – (s) (a) nylon (b) poly (vinyl chloride)
(c) A – (q), B – (p), C– (r), D – (s) (c) cellulose (d) natural rubber
(d) A – (q), B – (r), C– (s), D – (p) 112. Which one of the following polymers is prepared by
condensation polymerisation?
ASSERTION-REASON TYPE QUESTIONS
(a) Teflon (b) Natural rubber
Directions : Each of these questions contain two statements,
(c) Styrene (d) Nylon-66
Assertion and Reason. Each of these questions also has four
alternative choices, only one of which is the correct answer. You 113. When condensation product of hexamethylenediamine and
have to select one of the codes (a), (b), (c) and (d) given below. adipic acid is heated to 525K in an atmosphere of nitrogen
(a) Assertion is correct, reason is correct; reason is a correct for about 4-5 hours, the product obtained is
explanation for assertion.
(a) solid polymer of nylon 66
(b) Assertion is correct, reason is correct; reason is not a
correct explanation for assertion (b) liquid polymer of nylon 66
(c) Assertion is correct, reason is incorrect (c) gaseous polymer of nylon 66
(d) Assertion is incorrect, reason is correct. (d) liquid polymer of nylon 6
EBD_7207
506 POLYMERS

114. Identify A, B and C in the following sequence of reactions O O


O O O (c) Nylon-66 –NH–(CH2)6–NH–C–(CH2)4–C–O–n
C6H5–C–O–O–C–C6H5 2 C6H5–C–O 2A
CH3
A + CH2 = CH2 B (d) PMMA – CH2–C
B + nCH2 = CH2 C COOCH3 n

C+C C6H5–(CH2– CH2)n—CH2– CH2 – CH2 118. In which of the following polymers ethylene glycol is one
of the monomer units?
–(CH2–CH2)n– C6H5
(a) (
—OCH 2–CH2OOC CO)n
(a) A = C 6H5, B = C6H5 – CH2– CH 2 and

C = C 6H5 –(CH2 – CH2)n—CH2 – CH 2 .

(b) A = C6H5 CO, B = C6H5 – CH2– CH 2 and (


(b) —CH 2–CH2 —
n (

C = C6H5 –(CH2–CH2)n—CH2– CH 2

(c) A = C 6H5, B = C6H5 – CH – CH3 and


(c) (
—CH2–CH=CH–CH2 –CH–CH2—n (
C = C6H5—(CH2 –CH2)n—CH2 CH 2
(
(d) —O–CH–CH 2–C–O–CH–CH2–C—n (
(d) A= C 6H5, B=C6H5 –CH2– CH 2 and CH3 O CH2CH3 O
C = C6H5 –(CH2– CH2)n—C6H5 119. Arrange the following in increasing order of their melting
115. In which of the following the formation of radical was found point. Nylon 2,2 (1); Nylon 2,4 (2), Nylon 2,6 (3), Nylon
to be more stable; the formation of radical of (acrylonitrile) 2,10(4)
(a) 1, 2, 3, 4 (b) 3, 4, 2, 1
or CH 2 CH 2 n (ethene).
(c) 2, 1, 3, 4 (d) 4, 3, 2, 1
(a) Both are equally stable (b) Acrylonitrile. 120. Which of the following rubber is not a polydiene?
(c) Ethene (d) Cannot say (a) Polyisoprene (b) Polychloroprene
116. Which one of the following monomers gives the polymer (c) Thiokol rubber (d) Nitrile rubber
neoprene on polymerization ? 121. Which of the following polymer is biodegradable?
(a) CF2 = CF2 (a) (
—CH 2–C=CH–CH2—n (
(b) CH2 = CHCl Cl
(c) CCl2 = CCl2 CN

Cl (b) —CH
( 2–CH=CH–CH2–CH 2–CH —n (
|
(d) CH 2 C — CH CH 2 (c) (
—O–CH–CH 2–C–O–CH–CH2–C—n (
CH3 O CH2CH3 O
117. Which of the following is not correctly matched?
O O H H O O

(a) Terylene –OCH2–CH2– C– –C – (


(d) —N–(CH2)6–N–C–(CH2)4–C—n (
n 122. The mass average molecular mass & number average
molecular mass of a polymer are 40,000 and 30,000
— CH 2 — C = CH — CH 2 — respectively. The polydispersity index of polymer will be
(b) Neoprene
(a) < 1 (b) > 1
Cl (c) 1 (d) 0
n
POLYMERS 507

FACT / DEFINITION TYPE QUESTIONS 26. (d) Silk is protein fibre. Dacron is polyester fibre and
Nylon-66 is polyamide fibre.
1. (d) 27. (d) All the given statements about fibres are correct.
2. (d) All these are natural polymers and exist in nature. 28. (d) Both polyesters and polyamides are examples of fibres.
3. (c) ATP is a monomer molecule. 29. (b) Polystyrene and polyethylene belong to the category
4. (a) It is present in the cell wall of plant cell. of thermoplastic polymers which are capable of
5. (b) Since proteins, cellulose and RNA control various repeatedly softening on heating and harden on cooling.
activities of plants and animals, they are called 30. (d) Vinyl chloride, butadiene and styrene being
biopolymers. unsaturated undergoes addition polymerization.
31. (b) 32. (a) 33. (c)
6. (d) Rayon is a manufactured from regenerated cellulosic
fiber. Rayon is produced from naturally occurring 34. (a) Ethene on free radical polymerisation gives low density
polymers and therefore it is not a truly synthetic fiber, polythene.
nor is it a natural fiber. It is known by the names viscose 35. (b) Monomer of teflon is to Tetrafluoro ethylene C2F4.
rayon and artificial silk in the textile industry. So, 36. (d) Acrylonitrile is the monomer used in the preparation of
option (d) is the correct choice. orlon.
7. (c) Protein is a natural polymer of amino acids. 37. (d) Orlon is a trade name of polyacrylonitrile
8. (b) 38. (b) High density polythene is used for manufacturing of
9. (b) Polymers are substances of high molecular weight buckets, dustbins, pipes etc.
(usually more than a few thousand) formed by the union 39. (b) High density polythene is formed when addition
of small molecular weight substances by covalent polymerisation of ethene takes place in a hydrocarbon
bonds. solvent in presence of catalyst such as ziegler-natta
10. (d) Cellulose acetate also known as rayon and cellulose catalyst.
nitrate are semisynthetic polymers. 40. (b) Nylon-66 is an example of first synthetic fibres produced
11. (a) 12. (a) from the simple molecules. It is prepared by
condensation polymer-isation of adipic acid and
13. (a) Polymerisation starts either by condensation or addition
haxamethylene diamine.
reactions between monomers. Condensation polymers
41. (a) Melamine plastic crockery is a copolymer of HCHO
are formed by the combination of monomers with the
and Melamine.
elimination of simple molecules.Whereas the addition
42. (d) 43. (a)
polymers are formed by the addition together of the
molecules of the monomer or monomers to form a large 44. (b) nHOOC(CH 2 )4 COOH nH 2 N(CH 2 )6 NH 2
molecule without elimination of any thing. adipic acid Hexamethylene
diamine
14. (d) 15. (a) 16. (d) 17. (c)
18. (b) Elastomers are the polymers having very weak O O
intermolecular forces of attraction between the polymer 525K
[– C – (CH 2)4 – C –NH–(CH2) 6 – NH –]n
chain. The weak forces permit the polymer to be Polymerisation Nylon 6, 6
streched.
45. (b)
19. (b) Those polymers in which process of heat softening
46. (d) The unbreakable plastic household crockery is made
and cooling can be repeated as many times as desired.
from copolymer of formaldehyde (HCHO) and melamine.
Example : polystyrene PVC, teflon, etc.
20. (c) Bakelite is a thermosetting polymer. On heating it is N
H2N NH2
infusible and cannot be remoulded.
21. (b) Polymerisation
HCHO Melamine
22. (d) These are characteristics of thermosets. N N formaldehyde polymer
23. (d) All these are characteristics of elastomers.
NH2
24. (d) Thermosetting polymers have strongest molecular Melamine
forces. These are crosslinked polymers.
25. (b) Thermosetting plastics have three dimensional cross 47. (d) Nylon tyre cord is made from high tenacity continuous
linked structure. filament yarn by twisting and plying.
EBD_7207
508 POLYMERS
48. (a) Polyesters are condensation polymers of a dibasic acid 53. (d) Nylon–6 can be manufactured from
and a diol. e.g., Terylene O

NH2OH

Ethylene glycol Terephthalic acid


O
NOH C
conc H2SO4
NH
(Beckmann
rearrangement) C
Terylene H
49. (d) Neoprene is an addition polymer of isoprene. H
caprolactum
Cl H2O,
|
nCH 2 CH C CH 2
O 2 or peroxides é Où
ê || úú -(n-1)H 2O
Chloroprene ê polymerization
–êë NH - (CH 2 )5 - C –úû ¬¾¾¾ ¾¾¾
Cl Nylon-6
| é + ù
ê H 3 N- (CH 2 )5 - COO- ú
— CH 2 C CH CH2 — ë û
Neoprene 54. (b) Nylon 6 is
50. (d) Phenol and formaldehyde undergo condensation O O
polymerisation under two different conditions to give —(NH(CH 2 )5 —C— NH— (CH2)5 —C— )n
a cross linked polymer called bakelite.
Hence (b) is the correct option.
OH OH OH 55. (b) Nylon is a polyamide fibre. It is prepared by the

CH2OH condensation polymerisation of adipic acid
OH (HOOC.(CH2)4COOH) and hexamethylene diamine
+ HCHO +
(H2N.(CH2)6.NH2).
CH2OH 56. (c) 57. (b)
o & p-hydroxymethylphenol 58. (a)
(intermediate) OH OH
OH OH CH2OH
OH OH
CH2OH
Polymerisation
+ HCHO + ;
n CH2
– (n – 1) H2 O

Linear Polymer (Novalac) CH2OH


59. (a) Melamine plastic crockery is a copolymer of HCHO
OH OH
and Melamine.
CH2OH
Polymerisation 60. (a) Bakelite is used for making phonograph records.
n + n
– H2O 61. (b) Novolac is a linear polymer.
62. (a) Dacron is a polyester and is the condensation polymer
CH2OH of ethylene glycol and terephthalic acid. It is crease
resistant
63. (c) Terylene is made from glycol and Terephthalic acid

HO CH2 CH2 OH and HOOC COOH


(Glycol)
(Terephthalic acid)

64. (d) 65. (a) 66. (d)


67. (d) Butadiene – styrene copolymer is used for the
manufacture of autotyres, floortiles, footwear
components, cable insulation etc.
68. (a) This is definition of homopolymer.
51. (b) Nylon 6, 6 has amide linkage capable of forming
69. (a) Butyl rubber is a copolymer of isobutylene and
hydrogen bonding.
isoprene.
52. (b) Nylon is a polyamide polymer.
POLYMERS 509
70. (d) Neoprene is a homopolymer of 2-chloro-buta-1, 3-diene STATEMENT TYPE QUESTIONS
or chloroprene.
71. (b) It is the definition of copolymerisation. 91. (a) Monomeric units in polymers are joined together by
covalent bonds only.
72. (b) n CH2=CH–C=CH2 Polymerisation 92. (b) Condensation polymers are formed by repeated
condensation reaction between two different bi-
CH3 functional or tri–functional monomeric units.
Isoprene
–CH2–CH=C–CH2 – Buna–S and Buna–N being elastomeric consists of
n polymeric chains held together by weak intermolecular
CH3
Polyisoprene forces thus they are elastic in nature.
(Natural rubber) 93. (d) Number of hydrogen bonds is greater in polymer (1)
than in (2) as the density of amide bond is greater in (1)
73. (b)
therefore the chain links to each other strongly in (1)
CH3 H CH2 CH2 CH3 H than in (2) hence (1) melts at higher temperature.
C=C C=C C=C
– CH 2 CH2 CH 3 H CH 2
94. (d) Nylon–6 is used in the manufacturing of tyre cords
CH2–
Natural rubber not nylon–6,6 this is used in making sheets, bristles
Natural rubber
for brushes and in textile industry. Melamine –
(All cis configuration) formaldehyde polymer is used in the manufacture of
All statements except (b) are correct unbreakable crockery.
74. (b) Natural rubber is a linear 1, 4-addition polymer of 95. (b) Vulcanisation of natural rubber with sulphur and an
isoprene (2-methyl-1, 3-butadiene). appropriate additive is carried out within temperature
range of 373K to 415K. Synthetic rubbers are either
75. (d) 76. (b) 77. (b)
homopolymersof 1, 3– butadiene derivatives or
78. (c) Rubber is a polymer of isoprene. Its chemical formula is copolymers of 1, 3–butadiene or its derivatives with
(C5H8)n. another unsaturated monomer.
79. (a) Neoprene is a synthetic polymer that resembles natural 96. (c) Biodegradable polymers are manufactured because
rubber. Neoprene is a polymer of chloroprene which conventional polymers are quite resistant to the
polymerises 700 times faster than the isoprene environmental degradation which leads to accumulation
(monomer of natural rubber) and no specific catalyst is of polymeric solid waste materials causing acute
environmental problems.
needed for this purpose.

CH 2 C CH CH 2 MATCHING TYPE QUESTIONS


| 97. (b) A – (q), B – (s), C – (r), D – (p)
Cl
Chloroprene
98. (c) A – (s), B – (r), C – (p), D – (q)
(2 chloro 1,3 butadiene) 99. (b) A – (t), B – (p), C – (r), D – (q)
100. (b) A – (q), B – (r), C – (p), D – (t), E – (s)
101. (a) A – (r), B – (p), C – (q), D – (t), E – (s)
( CH 2 C CH CH 2 C CH CH 2 ) n
| | 102. (c) A – (r), B – (p), C – (s), D – (q)
Cl Cl 103. (a)
Neoprene (A) Polystyrene is used as insulator.
(B) Glyptal a polymer of ethylene glycol and phthali acid
80. (d) Synthetic rubber (neoprene) is a polymer of is used in manufacture of paints and lacquers.
Cl (C) Bakelite, a polymer of phenol & formal dehyde is used
|
for making electrical switches, handles of utensils and
CH 2 CH C CH 2 or chloroprene.
computer disc’s.
81. (a) (D) PVC, a polymer of vinyl chloride is used in manufacture
82. (b) Buna – N is a copolymer of butadiene of raincoat and flooring.
(CH2=CH–CH=CH2) and acrylonitrile (CH2 = CHCN).
83. (a) Neoprene is a polymer of chloroprene ASSERTION-REASON TYPE QUESTIONS
(2 – chloro – 1, 3 – butadiene). 104. (a)
84. (c) Neoprene is a homopolymer of chloroprene. 105. (b) Due to the presence of strong C–F bonds, teflon has
85. (b) 5% of sulphur is used as a crosslinking agent in the high thermal stability and chemical inertness.
manufacture of tyre rubber. 106. (c) Bakelite can be heated only once.
86. (b) Buna – N is obtained by copolymerisation of 107. (b) Vulcanisation is a process of treating natural rubber
with sulphur or some compounds of sulphur under
1, 3–butadiene and acrylonitrile.
heat so as to modify its properties. This cross-linking
87. (d) 88. (c) 89. (d) 90. (b) give mechanical strength to the rubber.
EBD_7207
510 POLYMERS
108. (d) The time of vulcanisation is reduced by adding 115. (b) Considering the resonance structure, the radical of
accelerators and activators. acrylonitrile is found to be more stable.
109. (d)
H
H
CRITICAL THINKING TYPE QUESTIONS CH = CH – C N C=C H
H
110. (b) Nylon 6, 6 has amide linkage capable of forming + C 6H 5
hydrogen bonding. + C6H5
111. (d) Nylon and cellulose, both have intermolecular
hydrogen bonding, polyvinyl chloride has dipole-
dipole interactions, while natural rubber has van der
Waal forces which are weakest.
112. (d) Copolymer of adipic acid (6C) and hexamethylene C6H5 – CH – CH – C N C 6H5 – CH 2 –CH 2
diamine (6C).
n HOOC(CH 2 ) 4 COOH nH 2 N (CH 2 ) 6 NH 2
Adipic acid Hexamethyl ene diamine

O O
|| || C6H5 – CH – CH C N
( C (CH 2 ) 4 C NH (CH 2 ) 6 NH ) n
Nylon 66 (Resonance stabilised)
It has high tenacity and elasticity. It is resistant to
abrasion and not affected by sea water. It is used for Cl
reinforcement of rubber tyres, manufacture of |
O 2 or peroxides
parachute, safety belts, carpets and fabrics. 116. (d) nCH 2 CH C CH 2
113. (b) The condensation polymerisation of hexamethylene Chloroprene
diamine and adipic acid is done in solution form by
interface technique. In this liquid nylon polymer is Cl
|
obtained. ( CH 2 CH C CH 2 )n
n.H 2 N (CH 2 )6 NH 2 + Neoprene

Polymerisation 117. (a) Terylene is prepared by condensing terephthalic acid


nHOOC – (CH 2 ) 4 – COOH nH 2O and ethylene glycol
[ HN CH 2 NHCO CH 2 CO ]n
6 Nylon 4 nHOOC COOH + nHOCH 2CH 2OH
Terephthalic acid Ethylene glycol
O O O
|| || ||
114. (a) C6 H5 C O O C C6 H5 2C6 H5 C O O O
|| ||
C C – OCH2–CH2–O n
2C6 H5
Terylene
(A)
Phenyl radical
118. (a)
C6 H5 CH2 CH 2 C6 H5 CH2 CH 2 CH 2 C H 2 119. (d) As the amide density along the chain increases the
(A) (B) melting point increases.
120. (c) Thiokol is polymer of CH2ClCH2Cl and sodium
C6 H 5 CH 2 CH 2 CH 2 C H 2 nCH 2 CH 2 polysulphide Na–S–S–Na and thus, not polydiene
(B) rubber.
121. (d)
C6 H 5 (CH 2 CH 2 ) n CH 2 C H 2 .
(C) 122. (b) Average number molecular weight M n 30,000

C6 H 5— (CH 2 CH 2 )n—CH 2 C H 2 C6 H—(CH Average mass molecular weight M w 40,000


5 2 CH 2 ) n—CH 2 C H 2

C6 H5 (CH 2— CH 2 )n—CH 2 CH 2 CH 2 Mw 40,000


Polydispersity index (PDI) = 1.33
Mn 30,000
(CH 2 CH 2 )—C
n 6 H5
30
CHEMISTRY IN
EVERYDAY LIFE
FACT / DEFINITION TYPE QUESTIONS 8. Which one of the following is employed as a tranquilizer
drug?
1. The use of chemicals for treatment of diseases is called as (a) Promethazine (b) Valium
(a) isothermotherapy (b) angiotherapy (c) Naproxen (d) Mifepristone
(c) physiotherapy (d) chemotherapy 9. Terfenadine is commonly used as a/an
2. Which of the following statements is not true aboult enzyme (a) tranquilizer (b) antihistamine
inhibitors? (c) antimicrobial (d) antibiotic
(a) Inhibit the catalytic activity of the enzyme. 10. Which one of the following is not a tranquilizer?
(b) Prevent the binding of substrate. (a) Equanil (b) Veronal
(c) Generally a strong covalent bond is formed between (c) Salvarsan (d) Serotonin
an inhibitor and an enzyme 11. Tranquillizers are substances used for the treatment of
(d) Inhibitors can be competitive or non-competitive. (a) cancer (b) AIDS
3. Which of the following is not a target molecule for drug (c) mental diseases (d) physical disorders
function in body? 12. Which one of the following is employed as a tranquilizer
(a) Carbohydrates (b) Lipids drug?
(c) Vitamins (d) Proteins (a) Promethazine (b) Valium
4. Which of the following comounds are administered as (c) Naproxen (d) Mifepristone
antacids? 13. Which of the following drugs is a tranquilizer and sedative
(i) Sodium carbonate (a) Sulphadiazine (b) Papaverine
(ii) Sodium hydrogencarbonate (c) Equanil (d) Mescaline
(iii) Aluminium carbonate 14. Drug which helps to reduce anxiety and brings about
(iv) Magnesium hydroxide calmness is
(a) (i) and (ii) (b) (ii) and (iv) (a) tranquillizer (b) diuretic
(c) (i), (ii) and (iii) (d) All of these (c) analgesic (d) antihistamine
5. The drug H is used as 15. The drug used as an antidepressant is
N (a) Luminol (b) Tofranil
(c) Mescaline (d) Sulphadiazine
16. Barbituric acid and its derivatives are well known
N CH2 (a) antipyretics (b) analgesics
(c) antiseptics (d) traquillizers
CH2 NH2
17. Which of the following is a hypnotic drug?
(a) Antacid (b) Analgesic (a) luminal (b) salol
(c) Antimicrobial (d) Antiseptic (c) catechol (d) chemisol
6. The function of enzymes in the living system is to 18. Which of the following is used for inducing sleep?
(a) transport oxygen (a) Paracetamol
(b) provide energy (b) Chloroquine
(c) provide immunity (c) Bithional
(d) catalyse biochemical reactions (d) Barbituric acid derivatives
7. Which one of the following is employed as a tranquilizer? 19. Aspirin is
(a) Naproxen (b) Tetracycline (a) antibiotic (b) antipyretic
(c) Chlorpheninamine (d) Equanil (c) sedative (d) psychedelic
EBD_7207
512 CHEMISTRY IN EVERYDAY LIFE
20. An antipyretic is 33. Sulpha drugs are used for
(a) quinine (b) paracetamol (a) precipitating bacteria
(c) luminal (d) piperazine (b) removing bacteria
21. The following compound is used as (c) decreasing the size of bacteria
O (d) stopping the growth of bacteria
O – C – CH3 34. Streptomycin is effective in the treatment of
(a) tuberculosis (b) malaria
(c) typhoid (d) cholera
COOH
35. An antibiotic with a broad spectrum
(a) an anti-inflammatory compound
(a) kills the antibodies
(b) analgesic
(b) acts on a specific antigen
(c) hypnotic
(d) antiseptic (c) acts on different antigens
22. Barbituric acid and its derivatives are well known (d) acts on both the antigens and antibodies
(a) antipyretics (b) analgesics 36. Which of the following is not an antiseptic drug?
(c) antiseptics (d) traquillizers (a) Iodoform (b) Dettol
23. The drug used for prevention of heart attacks is (c) Gammexane (d) Genation violet
(a) aspirin (b) valium 37. Penicillin was first discovered by
(c) chloramphenicol (d) cephalsoprin (a) A. Fleming (b) Tence and Salke
24. Sulpha drugs are used for (c) S.A. Waksna (d) Lewis Pasteur
(a) precipitating bacteria 38. Veronal, a barbiturate drug is used as
(b) removing bacteria (a) anaesthetic (b) sedative
(c) decreasing the size of bacteria (c) antiseptic (d) None of these
(d) stopping the growth of bacteria 39. A drug effective in the treatment of pneumonia, bronchitis,
25. Aspirin falls under which class of drugs ? etc, is
(a) Analgesic (b) Antibiotic (a) streptomycin (b) chloramphenicol
(c) Antifertility (d) antacid
(c) penicillin (d) sulphaguanidine
26. Which of the following term means pain killer
40. Commonly used antiseptic 'Dettol' is a mixture of
(a) Antibiotic (b) Analgesic
(a) o-chlorophenozylenol + terpeneol
(c) Antipyretic (d) Penicillin
27. Which one of the following can possibly be used as analgesic (b) o-cresol + terpeneol
without causing addiction and mood modification ? (c) phenol + terpeneol
(a) Diazepam (d) chloroxylenol + terpeneol
(b) Morphine 41. Chloroamphenicol is an :
(c) N-Acetyl-para-aminophenol (a) antifertility drug
(d) Tetrahydrocannabinol (b) antihistaminic
28. Aspirin is known as (c) antiseptic and disinfectant
(a) acetyl salicylic acid (b) phenyl salicylate (d) antibiotic-broad spectrum
(c) acetyl salicylate (d) methyl salicylic acid 42. The drug which is effective in curing malaria is
29. Which one among the following is not an analgesic? (a) quinine (b) aspirin
(a) Ibuprofen (b) Naproxen (c) analgin (d) equanil
(c) Aspirin (d) Valium
43. An antibiotic contains nitro group attached to aromatic
30. Which of the following statements about aspirin is not true? nucleus. It is
(a) It is effective in relieving pain.
(a) penicillin (b) streptomycin
(b) It is a neurologically active drug.
(c) tetracycline (d) chloramphenicol
(c) It has antiblood clotting action.
44. The structure given below is known as
(d) It belongs to narcotic analgesics.
31. Salol can be used as O
H H S CH3
(a) antiseptic (b) antipyretic
CH 2 C NH
(c) analgesic (d) None of these CH3
32. Various phenol derivatives, tincture of iodine (2 – 3%) I2 N COOH
in (water / alcohol) and some dyes like methylene blue are O H
(a) antiseptics (b) disinfectants (a) Penicillin F (b) Penicillin G
(c) analgesics (d) antipyretics (c) Penicillin K (d) Ampicillin
CHEMISTRY IN EVERYDAY LIFE 513
45. Arsenic drugs are mainly used in the treatment of 57. Which of the following method of classification of drugs is
(a) Jaundice (b) Typhoid useful for medicinal chemists?
(c) Syphilis (d) Cholera (a) On the basis of molecular targets.
46. Bithional is an example of (b) On the basis of chemical structure.
(a) disinfectant (b) antiseptic (c) On the basis of drug action.
(c) antibiotic (d) analgesic (d) All of these.
47. Penicillin is an :
58. Which of the following statements is true about the catalytic
(a) antibiotic (b) anaesthetic activity of enzyme?
(c) antiseptic (d) antipyretic
(a) Enzyme holds the substrate for a biochemical reaction.
48. Which of the following is a broad spectrum drug?
(b) En zyme binds substrate through a variety of
(a) Plasmoquine (b) Chloroquine
interactions such as ionic bonding, hydrogen bonding,
(c) Chloramphenicol (d) D.D.T.
van der Waal’s interaction or dipole – dipole
49. Bithional is added to soap as an additive to function as a/an
interaction.
(a) softener (b) hardener
(c) Enzyme provides functional group that will attack the
(c) dryer (d) antiseptic
substrate and carry out biochemical reaction.
50. Antiseptics and disinfectants either kill or prevent growth
of microorganisms. Identify which of the following (d) All of the above.
statements is not true: 59. Drug tegamet is used as
(a) Chlorine and iodine are used as strong disinfectants. (a) Antacid (b) Antimalarial
(b) Dilute solutions of boric acid and hydrogen Peroxide (c) Analgesic (d) Antiseptic
are strong antiseptics. 60. Which of the following is an essential component of sleeping
(c) Disinfectants harm the living tissues. pills?
(d) A 0.2% solution of phenol is an antiseptic while 1% (a) Analgesics (b) Tranquilizers
solution acts as a disinfectant. (c) Antihistamines (d) Both (b) and (c)
51. Arsenic containing medicine used for the treatment of 61. Which type of drugs inhibit the enzymes which catalyse
syphilis, is the degradation of noradrenaline?
(a) erythromycin (b) ofloxacin
(a) Narcotic analgesics (b) Antacids
(c) tetracycline (d) salvarsan
(c) Antidepressant (d) Non–narcotic analgesic.
52. Novestrol is an
(a) antibiotic (b) analgesic 62. Which of the following is/are example(s) of narcotic
(c) antacid (d) antifertility drug analgesics?
53. Which is the correct statement about birth control pills? (a) Morphine (b) Heroin
(a) Contain estrogen only (c) Codeine (d) All of these
(b) Contain progesterone only 63. Which of the following was the first effective treatment
(c) Contain a mixture of estrogen and progesterone discovered for syphilis?
derivatives. (a) Penicillin (b) Arsphenamine
(d) Progesterone enhances ovulation. (c) Chloramphenicol (d) Sulphanilamide
54. Compounds with antiseptic properties are ____________ 64. Which of the following is an example of narrow spectrum
(i) CHCl3 antibiotic?
(ii) CHI3 (a) Chloramphenicol (b) Penicillin G
(iii) Boric acid (c) Ampicillin (d) Ofloxacin
(iv) 0.3 ppm aqueous solution of Cl2 65. Antibiotic that can be given orally in case of typhoid, acute
(a) (i) and (ii) (b) (ii) and (iii) fever, dysentery, meningitis and pneumonia is_______.
(c) (i) and (iv) (d) (i) and (iii)
(a) vancomycin (b) salvarsan
55. Which of the following is not a function of aspirin?
(c) ofloxacin (d) chloramphenicol
(a) Relief from arthritic pain
(b) Relief from postoperative pain. 66. Veronal and luminal are derivatives of barbituric acid which
(c) Prevents platelet coagulation. are______.
(d) Prevention of heart attacks. (i) Tranquilizers
56. Which of the following method of classification of drugs is (ii) Non– narcotic analgesic.
useful for doctors? (iii) Antiallergic drug
(a) On the basis of drug action. (iv) Neurologically active drug.
(b) On the basis of chemical structure. (a) (i) and (iv) (b) (i) and (iii)
(c) On the basis of molecular targets. (c) (ii) and (iii) (d) (i) only
(d) On the basis of pharmacological effect.
EBD_7207
514 CHEMISTRY IN EVERYDAY LIFE
67. Which is correct about saccharin? 81. Washing soap can be prepared by saponification with alkali
O of which of the following oil
(a) Rose oil (b) Paraffin oil
C
(a) It is NH (c) Groundnut oil (d) Kerosene oil
SO2 82. Palmitic acid and stearic acid are used as :
(b) It is 600 times sweeter than sugar (a) medicine (b) soap
(c) It is used as sweetening agent (c) antiseptic cream (d) pickle
(d) All of these 83. Soaps can be classified as :
68. Which of the following acts as an antioxidant in edible oils (a) esters (b) salts of fatty acids
(a) Vitamin B (b) Vitamin C (c) alcohols (d) phenols
(c) Vitamin D (d) Vitamin E 84. Cetyltrimethyl ammonium bromide is a popular
69. Salts of sorbic acid and propionic acid are used as (a) anionic detergent (b) cationic detergent
(a) antioxidants (b) flavouring agents (c) non-ionic detergent (d) sweetener
(c) food preservatives (d) nutritional supplements 85. The cationic detergent that is used in hair conditioners is
70. Which of the following add nutritive value to food? (a) sodium dodecylbenzene sulphonate
(a) Sweeteners (b) Antioxidants (b) sodium lauryl sulphate
(c) Fat emulsifiers (d) None of these (c) tetramethyl ammonium chloride
71. Arrange the following artificial sweeteners in increasing (d) cetyltrimethyl ammonium bromide
order of their sweetness value? 86. Which of the following enhances leathering property of
(a) Sucralose < Saccharin < Alitame < Aspartame soap?
(b) Aspartame < Saccharin < Sucralose < Alitame (a) Sodium carbonate (b) Sodium rosinate
(c) Aspartame < Sucralose < Saccharine < Alitame (c) Sodium stearate (d) Trisodium phosphate
(d) Saccharine < Aspartame < Sucralose < Alitame 87. Polyethyleneglycols are used in the preparation of which
72. Which of the following artificial sweetener does not provide type of detergents?
calories? (a) Cationic detergents (b) Anionic detergents
(a) Alitame (b) Aspartame (c) Non-ionic detergents (d) Soaps
(c) Sucralose (c) Both (b) and (c) 88. Which of the following are anionic detergents?
73. Sodium benzoate is used as (i) Sodium salts of sulphonated long chain alcohol.
(a) food preservative (b) analgesic (ii) Ester of stearic acid and polyethylene glycol.
(c) filler in detergents (d) antiseptic (iii) Quarternary ammonium salt of amine with acetate ion.
74. Structuraly biodegradable detergent should contain (iv) Sodium salts of sulphonated long chain hydrocarbons.
(a) normal alkyl chain (b) branched alkyl chain (a) (i) and (iv) (b) (ii) and (iii)
(c) phenyl side chain (d) cyclohexyl side chain (c) (iii) and (iv) (d) (i), (ii) and (iv)
75. Detergents are prepared by the action of H2SO4 on which 89. Which type of detergent is formed when stearic acid reacts
of the following? with polyethylene glycol?
(a) Cholesterol (b) Lauryl alcohol (a) Cationic detergent (b) Anionic detergent
(c) Cyclohexanol (d) p-Nitrophenol (c) Non– ionic detergent (d) None of these
76. Sodium alkyl benzene sulphonate is used as 90. Glycerol is added to soap. It functions ___________
(a) soap (b) fertilizers (a) as a filler.
(c) pesticides (d) detergents (b) to increase leathering.
77. Which of the following represents a synthetic detergent? (c) to prevent rapid drying.
(a) C15H31COOK (d) to make soap granules.
(b) CH3[CH2]16COONa
STATEMENT TYPE QUESTIONS
(c) C12H25 SO3Na 91. Which of the following statements are incorrect about
receptor proteins?
(d) None of these (i) Majority of receptor proteins are embedded in the cell
78. Which of the following represents soap membranes.
(a) C17H35COOK (b) C17H35COOH (ii) The active site of receptor proteins opens on the inside
(c) C15H31COOH (d) (C17H35COO)2Ca region of the cell.
79. Alkaline hydrolysis of esters is known as : (iii) Chemical messengers are received at the binding sites
(a) Esterification (b) Saponification of receptor proteins.
(c) dehydration (d) alkalination (iv) Shape of receptor doesn’t change during attachment
80. Commercial detergent contains mainly of messenger.
(a) RCOONa (b) RONa (a) (i), (ii) and (iii) (b) (ii) and (iv)
(c) RSNa (d) RSO3Na (c) (ii), (iii) and (iv) (d) (i) and (iv)
CHEMISTRY IN EVERYDAY LIFE 515
92. Which of the following statements are incorrect about Which of the following is the correct code for the statements
penicillin? above ?
(i) An antibacterial medicine. (a) TFTT (b) FTTT
(ii) Ampicillin is its synthetic modification. (c) FTFT (d) TTTT
(iii) It has bacteriostatic effect. 97. Which of the following statements are correct?
(iv) It is a broad spectrum antibiotic. (i) Detergents give foam easily even in hard water.
(a) (i) and (ii) (b) (ii) and (iv) (ii) Anionic detergents are used in toothpastes.
(c) (iii) and (iv) (d) (i) and (ii) (iii) Cationic detergents being inexpensive are extensively
93. Which of the following statements are correct? used as germicide.
(i) Before 1970 for treatment of stomach acidity Al(OH)3 (iv) Detergents with linear alkyl chains are more polluting
is a better antacid in comparison to NaHCO3 . as compared to detergents having branched alkyl
(ii) Discovery of cimetidine was a major break through in chains.
the treatment of hyperacidity. Which of the following is the correct code for the statements
(iii) Terfenadine is a drug which competes with histamine above?
for binding sites of receptor. (a) TFTF (b) TFFT
(iv) Antidepressant drugs like equanil inhibit the enzymes (c) FFFT (d) TTFF
which catalyze the degradation of noradernaline.
(v) Veronal and luminal belongs to the class of tranquilizers MATCHING TYPE QUESTIONS
called barbiturates.
(a) (i), (ii) and (iii) (b) (i), (iii) and (v) 98. Match the columns
(c) (ii), (iii) and (iv) (d) (ii), (iv) and (v) Column-I Column-II
94. Consider the following statements. (A) Sodium Perborate (p) Disinfectant
(i) Antiseptics are not ingested like antibiotics. (B) Chlorine (q) Antiseptic
(ii) 1% solution of iodine in alcohol–water mixture is known (C) Bithional (r) Milk bleaching agent
as tincture of iodine. (D) Potassium stearate (s) Soap
(iii) SO2 in low concentrations are used as antiseptics (a) A – (p), B – (q), C – (r), D – (s)
whereas in higher concentration are used as (b) A – (q), B – (r), C – (s), D – (p)
disinfectants.
(c) A – (r), B – (p), C – (q), D – (s)
(iv) Birth control pills essentially contain a mixture of
synthetic estrogen and progesterone derivates. (d) A – (s), B – (p), C – (q), D – (r)
Which of the following is the correct code for the statements 99. Match the columns
above ? Column-I Column-II
(a) FTFT (b) TTTF (A) Ranitidine (p) Tranquilizer
(c) TFFT (d) FFTT (B) Furacine (q) Antibiotic
95. Which of the following statements are correct? (C) Phenelzine (r) Antihistamine
(i) Cationic detergents have germicidal properties (D) Chloramphenicol (s) Antiseptic
(ii) Bacteria can degrade the detergents containing highly (a) A – (r), B – (s), C – (p), D – (q)
branched chains. (b) A – (s), B – (p), C – (q), D – (r)
(iii) Some synthetic detergents can give foam even in ice (c) A – (p), B – (q), C – (r), D – (s)
cold water.
(d) A – (q), B – (r), C – (s), D – (p)
(iv) Synthetic detergents are not soaps.
100. Match the columns
(a) (i), (ii) and (iii) (b) (i), (iii) and (iv)
Column – I Column – II
(c) (ii), (iii) and (iv) (d) (iii) and (iv)
(A) First antibacterial (p) Broad spectrum
96. Consider the following statements.
drug antibiotic
(i) Potassium soaps are soft to the skin than sodium
(B) Protosil (q) Arsphenamine
soaps.
(ii) Shaving soaps contain ethanol to prevent rapid drying. (C) Chloramphenicol (r) 1932
(iii) Builders like Na2CO3 and Na3PO4 make soaps act more (D) Ofloxacin (s) 1947
rapidly. (a) A – (q), B – (r), C – (p, s), D – (p)
(iv) Hard water contains Ca2+ and Mg2+ ions which forms (b) A – (r), B – (q), C – (p, s), D – (p)
insoluble Ca2+ and Mg 2+ soaps separates out as (c) A – (q), B – (p, s), C – (r), D – (p)
scum. (d) A – (p), B – (r), C – (p, s), D – (q)
EBD_7207
516 CHEMISTRY IN EVERYDAY LIFE
101. Match the columns 105. Assertion : Tetracycline is a broad spectrum antibiotic.
Column-I Column-II Reason : Tetracyclin is effective against a number of types
+ of bacteria, large viruses and typhus fever.
CH3 106. Assertion : Antiseptics are applied to living tissues.

(A) CH3(CH2)15–N–CH3 Br (p) Dishwashing Reason : Iodine is a powerful antiseptic.
107. Assertion : Sedatives are given to patients who are mentally
CH3 powder agitated and violent.
Reason : Sedatives are used to suppress the activities of
central nervous system.
– +
(B) CH3– (CH2)11 SO3 Na (q) Laundry soap 108. Assertion : Non-competitive inhibitor inhibits the catalyic
activity of enzyme by binding with its active site.
(C) C17H35COO–Na+ + Na2CO3 + Rosin (r) Hair Reason : Non-competitive inhibitor changes the shape of
conditioners the active site in such a way that substrate can’t recognise
(D) CH3(CH2)16COO(CH2CH2O)nCH2 CH2OH (s) Toothpaste it.
(a) A – (p), B – (q), C – (r), D – (s) 109. Assertion : Sodium chloride is added to precipitate soap
(b) A – (q), B – (r), C – (p), D – (s) after saponification.
(c) A – (r), B – (s), C – (q), D – (p) Reason : Hydrolysis of esters of long chain fatty acids by
(d) A – (p), B – (r), C – (q), D – (s) alkali produces soap in colloidal form.
102. Match the columns
Column -I Column -II CRITICAL THINKING TYPE QUESTIONS
(A) Toilet soap (p) Made by beating
110. Which of the following are sulpha drugs?
tiny air bubbles before
(i) Sulphapyridine (ii) Prontosil
their hardening.
(iii) Salvarsan (iv) Nardil
(B) Transparent soap (q) Contain glycerol
(a) (i) and (ii) (b) (ii) and (iv)
to prevent rapid drying.
(c) (i), (ii) and (iv) (d) (ii), (iii) and (iv)
(C) Shaving soaps (r) Prepared by
using better grades of 111. Among the following antihistamines, which are antacids
fats and oils. (i) Ranitidine (ii) Brompheniramine
(D) Soaps that float in (s) Made by dissolving (iii) Terfenadine (iv) Cimetidine
water the soap in ethanol (a) (i) and (iii) (b) (i), (ii) and (iv)
and then evaporating (c) (i) and (iv) (d) (ii) and (iii)
excess alkali. 112. Which one of the following is an antihistamine?
(a) A – (s), B – (p), C – (q), D – (r) (a) Iproniazid (b) Salvarsan
(b) A – (r), B – (s), C – (q), D – (p) (c) Zantac (d) Chloramphenicol
(c) A – (r), B – (q), C – (p), D – (s) 113. Morphine is
(d) A – (q), B – (s), C – (p), D – (r) (a) an alkaloid (b) an enzyme
(c) a carbohydrate (d) a protein
ASSERTION-REASON TYPE QUESTIONS 114. H1 – Receptor antagonists is a term associated with :
(a) Antiseptics (b) Antihistamins
Directions : Each of these questions contain two statements, (c) Antacids (d) Analgesics
Assertion and Reason. Each of these questions also has four
115. Amoxillin is semi-synthetic modification of
alternative choices, only one of which is the correct answer. You
(a) penicillin (b) streptomycin
have to select one of the codes (a), (b), (c) and (d) given below.
(c) tetracycline (d) chloroampheniol
(a) Assertion is correct, reason is correct; reason is a correct
116. Which of the following is used as an antibiotic ?
explanation for assertion.
(a) Ciprofloxacin (b) Paracetamol
(b) Assertion is correct, reason is correct; reason is not a
correct explanation for assertion (c) Ibuprofen (d) Tocopherol
(c) Assertion is correct, reason is incorrect 117. Select the incorrect statement.
(d) Assertion is incorrect, reason is correct. (a) Equanil is used to control depression and hypertension.
103. Assertion : The drugs which act on the central nervous (b) Mifepristone is a synthetic steroid used as “morning
system and help in reducing anxiety are called antibiotics. after pill”.
Reason : Pencillin is an antibiotic. (c) 0.2 percent solution of phenol is an antiseptic while its
1.0 percent solution is a disinfectant.
104. Assertion : Equanil is a tranquilizer.
(d) A drug which kills the organism in the body is called
Reason : Equan il is used to cure depression and
bacteriostatic.
hypertension.
CHEMISTRY IN EVERYDAY LIFE 517
118. A large number of antibiotics have been isolated from (c) X = Narrow spectrum antibiotics.
(a) Bacteria actinomycetes Y = Limited spectrum antibiotics.
(b) Acids (d) X = Narrow spectrum antibiotics.
(c) Alkanals Y = Broad spectrum antibiotics.
(d) Bacteria rhizobium 125. Which of the following is an example of synthetic
119. Antiseptic chloroxylenol is progesterone derivative which is most widely used as
(a) 4-chloro-3, 5-dimethylphenol antifertility drug?
(b) 3-chloro-4, 5-dimethylphenol (a) Norethindrone (b) Novestrol
(c) 4-chloro-2, 5-dimethylphenol (c) Ethynylestradiol (d) All of these
(d) 5-chloro-3, 4-dimethylphenol 126. Substance used for the preservation of coloured fruit juices
120. Which of the following is not correctly matched? is
(i) Proteins that are – Receptors (a) benzene (b) benzoic acid
crucial to body’s communication (c) phenol (d) sodium meta bisulphite
process. 127. The artificial sweetener containing chlorine that has the
(ii) Drugs that mimic – Antagonists appearance and taste as that of sugar and is stable at cooking
the natural messenger by temperature is
switching on the receptor. (a) Aspartame (b) Saccharin
(iii) Drugs that binds to – Agonists (c) Sucrolose (d) Alitame
the receptor site and inhibit 128. Benzalkonium chloride is a
its natural function. (a) cationic surfactant and antiseptic
(a) (ii) only (b) (iii) only (b) anionic surfactant and soluble in most of organic
(c) (i) and (iii) (d) (ii) and (iii) solvents
121. Which of the following drug inhibits the synthesis of (c) cationic surfactant and insoluble in most of organic
chemicals known as prostaglandins which stimulate solvents
inflammation in tissue and cause pain? (d) cationic surfactant and antimalarial
(a) Barbiturates (b) Aspirin 129. Which one of the following is not used as a filler in laundry
(c) Seldane (d) Iproniazid soaps?
122. Bactericidal antibiotics are those which (a) Sodium silicate (b) Glycerol
(a) have inhibitory effect on microbes. (c) Sodium rosinate (d) Borax
(b) have killing effect on microbes. 130. Which of the following is an example of liquid dishwashing
(c) have both inhibitory and killing effect on microbes. detergent?
(d) intervene in metabolic process of microorganism. (a) CH3(CH2)10–CH2OSO3–Na+
123. Which of the following antibiotics is not correctly classified?
Bactericidal Bacteriostatic (b) C9H19 — (
—O—CH )5 –CH2CH2OH
2–CH2–O—

(A) Penicillin Erythromycin


(B) Aminoglycosides Tetracycline – +
(C) Chloramphenicol Ofloxacin (c) CH3— —SO3 Na
(a) A and B (b) C only
(c) B and C (d) B only +
CH3
124. Antibiotics that are effective mainly against Gram-positive –
or Gram-negative bacteria X. Antibiotics that are effective (d) CH3(CH2)15–N–CH3 Br
against a single organism or disease are Y CH3
What is X and Y ?
(a) X = Broad spectrum antibiotics. 131. Which of the following statements is incorrect?
Y = Narrow spectrum antibiotics. (a) Saccharin is about 550 times as sweet as cane sugar.
(b) X = Broad spectrum antibiotics. (b) Aspartame is used in the manufacture of baked sweets.
Y = Limited spectrum antibiotics. (c) Alitame is more sweet than saccharin and aspartame.
(d) Sodium benzoate is commonly used preservative.
EBD_7207
518 CHEMISTRY IN EVERYDAY LIFE

FACT / DEFINITION TYPE QUESTIONS O–COCH3

1. (d) 2. (c) 3. (c) 4. (b) COOH


28. (a) Aspirin (Acetyl salicylic acid)
5. (a) Given drug is used as Antacid.
6. (d) The function of enzymes in the living system is to
catalyse biochemical reactions which occure in living 29. (d) Valium is a tranquilizer and not an analgesic. It is used
systems. e.g. invertase, pepsin, amylase. for treatment of stress, fatigue, mild and severe mental
diseases.
Invertase
Sucrose glucose + fructose 30. (d) Aspirin is an non-narcotics analgesic.
(polymer) (monomer) 31. (a) Salol is phenyl salicylate used as antiseptic.
amylase
32. (a) Antiseptic drugs cause destruction of micro-organism
Starch glucose that produce septic disease e.g. Dettol, Savlon, Boric
(polymer) (monomer) acid, Phenol, Iodoform, KMnO4 and some dye such
7. (d) Equanil is an important medicine used in depression as methylene blue, genation violet.
and hypertension. 33. (d) Sulpha drugs (antibacterial and antibiotic) are group
8. (b) of drugs which are derivative of sulphanilamide.
9. (b) Terfenadine is commonly used as antihistamine. 34. (a) It is the very effective antibiotic for tuberculosis.
10. (c) Salvarsan is an organoarsenic compound, used in the 35. (c) Broad spectrum antibiotics act on different antigens.
treatment of syphilis. It was the first modern 36. (c) It is an insecticide.
chemotherapeutic agent. 37. (a) A. Fleming discovered penicillin in 1929.
11. (c) 12. (b) 38. (b)
13. (c) Tranquilizers reduce anxiety and tension they are also 39. (c) Penicillin is an effective medicine for pneumonia
called psychototropic drugs. These are two type disease.
(i) Sedative the drugs used for violent and mentaly 40. (d) The mixture of chloroxylenol and terpenol is dettol
agitated patient e.g.., Equanil and diazepam. which is used as antiseptic.
(ii) Antidepressant- The drug are used to patients who 41. (d) Chloroamphenicol is a broad spectrum antibiotic.
are highly depressed and lose self confidence e.g. 42. (a) Substances used for the treatment of malaria are
tofranil vitalin, amphetamine etc. antimalarial e.g. Quinine, chloroquine.
14. (a) 15. (b) 16. (d) 43. (d) Chloramphenicol is
17. (a) These drugs induce sleep and are habit forming O
common example of hypnotic drugs are Luminal and ||
NO2 CH CH NH C – CHCl 2
Saconal. | |
18. (d) OH CH 2OH
19. (b) Aspirin is antipyretic i.e., a drug which is responsible 44. (b) It is the known structure of Penicillin G
for lowering the temperature of feverish organism to 45. (c) Arsenic drugs are poisonous for syphilis.
normal, other antipyretic drugs are Paracetamol, 46. (b) Bithional is a well known antiseptic, added in soaps to
Phenacetin. reduce odours produced by bacterial decomposition
20. (b) Paracetamol is an antipyretic of organic matter of skin.
21. (b) It is acetyl salicylic acid i.e., aspirin, analgesic and
antipyretic. Cl OH HO Cl
22. (d)
23. (a) Due to anti-blood clotting action of aspirin, it is used S
to prevent heart attack. Cl
Bithional
Cl
24. (d) Sulpha drugs (antibacterial and antibiotic) are group
of drugs which are derivative of sulphanilamide. 47. (a) Penicillin is an antibiotic.
25. (a) Analgesic are pain killers. 48. (c) Chloramphenicol is a broad spectrum drug.
26. (b) Analgesic means painkiller. [Broad spectrum antibiotics are medicines effective
27. (c) We know that N-acetyl-para-aminophenol (or against gram positive as well as gram negative bacteria,
paracetamol) is an antipyretic which can also be used e.g., tetracycline, chloramphenicol, etc.]
as an analgesic to relieve pain.
CHEMISTRY IN EVERYDAY LIFE 519
49. (d) Bithional is another well known antiseptic which is 78. (a) Soaps are the sodium or potassium salt of higher fatty
added to good quality soaps to reduce the odours acids e.g., C17H37COOK (Potassium stearate). These
produced by bacterial decomposition of organic matter are obtained by alkaline hydrolysis of oils and fats.
on the skin. The reaction is called saponification.
50. (b) Dilute solutions of boric acid and hydrogen peroxide 79. (b) Alkaline hydr olysis of esters is known as
are weak antiseptics. saponification.
51. (d) R COOR ' NaOH R 'OH RCOONa
52. (d) Novestrol is an antifertility drug. 80. (d) Commercial detergent are the sodium salts of long chain
53. (c) 54. (b) (linear) alkyl substituted benzene sulphonic acids
55. (b) Morphine narcotics are chiefly used for the relief of (LAB) and are most widely used. The most common is
post operative pain. sodium dodecylbenzene sulphonate.
56. (d) Classification of drugs on the basis of pharmacological
effect is useful for doctors because it provides them – +
CH3(CH2)10 CH2 SO2O Na
the whole range of drugs available for the treatment of
a particular type of problem.
81. (c) Any oils which are good for eating or cooking, can be
57. (a) 58. (d)
used in making soap. One of the best is said to be
59. (a) Drug tegamet was designed to prevent the interaction
coconut oil. Groundnut, Shea butter, Cocoa butter, Sun
of histamine with the receptors present in the stomach
flower and many other vegetable oils are also used.
wall. This resulted in release of lesser amount of acid.
60. (b) 82. (b) Sodium or potassium salts of palmitic acid
61. (c) Antidepressant drugs inhibit the enzymes which (CH 3 (CH 2 ) 14 COO – Na + ) and stearic acid
catalyse the degradation of noradrenaline. (CH3(CH2)16COO–Na+) are used as soaps.
62. (d) 83. (b) Soaps are actually salts of higher fatty acids.
63. (b) Arsphenamine also known as salvarsan was the first Example ; C17 H 35 COONa
effective treatment discovered for syphilis. (sodium stearate)
64. (b) Penicillin G has a narrow spectrum, while all other
options have broad spectrum. 84. (b)
CH 3
65. (d) Chloramphenicol is rapidly absorbed from the |
gastrointestinal tract and hence can be given orally in CH3 (CH 2 )15 N CH3 Br
case of typhoid, acute fever, meningitis, pneumonia |
etc. CH3
66. (a) cetyl trimethyl ammonium bromide
67. (d) All are characteristics of Saccharin.
68. (d) Vitamin E is an antioxidant present in edible oils. 85. (d) Cetyltrimethyl ammonium bromide possess germicidal
69. (c) Salts of sorbic acid and propionic acid are used as properties. Thus it is used as a cationic detergent in
food preservatives because these chemicals inhibit the hair conditioners.
growth of yeast bacteria or moulds. 86. (b) 87. (c) 88. (a)
70. (d) Neither any of the substances among given options 89. (c) Non– ionic detergent is formed when stearic acid reacts
possess nutritive value. with polyethylene glycol.
71. (b) Artificial sweetener Sweetness value 90. (c)
Aspartame 100
Saccharin 550 STATEMENT TYPE QUESTIONS
Sucralose 600 91. (b) 92. (c)
Alitame 2000
93. (b) For statement (ii), drug which brings major change in
72. (c) Sucralose does not provide calories.
the treatment of hyperacidity was histamine. For
73. (a) Sodium benzoate is used as a food preservative.
statement (iv), antidepressant drugs like iproniazid and
74. (b) Structurally biodegradable detergents should contain
phenelzine inhibit the enzymes which catalyse the
branched alkyl chain.
degradation of noradrenaline when the enzyme is
75. (b)
inhibited, this important neurotransmitter is slowly
76. (d) It is used as detergent.
metabolised and can activate its receptor for longer
77. (c) The most widely used domestic detergent is the
periods of time, thus counteracting the effect of
sodium dodecyl benzene sulphonate (SDS).
depression.
94. (c) For statement (ii), 2–3% solution of iodine in alcohol
CH3 – (CH2)11 SO3Na water mixture is known as tincture of iodine. For
statement (iii), SO2 in very low concentrations are used
(Sodium dodecyl benzene sulphonate) as disinfectants.
EBD_7207
520 CHEMISTRY IN EVERYDAY LIFE
95. (b) 117. (d) Bacteriostatic drugs inhibit the growth of organism
96. (a) Shaving soaps contain glycerol to prevent rapid drying. while bactericidal drugs kill the microorganisms.
97. (d) For statement (iii), cationic detergents are expensive 118. (a)
and thus have limited use. For statement (iv), 119. (a) OH
detergents having unbranched chains can be
biodegraded more easily thus are less polluting as
compared to detergents having branched chains. H3C CH3
Cl
MATCHING TYPE QUESTIONS Chloroxylenol

98. (c) 99. (a) 100. (a) 101. (c) 102. (b) (4-chloro-3, 5-dimethylphenol)
120. (d) Drugs that mimic the natural messenger by switching
ASSERTION-REASON TYPE QUESTIONS on the receptor are called agonists. While drugs that
binds to the receptor site and inhibit its natural function
103. (d) The drugs which act on the central nervous system are called antagonists.
and help in reducing anxiety are called tranquilizers. 121. (b)
104. (a) Tranquilizers are chemicals which are used to cure 122. (b) Bactericidal have killing effect on microbes while
mental diseases. bacteriostatic have inhibitory effect on microbes.
105. (a) Broad spectrum antibiotics are those medicines which 123. (b) Chloramphenicol is bacteriostatic antibiotic while
are effective against several different types of harmful ofloxacin is bactericidal type antibiotic.
micro organisms. 124. (c) Narrow spectrum antibiotics are effective against
106. (b) Antiseptics are those chemical which kill or prevent Gram–positive or Gram-negative bacteria. Limited
the growth of micro organism. Antiseptics do not harm spectrum antibiotics are effective against a single
the living tissues and can be applied on cuts and organism or disease.
wounds. They help to reduce odour resulting from the 125. (a) Norethindrone is an example of synthetic progestrone
bacterial decomposition in the mouth and on the body. derivative most widely used as antifertility drug.
107. (a) A small quantity of sedative produces a feeling of 126. (b) Benzoic acid used as preservative as sodium benzoate.
relaxation, calmness and drowsiness. 127. (c) HO OH
108. (d) 109. (b) Cl
Cl HO
O
CRITICAL THINKING TYPE QUESTIONS HO O O
OH Cl
110. (a) 111. (c) Sucrolose
112. (c) Iproniazid Tranquilizer 128. (a) Benzalkonium chloride, also known as
Salvarsan Antimicrobial alkyldimethylbenzylammonium chloride is nitrogenous
Zantac (ranitidine) Antihistamine cationic surface active agent belonging to the
Chloramphenicol Antibiotic quaternary ammonium group. It is used as antiseptic.

113. (a) It is an alkaloid, a class of organic compound which is Cl
basic in nature and of plant origin containing atleast +
N
one nitrogen atom in a ring structure of molecule.
CnH2n+1
114. (b) The term “antihistamine” refers only to H1 antagonists,
which is also known as H1-receptor antagonists and n = 8, 10, 12, 14, 16, 18
H1-antihistamine. 129. (b) Laundry soaps contain fillers like sodium rosinate,
115. (a) Amoxillin is semisynthetic modification of Penicillin sodium silicate, borax and sodium carbonate.
116. (a) Ciprofloxacin is used as antibiotic while paracetamol, 130. (b)
ibuprofen and tocopherol are respectively antipyretic, 131. (b) Aspartame cannot be used in baked food as it is
pain killer and Vit. E. unstable at cooking temperature thus its use is limited
to cold foods and soft drinks.
Mock Test-1
Time : 1 hr Max. Marks -180
1. An acidic solution of 'X' does not give precipitate on 8. In sodium fusion test of organic compounds, the nitrogen
passing H2S through it. 'X' gives white precipitate when of the organic compound is converted into
NH4OH is added to it. The white precipitate dissolves in (a) Sodamide (b) Sodium cyanide
excess of NaOH solution. Pure 'X' fumes in air and dense (c) Sodium nitrite (d) Sodium nitrate
white fumes are obtained when a glass rod dipped in NH4OH 9. Specific volume of cylindrical virus particle is 6.02 × 10–2
is put in the fumes. Compound 'X' can be cc/gm. whose radius and length 7 Å & 10 Å respectively. If
(a) ZnCl2 (b) FeCl3 NA = 6.02 × 1023, find molecular weight of virus
(c) AlCl3 (d) SnCl2 (a) 3.08 × 103 kg/mol (b) 3.08 × 104 kg/mol
4
(c) 1.54 × 10 kg/mol (d) 15.4 kg/mol
2. CN– is a strong field ligand. This is due to the fact that
10. Inductive effect involves
(a) it carries negative charge (a) displacement of -electrons
(b) it is a pseudohalide (b) delocalisation of -electrons
(c) it can accept electrons from metal species (c) delocalisation of -electrons
(d) it forms high spin complexes with metal species (d) displacement of -electrons
3. The weight of NaCl decomposed by 4.9g of H2SO4, if 6 g of 11. The energy of a photon is 3 × 10–12 erg. What is its
sodium hydrogen sulphate and 1.825 g of HCl, were wavelength in nm ?
produced in the reaction is: (h = 6.62 × 10–27 erg-sec; c = 3 × 1010 cm/s)
(a) 6.921 g (b) 4.65 g (a) 662 (b) 1324
(c) 2.925 g (d) 1.4 g (c) 66.2 (d) 6.62
4. Which one of the following statement is not 12. Among the following compounds (I - III), the ease of their
true ? reaction with electrophiles is,
(a) pH of dr inking water should be between OCH3 NO2
5.5 – 9.5.
(b) Concentration of DO below 6 ppm is good for the
growth of fish.
(c) Clean water would have a BOD value of less than 5 ppm.
(d) Oxides of sulphur, nitrogen and carbon are the most I II III
widespread air pollutant.
5. Which of the following statements is not correct for nitrogen ? (a) II > III > I (b) III > II > I
(a) Its electronegativity is very high (c) II > I > III (d) I > II > III
13. Aluminium vessels should not be washed with materials
(b) d-orbitals are available for bonding
containing washing soda since
(c) It is a typical non-metal
(a) washing soda is expensive
(d) Its molecular size is small
(b) washing soda is easily decomposed
6. Which of the following statement is false ?
(c) washing soda reacts with Al to form soluble aluminate
(a) For 1 mole of an ideal gas, Cp – Cv = R (d) washing soda reacts with Al to form insoluble
E aluminium oxide
(b) 0 for an ideal gas 14. The following data are for the decomposition of ammonium
T T
nitrite in aqueous solution :
(c) q w p v Vol. of N2 in cc Time (min)
(d) For reversible isothermal expansion of 1 mole of an 6.25 10
ideal gas from volume V1 to V2, work done is equal to 9.00 15
RT ln (V2/V1) 11.40 20
13.65 25
7. 0.4 moles of HCl and 0.2 moles of CaCl 2 were dissolved in 35.65 Infinity
water to have 500 mL of solution, the molarity of Cl– ion is: The order of rection is :
(a) 0.8 M (b) 1.6 M (a) Zero (b) One
(c) 1.2 M (d) 10.0 M (c) Two (d) Three
EBD_7207
MT-2 CHEMISTRY

15. Which of the following reagents convert propene to 1- 23. The following equilibrium constants are given:
propanol? N 2 3H 2 2NH3 ; K1
(a) H2O, H2SO4 (b) aqueous KOH N2 O2 2NO; K 2
(c) MgSO4, NaBH4/H2O (d) B2H6, H2O2, OH– 1
16. A closed container contains equal number of oxygen and H2 O2 H 2 O; K 2
2
hydrogen molecules at a total pressure of 740 mm. If oxygen The equilibrium constant for the oxidation of NH3 by
is removed form the system then pressure will oxygen to give NO is
(a) Become double of 740 mm
K 2 K32 K 22 K3
(b) Become half of 740 mm (a) (b)
(c) Become 1/9 of 740 mm K1 K1
(d) Remains unchanged K1 K 2 K 2 K33
17. Compound X of molecular formula C4H6 takes up one (b) (d)
K3 K1
equivalent of hydrogen in presence of Pt to form another
24. Four successive members of the first row transition
compound Y which on ozonolysis gives only ethanoic acid.
elements are listed below with their atomic numbers.
The compound X can be Which one of them is expected to have the highest
(a) CH 2 CH CH CH 2 third ionization enthalpy?
(b) CH2 = C = CHCH3 (a) Vanadium (Z = 23) (b) Chromium (Z = 24)
(c) Manganese (Z = 25) (d) Iron (Z = 26)
(c) CH 3C CCH 3 25. Which of the following organometallic compound is and
(d) All the three bonded?
18. 1 M solution of CH3COOH should be diluted to ............... (a) [Fe ( 5 – C5H5)2] (b) Fe (CH3)3
times so that pH is doubled. (c) K [PtCl3( 2 – C2H4)] (d) [Co(CO)5 NH3]2+
(a) four times (b) 5.55 × 104 times 26. In the balanced chemical reaction
6
(c) 5.55 × 10 times (d) 10–2 times IO3 aI bH cH 2 O dI 2
19. Which of the following statements, about the advantage a, b, c and d respectively corresponds to
of roasting of sulphide ore before reduction is not true? (a) 5, 6, 3, 3 (b) 5, 3, 6, 3
(a) The G of of the sulphide is greater than those for (c) 3, 5, 3, 6 (d) 5, 6, 5, 5
27. Which of the following statements is true?
CS2 and H2S. (a) Silicon exhibits 4 coordination number in its
(b) The G of is negative for roasting of sulphide ore to compound
oxide. (b) Bond energy of F2 is less than Cl2
(c) Roasting of the sulphide to the oxide is (c) Mn(III) oxidation state is more stable than Mn(II) in
thermodynamically feasible. aqueous state
(d) Carbon and hydrogen are suitable reducing agents (d) Elements of 15th group shows only +3 and +5
oxidation states
for metal sulphides.
28. Which of the following compounds has the highest boiling
20. Which one of the following is NOT a buffer solution?
point?
(a) 0.8 M H2 S + 0.8 M KHS
(a) CH3CH2CH2 Cl
(b) 2MC6H5NH2 + 2MC6H5 N H 3 Br – (b) CH 3CH 2 CH 2 CH 2 Cl
(c) 3MH2CO3 + 3MKHCO3 (c) CH 3CH (CH 3 )CH 2 Cl
(d) 0.05 M KClO4 + 0.05 M HClO4 (d) (CH3 ) 3 CCl
21. Which of the following statements is false ? 29. Which one of the following statements is not correct ?
(a) Radon is obtained from the decay of radium (a) Nickel forms Ni(CO)4
(b) Helium is inert gas (b) All the transition metals form monometallic carbonyls
(c) Xenon is the most reactive among the rare gases (c) Carbonyls are formed by transition metals
(d) The most abundant rare gas found in the atmosphere (d) Transition metals form complexes
is helium 30. Hydrogen has an ionisation energy of 1311 kJ mol–1 and
22. Which one of the following is expected to exhibit optical for chlorine it is 1256 kJ mol–1. Hydrogen forms H+ (aq)
isomerism? ions but chlorine does not form Cl+ (aq) ions because
(en = ethylenediamine) (a) H+ has lower hydration enthalpy
(b) Cl+ has lower hydration enthalpy
(a) cis-[Pt(NH3)2 Cl2] (b) trans-[Pt(NH3)2Cl 2]
(c) Cl has high electron affinity
(c) cis-[Co(en)2Cl 2] (d) trans-[Co(en)2Cl 2]
(d) Cl has high electronegativity
MOCK TEST 1 MT-3

31. The number of en antiomers of the compound 39. In qualitative analysis, the metals of Group I can be
CH 3 CHBr CHBr COOH is : separated from other ions by precipitating them as chloride
(a) 2 (b) 3 salts. A solution initially contains Ag + and Pb2+ at a
(c) 4 (d) 6 concentration of 0.10 M. Aqueous HCl is added to this
32. Equivalent weighs of KMnO4 acidic medium, neutral solution until the Cl– concentration is 0.10 M. What will
medium and concentrated alkaline medium respectively are the concentrations of Ag+ and Pb2+ be at equilibrium?
M M M (Ksp for AgCl = 1.8 × 10–10,
, , . Reduced products can be Ksp for PbCl2 = 1.7 × 10–5)
5 1 3
(a) [Ag+] = 1.8 × 10–7 M ; [Pb2+] = 1.7 × 10–6 M
(a) MnO 2 , MnO 24 , Mn 2
(b) [Ag+] = 1.8 × 10–11 M ; [Pb2+] = 8.5 ×10–5 M
(b) MnO 2 , Mn 2 , MnO 24 (c) [Ag+] = 1.8 × 10–9 M ; [Pb2+] = 1.7 × 10–3 M
(d) [Ag+] = 1.8 × 10–11 M ; [Pb2+] = 8.5 ×10–4 M
(c) Mn 2 , MnO24 , MnO 2
40. In the diazotization of arylamines with sodium nitrite and
(d) Mn 2 , MnO2 , MnO24 hydrochloric acid, an excess of hydrochloric acid is used
33. Which of these have no unit? primarily to
(a) Electronegativity (b) Electron affinity (a) Supress the concentration of free aniline available
(c) Ionisation energy (d) Excitation potential for coupling
34. Which of the following statements is not correct for sigma (b) Supress hydrolysis of phenol
and pi-bonds formed between two carbon atoms? (c) Ensure a stoichiometric amount of nitrous acid
(a) Sigma-bond determines the direction between carbon (d) Neutralise the base liberated
atoms but a pi-bond has no primary effect in this
regard 41. In lake test of Al 3 ion, there is formation of coloured
(b) Sigma-bond is stronger than a pi-bond floating lake. It is due to
(c) Bond energies of sigma- and pi-bonds are of the order (a) adsorption of litmus by H2O
of 264 kJ/mol and 347 kJ/mol, respectively (b) adsorption of litmus by Al(OH)3
(d) Free rotation of atoms about a sigma-bond is allowed (c) adsorption of litmus by Al(OH)4–
but not in case of a pi-bond (d) none of these
35. The reactivity of metals with water is in the order of 42. For the reaction, 2Cl(g) — Cl2(g), the signs of H and S
(a) Na > Mg > Zn >Fe > Cu respectively, are:
(b) Cu > Fe > Zn > Mg > Na (a) +, – (b) +, +
(c) Mg > Zn > Na > Fe > Cu
(c) –, – (d) –, +
(d) Zn > Na > Mg > Fe > Cu
43. Ethanol and dimethyl ether form a pair of functional isomers.
36. The emf of Daniell cell at 298 K is E1
The boiling point of ethanol is higher than that of dimethyl
Zn | ZnSO4 (0.01 M) | | CuSO4 (1.0 M) | Cu
ether, due to the presence of
When the concentration of ZnSO4 is 1.0 M and that of
CuSO4 is 0.01 M, the emf changed to E2 What is the relation (a) H-bonding in ethanol
between E1 and E2? (b) H-bonding in dimethyl ether
(c) CH3 group in ethanol
(a) E1 = E2 (b) E2 0 E2
(d) CH3 group in dimethyl ether
(c) E1 E2 (d) E1 E2 44. Which of the following reactions will not result in the
CH 2 O formation of anisole?
37. (a) Phenol + dimethyl sulphate in presence of a base
O CH 2
(b) Sodium phenoxide is treated with methyl iodide
CH 2 O
(c) Reaction of diazomethane with phenol
The above shown polymer is obtained when a carbonyl (d) Reaction of methylmagnesium iodide with phenol
compound is allowed to stand. It is a white solid. The 45. What will be the heat of formation of methane, if the heat of
polymer is combustion of carbon is '–x' kJ, heat of formation of water
(a) Trioxane (b) Formose is '–y' kJ and heat of combustion of methane is 'z' kJ ?
(c) Paraformaldehyde (d) Metaldehyde.
38. The correct order of atomic/ionic sizes is (a) (–x – y + z) kJ (b) (–z – x + 2y) kJ
(a) N < Li < B (b) F O2 N3 (c) (–x – 2y – z) kJ (d) (–x – 2y + z) kJ
(c) Ca 2 S2 Cl (d) Na Mg 2 Cl
EBD_7207
MT-4 CHEMISTRY

ANSWER KEY

1. (a) 2. (b) 3. (c) 4. (b) 5. (b) 6. (c) 7. (b) 8. (b) 9. (d) 10. (a)
11. (a) 12. (d) 13. (d) 14. (b) 15. (d) 16. (b) 17. (d) 18. (b) 19. (d) 20. (d)
21. (d) 22. (c) 23. (d) 24. (c) 25. (d) 26. (a) 27. (b) 28. (b) 29. (b) 30. (b)
31. (c) 32. (c) 33. (a) 34. (c) 35. (a) 36. (c) 37. (a) 38. (b) 39. (c) 40. (a)
41. (b) 42. (c) 43. (a) 44. (d) 45. (d)

HINTS & SOLUTIONS


1. (a) NH 4 OH 8. (b) Sodium cyanide (Na + C + N NaCN).
X White ppt
(Lassaigne's test)
excess
Acidic solution (soluble ) 9. (d) Specific volume (volume of 1 gm) of cylindrical virus
NaOH ( No. ppt with H 2S) particle = 6.02 × 10–2 cc/gm
Given reactions (white precipitate with H2S in presence Radius of virus (r) = 7 Å = 7 × 10–8 cm
of NH4OH) indicate that 'X' should be ZnCl2 which Length of virus = 10 × 10–8 cm
explains all given reactions. Volume of virus
22
ZnCl 2 2H 2 O Zn (OH ) 2 HCl r 2l (7 10 8 ) 2 10 10 8
White fumes 7
= 154 × 10–23 cc
NH 4 OH HCl NH 4Cl volume
White fumes –H 2O Wt. of one virus particle
Dense white fumes specific volume
ZnCl 2 2 NaOH Zn(OH) 2 2NaCl Mol. wt. of virus = Wt. of NA particle
23
154 10
Zn (OH ) 2
2 NaOH
Na 2 ZnO 2 2H 2 O = 6.02 10 23
Excess 6.02 10 2
2. (b) CN– is a strong field ligand as it is a psuedohalide ion. = 15400 g/mol = 15.4 kg/mole
These ions are strong coordinating ligands and hence 10. (a) Inductive effect involves displacement of -electrons.
have the tendency to form -bond (from the pseudo hc
halide to the metal) and -bond. (from the metal to 11. (a) Using the relation
E
pseudo halide) Substituting given values, we get
3. (c) NaCl + H 2SO 4 NaHSO 4 HCl
xg 4.9g 6g 1.825g 6.62 10 27 erg-sec 3 1010 cm s 1
According to law of conservation of mass "mass is 3 10 12 erg
neither created nor destroyed during a chemical = 6.62 × 10–5 cm
change" = 6.62 × 10–5 × 107 nm [1 cm = 107 nm]
Mass of the reactants = Mass of products = 662 nm
x + 4.9 = 6 + 1.825 12. (d) –OCH3 activates the benzene ring. –NO2 deactivates
or x = 2.925 g the ring. Hence the reaction of the given compounds
4. (b) The ideal value of D.O for growth of fishes is 8 mg/ . with electrophiles is in the order, I > II > III.
7mg is desirable range, below this value fishes get 13. (d)
susceptible to desease. A value of 2 mg/ or below is 14. (b) NH 4 NO2 N 2 2H 2O
lethal for fishes. Volume of N2 formed in successive five minutes are
5. (b) In case of nitrogen, d-orbitals are not available. 2.75 cc, 2.40 cc and 2.25 cc which is in decreasing
6. (c) C is incorrect ; The correct is E = q + w order. So rate of reaction is dependen t on
7. (b) HCl H Cl concentration of NH4NO2. As decrease is not very
0.4moles 0.4moles fast so it will be first order reaction.
2 15. (d) We know that
CaCl2 Ca 2Cl
0.2moles 2 0.2 0.4moles B 2H 6
6 CH3 CH CH 2
Total Cl– moles = 0.4 + 0.4 = 0.8 moles 1, Pr opene ether, 0°C
Moles
Molarity = 2(CH3 CH 2 CH 2 )3
H2O2
Vol.in L
OH
0.8 6CH 3CH 2 CH 2 OH 2H 3 BO 3
Molarity of Cl– = 1.6 M.
0.5 Pr opanol
MOCK TEST 1 MT-5

16. (b) p tot 740 mm 23. (d) Given,


N 2 3H 2 2NH 3 ; K1 ....(i)
p tot p O2 pH 2
N2 O2 2NO; K 2 ....(ii)
Number of moles of O 2 and H2 are equal 1
p O2 p H 2 H2 O2 H 2 O; K 3 ....(iii)
2
pO 2 pH 2 740 We have to calculate
4NH 3 5O2 4NO 6H 2O; K ?
pO2 370mm p H2
5
17. (d) Formation of only CH3COOH by ozonolysis indicates or 2NH 3 O2 2NO 3H 2 O
2
that the compound Y should be CH3CH = CHCH3
which can be formed by all of the three given [NO]2 [H2O]3
For this equation, K
compounds [NH3 ]2 [O2 ]5 / 2
1H 2 / Pt
CH 2 CH CH CH 2
X [NH3 ]2 [NO]2
but K1 3
, K2
CH 3 CH CH CH 3 [N 2 ] [H 2 ] [N 2 ] [O 2 ]
Y
1H 2 / Pt [H 2O] [H 2O]3
CH3C CCH3 CH3CH CHCH3 & K3 or K 3
½
X Y [H 2 ] [O 2 ] [H 2 ]3 [O 2 ]3 / 2
1H 2 / Pt
CH 2 C CHCH3 K 2 . K 33
X
Now operate,
O3
K1
CH 3 CH CHCH3 2CH 3 COOH
Y [NO]2 [H 2 O]3 [N ] [H2 ]3
3 3 / 2
. 2
pH
1
[pK a log1]
pK a [N 2 ] [O2 ] [H 2 ] [O2 ] [NH3 ]2
18. (b)
2 2
pH' (twice of pH) = pKa [NO]2 [H 2 O]3
K
1 [NH 3 ]2 [O 2 ]5 / 2
pK a [pK a log C ]
2
– log C = pKa = – log Ka K 2 . K 33
K
C = Ka = 1.8 × 10–5 M K1
1 24. (c) For third ionization enthalpy last configuration of
dilution 5.55 × 10 4 times
C 3d 4s
19. (d) The sulphide ore is roasted to oxide before reduction V – 4s0 3d3
because the Gof of most of the sulphides are greater
than those of CS2 and H2S, therefore neither C nor H Cr – 4s0 3d4
can reduce metal sulphide to metal. Further, the Mn – 4s0 3d5
standard free energies of formation of oxide are much
Fe – 4s0 3d6
less than those of SO2. Hence oxidation of metal
sulphides to metal oxide is thermodynamically For third Ionization enthalpy Mn has stable
favourable. configuration due to half filled d-orbital.
20. (d) Buffer solution contains weak base + salt of weak 25. (d) [ Co (CO ) 5 NH 3 ]2 . In this complex. Co-atom attached
base with strong acid or weak acid + salt of weak acid with NH3 through bonding with CO attached with
with strong base. dative -bond.
In option (d) the acid used is HClO4 which is strong 26. (a) Given reaction is
acid and KClO4 is salt of this acid with strong base.
So it is not an example of buffer solution. IO3 aI bH cH 2 O dI 2
21. (d) The most abundant rare gas found in the atmosphere Ist half reaction
is argon and not helium. I I2 ...(i)
–1 0 (oxidation)
22. (c) en +
en
+ IInd half reaction
Cl Cl
IO 3 I2 ...(ii)
Co Co +5 0 (reduction)
On balancing equation (ii) we have
Cl Cl
en 10e 2IO3 12H I 2 6H 2 O ...(iii)
en
Cis-d-isomer Cis- -isomer
Now, balance equation (i)

Mirror
EBD_7207
MT-6 CHEMISTRY

2I I2 2e ....(iv) 0.0591 1
E2 E0 log
Multiply eqn (iv) by 5 and add it to eqn (iii), we get 2 0.01
2IO 3 10I 12H 6I 2 6H 2 O 0.0591
E0 – log10 2
or, IO3 5I 6H 3I 2 3H 2 O 2
Hence a = 5, b = 6, c = 3, d = 3 2 0.0591
E0 – or (E 0 – 0.0591)V
27. (b) This is because of inter-electronic replusions between 2
lone pairs. Thus, E1 E 2
B.E. : F – F Cl – Cl
CH2
(kJ mol–1) : 158.8 242.6
.. ..
On keeping O O
: .F. F 37. (a) 3 HCHO
..: CH2 CH2
aq. solution
28. (b) Molecules having higher molecular weight and less O
branching have higher boiling point. Trioxane
29. (b) Always transition metals combines with more than (meta formaldehyde)
one carbonyl group.
38. (b) Amongst the isoelectronic species, the anion having
30. (b) Hydration energy of Cl+ is very less than H+ hence it
more negative charge would have the larger size.
doesn’t form Cl+ (aq) ion.
39. (c) Ksp = [Ag+] [Cl–]
31. (c) No. of asymmetric carbon = 2
1.8 × 10–10 = [Ag+] [0.1]
No. of enantiomers = 22 = 4. [Ag+] = 1.8 × 10–9 M
32. (c) Change Equiv. mass Ksp = [Pb+2] [Cl–]2
H M 1.7 × 10–5 = [Pb+2] [0.1]2
MnO 4 Mn 2 5
7 2 5 [Pb+2] = 1.7 × 10–3 M
H 2O
40. (a) Excess of HCl is used to convert free aniline to aniline
MnO 4 MnO 42 1 M hydrochloride otherwise free aniline would undergo
7 6 coupling reaction with benzenediazonium chloride.
OH M 41. (b) In lake test of Al3+, there is formation of coloured
MnO 4 MnO2 3 floating lake. It is due to the adsorption of litmus by
7 4 3
Al(OH)3.
33. (a) Electronegativity is the tendency of the atom to attract
42. (c) 2Cl(g) — Cl2(g)
electrons to itself when combined in a compound as
Entropy is decreasing (–ve) in the reaction. Further
defined by Pauling. Electronegativity, is a relative term
the reaction is exothermic since a bond is being
so it does not have any unit.
formed, i.e., H is also –ve.
34. (c) As sigma bond is stronger than the (pi) bond, so it
43. (a) Due to H-bonding, the boiling point of ethanol is much
must be having higher bond energy than (pi) bond.
higher than that of the isomeric diethyl ether.
35. (a) The reactivity may be attributed to size factor, larger
44. (d) Phenol has active (acidic) hydrogen so it reacts with
is the size, higher is tendency to lose electron (low
CH3MgI to give CH4, and not anisole
I.E.). Zn is the last element of 3d series and it has
C 6 H 5 OH CH 3MgI CH 4 C6 H5OMgI
larger size than Cu due to (3d10 4s 2 ) configurations.
45. (d) From given data, we have
Hence, the reactivity order is C + O2 CO2 – x kJ … (i)
Na > Mg > Zn > Fe > Cu.
1
36. (c) Using the relation H2 O2 H 2 O y kJ …(ii)
2
0 0.0591 [anode]
E cell E cell log CH 4 2O 2 CO 2 2H 2 O z kJ …(iii)
n [cathode]
The required equation is
2+
0.0591 [Zn ] C 2H 2 CH 4 Q
E0cell log
n [Cu 2+ ] To get the required equation, operate
Substituting the given values in two cases. (i) + 2 × (ii) – (iii)
Thus, we get
0.0591 0.01
E1 E 0 log C 2H 2 CH 4 [( x ) ( 2 y ) ( z )]
2 1.0
0.0591 Thus, heat of formation of methane is
E0 log10 2 (–x – 2y + z) kJ
2
0.0591
E0 2 or (E 0 0.0591)V
2
Mock Test-2
Time : 1 hr Max. Marks -180
1. The angular momentum of the electron in first excited
1
energy state of hydrogen atom is (c) n = 3, l = 2, m = +1, s =
2
h h
(a) (b) 1
2
(d) n = 3, l = 1, m = –1, s =
h 2
(c) 2(2 1) (d) None of these 4+ – 2+
2 9. Sn + 2e Sn E° = 0.13 V
Br2 + 2e– 2Br – E° = 1.08 V
2. When NaCl is dopped with 1.0 × 10–3 mole of SrCl2, the
number of cation vacancy is Calculate Keq for the cell at 20°C formed by two electrodes
(a) 6.023 × 1018 (b) 6.023 × 1020 (a) 1041 (b) 1032
20 –32 (d) 10–42
(c) 2 × 6.023 × 10 (d) 3.011 × 1020 (c) 10
3. A 0.5 M NaOH solution offers a resistance of 31.6 ohm in a 10. Calculate the pH of a solution obtained by mixing 2 ml of
conductivity cell at room temperature. What shall be the HCl of pH 2 and 3 ml of solution of KOH of pH = 12
approximate molar conductance of this NaOH solution if (a) 10.30 (b) 3.70
cell constant of the cell is 0.367 cm–1 . (c) 11.30 (d) None of these
(a) 23.4 S cm2 mole–1 (b) 23.2 S cm2 mole–1 11. Which of the following represents a correct sequence of
(c) 46.45 S cm2 mole–1 (d) 54.64 S cm2 mole–1 reducing power of the following elements?
4. Ammonium dichromate on heating gives (a) Li > Cs > Rb (b) Rb > Cs > Li
(a) chromic acid & ammonia (c) Cs > Li > Rb (d) Li > Rb > Cs
(b) chromium sesquioxide & nitrogen 12. Paramagnetism of Cr (Z = 24), Mn2+ (Z = 25) and Fe3+
(c) chromium sesquioxide & ammonia (Z = 26) are x, y and z respectively. They are in the order
(d) chromic acid and N2 (a) x = y = z (b) x > y > z
5. Predict the relative acidic strength among the following (c) x = y > z (d) x > y = z
(a) H2O, H2S, H2Se, H2Te 13. Formaldehyde reacts with ammonia to give urotropine is
(b) H2O < H2S < H2Se < H2Te (a) (CH2)6N4 (b) (CH2)4N3
(c) H2Te < H2Se < H2S < H2O (c) (CH2)6N6 (d) (CH2)3N3
(d) H2O < H2Se < H2S < H2Te 14. Indicate the wrongly named compound
6. The catalyst used in the preparation of an alkyl chloride by (a) CH 3 C H CH 2 CH 2 CHO
the action of dry HCl on an alcohol is |
(a) anhydrous AlCl3 (b) FeCl3 CH 3
(c) anhydrous ZnCl2 (d) Cu
(4-methyl -1- pentanal)
7. In which of the following, resonance will be possible?
(a) CH2 CH2 CH2 CHO (b) CH 3 CH C C COOH
|
(b) CH 2 CH CH O CH 3
(c) CH 3COCH 3 (4- methyl -2- pentyn -1- oic acid)
(d) CH 2 CH CH 2 CH CH 2
(c) CH 3CH 2 CH 2 C H COOH
8. The four quantum numbers that could identify the third 3p |
electron in sulphur are CH 3
1 (2- methyl -1- pentanoic acid)
(a) n = 3, l = 0, m = +1, s =
2
O
1 ||
(b) n = 2, l = 2, m = –1, s = (d) CH3 CH2 CH CH C CH3
2
EBD_7207
MT-8 CHEMISTRY

(3- hexen -5- one) Alc. NH 3


15. The favourable condition for a process to be spontaneous C 6 H 5 CH CHCO 2 H
is : 22. The product obtained on reaction of C2H5Cl with hydrogen
(a) T S H, H ive, S ive over palladium carbon is :
(a) C3H8 (b) C4H10
(b) T S H, H ive, S ive (c) C2H6 (d) C2H4
(c) T S H, H ive, S ive 23. Which of the following can be predicted from
electronegativity values of elements ?
(d) T S H, H ive, S ive
(a) Dipole moment of a molecule
16. Vapour pressure (in torr) of an ideal solution of two liquids (b) Valency of elements
A and B is given by : P = 52XA + 114
(c) Polarity of bonds
where XA is the mole fraction of A in the mixture. The vapour
(d) Position in electrochemical series
pressure (in torr) of equimolar mixture of the two liquids
will be : 24. The reaction 3ClO ( aq ) ClO 3( aq ) 2Cl ( aq ) is an
(a) 166 (b) 83 example of
(c) 140 (d) 280 (a) oxidation reaction
17. The decomposition of a substance follows first order (b) reduction reaction
kinetics. Its concentration is reduced to 1/8th of its initial (c) disproportionation reaction
value in 24 minutes. The rate constant of the decomposition (d) decomposition reaction
process is 25. The concentration of a reactant X decreases from 0.1 M to
0.692 1 0.005 M in 40 min. If the reaction follows first order kinetics,
(a) 1/24 min–1 (b) min
the rate of the reaction when the concentration of X is 0.01
24
M will be
2.303 1 1 2.303 8 1 (a) 1.73 × 10–4 M min–1 (b) 3.47 × 10–4 M min–1
(c) log min (d) log min
24 8 24 1 (c) 3.47 × 10–5 M min–1 (d) 7.5 × 10–4 M min–1
18. Fluorine does not show highest oxidation state opposite 26. Mark the false statement?
to other halogens, because (a) A salt bridge is used to eliminate liquid junction
(a) it is most electronegative potential
(b) it has no d-orbital (b) The Gibbs free energy change, G is related with
(c) its atomic radius is very small electromotive force E as G = –nFE
(d) F– ion is stable and isoelectronic with neon (c) Nernst equation for single electrode potential is
19. Glucose contains in addition to aldehyde group. RT
E Eo log a M n
(a) one secondary –OH and four primary –OH groups nF
(b) one primary –OH and four secondary –OH groups (d) The efficiency of a hydrogen-oxygen fuel cell is 23%
(c) two primary –OH and three secondary –OH groups 27. The paramagnetism of transition element compounds is
(d) three primary –OH and two secondary –OH groups due to
20. Which of the following product is obtained by treating 1- (a) paired eletrons spining in opposite directions
butyne with HgSO4 and H2SO4? (b) unpaired eletrons in d and f-orbitals
(a) CH3CH2COCH3 (b) CH3CH2CH2CHO (c) shared valance electrons
(c) CH3CH2CH2COOH (d) CH3CH2CH = CH2
(d) unpaired electrons in s or p-orbitals.
21. An example of Perkin’s reaction is
28. Aniline, chloroform and alcoholic KOH react to produce a
(a) C6 H5CHO CH3 NO 2 bad smelling substance which is
KOH (a) phenyl isocyanide
C6 H 5 CHCHNO 2
(b) phenyl cyanide
(b) C 6 H 5 CHO (CH 3CO ) 2 O (c) chlorobenzene
(d) benzyl alcohol.
CH 3COONa
C 6 H 5 CH CHCOOH 29. The species with a radius less than that of Ne is
(c) C6 H5CHO CH3CHO (a) Mg2+ (b) F–
(c) O 2– (d) K+
NaOH
C6 H5 CH= CHCHO 30. Vapour density of the equilibrium mixture of the reaction
SO2Cl2(g) SO2(g) + Cl2(g) is 50.0. Percent dissociation
(d) C 6 H 5CHO CH 2 (COOH) 2
of SO2Cl2 is :
MOCK TEST 2 MT-9

(a) 33.33 (b) 35.0 (c) in the presence of cryolite which forms a melt with
(c) 30.0 (d) 66.67 lower melting temperature
31. What will be the emf for the given cell (d) in the presence of cryolite which forms a melt with
Pt | H2 (P1) | H+ (aq) | H2 (P2) | Pt higher melting temperature
39. Consider the following complex
RT P1 RT P1
(a) ln (b) ln [Co(NH3)5CO3]ClO4.
F P2 2F P2
The coordination number, oxidation number, number of d-
RT P2 electrons and number of unpaired d-electrons on the meal
(c) ln (d) None of these are respectively
F P1
(a) 6, 3, 6, 0 (b) 7, 2, 7, 1
32. H3PO3 is (c) 7, 1, 6, 4 (d) 6, 2, 7, 3
(a) neutral (b) basic 40. Nylon is a :
(c) a tribasic acid (d) a dibasic acid
(a) polysaccharide (b) polyester
33. Of the following which is diamagnetic in nature?
(c) polyamide (d) all of the above
(a) [ Co F6 ]3 (b) [ Ni Cl 4 ]2
Br2
(c) [ Cu Cl 4 ]2 (d) [Ni(CN) 4 ]2 41. OH SO3H X,
H2O
34. Which of the following products is formed when
X is identified as
benzaldehyde is treated with CH3MgBr and the addition
product so obtained is subjected to acid hydrolysis ? (a) 2, 4, 6-tribromophenol
(a) A secondary alcohol (b) A primary alcohol (b) 2-bromo-4-hydroxylbenzene sulphonic acid
(c) Phenol (d) tert-Butyl alcohol (c) 3, 5-dibromo-4-hydroxybenzene sulphonic acid
35. Mole fraction of methanol in its aqueous solution is 0.5. (d) 2-bromophenol
The concentration of solution in terms of percent by mass 42. The non-polar molecule is :
of methanol is (a) NF3 (b) SO3
(a) 36 (b) 50 (c) CHCl3 (d) ClO2
(c) 64 (d) 72
43. The hybridization of P in PO 34 is the same as of
36. The unit cell of an ionic compound is a cube in which
cations (A) occupy each of the corners and anions (B) are (a) S in SO3 (b) N in NO–3
at the centres of each face . The simplest formula of the (c) S in SO4– – (d) I in ICl4–
ionic compound is 44. Solution of potash alum is acidic in nature. This is due to
(a) AB2 (b) A3B hydrolysis of
(c) AB3 (d) A4 B 3
37. For orthorhombic system axial ratios are a b c and (a) SO 24 (b) K+
the axial angles are (c) Al2(SO4)3 (d) Al3+
(a) 90 45. When conc. HNO3 acts on our skin, the skin becomes
(b) 90 yellow, because
90 , 90 (a) HNO3 acts as an oxidising agent
(c)
(b) HNO3 acts as a dehydrating agent
(d) 90
38. Electrolytic reduction of alumina to aluminium by Hall- (c) Nitro-cellulose is formed
Heroult process is carried out (d) The proteins are converted into xanthoproteins
(a) in the presence of NaCl
(b) in the presence of fluorite
EBD_7207
MT-10 CHEMISTRY

ANSWER KEY

1. (a) 2. (b) 3. (b) 4. (b) 5. (a) 6. (c) 7. (b) 8. (d) 9. (b) 10. (c)
11. (a) 12. (d) 13. (a) 14. (d) 15. (b) 16. (c) 17. (d) 18. (b) 19. (b) 20. (a)
21. (b) 22. (c) 23. (c) 24. (c) 25. (d) 26. (c) 27. (b) 28. (a) 29. (a) 30. (b)
31. (b) 32. (d) 33. (d) 34. (a) 35. (c) 36. (c) 37. (b) 38. (c) 39. (a) 40. (c)
41. (c) 42. (b) 43. (c) 44. (a) 45. (d)

HINTS&SOLUTIONS
h 9. (b) Sn 4 2e Sn 2 E 0.13 V
1. (a) Angular momentum, mvr n
2 Br2 2e 2Br E 1.08 V
(n = 2 for first excited state) E° values shows Br 2 has higher reduction potential.
2. (b) Two Na+ ions are replaced by one Sr 2+ion to maintain Hence Ecell = ER – EL
electrical neutrality.
= E E
Hence, number of vacancies Br2 / Br Sn 4 / Sn 2
= Number of Sr 2+ ions doped = 1.08 – 0.13 = 0.95 V
3 23 20
= 1.0 10 6.02 10 6.02 10 Now G nFE cell
3. (b) Here, R = 31.6 ohm n = 2, F = 96500.
1 1 G 2 96500 0.95 kJ/mol
C= ohm 1 = 0.0316 ohm–1
R 31.6
Specific conductance = conductance × cell Also, G 2.303RT log K eq

G
constant log K eq
2.303 R T
= 0.0316 ohm–1 × 0.367 cm–1
= 0.0116 ohm–1 cm–1 ( 2 96500 0.95)
Now, molar concentration = 0.5 M = 32.6820
2.303 8.314 293
(given)
Keq = antilog 32.682
= 0.5 × 10–3 mole cm–3
= 4.78 × 1032 1032
K 0.0116
Molar conductance = 10. (c) [H ] in HCl solution (pH = 2) = 10 2
M ; [OH ] in
C 0.5 10 3
= 23.2 S cm2 mol–1 KOH solution (pOH = 14 – 12 = 2) = 10 2
M
4. (b) heat Cr2O3 4H 2O N 2
(NH 4 ) 2 Cr2 O7 Excess m Mol of OH in 5 ml mixture
5. (a) Assume that each has lost a proton . So we get : HO– 2 2 2
= 3 10 2 10 1.0 10 ;
, HS–, HSe–, HTe–
It can be easily seen that the volume available for the 1.0 10 2
negative charge is increasing from HO– to HTe– , [OH ] in mixture =
5
therefore
(i) volume available for the negative charge is = 2 10 3
M;
increasing from left to right
3
(ii) charge density is decreasing from left to right pOH = log 2 10 = 3 – log 2;
(iii) basicity is decreasing from left to right pH = 14 – (3 – log 2) = 11.30
(iv) acidity of conjugate acids is increasing from left 11. (a) A reducing agent is a substance which can loose
to right electron and hence a reducing agent should have low
H2O < H2S < H2Se < H2Te ionisation energy. Now since ionisation energy
6. (c) In preparation of an alkyl chloride by the action of dry decreases from Li to Cs, the reducing property should
HCl, the catalyst generally used is anhydrous ZnCl2. increase in the opposite manner. The only exception
7. (b) Only structure (b) has a conjugated system, which is to this is lithium. This is because the net process of
necessary for resonance. converting an atom to an ion takes place in 3 steps.
8. (d)
MOCK TEST 2 MT-11

(i) M(s) M(g) H = Sublimation energy 21. (b) In general,


(ii) M(g) M+(g) + e– H = Ionisation energy
OH O
(iii) M+(g)+H2O M+ (aq) H = Hydration energy | ||
CN
The large amount of energy liberated in hydration of ArCHO Ar C H C Ar
Aromatic aldehyde Acyloins
Li (because of its small size) makes the overall H
negative. This accounts for the higher oxidation Pd
22. (c) C2 H 5Cl H 2 C2H 6
potential of lithium i.e., its high reducing power.
12. (d) Number of unpaired electrons in Cr, Mn 2+ and Fe3+ Reduction of alkyl halide by H 2 in the presence of Pd
are 6, 5 and 5 respectively. which serves as catalyst. The accumulation of
13. (a) 6HCHO 4NH 3 hydrogen gas occurrs on the surface of active
palladium so that fast reduction may be achieved.
(CH 2 )6 N 4 6H 2 O 23. (c) Greater the difference of electronegativities, more polar
urotropine is the bond.
hexamethylene
tetramine 24. (c) O.N. of Cl in ClO– changes from +1 to +5 and –1.
25. (d) For a first order reaction, we have
O
2 || 1 2.303 N
14. (d)
6 5 4 3 k log 0
C H3 C H2 C H C H C C H3 t N
(hex 3-ene-5-one)
2.303 0.1
15. (b) k log
16. (c) Total V.P., 40 min 0.005

P PAº X A PBº X B PAº X A PBº ( 1 X A ) 2.303 2.303


log 20 1.3010
40 min 40
= (PAº PBº )X A PBº
Now rate = k × [reactant]
Thus, PBº 114 torr ; PAº PBº 52 When [x] = 0.01 M

or 2.303
PAº 166 torr rate × 1.3010 × 0.01 M min–1
40
1 1 = 7.5 × 10–4 M min–1.
Hence P 166 114 140 torr
2 2 26. (c) Correct Nernst equation is
2.303 a 2.303 RT
17. (d) k
t
log
a x
E Eo log a M n .
nF
2.303 1 2.303 27. (b)
log log 8
24 1 24 28. (a) This is isocyanide test
8
NH2
18. (b)
19. (b) Structural formula of glucose is
+ CHCl3 3KOH
CHO Chloroform
| Aniline
(CHOH)4
|
N C
CH 2OH
In addition to – CHO group it contains one primary + 3KCl + 3H2O
and four secondary – OH groups.
Hg 2 Phenyl isocyanide
20. (a) CH3 CH 2 C CH (Offensive smell)
H3O

29. (a)
OH
| 30. (b) SO 2Cl 2(g ) SO 2(g ) Cl 2(g)
tautomerization
CH3 CH 2 C CH 2
D d
CH3CH 2COCH3 % 100 (y = 2)
d ( y 1)
EBD_7207
MT-12 CHEMISTRY

(An atom at corner is shared by 8 unit cells)


molar mass of SO2 Cl 2 135
D 67.5;
2 2 1
Number of B atoms per unit cell = 6 3
d = 50.0 (given) 2
(An atom at the face centre is sh ared by
67.5 50.0
% 100 = 35% 2 unit cells)
50.0(2 1) Hence, formula is : AB3
31. (b) LHS : H2 (P1) 2H+ + 2e– Oxidation 37. (b) For orthorhombic 90
RHS : 2H+ + 2e– H2 (P2) Reduction 38. (c)
Net reaction : 39. (a) [Co(NH3)5CO3]ClO4. Six monodentate ligands are
allached to Co hence C. N. of Co = 6;
H2 (P1) H2 (P2)
O. N. = x + 5 × (0) + 1× (–2) + 1× (–1) = 0
RT P2 RT P1 x = + 3 ; electronic configuration of Co3+[Ar] 3d64s0
E = E° – nF ln . P 0 ln
2F P2 hence number of d electrons is 6 : All d electrons are
1
paired due to strong ligand hence unpaired electrons
O zero.
32. (d) H P OH , so dibasic acid O
| ||
OH 40. (c) Nylon has C NH – group linkage
3d
33. (d) [Co F6 ]3 : Br2
41. (c) HO SO3H
H2 O
.. .. 4p.. .. .. ..
4s 4d
Br
3 2
sp d (Paramagnetic)
HO SO3H
[Ni Cl 4 ]2 :
.. .. .. .. Br
– OH group is highly activating. This is a type of
sp 3(paramagnetic) electrophillic substitution reaction at ortho and para
position.
[CuCl 4 ]2 :
42. (b) SO3 has trigonal planar geometry (sp2 hybridisation
.
.
.. .. .. of S) which is symmetrical.
dsp2 (paramagnetic) 43. (c) Number of hybrid orbitals of P in PO 34 = ½ [5 + 0 +
[Ni(CN)4]2– :
. 3] = 4 (sp3)
. No. of hybrid orbitals of N in NO–3
(diamagnetic) .. .. .. dsp2 : = ½ [5 + 0 + 1] = 3 (sp2)
No. of hybrid orbitals of S in SO4– –
34. (a) Aldehydes, other than formaldehyde, when treated
with RMgX give 2º alcohols. 1
35. (c) Mass of methanol in 1 mol solution = 0.5 × 32 = 16 g = [6 + 0 + 2 – 0] = 4 (sp3)
2
44. (a) Al2(SO4)3 is a salt of weak base and a strong acid
Mass of water in solution = 0.5 × 18 = 9 g hence on hydrolysis it will produce Al(OH)3 and
16 100 H2SO4. Since H2SO4 is a strong acid and Al(OH)3 is
% by mass of methanol 64 is a weak base hence the solution will be acidic due to
16 9
SO4– –.
1 45. (d) It is the correct answer.
36. (c) Number of A atoms per unit cell = 8 1
8
Mock Test-3
Time : 1 hr Max. Marks -180
1. Fluorine is more electronegative than either boron or 6. Which of the following molecules is most suitable to
phosphorus. What conclusion can be drawn from the fact disperse benzene in water ?
that BF3 has no dipole moment but PF3 does ? O
(a) – +
(a) BF3 is not spherically symmetrical but PF3 is spherically O Na
symmetrical.
+ – O
(b) BF3 molecule must be linear (b) Na O – +
(c) The atomic radius of P is larger than that of B O Na
(d) The BF3 molecule must be planar triangular O
2. The volume-temperature graphs of a given mass of an ideal
gas at constant pressure are shown below. (c)
CH3

p2 p
3
V p1 (d) Cl
7. Ozone hole refers to
(a) Increase in concentration of ozone
(b) Hole in ozone layer
(c) Reduction in thickness of ozone layer in troposphere
O 273 T(K) (d) Reduction in thickness of ozone layer in stratsophere
8. Which of the following does not represent the correct order
What is the correct order of pressures ? of the properties indicated
(a) p1 > p3 > p2 (b) p1 > p2 > p3 (a) Ni2+ > Cr2+ > Fe2+ > Mn2+ (size)
(c) p2 > p3 > p1 (d) p2 > p1 > p3 (b) Sc > Ti > Cr > Mn (size)
3. Which is the most suitable reagent among the following to (c) Mn2+ > Ni2+ < Co2+ < Fe2+
distinguish compound (3) from rest of the compounds ? (unpaired electron)
1. CH 3 C C CH 3 (d) Fe2+ > Co2+ > Ni2+ > Cu2+
(unpaired electron)
2. CH 3 CH 2 CH 2 CH 3
9. The major product formed in the following reaction is :
3. CH 3 CH 2 C CH
4. CH 3 CH CH 2. CH 3
|
(a) Bromine in carbon tetrachloride CH 3O –
CH 3— C — CH 2 Br
(b) Bromine in acetic acid | CH 3OH
H
(c) Alk KMnO4
CH3
(d) Ammonical silver nitrate. |
4. In the extraction of Cu, the metal is formed in the bessemer (a) CH 3—C — CH 2 OCH3
|
converter due to the reaction : H
(a) Cu2S + 2Cu2O 6Cu + SO2 (b) CH 3— C H — CH 2 CH 3
(b) Cu2S 2Cu + S |
OCH3
(c) Fe + Cu2O 2Cu + FeO
(d) 2Cu2O 4Cu + O2 CH 3
|
5. An example of electrophilic substitution reaction is (c) CH 3 — C CH 2
(a) Chlorination of methane
CH3
(b) Conversion of methyl chloride to methyl alcohol |
(c) Nitration of benzene (d) CH3 — C — CH3
|
(d) Formation of ethylene from ethyl alcohol. OCH3
EBD_7207
MT-14 CHEMISTRY

10. (I) n = 3, l = 2, m1 = –2 18. IUPAC name of the following compound :


(II) n = 3, l = 1, m1 = 0
O
(III) n = 3, l = 0, m1 = – 1 || | CH3
(IV) n = 3, l = 2, m1 = 0 C N

|
(V) n = 3, l = 3, m1 = –2 CH3
Of these question state designation which does not
(a) N, N-dimethylcyclopropanecarboxamide
describe an allowed state for an electron in an atom ?
(b) N-methylcyclopropanamide
(a) I and IV (b) III and V
(c) cyclopropionamide
(c) II and V (d) IV and V (d) none of the above
11. In an adiabatic process which of the following is true? 19. Which of the following shows iso-structural species?
(a) q = + w (b) q = 0
(a) NH 4 and NH 2
(c) E=q (d) P V = 0
12. In which of the following cases, the stability of two (b) CH 3 and CH 3
oxidation states is correctly represented
(a) Ti3+ > Ti4+ (b) Mn2+ > Mn3+ (c) SO 24 , PO 34 and [ BF4 ]
2+
(c) Fe > Fe 3+ (d) Cu+ > Cu2+ (d) NH 4 and NH 3
13. Which is not the disproportionation reaction ? 20. When dihydroxyacetone reacts with HIO4, the product is/
(a) 3H 3PO 2 2H 3 PO 2 PH 3 are :
(b) HCHO OH HCOO CH 3OH (a) HCHO (b) HCOOH
(c) NH 4 NO 3 N 2 O 2H 2 O (c) HCHO and HCOOH (d) HCHO and CO2
(d) 3Cl2 6OH 5Cl ClO 3 3H 2 O 21. Hard water when passed through ion exchange resin
containing R’COOH groups, becomes free from :
14. The major organic product in the reaction,
CH3 — O — CH(CH3)2 + HI Product is (a) Cl (b) SO 24
(a) ICH2OCH(CH3)2 (c) H3O (d) Ca 2
(b) CH 3O C( CH 3 ) 2 22. A certain compound (X) when treated with copper sulphate
|
I solution yields a brown precipitate. On adding hypo
(c) CH3I + (CH3)2CHOH solution, the precipitate turns white. The compound is
(d) CH3OH + (CH3)2CHI (a) K2CO3 (b) KI
(c) KBr (d) K3PO4
15. For the reaction H 2(g) Br2(g) 2HBr(g) , the rate law
23. Consider the following reactions:
is rate k[H 2 ][Br2 ]1/ 2 . Which of the following statement (i) H +(aq) + OH–(aq) = H2O(l),
is true about this reaction H = – X1 kJ mol–1
(a) The reaction is of second order. 1
(ii) H2(g) + O = H2O(l),
(b) Molecularity of the reaction is 3/2 2 2(g)
(c) The unit of k is s–1 H = – X2 kJ mol–1
(d) Molecularity of the reaction is 2 (iii) CO2(g) + H2(g) = CO(g) + H2O,
16. The correct order of solubility in water for He, Ne, Ar, Kr, H = – X3 kJ mol–1
Xe is 5
(a) He > Ne > Ar > Kr > Xe (iv) C 2 H 2(g) O 2(g) = 2CO2(g) + H2O(l)’
2
(b) Xe > Kr > Ar > Ne > He
H = + 4X4 kJ mol–1
(c) Ne > Ar > Kr > He > Xe
Enthalpy of formation of H2O (l) is
(d) Ar > Ne > He > Kr > Xe
(a) + X3 kJ mol– 1 (b) – X4 kJ mol– 1
17. If uncertainty in position and velocity are equal then (c) + X1 kJ mol– 1 (d) – X2 kJ mol– 1
uncertainty in momentum will be 24. The restricted rotation about carbon carbon double bond
1 mh 1 h in 2-butene is due to
(a) (b) (a) Overlap of one s- and sp2-hybridized orbitals
2 2 m
(b) Overlap of two sp2- hybridized orbitals
h mh (c) Overlap of one p- and one sp2-hybridized orbitals
(c) (d)
4 m 4 (d) Sideways overlap of two p- orbitals.
MOCK TEST 3 MT-15

25. A balloon has maximum capacity of 20 L. At one atmospheric 33. For reaction aA xP , when [A] = 2.2 mM, the rate was
pressure 10 L of air is filled in the balloon. It will burst when found to be 2.4 mMs–. On reducing concentration of A to
pressure is (assuming isothermal condition) half, the rate changes to 0.6 mMs–1. The order of reaction
(a) > 0.5 atm (b) < 0.5 atm with respect to A is :
(c) = 0.5 atm (d) 0.5 atm (a) 1.5 (b) 2.0
26. Which one of the following complexes will have four (c) 2.5 (d) 3.0
different isomers ? 34. Which of the following is used for making optical
(a) [ Co (en ) 2 Cl 2 ]Cl (b) [ Co ( en )( NH 3 )Cl 2 ]Cl instruments?
(a) SiO2 (b) Si
(c) [Co(PPh 3 ) 2 Cl2 ]Cl (d) [Co (en )3 ]Cl 3
(c) SiH4 (d) SiC
27. Solubility product of a salt AB is 1 × 10–8 in a solution in
35. If ‘a’ stands for the edge length of the cubic systems :
which the concentration of A+ ions is 10–3 M. The salt will
simple cubic, body centred cubic and face centred cubic,
precipitate when the concentration of B– ions is kept
then the ratio of radii of the spheres in these systems will
(a) between 10–8 M to 10–7 M be respectively,
(b) between 10–7 M to 10–8 M
(c) > 10–5 M 1 3 1 1 1
(a) a: a: a (b) a : 3a : a
(d) < 10–8 M 2 4 2 2 2 2
28. Sucrose in water is dextro-rotatory, [ ]D= + 66.4º. When
1 3 3
boiled with dilute HCl, the solution becomes leavo-rotatory, (c) a: a: a (d) 1a : 3a : 2a
2 2 2
[ ]D= –20º. In this process the sucrose molecule breaks
into 36. Which of the following is correct order of acidity?
(a) L-glucose + D-fructose (a) HCOOH > CH3COOH > ClCH2COOH
(b) L-glucose + L-fructose > C2H5 COOH
(c) D-glucose + D-fructose (b) ClCH2COOH > HCOOH > CH3COOH
(d) D-glucose + L-fructose > C2H5 COOH
29. Select correct statement(s). (c) CH3COOH > HCOOH > ClCH2COOH
(a) Cyanamide ion (CN22–) is isoelectronic with CO2 and > C2H5COOH
has the same linear structure (d) C2H5COOH > CH3COOH > HCOOH
(b) Mg2C3 reacts with water to form propyne > ClCH2COOH
(c) CaC2 has NaCl type lattice 37. Penicillin is :
(d) All of the above (a) analgesic (b) antipyretic
30. In DNA the complementary bases are (c) antimalarial (d) antibiotic
(a) adenine and thymine; guanine and cytosine 38. The rate constant k, for the reaction
(b) uracil and adenine; cytosine and guanine 1
N 2 O 5 (g ) 2 NO 2 (g ) O 2 (g)
(c) adenine and guanine; thymine and cytosine 2
(d) adenine and thymine; guanine and uracil is 1.3 × 10–2s–1. Which equation given below describes the
31. Vapour pressure of benzene at 30°C is 121.8 mm. When 15 change of [N2 O5 ] with time ? [N2 O5 ]0 and [N2O 5] t
g of a non volatile solute is dissolved in 250 g of benzene corrospond to concentration of N2O5 initially and at time t.
its vapour pressure decreased to 120.2 mm. The molecular (a) [N2O5]t = [N2O5]0 + kt
weight of the solute (Mo. wt. of solvent = 78) (b) [N2O5]0 = [N2O5]t ekt
(a) 356.2 (b) 456.8 (c) log [N2O5]t = log [N2O5]0 + kt
(c) 530.1 (d) 656.7
[N 2 P5 ]0
32. In nitrogen family, the H-M-H bond angle in the hydrides (d) In [N P ] kt
2 5 t
gradually becomes closer to 90º on going from N to Sb.
This shows that gradually 39. Which of the following reactions can produce aniline as
(a) The basic strength of the hydrides increases main product?
(b) Almost pure p-orbitals are used for M-H bonding (a) C6H5NO2 + Zn/KOH
(c) The bond energies of M-H bonds increase (b) C6H5NO2 + Zn/NH4Cl
(d) The bond pairs of electrons become nearer to the (c) C6H5NO2 + LiAlH4
central atom (d) C6H5NO2 + Zn/HCl
EBD_7207
MT-16 CHEMISTRY

40. Which of the following reactions is used to make a fuel (c) aq. FeCl3 reacts with aq. K4[Fe(CN)]6
cell? (d) KMnO4 reduced to MnO2 in alkaline medium
(a) Cd(s) + 2Ni(OH)3 (s) CdO(s) 43. The unit of equivalent conductivity is
(a) S cm–2
+ 2Ni(OH)2(s) + H2O(l)
(b) ohm cm2 (g - equivalent)
(b) Pb(s) + PbO2(s) + 2H2SO4(aq) (c) ohm cm
2PbSO4(s) + 2H2O (l) (d) ohm 1cm2 (g equivalent)–1
(c) 2H2(g) + O2 (g) 2H2O (l) 44. Perlon is
(a) Terylene (b) Rubber
(d) 2Fe(s) + O2 (g) + 4 H+ (aq)
(c) Nylon – 6 (d) Polyester
2Fe2+ (aq) + 2H2O (l) 45. Tertiary nitro compounds do not tautomerise because
41. Which does not exist ? (a) there is no double bond
(a) [SiCl6]2– (b) [GeF6]2– (b) there is no -hydrogen
(c) [CCl6] 2– (d) [SnCl6]2–
(c) oxygen is more electronegative than hydrogen
42. In which case, van’t Hoff factor i remains unchanged ?
(d) all of the above
(a) PtCl4 reacts with aq. KCl
(b) aq. ZnCl2 reacts with aq. NH3
MOCK TEST 3 MT-17

ANSWER KEY

1. (d) 2. (a) 3. (d) 4. (d) 5. (c) 6. (c) 7. (d) 8. (a) 9. (d) 10. (b)
11. (b) 12. (b) 13. (c) 14. (c) 15. (d) 16. (b) 17. (a) 18. (a) 19. (c) 20. (d)
21. (d) 22. (d) 23. (d) 24. (d) 25. (b) 26. (b) 27. (c) 28. (c) 29. (d) 30. (a)
31. (a) 32. (b) 33. (b) 34. (a) 35. (a) 36. (b) 37. (d) 38. (d) 39. (d) 40. (c)
41. (c) 42. (b) 43. (d) 44. (c) 45. (b)

HINTS&SOLUTIONS
1. (d) BF3 is planar triangular while PF3 is pyramidal. CH3
2. (a) From the graph we can see the correct order of |
hydrideshift
pressures p1 > p3 > p2 CH3 — C — CH3
3 carbocation
3. (d) Br2 in CCl4 (a), Br2 in CH3 COOH (b) and alk. KMnO4
(c) will react with all unsaturated compounds, i.e., 1, 3 CH3
and 4 while ammonical AgNO3 (d) reacts only with –
OCH3
|
terminal alkynes, i.e., 3 and hence compund 3 can be CH3 — C — CH3
|
distinguished from 1, 2 and 4 by. ammonical AgNO3 OCH3
(d).
10. (b) A set of question number is valid when
4. (d) Decomposition of carbonates and hydrated oxides.
l < n and m1 lies between – l to + l. Thus sets I, II, IV
5. (c) Chlorination of methane proceeds via free radical are valid, III, V invalid.
mechanism. Conversion of methyl chloride to methyl
11. (b) For adiabatic process, q = 0
alcohol proceeds via nucleophilic substitution.
12. (b) Mn2+ (3d5) is more stable than Mn3+ (3d4).
Formation of ethylene from ethyl alcohol proceeds
13. (c) In NH4NO3, there are two different N-atoms (NH4+,
via dehydration reaction. Nitration of benzene is
NO3– ) with different oxidation numbers, thus reaction
electrophilic substitution reaction.
is not disproportionation.
6. (c) Benzene is non-polar and hence dissolves non-polar 14. (c) In case of unsymmetrical ethers, the site of cleavage
compounds (like dissolves like). Among the given depends on the nature of alkyl group e.g.,
compounds, only (c) is non-polar hence it dissolves
373K
in benzene. CH 3O CH(CH3 ) 2 HI
7. (d) Ozone hole is reduction in ozone layer in stratosphere.
CH3I (CH3 )2 CHOH
8. (a) In a period on moving from left to right ionic radii Methyl Isopropyl
decreases. iodide alcohol
(a) So order of cationic radii is The alkyl halide is always formed from the smaller
Cr2+ > Mn2+ > Fe2+ > Ni2+ and alkyl group.
(b) Sc > Ti > Cr > Mn 15. (d) H 2(g ) Br2(g ) 2HBr(g )
(correct order of atomic radii)
Rate law, R = k[H2] [Br2]½
(c) For unpaired electrons
Order of reaction = 1 + ½ = 3/2
Mn 2 (Five) Ni2 (Two) Molecularity of reaction = 2
Co 2 (Three) Fe2 (Four) The unit of k
R
(d) For unpaired electrons [H 2 ] [Br2 ]½

Fe2 (Four) Co 2 (Three) mole. lit 1s 1

Ni 2 (Two) Cu 2 (One) [mole. lit 1 ] [mole. lit 1 ]½


9. (d) The reaction is an example of SN1 reaction = mole–½ . lit½ . s–1
CH3 CH3 16. (b) As the molecular weight of noble gas atoms increases
|
–Br – | down the group its polarity increases due to which
CH 3 — CH — CH 2 Br CH3 — C H — C H 2 van-der-waal’s force between them increases. Due to
1 carbocation
increased polarity of heavier inert gas, its solubility in
EBD_7207
MT-18 CHEMISTRY

water also increases. So, most soluble gas will be Xe 23. (d) This reaction shows the formation of H2O, and the
and least soluble will be He. X2 represents the enthalpy of formation of H2 O
So correct order is Xe > Kr > Ar > Ne > He because as the definition suggests that the enthalpy
17. (a) x v of formation is the heat evolved or absorbed when
one mole of substance is formed from its constituent
h
x. v atoms.
4 m 24. (d) Rotation around bond is not possible. If any attempt
2 h is made to rotate one of the carbon atoms, the lobes
v
4 m of -orbital will no longer remain coplanar i.e no
parallel overlap will be possible and thus -bond will
h
v break . This is known as concept of restricted rotation.
4 m
In other words the presence of -bonds makes the
position of two carbon atom.
h
p m. v m 25. (b) The balloon would burst when V > 20 L
4 m
P1V1 = P2V2
1 hm 1 × 10 = P2 × 20
2 P2 = 0.5 atm (no bursting)
18. (a) It is N, N-dimethylcyclopropane –carboxamide. Thus, a pressure below 0.5 atm, it would burst.
19. (c) Hybridisation can be calculated by calculating the no 26. (b) Complex Co(en)(NH3)2Cl2]Cl will have four different
of valence electron and dividing it by 8. isomers.
In SO 4 2 = Total no. of e– (i) Geometrical isomers
= 6 + (6 × 4) + 2 = 32 + +
NH3 Cl
Cl NH3
32
So, no. of hybrid orbitals = 4 en Co en Co
8
sp3 hybridization. NH3
Cl
Cl
NH3

cis-form trans-form
Similarly, for PO 43 ; no. of hybrid orbitals
(ii) Optical isomers
5 24 3 32
= 4 + +
8 8 NH3
NH3
NH3
Cl

Hybridisation = sp3 en Co en Co

Similarly, for BF4 , it is sp3. Cl NH3


Cl Cl
cis-form trans-form
CH 2 OH
| 27. (c) AB A B
HIO4
20. (d) CO 2CH 2 O CO 2
| [A ][B ]
Ksp
CH 2 OH [AB]
21. (d) An ion exchange resin containing R–COOH group Salt will precipitate if ionic conc. > Ksp
exchange cations like Ca2+, Mg2+, Na+, Fe2+ with H+ [A+][B–] > 1 × 10–8
when hard water is passed through it. This resin is (1× 10–3)[B–] > 1 ×10–8
called cation exchange resin.
8
22. (b) KI reacts with CuSO4 solution to produce cuprous 1 10
[B ] or 1 × 10–5
iodide (white precipitate) and I2 (which gives brown 1 10 3
colour) Iodine reacts with hypo (Na 2S2O35H2 O) 28. (c) The hydrolysis of sucrose by boiling with mineral
solution. Decolourisaiton of solution shows the acid or by enzyme invertase or sucrase produces a
appearance of white precipitate. mixture of equal molecules of D(+) glucose and
2CuSO4 4KI 2K 2SO4 2CuI I2 D(–) Fructose.
Cuprous iodide (Brown colour
(White ppt.) in solution) HCl
C12 H 22O11 H 2O C 6 H12O 6 C 6 H12O 6
sucrose D glu cos e D Fructose
2Na 2S2 O3 I2 Na 2S4 O6 2NaI [ D] 66.5º [ D ] 52.5º [ D ] 92 º
Sod. tetrathionate
(coloourless) Invert sugar ,[ D ] 20º
MOCK TEST 3 MT-19

29. (d) In CO2 we have 22 (6 + 8 + 8 = 22) electrons. In (CN22– Thus the ratio of radii of spheres for these will be
), we have 22 (6 + 7 + 7 + 2 =22) electrons. Both CO2 simple : bcc : fcc
and (CN22–) have linear structures. Thus, statement
(a) is correct. a 3 1
= : a: a
Mg 2C3 4H 2O 2Mg(OH) 2 CH 3C CH 2 4 2 2
Propyne i.e. option (a) is correct answer.
i.e., statement (b) is also correct . 36. (b) Recall that presence of electron-withdrawing group
The structure of CaC2 is of NaCl type increases, while presence of electron-releasing group
i.e., statement (c) is also correct. decreases the acidity of carboxylic acids.
30. (a) In DNA, adenine faces thymine and guanine faces O O
cytosine. || ||
ClCH 2COOH H C OH CH3 C OH
31. (a) Given vapour pressure of pure solute (electron-withdrawing gp.) (Electron-releasing character
increasing from left to right)
(P 0 ) 121.8 mm; Weight of solute (w) = 15 g
O
Weight of solvent (W) = 250 g; Vapour pressure of ||
pure solvent (P) = 120.2 mm and Molecular weight of C 2H 5 C OH
solvent (M) = 78
37. (d) Penicillin is an antibiotic which was first obtained from
From Raoult’s law a fungus, penicillium notatum by the scientist,
Flemming.
Po P w M 121.8 120.2 15 78
o m W 121.8 m 250 38. (d) As the unit of rate constant is sec–1, so the reaction is
P
first order reaction. Hence
15 78 121.8
or m 356.2 1 a [N O ]
250 1.6 k = log or kt = log 2 5 0
t ( a x) [N 2 O5 ]t
32. (b) With the decrease in the electronegativity of central
atom the bond angle decreases 39. (d) Various products are formed when nitroarenes are
reduced. These are given below for C6H5NO2.
33. (b) When the concentration of reactant is reduced to half
Medium Main product
its initial value, the rate is reduced by 2.4 4 times In acidic medium Aniline (C6H5NH2) ( m et a l /
0.6
HCl)
It means, rate [ reactant]2 In neutral medium Phenyl hydroxylamine,( Z n /
So, order of reaction = 2
NH4Cl) NHOH
34. (a) SiO2 is used for this purpose.
35. (a) Following generalization can be easily derived for
various types of lattice arrangements in cubic cells
between the edge length (a) of the cell and r the
In alkaline medium NH NH
radius of the sphere.
Hydrazobenzene

a Thus, Aniline will be main product in case of (d).


For simple cubic : a = 2r or r
2 40. (c) Reaction used in fuel cell is
For body centred cubic : 2H 2 (g) O 2 (g) 2H 2 O( )

4 3 At anode : [H 2 2H 2e] 2
a r or r a
3 4
At cathode : O 2 2H 2 O 4e 4OH
For face centred cubic :
41. (c) Carbon cannot expand its coordination number beyond
1 four due to the absence of d-orbitals, hence it cannot
a 2 2r or r a
2 2 form [CCl6 ]2 ion
EBD_7207
MT-20 CHEMISTRY

42. (b) i remains unchanged when number of ions before and


O NOH
after complex ion remains constant.
NH2OH Beckmann
Solute y Complex y
rearrangement
(a) PtCl4 5 K2[PtCl6] 3
Cyclo hexanone Oxime Caprolactam
(b) ZnCl2 3 Zn[(NH3)4]Cl2 3
43. (d) The equivalent conductivity of a solution,
O O
1000
eq = × NH H2O
C H2N(CH2)5COOH
Where, Caprolactam -Amino Caproic acid
= specific conductance = Unit ohm–1 cm–1
C = normality of the solution unit gm eq/cm3 O O
Hence, the unit of eq is Ohm–1.cm2 (gm equivalent)– –(NH(CH2)5–C–NH–(CH2)5 –C)–n
1. Polymerise
Nylon-6
44. (c) Nylon-6 is also called as perlon. It is a polymer of
45. (b)
caprolactam.
Mock Test-4
Time : 1 hr Max. Marks -120
Which non-conductive solid when melts converts into
CH3 conductive liquid?
(a) C, D (b) Only C
1. IUPAC name of is (c) Only B (d) A, B and C
CH2CH3 8. On applying pressure to the equilibrium
(a) 1-methyl-3 ethyl cyclohexane ice water, which phenomenon will happen
(b) 1-ethyl-3 methyl benzene (a) More ice will be formed
(c) 1-ethyl-3 methyl cyclo hexane (b) More water will be formed
(d) Cyclo hexane-1-ethyl-3-methyl (c) Equilibrium will not be disturbed
(d) Water will evaporate
2. Which of the following structures does not contain any
9. Let 1 be the frequency of the series limit of the Lyman series,
chiral C atom but represent the chirality in the structure.
2 be the frequency of the first line of the Lyman series, and 3
(a) 2 – Ethyl – 3 – hexene (b) 2, 3-Pentadiene be the frequency of the series limit of the Balmer series, then –
(c) 1,3 – Butadiene (d) Pent – 3 – en – 1 – yne
3. N2 and O2 are converted to mono cations N2+ and O2+ 1
(a) 3= ( – 3) (b) 2– 1=
respectively, which of the following is wrong? 2 1 3

(a) In N2+, the N – N bond weakens (c) 1– 2= 3 (d) 1+ 2= 3


(b) In O2+, the O – O bond order increases
(c) In O2+, paramagnetism decreases Ka
(d) N2+ becomes diamagnetic 10. Given, HF H 2 O H 3O F— ;
4. In a compound AOH, electronegativity of ‘A’ is 2.1, the Kb
F— H 2O HF OH — .
compound would be
(a) Acidic Which relation is correct
(b) Neutral towards acid & base 1
(a) Kb = Kw (b) Kb
(c) Basic Kw
(d) Amphoteric
Ka
5. Which of the following orders is wrong? (c) Ka × Kb = Kw (d) Kw
(a) Electron affinity– N < O < F < Cl Kb
(b) Ist ionisation potential – Be < B < N < O 11. In an amino acid, the carboxyl group ionises at pK a1 =
(c) Basic property– MgO < CaO < FeO < Fe2O3
2.34 and ammonium ion at pK a 2 = 9.60. The isoelectric
(d) Reactivity–Be < Li < K < Cs Cl
point of the amino acid is at pH
(a) 5.97 (b) 2.34
6. The dipole moment of chlorobenzene is 1.5 D. The (c) 9.60 (d) 6.97
12. AB, A2 and B2 are diatomic molecules. If the bond enthalpies
Cl of A2, AB and B2 are in the ratio 1:1 :0.5 and enthalpy of
Cl formation of AB from A2 and B2 is –100 kJ mol–1 . What is
dipole moment of is the bond energy of A2 :
Cl Cl (a) 200 kJ mol–1 (b) 100 kJ mol–1
(c) 300 kJ mol –1 (d) 400 kJ mol–1
(a) 2.86 D (b) 2.25 D
13. Equal volume of 0.1 M urea and 0.1 M glucose are mixed.
(c) 1.5 D (d) 0 D
The mixture will have
7. Following substances are in solid state : (a) Lower osmotic pressure
(A) Methane (B) Cesium chloride (b) Same osmotic pressure
(C) Ice (D) Lithium (c) Higher osmotic pressure
(d) None of these
EBD_7207
MT-22 CHEMISTRY

14. If the following half cells have the E° values as 19. A metal which is not affected by conc. H2SO4, HNO3 or
3 – 2 alkalis forms a compound X. This compound X can be
Fe e Fe ; E° = + 0.77V and
2
used to give a complex which finds its application for toning
Fe 2e Fe ; E° = – 0.44V. The E° of the half cell in photography? The metal is
3 – (a) Au (b) Ag
Fe 3e Fe will be
(a) 0.33 V (b) 1.21 V (c) Hg (d) Cu
(c) 0.04 V (d) 0.605 V 20. If Cl2 gas is passed into aqueous solution of KI containing
15. The oxidation states of sulphur in the anions some CCl4 and the mixture is shaken, then”.
(a) Upper layer becomes violet
SO 32 , S2 O 24 and S2 O 62 follow the order (b) Lower layer becomes violet
(c) Homogenous violet layer is formed
(a) SO 32 S 2 O 24 S2 O 62 (d) None
(b) S 2O 24 21. In Lassaigne’s test, the organic compound is fused with a
S2 O62 SO 32
piece of sodium metal in order to
(c) S 2 O 62 S2 O 24 SO 32 (a) increase the ionisation of the compound
(b) decrease the melting point of the compound
(d) S 2O 24 SO 32 S2 O 62 (c) increase the reactivity of the compound
16. Which of the following statements is not correct? (d) convert the covalent compound into a mixture of ionic
compounds
(a) C– Cl bond in vinyl chloride is less polar than in CH3Cl
(b) C – Cl bond in vinyl chloride is stronger than in CH3Cl 22. An aqueous solution of colourless metal sulphate M gives
a white precipitate with NH4OH. This was soluble in excess
(c) C – Cl bond in vinyl chloride is shorter than in CH3Cl
of NH4OH. On passing H2S through this solution a white
(d) Vinyle chloride undergo nucleophilic substitution ppt. is formed. The metal M in the salt is
more readily than CH3Cl.
(a) Ca (b) Ba
NO2 (c) Al (d) Zn
17. 23. Choose the correct order of T (True) and F (False) –
Br2 CH3ONa
X Y (1) When the pH of rain water is below 3.6, it is called
Fe, CH3OH, acid rain.
(2) Ozone hole occurs over Antarctica mainly during
Cl September–October and it gets replenished in
November–December.
Product (Y) of this reaction is – (3) Methylcyclohexane is an ozone-depleting molecule.
NO2 (4) COD is always larger than BOD.
NO2
(a) TTTF (b) FTFT
(c) FFFF (d) FTTT
(a) (b) 24. Which of the following compound can not used in
preparation of iodoform?
OCH3 Br
OCH3 (a) CH3CHO (b) CH3COCH3
Cl
(c) HCHO (d) 2-propanol
NO2 NO2 25. The correct priorities for the substituents shown below,
according to the E-Z sequence rule is
I. – CN II. – CBr (CH 3 ) 2
(c) (d)
OCH3 O
||
OCH3 III. – COOH IV. – CH 2 C OCH 3

18. Although Al has a high oxidation potential it resists O


corrosion because of the formation of a tough, protective ||
V. – C H
coat of
(a) Al(NO3)2 (b) AlN (a) II, III, V, I, IV (b) V, II, I, IV, III
(c) Al2O3 (d) Al2(CO3)2 (c) III, IV, I, II, V (d) II, V, I, IV, III
MOCK TEST 4 MT-23

26. Identify X in the sequence given :


(b)
NH2

CHCl3 HCl
(Y) X + methanoic acid
KOH (300 K)
(c)
Cl

(d)
(a) NH2 Cl (b) C N Cl

29. 1 mol of N 2 and 3 mol of H 2 are placed in a closed


container at a pressure of 4 atm. The pressure falls to 3 atm
at the same temperature when the following equilibrium is
(c) N C Cl (d) CH3–NH Cl
attained

27. Select the rate law that corresponds to the data shown for N 2 (g) 3H 2 (g ) 2 NH 3 (g ) .
the following reaction A B C
The K p for the dissociation of NH 3 is
Expt. No. (1) (2) Initial Rate
3 3
1 0.012 0.035 0.10 (a) atm 2 (b) 0.5 (1.5) 3 atm 2
0.5 (1.5)3
2 0.024 0.070 0.80
3 0.024 0.035 0.10 0.5 (1.5)3 (1.5) 3 2
(c) atm 2 (d) atm
3 3 0.5
4 0.012 0.070 0.80
(a) Rate = K[B]3 (b) Rate = K [B]4 30. 0.5 g mixture of K2Cr2O7 and KMnO4 was treated with
3 excess of KI in acidic medium. I2 liberated required 100 cm3
(c) Rate = K [A] [B] (d) Rate = K [A]2 [B]2
of 0.15N. Na2S2O3 solution for titration. The percentage
28. An alkene upon ozonolysis yield
amount of K2Cr2O7 in the mixture is
CHO – CH2– CH2– CH2 – CHO only. The alkene is (a) 85.36 % (b) 14.64 %
(a) CH2= CH – CH2 –– CH2 –– CH2 –– CH2 –– CH3 (c) 58.63 % (d) 26.14 %
EBD_7207
MT-24 CHEMISTRY

ANSWER KEY

1. (d) 2. (b) 3. (d) 4. (b) 5. (b) 6. (c) 7. (c) 8. (b) 9. (c) 10. (c)
11. (a) 12. (d) 13. (b) 14. (c) 15. (d) 16. (d) 17. (b) 18. (c) 19. (a) 20. (b)
21. (d) 22. (d) 23. (b) 24. (c) 25. (a) 26. (a) 27. (a) 28. (b) 29. (b) 30. (b)

HINTS&SOLUTIONS
CH3
3
2
1. (d) 1 1 3RcZ 2
1 CH CH Ethyl group comes first in alphab- 2 RcZ2
2 3 12 22 4
C2H5
3 1 1 RcZ2
3 RcZ2 2 2
atical order but IUPAC name of 1
2 Br , 2-bromo- 2 4
CH3
1 2 3
4-ethyl-1-methyl cyclohexane. It follows lowest sum
rule [H 3 O ][F – ] [HF][OH – ]
10. (c) Ka and K b .
(i.e., lowest set of locant is preferred.) [HF][H 2 O] [F – ][H 2 O]
Lowest sum of locant is = 1 + 2+ 3 = 6 Therefore, Ka × Kb = [H3O+] [OH–] = Kw.
2. (b) The molecule 2,3 - pentadiene does not have any 11. (a) Isoelectric point (pH)
chiral C but at the same time it does not have any
mirror plane which makes the molecule chiral. pKa1 pK a 2 2.34 9.60
= 5.97
3. (d) In N2+
, there is one unpaired electron hence it is 2 2
paramagnetic.
12. (d) Let bond energy of A2 be x then bond energy of AB
4. (b) In A — O — H, if EN of ‘A’ is 2.1 then it will be neutral, is also x and bond energy of B2 is x/2.
as XA – X0 = X0 – XH. (where X is EN)
Enthalpy of formation of AB is – 100 KJ/mole:
5. (b) Correct order is B < Be < O < N.
Cl 120
1 1 1
Cl A2 B2 2 AB; A2 B2 AB; 100KJ
6 2 2 2
6. (c) Dipole moments of 2Cl and
3 x x 2x x 4x
Cl 5 4 Cl or 100 x 100 x 400 KJ
2 4 4
5Cl are vectorically cancelled.
(n 1 n 2 )
2 13. (b) M total st ,
It is due 1 Cl and 3 Cl v1 v 2
2 2 n1 = n2 = 0.1, V1 = V2 = 1litre M total 0.1 M
= 1 2 2 1 2 cos
0.77 2( 0.44) 0.77 0.88 0.11
= (1.5) 2 (1.5) 2 2 1.5 1.5 cos 120 = 1.5 D 14. (c) E0
3 3 3
7. (c) CsCl is ionic solid.
0.04
8. (b) Volume of ice > volume of water & thus increase in
pressure favours forward reaction showing decrease 15. (d) The chemical bond method gives the O.N.
in volume. – O O O O
O – – – –
– S = O: O – S – S – O : O–S– S–O
1 1 O +4 +3
9. (c) RcZ2 +5 O O
n12 n 22
+ – + –
16. (d) CH2 = CH– Cl CH2 – CH = Cl
1 1 Vinyl chloride
1 RcZ2 2 2
RcZ2
1
MOCK TEST 4 MT-25

24. (c) Formaldehyde can not produce iodoform, as only


17. (b) NO2 NO2
those compound which contains either CH3 CH
|
CH3ONa
OH
X=
SNAr
Br Br group or CH3 CH group on reaction with
||
Cl OCH3 O
18. (c) Because the layer of Al2O3 (oxide) is inert, insoluble potassium iodide and sod. hypochlorite yield
and impervious. iodoform.
19. (a) Au, the gold is not attacked by acids and alkalis. It
N C CH 3
forms AuCl3.AuCl3 further reacts with HCl to form |
| |
H[Au(Cl)4] which is used in photography. 25. (a) (i) C N, (ii) C CH 3 ,
| | |
AuCl3 HCl H[Au(Cl)4 ] N C Br
Complex

20. (b) 2KI Cl 2 2KCl I 2 OH H O C


| | |
I2 CCl 4 Violet Colour (iii) C O, (iv) C C OCH 3 ,
| | | |
Note: The excess of Cl2 should be avoided. The layer H O
O C
may become color less due to conversion of
I 2 to HIO 3 O C
| |
I 2 5Cl 2 6H 2 O 2HIO 3 10HCl C O
(v)
In case of Br 2 : |
H
Br22H 2 O Cl 2 2HBrO 2HCl
Arrange (NNN), (BrCC), (OOO), (CHH), (OOH) in
21. (d) To convert covalent compounds into ionic
increasing atomic number. The order is ii, iii, v, i, iv.
compounds such as NaCN, Na2S, NaX, etc.
CHCl3
26. (a) Cl
22. (d) 2 KOH
Zn 2NH 4 OH Zn(OH) 2 2NH 4
White ppt.

Zn(OH)2 2NH 4 OH (NH 4 ) 2 ZnO 2 2H 2 O HCl


Cl NC [Y]
Soluble 300 K

(NH 4 )2 ZnO2 H 2S ZnS 2NH 4 OH


White ppt.
Cl + HCOOH
23. (b). (1) When the pH of rain water is below 5.6, it is called
[X]
acid rain.
(2) Ozone hole occurs over Antarctica mainly during 27. (a) Let the rate law be r = [A]x[B]y
September–October and it gets replenished in
November–December. 0.10 [0.024]x [0.035] y
Divide (3) by (1)
(3) Methylcyclohexane is not an ozone-depleting 0.10 [0.012]x [0.035] y
molecule.
1 = [2]x, x = 0
(4) BOD (Biological oxygen demand) is a measure
of organic pollutant present in the sample of 0.80 [0.024]x [0.070] y
water. Higher is the value of BOD, higher is the Divide (2) by (3)
0.10 [0.024]x [0.035]y
level of organic pollution in water. The amount
of oxygen (in mg/L) consumed for oxidising all 8 = (2)y , y = 3
organic and oxidisable inorganic material in a Hence rate equation, R = K[A]0[B]3 =K[B]3
sample of water is called chemical oxygen
demand (COD).
COD is always larger than BOD.
EBD_7207
MT-26 CHEMISTRY

CH2– C – H 3
1 0 .5 3 3 0 .5
O CH2– CH 3 3
Ozonolysis 3 3
28. (b) CH2 CH2 = 2
O CH2– CH 2 0 .5 3
CH2– C – H
3

29. (b) N 2 (g) 3H 2 (g) 2 NH 3(g) = 0.5 × (1.5) 3 atm 2


1–x 3 – 3x 2x at equilibrium
30. (b) Let the amount of the K 2Cr2 O 7 in the mixture be x g,
Total moles,
1 – x + 3 – 3x + 2x = 4 – 2x = 3 (given) then amount of KMnO4 will be (0.5 – x) g
(Since, 4 moles = 4 atm given)
x 0.5 x 100 0.15
x = 0.5 =
49 31.6 1000
p N2 p3H2
K p for dissociation of NH 3 where 49 is Eq. wt. of K 2Cr2 O 7 and 31.6 is Eq. wt. of
p 2 NH 3
KMnO4 .
On solving, we get x = 0.073 g
0.0732 100
% age of K 2Cr2 O 7 = 14.64%
0.5
Mock Test-5
Time : 1 hr Max. Marks -120

1. An atom X belongs to 4th period of the periodic table and (c) C(graphite) + O2(g) CO2(g)
has highest number of unpaired electrons in comparison (d) C(g) + O2(g) CO2(g)
to the other elements of the period. The atomic number of 8. Among the reactions given below for B2H6, the one which
X is does not take place is
(a) 23 (b) 25 (a) B2H6 + HCl B2H5Cl + H2
(c) 24 (d) 33
(b) 2B2H6 + 6NH3 B3N3H6 (borazine)
2. In O2 , O 02 and O 22 molecular species, the total number
(c) B2H6 + 2N(CH3)3 2(CH3)3 NBH3
of antibonding electrons respectively are :
H O3
(a) 7, 6, 8 (b) 1, 0, 2 (d) B2H6 + 6C2H4 3C2H5OH + 2B(OH)3
(c) 6, 6, 6 (d) 8, 6, 8 9. The pure crystalline substance on being heated gradually
3. Aluminothermy used for on the spot welding of large iron first forms a turbid liquid at constant temperature and still at
structure is based on the fact that- higher temperature turbidity completely disappears. The
(a) As compared to iron, aluminium has greater affinity behaviour is a characteristic of substance forming.
for oxygen. (a) Allotropic crystals
(b) As compared to aluminium, iron has greater affinity (b) Liquid crystals
for oxygen. (c) Isomeric crystals
(c) Reaction between aluminium and oxygen is endothermic. (d) Isomorphous crystals.
(d) Reaction between iron and oxygen is endothermic. 10. Silver bromide when dissolve in hypo solution gives
4. Which of the following shows the tendency to form complex ..... in which oxidation state of silver is ....
peroxide? (a) Na3[Ag(S2O3)2], (I) (b) Na3[Ag(S2O3)3], (III)
(a) Lithium (b) Magnesium (c) Na3[Ag(S2O3)2], (II) (d) Na3[Ag(S2O3)4], (I)
(c) Beryllium (d) Radium 11. Which of the following can be termed as a mixed complex?
5. A 1.0 M solution with respect to each of the metal halides (a) K4 [Fe(CN)6] (b) [Cu(NH3)4] SO4
AX 3 , BX 2 , CX 3 and DX 2 is electrolysed using (c) [Co(NH3)4NO2Cl] Cl (d) K2FeO4
platinum electrodes. If E° = 1.50 V, E° = 0.3 V, 12. The relationship between the values of osmotic pressures
A3+ / A B2+ /B
of solutions obtained by dissolving 6.00 g L–1 of CH3COOH

C3+ /C
= – 0. 74 V, E°
D2+ /D
= – 2.37 V. ( 1) and 7.45 g L–1 of KCl ( 2) is
The correct sequence in which the various metals are (a) 1 2 (b) 1 2
deposited at the cathode is
(a) A, B, C, D (b) A, B, C 1 2
(c) 1 2 (d)
(c) D, C, B, A (d) C, B, A 1 2 1 2
6. Arrange hypophosphorous acid (H3PO2), phosphorous 13. In the reaction of KMnO4 with an oxalate in acidic medium,
acid (H3 PO 3 ) and Phosphoric acid (H3 PO 4 ) in the
decreasing order of acidic strength MnO4 is reduced to Mn 2 and C 2 O 24 is oxidised to
(a) H3PO3 > H3PO4 > H3PO2
CO2. Hence, 50 ml of 0.02 M KMnO4 is equivalent to
(b) H3PO4 > H3PO3 > H3PO2
(c) H3PO4 > H3PO2 > H3PO3 (a) 100 ml of 0.05 M H 2C 2O 4
(d) H3PO4 H3PO3 H3PO2
(b) 50 ml of 0.05 M H 2C 2O 4
7. Which of the following reactions corresponds to the
definition of enthalpy of formation ? (c) 25 ml of 0.2 M H 2C 2O 4
(a) C(diamond) + O2(g) CO2(g)
(d) 50 ml of 0.10 M H 2C 2O 4
(b) C(graphite) + O2(l) CO2(g)
EBD_7207
MT-28 CHEMISTRY

14. In which reaction, there is change in oxidation number of N (b) purple colour of CCl4
(a) 2NO2 N2O4 (c) brown colour in the organic layer of CCl4
(b) NH4OH NH4+ + OH– (d) deep blue colour in CCl4
(c) N2O5 + H2O 2HNO3 21. An element (atomic mass =100g/mol) having bcc structure
has unit cell edge 400pm. The density (in g/cm3) of the
(d) 2NO2 + H2O HNO3 + HNO2 element is
15. Potassium permanganate acts as an oxidant in neutral, (a) 10.376 (b) 5.19
alkaline as well as acidic media. The final products obtained
(c) 7.289 (d) 2.144
from it in the three conditions are, respectively
(a) MnO42–, Mn3+ and Mn2+ 22. An organic compound A (C4H10O) has two enantiomeric
forms and on dehydration it gives B(major product) and C
(b) MnO2, MnO2 and Mn2+
(minor product). B and C are treated with HBr/ Peroxide
(c) MnO2, MnO2+ and Mn3+ and the compounds so produced were subjected to alkaline
(d) MnO, MnO2 and Mn2+ hydrolysis then-
16. The number of electrons present in 6.4 g of calcium (a) B will give an isomer of A
carbide is – (NA = Avagadro’s number) (b) C will give an isomer of A
(a) 4 NA (b) 0.4 NA (c) Neither of them will give isomer of A
(c) 0.1 NA (d) 0.2 NA
(d) Both B and C will give isomer of A
C–C H 23. A reaction is found to be second order w.r.t. one of the
17. CH 3 CH 2 MgBr gives reactants & has rate constant of 0.5 mol–1 dm3 min–1. If
initial concentration is 0.2 mol dm–3 then t1/2 of reaction is
C CMgBr CH3 CH2 (a) 5 min (b) 10 min
(a)
(c) 15 min (d) 20 min

C CMgBr C2H6 CH 2 O
(b)
24. O CH 2
CH 2 O
MgBr CH3 CH2 C CH
(c)
The above shown polymer is obtained when a carbonyl
compound is allowed to stand. It is a white solid. The
C2H5 HC CMgBr polymer is
(d)
(a) trioxane (b) formose

18. The rate of SN1 reaction is fastest in the hydrolysis of (c) paraformaldehyde (d) metaldehyde.
which of the following halides ? 25. Concentration of NH4Cl and NH4OH in a buffer solution is
(a) C6H5CH2Br (b) CH3Br in the ratio of 1 : 1, Kb for NH4OH is 10–10. The pH of the
buffer is
(c) (CH3)2CHBr (d) (CH3)3CBr
19. Two elements A & B form compounds having molecular (a) 4 (b) 5
formulae AB2 and AB4. When dissolved in 20.0 g of benzene (c) 9 (d) 11
1.00g of AB2 lowers f.p. by 2.3°C whereas 1.00g of AB4 26. Aniline when diazotized in cold and when treated with
lowers f.p. by 1.3°C. The molal depression constant for dimethyl aniline gives a coloured product. Its structure
benzene in 1000g is 5.1. The atomic masses of A and B are would be
(a) 52, 48 (b) 42, 25
(a) CH3NH N=N NHCH3
(c) 25, 42 (d) None
20. To detect iodine in presence of bromine, the sodium extract (b) CH3 N=N NH2
is treated with NaNO 2 + glacial acetic acid + CCl4 . Iodine
(c) (CH3)2N N=N
is detected by the appearance of
(a) yellow colour of CCl4 layer (d) (CH3)2N NH
MOCK TEST 5 MT-29

27. An organic amino compound reacts with aqueous nitrous (b) one carboxylic acid residue and two phosphate
acid at low temperature to produce an oily nitrosoamine. groups
The compound is: (c) three phosphate groups
(a) CH3 NH2 (b) CH3CH2NH2
(d) three carboxylic acid residues
(c) CH3CH2NH.CH2CH3 (d) (CH3 CH2)3N
28. The standard reduction potential of C N
+ C6H5MgBr Ether H3O
Li+/Li, Ba2+/Ba, Na+/Na and Mg2+/Mg are –3.05, –2.73, – 30. A Ba
2.71 and –2.37 volts respectively. Which one of the following
is strongest oxidising agent?
O N–MgBr
(a) Na+ (b) Li+ C C
(c) Ba2+ (d) Mg2+ (a) (b)
29. Phospholipids are esters of glycerol with
(a) two carboxylic acid residues and one phosphate N–H N–OH
group C C
(c) (d)
EBD_7207
MT-30 CHEMISTRY

ANSWER KEY

1. (c) 2. (a) 3. (a) 4. (d) 5. (b) 6. (d) 7. (c) 8. (d) 9. (b) 10. (a)
11. (c) 12. (a) 13. (b) 14. (d) 15. (b) 16. (b) 17. (b) 18. (a) 19. (c) 20. (b)
21. (b) 22. (b) 23. (b) 24. (a) 25. (a) 26. (c) 27. (c) 28. (d) 29. (a) 30. (a)

HINTS&SOLUTIONS
1. (c) The configuration of atom of 4th period with maximum 11. (c) By definition, a mixed complex contains more than
unpaired electrons is 1s2 2s2 2p6 3s2 3p6 3d5 4s1 . one type of ligands.
Hence its atomic number is 24. 12. (a) Osmotic pressure, = CRT ;
2. (a) The total numberof electrons in the molecular species 6 7.45
n CH 3COOH , n KCl
given, respectively are 17, 16 and 18. Write down the 60 74.5
electronic configuration of the molecular species and Since KCl ionises. Therefore its effective conc., in
observe the number of electrons in antibonding solution increases.
orbitals which are respectively are 7, 6 and 8. 13. (b)
3. (a) Aluminium has greater affinity for oxygen and the 14. (d) In reaction (d) oxidation number changes from + 4 in
reaction is highly exothermic. NO2 to + 3 in HNO2 and + sin HNO3
4. (d) Among alkaline earth metals, barium and radium have
15. (b) In neutral and alkaline medium
the tendency to form peroxides.
5. (b) The more the reduction potential, the more is the MnO 4 2H 2 O 3e MnO 2 4OH
deposition of metals at cathode. Cation having E° value
less than – 0.83 V (reduction potential of H2O) will not In acidic medium:
deposit from aqueous solution.
MnO 4 8H 5e Mn 2 4H 2 O
6. (d) There is very little difference in acid strength in the
series H3 PO 4 , H3 PO 3 and H3 PO 2 because the C
hydrogen in these acids are not all bonded to
phosphorus. 16. (b) Ca , 6.4g of CaC2 contain -electron = 4NA
In the above three acids although the number of –OH
C
groups (ionisable hydrogen increases, yet the acidity
does not increase very much. This is due to the fact C CH
that the number of unprotonated oxygen, responsible
17. (b) + CH 3– CH2MgBr
for the enhancement of acidity due to inductive effect,
remains the same with the result dissociation constant
also remains nearly same. C CMg Br
7. (c) C(graphite) is the stable state of aggregation of C2 H6 +
carbon.
8. (d) Reaction between diborane and alkene are carried out
18. (a) Because of the formation of the most stable
in dry ether under an atmosphere of N2 because B2H6
and the products are very reactive. The products carbonium ion, C6 H5 C H 2
further treated with alkaline H2O2 to convert into
19. (c) Let atomic masses of A and B be a and b amu
alcohols.
respectively
alkaline
B2 H 6 6C2 H 4 B(C2 H4 )3
H 2O 2 Molar mass of AB2 = (a + 2b) g mol–1
reactive
and Molar mass of AB4 = (a + 4b) g mol–1
3CH3CH 2OH H3BO3 For compound AB2
9. (b) Liquid crystals on heating first become turbid and Tb = Kb WB 1000/ WA MB
then clear. 2.3 = 5.1 1 1000/ 20.0 (a + 2b)....I
10. (a) AgX + 2 Na2 S2O3 Na3 [Ag(S2O3)2]+ NaX
For compound AB4
Sodium argento thiosuphate
1.3 = 5.1 1 1000/ 20.0 (a + 4b)....II
(soluble complex)
Solving (I) and (II), a = 25.49 b = 42.64
MOCK TEST 5 MT-31

20. (b) 2NaI + 2 NaNO 2 + 4 CH 3COOH I 2 + 2NO + NaNO 2 ,HCl


26. (c) NH 2
4 CH 3COONa + 2 H 2 O 0 5 C

The colour of CCl4 layer turns purple due to liberated


I2.
NMe 2
21. (b) For bcc lattice, number of atoms per unit cell = 2
N 2 Cl +
n M 2 100 H
Now d
a3 No (4 10 8 cm)3 6.02 1023
= 200/38.528 = 5.19 g/cc N=N NMe 2
22. (b) Entiomers of C4H10O are

OH HO
27. (c) The secondary amines react with HNO2 to give the
| | oily nitroso derivative. Amongst the options, (c) is
H3CH 2C C CH3 H 3C C CH 2 CH 3 the secondary amine.
| |
H
H 28. (d) The strongest oxidising agent is one which has
maximum tendency to gain electrons, i.e. whose E°Red
is maximum
OH
|
H 2SO 4
29. (a) Phospholipids - Phosphate + glycerol + fatty acids +
CH 3 CH 2 C H CH 3 a nitrogen containing base.
A
General formula : CH 2 O . COR '
CH 3 CH CH CH 3 CH 2 CHCH 2 CH 3 |
Major ( B ) Minor ( C ) CHO COR ' '
| OH
1 |
23. (b) In general t1/ 2 of reaction CH 2 O P O X
n 1
(a 0 ) ||
O
For a second order reaction,
1 1 1 X = OHCH 2CH 2 NH 2 ,
t1/ 2 10 min Ethanolamine
K(a 0 )n 1 K(a 0 ) 0.5 0.2 30. (a) Alkyl or Aryl cyanide react with grignard reagent to
form ketones
CH2
On keeping O O C N
24. (a) 3 HCHO CH2 CH2 Ether
aq. solution O +C6H5MgBr
Trioxane
(meta formaldehyde)
O
25. (a)
salt
pOH pK b log C
base H 3O+
C6 H5– C=NMgBr
10
log 10 log 1 10; pH 14 10 4 C6H5

or C6H5 C O MgBrNH 2
|
C6 H 5

You might also like